Ambulatory Medicine

You might also like

Download as pdf or txt
Download as pdf or txt
You are on page 1of 1693

STEP2 CK QBank (Shelf Review) - Ambulatory

Medicine
Question #1

A 35-year-old man comes to the office due to 3 months of persistent cough and shortness of breath with exertion.
The patient was treated for sinusitis 3 times and pneumonia twice in the past 2 years; symptoms improved after
antibiotic therapy but recurred quickly thereafter. He has never smoked cigarettes and takes no medications.
Temperature is 37 C (98.6 F), blood pressure is 110/78 mm Hg, pulse is 86/min, and respirations are 15/min. BMI is
23 kg/m2. The oropharynx is clear. There is no lymphadenopathy or hepatosplenomegaly. Complete blood count,
serum electrolytes, and renal function tests are normal. HIV testing is negative. CT scans reveal mucosal
thickening of the paranasal sinuses and fibrosis at bilateral lung bases. What is the best next step in management
of this patient?

A) Bronchoalveolar lavage

B) Flow cytometry of peripheral blood

C) Measurement of serum alpha-1 antitrypsin level

D) Quantitative immunoglobulin assay

E) Sputum culture for acid-fast bacilli


Explanation
Correct Answer:

D) Quantitative immunoglobulin assay

Common variable immunodeficiency

• Abnormal differentiation of B cells into plasma cells


Pathophysiology
→ decreased immunoglobulin production

• Symptom onset classically age 20-40, as early as


puberty
• Recurrent respiratory infections (eg, pneumonia,
sinusitis, otitis)
Clinical • Recurrent GI infections (eg, Salmonella,
manifestations Campylobacter, Giardia)
• Chronic disease:
◦ Autoimmune (eg, RA, thyroid disease)
◦ Pulmonary (eg, bronchiectasis, fibrosis)
◦ GI (eg, chronic diarrhea, IBD-like conditions)

• ↓↓ IgG, ↓ IgA/IgM
Diagnosis
• No response to vaccination
Management • Immunoglobulin replacement therapy

GI = gastrointestinal; IBD = inflammatory bowel disease; RA = rheumatoid


arthritis.

This patient has recurrent sinopulmonary infections and chronic lung disease, findings suggestive of an
immunoglobulin deficiency disorder. Symptom onset in young adulthood (age 20-40) is characteristic of
common variable immunodeficiency (CVID), a primary humoral immunodeficiency in which antibody production
is decreased because B cells cannot differentiate into plasma cells.

The most common initial manifestations of CVID include acute and chronic rhinosinusitis, as seen on this patient's
imaging. Patients may subsequently develop progressive cough and dyspnea due to restrictive or obstructive lung
disease. Mechanisms for chronic lung disease include the following:

• Immune-mediated interstitial lung disease resulting in pulmonary fibrosis and scarring

• Bronchiectasis (ie, dilated, thickened airways) due to frequent infection and inflammation

Other manifestations can include chronic gastrointestinal illness, autoimmune disorders, and atopic disease.

The next step in evaluation of a humoral immunodeficiency is a quantitative immunoglobulin panel. Patients with
CVID have low IgG in combination with low IgM and/or IgA. Antibody response to vaccines is also low in CVID,
but leukocyte count is generally normal. Treatment is immunoglobulin replacement therapy and management of
chronic lung disease.

(Choice A) Bronchoalveolar lavage can be used to diagnose a primary lung disease (eg, pneumonia, malignancy).
The presence of concomitant, chronic sinus disease in this patient makes an underlying immunodeficiency more
likely.

(Choice B) Flow cytometry may be considered in an infant or toddler with low immunoglobulin levels concerning for
X-linked agammaglobulinemia, which is characterized by markedly low B cells. This patient's age makes this
diagnosis unlikely. Flow cytometry is not required for the diagnosis of CVID but typically reveals normal B and T
lymphocytes.

(Choice C) Alpha-1 antitrypsin deficiency is characterized by tissue damage in the lungs and liver. The sinuses
are not involved, and CT scan would reveal emphysematous changes (eg, large air spaces) rather than fibrosis in
the lower lungs.

(Choice E) Sputum culture for acid-fast bacilli is diagnostic for Mycobacterium tuberculosis, which can cause
chronic pulmonary infection but very rarely affects the sinuses. Classic x-ray findings of pulmonary tuberculosis
include consolidation, cavitation, hilar adenopathy, and pleural effusions.

Educational objective:
Recurrent sinopulmonary infections and evidence of chronic lung disease (eg, bronchiectasis, fibrosis/scarring) in a
young adult are suspicious for an underlying immunoglobulin deficiency such as common variable
immunodeficiency.

Reference
• CT screening for pulmonary pathology in common variable immunodeficiency disorders and the correlation
with clinical and immunological parameters.

• Common variable immunodeficiency–associated granulomatous and interstitial lung disease.


Question #2

A 40-year-old man comes to the office due to 3 days of fever, joint pain, and pruritic rash. The patient has a long
history of Crohn disease treated with infliximab. Temperature is 38.5 C (101.3 F), blood pressure is 110/60 mm Hg,
pulse is 85/min, and respirations are 16/min. Examination shows an urticarial rash over the trunk and extremities.
Oral mucous membranes are normal. Lungs are clear to auscultation, and no heart murmurs are present. The
abdomen is soft; mild right lower quadrant tenderness is present, which is unchanged from the prior examination.
Diffuse joint tenderness is present. The remainder of the examination shows no abnormalities. Complete blood
count and comprehensive metabolic panel show no new abnormalities compared to baseline. Urinalysis shows 1+
protein. Blood and urine cultures are negative. Which of the following is the most likely cause of this patient's
current symptoms?

A) Bacterial endocarditis

B) IgE-mediated allergic reaction

C) Immune complex deposition

D) Mycobacterial infection

E) Type IV hypersensitivity reaction


Explanation
Correct Answer:

C) Immune complex deposition

Serum sickness & serum sickness–like reaction

SS SSLR

Common Foreign proteins in antivenom, antitoxin, or Medications, particularly cefaclor, penicillin &
triggers monoclonal antibody TMP-SMX

Immune
High titer Mild or none
complexes

Complement
Extensive Minimal or none
activation

Onset 5-14 days after exposure 5-14 days after exposure

Fever High Low-grade


Arthralgia Yes Yes

Urticaria Yes Yes

Spontaneous (discontinue drug if still


Resolution Spontaneous
receiving)

SS = serum sickness; SSLR = serum sickness–like reaction; TMP-SMX = trimethoprim-sulfamethoxazole.

This patient developed fever, arthralgia, urticarial rash, and mild proteinuria while taking the chimeric monoclonal
antibody infliximab. Chimeric and humanized monoclonal antibodies contain foreign proteins derived from other
animals (eg, mice). Exposure to foreign proteins triggers an adaptive immune response marked by the generation
of high affinity antibodies. Binding of the high affinity antibody to the foreign portion of the medication generates
immune complexes (ICs), which are subsequently cleared by mononuclear cells in the reticuloendothelial system.

Although the formation and removal of ICs often proceeds without symptoms, large quantities of ICs can saturate
the phagocytic system and lead to IC aggregation. The aggregated ICs can then deposit into tissue (eg, skin,
joints, glomeruli), activate the classical complement system, and cause a type III hypersensitivity reaction called
serum sickness.

Common manifestations include fever, urticarial rash, arthralgia, and mild proteinuria. These symptoms resolve
spontaneously as the ICs are cleared, but patients may develop recurrent symptoms 5-14 days after each dose of
the medication. In addition to monoclonal antibodies, antitoxins, antivenoms, or antigenic vaccines can cause
serum sickness reactions. Some antibiotics (eg, penicillin) can cause a serum sickness–like reaction that is not due
to high IC titers.

(Choice A) Bacterial endocarditis often causes fever, arthralgia, and rash. However, the rash typically presents as
macular lesions on the palms (Janeway lesions) or violaceous nodules on the toes/fingers (Osler nodes); urticarial
rash is uncommon. In addition, negative blood cultures, no heart murmur, and normal complete blood count make
endocarditis unlikely.

(Choice B) IgE can form against chimeric antibodies and trigger urticarial rash upon reexposure due to mast cell
and basophil degranulation (type I hypersensitivity reaction). However, fever, arthralgia, and proteinuria would be
somewhat atypical. In addition, type I reactions typically begin within minutes of drug infusion.

(Choice D) Pulmonary tuberculosis usually causes subacute fever, malaise, and pulmonary symptoms; proteinuria
and urticarial rash would be atypical. In addition, because immunomodulators (eg, infliximab) dramatically increase
risk of Mycobacteria tuberculosis reactivation, most patients receive screening for latent tuberculosis prior to
medication initiation, also making symptomatic tuberculosis unlikely.

(Choice E) Type IV hypersensitivity reactions (eg, Stevens-Johnson syndrome) are mediated by cytotoxic T cells.
Stevens-Johnson syndrome causes high fever, arthralgia, and rash, but the rash is typically sunburn-like and
diffuse; in addition, mucocutaneous lesions are usually present.

Educational objective:
Serum sickness is an immune complex–mediated type III hypersensitivity reaction that occurs after exposure to
foreign proteins in antivenoms, antitoxins, monoclonal antibodies, or antigenic vaccinations. Manifestations typically
arise 5-14 days after exposure and include fever, urticarial rash, arthralgia, and mild proteinuria; symptoms usually
resolve over days without intervention as the immune complexes are cleared.
Question #3

A 26-year-old woman comes to the office due to an intensely pruritic rash on her back that has been present for the
past 24 hours. The rash developed shortly after the patient awoke from a nap and has been spreading. Other than
the rash, she has no symptoms. The patient reports no changes in cosmetic products or detergents. She has not
been ill recently and has never had a similar rash. The patient has no known chronic medical conditions and takes
no medications or supplements. She is not sexually active, and her last menstrual period was 2 weeks ago. Vital
signs are normal. Appearance of the skin on examination is shown in the exhibit. Oropharyngeal and
cardiopulmonary examinations reveal no abnormalities. Heart and lung sounds are normal. Which of the following
is the most appropriate next step in management?

A) Administer epinephrine

B) Obtain serum IgE level

C) Obtain skin biopsy

D) Prescribe a high-potency topical corticosteroid

E) Prescribe an H1-receptor antagonist


Explanation
Correct Answer:

E) Prescribe an H1-receptor antagonist

Acute urticaria

• Medications, insect stings


Etiologies • Infections, rheumatologic diseases
• Many cases are idiopathic

Pathophysiology • Mast cells release histamine → dermal edema

• Pruritic, erythematous plaques (ie, wheals)


Clinical features • Each wheal lasts <24 hr, urticaria duration <6 weeks
• Diffuse or localized

• No workup for 1st episode


• Allergy testing if specific trigger (eg, food) is
Evaluation suspected
• Laboratory testing/biopsy if systemic symptoms are
present
• 2nd-generation H1 antihistamines
Treatment & • Add H2 antihistamines or systemic corticosteroids if
prognosis severe
• Two-thirds of cases self-resolve

This patient has acute urticaria, intensely pruritic, well-circumscribed erythematous plaques (ie, wheals) caused by
mast cell activation in the superficial dermis. Acute urticaria usually represents an IgE-mediated allergic reaction
(eg, medication, insect bite, food); additional etiologies include infection (eg, bacterial, viral) and systemic illness
(eg, rheumatologic disorder, mastocytosis). However, many cases are idiopathic.

Initial management includes a second-generation H1 blocker (eg, cetirizine). A single episode, such as this
patient's presentation, typically requires no additional evaluation. Laboratory evaluation and biopsy are
recommended when urticaria is unresponsive to therapy, exhibits atypical features (eg, purpura), or is accompanied
by systemic symptoms (eg, arthritis, weight loss) (Choice C).

(Choice A) Intramuscular epinephrine is indicated for treatment of anaphylaxis, a severe, multisystem


hypersensitivity reaction. Although patients with anaphylaxis may have urticaria, they typically have abnormal vital
signs (eg, hypotension) and extradermal (eg, respiratory, gastrointestinal) manifestations. Treatment for isolated
urticaria does not require epinephrine.

(Choice B) Serum IgE level is typically elevated in atopic dermatitis, which presents acutely with erythematous
papules and vesicles, not wheals. Although acute urticaria is IgE mediated, the serum IgE level is not typically
elevated.

(Choice D) High-potency topical corticosteroids are used to treat plaque psoriasis, which presents with pruritic,
erythematous plaques with silvery scales, which are absent in this patient. Lesions typically develop over time on
the extensor extremities rather than abruptly throughout the back.

Educational objective:
Initial management of acute urticaria includes a second-generation H1 blocker (eg, cetirizine). No additional
evaluation is needed for a single episode of urticaria, although laboratory evaluation and biopsy are recommended
for episodes that are unresponsive to therapy, that exhibit atypical features, or that are accompanied by systemic
symptoms.

Reference
• Acute and chronic urticaria: evaluation and treatment.
Question #4

A cohort study was conducted to assess the relationship between high saturated fat consumption and the
occurrence of colorectal carcinoma among women. A group of women aged 40-65 was selected. The baseline
saturated fat consumption was calculated using a food questionnaire, and the cohort was followed for seven years
for the development of colon cancer. The study showed that women with high baseline saturated fat consumption
have four times the risk of colorectal cancer in a 7-year period, compared to women with low fat consumption (RR =
4.0, 95% CI = 1.5 - 6.5). According to the study results, what percent of colorectal carcinoma in women with high
fat consumption could be attributed to their diet?

A) 25%

B) 50%

C) 75%

D) 90%

E) 100%
Explanation
Correct Answer:

C) 75%

Attributable risk percent (ARP) or etiologic fraction is an important measure of the impact of a risk factor being
studied. ARP represents the excess risk in a population that can be explained by exposure to a particular risk
factor. It is calculated by subtracting the risk in the unexposed population (baseline risk) from the risk in the
exposed population, and dividing the result by the risk in the exposed population:

ARP = (risk in exposed - risk in unexposed)/risk in exposed.

An easier way to calculate the ARP is to derive it from the relative risk (RR):

ARP = (RR - 1)/RR.

In this case, ARP = (4.0 - 1)/4.0 = 0.75 (75%). According to the study results, 75% of colorectal carcinoma in the
high consumption group was attributable to high saturated fat intake.

Educational Objective:
ARP represents the excess risk in the exposed population that can be attributed to the risk factor. It can be easily
derived from the relative risk using the following formula: ARP = (RR - 1)/RR.
Question #5

A group of investigators conducted a randomized placebo-controlled clinical trial to assess the effect of a new
aldosterone receptor antagonist on the progression of chronic heart failure. The primary outcome was all-cause
mortality. A decrease in all-cause mortality in the treatment group was reported, with a relative risk of 0.71 (p =
0.001). Which of the following statements is the best interpretation of the reported association?

A) The 95% confidence interval for the relative risk of all-cause mortality includes 1.0

B) There is a 0.1% probability of observing the given relative risk (or more extreme) by chance alone assuming
no differences in mortality

C) There is a 71% decrease in all-cause mortality in the new aldosterone receptor antagonist compared to the
placebo group

D) There is only a 0.1% chance that the relative risk calculated is biased in favor of the aldosterone receptor
antagonist group

E) The results obtained are not statistically significant


Explanation
Correct Answer:

B) There is a 0.1% probability of observing the given relative risk (or more extreme) by chance alone assuming
no differences in mortality

Relative risk (RR) is a point estimate of association, but it does not account for random error. It is always possible
that the calculated RR occurred by chance. The p-value is used to strengthen the results of the study and
represents the probability of observing a given (or more extreme) result by chance alone, assuming the null
hypothesis is true. In this scenario, assuming there were no difference in all-cause mortality between the treatment
and placebo groups (null hypothesis), the probability of obtaining a RR of 0.71 or a more extreme value due to
chance alone is 0.1% (ie, p = 0.001 because 0.1/100 = 0.001). The commonly accepted upper limit (cut-off point) of
the p-value for a result to be considered statistically significant is 0.05 (ie, p <5% is statistically significant).

(Choices A and E) Both the p-value and the confidence interval are measures of statistical significance. A 95%
confidence interval that contains 1.0 (null value for RR) would mean that the results are not statistically significant.
This is not consistent with the obtained p-value of 0.001, which is statistically significant.

(Choice C) A RR = 0.71 shows that the drug decreased the risk of mortality by 29% (as 1.0 - 0.71 = 0.29 or 29%).

(Choice D) The p-value reflects random variability, not bias (which is systematic error).

Educational objective:
The p-value is the probability of observing a given (or more extreme) result due to chance alone, assuming the null
hypothesis is true. A result is generally considered statistically significant when p <0.05.
Question #6

A study was done to evaluate a new serologic screening test for human immunodeficiency virus (HIV) infection. A
total of 1000 people were randomly selected from the population, screened using the new test, and screened again
using the gold standard (i.e., Western blot analysis) to determine true infection state. The findings are as follows:

Test Results True HIV Infection No HIV Infection Total

Positive 140 60 200

Negative 40 760 800

Total 180 820 1000

What is the sensitivity of this new serologic screening test?

A) 140/180

B) 140/200

C) 60/820

D) 60/200
E) 180/1000
Explanation
Correct Answer:

A) 140/180

Sensitivity and specificity are used to measure a test's validity. Sensitivity is defined as the proportion of positive
test results among the subjects with the disease. Specificity is defined as the proportion of negative test results
among the subjects without the disease.

Test Results Disease Present Disease Absent Total

A B
Positive A+B
True positive (TP) False positive (FP)

C D
Negative C+D
False negative (FN) True negative (TN)

Total A+C B+D A+B+C+D

Sensitivity = TP / (TP + FN) = A / (A + C)

Sensitivity determines how well the test identifies the people with the disease. It is usually the first test used, similar
to the use of a screening test when there is a public health threat. The true-positive rate describes the sensitivity of
the test.
Specificity = TN / (TN + FP) = D / (B + D)

Specificity determines how well the test identifies the people without the disease. The true-negative rate describes
the specificity of the test.

A perfectly valid diagnostic test should have sensitivity and specificity equal to 1, but this is seldom possible. The
more sensitive the test, the less specific it is (and vice versa).

In this example, the sensitivity is 140/180.

Educational objective:
Know how to calculate the sensitivity and specificity of a test.
Question #7

A study evaluated the relationship between the common cold and the number of cigarettes smoked per day. The
research was conducted among fourth-year medical students and sponsored by the public health department of the
medical school. Medical students with symptoms of common cold were interviewed about their smoking status,
number of packs smoked per day, and duration of smoking. All participants completed the interview and their data
analyzed. Based on this information, which of the following is of most concern regarding the study?

A) Admission rate bias

B) Lead-time bias

C) Nonresponse bias

D) Regression to the mean

E) Response bias
Explanation
Correct Answer:

E) Response bias

Response bias occurs when participants in cross-sectional studies (eg, surveys, polls, questionnaires, interviews)
purposely give desirable responses to questions about topics perceived to be sensitive (eg, health behaviors).
Biased responses become less useful as they are inaccurate and may lead to incorrect conclusions (eg, lower
than expected prevalence of disease or frequency of risk factors).

In this example, medical students likely know the risk of smoking and may purposely report smoking a lesser
number of cigarettes than they truly smoke, especially to interviewers from the public health department of their
institution. This underreporting may significantly affect the results of the study.

(Choice A) Admission rate bias occurs when a distortion in risk ratio exists due to hospitals' differing admission
rates for certain cases. For instance, patients with cardiac diseases may be admitted to hospitals with more
specialized cardiology services.

(Choice B) Lead-time bias occurs when a screening test diagnoses a disease earlier than it would have appeared
by natural history alone, so that the time from diagnosis until death appears prolonged even though there might
actually be no improvement in survival.

(Choice C) Nonresponse bias occurs when respondents differ from nonrespondents in such meaningful ways that
threaten the generalizability of study results. All participants in this study completed the interview, so there were no
nonrespondents.

(Choice D) Regression to the mean refers to a set of data where the first assessment of a variable reveals an
extreme value but repeat assessment reveals values closer to the center of the distribution of that variable.

Educational objective:
Response bias occurs when participants purposely give desirable responses to questions about topics perceived to
be sensitive (eg, health behaviors). This practice results in responses that are inaccurate and may lead to incorrect
conclusions (eg, lower than expected prevalence of disease or frequency of risk factors).
Question #8

A prospective cohort study revealed a strong positive association between smoking and liver cirrhosis (relative risk
= 2.8). The researchers then divided the cohort into two groups: alcohol consumers and non-consumers.
Subsequent statistical analysis did not reveal any association between smoking and liver cirrhosis with either
group. The scenario described above is an example of which of the following?

A) Selection bias

B) Observer's bias

C) Measurement bias

D) Recall bias

E) Confounding
Explanation
Correct Answer:

E) Confounding

Confounding refers to the bias that results when the exposure-disease relationship is mixed with the effect of
extraneous factors (i.e., confounders). Confounders influence both the exposure and outcome. In the given study,
crude analysis of the data initially revealed an association between smoking and liver cirrhosis. The most likely
potential confounder is concomitant alcohol consumption in people who smoke. It is a well-known fact that alcohol
consumption is strongly associated with liver cirrhosis. Furthermore, alcohol consumption can explain at least part
of the association observed between smoking and liver cirrhosis.

Methods to deal with confounding include matching of cases and controls based on the confounding factor, or
stratification of the study population based on the confounding factor. In this case, running separate analyses for
alcohol consumers and non-consumers (this technique is called stratified analysis) can unmask confounding and
disclose the true unconfounded value of the RR.

(Choice A) Selection bias results from the manner in which people are selected for the study, or from the selective
losses from follow-up. The scenario does not mention any of these problems.

(Choices B and C) Observer's bias and measurement bias distort the measure of association by misclassifying
exposed/unexposed and/or diseased/non-diseased subjects. The scenario does not describe this classification
process.

(Choice D) Recall bias results from the inaccurate recall of past exposure by people in the study. It applies mostly
to case-control studies, not cohort studies.

Educational Ojective:
Know the concept of confounding. Distinguish between crude and adjusted measures of association. Confounding
refers to the bias that can result when the exposure-disease relationship is mixed with the effect of extraneous
factors (i.e., confounders).
Question #9

A study is undertaken to evaluate a new serological screening test for the diagnosis of tuberculosis infection. In the
study, 1000 people were randomly selected from the population and given the new diagnostic test, and positive and
negative results were recorded. Each study participant also was given the gold standard diagnostic procedure to
determine the true infection state of the individual. The findings are as follows:

True Tuberculosis infection state


Test results Yes No Total
Positive 130 60 190
Negative 50 760 810
Total 180 820 1000

Which of the following is the positive predictive value of the screening test under study?

A) 130/180

B) 130/190

C) 50/810

D) 760/810

E) 60/190
Explanation
Correct Answer:

B) 130/190

Although predictive values are not true indices of the validity of a test, these are still of prime importance to
physicians. In clinical practice, patients will more often present with a positive or negative test result, rather than a
disease or disease-free state; therefore, one should have sufficient knowledge about the predictive values of a test.

Positive predictive value is defined as the proportion of subjects with a positive test result who actually have the
disease. Negative predictive value is defined as the proportion of subjects with a negative test result who are free
of disease. Consider the following 2 x 2 table:

Disease
Test results Present Absent Total
A B
Positive A+B
True positive (TP) False positive (FP)
C
D
Negative False Negative C+D
True Negative (TN)
(FN)
Total A+C B+D A+B+C+D

Positive predictive value = TP/(TP+FP) or A/(A+B)


Negative predictive value = TN/(TN+FN) or D/(C+D)
In this case, the PPV is 130 / 190.

(Choice A) Sensitivity and specificity are useful for assessing the validity of a test. 130/180 is the sensitivity of the
test.
(Choice D) 760/810 is the negative predictive value of the test.

Educational objective:
Know how to calculate the predictive values of a test.
Question #10

A group of investigators is studying the relationship between a particular 5-lipoxygenase genotype and
atherosclerosis. A random sample of patients from a local university hospital is invited to participate in the study.
Blood samples for leukocyte genotyping are obtained, and ultrasonography to assess carotid intima-media
thickness (CIMT), a marker of atherosclerosis, is performed. Results show that the particular 5-lipoxygenase
genotype is associated with abnormally increased CIMT, a predisposition to atherosclerosis. Which of the following
best describes this study design?

A) Case-control study

B) Cross-sectional study

C) Prospective cohort study

D) Randomized clinical trial

E) Retrospective cohort study


Explanation
Correct Answer:

B) Cross-sectional study

https://afkebooks.com
https://afkebooks.com
A cross-sectional study is an observational study design that may be employed to estimate the prevalence of
disease, or to examine associations between risk factors and disease as they exist in a well-defined population at
one particular time. This type of design typically takes a snapshot and measures prevalence of risk factor and
outcome simultaneously.

In this case, subjects are classified according to their risk factor (ie, presence or absence of the particular
5-lipoxygenase genotype) and disease status (ie, presence or absence of abnormally increased carotid intima-
media thickness [CIMT]) at the same time (ie, snapshot). Then, the association between the presence of the risk
factor and the disease is estimated. A major limitation of cross-sectional studies is that the temporal relationship
between risk factor and disease is not always clear. In this example, however, demonstrating a temporal
relationship is possible due to the nature of the risk factor (acquiring a particular genotype precedes
atherosclerosis).

(Choice A) A case-control study is designed by selecting both patients with a particular disease (cases) and
patients without that disease (controls), and then determining their previous exposure status. In this case, the
researchers selected a single sample from the target population and categorized subjects according to their
presence or absence of the risk factor (ie, a particular 5-lipoxygenase genotype) and disease (ie, CIMT status) at a
single point in time.

(Choices C and E) In cohort studies, a group of exposed and unexposed subjects is followed over time for
development of the outcome of interest. Contrary to the prospective cohort study, in the retrospective case,
exposure and outcome have already occurred at the beginning of the study; therefore, exposure and outcome
status are ascertained retrospectively. In both prospective and retrospective cohort studies, however, there is a
follow-up of exposed and unexposed individuals across time. By contrast, in this example, a snapshot of the
subjects was obtained at one particular time.

(Choice D) A randomized clinical trial is an experimental study that directly compares ≥2 treatments or
interventions. Typically, the subjects are randomly assigned to an intervention (eg, a medication) or placebo, and
then followed for the development of the outcome of interest (eg, disease).

Educational objective:
In a cross-sectional study, risk factor and outcome are measured simultaneously at a particular point in time
(snapshot study). In other study designs, a certain period separates the exposure from the outcome.

https://afkebooks.com
Question #11

A group of investigators plans to conduct a study to assess the relationship between colon cancer and elevated
plasma C-reactive protein (CRP) levels. The study design involves determining the prevalence of elevated plasma
CRP levels (based on prespecified cutoff values) and of colon cancer in a sample of individuals at a given point in
time. The prevalence of elevated CRP levels is compared between patients with and without colon cancer. Which
of the following is the best statement of the null hypothesis for this study?

A) Colon cancer is more prevalent among subjects with elevated CRP levels

B) Having a diagnosis of colon cancer does not affect plasma CRP levels

C) Subjects with elevated plasma CRP levels are prone to colon cancer

D) The risk of colon cancer is the same for subjects with and without elevated plasma CRP levels

E) There is no association between elevated plasma CRP level and colon cancer

https://afkebooks.com
Explanation
Correct Answer:

E) There is no association between elevated plasma CRP level and colon cancer

To state the null hypothesis correctly, the study design must be considered. In this case, the study is cross-
sectional as the investigators are taking a snapshot of a sample and measuring prevalence of elevated C-reactive
protein (CRP) levels and of colon cancer at the same time. Therefore, the null hypothesis should state that there is
no association between elevated plasma CRP levels (equivalent to exposure) and colon cancer (equivalent to
outcome). In general, the null hypothesis represents a statement of no association between variables of interest.

As this is a cross-sectional study, the temporal relationship between the 2 variables (ie, whether the exposure
preceded the outcome, or whether the outcome preceded the exposure) cannot be determined. It would not be
suitable for the null hypothesis in this case to suggest a temporality or risk profile, whether having colon cancer is a
risk factor impacting CRP levels (Choice B) or having elevated CRP levels is a risk factor for colon cancer (Choice
C).

(Choice A) This would not be a suitable null hypothesis as it states that there is an association ("more prevalent")
between an exposure and outcome.

(Choice D) This would have been a valid statement of the null hypothesis for a cohort study, in which individuals
with and without elevated CRP levels are followed over time to determine the risk of colon cancer.

Educational objective:
The null hypothesis is the statement of no relationship between the exposure and the outcome. To state the null
hypothesis correctly, the study design should be considered.

https://afkebooks.com
Question #12

A study was conducted to assess the relationship between race and end-stage renal disease. Two groups of
pathologists independently studied specimens from 1,000 kidney biopsies. The first group of pathologists was
aware of the race of the patient from whom the biopsy came, while the second group was blinded from the patient's
race. The first group reported "hypertensive nephropathy" much more frequently for Black patients than the second
group. This study best demonstrates which of the following types of bias?

A) Confounding

B) Respondent bias

C) Recall bias

D) Selection bias

E) Observer bias

https://afkebooks.com
Explanation
Correct Answer:

E) Observer bias

Observer bias occurs when the investigator's decision is adversely affected by knowledge of the exposure status.
In this case, some pathologists' decisions were influenced by the fact that hypertensive nephropathy is a common
cause of endstage renal disease in the Black population. The pathologists who were blinded to the subjects'
race were not under this influence, so their interpretation was more unbiased.

(Choice A) Confounding is present when at least part of the exposure-disease relationship can be explained by
another variable (confounder). No information on possible confounders is given in this scenario.

(Choice B) Respondent bias is present when the outcome is obtained by the patient's response, and not by
objective diagnostic methods (e.g., migraine headache). In this case, the diagnosis was ascertained via kidney
biopsy.

(Choice C) Recall bias results from inaccurate recall of past exposure by patients. It is not applicable to this case.

(Choice D) Selection bias results from the manner in which the subjects are selected for the study, or from the
selective losses from the follow-up.

Educational objective:
Observer bias occurs when the investigator's decision is adversely affected by knowledge of the exposure status.

https://afkebooks.com
Question #13

Two cross-sectional studies were conducted using different questionnaires to determine the prevalence of over-the-
counter analgesics use in a population. The first study showed a prevalence of 7.5% (95% confidence interval 6.0 -
9.0), and the second study demonstrated a prevalence of 7.3% (95% confidence interval 6.9 - 7.6). If the true
prevalence of over-the-counter analgesics use in the population is 7.4%, which of the following statements about
the results of the study is the most accurate?

A) The first study results are more specific

B) The second study results are more sensitive

C) The first study results are more valid

D) The first study results are more accurate

E) The second study results are more precise

https://afkebooks.com
Explanation
Correct Answer:

E) The second study results are more precise

Precision is the measure of random error in the study. The study is precise if the results are not scattered widely;
this is reflected by a tight confidence interval. The first study has a wider confidence interval compared to the
second study; therefore, the second study is more precise.

(Choices A and B) Specificity and sensitivity are measures of validity. The sensitivity and specificity of the
questionnaires used in these studies cannot be determined from the given information.

(Choices C and D) Validity and accuracy are measures of systematic error (bias). Accuracy is reduced if the result
does not reflect the true value of the parameter measured. Increasing the sample size increases the precision of
the study, but does not affect accuracy. In our case, the results of both studies are pretty close to the true value,
and are thus seemingly accurate.

Educational Objective:
Precision is the measure of random error. The tighter the confidence interval, the more precise the result.
Increasing the sample size increases precision.

https://afkebooks.com
Question #14

A study was conducted to assess the association between L-tryptophan use and the development of Eosinophilia-
Myalgia Syndrome (EMS). Patients with EMS were asked about the use of products containing L-tryptophan during
the last 6 months. At the same time, people without EMS were randomly selected from the same population where
the patients came from, and asked about their experience with L-tryptophan containing products within the last 6
months. The study showed that the use of L-tryptophan is significantly associated with EMS. Which of the
following measures of association are the investigators most likely to report?

A) Relative risk

B) Median survival

C) Exposure odds ratio

D) Relative rate

E) Prevalence odds ratio

https://afkebooks.com
Explanation
Correct Answer:

C) Exposure odds ratio

The above case describes a typical case-control study design. Patients with the disease of interest (cases) and
people without the disease (controls) are asked about previous exposure to the variable being studied (L-tryptophan
use). The main measure of association is the exposure odds ratio, in which the exposure of people with the
disease (cases) is compared to the exposure of those without the disease (controls).

(Choices A and D) Incidence measures (e.g., relative risk or relative rate) cannot be directly measured in case-
control studies because the people being studied are those who have already developed the disease. Relative risk
and relative rate are calculated in cohort studies, where people are followed over time for the occurrence of the
disease.

(Choice B) Median survival is calculated in cohort studies or clinical trials, and is usually used to compare the
median survival times in two or more groups of patients (e.g., receiving a new treatment or placebo).

(Choice E) Prevalence odds ratio is calculated in cross-sectional studies to compare the prevalence of a disease
between different populations.

Educational Objective:
A case-control study is used to compare the exposure of people with the disease (cases) to the exposure of the
people without the disease (controls). The main measure of association is the exposure odds ratio.

https://afkebooks.com
Question #15

A new screening test was devised to detect pancreatic cancer at early stages using a serum marker (CA19-9) of the
disease. A study of this new test showed that its use prolongs the survival of patients with pancreatic cancer by
several months. The researchers concluded that use of the test improves the prognosis of patients with pancreatic
cancer. Which of the following is a potential problem with this conclusion?

A) Observer's bias

B) Measurement bias

C) Lead-time bias

D) Confounding

E) Ascertainment bias

https://afkebooks.com
Explanation
Correct Answer:

C) Lead-time bias

Lead-time bias should always be considered while evaluating any screening test. This bias occurs when there is an
incorrect assumption or conclusion of prolonged apparent survival and better prognosis due to a screening test.
What actually happens is that detection of the disease was made at an earlier point in time, but the disease course
itself or the prognosis did not change, so the screened patients appeared to live longer from the time of diagnosis to
the time of death. (USMLE tip: think of lead-time bias when you see "a new screening test" for poor prognosis
diseases like lung or pancreatic cancer.)

(Choices A, B and E) Observer's bias, measurement bias and ascertainment bias refer to misclassification of an
outcome and/or exposure (e.g., labeling diseased subjects as non-diseased and vice versa) and are related to the
design of the study. The scenario described does not mention how the study was designed.

(Choice D) Although the results of the study could be potentially confounded, there is no information on how
potential confounders were treated during the design or analysis stage of this study.

Educational Objective:
Understand the concept of lead-time bias in screening tests. The typical example of lead-time bias is prolongation
of apparent survival in patients to whom a test is applied, without changing the prognosis of the disease.

https://afkebooks.com
Question #16

A 20-year-old woman comes to the office for a routine employment physical examination. She reports no
symptoms, and her medical history includes no significant conditions. The patient has good exercise tolerance and
does not take any medications or use illicit drugs. Her blood pressure is 125/65 mm Hg and pulse is 80/min.
Cardiac auscultation reveals an early diastolic murmur at the left sternal border that is best heard with expiration.
Carotid pulses are normal bilaterally, and there are no bruits. ECG shows normal sinus rhythm without any
significant abnormalities. What is the best next step in management of this patient?

A) Chest x-ray

B) Coronary CT angiography

C) Echocardiography

D) Exercise ECG

E) No further workup

https://afkebooks.com
Explanation
Correct Answer:

C) Echocardiography

Diastolic and continuous murmurs are usually due to an underlying pathologic cause. Their presence should
prompt further evaluation with a transthoracic echocardiogram, which can identify valvular regurgitation and
evaluate for any associated structural abnormalities or hemodynamic consequences.

Aortic regurgitation leads to an early and gradually decreasing (decrescendo) diastolic murmur that begins
immediately after A2 (aortic component of the second heart sound). The murmur is high-pitched, has a blowing
quality, and, when it is due to valvular (rather than aortic root) abnormalities, is best heard along the left sternal
border at the third and fourth intercostal spaces with the patient sitting up and leaning forward while holding the
breath in full expiration.

(Choice A) Chest radiography in patients with chronic aortic regurgitation can reveal dilation of the ascending aorta
or cardiomegaly. These findings are nonspecific and are not useful in the diagnosis.

(Choices B and D) Exercise ECG is a useful screening test in patients with an intermediate probability of coronary
heart disease. Coronary CT angiography is a noninvasive method to assess coronary artery calcification and
intraluminal stenosis in patients with suspected coronary heart disease. These studies are not indicated in the initial
evaluation of the murmur of aortic regurgitation.

(Choice E) A midsystolic murmur can be detected occasionally in young, asymptomatic adults. This murmur is
usually benign and, in the absence of symptoms or other abnormal findings, does not require further evaluation.

Educational objective:
Diastolic and continuous murmurs are usually due to an underlying pathologic cause, and their presence should
prompt further evaluation with a transthoracic echocardiogram. Midsystolic murmurs in otherwise young,
asymptomatic adults are usually benign and do not require further evaluation.

https://afkebooks.com
Question #17

A 55-year-old woman comes to the office due to persistent shortness of breath. Over the last year, she has had
difficulty climbing the stairs to her bedroom and performing household chores. The patient also feels fatigued at the
end of the day. She has no chest pain, cough, syncope, or lower extremity swelling. Medical history is significant
for carpal tunnel syndrome and hypothyroidism, for which she takes levothyroxine. The patient is a lifetime
nonsmoker. She has no family history of early coronary artery disease. Blood pressure is 133/75 mm Hg and pulse
is 85/min and regular. Lungs are clear on auscultation. The apical impulse is displaced to the left, and there is a
palpable systolic thrill. A 4/6 blowing and high-pitched holosystolic murmur is heard at the apex. Which of the
following additional physical findings would most likely be present in this patient?

A) Clubbed fingers

B) Opening snap

C) S3

D) S4

E) Uvular pulsation

F) Wide and fixed splitting of S2

https://afkebooks.com
Explanation
Correct Answer:

C) S3

An intense holosystolic murmur best heard at the cardiac apex is consistent with severe mitral regurgitation
(MR). Due to backflow of blood from the left ventricle (LV) to the left atrium during systole, the total amount of blood
entering the LV during diastole is increased. To compensate for the increased volume load, the LV undergoes
eccentric hypertrophy and LV cavity dilation. Patients initially develop symptoms (eg, dyspnea on exertion) due to
decreased cardiac output (the lungs are clear during this time). Further progression can lead to decompensated
heart failure with increased left atrial pressure and pulmonary edema.

An audible S3 gallop is an expected finding in patients with severe MR. It results from reverberation of the walls
of the dilated LV during the passive filling phase of diastole. The S3 is present prior to the development of
decompensated heart failure, and the absence of an S3 helps exclude severe chronic MR. An S3 is also commonly
heard with decompensated heart failure of any etiology.

(Choice A) Finger clubbing can result from chronic hypoxemia, such as occurs in disorders with right-to-left
intracardiac shunting (eg, tetralogy of Fallot). MR causes dyspnea due to reduced cardiac output; significant
hypoxemia is unlikely unless there is pulmonary edema (this patient's lungs are clear on auscultation).

(Choice B) An opening snap occurs with mitral stenosis. It can be appreciated just after S2, as the stenotic mitral
valve opens at the beginning of diastole.

(Choice D) An S4 is often heard in patients with concentric LV hypertrophy due to systemic hypertension or severe
aortic stenosis. It is caused by blood striking a stiff left ventricle during atrial systole, just before mitral valve closure
(S1).

(Choice E) Uvular pulsation (Müller's sign) may be appreciated with the high-amplitude systolic pulsation and rapid
diastolic collapse that occurs with severe aortic regurgitation.

https://afkebooks.com
(Choice F) Wide and fixed splitting of S2 occurs with an atrial septal defect. Because blood passes from the left
atrium into the right atrium, blood flow increases across the pulmonic valve during systole, resulting in delayed
pulmonic valve closure (wide splitting). The communication between the atria also causes equilibration of left atrial
and right atrial pressures and eliminates respiratory variation in the splitting of S2 (fixed).

Educational objective:
Patients with severe mitral regurgitation develop left ventricular volume overload and LV dilation, with an audible S3
gallop typically present.

Reference
• Valvular heart disease: diagnosis and management.

https://afkebooks.com
Question #18

A 56-year-old man comes to the office for follow-up of high blood pressure. At his 3 previous visits, systolic blood
pressure has been 140-145 mm Hg, and diastolic blood pressure has been 90-96 mm Hg. The patient has no other
medical problems and takes no medications. He has smoked 2 or 3 cigarettes daily for 30 years and drinks 1 or 2
glasses of wine nightly. The patient's diet mainly consists of fried foods and meat, with limited intake of fruits and
vegetables. There is no family history of diabetes mellitus, coronary artery disease, hyperlipidemia, or
hypertension. Blood pressure is 146/97 mm Hg and pulse is 80/min. BMI is 27 kg/m2. Physical examination is
normal. Which of the following nonpharmacologic interventions is likely to have the greatest impact on this patient's
blood pressure?

A) DASH (Dietary Approaches to Stop Hypertension) diet

B) Isometric resistance training

C) Reduced sodium intake

D) Smoking cessation

https://afkebooks.com
Explanation
Correct Answer:

A) DASH (Dietary Approaches to Stop Hypertension) diet

Lifestyle interventions for hypertension

Approximate ↓
Modification Recommended plan
systolic BP (mm Hg)

DASH diet Diet high in fruits & vegetables & low in saturated & total fats 11

Weight loss Reduction of BMI to <25 kg/m2 6 per 10-kg loss

Aerobic exercise 30 minutes/day for 5+ days/week 7

Dietary sodium <1.5-2.3 g/day (response varies) 5-8

Alcohol limitation ≤2 drinks/day in men, ≤1 drink/day in women 5

https://afkebooks.com
DASH = Dietary Approaches to Stop Hypertension.

Guidelines define hypertension as systolic blood pressure ≥130 mm Hg and/or diastolic blood pressure ≥80 mm
Hg, as confirmed by multiple blood pressure readings. Although most patients with hypertension require
pharmacologic therapy, all patients with hypertension should also institute lifestyle interventions to reduce blood
pressure. Major interventions include dietary modification, weight loss (for overweight and obese patients), regular
aerobic exercise, and limitation of alcohol intake.

One of the most effective interventions to treat hypertension is the DASH (Dietary Approaches to Stop
Hypertension) diet, which is a combination diet rich in fruits, vegetables, legumes, low-fat dairy products, and low
in saturated and total fat. In patients with hypertension, the DASH diet can produce a reduction in systolic blood
pressure of approximately 11 mm Hg, with a corresponding reduction in diastolic pressure. Overweight and obese
patients should also be counseled on additional dietary and exercise interventions for weight loss, which can further
reduce blood pressure.

(Choice B) Although regular aerobic exercise consistently reduces blood pressure in hypertensive patients,
isometric resistance training has not demonstrated this effect.

(Choice C) The base DASH diet generally provides less sodium than many patients with hypertension typically
consume; a specific low-sodium version of the DASH diet can further reduce blood pressure. However, blood
pressure reduction from sodium restriction alone is variable and generally less than either version of the DASH diet.

(Choice D) Smoking cessation should be recommended to all patients to reduce the overall risk of cardiovascular
complications. However, the effect of smoking on blood pressure is small and generally inconsistent;
discontinuation of this patient's light smoking habit is unlikely to have a significant effect in reducing his blood
pressure.

Educational objective:
In patients with hypertension, the DASH (Dietary Approaches to Stop Hypertension) diet, which is a combination
diet rich in fruits, vegetables, legumes, low-fat dairy products, and low in saturated and total fat, can produce a
reduction in systolic blood pressure of approximately 11 mm Hg, with a corresponding reduction in diastolic

https://afkebooks.com
pressure.

Reference
• Non-pharmacological aspects of blood pressure management: what are the data?

• Nonpharmacologic therapy for hypertension: does it really work?

• A clinical trial of the effects of dietary patterns on blood pressure. DASH Collaborative Research Group.

https://afkebooks.com
Question #19

A 40-year-old man is found to have severe hypertriglyceridemia on routine evaluation. He has no present
complaints. Medical history includes hypertension controlled with hydrochlorothiazide and asymptomatic
gallstones. His father had a nonfatal myocardial infarction at age 47. The patient does not smoke or consume
alcohol. Blood pressure is 130/75 mm Hg and pulse is 70/min. He is prescribed niacin for his hypertriglyceridemia.
The patient returns 1 week later due to intense, generalized pruritus and flushing. What is the most likely cause of
his complaint?

A) Hypersensitivity reaction

B) Prostaglandin-related reaction

C) Drug-drug interaction

D) Drug-induced vasoconstriction

E) Psychogenic reaction

https://afkebooks.com
Explanation
Correct Answer:

B) Prostaglandin-related reaction

Cutaneous flushing and intense generalized pruritus are well-known side effects of high-dose niacin therapy
(high dosage is required to treat lipid abnormalities). These effects are explained by niacin-induced peripheral
vasodilation, not vasoconstriction (Choice D).

The mechanism involved in this reaction is believed to be drug-induced release of histamine and
prostaglandins, not a true hypersensitivity reaction (Choice A). The role of prostaglandins is confirmed by the fact
that low-dose aspirin can greatly reduce or prevent cutaneous flushing and pruritus if taken 30 minutes before
niacin. Therefore, niacin is frequently prescribed with aspirin. Flushing and pruritus usually improve after 2-4
weeks of therapy.

(Choice C) Thiazide diuretics do not intensify this reaction, although they may predispose the patient to
hypotension.

(Choice E) A psychogenic cause of the patient's complaint is unlikely.

Educational objective:
High-dose niacin therapy to treat lipid abnormalities frequently produces cutaneous flushing and pruritus. This side
effect is explained by prostaglandin-induced peripheral vasodilatation and can be reduced by low-dose aspirin.

https://afkebooks.com
Question #20

An 18-year-old woman comes to the office for a physical prior to beginning a community college program. She
recently moved into a fourth-floor apartment and reports some mild dyspnea with climbing stairs. The patient has
no chest pain, lightheadedness, or syncope. She has a history of heavy menstrual bleeding for which she takes
oral contraceptive pills. She does not use tobacco and consumes a few drinks at parties. The patient reports that
one of her younger sisters was born with "a hole" in the heart that closed spontaneously in infancy. Temperature is
36.7 C (98 F), blood pressure is 100/60 mm Hg, pulse is 92/min, and respirations are 14/min. Pulse oximetry
shows 98% on room air. Weight is 75 kg (165.3 lb) and height is 160 cm (5 ft 3 in) tall. BMI is 29 kg/m2. Her
conjunctivae are pink and mucous membranes are moist. There is no jugular venous distension. Lungs are clear
on auscultation. During expiration, there is an extra high-pitched sound heard after the S1. There is a grade 3/6
systolic crescendo-decrescendo murmur that is loudest at the left upper sternal border. The S2 is split throughout
the respiratory cycle and splitting increases with inspiration. The murmur does not increase with standing. Her
dorsalis pedis pulses are 2+ bilaterally. Which of the following is the most likely mechanism of this patient's
murmur?

A) Atrial septal defect

B) Bicuspid aortic valve

C) Functional murmur

D) Hypertrophic cardiomyopathy

E) Pulmonic stenosis

F) Tricuspid regurgitation

https://afkebooks.com
Explanation
Correct Answer:

E) Pulmonic stenosis

Pulmonic valve stenosis

• Congenital (usually isolated defect)


Etiology
• Rarely acquired (eg, carcinoid)

• Severe: Right-sided heart failure in childhood


• Mild: Symptoms (eg, dyspnea) in early adulthood
Clinical presentation
• Crescendo-decrescendo murmur (↑ on inspiration)
• Systolic ejection click & widened split of S2

Diagnosis • Echocardiography

• Percutaneous balloon valvulotomy (preferred)


Treatment
• Surgical repair in some cases

This patient most likely has pulmonic valve stenosis (PS), which most commonly occurs as an isolated
congenital defect and rarely occurs as an acquired defect (eg, rheumatic fever, carcinoid syndrome). Severe PS is
typically diagnosed early in life due to presentation of right-sided heart failure, but patients with relatively mild PS
often remain asymptomatic throughout childhood and develop symptoms (eg, dyspnea with exertion) in early

https://afkebooks.com
adulthood. Cardiac auscultation reveals a pulmonic ejection click (high-pitch sound after S1 best heard during
expiration) followed by a crescendo-decrescendo systolic murmur over the left second intercostal space. The
murmur intensifies with inspiration when there is increased blood volume in the right side of the heart. The stenosis
also causes the pulmonic valve to close later than usual, resulting in widened splitting of the aortic and pulmonic
components of S2; the splitting is further increased during inspiration.

(Choice A) Atrial septal defect is a common congenital defect that leads to increased blood flow in the right side of
the heart. Patients can have a mid-systolic murmur due to increased flow across the pulmonic valve; however, the
S2 is widely split without variation during respiration (wide and fixed splitting).

(Choice B) A bicuspid aortic valve is the most common congenital defect seen in adults. Premature valvular
calcification can lead to aortic stenosis (AS) in patients in their 40s and 50s, but AS would be unlikely in this
18-year-old woman. AS typically causes a systolic murmur in the right second intercostal space; it also leads to
narrowed or paradoxical splitting of S2 due to delayed closure of the aortic valve.

(Choice C) A functional (benign) flow murmur can occur in patients with anemia (eg, due to menorrhagia) due to
increased flow across the aortic and pulmonic valves; however, an ejection click or abnormal splitting of S2 is not
present.

(Choice D) Cardiac auscultation in patients with hypertrophic cardiomyopathy reveals a harsh systolic murmur at
the left mid-sternal border; the murmur increases in intensity with maneuvers that decrease left ventricular blood
volume (eg, Valsalva, standing).

(Choice F) Patients with tricuspid regurgitation have a holosystolic murmur over the left lower sternal border that is
intensified by an increase in venous return (eg, leg raising, deep inspiration, hepatic compression). An ejection click
or wide splitting of S2 is not present.

Educational objective:
Pulmonic valve stenosis usually occurs as a congenital defect and can often remain asymptomatic until adulthood.
Cardiac auscultation reveals an ejection click, followed by a crescendo-decrescendo systolic murmur over the left
second intercostal space and widened splitting of S2.

Reference

https://afkebooks.com
• Long-term outcomes of balloon valvuloplasty for isolated pulmonary valve stenosis.

https://afkebooks.com
Question #21

A 78-year-old man comes to the office due to lower extremity swelling. He has had progressive exertional dyspnea
over the past 2 years. Over the past 2 months, the patient has noticed lower extremity swelling, decreased
appetite, and increased abdominal girth. He has a history of poorly controlled hypertension but no known coronary
artery disease. The patient is a lifetime nonsmoker. He has no history of prior surgeries. Blood pressure is 165/88
mm Hg and pulse is 72/min and regular. He is afebrile. On physical examination, the jugular veins are distended
and there are prominent V waves. A holosystolic murmur is heard at the lower sternal border, and there is 3+ pitting
edema of the lower extremities bilaterally. Which of the following best explains the physical examination findings in
this patient?

A) Dilation of the pulmonary arteries

B) Dilation of the tricuspid valve annulus

C) Flailing of a tricuspid valve leaflet

D) Fusion of the tricuspid valve commissures

E) Increased intrapericardial pressure

https://afkebooks.com
Explanation
Correct Answer:

B) Dilation of the tricuspid valve annulus

This patient with prolonged uncontrolled hypertension and progressive dyspnea, lower extremity swelling, and
jugular venous distension likely has decompensated left-sided heart failure due to hypertensive heart disease.

https://afkebooks.com
The holosystolic murmur at the left lower sternal border is consistent with tricuspid regurgitation (TR).

An estimated 90% of TR is secondary (functional), defined as that occurring with an anatomically normal tricuspid
valve. Secondary TR usually results from right ventricular (RV) cavity enlargement in the setting of chronic right-
sided volume or pressure overload (eg, right-sided heart failure). Left-sided heart failure (as in this patient) is a
common cause of RV overload. Similar to left ventricular enlargement in secondary mitral regurgitation, RV
enlargement causes tricuspid annular dilation as well as tethering (increased tension) of the chordae
tendineae, both of which restrict adequate closure of normal tricuspid valve leaflets.

On the jugular venous pulsation (JVP) waveform, prominent V waves are highly specific for TR. The C wave of
the JVP waveform occurs at the start of RV contraction and is followed by the x descent as right atrial (RA) pressure
decreases. The V wave then follows as RA pressure increases with atrial refilling. Because incompetent tricuspid
valve closure leads to a sustained elevation in RA pressure during RV systole, TR causes an absent x descent and
prominent V wave.

(Choice A) Pulmonary artery dilation may be present with pulmonary hypertension due to decompensated heart
failure, but it does not cause a prominent V wave or holosystolic murmur.

(Choice C) Flailing of a tricuspid valve leaflet can occur due to chordae tendineae rupture in the setting of tricuspid
valve myxomatous degeneration. This is an example of primary TR, which is far less common than secondary TR.
In addition, it typically causes acute TR with rapid onset of right-sided heart failure and is less likely in this patient
with progressive symptoms.

(Choice D) Fusion of the tricuspid valve commissures can occur with rheumatic heart disease and results in
tricuspid stenosis. A rumbling diastolic murmur is expected, and the JVP demonstrates a flattened y descent and
prominent A wave due to obstructed RV filling.

(Choice E) Increased intrapericardial pressure occurs in cardiac tamponade. Muffled heart sounds are expected,
and the JVP demonstrates a flattened y descent due to restriction of passive RV filling.

Educational objective:
Tricuspid regurgitation is usually secondary (functional), resulting from right ventricular cavity enlargement in the
setting of chronic right-sided volume or pressure overload. A prominent V wave in jugular venous pulsation is highly
specific for tricuspid regurgitation.

Reference
• Secondary tricuspid valve regurgitation: a forgotten entity.

• Images in clinical medicine. Lancisi's sign.


Question #22

A 23-year-old man comes to the clinic for a physical examination prior to joining a professional soccer team. The
patient goes on a 10-mile run 3 or 4 times per week, during which he experiences no chest pain, lightheadedness,
or shortness of breath. Medical history is unremarkable. He does not smoke or drink alcohol. The patient lives in
Texas and has not traveled recently. Family history includes myocardial infarction in his father at age 56. Blood
pressure is 114/62 mm Hg and pulse is 54/min. Cardiac examination reveals normal heart sounds with no
murmurs. The lungs are clear to auscultation. ECG shows sinus bradycardia with occasional dropped QRS
complexes preceded by progressive lengthening of the PR interval. No ST-segment abnormalities are present, and
the QTc interval is normal. Which of the following is the most appropriate next step in the management of this
patient?

A) Exercise stress testing

B) Lyme serology

C) Pacemaker placement

D) Reassurance with no further intervention

E) Transthoracic echocardiography
Explanation
Correct Answer:

D) Reassurance with no further intervention

Atrioventricular block

Clinical presentation ECG features Management

First degree Asymptomatic PR interval prolongation Observation

Observation
Mobitz type I Progressive PR interval lengthening followed by
Usually asymptomatic
second degree dropped QRS complex (rarely PPM
placement)

Fatigue,
Mobitz type II Constant PR interval with randomly dropped QRS
lightheadedness, PPM placement
second degree complexes
syncope

Fatigue,
Third degree Complete dissociation of P waves & QRS
lightheadedness, PPM placement
(complete) complexes
syncope
PPM = permanent pacemaker.

This patient's ECG shows progressive lengthening of the PR interval followed by a dropped QRS complex,
consistent with Mobitz type I second-degree atrioventricular (AV) block. Highly trained endurance athletes
typically have very high resting vagal tone, making sinus bradycardia and low-grade AV block (ie, first degree and
Mobitz type I second degree) common findings on resting ECG. These are benign findings and require no
intervention.

(Choice A) Exercise stress testing is used to evaluate for obstructive atherosclerotic coronary artery disease
(CAD). AV nodal ischemia can lead to AV block, but there is no suspicion for significant CAD in this young patient
who experiences no symptoms (eg, chest pain, shortness of breath) at high levels of exertion.

(Choice B) Varying degrees of AV block can occur in a small percentage of patients with Lyme disease who do not
receive antibiotic treatment. However, transmission of Lyme disease is uncommon in Texas (it is usually transmitted
in the northeastern and upper Midwestern parts of the United States), and low-grade AV block is an expected
finding in this highly trained endurance athlete.

(Choice C) Permanent pacemaker placement is indicated for advanced AV block, including Mobitz type II second-
degree AV block (ie, randomly dropped QRS complexes) and third-degree AV block (ie, complete dissociation of P
waves and QRS complexes).

(Choice E) The presence of a systolic ejection murmur can suggest aortic stenosis or hypertrophic
cardiomyopathy, which would warrant transthoracic echocardiography for further investigation. However,
echocardiography is not indicated in this asymptomatic patient with no cardiac murmurs.

Educational objective:
Low-grade atrioventricular block, including first degree and Mobitz type I second degree, are common findings on
resting ECG in highly trained endurance athletes because of their high levels of resting vagal tone. These findings
are benign and require no intervention.
Reference
• ECG variants and cardiac arrhythmias in athletes: clinical relevance and prognostic importance.
Question #23

A 76-year-old man with coronary artery disease comes to the office for follow-up 6 months after an uncomplicated
coronary artery bypass surgery. The exertional chest pain that was bothering the patient before the surgery has
completely resolved. He reports no palpitations, shortness of breath, light-headedness, or syncope. The patient
has a history of hypertension, diet-controlled type 2 diabetes mellitus, and gout. Medications include low-dose
aspirin, metoprolol, and rosuvastatin. He has a 30-pack-year smoking history but quit 5 years ago. The patient
does not use alcohol or illicit drugs. Physical examination shows an irregular pulse. The chest surgical incision is
well healed. There are no heart murmurs, and the lungs are clear on auscultation. There is no peripheral edema.
ECG obtained in the office is shown in the exhibit. Which of the following is the best management for this patient?

A) Atrioventricular nodal slow pathway ablation

B) Exercise stress testing

C) Flecainide initiation

D) Oral anticoagulant therapy

E) Routine follow-up in 6 months


Explanation
Correct Answer:

D) Oral anticoagulant therapy

This patient's ECG demonstrates saw-toothed flutter waves consistent with atrial flutter. The atrial rate is typically
approximately 300/min, and the ventricular response can be constant or variable. This patient has a variable
ventricular response rate that creates an irregular rhythm, whereas a constant ventricular response rate (eg, a
constant 3:1 ventricular response at a rate of 100/min) can create a regular-appearing rhythm.

Atrial flutter usually results from a large reentrant circuit involving the cavotricuspid isthmus of the right atrium. Risk
factors are similar to those for atrial fibrillation and include age, hypertension, and heart failure. The arrhythmia is
also frequently seen following cardiac surgery. Atrial flutter carries a similar risk of arterial thromboembolism to
atrial fibrillation; therefore, this patient with multiple CHA2DS2-VASc risk factors should be started on chronic
anticoagulation (eg, rivaroxaban, apixaban) therapy.

Although not needed in this patient (whose pulse is ~60-70/min), management also involves rate control, which can
sometimes be difficult to accomplish with medications alone (eg, metoprolol, verapamil). Definitive management
often requires radiofrequency ablation to disrupt the reentrant circuit, or electric cardioversion is also sometimes an
option. These interventions increase the risk of a thromboembolic event and should be preceded by several weeks
of anticoagulation therapy.

(Choice A) Atrioventricular (AV) nodal slow pathway ablation is used to treat AV nodal reentrant tachycardia, a type
of paroxysmal supraventricular tachycardia. ECG usually shows a regular rhythm with rapid rate and no visible P
waves. The reentrant circuit in atrial flutter does not involve the AV node.

(Choice B) Exercise stress testing is used to evaluate for coronary artery disease in patients with suggestive
symptoms (eg, chest pain on exertion), but it is not indicated in this patient.

(Choice C) Flecainide is a class IC antiarrhythmic that is sometimes used to treat atrial fibrillation. Such rhythm-
control agents are less preferable for atrial flutter because of poor efficacy and the risk of precipitating rapid
ventricular response.

(Choice E) Routine follow-up in 6 months is not appropriate because atrial flutter significantly increases the risk of
thromboembolic stroke in this patient with multiple CHA2DS2-VASc risk factors.

Educational objective:
Atrial flutter is recognized by saw-toothed flutter waves on ECG; the rhythm can be regular or irregular depending
on the variability of the ventricular response rate. Atrial flutter carries a similar risk of arterial thromboembolization
to atrial fibrillation and should be similarly managed with chronic anticoagulation.

Reference
• Atrial flutter and the risk of thromboembolism: a systemic review and meta-analysis.
Question #24

A 75-year-old man comes to the clinic due to a 6-month history of periodic substernal chest pressure, which he
experiences when walking uphill or climbing 2 flights of stairs. His medical history is significant for hyperlipidemia,
for which he takes atorvastatin. The patient smokes a pack of cigarettes daily and occasionally consumes alcohol.
Blood pressure is 120/78 mm Hg and pulse is 75/min. Physical examination shows no abnormalities. Resting ECG
is normal. A treadmill stress test shows a horizontal ST-segment depression in leads V1-V4 at 73% of predicted
maximal heart rate. Echocardiography demonstrates normal resting left ventricular systolic function. The patient
prefers medical management. He is prescribed sublingual nitroglycerin to take as needed when anginal pain occurs
and he is also prescribed a daily medication to help prevent anginal episodes. The daily medication most likely
functions through which of the following mechanisms?

A) Altered myocardial calcium level

B) Coronary artery vasodilation

C) Decreased cardiac afterload

D) Decreased cardiac preload

E) Decreased myocardial contractility


Explanation
Correct Answer:

E) Decreased myocardial contractility

Treatment of chronic stable angina

• First-line therapy
Beta blockers
• ↓ Myocardial contractility & heart rate

• Alternative to beta blocker


Nondihydropyridine CCBs
• ↓ Myocardial contractility & heart rate

• Added to beta blocker when needed


• Coronary artery vasodilation
Dihydropyridine CCBs
• ↓ Afterload by systemic vasodilation

• Long-acting added for persistent angina


Nitrates
• ↓ Preload by dilation of capacitance veins

• Alternative therapy for refractory angina


Ranolazine
• ↓ Myocardial calcium influx
CCBs = calcium channel blockers.

This patient has coronary artery disease with symptoms and clinical findings consistent with stable angina.
Stable angina is defined as chest discomfort that predictably occurs with exertion and is relieved with rest; it results
from a mismatch of myocardial oxygen supply and demand. In patients for whom coronary revascularization is
not possible or desired, stable angina is treated medically. The 3 main medication classes for the prevention of
stable angina symptoms are beta blockers, calcium channel blockers, and long-acting nitrates.

Beta blockers (eg, metoprolol, atenolol) are recommended as first-line therapy for controlling anginal symptoms
and improving exercise tolerance in patients with stable angina. These drugs primarily function by reducing
myocardial oxygen demand through a decrease in heart rate and myocardial contractility and are highly
effective in minimizing or eliminating exertional angina. Nondihydropyridine calcium channel blockers (eg,
verapamil, diltiazem) treat angina primarily through the same mechanisms and are an alternative first-line therapy in
patients with a beta blocker contraindication.

(Choice A) Ranolazine exerts its antianginal effect by inhibiting the late phase of sodium influx into ischemic
cardiac myocytes. This causes reduced calcium influx through the sodium-calcium exchanger, leading to a
reduction in ventricular wall stress and myocardial oxygen demand. Ranolazine is often reserved for patients with
stable angina that is refractory to other medical therapies.

(Choices B and C) The dihydropyridine calcium channel blockers (eg, amlodipine, felodipine) treat angina in 2
ways: They increase myocardial oxygen supply through coronary artery vasodilation and reduce myocardial oxygen
demand through systemic arterial vasodilation and a reduction in cardiac afterload (beta blockers only mildly reduce
afterload). The dihydropyridines are typically used in combination with beta blockers because monotherapy can
result in reflex tachycardia and worsening anginal symptoms.

(Choice D) Nitrates treat angina primarily through the dilation of venous capacitance vessels and a reduction in
cardiac preload. The result is a reduction in left ventricular wall stress and reduced myocardial oxygen demand.
Long-acting nitrates (eg, isosorbide mononitrate) are often added to beta blockers in patients with persistent, stable
angina. As-needed, short-acting nitrates (eg, sublingual nitroglycerin) can provide brief anginal relief but are not
used for long-term anginal prevention.

Educational objective:
Beta blockers are first-line therapy for preventing symptoms and improving exercise tolerance in patients with stable
angina. They help prevent angina by decreasing exertional heart rate and myocardial contractility, leading to a
reduction in myocardial oxygen demand. Calcium channel blockers and long-acting nitrates are also used.

Reference
• Treatment of stable angina pectoris.

• 2012 ACCF/AHA/ACP/AATS/PCNA/SCAI/STS guideline for the diagnosis and management of patients with
stable ischemic heart disease: executive summary: a report of the American College of Cardiology
Foundation/American Heart Association Task Force on Practice Guidelines, and the American College of
Physicians, American Association for Thoracic Surgery, Preventive Cardiovascular Nurses Association,
Society for Cardiovascular Angiography and Interventions, and Society of Thoracic Surgeons.
Question #25

A 61-year-old woman comes to the office due to elevated blood pressure. Blood pressure was 152/92 mm Hg a
month ago; she has been monitoring it at home since then, with an average of 148/88 mm Hg. The patient's only
other medical condition is osteoporosis, diagnosed 6 months ago and treated with vitamin D, calcium, and
alendronate. Today in the office, blood pressure is 156/90 mm Hg on the left arm and 152/92 mm Hg on the right
and pulse is 70/min. Physical examination is normal. Complete blood count, serum creatinine, serum electrolytes,
TSH, and urinalysis are normal. ECG is normal. Which of the following is the best next step in management of this
patient's blood pressure?

A) Advise lifestyle modifications only

B) Change alendronate to denosumab

C) Discontinue calcium supplements

D) Start atenolol

E) Start chlorthalidone
Explanation
Correct Answer:

E) Start chlorthalidone

Choice of antihypertensive drug


for comorbid conditions

• Angina pectoris: β-blocker, CCB


Coronary
• Post–myocardial infarction: ACE inhibitor
atherosclerosis
or ARB, β-blocker

Heart failure
• ACE inhibitor or ARB, β-blocker, diuretic,
with reduced
aldosterone antagonist
ejection fraction

Atrial fibrillation
• β-Blocker, nondihydropyridine CCB
or flutter

Chronic kidney
• ACE inhibitor or ARB
disease
• Losartan, other ARB, CCB (avoid
Gout
diuretics)

Osteoporosis • Thiazide diuretic

Migraine • β-Blocker, CCB

ARB = angiotensin II receptor blocker; CCB = calcium channel


blocker.

This patient has hypertension, confirmed on multiple occasions in both a health care setting and at home. A short-
term trial of lifestyle modification only (eg, dietary modification, weight loss) may be considered for low-risk patients
with minimal blood pressure elevation (ie, 130-140 mm Hg systolic, 80-90 mm Hg diastolic). In patients who have
confirmed significant out-of-office blood pressure elevation (as in this patient), starting antihypertensives without
delay is recommended (possibly with a more liberal goal in those with advanced age) (Choice A).

First-line antihypertensive agents include ACE inhibitors, angiotensin II receptor blockers, calcium channel blockers,
and thiazide diuretics (eg, chlorthalidone). The choice of drug is determined largely by patient factors (eg,
comorbidities, concurrent medications, adverse effects profile).

For patients with osteoporosis, thiazide diuretics are generally preferred; thiazides increase calcium reabsorption
in the distal tubule, reducing renal calcium wasting and slowing the rate of bone loss. Although thiazides can
cause hypercalcemia, especially in patients taking supplemental calcium, it is rarely significant (unless
hyperparathyroidism is present), and the calcium supplements may be continued; regular electrolyte monitoring is
necessary with all diuretics.

(Choices B and C) Common medications that raise blood pressure include decongestants (eg, pseudoephedrine),
stimulants (eg, methylphenidate), some antidepressants (eg, bupropion), erythropoietin, and nonsteroidal anti-
inflammatory drugs; alendronate and calcium do not generally affect blood pressure.

(Choice D) Beta blockers (eg, atenolol) are commonly used for hypertension in patients with comorbid coronary
heart disease (with angina or a history of myocardial infarction), heart failure with reduced ejection fraction, and
atrial fibrillation. Although the benefit of hypertensive treatment is determined primarily by the degree of blood
pressure lowering rather than the choice of drug, beta blockers appear to not confer the same benefits as other
classes, especially in patients age >60.

Educational objective:
First-line antihypertensive medications include ACE inhibitors, angiotensin II receptor blockers, calcium channel
blockers, and thiazide diuretics; the choice of drug is determined largely by comorbid conditions, concurrent
medications, and adverse effects profile. Thiazide diuretics reduce renal calcium wasting and slow the rate of bone
loss and are therefore preferred in patients with osteoporosis.

Reference
• 2017 ACC/AHA/AAPA/ABC/ACPM/AGS/APhA/ASH/ASPC/NMA/PCNA guideline for the prevention,
detection, evaluation, and management of high blood pressure in adults: executive summary: a report of the
American College of Cardiology/American Heart Association Task Force on Clinical Practice Guidelines.

• Association between hypertension, antihypertensive drugs, and osteoporosis in postmenopausal Syrian


women: a cross-sectional study.

• Long-term thiazide use and risk of low-energy fractures among persons with Alzheimer's disease—nested
case-control study
Question #26

A 70-year-old man comes to the office for a follow-up for gout. It has been well controlled, with no joint pain since
he started taking allopurinol 6 months ago. The patient has had no chest pain, trouble breathing, headaches, or
lightheadedness. He does not use tobacco, alcohol, or illicit drugs. The patient's blood pressure was 145/90 mm
Hg and 150/95 mm Hg at his 2 previous visits. Today, his blood pressure is 148/92 mm Hg on the left arm and 144/
90 mm Hg on the right arm. Physical examination is normal. Complete blood cell count, serum creatinine, serum
electrolytes, TSH, uric acid, and urinalysis results are normal. ECG indicates left ventricular hypertrophy. Which of
the following is the best next step in managing this patient?

A) Discontinue allopurinol and recheck blood pressure in 4 weeks

B) Order exercise stress testing

C) Reassure patient and follow up in 3 months to recheck blood pressure

D) Start hydrochlorothiazide

E) Start losartan
Explanation
Correct Answer:

E) Start losartan

Choice of antihypertensive drug


for comorbid conditions

• Angina pectoris: β-blocker, CCB


Coronary
• Post–myocardial infarction: ACE inhibitor
atherosclerosis
or ARB, β-blocker

Heart failure
• ACE inhibitor or ARB, β-blocker, diuretic,
with reduced
aldosterone antagonist
ejection fraction

Atrial fibrillation
• β-Blocker, nondihydropyridine CCB
or flutter

Chronic kidney
• ACE inhibitor or ARB
disease
• Losartan, other ARB, CCB (avoid
Gout
diuretics)

Osteoporosis • Thiazide diuretic

Migraine • β-Blocker, CCB

ARB = angiotensin II receptor blocker; CCB = calcium channel


blocker.

This patient has newly diagnosed hypertension, confirmed by measurements on 3 different office visits. His initial
workup shows evidence of end-organ injury (ie, left ventricular hypertrophy [LVH]), so antihypertensive
treatment must be started without delay (Choice C).

First-line antihypertensive agents that can lower blood pressure and minimize LVH include angiotensin II receptor
blockers (ARBs), ACE inhibitors, calcium channel blockers, and thiazide diuretics. However, comorbid gout in this
patient makes thiazide diuretics (Choice D) a poor choice because they can raise uric acid levels and increase the
risk of gout attack. In contrast, the ARB losartan has a mild uricosuric effect, which decreases the risk of gout
attack. In addition, ARBs provide the greatest reduction in LVH of all first-line antihypertensive agents. Other
antihypertensive options for patients with gout include other ARBs, ACE inhibitors, and calcium channel blockers.

(Choice A) Discontinuing allopurinol would not decrease the patient's blood pressure or address LVH, putting him
at increased risk for stroke, myocardial infarction, and additional hypertensive damage. Without allopurinol, the
patient's uric acid level would increase, causing more frequent gout attacks.

(Choice B) Exercise stress testing is used to evaluate cardiac ischemia; this patient has no evidence of ischemia
(eg, chest pain, dyspnea, ST-segment depression). Echocardiography should be considered to further assess LVH,
but obtaining this test should not delay initiation of blood pressure medication.

Educational objective:
Losartan is an angiotensin II receptor blocker (ARB) that has a mild uricosuric effect and is the preferred first-line
antihypertensive drug for patients with gout. Other options include other ARBs, ACE inhibitors, and calcium
channel blockers. Thiazide diuretics can raise circulating uric acid levels and should be avoided.

Reference
• Antihypertensive drugs and risk of incident gout among patients with hypertension: population based case-
control study.
Question #27

A 43-year-old woman complains of fatigue and shortness of breath over the last 2 weeks. She has no chest pain,
nausea, vomiting, or weight loss. She reports a recent upper respiratory tract infection. The patient does not use
tobacco, alcohol, or illicit drugs. Her blood pressure is 98/55 mm Hg and pulse is 105/min. Jugular veins are
distended with the patient in the seated position. Lungs are clear to auscultation. Chest x-ray is shown below.
Which of the following is the most likely additional finding in this patient?

A) Audible fourth heart sound

B) Fixed splitting of the second heart sound

C) Intermittent ptosis

D) Nonpalpable point of maximal impulse

E) Opening snap

F) Pulsus bisferiens
Explanation
Correct Answer:

D) Nonpalpable point of maximal impulse

This patient's clinical presentation - recent upper respiratory tract infection, dyspnea, elevated jugular venous
pressure (JVP), clear lung fields, and increased cardiac silhouette on chest x-ray - is suggestive of early cardiac
tamponade due to a large pericardial effusion. Pericardial effusion is most commonly idiopathic (thought to be
likely from a viral illness) but can also occur due to bacterial or fungal infections, neoplasms (metastatic or primary),
post myocardial infarction, trauma, uremia, autoimmune diseases, and hypothyroidism.

Large pericardial effusions typically appear on chest x-ray as an enlarged and globular cardiac silhouette with
clear lung fields. Physical examination findings in pericardial effusion are nonspecific and vary depending on size
of the effusion. However, the inability to palpate the point of maximal apical impulse is consistent with large
pericardial effusion. Patients with cardiac tamponade may also have Beck's triad of hypotension, elevated JVP, and
muffled heart sounds.

(Choice A) Patients with viral myocarditis may present similarly with a history of recent upper respiratory infection,
fatigue, dyspnea, elevated JVP, and cardiomegaly on imaging. However, patients typically have an audible S3 and
bibasilar rales on physical examination, along with pulmonary vascular congestion on imaging studies. An
abnormal S4 may be heard in patients with decreased left ventricular compliance, including those with hypertensive
heart disease, aortic stenosis, or hypertrophic cardiomyopathy, and during the acute phase of myocardial infarction.

(Choice B) Fixed splitting of the second heart sound is heard in patients with atrial septal defect. Chest x-ray may
show an enlarged right atrium and ventricle along with prominent hilar or proximal pulmonary arterial vasculature.

(Choice C) Intermittent ptosis can be seen in patients with myasthenia gravis, which can occur as an autoimmune
disorder or as a paraneoplastic disorder in association with thymoma. These tumors present as anterior
mediastinal (or retrosternal) masses on lateral chest x-ray projection. Cardiac size and silhouette are typically
normal.
(Choice E) An opening snap is heard in patients with mitral stenosis. Significant mitral stenosis leads to left atrial
enlargement, which presents on chest x-ray as straightening of the left heart border, elevated left main bronchus,
and pulmonary vascular congestion with cephalization of the blood flow to upper lobes. Lateral chest x-ray
projection can reveal posterior displacement and impingement of the esophagus.

(Choice F) Pulsus bisferiens (or biphasic pulse) refers to 2 strong systolic peaks of the aortic pulse from left
ventricular ejection separated by a midsystolic dip. It can be palpated in patients with significant aortic
regurgitation (with or without aortic stenosis), hypertrophic obstructive cardiomyopathy, and, occasionally, large
patent ductus arteriosus.

Educational objective:
A pericardial effusion appears as an enlarged, globular cardiac silhouette on chest x-ray. Physical examination
findings of effusion without cardiac tamponade include diminished heart sounds on auscultation and a maximal
apical impulse that is difficult to palpate.

Reference
• Clinical clues to the causes of large pericardial effusions.

• A systematic approach to evaluation of pericardial effusion and cardiac tamponade.


Question #28

A 42-year-old woman comes for follow-up after a recent urinary tract infection treated with a short course of
antibiotics. During her emergency department visit, the patient was told that her blood pressure was "mildly
elevated." She currently has no symptoms. Medical history is otherwise insignificant, and the patient does not
regularly see a physician. Family history is unremarkable. The patient jogs twice a week and eats a healthy diet.
She takes no medications. Blood pressure is 142/91 mm Hg. BMI is 28 kg/m2. Physical examination is
unremarkable. ECG and laboratory tests, including a basic metabolic panel, are unremarkable. Which of the
following is the best next step in managing this patient?

A) Advise weight loss and follow-up in 12 months

B) Obtain multiple home blood pressure readings

C) Obtain polysomnography

D) Start double antihypertensive therapy

E) Start single antihypertensive agent


Explanation
Correct Answer:

B) Obtain multiple home blood pressure readings

This patient has had 2 separate episodes of mildly elevated blood pressure (BP) while she was in a medical
setting (in the office and the emergency department), which suggest hypertension. However, office-based BP
measurements are a poor predictor of high BP outside of the office because of the relatively common occurrence of
white coat hypertension (eg, BP 5-10 mm Hg higher in the office than at home). Therefore, in the absence of end-
organ damage, the diagnosis of hypertension must be confirmed by one of the following:

• Ambulatory BP readings, as measured by an automatic device worn continuously by the patient for 24-48
hours. The device monitors BP at regular intervals (eg, 15-60 min).

• If continuous ambulatory BP monitoring is not available, an acceptable alternative is home BP monitoring


done twice a day (morning and evening) for a week.

• If home BP monitoring is not possible, 3 office readings (preferably by an automated machine while the
patient is alone) at least a week apart are needed.

In contrast, if end-organ damage (eg, left ventricular hypertrophy, retinopathy, nephropathy) is present or if the
patient has a systolic BP ≥180 mm Hg or diastolic BP ≥120 mm Hg, then the diagnosis of hypertension can be
established in the office without additional confirmation, and therapy should be started quickly. This patient has
neither end-organ damage nor confirmed hypertension, so antihypertensive therapy does not need to be started yet
(Choices D and E).

(Choice A) Weight loss is unlikely to improve BP significantly in this nonobese patient who is already exercising
and following a good diet. In addition, waiting 12 months for an evaluation is too long and risks the patient being
lost to follow-up or developing end-organ damage; a shorter interval is advised.

(Choice C) Polysomnography is used to diagnose obstructive sleep apnea (OSA), which is associated with
hypertension and obesity. This patient does not have confirmed hypertension, and although she is overweight, she
is not at high risk for OSA. In general, OSA screening is typically considered in patients with excessive daytime
somnolence who have 2 other clinical manifestations (eg, hypertension, loud snoring, choking while asleep).

Educational objective:
Patients with severe hypertension or evidence of end-organ damage should be prescribed antihypertensive therapy
immediately. For those with only mild blood pressure elevations, the diagnosis should first be confirmed outside of
a health care setting with ambulatory blood pressure monitoring or home measurement.

Reference
• 2017 ACC/AHA Blood Pressure Guideline Treatment Recommendations and Risk For Cardiovascular
Events And All-Cause Mortality.

• What is the evidence base for diagnosing hypertension and for subsequent BP treatment targets in the
prevention of cardiovascular disease?
Question #29

A 24-year-old woman comes to the office due to pressure-like, substernal chest pain that occurs when she
exercises. The patient began noticing the pain approximately 6 months ago when she started to exercise to lose
weight. Prior to that, her lifestyle was largely sedentary. She has no associated nausea, vomiting, diaphoresis,
dyspnea, palpitations, or syncope. The patient has no significant past medical history but was told as a child that
she has a "murmur." She has no significant family history of heart disease. Blood pressure is 130/70 mm Hg on the
right and 105/55 mm Hg on the left, and pulse is 72/min and regular. BMI is 29 kg/m2. A palpable thrill is present in
the suprasternal notch. There is a loud midsystolic murmur best heard at the first right intercostal space. The lungs
are clear to auscultation. What is the most likely cause of this patient's chest pain?

A) Anomalous origin of the right coronary artery

B) Atherosclerotic narrowing of the coronaries

C) Increased myocardial oxygen demand

D) Stretching of the papillary muscles

E) Systolic anterior motion of the mitral valve


Explanation
Correct Answer:

C) Increased myocardial oxygen demand

Supravalvular aortic stenosis (AS), the second most common type of AS, usually refers to congenital left
ventricular outflow tract obstruction due to discrete or diffuse narrowing of the ascending aorta. This causes a
systolic murmur similar to that seen with valvular AS; however, the murmur is usually best heard at the first right
intercostal space, higher than where the valvular AS murmur is best heard. Patients may also have unequal carotid
pulses, differential blood pressure in the upper extremities (high-pressure jet in ascending aorta), and a palpable
thrill in the suprasternal notch.

Patients with significant supravalvular AS develop left ventricular hypertrophy over time and can also have
coronary artery stenosis as an associated anomaly. These changes, along with the increase in myocardial
oxygen demand during exercise, can lead to subendocardial or myocardial ischemia, which is likely responsible for
this patient's anginal symptoms during exercise.

(Choice A) An anomalous right coronary artery that courses between the aortic root and the main pulmonary trunk
can become compressed during exercise, leading to anginal symptoms. However, no associated murmurs or
palpable thrill would be expected.

(Choice B) Atherosclerotic coronary artery disease is unlikely in this patient given her young age and lack of
cardiovascular risk factors other than obesity.

(Choice D) Stretching of the papillary muscles can lead to mitral regurgitation, which causes a holosystolic or mid
to late systolic murmur best heard at the cardiac apex. Exertional angina is not typically seen.

(Choice E) Systolic anterior motion of the mitral valve is seen in patients with hypertrophic obstructive
cardiomyopathy (HOCM). Patients with HOCM can have exertional angina, dyspnea, dizziness, presyncope, or
syncope. However, the systolic murmur of HOCM has maximal intensity along the lower left sternal border and
typically changes in intensity with various physiologic maneuvers.
Educational objective:
Patients with aortic outflow obstruction from supravalvular aortic stenosis develop left ventricular hypertrophy and
can have exertional angina due to subendocardial ischemia with increased myocardial oxygen demand during
exercise.

Reference
• Pathology of coronary arteries, myocardium, and great arteries in supravalvular aortic stenosis. Report of
five cases with implications for surgical treatment.
Question #30

A 38-year-old man comes to the office for follow-up after his blood pressure was found to be elevated at a health
fair. The patient has had no headache, blurry vision, or chest pain. He does not use tobacco, alcohol, or illicit
drugs. Temperature is 36.7 C (98.1 F), blood pressure is 148/96 mm Hg in the right arm and 146/94 mm Hg in the
left, pulse is 74/min, and respirations are 14/min. BMI is 36 kg/m2. Examination shows a regular heart rate and
rhythm with no heart murmurs or carotid and abdominal bruits. Peripheral pulses are 2+ without radiofemoral
delay. Breath sounds are normal. ECG is normal, and a 48-hour ambulatory blood pressure monitor shows an
average pressure of 138/88 mm Hg. Which of the following is the best next step in management of this patient?

A) Order 24-hour urinary free cortisol

B) Order plasma renin and aldosterone

C) Order serum creatinine

D) Order transthoracic echocardiography


Explanation
Correct Answer:

C) Order serum creatinine

Laboratory evaluation of hypertension

• Serum electrolytes (Na, K, Ca)


• Serum creatinine
Renal function tests
• Urinalysis
• Urine albumin/creatinine ratio (optional)

• Fasting glucose or hemoglobin A1c


Endocrine tests • Lipid profile
• TSH

• ECG
Cardiac tests
• Echocardiography (optional)

• Complete blood count


Other tests
• Uric acid (optional)

This patient has a new diagnosis of hypertension confirmed with office and ambulatory blood pressure
measurements. Diagnostic criteria for hypertension vary according to different guidelines, but a blood pressure
persistently at ≥130 mm Hg systolic and/or ≥80 mm Hg diastolic is generally considered hypertensive.

The initial evaluation of hypertension is directed to rule out common secondary causes, end-organ complications,
and additional risk factors that may influence management. Hypertension is a major risk factor for chronic kidney
disease, and conversely, kidney disease can cause secondary hypertension. Therefore, patients' renal function,
including serum creatinine, electrolytes, and urinalysis, should be assessed at the time of diagnosis.

Because the risk of cardiovascular complications is greater in patients with comorbid hyperlipidemia or diabetes
mellitus, a blood glucose (or hemoglobin A1c) and lipid panel are indicated. Also, many experts recommend a
serum TSH for all patients with hypertension because thyroid disorders are common and often present with
asymptomatic hypertension.

(Choices A and B) Screening for infrequent secondary causes is needed only for patients with an atypical
presentation (eg, onset at age <30), resistant hypertension (ie, uncontrolled despite an appropriate 3-drug regimen),
or specific signs of a secondary hypertensive disorder. Examples of signs of a secondary hypertension include
hypokalemia, suggesting hyperaldosteronism and confirmed with a plasma renin/aldosterone ratio, or atypical body
fat distribution, suggesting Cushing syndrome and confirmed with 24-hour urinary free cortisol.

(Choice D) ECG is recommended in the initial evaluation of hypertension to rule out common cardiovascular
complications (eg, left ventricular hypertrophy, myocardial infarction). However, in the absence of abnormal
examination (eg, murmur) or ECG findings, more advanced testing (eg, echocardiography) is not generally
necessary.

Educational objective:
Hypertension is a major risk factor for chronic kidney disease and conversely, kidney disease can cause secondary
hypertension. Therefore, patients with hypertension should have their renal function (including serum creatinine,
electrolytes, and urinalysis) assessed at the time of diagnosis.

Reference
• 2017 ACC/AHA/AAPA/ABC/ACPM/AGS/APhA/ASH/ASPC/NMA/PCNA guideline for the prevention,
detection, evaluation, and management of high blood pressure in adults: executive summary: a report of the
American College of Cardiology/American Heart Association Task Force on Clinical Practice Guidelines.
Question #31

A 38-year-old woman comes to the physician complaining of occasional palpitations. She describes a fast and
irregular heartbeat. She has had 3 such episodes over the past 2 months, each lasting about 2 hours. The patient
has no associated chest pain, shortness of breath, cough, or ankle swelling. She drinks alcohol on social occasions
and does not smoke. She has no other medical problems and takes no medications. Her temperature is 37.1°C
(98.9°F), blood pressure is 130/70 mm Hg, pulse is 80/min, and respirations are 14/min. The apical impulse is
displaced to the left, and a third heart sound is heard at the apex in the left decubitus position. There is also a
holosystolic murmur that is loudest at the apex and radiates to the axilla. Which of the following is the most likely
cause of this patient's condition?

A) Bacterial infection of the mitral valve

B) Mitral annular calcification

C) Myocardial ischemia

D) Myxomatous degeneration of the mitral valve

E) Rheumatic mitral valve disease


Explanation
Correct Answer:

D) Myxomatous degeneration of the mitral valve

This patient's examination findings (displaced apical impulse, holosystolic murmur, and third heart sound) are
consistent with chronic severe mitral regurgitation (MR). Mitral valve prolapse (MVP) is the most common cause of
chronic MR in developed countries. MVP occurs due to myxomatous degeneration of the mitral valve leaflets and
chordae and causes a mid-systolic click followed by a mid-to-late systolic murmur. As the severity of leaflet
dysfunction and MR worsens, the murmur becomes holosystolic and the click may not be audible. The third heart
sound is from left ventricular volume overload and does not necessarily represent congestive heart failure. Chronic
severe MR can cause left atrial dilation, which can eventually lead to atrial fibrillation (symptoms of palpitations).

(Choice A) Infective endocarditis can cause MR due to inadequate leaflet coaptation or, rarely, leaflet perforation.
It is unlikely in this patient, who has no accompanying signs or symptoms (eg, fever, embolic findings) suggesting
infective endocarditis.

(Choice B) Mitral annular calcification refers to a degenerative process involving the fibrous annulus of the mitral
valve. It is a common incidental finding in older adults and is usually associated with mild-to-moderate MR. Mitral
annular calcification is less likely to cause significant MR in a younger patient.

(Choice C) Myocardial ischemia can cause papillary muscle dysfunction, which in turn can cause MR. It usually
presents with chest discomfort, dyspnea, or sudden onset of pulmonary edema.

(Choice E) Rheumatic heart disease can cause mitral stenosis, MR, and aortic regurgitation. However, it is a much
less common cause of chronic MR than MVP.

Educational objective:
Mitral valve prolapse is the most common cause of mitral regurgitation (MR) in developed countries. It usually
causes mild MR with mid-systolic click and mid-to-late systolic murmur. Patients with severe leaflet dysfunction and
prolapse can develop severe MR and holosystolic murmur on physical examination. Chronic severe MR causes left
atrial and ventricular enlargement leading to atrial fibrillation, left ventricular dysfunction, and congestive heart
failure.

Reference
• Mitral valve prolapse.
Question #32

A 72-year-old man comes to the office due to palpitations for the past 1 week. The patient has had no fever, cough,
chest pain, or dizziness. Medical history is significant for mild chronic obstructive pulmonary disease, long-standing
hypertension, hyperlipidemia, and chronic stable angina. He uses an albuterol/ipratropium inhaler as needed. The
patient is a former smoker; he does not use alcohol but drinks 2 or 3 cups of coffee daily. Temperature is 37.1 C
(98.8 F), blood pressure is 136/78 mm Hg, pulse is 116/min, respirations are 16/min. Oxygen saturation is 97% on
room air. On physical examination, the patient appears comfortable with normal work of breathing. Lung
auscultation reveals mildly prolonged expiration with no wheezing or crackles. Cardiac examination shows irregular
tachycardia. The remainder of the physical examination is unremarkable. ECG rhythm strip is shown in the
exhibit. Which of the following most likely increased the risk of development of this patient's arrhythmia?

A) Acquired long QT syndrome

B) Beta-2 agonist use

C) Caffeine consumption

D) Chronic hypertension

E) Coronary artery disease


Explanation
Correct Answer:

D) Chronic hypertension

Comorbidities that encourage atrial fibrillation

• Advanced age
• Systemic hypertension
• Mitral valve dysfunction
Precipitants of atrial dilation &/or • Left ventricular failure
conduction remodeling • Coronary artery disease & related factors (eg, DM, smoking)
• Obesity & obstructive sleep apnea

• Chronic hypoxic lung disease (eg, COPD)

• Hyperthyroidism
• Excessive alcohol use
• Increased sympathetic tone
Triggers of increased automaticity ◦ Acute illness (eg, sepsis, PE, MI)
◦ Cardiac surgery

• Sympathomimetic drugs (eg, cocaine)

COPD = chronic obstructive pulmonary disease; DM = diabetes mellitus; MI = myocardial infarction;


PE = pulmonary embolism.

This patient's ECG rhythm strip shows a rapid rhythm with irregularly irregular R-R intervals and an absence of
organized P waves, consistent with atrial fibrillation with rapid ventricular response (ie, rate >100/min). The
pathogenesis of atrial fibrillation involves an underlying substrate created by atrial remodeling, or changes in the
atrial structure and conduction system that result from aging and comorbidities. Chronic hypertension is the
most common comorbidity associated with atrial fibrillation and is likely one of the strongest contributors to atrial
remodeling.

Most of the contribution of chronic hypertension to atrial remodeling is likely via left atrial dilation. With the left
ventricle pumping against chronically increased pressure load, compensatory concentric left ventricular (LV)
hypertrophy develops, which decreases LV diastolic filling. Given the thin left atrial walls, even subtle pressure
increases transmitted back to the left atrium can have a significant effect on left atrial stretching and dilation. The
left atrial stretching likely also contributes to atrial fibrosis and some degree of atrial conduction remodeling.

Chronic obstructive pulmonary disease (COPD) can also contribute to atrial fibrillation in several ways. Severe
COPD associated with significant hypoxemia can cause right atrial dilation due to hypoxic pulmonary
vasoconstriction and resulting pulmonary hypertension. In addition, COPD exacerbation can increase sympathetic
tone and act as a trigger for atrial fibrillation. Beta-2 agonists (eg, albuterol, salmeterol) used for the treatment of
COPD may also be occasional triggers of atrial fibrillation. However, the underlying substrate (created by chronic
hypertension in this patient) is a more important factor (Choice B).

(Choice A) Acquired long QT syndrome typically results from medications that prolong the QT interval. It increases
the risk of polymorphic ventricular tachycardia (ie, torsade de pointes), but it is not a contributor to atrial fibrillation.

(Choice C) Despite the common belief that caffeine consumption is considered a trigger for atrial fibrillation and
other arrhythmias (due to increased sympathetic tone), studies have not shown any significant association.

(Choice E) Coronary artery disease (CAD) and its associated risk factors (eg, diabetes mellitus, smoking) likely
somewhat contribute to atrial remodeling but less so than chronic hypertension. In patients with atrial fibrillation, the
association of CAD with the arrhythmia is primarily seen among those who develop acute myocardial infarction or
ischemic heart failure.

Educational objective:
Chronic hypertension is the most common comorbidity associated with atrial fibrillation and is likely one of the
strongest contributors to the predisposing atrial remodeling. Aging is also a strong contributor.

Reference
• The natural history of atrial fibrillation: incidence, risk factors, and prognosis in the Manitoba follow-up study.

• Atrial fibrillation in hypertension: predictors and outcome.


Question #33

A 54-year-old man comes to the physician because of edema of his right ankle. He reports heaviness and
cramping in the same leg that is worse after a long day at work. The swelling is usually reduced significantly when
he wakes up in the morning and worsens progressively throughout the day. He denies any other symptoms. He
has no significant medical problems except hypertension, for which he takes atenolol. His temperature is 36.7° C
(98° F), blood pressure is 120/76 mm Hg, pulse is 80/min and respirations are 16/min. JVP is normal. Lungs are
clear to auscultation. There are no murmurs. There is no hepatosplenomegaly. Examination shows edema of the
right ankle. Doppler examination of the leg shows no evidence of thrombosis. Which of the following is the most
likely cause of his edema?

A) Lymphatic obstruction

B) Impaired cardiac contraction

C) Reduced diastolic filling of the heart

D) Increased urinary loss of protein

E) Venous valve incompetence

F) Decreased liver protein synthesis

G) Arterial occlusion
Explanation
Correct Answer:

E) Venous valve incompetence

The patient described is experiencing unilateral lower extremity edema that worsens when the leg is dependent (i.e.
while the patient is at work) and improves with leg elevation (i.e. when the patient is sleeping). There are many
potential causes of lower extremity edema. Indeed, all of the above mentioned conditions may cause edema of the
legs, but in the clinical scenario provided, venous valvular incompetence is the most likely cause. Venous
insufficiency is the most common cause of lower extremity edema; it affects approximately 2% of the population at
large. Failure of venous valves allows blood to pool in dependent areas such as the legs resulting in an increase in
capillary hydrostatic pressure. This increased pressure favors increased filtration of fluid out of the capillaries into
the interstitial tissue. This process causes a decrease in intravascular volume, which stimulates the kidneys to
retain water and salt ultimately leading to further progression of edema.

(Choice A) Lymphatic obstruction is an uncommon cause of edema. It may result from malignant obstruction of
lymph nodes, lymph node resection, trauma and filariasis. It classically affects the dorsa of the feet and causes
marked thickening and rigidity of the skin.

(Choices B & C) Impaired cardiac contraction and reduced diastolic filling of the heart may cause bilateral lower
extremity edema due to pooling of blood in the venous circulation causing increased capillary hydrostatic pressure.
However, respiratory symptoms and crackles on examination are common.

(Choices D & F) Increased urinary loss of protein and decreased liver protein synthesis causes decreased plasma
oncotic pressure, which results in decreased reabsorption of interstitial fluid in distal capillaries and edema. Urinary
protein loss occurs in nephrotic syndromes as well as in most cases of nephritis. Failure of liver protein synthesis
typically occurs in the setting of cirrhosis or other forms of liver failure where the synthetic function of the liver is
impaired. Other signs of liver failure and nephrotic syndrome are usually evident.

(Choice G) Arterial occlusion causes pain, pallor, paresthesias, pulselessness and coolness to the touch in affected
extremities.

Educational objective:
Venous insufficiency (valvular incompetence) is the most common cause of lower extremity edema. It classically
worsens throughout the day and resolves overnight when the patient is recumbent.
Question #34

A 76-year-old woman comes to the office due to intermittent palpitations for the last 6 months. The patient
describes episodes of her heart beating fast. These happen on average once a week, last up to an hour, and
resolve spontaneously. She cannot identify any provoking factors but thinks they occur typically when she is tired.
The patient has had no chest pain, dyspnea, syncope, or lower extremity swelling. She has a history of
hypertension. The patient is a lifetime nonsmoker. Blood pressure is 145/85 mm Hg, and pulse is 75/min and
regular. There are no heart murmurs. The lungs are clear on auscultation. Peripheral pulses are full and
symmetric. ECG shows normal sinus rhythm. Echocardiogram shows moderate left atrial enlargement, concentric
left ventricular hypertrophy, ejection fraction of 65%, and no valvular abnormalities. Further workup is most likely to
identify which of the following in this patient?

A) Atrial fibrillation

B) Atrioventricular nodal reentrant tachycardia

C) Second-degree atrioventricular block

D) Sinus node dysfunction

E) Ventricular tachycardia
Explanation
Correct Answer:

A) Atrial fibrillation

Comorbidities that encourage atrial fibrillation

• Advanced age
• Systemic hypertension
• Mitral valve dysfunction
Precipitants of atrial dilation &/or • Left ventricular failure
conduction remodeling • Coronary artery disease & related factors (eg, DM, smoking)
• Obesity & obstructive sleep apnea

• Chronic hypoxic lung disease (eg, COPD)

• Hyperthyroidism
• Excessive alcohol use
• Increased sympathetic tone
Triggers of increased automaticity ◦ Acute illness (eg, sepsis, PE, MI)
◦ Cardiac surgery

• Sympathomimetic drugs (eg, cocaine)

COPD = chronic obstructive pulmonary disease; DM = diabetes mellitus; MI = myocardial infarction;


PE = pulmonary embolism.

Atrial fibrillation (AF) is the most common cardiac arrhythmia, with a prevalence that increases dramatically with
age; approximately 70% of those affected are age ≥65. Patients often have episodes of paroxysmal (intermittent)
AF in the initial stages and progress to persistent or permanent AF over time. Those affected may be asymptomatic
or have palpitations, dyspnea, and/or fatigue.

Atrial enlargement (as seen on this patient's echocardiogram) and subsequent atrial remodeling likely play an
important role in the development of AF, and patients usually have some form of underlying heart disease that
contributes to this pathophysiologic mechanism. Hypertension is the most common underlying comorbidity,
contributing to left atrial enlargement via concentric left ventricular hypertrophy and transmission of increased
pressure to the left atrium.

AF is recognized on ECG by an irregularly irregular rhythm with varying R-R intervals and the absence of P waves.
However, due to the intermittent nature of paroxysmal AF, patients are commonly in normal sinus rhythm (with a
regular pulse) at the time of evaluation; therefore, a single ECG often fails to make the diagnosis and continuous
ambulatory ECG monitoring (eg, for 24 hours) may be needed.

(Choice B) Atrioventricular nodal reentrant tachycardia (AVNRT) and atrioventricular reentrant tachycardia (AVRT)
are types of paroxysmal supraventricular tachycardia, which have a regular rhythm. These arrhythmias typically
present with episodes of palpitations, dyspnea, or presyncope/syncope. They are less common than AF and
usually occur in adolescents and young adults.

(Choices C and D) Like AF, both second-degree atrioventricular block (with dropped beats) and sinus node
dysfunction (ie, sick sinus syndrome, with periods of bradycardia sometimes alternating with tachycardia) increase
in prevalence with age. However, these bradyarrhythmias are less likely than AF in this patient with intermittent
palpitations, left ventricular hypertrophy, and left atrial enlargement. When symptomatic, these bradyarrhythmias
are more likely to cause dyspnea, fatigue, or symptoms of cerebral hypoperfusion (eg, lightheadedness, syncope).

(Choice E) Ventricular tachycardia can present with intermittent palpitations, chest discomfort, and presyncope or
syncope, and it is far more likely than AF to cause sudden cardiac death. It is usually seen in patients with heart
failure with reduced ejection fraction or previous myocardial infarction.

Educational objective:
Atrial fibrillation (AF) is the most common cardiac arrhythmia, with age and comorbidities that contribute to left atrial
enlargement (eg, hypertension) being the most common risk factors. AF is often paroxysmal (intermittent), making
diagnostic confirmation via a single ECG difficult, and continuous ECG monitoring is often needed.

Reference
• Prevalence of diagnosed atrial fibrillation in adults: national implications for rhythm management and stroke
prevention: the anticoagulation and risk factors in atrial fibrillation (ATRIA) study.

• Prevalence, incidence and lifetime risk of atrial fibrillation: the Rotterdam study.
Question #35

A 75-year-old man is being evaluated for a heart murmur. He has had no exertional chest pain, shortness of breath,
palpitations, fatigue, dizziness, or syncope. The patient's only other medical condition is hypertension, for which he
takes chlorthalidone and amlodipine. He has a 20-pack-year smoking history but quit 25 years ago. Temperature is
36.9 C (98.4 F), blood pressure is 130/80 mm Hg, pulse is 80/min, and respirations are 16/min. Physical
examination shows a sustained apical impulse. S1 is normal; a single, soft S2 and an S4 are present. A grade 3/6
late-peaking systolic murmur is heard best at the right 2nd intercostal space, with radiation to the right carotid
artery. Carotid pulses are delayed. Transthoracic echocardiogram shows a thickened left ventricular wall with no
regional wall motion abnormalities. The ejection fraction is 45%. There is a trileaflet aortic valve with heavy
calcification, valve area of 0.9 cm2, and transvalvular gradient consistent with severe aortic stenosis. Which of the
following is the most appropriate next step in management of this patient?

A) Follow with serial echocardiography

B) Reassure with no additional intervention

C) Recommend aortic valve intervention

D) Recommend cardiac exercise program

E) Recommend medical therapy


Explanation
Correct Answer:

C) Recommend aortic valve intervention

Valve replacement in aortic stenosis

• Aortic jet velocity ≥4.0 m/sec, or


Severe AS criteria • Mean transvalvular pressure gradient ≥40 mm Hg
• Valve area usually ≤1.0 cm2 but not required

Severe AS & ≥1 of the following:

Indications for valve replacement • Onset of symptoms (eg, angina, syncope)


• Left ventricular ejection fraction <50%
• Undergoing other cardiac surgery (eg, CABG)

AS = aortic stenosis; CABG = coronary artery bypass grafting.

Severe aortic stenosis (AS) is diagnosed based on aortic jet velocity or mean transvalvular pressure gradient; in
most cases the valve area is ≤1 cm2, but valve area is not considered for diagnosis. Physical examination findings
that suggest severe AS include a late-peaking, crescendo-decrescendo systolic murmur best heard at the right
upper sternal border and diminished and delayed pulses (pulsus parvus et tardus). Patients with severe AS often
have symptoms of angina, syncope/presyncope, or heart failure (eg, dyspnea).
Surgical aortic valve replacement (AVR) should be considered in patients with severe AS and ≥1 of the following
criteria:

• Presence of symptoms attributable to AS: Patients with symptomatic, severe AS have a relatively high risk
of sudden cardiac death.
• Left ventricular ejection fraction (LVEF) <50%, regardless of symptoms (as in this patient with LVEF
45%): A depressed LVEF is often due to excessive afterload created by the stenotic valve, and it frequently
normalizes with AVR.
• Undergoing other cardiac surgery (eg, coronary artery bypass grafting): The valve can be repaired
concomitantly.

The definition of severe AS encompasses many patients who are asymptomatic because it was designed to
identify nearly all patients who may benefit from AVR (high sensitivity). Some of these patients are truly
asymptomatic. Others lack symptoms only because of a sedentary lifestyle; when subjected to exertion (eg, stress
testing) they have typical severe AS symptoms.

(Choice A) Serial echocardiography is appropriate in patients with severe AS who are truly asymptomatic and
have normal LVEF, but not in this patient with LVEF <50%.

(Choice B) Severe AS is a progressive disease that requires either prompt AVR or close monitoring; reassurance
alone is not appropriate.

(Choices D and E) Severe AS creates a tenuous hemodynamic situation because adequate preload is required to
maintain cardiac output and adequate diastolic pressure is required to maintain coronary artery perfusion. This
limits the use of medical therapy because diuretics (eg, furosemide) reduce preload and vasodilators (eg,
amlodipine, hydralazine, ACE inhibitors) reduce diastolic pressure, often precipitating clinical decompensation.
Cardiac exercise programs are also of limited benefit. AVR is the only therapy proven to improve survival in
patients with severe AS.

Educational objective:
In patients with severe aortic stenosis, surgical aortic valve replacement is indicated in all symptomatic patients. It
is also indicated in asymptomatic patients with left ventricular ejection fraction <50% and those undergoing other
cardiac surgery.
Reference
• 2014 AHA/ACC guideline for the management of patients with valvular heart disease: a report of the
American College of Cardiology/American Heart Association task force on practice guidelines.
Question #36

A 60-year-old woman comes to the office due to several months of lower extremity swelling. Medical history is
significant for hypertension, type 2 diabetes mellitus, and hepatitis C infection. The patient was also diagnosed with
latent tuberculosis 10 years ago, but she declined antibiotic therapy. Blood pressure is 120/80 mm Hg and pulse is
90/min. Physical examination shows symmetric pitting edema of the lower extremities. The liver is palpated 4 cm
below the costal margin, and ascites is present. The tip of the spleen is palpated on deep inspiration.
Hepatojugular reflux is present when sustained pressure is applied to the upper abdomen. The lungs are clear on
auscultation. Which of the following findings is most suggestive of a cardiac cause for this patient's edema?

A) Ascites

B) Clear lungs

C) Hepatojugular reflux

D) Hepatomegaly

E) Lower extremity edema

F) Splenomegaly
Explanation
Correct Answer:

C) Hepatojugular reflux
Hepatojugular reflux (or abdominojugular reflux) is elicited by applying firm, sustained pressure to the upper
abdomen for 10-15 seconds. A positive response is defined as a sustained elevation in jugular venous
pressure (JVP) >3 cm and is highly specific for elevated right-sided cardiac pressures and right ventricular (RV)
failure.

With RV failure, elevated hydrostatic pressure backs up through the inferior vena cava (IVC), hepatic veins, portal
sinusoids, portal vein, and splanchnic circulation (eg, splenic vein, inferior mesenteric vein), creating a continuous
column of elevated hydrostatic pressure. Compression of the upper abdomen forces this continuous column of
blood back toward the heart; because the failing right ventricle cannot handle the increased venous return, the
blood backs up into the jugular veins and causes a perceptible rise in JVP. This patient with a positive
hepatojugular reflux may have constrictive pericarditis (a common cause of right-sided heart failure) due to
reactivation of her latent tuberculosis.

Ascites, splenomegaly, and lower extremity edema are commonly seen with RV failure, but they are nonspecific
findings because they are also seen with liver cirrhosis (eg, from hepatitis C) (Choices A, E, and F). Hepatojugular
reflux is not present with liver cirrhosis because the pressure backup starts at the liver and there is no continuous
column of elevated hydrostatic pressure within the IVC.

(Choice B) Clear lungs are expected with both liver cirrhosis and isolated RV failure and are not useful in
differentiating these disease processes. Clear lungs are helpful in differentiating isolated RV failure from left
ventricular (LV) failure, as pulmonary edema is expected with LV failure. A positive hepatojugular reflex is
commonly present with LV failure once hydrostatic pressure backs up through the lungs to cause RV failure.

(Choice D) Hepatomegaly is expected with RV failure and is not expected with liver cirrhosis (cirrhosis typically
causes a shrunken liver). However, hepatomegaly is less specific for RV failure than hepatojugular reflux because
hepatomegaly can also occur with an obstruction to hepatic vein or IVC blood flow (eg, hepatic vein thrombosis
[Budd-Chiari syndrome]).

Educational objective:
Hepatojugular reflux is useful for differentiating between cardiac disease– and liver disease–related causes of
ascites, splenomegaly, and lower extremity edema. Positive hepatojugular reflux (ie, sustained >3-cm rise in jugular
venous pressure elicited by compression of the upper abdomen) is highly specific for right ventricular failure.
Hepatojugular reflux is not expected with liver cirrhosis.
Reference
• The abdominojugular reflux sign.

• Hepatojugular reflux.
Question #37

A 34-year-old woman comes to the office for a follow-up blood pressure check. Over the last 12 months, the
patient's blood pressure readings have ranged from 145/90 to 150/95 mm Hg. Her blood pressure was measured
during a health fair 2 weeks ago and it was 145/90 mm Hg. The patient feels well, has no other medical conditions,
and has never been pregnant. Medications include a combination oral contraceptive for the past 5 years and,
occasionally, acetaminophen for relief of tension headaches. She does not use tobacco, alcohol, or illicit drugs.
There is no family history of hypertension, stroke, deep venous thrombosis, or heart attack. Blood pressure is 150/
90 mm Hg and pulse is 80/min. BMI is 22 kg/m2. Physical examination and ECG are normal. Six months ago, the
patient's total cholesterol level was 170 mg/dL. Complete blood count, urinalysis, and basic metabolic panel are
unremarkable. Which of the following is the most appropriate next step in management of this patient's
hypertension?

A) Initiate a low-dose thiazide diuretic

B) Perform a CT angiogram of the abdomen

C) Prescribe an alternate form of contraception

D) Reassure the patient; no intervention is required

E) Recommend a diet and exercise regimen


Explanation
Correct Answer:

C) Prescribe an alternate form of contraception

This patient has stage 2 hypertension, defined as systolic blood pressure ≥140 mm Hg and/or diastolic blood
pressure ≥90 mm Hg. Oral contraceptives (OCs) commonly cause a mild elevation in blood pressure and
sometimes (in up to 5% of chronic users) can lead to overt hypertension. The mechanism is unclear but is possibly
due to an estrogen-mediated increase in hepatic angiotensinogen synthesis or other effects on the renin-
angiotensin system. The risk of hypertension increases with the duration of OC use and is elevated in those who
have a family history of hypertension or who developed hypertension during a previous pregnancy.

The Centers for Disease Control and Prevention recommends against the use of combination estrogen-progestin
OCs in women with hypertension. In patients whose hypertension is caused by OC use, discontinuation can reduce
blood pressure and often corrects the condition. The physician should have a discussion with the patient about the
benefit of stopping the OC, the risks of drug continuation, and alternate contraceptive methods.

(Choices A and E) If elevated blood pressure persists after OC discontinuation, lifestyle modification and/or
medication (eg, low-dose thiazide diuretic) may be required.

(Choice B) Renovascular hypertension and fibromuscular dysplasia are uncommon causes of hypertension;
therefore, routine CT angiogram of the abdomen is not recommended. Investigation for secondary causes of
hypertension is considered for patients with more severe or refractory hypertension or with other signs of
renovascular disease (eg, acute kidney injury after initiation of an ACE inhibitor, recurrent flash pulmonary edema).

(Choice D) Reassurance without intervention is inappropriate because hypertension is an important cardiovascular


risk factor that should be treated properly to reduce the risk of cardiovascular events.

Educational objective:
Oral contraceptives (OCs) commonly cause a mild elevation in blood pressure and can sometimes lead to overt
hypertension. OCs should not be used in patients with hypertension, and those who develop hypertension while
taking OCs should discontinue the medication.

Reference
• Hypertension in women.

• Drug-induced hypertension: an unappreciated cause of secondary hypertension.


Question #38

A 56-year-old woman comes to the office due to left lower extremity pain and swelling. One year ago, the patient
had a provoked left femoropopliteal deep venous thrombosis and completed 6 months of antithrombotic therapy.
Over the past 3 months, she has had slowly worsening intermittent pain, fatigue, and swelling of the left leg,
especially toward the end of the day. Medical history is significant for obesity and tobacco use. Temperature is 36.6
C (97.9 F), blood pressure is 122/74 mm Hg, pulse is 83/min, and respirations are 14/min. Oxygen saturation is
98% on room air. Mild pitting edema is noted on left lower extremity examination. There are scattered dilated
superficial veins, but no skin erythema or calf tenderness is present. The right lower extremity has no
abnormalities. Lower extremity venous ultrasonography is negative for thrombus. Which of the following is the
most appropriate next step in management of this patient?

A) Arterial Doppler ultrasonography

B) Exercise and compression therapy

C) Inferior vena cava filter placement

D) Long-term antithrombotic therapy

E) Salt restriction and diuretics

F) Smoking cessation and cilostazol


Explanation
Correct Answer:

B) Exercise and compression therapy

Postthrombotic syndrome

• Chronic venous insufficiency following


acute DVT
Epidemiology
• Most cases arise within 2 years of
thrombus

• Leg edema, fatigue, pain, superficial


Manifestations venous dilation, venous stasis ulcers
• Often worse at the end of the day

• Exercise (eg, ankle flexion, walking)


Treatment • Compression (eg, bandages,
stockings)

DVT = deep vein thrombosis.

This patient had an acute left lower extremity deep venous thrombosis (DVT) a year ago and now has left leg pain,
fatigue, and edema. Although these manifestations could indicate recurrent DVT, ultrasound examination is
negative for lower extremity thrombus. Therefore, the most likely diagnosis is postthrombotic syndrome, a
disorder characterized by chronic venous insufficiency following acute DVT.

Approximately 50% of patients with acute DVT develop postthrombotic syndrome within 2 years due to venous
hypertension distal to the site of previous thrombus. This is thought to arise due to venous valve damage from the
release of inflammatory mediators and venous lumen stenosis due to wall remodeling. Increased venous pressure
typically causes progressive lower extremity pain, fatigue, edema, superficial venous dilation, trophic skin changes,
and/or ulcers.

Patients with suspected postthrombotic syndrome often undergo Doppler ultrasonography to rule out acute DVT
because the symptoms can overlap. Treatment of postthrombotic syndrome should be initiated to prevent the
development of venous stasis ulcers; exercise (eg, daily walking, ankle flexion exercises) and compression (eg,
stockings, bandages) help improve venous return and are the mainstays of therapy.

(Choices A and F) Patients with peripheral artery disease often have leg pain that worsens with activity and
improves with rest (claudication); nonhealing ulcers or gangrene can also occur. Medical management often
includes smoking cessation and cilostazol, an arterial vasodilator (eg, for those with no improvement with
supervised exercise therapy). This patient's leg pain, edema, and history of recent DVT make postthrombotic
syndrome far more likely.

(Choices C and D) Inferior vena cava (IVC) filter placement is usually indicated for those who have lower extremity
thrombus and a contraindication to antithrombotic therapy (eg, gastrointestinal bleed, recent cranial hemorrhage).
Long-term anticoagulation is often indicated for those with recurrent thrombus, thrombus in the setting of
malignancy, or hypercoagulable state with thrombus. This patient has no evidence of lower extremity thrombus;
additional anticoagulation and IVC filter are not indicated at this time.

(Choice E) Salt restriction and diuretics can improve volume overload in cirrhosis or heart failure, which often
causes bilateral lower extremity pitting edema. This patient's unilateral edema following DVT indicates localized
venous hypertension, not excessive intravascular volume; exercise and compression treatment targeted to that limb
are indicated.

Educational objective:
Postthrombotic syndrome occurs in >50% of patients with a history of acute deep venous thrombosis and is marked
by the development of chronic venous insufficiency distal to the site of thrombus. It usually presents with leg pain,
edema, fatigue, superficial venous dilation, and/or ulcer. Treatment includes exercise and compression.
Question #39

A 78-year-old man comes to the clinic for a routine outpatient visit. The patient reports no symptoms. Medical
history is significant for hypertension, early-stage chronic kidney disease, and calcific aortic stenosis. Eight months
ago, he underwent transcatheter aortic valve implantation (TAVI) using femoral approach without significant
complications. Recent echocardiogram reveals normal left ventricular function and no evidence of transcatheter
valve dysfunction. The patient is scheduled to undergo routine dental cleaning, which was last done a year ago.
He asks about the need to take an antibiotic before the procedure. Which of the following is the best response?

A) "No, because it has been more than 6 months after the procedure."

B) "No, because you are scheduled for a low risk procedure."

C) "No, because you have a sutureless valve."

D) "Yes, because you have a prosthetic valve."

E) "Yes, because you have calcium around your new valve."


Explanation
Correct Answer:

D) "Yes, because you have a prosthetic valve."

High-risk conditions for infective endocarditis

• Prosthetic heart valve


• Previous infective endocarditis
• Structural valve abnormality in transplanted heart
• Unrepaired cyanotic congenital heart disease
• Repaired congenital heart disease with residual defect

Patients with prosthetic heart valves (eg, following transcatheter aortic valve replacement) are at high risk for
infective endocarditis (IE). Incidence is thought to be 0.3%-3.4% per patient year. Most cases arise when a small
thrombus forms on the surface of the prosthetic valve and is subsequently seeded by bacteria during a period of
transient bacteremia. Although seeding primarily occurs during normal daily activities (eg, teeth brushing), risk is
increased during the following invasive procedures:

• Dental work that penetrates the apical tooth or gingival surface, including routine dental cleaning

• Respiratory procedures that penetrate the mucosa (eg, tonsillectomy, bronchoscopic biopsy)

• Skin/soft tissue procedures in areas of active infection

• Genitourinary (GU)/gastrointestinal (GI) procedures in areas of active GU/GI infection


Prophylactic antibiotics administered 30-60 minutes prior to these procedures reduce the quantity and duration
of transient bacteremia; therefore, antibiotics should be administered to those with a high risk of IE (eg, prosthetic
valve, prosthetic cardiac graft, certain congenital cyanotic heart diseases). Oral amoxicillin (one 2 g dose) is used
prior to dental and respiratory procedures to reduce Streptococcus viridans bacteremia; it can also be used prior to
GU/GI procedures in patients with active GU/GI infection to reduce Enterococcus bacteremia. Patients undergoing
procedures in areas of skin and soft tissue infection usually receive intravenous vancomycin to reduce
Staphylococcus bacteremia.

(Choice A) Risk of IE is greatest immediately following valve replacement because the valve is not covered with
endothelium. However, because the risk of IE persists, there is no specific time cutoff following prosthetic valve
placement to discontinue prophylaxis; therefore, antibiotic prophylaxis is required for procedures associated with
increased risk of transient bacteremia (eg, dental cleaning).

(Choices B and C) Routine dental cleaning is associated with a high risk of transient bacteremia; prophylactic
antibiotics are required for patients with high risk heart conditions (eg, prosthetic valve). All prosthetic valves (eg,
surgical, transaortic [ie, sutureless]), confer an increased risk of IE because they promote the formation of small
clots, which can subsequently be infected during transient bacteremia.

(Choice E) Calcification of the aortic valve reflects an inflammatory process similar to atherosclerosis. However, it
does not affect guidelines for antibiotic prophylaxis.

Educational objective:
Patients with prosthetic heart valves (eg, following transcatheter aortic valve replacement) are at increased risk for
infective endocarditis. They should be given prophylactic antibiotics prior to dental procedures that penetrate the
gingival surface, including routine dental cleaning. Prophylactic antibiotics are also typically required for procedures
that penetrate the respiratory mucosa, areas of infected skin, and areas of infected genitourinary/gastrointestinal
mucosa.
Question #40

A 73-year-old man is brought to the emergency department after repeat episodes of lightheadedness. Yesterday,
the patient felt lightheaded while jogging and sat down on the curb to prevent a fall. A similar episode occurred 6
weeks ago when he felt lightheaded while climbing stairs. He has had some exertional shortness of breath
associated with chest discomfort for the past few months but has had no headache, blurry vision, or chest pain.
Medical history includes hypertension. The patient takes hydrochlorothiazide and low-dose aspirin daily. He has a
20-pack-year history and occasionally drinks a glass of wine. Blood pressure is 142/90 mm Hg, pulse is 80/min,
and respirations are 16/min. ECG shows normal sinus rhythm with left ventricular hypertrophy. Which of the
following is most likely to diagnose this patient's condition?

A) 24-hour ECG monitoring

B) Carotid doppler study

C) Echocardiography

D) Tilt table testing

E) Treadmill stress testing


Explanation
Correct Answer:

C) Echocardiography

Aortic stenosis

• Calcific disease (most common, age >70)


• Congenital bicuspid valve (younger patients)
Causes
• Rheumatic heart disease (common worldwide but rare in
developed countries)

• Dyspnea on exertion, decreased exercise tolerance


Clinical • Angina pectoris (severe AS)
manifestations • Syncope (severe AS)
• Heart failure (severe AS)

• Crescendo-decrescendo systolic murmur best heard at RUSB


Physical
with radiation to carotids & axillae
examination
• Slow-rising (delayed) & weak carotid pulse

• Aortic valve replacement for severe AS causing symptoms or


Management
LV systolic dysfunction
AS = aortic stenosis; LV = left ventricular; RUSB = right upper sternal border.

This patient's clinical presentation is most suggestive of severe aortic stenosis (AS). Patients with AS are usually
asymptomatic for a prolonged period, with classic symptoms typically occurring only after progression to severe
AS (valve area usually <1 cm2). Classic symptoms of severe AS include the following:

• Exercise intolerance and exertional dyspnea: Severe valvular stenosis obstructs left ventricular outflow,
preventing an increase in cardiac output during exercise.

• Exertional angina: Left ventricular hypertrophy (as seen on this patient's ECG) compensates for the
increased afterload caused by the stenosed aortic valve. On exertion, oxygen delivery is inadequate (due to
inability to appropriately increase cardiac output) to supply the increased oxygen requirements of the
hypertrophied myocardium, resulting in angina.

• Presyncope (ie, lightheadness) or syncope: Cerebral perfusion pressure decreases during exercise
because cardiac output cannot increase sufficiently to offset the decrease in systemic vascular resistance
that occurs with exercise-induced peripheral vasodilation.

Transthoracic echocardiography should be obtained in all patients with suspected AS to confirm the diagnosis,
assess the severity of AS, and evaluate left ventricular size and function. Failure to treat severe AS can lead to
irreversible heart failure and sudden death. Definitive treatment is surgical valve replacement.

(Choice A) 24-hour ECG monitoring can detect paroxysmal arrhythmias (eg, ventricular tachycardia, atrial
fibrillation with rapid ventricular response) that may cause presyncope. However, the consistent association of this
patient's symptoms with exertion and left ventricular hypertrophy on ECG are more suggestive of AS than of
intermittent arrhythmias.

(Choice B) Carotid stenosis (which can be evaluated with carotid ultrasound) can lead to decreased cerebral
perfusion, which rarely presents with lightheadedness on exertion. However, it would not explain this patient's
exertional dyspnea and chest discomfort.
(Choice D) Upright tilt table testing may be used to diagnose orthostatic or vasovagal syncope in patients with
recurrent unexplained syncope. However, other more dangerous causes (eg, structural heart disease) of
presyncope/syncope should be ruled out first.

(Choice E) Exercise stress testing is considered when exertional symptoms (eg, angina, dyspnea) occur in the
setting of known or suspected coronary artery disease. However, severe, symptomatic AS is a contraindication for
exercise stress testing due to the risk of complications, including syncope and death; therefore, echocardiography
should be performed first when severe AS is suspected.

Educational objective:
Symptoms of severe aortic stenosis include exercise intolerance, exertional dyspnea and angina, and exertional
presyncope/syncope. Echocardiography should be obtained in all patients with suspected structural heart disease
(eg, aortic stenosis).

Reference
• Evaluation of syncope.

• Diagnostic patterns and temporal trends in the evaluation of adult patients hospitalized with syncope.
Question #41

A 52-year-old man comes to the clinic due to frequent chest discomfort over the past 6 weeks. The discomfort,
described as substernal and "dull," occurs for hours when the patient lies down; it has awakened him from sleep
most nights. Use of sublingual nitroglycerin has not relieved the chest discomfort. Review of systems is notable for
intermittent nocturnal cough. Medical history includes coronary artery disease with drug-eluting stent placement 3
years ago. Current medications include low-dose aspirin, atorvastatin, metoprolol, and losartan. Blood pressure is
128/78 mm Hg, and heart rate is 85/min. Physical examination is unremarkable. Resting ECG reveals no
significant abnormalities. Which of the following is the most appropriate next step in management of this patient?

A) Increase metoprolol dose to a resting heart rate <60/min

B) Order Helicobacter pylori stool antigen testing

C) Prescribe proton pump inhibitor therapy

D) Refer for coronary catheterization

E) Refer for pulmonary function testing


Explanation
Correct Answer:

C) Prescribe proton pump inhibitor therapy

Distinguishing acid reflux from angina

GERD Angina

• Substernal burning or dull pain; may


• Substernal pressure; may radiate to arms,
radiate to arms, neck, jaw
Symptoms neck, jaw
• Frequently accompanied by cough or
• Frequently accompanied by dyspnea
reflux

Symptom • Short, intermittent (eg, 2-5 min for chronic


• Prolonged, constant (eg, >1 hr)
duration angina; >15 min suggests ACS)

Precipitating • Recumbency • Physical exertion


factors • Trigger foods (eg, alcohol, coffee) • Stress

Alleviating • Rest
• Antacids
factors • Nitroglycerin
ACS = acute coronary syndrome; GERD = gastroesophageal reflux disease.

Substernal chest pain can occur in both angina and gastroesophageal reflux disease (GERD). However, the
likelihood of GERD versus angina can often be distinguished based on a thorough history. Although this patient
has a history of coronary artery disease, his current symptom pattern is more suggestive of GERD due to the
following features:

• Prolonged symptom duration (eg, pain lasting for hours)


• Positional nature of symptoms (eg, pain worse with recumbency)
• Accompanying symptoms of cough (caused by aspiration, cough reflex, or vagal nerve stimulation)
• Nonresponse to nitroglycerin

GERD is diagnosed clinically. Management includes lifestyle changes (eg, weight loss, head-of-bed elevation) and
acid-reducing medication. Proton pump inhibitor therapy is recommended for patients with frequent (eg, >2 days
a week) or severe symptoms and should be started next in this patient.

In contrast, angina is characterized by intermittent pain of short duration (eg, 2-5 min) that is frequently aggravated
by activity and relieved with rest and nitroglycerin. Patients with stable angina should first undergo noninvasive
testing for coronary artery disease (eg, stress testing). Coronary catheterization is reserved for acute coronary
syndrome, persistent angina despite maximal medication therapy, or extensive ischemia seen on noninvasive
testing (Choice D).

(Choice A) Increasing beta blocker therapy to achieve a resting heart rate of 55-60/min is recommended in
patients with angina from obstructive coronary artery disease, which is not evident in this patient.

(Choice B) Helicobacter pylori stool antigen testing is indicated in patients with dyspepsia, which is characterized
by epigastric (not substernal) pain/burning, postprandial fullness, and early satiety. This patient's provocation of
symptoms by recumbency and accompanying cough are more characteristic of GERD; H pylori testing is not
indicated in most patients with GERD.

(Choice E) Pulmonary function tests are used to diagnose asthma, which can present with cough and chest
tightness. However, recumbency is not a typical provoking factor.

Educational objective:
Chest pain due to gastroesophageal reflux disease is distinguished from angina by the presence of prolonged
symptom duration (hours rather than minutes), occurrence with rest or recumbency (rather than exertion),
nonresponse to nitroglycerin, and accompanied cough. The first step in managing frequent symptoms is proton
pump inhibitor therapy.

Reference
• Evaluation of chest pain in primary care patients.
Question #42

A 59-year-old man comes to the office for follow-up after a recent acute myocardial infarction. Three months ago,
the patient was hospitalized with chest pain and was found to have subtotal occlusion of the mid-left anterior
descending artery, which was treated with a drug-eluting stent. He currently has no chest pain, shortness of breath,
or palpitations. Medical history is significant for hypertension. His father had a myocardial infarction at age 55. The
patient currently takes aspirin, ticagrelor, metoprolol, and lisinopril; he was prescribed high intensity rosuvastatin
therapy but stopped taking it a month ago due to muscle aches. Blood pressure is 122/73 mm Hg, and pulse is 65/
min and regular. BMI is 28 kg/m2. Physical examination is unremarkable. Lipid panel results are as follows:

LDL cholesterol 105 mg/dL


HDL cholesterol 48 mg/dL
Triglycerides 112 mg/dL

Which of the following is the best recommendation for lipid management in this patient?

A) Advise fish oil supplement

B) Advise lifestyle modification only

C) Offer ezetimibe therapy

D) Offer fenofibrate therapy

E) Offer moderate-intensity statin therapy


Explanation
Correct Answer:

E) Offer moderate-intensity statin therapy

This patient has established atherosclerotic cardiovascular disease (ASCVD) and was appropriately prescribed
high-intensity statin therapy (ie, rosuvastatin). Statins have been shown to reduce the risk of myocardial
infarction and ischemic stroke in patients with ASCVD. However, these drugs are associated with a small risk of
adverse effects, including muscle injury (eg, myalgia, myonecrosis).

The muscle injury associated with statins typically resolves with cessation of the drug. Those who develop
rhabdomyolysis, a rare side effect of severe statin-induced myopathy, should discontinue statin use entirely.
However, most muscle toxicity is mild (eg, myalgias), and the majority of patients can tolerate a different statin.

Options include an alternate high-intensity statin (eg, atorvastatin 80 mg) or a moderate-intensity statin, which
should be titrated to the maximal tolerated dose. Pravastatin, pitavastatin, and fluvastatin are moderate-intensity
statins that have a lower risk of myopathy and are frequently used as alternate therapy.

Subclinical hypothyroidism is associated with an increased risk of statin myopathy, and many experts advise
measuring a serum TSH level before starting statins or when side effects occur in patients receiving treatment.

(Choice A) Although high-fish diets are associated with a lower risk of ASCVD in some populations, studies on the
risk of atherosclerotic events with fish oil supplements are inconsistent.

(Choice B) Although lifestyle modifications (eg, exercise, smoking cessation) should be recommended to reduce
the risk of myocardial infarction and stroke, this patient should also receive a lipid-lowering medication, preferably a
statin, to reduce his risk.

(Choice C) Ezetimibe can be considered as third-line therapy for patients who do not tolerate high- or moderate-
intensity statins, but it is not as effective as statins in preventing cardiovascular events. PCSK9 inhibitors (eg,
alirocumab) are another alternate therapy for these patients.
(Choice D) Fibrates (eg, fenofibrate) do not lower the risk of atherosclerotic events in patients with normal
triglyceride levels at baseline. They have been associated with improvements in cardiovascular risk in patients with
very high triglycerides (ie >400 mg/dL) and low HDL (≤40 mg/dL) but are typically given with a statin.

Educational objective:
High-intensity statin therapy can reduce the risk of myocardial infarction and ischemic stroke in patients with
atherosclerotic cardiovascular disease. For patients who are intolerant of one high-intensity statin, switching to a
different high-intensity or a moderate-intensity statin should be considered.

Reference
• Statin non-adherence: clinical consequences and proposed solutions.
Question #43

A 35-year-old woman is seen in the outpatient clinic due to palpitations. Over the past 6 months, she has noticed
decreased exercise tolerance and episodes of her heart "racing in my chest." Temperature is 37.2 C (98.9 F) and
blood pressure is 125/75 mm Hg. On examination, there is an early diastolic sound followed by a middiastolic
murmur. The rest of the examination is unremarkable. ECG is shown in the exhibit. Which of the following is the
most likely finding on echocardiography?

A) Atrial septal defect

B) Left atrial dilation

C) Left atrial mass attached to interatrial septum

D) Left ventricular dilation

E) Left ventricular hypertrophy


Explanation
Correct Answer:

B) Left atrial dilation

Mitral stenosis in adults

• Rheumatic heart disease (vast majority of cases)


Etiology
• Age-related calcification, radiation induced

• Exertional dyspnea, orthopnea, paroxysmal nocturnal dyspnea, hemoptysis


Clinical presentation • Pulmonary edema ± right-sided heart failure (eg, lower extremity edema)
• Atrial fibrillation, ↑ risk for systemic embolization

• Opening snap with middiastolic rumble at the apex


Diagnosis
• Echocardiography: ↑ transmitral flow velocity

Treatment • Percutaneous valvotomy or surgical repair/replacement

This patient's middiastolic murmur preceded by an early diastolic sound (ie, opening snap) is consistent with mitral
stenosis (MS), which usually occurs in the setting of rheumatic heart disease. The irregularly irregular rhythm
and absent P waves on ECG demonstrate atrial fibrillation, which is a common complication of MS that occurs
due to left atrial dilation. Among patients with untreated MS, the prevalence of atrial fibrillation is up to 50%.
MS impairs diastolic filling of the left ventricle and causes an increase in left atrial pressure, which leads to
stretching of the left atrial walls. This atrial stretch can disrupt electrical conduction and trigger arrhythmogenic
electrical foci; atrial fibrillation most commonly originates from arrhythmogenic foci in the pulmonary veins.

Atrial fibrillation can induce acute decompensated heart failure in the setting of MS because many patients with
MS require the additional left ventricular filling supplied by atrial contraction (ie, atrial kick at end-diastole) to remain
compensated. Atrial fibrillation in the setting of MS also carries an even higher thrombogenic risk than
nonvalvular atrial fibrillation, and routine anticoagulation with warfarin is indicated regardless of CHA2DS2-Vasc
score.

(Choice A) An atrial septal defect can cause increased blood flow in the right side of the heart. The defect itself
does not generate a murmur, but right-sided flow murmurs (ie, low-grade systolic murmur of the pulmonic valve,
low-grade diastolic murmur of the tricuspid valve) may be heard. An opening snap would not be present, and atrial
fibrillation would be only a rare complication.

(Choice C) A left atrial myxoma can obstruct the mitral valve and cause a presentation similar to MS; however,
because the mitral valve is healthy, an opening snap would not be present. Thrombus may be seen in the left
atrium in patients with MS, but it typically develops in the atrial appendage and would not be attached to the
interatrial septum.

(Choices D and E) Because in MS the obstruction to blood flow is upstream of the left ventricle, left ventricular
structure remains normal (ie, no cavity dilation or wall hypertrophy).

Educational objective:
Mitral stenosis (MS) is recognized by a middiastolic murmur preceded by an opening snap. Atrial fibrillation is
common in MS and results from an increase in left atrial pressure with consequent left atrial stretching. In the
setting of MS, atrial fibrillation can trigger decompensated heart failure and also carries a higher thrombogenic risk
than nonvalvular atrial fibrillation.

Reference
• Management of atrial fibrillation in patients with rheumatic mitral stenosis.
Question #44

A 54-year-old man who is homeless comes to the emergency department 24 hours after the onset of substernal
chest pain and is diagnosed with an anterior wall myocardial infarction. The patient has no history of hypertension
or diabetes mellitus and has had no previous chest pain, dyspnea, palpitations, syncope, or leg swelling. He has a
40-pack-year smoking history. Echocardiography shows normal left ventricular size and left ventricular anterior wall
hypokinesis, as well as an ejection fraction of 40%. The patient declines pharmacologic therapy on discharge and
is scheduled for an outpatient clinic visit but becomes lost to follow up. Two years later, the patient is found dead by
one of his friends. Autopsy reveals a dilated left ventricle with a globular shape and thinned walls with a scar on the
anterior wall. Which of the following would most likely have helped prevent this patient's pathologic findings?

A) Amlodipine

B) Apixaban

C) Aspirin

D) Enalapril

E) Isosorbide dinitrate

F) Prasugrel
Explanation
Correct Answer:

D) Enalapril
This patient's myocardial infarction (MI) 2 years ago resulted in left ventricular (LV) systolic dysfunction, and autopsy
shows a dilated left ventricle with thinned walls and evidence of scarring. These findings are consistent with
eccentric hypertrophy due to ischemic heart disease. Both infarction and LV systolic dysfunction induce
deleterious cardiac remodeling with pathologic cardiomyocyte hypertrophy and collagen deposition; these
changes commonly progress to cause severe LV systolic dysfunction and create an increased risk for fatal cardiac
arrhythmia (eg, ventricular tachycardia or fibrillation), which was the likely cause of death in this patient.

Cardiac remodeling is mostly driven by neurohormonal pathways, including the renin-angiotensin-aldosterone


pathway; inhibition of this pathway with an ACE inhibitor (eg, enalapril, lisinopril) has been shown to reduce
cardiac remodeling and improve survival in patients who have had MI, especially those with an LV ejection
fraction of ≤40% and/or anterior wall MI.

Certain beta blockers (eg, metoprolol, carvedilol) have demonstrated similar benefit in all patients following MI, likely
through inhibition of sympathetically driven cardiac remodeling pathways. Aldosterone antagonists (eg,
spironolactone) have shown survival benefit in patients with MI and LV ejection fraction of ≤40%.

(Choice A) Dihydropyridine calcium channel blockers (eg, amlodipine) are appropriate for treating hypertension
and may reduce a component of cardiac remodeling that is stimulated by increased LV pressure load. However,
these drugs have not shown the degree of survival benefit following MI that ACE inhibitors and other
neurohormonal-inhibiting drugs have shown.

(Choice B) Anticoagulation agents (eg, apixaban) are not routinely used following MI. These are appropriate to
prevent systemic embolization (eg, stroke) in patients with LV aneurysm and mural thrombus, of which this patient
has no evidence on autopsy.

(Choice C) Low-dose aspirin is beneficial for secondary prevention of further cardiovascular events in patients who
have had MI, but it is not known to benefit cardiac remodeling.

(Choice E) Isosorbide dinitrate is useful in treating symptomatic angina, but it has not shown survival benefit
following MI.

(Choice F) P2Y12 inhibitors (eg, prasugrel, clopidogrel), in combination with aspirin, have been shown to reduce
cardiovascular events within the first year after MI; however, these drugs do not reduce cardiac remodeling.
Educational objective:
Following myocardial infarction (MI), deleterious cardiac remodeling is mostly driven by neurohormonal
mechanisms. ACE inhibitors (eg, enalapril, lisinopril), beta blockers (eg, metoprolol), and aldosterone antagonists
(eg, spironolactone) improve survival following MI, likely due to inhibition of neurohormonal-mediated cardiac
remodeling.

Reference
• Effect of ramipril on morbidity and mode of death among survivors of acute myocardial infarction with clinical
evidence of heart failure. A report from the AIRE Study Investigators.
Question #45

An imaging study detects an incidental 3.5-cm infrarenal abdominal aortic aneurysm in a 70-year-old man. He has
no associated symptoms. Medical history is significant for hypertension, type 2 diabetes, and
hypercholesterolemia. The patient is maintained on anticoagulation due to paroxysmal atrial fibrillation. He has
smoked 1-2 packs of cigarettes a day for the past 40 years, and he drinks 1 or 2 glasses of wine daily. The patient
is physically active, bikes regularly, and enjoys hiking. On physical examination, blood pressure is 150/78 mm Hg
and pulse is 80/min. Heart sounds are normal, and the lungs are clear. Laboratory results are as follows:

Serum creatinine 1.7 mg/dL


Low-density lipoprotein (LDL) 150 mg/dL
Hemoglobin A1c 7.8%

Which of the following is most strongly associated with aneurysm progression in this patient?

A) Active smoking

B) Anticoagulation

C) Exercise activities

D) LDL >130 mg/dL

E) Systolic hypertension

F) Uncontrolled diabetes mellitus


Explanation
Correct Answer:

A) Active smoking

Abdominal aortic aneurysm

• Advanced age (eg, >60)


Risk factors • Smoking, male sex, hypertension
• History of atherosclerosis or CTD

• Mostly asymptomatic
• Rapid expansion
◦ Dull abdominal/back pain
Clinical presentation ◦ Distal embolization
• Rupture
◦ Sudden, severe abdominal/back pain ± shock
◦ Umbilical/flank hematoma

• Smoking cessation
• Elective repair for size >5.5 cm (asymptomatic)
Management
• Urgent repair for symptomatic & HD stable patients
• Emergency repair for symptomatic & HD unstable patients
CTD = connective tissue disease; HD = hemodynamically.

Abdominal aortic aneurysm (AAA) is generally defined as an enlargement of the abdominal aorta to a diameter ≥3
cm. It is often incidentally detected on imaging, as many patients have no symptoms unless and until rupture
occurs. Rupture is more likely with large aneurysmal size (>5.5 cm) or rapid rate of expansion (eg, >1 cm/year),
and preemptive surgical repair is the definitive treatment. However, surgery carries a relatively high risk of
complication, with the risk of repair typically greater than the risk of rupture until the aneurysm reaches a large size.
Therefore, management of small to moderately sized AAA involves surveillance with serial abdominal ultrasound
and lifestyle modification to limit AAA progression.

Risk factors for AAA include advanced age (eg, >60), smoking, male sex, family history, hypertension, and a
history of atherosclerosis or underlying connective tissue disease. Of these, active smoking is the strongest
modifiable influence for AAA development and progression. Smoking is believed to disrupt the arterial wall elastin
matrix via stimulation of enhanced protease activity, chronic inflammation, and smooth muscle dysfunction, and
smoking cessation is considered the best intervention to minimize AAA progression.

Because of the relationship between smoking and AAA, a one-time abdominal ultrasound is recommended to
screen for AAA in men ages 65 to 75 with any smoking history.

(Choice B) Anticoagulation has not been shown to either slow or accelerate AAA progression. Patients with AAA
who have an appropriate indication for anticoagulation (eg, atrial fibrillation) should remain on it.

(Choices C and D) AAA is considered a coronary artery disease equivalent; therefore, moderate exercise and
enhanced lipid control are associated with overall improved cardiovascular outcomes in such patients. However,
exercise and lipid control do not limit AAA progression to the same extent as smoking cessation.

(Choice E) AAA is more common in individuals with hypertension, indicating that hypertension may play a role in
initial AAA development. However, hypertension, especially isolated systolic hypertension, has not been reliably
correlated with AAA progression.

(Choice F) Diabetes is a strong risk factor for atherosclerosis and cardiovascular disease. However, the risk of
AAA development and progression is lower in patients with diabetes compared to those without, for reasons that
are unclear.

Educational objective:
The management of small to moderately sized (ie, 3 cm to 5.5 cm) abdominal aortic aneurysms (AAAs) involves
lifestyle modification, with smoking cessation as the best intervention to minimize AAA progression.

Reference
• Abdominal aortic aneurysm expansion: risk factors and time intervals for surveillance.

• The association between cigarette smoking and abdominal aortic aneurysms.

• Association of hypertension with abdominal aortic aneurysm expansion.

https://afkebooks.com
Question #46

A 33-year-old man comes to the physician reporting mild exertional shortness of breath and a "pounding" heart over
the last 5 months. He is uncomfortably aware of his heartbeat while lying on his left side. Vital signs include blood
pressure of 150/45 mm Hg and pulse of 73/min. Which of the following is most likely responsible for his symptoms?

A) Aortic regurgitation

B) Aortic stenosis

C) Mitral stenosis

D) Pulmonic regurgitation

E) Tricuspid stenosis

https://afkebooks.com
Explanation
Correct Answer:

A) Aortic regurgitation

Chronic aortic regurgitation

• Congenital bicuspid aortic valve


Common etiologies • Postinflammatory (eg, rheumatic heart disease, endocarditis)
• Aortic root dilation (eg, Marfan syndrome, syphilis)

• Backflow from aorta into LV → ↑ LV end-diastolic volume


Pathophysiology • LV initially compensates with eccentric hypertrophy → ↑ SV & CO
• Eventual LV dysfunction → ↓ SV & CO → heart failure

• Decrescendo diastolic murmur


Clinical
• Widened pulse pressure (↑SBP & ↓DBP)
• Rapid rise-rapid fall (“water-hammer”) pulsation
findings
• Abrupt carotid distension & collapse, “pistol-shot” femoral pulses

CO = cardiac output; DBP = diastolic blood pressure; LV = left ventricle; SBP = systolic blood pressure; SV =
stroke volume.

This patient's presentation with exertional dyspnea, pounding heart sensation, and widened pulse pressure is most

https://afkebooks.com
likely due to chronic aortic regurgitation (AR). In chronic AR, a portion of left ventricular (LV) output leaks back
into the left ventricle, causing an increase in LV end-diastolic volume (LVEDV), myocardial hypertrophy, and
chamber enlargement. The increase in LV size brings the ventricular apex close to the chest wall, causing a
pounding sensation and an uncomfortable awareness of the heartbeat, especially in the left lateral decubitus
position. The most common cause of AR in developing countries is rheumatic heart disease; in developed
countries it is often due to aortic root dilation or congenital bicuspid valve.

The wide pulse pressure (systolic minus diastolic blood pressure) in patients with AR causes a characteristic
"water hammer" or Corrigan pulse: rapid, abrupt upstroke followed by rapid collapse of the peripheral pulse. AR
produces an early diastolic murmur that is high-pitched and sustained or decrescendo in intensity.

(Choice B) Patients with aortic stenosis have a characteristic pulse with delayed upstroke (pulsus tardus), delayed
peak, and small pulse amplitude (pulsus parvus). Pulse pressure remains normal in patients with aortic stenosis.

(Choice C) Mitral stenosis occurs most commonly due to rheumatic heart disease. Patients can have pounding
sensations or palpitations due to development of rapid atrial fibrillation. However, peripheral arterial pulses are
reduced in volume and wide pulse pressure is typically not present.

(Choice D) Pulmonic regurgitation occurs most commonly as a result of pulmonary hypertension or after Tetralogy
of Fallot repair in adults. It does not cause pounding awareness of heartbeat or widening of the pulse pressure.

(Choice E) Tricuspid stenosis typically presents with right-sided heart failure (eg, elevated jugular venous pressure,
hepatomegaly, ascites, peripheral edema) along with clear lungs. The peripheral pulse pressure would not be
increased.

Educational objective:
Aortic regurgitation leads to increased left ventricular (LV) end-diastolic volume due to leakage of blood from the
aorta back into the left ventricle. Features of aortic regurgitation include a wide pulse pressure, "water hammer"
pulse, and LV enlargement. The left lateral decubitus position brings the enlarged left ventricle closer to the chest
wall and causes a pounding sensation and increased awareness of the heartbeat.

Reference

https://afkebooks.com
• The rational clinical examination. Does this patient have aortic regurgitation?

• Valvular heart disease: aortic regurgitation.

https://afkebooks.com
Question #47

A 73-year-old woman comes to the office for preoperative evaluation prior to arthroscopic rotator cuff repair under
general anesthesia. The patient does not routinely exercise but keeps a large garden and does her own laundry in
the basement. She has no chest pain, palpitations, dyspnea, light-headedness, or orthopnea. Medical history is
notable for an ST-segment elevation myocardial infarction requiring stent placement to her right coronary artery 3
years ago. Other medical conditions include type 2 diabetes mellitus, hypertension, and dyslipidemia. She takes
oral medications and does not use insulin. Blood pressure is 140/86 and pulse is 76/min and regular. Jugular veins
are not distended. Cardiopulmonary examination is normal. There is no pedal edema. Serum creatinine is 1.1 mg/
dL. ECG shows normal sinus rhythm with evidence of previous inferior myocardial infarction. Which of the
following diagnostic tests is the most appropriate preoperative management for this patient?

A) Coronary angiography

B) Exercise treadmill stress testing

C) No further testing

D) Pharmacologic stress testing

E) Resting echocardiography

https://afkebooks.com
Explanation
Correct Answer:

C) No further testing

Revised Cardiac Risk Index (RCRI)


(cardiovascular risk of noncardiac surgery)

• High-risk surgery (eg, vascular, intrathoracic)


• Ischemic heart disease
• History of congestive heart failure
6 risk predictors
• History of cerebrovascular disease (stroke or TIA)
• Diabetes mellitus treated with insulin
• Preoperative creatinine >2 mg/dL

Risk of cardiac death,


• 0-1 factor: low risk*
nonfatal cardiac arrest,
• ≥2 factors: elevated risk
or nonfatal MI

*RCRI score of 0-1 originally reported as ≤1% and still accepted as low risk. Slightly higher
event rates of later studies probably due to using troponins (↑ sensitivity) and including
additional outcomes (eg, all-cause mortality).

MI = myocardial infarction; TIA = transient ischemic attack.

https://afkebooks.com
Patients being considered for noncardiac surgery should undergo preoperative cardiovascular risk assessment
with a validated prediction tool that incorporates both procedure- and patient-specific variables. Although newer
models (eg, American College of Surgeons surgical risk calculator) are available, the Revised Cardiac Risk Index
(RCRI) remains relevant due to its prolonged validation (ie, decades) and simplicity (ie, 1 point each for 6 risk
predictors).

This patient's only patient-specific risk factor is ischemic heart disease, and her procedure-specific risk is
intermediate. Therefore, her RCRI score is 1, indicating a ~1% estimated risk of perioperative major adverse
cardiac event (eg, myocardial infarction, cardiac arrest, cardiac death). An estimated risk ≤1% (eg, RCRI 0-1) is
the recommended threshold to undergo surgery without further testing.

In contrast, when estimated risk is >1% (eg, RCRI ≥2), further preoperative evaluation requires assessment of
cardiac functional capacity. Functional capacity is classically determined based on the ability to perform activities
requiring ≥4 metabolic equivalents (METs):

• Patients with increased risk (>1%) who can perform ≥4 METs (eg, climb a flight of stairs, do yardwork) can
proceed to surgery without further evaluation. (This patient who is able to climb up a flight of stairs from her
basement likely has ≥4 METs functional capacity and would be able to proceed to surgery even if she had
>1% risk of perioperative major adverse cardiac events.)

• Patients with increased risk (>1%) and unknown or poor (<4 METs) functional status require further
consideration. Cardiac stress testing is recommended if cardiac intervention (if necessary) will impact
surgical/perioperative management (eg, postponing elective surgery for revascularization followed by 6
months of antiplatelet therapy). Because these patients are typically less likely to achieve an adequate
exercise workload, pharmacologic-based stress testing is recommended over exercise-based (eg, treadmill)
stress testing (Choices B and D).

Generally, extensive preoperative cardiac testing is necessary only in patients with significant cardiac conditions
undergoing high-risk procedures.

(Choice A) Coronary angiography is indicated for patients with acute myocardial infarction, unstable angina, or
abnormal stress testing. This patient has adequate functional capacity and no anginal symptoms after
revascularization 3 years ago, making coronary angiography unnecessary.

https://afkebooks.com
(Choice E) Preoperative resting echocardiography is generally necessary only if indicated in the absence of
planned surgery (eg, new murmur, clinical status change in patient with heart failure). This patient's preoperative
assessment shows no evidence of heart failure or valvular disease (eg, no orthopnea, edema, or murmur).

Educational objective:
Patients undergoing noncardiac surgery should be assessed for perioperative cardiac risk based on procedure- and
patient-specific risk factors. Low-risk patients and those at increased risk (>1%) with adequate functional capacity
(≥4 metabolic equivalents) can typically undergo surgery without further testing.

https://afkebooks.com
Question #48

A 56-year-old man comes to the office due to 3 months of dyspnea that was initially exertional but has progressively
worsened to the point that the patient is now breathless even at rest. He has not had any chest pain, palpitations,
syncope, or ankle swelling. The patient has no significant past medical history, and he takes no medications. He
has been smoking a pack of cigarettes daily for the last 30 years and has been drinking alcohol heavily for the past
5 years. His mother died of breast cancer at age 57. The patient's temperature is 37.1 C (98.8 F), blood pressure
is 113/76 mm Hg, and pulse is 86/min. A third heart sound is heard on cardiac auscultation; there are no murmurs.
Chest auscultation indicates bibasilar crackles. His abdomen is soft and with no evidence of ascites. Laboratory
results are as follows:

Complete blood count


Hematocrit 32%
Mean corpuscular volume 101 µm3
Platelets 60,000/mm3
Leukocytes 6000/mm3

Liver function studies


Alkaline phosphatase 44 U/L
Aspartate aminotransferase 180 U/L
Alanine aminotransferase 66 U/L

Chest x-ray reveals cardiac silhouette enlargement and signs of pulmonary venous congestion. Echocardiography
shows a dilated left ventricle with an estimated left ventricular ejection fraction of 25%. Coronary angiography
reveals no evidence of obstructive coronary artery disease. Which of the following measures is most likely to
reverse this patient's heart function?

https://afkebooks.com
A) Abstinence from alcohol

B) Cessation of cigarette smoking

C) Initiation of ACE inhibitor therapy

D) Initiation of digoxin therapy

E) Niacin supplementation

F) Prednisolone therapy

G) Reduction of salt intake

https://afkebooks.com
Explanation
Correct Answer:

A) Abstinence from alcohol

This clinical presentation - dyspnea, third heart sound, bibasilar crackles, and low ejection fraction on
echocardiogram - is consistent with decompensated congestive heart failure due to left ventricular (LV) systolic
dysfunction. In this patient with no evidence of coronary artery disease on angiography, the most likely etiology is
dilated cardiomyopathy due to heavy alcohol consumption (macrocytic anemia [mean corpuscular volume >100
µm3], thrombocytopenia, >2:1 ratio of aspartate aminotransferase to alanine aminotransferase).

Alcoholic cardiomyopathy is a diagnosis of exclusion in patients with dilated cardiomyopathy and history of
alcohol abuse in whom no other potential causes of cardiomyopathy (eg, coronary artery disease, valvular heart
disease) are suspected or identified. The degree of LV dysfunction in alcoholic cardiomyopathy is directly related to
the daily amount and overall duration of alcohol intake. The primary therapy for such patients is complete
abstinence from alcohol use; this intervention is associated with improvement or normalization of LV function over
time.

(Choice B) Cigarette smoking is a strong risk factor for coronary heart disease, and smoking cessation should be
encouraged in all patients, regardless of the presence or absence of coronary artery disease. However, tobacco
cessation does not reverse the progression of alcoholic cardiomyopathy.

(Choices C, D, and G) Patients with alcoholic cardiomyopathy should be managed in the same way as patients
with other forms of dilated cardiomyopathy: salt and water restriction, diuretics as needed, ACE inhibitors or
angiotensin II receptor blockers, beta blockers, mineralocorticoid receptor antagonists, and (if indicated) digoxin.
However, strict abstinence from alcohol use remains the mainstay of therapy to reverse or halt further progression
of cardiomyopathy.

(Choice E) Niacin deficiency can occur with alcoholism, but supplementation has no role in management of
patients with alcoholic cardiomyopathy.

https://afkebooks.com
(Choice F) Glucocorticoid therapy (prednisolone) is occasionally used for patients with alcoholic hepatitis who have
a high discriminant function, but it has no role in management of alcoholic cardiomyopathy.

Educational objective:
Complete cessation of alcohol consumption is the mainstay of therapy in patients with alcoholic cardiomyopathy
and is associated with improvement or normalization of left ventricular function over time.

Reference
• Alcoholic cardiomyopathy: a review.

• Natural history and prognostic factors in alcoholic cardiomyopathy.

https://afkebooks.com
Question #49

A 60-year-old man comes to the office for evaluation of a 6-month history of intermittent chest pain. He describes
substernal tightness and pain that occur when he walks quickly or climbs stairs. The symptoms last about 10
minutes and slowly fade away with rest. These episodes do not happen at rest. The patient has a known history of
coronary artery disease with coronary artery bypass grafting surgery 7 years ago. Other medical problems include
hypertension and hyperlipidemia. Blood pressure is 140/78 mm Hg and pulse is 78/min and regular. There are no
murmurs on cardiac auscultation. Lungs are clear bilaterally. Treadmill stress test is performed. Seven minutes
into the test, the patient develops chest pain and the treadmill is stopped. Sublingual nitroglycerin is administered,
which almost immediately relieves the patient's pain. What is the predominant mechanism responsible for the rapid
pain relief in this patient?

A) Coronary vasodilation

B) Decreased left ventricular contractility

C) Decreased left ventricular wall stress

D) Dilation of small arteries

E) Negative chronotropic effect

https://afkebooks.com
Explanation
Correct Answer:

C) Decreased left ventricular wall stress

This patient with coronary artery disease has a classic presentation of chronic stable angina (exertional chest pain
relieved by rest), with rapid relief of his symptoms following the administration of sublingual nitroglycerin. Nitrates
exert their effect by direct vascular smooth muscle relaxation causing systemic venodilation and an increase in
peripheral venous capacitance. Their primary anti-ischemic effect is mediated by systemic vasodilation and
decrease in cardiac preload resulting in a decrease in left ventricular end-diastolic and end-systolic volume. This,
in turn, leads to a reduction in left ventricular systolic wall stress - which reflects afterload and is proportional to
(pressure * radius / thickness) - and a decrease in myocardial oxygen demand, resulting in relief of anginal
symptoms.

Although nitrates act as coronary vasodilators (via direct relaxation of coronary vascular smooth muscle cells), the
beneficial effect of this mechanism is uncertain (Choice A). The coronary arterioles in the area of flow-limiting
coronary stenosis are already dilated by innate mechanisms, allowing maintenance of resting blood flow across the
stenotic lesion. Nitrate-induced coronary vasodilation paradoxically decreases coronary perfusion pressure across
the stenotic lesion by diverting blood flow to arterioles supplying the normal myocardium.

(Choices B and E) Beta blockers and calcium channel blockers exert an antianginal effect partially via a decrease
in the heart rate (negative chronotropic effect) or contractility (negative inotropic effect). Nitrates do not have a
direct effect on cardiac chronotropy; they may cause decreased inotropy due to decreased preload, but their
antianginal effect is thought to be due primarily to a reduction in wall stress.

(Choice D) At low doses (such as those used in sublingual nitroglycerin preparations), nitrates have only a modest
effect on arterial and arteriolar dilation and cause minimal or no change in systemic vascular resistance. Higher
doses can lead to a drop in systemic blood pressure.

Educational objective:

https://afkebooks.com
Sublingual nitroglycerin is used as a first-line agent for rapid relief of symptoms in patients with angina pectoris.
The antiischemic effect of nitrates is mediated by systemic vasodilation with a decrease in left ventricular end-
diastolic volume and wall stress resulting in decreased myocardial oxygen demand.

Reference
• Mechanisms of action of nitrates.

https://afkebooks.com
Question #50

A 27-year-old man comes to the office due to syncope. Two days ago, the patient was sitting at his computer at
work and then suddenly awakened lying on the ground. His coworkers saw him collapse to the floor, jerk a few
times, and become still. The patient was unconscious for 1-2 minutes. He was not lethargic or confused after the
episode and did not experience chest pain or shortness of breath. The patient has had two similar episodes in the
last year but did not seek medical attention. He has become concerned following this third episode. He has no
medical history and takes no medications. Temperature is 36.7 C (98.1 F), blood pressure is 120/60 mm Hg, pulse
is 72/min, and respirations are 16/min. Pulse oximetry is 100% on room air. The patient is awake, alert, and
oriented, with normal speech. He has a small occipital hematoma. Oropharyngeal examination is normal. The
pupils are equal and reactive to light. Heart and lung examinations are normal. Strength is 5/5 in the upper and
lower extremities, and gait is normal. ECG is normal. What is the best next step in management of this patient?

A) Ambulatory ECG monitoring

B) CT angiography of the chest

C) Electroencephalography

D) No additional workup

E) Noncontrast CT scan of the head

https://afkebooks.com
Explanation
Correct Answer:

A) Ambulatory ECG monitoring

Clues to etiology of syncope

• Trigger (eg, emotional stress, prolonged standing)


Vasovagal
• Prodrome (eg, nausea, sweating, warmth)

Reflex syncope Situational • Occurs with micturition, defecation, swallowing, or coughing

Carotid • Tactile stimulation of carotid sinus while standing


hypersensitivity • Advanced age, carotid atherosclerosis

• Vasodilators (eg, alpha-1 blockers, antihypertensives)


Medications
• Inotropic/chronotropic blockade (eg, beta blockers)

Orthostatic
syncope Hypovolemia • History consistent with volume loss

Autonomic • Advanced age

https://afkebooks.com
dysfunction • Predisposing disease (eg, DM, Parkinson)

• Syncope with exertion


LV outflow obstruction
• Systolic ejection murmur

• No warning symptoms
Ventricular
• Cardiomyopathy or ischemic HD (monomorphic)
Cardiac syncope tachycardia
• QT-interval prolongation (polymorphic)

• Preceding fatigue or light-headedness


Conduction
• ECG abnormalities (eg, sinus pauses, dropped QRS
impairment*
complexes)

*Sick sinus syndrome or advanced atrioventricular block.

DM = diabetes mellitus; HD = heart disease; LV = left ventricular.

This patient's sudden, brief (eg, 1-2 min) episode of loss of consciousness (LOC) and postural tone, followed by
rapid recovery, is consistent with syncope. Syncope occurs due to global hypoperfusion of the brain and can
have many causes. In most cases, syncope is benign and self-limiting; however, it can be the initial manifestation
of a life-threatening disease process (eg, cardiac syncope).

A detailed initial evaluation can provide several clues to the underlying etiology. This patient's syncopal episode
had no obvious trigger or prodrome to implicate reflex syncope; it also did not occur with position change (ie,
standing from a seated position) to suggest orthostatic syncope. Instead, the syncopal episode occurred with the
patient at rest and without warning symptoms. This presentation raises concern for a cardiac etiology,

https://afkebooks.com
specifically an arrhythmic etiology, and should prompt further cardiac evaluation (Choice D).

Continuous ECG monitoring can evaluate for transient arrhythmia (eg, ventricular tachycardia) that may not be
detected on initial ECG. In relatively young patients (eg, age <40) without structural heart disease, ambulatory ECG
monitoring (eg, Holter monitor, insertable cardiac monitor) is usually appropriate. In contrast, patients likely need
hospital admission for closer monitoring and expedited cardiac evaluation when they are older, have underlying
structural heart disease (eg, prior myocardial infarction), or have an abnormal ECG.

(Choice B) CT angiography of the chest can evaluate for pulmonary embolism (PE), which sometimes presents
with syncope. However, other symptoms (eg, chest pain, dyspnea) or signs (eg, hypoxemia, tachycardia) are
usually present, and PE is unlikely to account for recurrent episodes of syncope.

(Choice C) Seizure activity (diagnosed on electroencephalography) can cause LOC with jerking movements;
however, other suggestive features (eg, prodromal aura, postictal period) are typically present. Muscle jerks can
occur in all types of syncope because sudden cerebral hypoperfusion often causes brief myoclonus.

(Choice E) Noncontrast CT scan of the head helps rule out trauma-induced intracranial injury (eg, hemorrhage). It
is not indicated in this patient who experienced a relatively low-impact mechanism of injury and has no abnormal
neurologic symptoms or findings 2 days after the incident.

Educational objective:
Patients with syncope not clearly attributable to a benign etiology (eg, reflex or orthostatic syncope) should undergo
further cardiac workup (eg, ambulatory ECG) to determine whether a cardiac etiology is present. Syncope that
occurs while supine or sitting, at rest, and without warning symptoms suggests an arrhythmic etiology.

Reference
• Guidelines for the diagnosis and management of syncope (version 2009).

• Syncope: a review of emergency department management and disposition.

https://afkebooks.com
Question #51

A 57-year-old woman comes to the office for preoperative evaluation prior to arthroscopic meniscectomy under
general anesthesia. The patient can perform activities of daily living but can no longer exercise due to recurrent
knee "locking." She has no chest pain, palpitations, dyspnea, or light-headedness. Medical history includes
hypertension and dyslipidemia. Her 61-year-old sister recently underwent coronary artery bypass grafting. The
patient has a 30-pack year history but quit 5 years ago. BMI is 29.2 kg/m2. Blood pressure is 138/82, pulse is 68/
min, and respirations are 16/min. Jugular veins are not distended. Cardiopulmonary examination is normal. Serum
creatinine is 1.0 mg/dL. ECG shows normal sinus rhythm with no ST-segment or T-wave changes. Which of the
following diagnostic tests is the most appropriate preoperative management for this patient?

A) Coronary angiography

B) Exercise treadmill stress testing

C) No further testing

D) Pharmacologic stress testing

E) Resting echocardiography

https://afkebooks.com
Explanation
Correct Answer:

C) No further testing

Revised Cardiac Risk Index (RCRI)


(cardiovascular risk of noncardiac surgery)

• High-risk surgery (eg, vascular, intrathoracic)


• Ischemic heart disease
• History of congestive heart failure
6 risk predictors
• History of cerebrovascular disease (stroke or TIA)
• Diabetes mellitus treated with insulin
• Preoperative creatinine >2 mg/dL

Risk of cardiac death,


• 0-1 factor: low risk*
nonfatal cardiac arrest,
• ≥2 factors: elevated risk
or nonfatal MI

*RCRI score of 0-1 originally reported as ≤1% and still accepted as low risk. Slightly higher
event rates of later studies probably due to using troponins (↑ sensitivity) and including
additional outcomes (eg, all-cause mortality).

MI = myocardial infarction; TIA = transient ischemic attack.

https://afkebooks.com
All patients being considered for noncardiac surgery should undergo preoperative cardiovascular risk
assessment with a validated prediction tool that incorporates both procedure- and patient-specific variables.
Although newer models (eg, American College of Surgeons surgical risk calculator) are available, the Revised
Cardiac Risk Index (RCRI) remains widely accepted due to its long validation and simplicity (1 point each for 6 risk
predictors).

This patient undergoing an intermediate-risk procedure with no patient-specific risks has an RCRI score of 0. An
RCRI of 0-1 indicates a low risk for perioperative major cardiac complication (eg, myocardial infarction, cardiac
arrest, cardiac death), so she can undergo surgery without further testing. Although useful for guiding primary
prevention, the presence of traditional coronary risk factors (eg, hypertension, dyslipidemia, tobacco use, family
history) are unhelpful in predicting perioperative risk.

In contrast, when estimated risk is >1% (eg, RCRI ≥2), further preoperative evaluation requires assessment of
cardiac functional capacity. Functional capacity is classically determined based on the ability to perform activities
requiring ≥4 metabolic equivalents (METs):

• Patients with increased risk (>1%) who can perform ≥4 METs can also proceed to surgery without further
evaluation.

• Patients with increased risk (>1%) and unknown or poor (<4 METs) functional status require further
consideration. Cardiac stress testing is recommended only if cardiac intervention (if necessary) will impact
surgical/perioperative management (eg, postponing elective surgery for revascularization followed by 6
months of antiplatelet therapy). Because these patients are typically less likely to achieve an adequate
exercise workload, pharmacologic-based testing is recommended over exercise-based (eg, treadmill) testing
(Choices B and D).

Generally, extensive preoperative cardiac testing is necessary only in patients with significant cardiac conditions
undergoing high-risk procedures.

(Choice A) Coronary angiography is indicated for patients with acute myocardial infarction, unstable angina, or
abnormal stress testing. This patient has no anginal symptoms, making coronary angiography unnecessary.

(Choice E) Preoperative resting echocardiography is generally necessary only if indicated in the absence of

https://afkebooks.com
planned surgery (eg, new murmur, clinical status change in a patient with heart failure). This patient's preoperative
assessment shows no evidence of heart failure or valvular disease (eg, no dyspnea or murmur).

Educational objective:
Patients undergoing noncardiac surgery should be assessed for perioperative cardiac risk based on procedure- and
patient-specific risk factors. Low-risk patients and those at increased risk (>1%) with adequate functional capacity
(≥4 metabolic equivalents) can typically undergo surgery without further testing.

https://afkebooks.com
Question #52

A 50-year-old man is evaluated prior to undergoing routine screening colonoscopy. Medical history is significant for
a mechanical aortic valve replacement 5 years ago for severe aortic stenosis due to congenital bicuspid aortic
valve. The patient takes warfarin. He has no known drug allergies. Vital signs are within normal limits. Cardiac
examination reveals a normal S1 and mechanical S2. Which of the following is the most appropriate prophylaxis
prior to colonoscopy?

A) Amoxicillin

B) Ceftriaxone

C) Clindamycin

D) No antibiotic prophylaxis needed

E) Vancomycin

https://afkebooks.com
Explanation
Correct Answer:

D) No antibiotic prophylaxis needed

Prevention of infective endocarditis with antimicrobial prophylaxis*

• Gingival/apical tooth manipulation (amoxicillin)


High-risk procedures
• Respiratory mucosa incision (amoxicillin)
• Surgery on infected skin or muscle (vancomycin)
(Prophylaxis needed)
• GI or GU procedure in setting of active infection (ampicillin)

Low-risk procedures
• GI or GU endoscopy/procedure in absence of infection
• Most vaginal/caesarian deliveries
(No prophylaxis)

*Indicated for high-risk patients (eg, prosthetic heart valve, previous infective endocarditis, certain congenital
cyanotic heart diseases).

GI = gastrointestinal; GU = genitourinary.

Patients with a mechanical heart valve are at high risk for infective endocarditis (IE). Most cases are thought to
occur when a small thrombus forms on the valve and is subsequently seeded during an episode of transient
bacteremia. Although transient bacteremia frequently occurs during normal day-to-day activities (eg, brushing
teeth), risk is increased during a subset of invasive procedures as follows:

https://afkebooks.com
• Dental work that penetrates the gingival surface, including routine teeth cleaning

• Respiratory procedures that penetrate the mucosal surface (eg, tonsillectomy, bronchoscopy with biopsy)

• Skin and soft tissue procedures in an area of active infection (eg, abscess)

• Genitourinary and gastrointestinal procedures in an area of active infection (eg, diverticulitis)

To reduce risk of transient bacteremia, patients with cardiac conditions associated with a high risk for IE are given
prophylactic antibiotics 30-60 minutes prior to the procedure. Amoxicillin is generally used to counter
Streptococcus viridans bacteremia during dental work/respiratory procedures and to counter enterococci during
genitourinary/gastrointestinal procedures involving infected tissue; vancomycin is generally given prior to skin and
soft tissue procedures in areas of infection (Choices A and E).

However, antimicrobial prophylaxis is not indicated for gastrointestinal and genitourinary procedures (eg,
screening colonoscopy) that do not involve areas of active infection because risk of transient bacteremia is low.
Prophylaxis is also not usually given for dental work that involves only orthodontics/radiographs or prior to vaginal/
cesarean delivery.

(Choice B) Ceftriaxone can be used in place of amoxicillin in those who are unable to take oral medication. It
would not be indicated for a low-risk procedure such as colonoscopy.

(Choice C) Clindamycin can be used in place of amoxicillin in patients with penicillin allergy. It is a second-line
agent due to high risk for Clostridioides difficile infection. It would also not be indicated for a low-risk procedure
such as colonoscopy.

Educational objective:
Patients with prosthetic heart valves, prosthetic cardiac grafts, and certain congenital heart abnormalities are at high
risk for infective endocarditis due to transient bacteremia. These patients should receive antibiotic prophylaxis (eg,
amoxicillin) 30-60 minutes prior to invasive procedures that involve the gingival surface, respiratory mucosa,
infected skin and soft tissue, and infected areas of the gastrointestinal/genitourinary tract. No prophylaxis is
required for most other genitourinary/gastrointestinal procedures (eg, screening colonoscopy).

https://afkebooks.com
Question #53

A 65-year-old woman comes to the office due to a 2-month history of shortness of breath and lower extremity
edema. The patient reports no chest pain, palpitations, or syncope. She takes chlorthalidone and lisinopril for
chronic hypertension. The patient has a sedentary lifestyle. She drinks 1-2 glasses of wine with dinner most days
of the week. Blood pressure is 145/94 mm Hg and pulse is 80/min. BMI is 40 kg/m2. Estimated jugular venous
pressure is 9 cm H2O. Cardiac examination reveals a regular rate and rhythm with no murmurs. Crackles are
heard bilaterally in the lung bases. Bilateral pitting pedal edema is present. Chest x-ray shows a normal cardiac
silhouette with pulmonary vascular congestion. Echocardiogram demonstrates left atrial enlargement, mild
concentric left ventricular (LV) hypertrophy, mild mitral regurgitation, and no pericardial effusion. LV ejection fraction
is 65% and there are no wall motion abnormalities. Serum creatinine is 0.8 mg/dL. Serum lipid studies show a total
cholesterol of 208 mg/dL, HDL cholesterol of 35 mg/dL, and LDL cholesterol of 136 mg/dL. Which of the following
is the strongest predisposing factor to this patient's current condition?

A) Alcohol use

B) Hereditary mutation in sarcomere gene

C) Hypercholesterolemia

D) Obesity and sedentary lifestyle

E) Valvular heart disease

F) Viral infection

https://afkebooks.com
Explanation
Correct Answer:

D) Obesity and sedentary lifestyle

Heart failure with preserved ejection fraction

Etiology • LV diastolic dysfunction due to impaired relaxation

• Chronic hypertension (concentric LV hypertrophy)


• Obesity & sedentary lifestyle (myocardial interstitial
Risk factors
fibrosis)
• CAD & related risk factors (eg, diabetes mellitus)

• Specific therapies to reduce hospitalization & possibly


mortality:
◦ MRAs (eg, spironolactone)
◦ SGLT2 inhibitors (eg, dapagliflozin)
Management • Loop diuretics as needed to treat volume overload
• Additional antihypertensives PRN to reduce afterload
• Treatment of exacerbating conditions (eg, CAD, OSA, A-
fib)
• Exercise training/cardiac rehabilitation

https://afkebooks.com
A-fib = atrial fibrillation; CAD = coronary artery disease; LV = left ventricular;
MRAs = mineralocorticoid antagonists; OSA = obstructive sleep apnea;
PRN = as needed; SGLT2 = sodium-glucose cotransporter 2.

This patient has clinical signs and symptoms of decompensated heart failure (eg, dyspnea, pulmonary and lower
extremity edema, jugular venous distension). Heart failure is characterized by reduced cardiac output due to
impaired contractility (ie, systolic dysfunction) and/or relaxation (ie, diastolic dysfunction). This patient's normal
ejection fraction on echocardiogram is indicative of heart failure with preserved ejection fraction (HFpEF).

HFpEF is the result of progressive cardiac remodeling that is induced by both systemic and local factors affecting
cardiac myocytes and the surrounding interstitium. Chronic hypertension results in elevated systemic vascular
resistance, which exerts a hydraulic stress on the left ventricle (LV) by increasing afterload. This stress on the LV
initiates concentric hypertrophy of cardiac sarcomeres, which is perpetuated by systemic inflammation commonly
seen in individuals who are sedentary and/or obese (eg, cytokines released from adipocytes).

Systemic inflammation predisposes patients to microvascular ischemia, which leads to further sarcomeric
hypertrophy and collagen deposition in the interstitial space. The result of these changes is stiffening of the LV,
impaired diastolic filling (decreased compliance), and heart failure (reduced cardiac output).

(Choices A and F) Heavy alcohol use and certain viruses (eg, coxsackie) can occasionally cause dilated
cardiomyopathy, which would present as heart failure with reduced ejection fraction (HFrEF) (ie, systolic
dysfunction) rather than HFpEF.

(Choice B) Hypertrophic cardiomyopathy, caused by a plethora of hereditary mutations in various sarcomere


genes, can also present with concentric hypertrophy and diastolic dysfunction. However, hypertrophic changes
typically occur in the septum (rather than the entire myocardium) and can be easily visualized on echocardiogram.

(Choice C) Hypercholesterolemia is a significant risk factor for developing coronary artery disease (CAD).
Although CAD is common in patients with HFpEF, it is not as significant a contributor to HFpEF as hypertension,
obesity, or a sedentary lifestyle. Myocardial ischemia due to CAD most commonly manifests as HFrEF.

https://afkebooks.com
(Choice E) Severe regurgitant valvular heart disease (eg, mitral regurgitation, aortic regurgitation) is a common
cause of HFrEF, but does not typically contribute to HFpEF; in addition, this patient only has mild (rather than
severe) mitral regurgitation. Aortic stenosis can contribute to the concentric hypertrophy that underlies HFpEF, but
it is unlikely in this patient with no murmurs.

Educational objective:
Heart failure with preserved ejection fraction is characterized by progressive stiffening of the left ventricle resulting
in impaired diastolic filling and reduced cardiac output. In addition to chronic hypertension, systemic inflammation
from a sedentary lifestyle and/or obesity plays a significant role.

Reference
• Sedentary lifestyle and the risk for HFpEF: are &quot;huff-puff health clubs&quot; the answer?
Question #54

A 50-year-old man comes to the office for a follow-up visit. He has essential hypertension that was diagnosed a
year ago. Laboratory results at that time for complete blood cell count, chemistry panel, and urinalysis were
normal. At his last office visit 4 weeks ago, the patient's blood pressure was 144/92 mm Hg. He exercises regularly
and has been compliant with a low-salt diet. Current medications include a thiazide diuretic and an ACE inhibitor.
He quit smoking 2 years ago after a 25-pack-year history. The patient drinks 3 or 4 glasses of wine every day and
6-8 beers on weekend nights with his friends. He does not use illicit drugs. His father died following a stroke at age
60. Today, the patient's blood pressure is 143/88 mm Hg, and pulse is 76/min. The rest of his physical examination
is normal. Which of the following is the most appropriate next step in management of this patient's hypertension?

A) Counsel for reduction in alcohol intake

B) Increase the dose of thiazide diuretic

C) Obtain a plasma aldosterone/renin ratio

D) Obtain a renal vascular sonogram

E) Start a beta blocker


Explanation
Correct Answer:

A) Counsel for reduction in alcohol intake

This patient with hypertension has a persistently elevated blood pressure (BP) despite taking appropriate
antihypertensive medication. Common causes of such persistent BP elevations include medication nonadherence,
dietary indiscretion (eg, excessive sodium intake), and concurrent medications (eg, nonsteroidal anti-inflammatory
drugs, decongestants, glucocorticoids) that raise BP.

In addition, excessive alcohol intake (ie, >2 drinks/day) and binge drinking (ie, ≥5 drinks in a single sitting) are
associated with increased incidence of hypertension, higher BP, and failure to respond to appropriate
antihypertensive therapy. In contrast, moderate alcohol intake (eg, ≤2 drinks/day for men, ≤1 drink/day for
women) is associated with lower BP, decreased incidence of coronary artery disease, and lower cardiovascular
mortality. Although adding other agents to control this patient's BP may eventually be necessary, the first step
should be counseling regarding his drinking.

(Choices B and E) Addition or titration of antihypertensive medication may be considered for this patient later if
moderation of alcohol intake is unsuccessful. If a third medication is needed, a calcium channel blocker (eg,
amlodipine) is preferred over a beta blocker in the absence of additional specific indications (eg, heart failure,
hyperthyroidism). Thiazide diuretics have a nonlinear dose-response curve; the BP-lowering effect is
proportionately greater at lower doses, and up-titration often induces greater adverse effects than benefits.

(Choices C and D) Screening tests (eg, plasma aldosterone/renin ratio, renal vascular ultrasonography) for
secondary causes of hypertension are recommended for patients with treatment-resistant hypertension (ie, BP
elevated despite ≥3 appropriate antihypertensive medications). They also are indicated for unusual presentations
(eg, young or very old age of onset, sudden onset) and when standard initial tests suggest a secondary cause (eg,
hypokalemia). Most patients have primary hypertension; in the absence of specific indications, routine testing for
secondary causes is not cost effective and may result in false-positive results.
Educational objective:
Excessive alcohol intake (ie, >2 drinks/day) and binge drinking (ie, ≥5 drinks in a single sitting) are associated with
increased incidence of hypertension, higher blood pressure, and failure to respond to appropriate antihypertensive
therapy. In contrast, moderate alcohol intake is associated with lower blood pressure.

Reference
• Hypertension.

• Alcohol-induced hypertension: mechanism and prevention.


Question #55

A 50-year-old woman comes to the office due to lower extremity edema. The edema started about 6 weeks ago
and has slowly progressed . Medical history is significant for hypertension, treated with metoprolol for 3 years. Two
months ago, amlodipine was added because of inadequate blood pressure control with metoprolol alone. The
patient does not use tobacco or alcohol. She has no known drug allergies. Blood pressure is 130/80 mm Hg and
pulse is 64/min. Physical examination reveals bilateral symmetric 3+ pitting edema of both lower extremities,
without any skin changes or varicosities. Her neck vein pulsation is normal. The rest of the examination is
unremarkable. Laboratory results are as follows:

Serum chemistry
Albumin 4.5 g/dL
Total bilirubin 0.8 mg/dL
Sodium 140 mEq/L
Potassium 4.0 mEq/L
Creatinine 0.8 mg/dL

Urinalysis is within normal limits. What is the most likely cause of the edema in this patient?

A) Heart failure

B) Liver disease

C) Renal disease

D) Venous insufficiency

E) Medication side effect


Explanation
Correct Answer:

E) Medication side effect

Peripheral edema is a common side effect of treatment with dihydropyridine calcium channel blockers (CCBs). It
is due to these medications' vasodilatory effects on peripheral blood vessels, resulting in increased pressure and
capillary permeability. It does not represent an allergic reaction. If the edema is significant, the CCB should be
discontinued; because the edema is not due to increased plasma volume, diuretics are generally unhelpful.

In this patient, clues to the diagnosis of medication-induced peripheral edema include the recent initiation of the
dihydropyridine CCB amlodipine in conjunction with unremarkable physical examination findings (apart from the
edema) and normal laboratory results. Other more serious causes of edema can often be excluded by clinical and
laboratory data.

(Choice A) Congestive heart failure is an important cause of lower extremity edema, but the absence of other
symptoms (eg, dyspnea, orthopnea) and physical findings (eg, elevated neck vein pulsation, liver enlargement) of
heart failure makes this diagnosis unlikely in this patient.

(Choice B) Liver diseases can cause lower extremity edema. However, ascites usually dominates over peripheral
edema, and abnormal laboratory findings characteristic of liver dysfunction (eg, hypoalbuminemia,
hyperbilirubinemia) are present.

(Choice C) Renal diseases can cause peripheral edema due to massive proteinuria (eg, nephrotic syndrome) or
fluid retention (eg, acute nephritic syndrome). This patient's normal creatinine, lack of proteinuria, and normal
albumin result (ie, no hypoalbuminemia) make renal causes of the edema unlikely.

(Choice D) Although venous insufficiency can cause edema, symmetric edema without skin changes and
varicosities argues against this diagnosis.

Educational objective:
Dihydropyridine calcium channel blockers can cause peripheral edema and should always be considered in the
differential diagnosis of this condition, along with other causes, such as heart failure, renal disease, and venous
insufficiency.
Question #56

A 29-year-old man comes to clinic 2 weeks after an emergency department visit for epistaxis requiring anterior
nasal packing. In the emergency department, his blood pressure was 170/110 mm Hg. He has occasional
headaches and fatigue but no chest pain, palpitations, or syncope. His medical history is unremarkable and he
does not use tobacco, alcohol, or recreational drugs. The patient's current blood pressure is 180/112 mm Hg and
pulse is 78/min and regular. Cardiac auscultation in the supine position reveals no murmurs or additional sounds.
Abdominal examination shows no periumbilical bruits. Laboratory results are as follows:

Hemoglobin 14.2 g/dL


Platelets 230,000/mm3
Creatinine 1.0 mg/dL

Urine toxicology screen is negative. ECG shows normal sinus rhythm, high-voltage QRS complexes, and
downsloping ST-segment depression as well as T wave inversion in leads V5 and V6. Which of the following is the
best next step in evaluation of this patient?

A) Ambulatory blood pressure monitoring

B) Bilateral arm and leg blood pressure measurements

C) Cardiac auscultation in squatting and standing positions

D) Carotid sinus massage

E) Exercise stress testing


Explanation
Correct Answer:

B) Bilateral arm and leg blood pressure measurements

Coarctation of the aorta

• Congenital
Etiology
• Acquired (rare) (eg, Takayasu arteritis)

• Upper body
◦ Well developed
◦ Hypertension (headaches, epistaxis)
• Lower extremities
Clinical features ◦ Underdeveloped
◦ Claudication
• Brachial-femoral pulse delay
• Upper & lower extremity blood pressure differential
• Left interscapular systolic or continuous murmur

• ECG: Left ventricular hypertrophy


• Chest x-ray
Diagnostic studies
◦ Inferior notching of the 3rd to 8th ribs
◦ "3" sign due to aortic indentation
• Echocardiography: diagnostic confirmation

• Balloon angioplasty ± stent placement


Treatment
• Surgery

This patient's elevated blood pressure and ECG findings consistent with left ventricular hypertrophy (LVH) (eg, high-
voltage QRS complexes, lateral ST segment depression, lateral T wave inversion) are suggestive of long-standing
systemic hypertension. Essential hypertension rarely leads to end-organ damage (eg, LVH) in young patients
(eg, age <40), and therefore a cause of secondary hypertension should be sought.

Coarctation of the aorta is a narrowing of the descending aorta (typically located just distal to the origin of the left
subclavian artery) that results in a proximal increase in arterial pressure and decreased blood flow to the lower
body. Patients usually present with asymptomatic hypertension; however, epistaxis, headaches, and lower
extremity claudication can occur. A continuous murmur is characteristically associated with coarctation but is not
always noted and may be particularly difficult to auscultate in the supine position. Patients should be initially
evaluated for coarctation of the aorta with:

• Simultaneous palpation of the brachial and femoral pulses to assess for brachial-femoral delay

• Bilateral upper extremity (supine position) and lower extremity (prone position) blood pressure measurement
to evaluate for upper and lower extremity blood pressure differential

Patients with abnormal findings should undergo diagnostic confirmation with echocardiogram.

(Choice A) Ambulatory blood pressure monitoring is sometimes used for patients in whom the diagnosis of
hypertension is uncertain. It is not indicated in this patient with significantly elevated blood pressure and evidence
of LVH.

(Choice C) Squatting increases the volume of blood in the left ventricle and allows for differentiating the systolic
murmur of aortic stenosis (intensifies with squatting, lessens with standing) from that of hypertrophic
cardiomyopathy (lessens with squatting, intensifies with standing). This patient's epistaxis, headaches, and
systemic hypertension are more suggestive of aortic coarctation.

(Choice D) Carotid sinus massage slows atrioventricular node conduction and can sometimes terminate
paroxysmal supraventricular tachycardia (eg, atrioventricular nodal reentrant tachycardia). However, it is not useful
in the evaluation or treatment of hypertension.

(Choice E) Exercise stress testing is often used to evaluate for coronary artery disease; however, this patient's
ECG changes are more suggestive of LVH. In addition, coronary artery disease is very unlikely in this young
patient, especially in the absence of chest pain.

Educational objective:
Coarctation of the aorta is a potential cause of secondary hypertension in young adults. Patients should be initially
evaluated with simultaneous palpation of the brachial and femoral pulses to assess for brachial-femoral delay, and
bilateral upper and lower extremity blood pressure measurement to assess for blood pressure differential.

Reference
• Coarctation of the aorta.

• ACC/AHA 2008 guidelines for the management of adults with congenital heart disease: a report of the
American College of Cardiology/American Heart Association Task Force on Practice Guidelines.
Question #57

A 21-year-old woman is evaluated for syncope. She has had recurrent episodes of syncope since her mid-teens.
The first episode happened when she was having blood drawn for a test. She has also passed out several times
while standing in a crowded church. The syncope is typically preceded by a feeling of warmth, lightheadedness,
diaphoresis, and nausea. During one episode, the patient had a minor head injury and facial laceration. Her
medical history is otherwise insignificant. She takes no medications and uses no illicit drugs. Blood pressure is
122/76 mm Hg supine and 125/80 mm Hg after 2 minutes of standing. Pulse is 72/min and regular. Physical
examination is unremarkable. ECG shows normal sinus rhythm with no significant abnormalities. Long-term ECG
monitoring is most likely to show which of the following immediately preceding and during the syncopal episode?

A) Atrial fibrillation

B) Bradycardia and sinus arrest

C) Sinus tachycardia

D) Third-degree atrioventricular block

E) Ventricular tachycardia
Explanation
Correct Answer:

B) Bradycardia and sinus arrest

Vasovagal & situational syncope

• Pain,* anxiety,* emotional stress, heat, prolonged standing


Triggers
• Situational: cough,* micturition,* defecation, eating, hair-combing*

Clinical • Prodrome (eg, warmth, pallor, nausea, diaphoresis)


presentation • Rapid recovery of consciousness (eg, <1-2 min)

• Mainly based on clinical history of event


Diagnosis
• Upright tilt table testing sometimes indicated in uncertain cases

• Reassurance & avoidance of triggers


Treatment
• Counterpressure techniques for recurrent episodes

*Particularly common triggers in the pediatric population.

Syncope is a quick-resolving loss of consciousness that results from transient cerebral hypoperfusion leading to
loss of postural tone (eg, collapse). This patient is most likely experiencing vasovagal syncope (neurocardiogenic
syncope), which occurs due to an alteration in autonomic drive. Most commonly, abrupt parasympathetic
activation leads to a cardioinhibitory response that manifests as bradycardia with sinus arrest. Less
commonly, decreased sympathetic drive leads to a vasodepressor response, resulting in peripheral vasodilation
and abrupt hypotension. In both scenarios, cerebral perfusion suddenly decreases and syncope occurs. The
episode resolves quickly (typically <1 minute) as cerebral perfusion is rapidly restored.

Vasovagal syncope most commonly affects young patients and is often triggered by emotional stress (eg, fear of
venipuncture) or prolonged standing. The diagnosis is usually clinical. A prodrome (eg, nausea, diaphoresis,
pallor, lightheadedness) typically precedes vasovagal syncope and helps differentiate it from serious causes of
syncope (eg, cardiogenic syncope). In addition, measurement of normal orthostatic blood pressure helps rule out
orthostatic syncope.

Isolated episodes of vasovagal syncope require no treatment other than reassurance. Recurrent episodes may
warrant counseling to avoid common triggers as well as training in counterpressure techniques (eg, leg-crossing,
hand grip) to try to avert a syncope episode once prodromal symptoms occur.

(Choices A and E) Atrial arrhythmias (eg, atrial fibrillation, atrial flutter) with a rapid ventricular rate (eg, >150/min)
and ventricular arrhythmias (eg, ventricular tachycardia) can cause cardiogenic syncope due to inadequate
ventricular filling and loss of cardiac output. The syncope is typically sudden; patients may experience palpitations
prior to the episode, but a prodrome of nausea and diaphoresis is not typical.

(Choice C) Sinus tachycardia does not cause syncope. Patients often have sinus tachycardia as a reflex response
after a syncope episode.

(Choice D) Third-degree atrioventricular block can cause cardiogenic syncope but typically in patients with a
history of heart disease (eg, myocardial infarction, systemic amyloidosis, Lyme carditis). It would be unusual in this
young woman with no significant medical history. Varying degrees of atrioventricular block can occur as a
cardioinhibitory response in vasovagal syncope; however, sinus bradycardia with sinus arrest is the typical finding
on cardiac monitoring.

Educational objective:
Vasovagal syncope is typically preceded by a prodrome (eg, nausea, diaphoresis, pallor, lightheadedness) and
usually occurs due to a cardioinhibitory response from increased parasympathetic activity. Cardiac monitoring
immediately preceding the syncope typically shows sinus bradycardia and asystole due to sinus arrest.

Reference
• Guidelines for the diagnosis and management of syncope (version 2009).

• Mechanism of syncope in patients with isolated syncope and in patients with tilt-positive syncope.
Question #58

A 68-year-old woman comes to the office for follow-up after a recent emergency department visit. Two weeks ago,
the patient experienced palpitations and mild dizziness; ECG showed atrial fibrillation with rapid ventricular
response. The episode resolved spontaneously in 2 hours, and she has had no symptoms since then. The patient
has a history of hypertension treated with valsartan and hydrochlorothiazide. She is a lifetime nonsmoker and does
not drink alcohol. Her exercise tolerance is good. Blood pressure is 128/72 mm Hg, and pulse is 74/min and
regular. Physical examination is unremarkable. TSH is 1.6, creatinine is 1.1 mg/dL, and fasting glucose is 85 mg/
dL. Echocardiography shows preserved left ventricular ejection fraction and no significant valvular abnormalities.
Which of the following is the best next step in management of this patient?

A) Ambulatory heart rhythm monitoring

B) Amiodarone

C) Apixaban

D) Aspirin

E) Carotid ultrasound

F) Clinical follow-up only


Explanation
Correct Answer:

C) Apixaban

CHA2DS2-VASc score for thromboembolic risk


in nonvalvular atrial fibrillation

Risk criteria Points

C Congestive heart failure 1

H Hypertension 1

A2 Age ≥75* 2

D Diabetes mellitus 1

S2 Stroke or TIA 2
V Vascular disease (eg, PAD, prior MI) 1

A Age 65-74* 1

Sc Sex category female 1

Maximum score 9

Total score
Generalized stroke risk Antithrombotic therapy
Male Female**

0 0 Low None

1 2 Moderate None or oral anticoagulant

≥2 ≥3 High Oral anticoagulant

*Patients are assigned to 1 of the 2 age categories.


**Different cutoffs are used for males & females. Female sex is considered a risk modifier that adds to the
CHA2DS2-VASc score only if other nonsex risk factors are present (female patients cannot have a total score of 1).

MI = myocardial infarction; PAD = peripheral artery disease; TIA = transient ischemic attack.

This patient had an episode of atrial fibrillation that spontaneously resolved within hours. At the time of her clinic
follow-up 2 weeks later, she appears to be in normal sinus rhythm (evidenced by a regular pulse rhythm). Given her
age and other risk factors (eg, hypertension), this patient will most likely continue to have intermittent episodes of
the arrhythmia (ie, paroxysmal atrial fibrillation).

The risk of systemic thromboembolism (eg, stroke) is increased in atrial fibrillation, likely to a similar degree in
both paroxysmal and persistent/permanent disease. Therefore, an important component in managing new-onset
atrial fibrillation is an assessment of the need for chronic anticoagulation to lower that risk. In patients with
nonvalvular atrial fibrillation, the CHA2DS2-VASc score is typically used to estimate annual risk, with chronic
anticoagulation indicated for men with a total score ≥2 and women with a total score ≥3 (indicating high
thromboembolic risk). Direct oral anticoagulants (eg, apixaban, rivaroxaban, dabigatran) are typically used for
chronic anticoagulation, although warfarin is sometimes used instead.

This patient has a CHA2DS2-VASc score of 3 (ie, hypertension, age, female sex); therefore, the initiation of
apixaban is most appropriate (Choice F). Chronic anticoagulation is not indicated for patients with a
CHA2DS2-VASc score of 0 and should be considered (but is not necessarily indicated) for patients with a score
indicating moderate risk (ie, 1 for men and 2 for women).

(Choice A) Ambulatory heart rhythm monitoring (eg, Holter monitor) is used to help diagnose an intermittent
arrhythmia that has not been captured on ECG. This patient's atrial fibrillation has already been recognized on
ECG and further diagnostic intervention is not needed.

(Choice B) Amiodarone is sometimes used as a rhythm-control agent for atrial fibrillation. However, it has multiple
side effects and is not a first-line agent. It is not indicated in this patient whose episode of atrial fibrillation has
resolved.
(Choice D) Aspirin is not significantly effective in lowering the risk of systemic thromboembolism in atrial fibrillation
and should not be used for this purpose, particularly in patients with an elevated CHA2DS2-VASc score.

(Choice E) Carotid ultrasound is not indicated in the management of new-onset atrial fibrillation. Stroke associated
with atrial fibrillation occurs due to embolization of atrial thrombus and is independent of carotid artery
atherosclerosis.

Educational objective:
The management of new-onset atrial fibrillation should involve an assessment of systemic thromboembolism risk.
Chronic anticoagulation (eg, apixaban) should be initiated for men with a CHA2DS2-VASc score ≥2 and women with
a score ≥3 to lower their risk.

Reference
• Anticoagulation in atrial fibrillation.
Question #59

A 64-year-old man comes to the emergency department due to palpitations and progressive shortness of breath
over the past several weeks. The patient says that he develops "a choking sensation accompanied by a dry cough"
every time he lies down. He has had no chest pain, sputum production, lightheadedness, or syncope. His other
medical conditions include hypertension for the past 20 years and medication nonadherence. He also has a
35-pack-year smoking history. The patient's father died of a heart attack at age 70, and his mother suffered from
asthma. Blood pressure is 182/105 mm Hg and pulse is 120/min and irregularly irregular. Lung examination
reveals bibasilar crackles. There is 2+ pitting edema of the lower extremities. Bedside echocardiography shows a
normal-sized left ventricle with wall hypertrophy, left ventricular ejection fraction of 55%, and no significant mitral or
aortic valve disease. Which of the following is most likely responsible for this patient's symptoms?

A) Cor pulmonale

B) Diastolic dysfunction

C) High-output heart failure

D) Multivessel coronary artery disease

E) Small airway bronchoconstriction


Explanation
Correct Answer:

B) Diastolic dysfunction

Heart failure with preserved ejection fraction

Etiology • LV diastolic dysfunction due to impaired relaxation

• Chronic hypertension (concentric LV hypertrophy)


• Obesity & sedentary lifestyle (myocardial interstitial
Risk factors
fibrosis)
• CAD & related risk factors (eg, diabetes mellitus)

• Specific therapies to reduce hospitalization & possibly


mortality:
◦ MRAs (eg, spironolactone)
◦ SGLT2 inhibitors (eg, dapagliflozin)
Management • Loop diuretics as needed to treat volume overload
• Additional antihypertensives PRN to reduce afterload
• Treatment of exacerbating conditions (eg, CAD, OSA, A-
fib)
• Exercise training/cardiac rehabilitation
A-fib = atrial fibrillation; CAD = coronary artery disease; LV = left ventricular;
MRAs = mineralocorticoid antagonists; OSA = obstructive sleep apnea;
PRN = as needed; SGLT2 = sodium-glucose cotransporter 2.

This patient's progressive dyspnea, orthopnea, bibasilar lung crackles, and lower extremity edema are classic for
decompensated heart failure. Echocardiography shows a normal left ventricular (LV) ejection fraction (ie, >50%),
consistent with heart failure with preserved ejection fraction (HFpEF).

HFpEF results from diastolic dysfunction (ie, impaired relaxation with decreased compliance) that leads to
elevated LV diastolic filling pressure and reduced cardiac output. Concentric hypertrophy due to chronic
hypertension is usually the primary contributor, causing a normal-sized LV cavity with thickened walls. Obesity
and sedentary lifestyle are also often strong contributors, in part through their association with hypertension but
also via systemic inflammation that leads to microvascular myocardial ischemia and deposition of interstitial
collagen.

Atrial fibrillation, which is likely present in this patient with palpitations and an irregularly irregular pulse, is a
common precipitant of an acute exacerbation of HFpEF. Due to impaired diastolic filling at baseline, patients with
HFpEF are highly sensitive to the loss of atrial kick that occurs with atrial fibrillation.

(Choice A) Cor pulmonale refers to right ventricular failure resulting from pulmonary pathology that leads to
pulmonary hypertension (eg, chronic obstructive pulmonary disease [COPD], pulmonary embolism). Pulmonary
edema is not present with isolated right ventricular failure; this patient's bibasilar crackles and orthopnea suggest LV
failure.

(Choice C) High-output heart failure involves increased cardiac output in response to a condition causing reduced
systemic vascular resistance (eg, arterial venous fistula, hyperthyroidism). Echocardiography typically shows
dilation of both the right ventricle and left ventricle as they accommodate the high flow state.

(Choice D) Coronary artery disease (CAD) is often present in patients with HFpEF; it likely does not significantly
contribute to the pathogenesis of the disease but it may worsen the clinical manifestations (eg, precipitate heart
failure exacerbation). Ischemic heart disease due to CAD most commonly leads to systolic dysfunction and heart
failure with reduced ejection fraction.

(Choice E) Small airway bronchoconstriction due to asthma or COPD would not explain this patient's evidence of
left-sided heart failure (eg, lung crackles, orthopnea).

Educational objective:

Heart failure with preserved ejection fraction results from impaired diastolic relaxation (ie, diastolic dysfunction) in
the setting of a normal left ventricular (LV) ejection fraction (ie, >50%). Concentric LV hypertrophy due to chronic
hypertension is usually the primary contributor. Patients develop typical heart failure symptoms (eg, dyspnea,
orthopnea, lower extremity edema) resulting from elevated LV diastolic filling pressure and reduced cardiac output.

Reference
• Clinical practice. Diastolic heart failure.

• 2013 ACCF/AHA guideline for the management of heart failure: executive summary: a report of the
American College of Cardiology Foundation/American Heart Association Task Force on practice guidelines.
Question #60

A 72-year-old man comes to the office for a health maintenance examination. The patient feels well and does not
report any symptoms, aside from occasional pain in his knees. Medical history is significant for hypertension and
osteoarthritis, for which he takes hydrochlorothiazide, lisinopril, and as-needed acetaminophen. The patient has a
30-pack-year smoking history but quit 15 years ago. He does not use alcohol. He is a retired plumber and lives
with his wife in a retirement community. During the visit, the patient reveals that a friend recently had an abdominal
aortic aneurysm repair and inquires whether he should be screened for the same disorder. Which of the following is
the most appropriate next step in management of this patient?

A) No further testing required

B) Obtain a CT scan of the abdomen

C) Obtain an abdominal ultrasound

D) Obtain an echocardiogram

E) Obtain an MRI of the abdomen


Explanation
Correct Answer:

C) Obtain an abdominal ultrasound

Screening: abdominal aortic aneurysm

Patient • Men
population • Age 65-75

Risk factor
• Any smoking history
indication

• One-time abdominal duplex


Test
ultrasonography

Abdominal aortic aneurysm (AAA) is a common disorder with a long asymptomatic phase. The mortality rate for
patients with ruptured AAA is very high (75%-90%), but the risk is mitigated if the aneurysm is repaired prior to
rupture, making it an appropriate subject for screening in patients at risk. The primary risk factors for AAA include
male gender, smoking history, and age >65.

The US Preventive Services Task Force recommends screening for AAA with a one-time abdominal ultrasound in
men age 65-75 who have ever smoked (ie, any lifetime exposure >100 cigarettes). Screening and surgical repair
of large AAAs (>5.5 cm) have been found to decrease AAA-specific mortality in this population. Ultrasonography is
the recommended screening modality because of its high sensitivity (>95%), low cost, noninvasive nature, and lack
of ionizing radiation.

Screening for AAA may also be beneficial for men age 65-75 who have not smoked but have other significant risk
factors (eg, first-degree family history of AAA rupture). Screening in younger men and in women appears to have
little, if any, benefit and is not recommended.

(Choices B and E) CT and MRI both have high sensitivity for AAA, and AAA is often detected incidentally on
abdominal imaging performed for other purposes. However, these tests are more expensive and less validated
than ultrasound for screening. In addition, CT exposes patients to ionizing radiation.

(Choice D) Transesophageal and, to a lesser degree, transthoracic echocardiogram can detect dilation of the
proximal ascending aorta, but they do not play a role in screening for AAA.

Educational objective:
The primary risk factors for abdominal aortic aneurysm include male gender, smoking history, and age >65. The US
Preventive Services Task Force recommends screening for this condition with a one-time abdominal ultrasound in
men age 65-75 who ever smoked.

Reference
• Ultrasonography screening for abdominal aortic aneurysms: a systematic evidence review for the U.S.
Preventive Services Task Force.

• Screening for abdominal aortic aneurysm: U.S. Preventive Services Task Force recommendation statement
Question #61

A 63-year-old woman comes to the office due to leg swelling that is especially bothersome in the evening. Her
symptoms have gradually worsened over the last year. Her medical problems include hypertension treated with
lisinopril and obstructive sleep apnea for which she uses continuous positive airway pressure during sleep. She
was hospitalized 2 years ago for a chest infection that was treated with antibiotics. The patient has smoked a pack
of cigarettes daily for 30 years and does not drink alcohol. Blood pressure is 160/90 mm Hg and pulse is 80/min.
BMI is 32 kg/m2. Jugular venous pulsation is seen 2 cm above the sternal angle with the head of the bed elevated
to 45°. Chest examination shows bilateral scattered wheezes and prolonged expirations. Her abdomen is soft and
nondistended. She has bilateral 2+ pitting edema in her lower extremities to the midshin with dilated and tortuous
superficial veins. A small ulcer is noted on the left medial ankle. All peripheral pulses are palpable. Which of the
following is most likely to relieve this patient's current symptoms?

A) Daily furosemide

B) Dietary sodium restriction

C) Frequent leg elevation

D) Improved control of hypertension

E) Smoking cessation
Explanation
Correct Answer:

C) Frequent leg elevation

This patient's lower extremity (LE) swelling is likely due to chronic venous insufficiency (CVI), which is most
commonly caused by incompetence of venous valves leading to venous hypertension in the deep venous system
of the legs. Patients may present with leg discomfort, pain, or swelling that is typically worse in the evening or
following prolonged standing and improves after walking or leg elevation. Pitting edema is the most common
physical examination finding. In relatively severe cases, redirection of blood from the deep venous system to the
superficial venous system may lead to other physical examination findings, including abnormal venous dilation (eg,
telangiectasia, varicose veins), skin discoloration, lipodermatosclerosis, or skin ulceration (characteristically on the
medial aspect of the lower leg). Risk factors for CVI include advancing age, obesity, family history, pregnancy,
sedentary lifestyle, previous LE trauma, and previous LE venous thrombosis.

The diagnosis of CVI is usually based on history and physical examination, and initial treatment includes leg
elevation, exercise, and compression stockings. Patients who do not respond to initial conservative measures
should undergo venous duplex ultrasound to confirm the diagnosis of CVI by identification of venous reflux
(retrograde venous blood flow) in the deep venous system.

(Choice A) Diuretics (eg, furosemide) are useful in the management of heart failure, which is unlikely in this patient
given the lack of dyspnea and the normal jugular venous pressure. Diuretics are likely to cause dehydration in
patients with CVI and are generally not recommended.

(Choices B and D) Dietary sodium restriction and hypertension control are important interventions in the
management of LE edema due to heart failure, but do not play a significant role in the management of CVI. This
patient's normal jugular venous pressure and absence of crackles on lung examination (wheezing is likely due to
underlying chronic obstructive pulmonary disease rather than pulmonary edema) make heart failure unlikely.

(Choice E) Smoking is considered a risk factor for CVI; however, there is no evidence to support smoking
cessation as an effective management strategy for CVI. Smoking cessation is a first-line intervention for peripheral
arterial disease, which does not cause significant LE edema and is unlikely in this patient with palpable peripheral
pulses.

Educational objective:
Chronic venous insufficiency is a common cause of lower extremity edema that may be accompanied by varicose
veins, skin discoloration, and medial skin ulceration. Initial treatment includes conservative measures with leg
elevation, exercise, and compression therapy.

Reference
• Effect of leg elevation on the skin microcirculation in chronic venous insufficiency.

• Chronic venous insufficiency and venous ulceration.

• Chronic venous insufficiency: mechanisms and management.


Question #62

A 46-year-old man comes to the physician due to exertional dyspnea and dry cough. He also has occasional
episodes of suffocating nighttime cough that is relieved only by sitting up. Medical history is significant for
myocardial infarction 6 months ago and hypercholesterolemia. Current medications include metoprolol, aspirin, and
rosuvastatin. The patient drinks alcohol on social occasions but does not use tobacco or illicit drugs. His father
died of a stroke and his mother has type 2 diabetes mellitus. Blood pressure is 150/100 mm Hg and pulse is 60/
min. Chest examination shows bibasilar crackles. The cardiac apex is palpated in the left sixth intercostal space.
Bilateral pitting leg edema is present. Which of the following is most likely to be associated with this patient's
condition?

A) Constriction of the renal efferent arterioles

B) Decreased plasma colloid pressure

C) Decreased renal venous pressure

D) Dilation of the renal afferent arterioles

E) High sodium delivery to the distal tubule

F) Increased chloride delivery to the macula densa


Explanation
Correct Answer:

A) Constriction of the renal efferent arterioles


This patient's presentation with exertional dyspnea, cough, orthopnea, and evidence of pulmonary and peripheral
edema following recent myocardial infarction is consistent with decompensated heart failure (DHF). The initial
disturbance in heart failure is usually a reduction in left ventricular function that leads to reduced cardiac output.
The reduced cardiac output causes decreased organ and tissue perfusion that is sensed by arterial baroreceptors
and the juxtaglomerular apparatus of the kidneys, which triggers compensatory activation of the sympathetic
nervous system and renin-angiotensin-aldosterone system (RAAS). These systems stimulate both
vasoconstriction and sodium retention (ie, increased blood volume) to maintain organ and tissue perfusion.
Specifically, angiotensin II causes vasoconstriction of both the afferent and efferent renal arterioles (more prominent
of the efferent arterioles) to maintain the glomerular filtration rate.

Ultimately, these compensatory mechanisms are maladaptive as vasoconstriction increases afterload and sodium
retention increases preload, both of which place additional strain on the failing heart and further reduce cardiac
output. A vicious cycle ensues, eventually progressing to clinical decompensation with overt volume overload.

(Choice B) Plasma colloid pressure is mostly driven by albumin concentration. It is low in nephrotic syndrome and
decompensated cirrhosis, but it is typically normal in DHF. Edema in DHF results from elevated venous hydrostatic
pressure.

(Choice C) Renal venous pressure is increased in DHF as elevated pressure is transmitted back from the heart to
the vena cava and to the renal veins (ie, central venous pressure is elevated).

(Choices D, E, and F) The release of natriuretic peptides is triggered by myocardial stretch; these peptides
stimulate renal afferent arteriole vasodilation and sodium excretion to counteract the sympathetic and RAAS
pathways and offset the downward spiral of DHF. However, the natriuretic pathway is eventually overcome. Renal
arteriolar vasoconstriction predominates with reduced renal blood flow and reduced glomerular filtration (ie, reduced
sodium delivery to the distal tubule, reduced chloride delivery to the macula densa).

Educational objective:
Decompensated heart failure involves the activation of compensatory mechanisms in the form of the sympathetic
nervous system and renin-angiotensin-aldosterone system. These systems stimulate vasoconstriction and sodium
retention to maintain organ and tissue perfusion in the setting of reduced cardiac output. The compensatory
mechanisms are ultimately maladaptive as they further decrease cardiac output and perpetuate a downward spiral
of clinical decompensation.
Reference
• Congestive heart failure: pathophysiologic consequences of neurohormonal activation and the potential for
recovery: part I.
Question #63

A 32-year-old man comes to the office for a routine preemployment physical. He has been feeling well. Ten years
ago, his father died suddenly at age 54. Blood pressure is 175/103 mm Hg in the right arm and 180/105 mm Hg in
the left, and pulse is 82/min. BMI is 22.1 kg/m2. The lungs are clear bilaterally and heart sounds are normal.
Bilateral, nontender, upper abdominal masses are palpated on examination. Hemoglobin level is 15.2 g/dL and
creatinine concentration is 0.8 mg/dL. Which of the following is the most appropriate next step in evaluating this
patient's condition?

A) 24-hour urine cortisol

B) Abdominal ultrasound

C) Captopril-enhanced radionuclide renal scan

D) Plasma aldosterone/renin ratio

E) Urine metanephrines
Explanation
Correct Answer:

B) Abdominal ultrasound

Autosomal dominant polycystic kidney disease

• Most patients asymptomatic until age 30-40


• Flank pain, hematuria
Clinical
• Hypertension
presentation
• Palpable abdominal masses (usually bilateral)
• Chronic kidney disease (CKD)

• Cerebral aneurysms
• Hepatic & pancreatic cysts
Extrarenal
• Mitral valve prolapse, aortic regurgitation
features
• Colonic diverticulosis
• Ventral & inguinal hernias

Diagnosis • Ultrasonography showing multiple renal cysts

• Aggressive control of risk factors for CV & CKD


Management
• ACE inhibitors preferred for hypertension
• Hemodialysis, renal transplant for ESRD

CV = cardiovascular; ESRD = end-stage renal disease.

This patient with early-onset hypertension and bilateral upper abdominal masses likely has autosomal dominant
polycystic kidney disease (ADPKD), which is the most common genetic cause of chronic kidney disease.
Mutation of the PKD1 or PKD2 gene disrupts the function of polycystin proteins and leads to cystic degeneration of
the renal parenchyma that begins early in life.

Most patients are asymptomatic until age 30-40, when cysts have become large and numerous. Hypertension is
a common early disease manifestation and is typically present before any significant decline in renal function.
Other early disease findings can include hematuria, nephrolithiasis, flank pain, and urinary tract infections. Some
patients have an elevated hemoglobin concentration due to increased erythropoietin production (possibly from local
ischemia around the renal cysts). As cystic degeneration progresses with age, patients experience progressive
renal dysfunction that eventually leads to end-stage renal disease.

Extrarenal complications of ADPKD include cerebral aneurysms, hepatic or pancreatic cysts, cardiac valvular
abnormalities (ie, mitral valve prolapse, aortic regurgitation), colonic diverticula, and ventral or inguinal hernias. The
sudden death of this patient's father suggests rupture of a cerebral aneurysm in the setting of ADPKD. The
diagnosis of ADPKD is based mainly on abdominal ultrasound or CT showing enlarged kidneys with numerous
cysts.

(Choice A) Elevated 24-hour urine cortisol excretion is seen in Cushing syndrome. Although Cushing syndrome
can cause secondary hypertension, it is unlikely in this patient who is not overweight and is feeling well, without
suggestive symptoms (eg, fatigue, irritability). Cushing syndrome may be due to adrenal tumors but is unlikely to
cause palpable abdominal masses.

(Choice C) Captopril-enhanced radionuclide renal scan is used occasionally to diagnose suspected renovascular
disease or renal artery stenosis as a cause of refractory hypertension. However, these conditions are not
associated with palpable renal enlargement.

(Choice D) A high plasma aldosterone/renin ratio is seen in patients with primary aldosteronism (Conn syndrome),
which is a common cause of secondary hypertension. However, it usually involves a unilateral adrenal adenoma
that is not easily palpable; bilateral palpable adenomas would be highly unusual.

(Choice E) Urinary excretion of metanephrines is increased in patients with pheochromocytoma. These patients
usually have a paroxysmal elevation in blood pressure accompanied by headaches, palpitations, and/or
diaphoresis.

Educational objective:
Most patients with autosomal dominant polycystic kidney disease are asymptomatic until age 30-40, when renal
cysts become large and numerous. Bilateral upper abdominal masses are typically palpable on physical
examination, and hypertension is an early disease manifestation that typically precedes a progressive decline in
renal function.

Reference
• Autosomal dominant polycystic kidney disease.

• Hypertension in autosomal dominant polycystic kidney disease.


Question #64

A 42-year-old woman is evaluated for depression and poor sleep. She also has mild headaches and muscle
weakness. The review of hospital records indicates emergency department visits for anxiety and kidney stones.
The patient is a lifetime nonsmoker and does not use illicit drugs. Blood pressure is 160/105 mm Hg and pulse is
85/min. Laboratory results are as follows:

Sodium 139 mEq/L


Potassium 3.8 mEq/L
Chloride 102 mEq/L
Bicarbonate 24 mEq/L
Blood urea nitrogen 13 mg/dL
Creatinine 0.9 mg/dL
Glucose 98 mg/dL
Calcium 11.7 mg/dL
Albumin 3.7 g/dL

Which of the following best explains this patient's hypertension?

A) Amphetamine abuse

B) Cushing syndrome

C) Hyperparathyroidism

D) Hypothyroidism
E) Primary hyperaldosteronism

F) Renal artery stenosis

G) Renal parenchymal disease


Explanation
Correct Answer:

C) Hyperparathyroidism

Secondary causes of hypertension

Condition Clinical clues/features

• Elevated creatinine level


Renal parenchymal disease
• Abnormal urinalysis (proteinuria, red blood cell casts)

• Recurrent flash pulmonary edema


Renovascular disease • Elevated creatinine level (particularly with ACE inhibitor use)
• Abdominal bruit

• Hypokalemia (spontaneous or thiazide induced)


Primary hyperaldosteronism
• Metabolic alkalosis

• Daytime somnolence
Obstructive sleep apnea
• Increased neck circumference
• Paroxysmal hypertension & tachycardia
Pheochromocytoma
• Headaches, palpitations, diaphoresis

• Cushingoid body habitus & proximal muscle atrophy


Cushing syndrome
• Hyperglycemia

• Hyperthyroidism: anxiety, heat intolerance, weight loss, tachycardia


Thyroid disease
• Hypothyroidism: fatigue, cold intolerance, weight gain, bradycardia

• Mild hypercalcemia ± symptoms (eg, constipation)


Primary hyperparathyroidism
• Kidney stones

• Upper extremity hypertension with brachial-femoral pulse delay (common)


Coarctation of the aorta
• Lateralizing hypertension (less common)

This patient with hypertension associated with hypercalcemia, muscle weakness, kidney stones, and
neuropsychiatric symptoms (eg, depression, poor sleep) likely has primary hyperparathyroidism (PHPT).

Excess parathyroid hormone causes hypercalcemia due to increased renal calcium reabsorption, gastrointestinal
calcium absorption, and bone resorption. In addition to kidney stones, neuropsychiatric symptoms, and muscle
weakness, other common manifestations include abdominal pain, constipation, and polyuria or polydipsia. The
majority (~80%) of PHPT cases are due to parathyroid adenoma.

PHPT is commonly associated with hypertension; proposed mechanisms include increased renin secretion,
sympathetic hyperresponsiveness, and peripheral artery vasoconstriction. Other cardiovascular manifestations of
hyperparathyroidism include left ventricular hypertrophy, arrhythmias, and vascular and valvular calcification.
Treatment of PHPT usually results in normalization of blood pressure.

(Choice A) Amphetamine abuse can cause hypertension associated with additional signs and symptoms of
sympathetic stimulation (eg, tachycardia, diaphoresis, hyperthermia) but would not cause hypercalcemia and kidney
stones.

(Choice B) Findings suggestive of Cushing syndrome include central obesity, muscle wasting, thin skin, and
abdominal striae. Laboratory findings include hyperglycemia, leukocytosis, and hypokalemia.

(Choice D) Hypertension can be induced by both hyperthyroidism (predominantly systolic hypertension) and
hypothyroidism (predominantly diastolic hypertension). Although hyperthyroidism can cause hypercalcemia due to
increased bone turnover, hypothyroidism does not have a significant effect on serum calcium concentration.

(Choice E) Primary hyperaldosteronism (Conn syndrome) can cause hypertension and muscle weakness, but it is
typically associated with hypokalemia (especially when patients are given diuretics) rather than hypercalcemia.

(Choice F) Although renal artery stenosis can occur in young women with fibromuscular dysplasia, it is much more
common in older men with atherosclerosis. Suggestive findings include resistant hypertension, abdominal bruit,
recurrent flash pulmonary edema, and acute kidney injury following initiation of an ACE inhibitor.

(Choice G) Renal parenchymal disease can cause secondary hypertension but usually presents with elevated
serum creatinine and abnormal urinalysis. In addition, patients are usually hypocalcemic rather than hypercalcemic.

Educational objective:
Hyperparathyroidism is a cause of secondary hypertension and should be suspected in patients who have
hypertension associated with hypercalcemia, renal stones, abdominal pain, or neuropsychiatric symptoms. Other
cardiovascular manifestations of hyperparathyroidism include left ventricular hypertrophy, arrhythmias, and vascular
and valvular calcification.

Reference
• Primary hyperparathyroidism predicts hypertension: results from the National Inpatient Sample.
• Endocrine hypertension: a practical approach.
Question #65

An 82-year-old man comes to the office for evaluation of chronic back pain. On physical examination, he is found to
have a blood pressure of 160/85 mm Hg while supine and 135/70 mm Hg while standing. He is otherwise healthy;
his only medicine is occasional ibuprofen for back pain. Which of the following age-related changes best explains
the observed finding?

A) Increased left ventricular wall stiffness

B) Decreased left ventricular contractility

C) Decreased baroreceptor responsiveness

D) Decreased stress-mediated adrenal catecholamine release

E) Decreased glomerular filtration rate


Explanation
Correct Answer:

C) Decreased baroreceptor responsiveness

This patient has orthostatic hypotension, which is defined as a postural decrease in blood pressure by 20 mm
Hg systolic or 10 mm Hg diastolic (sometimes accompanied by an increase in heart rate) that occurs on standing.

In general, orthostatic hypotension results from insufficient constriction of blood vessels in the lower extremities on
standing, which may be due to a defect in autonomic reflexes, decreased intravascular volume, or medications.
Some baroreceptor sensitivity is lost as a normal part of aging. Arterial stiffness, decreased norepinephrine
content of sympathetic nerve endings, and reduced sensitivity of the myocardium to sympathetic stimulation all
contribute to a tendency toward orthostatic hypotension with age.

(Choices A and B) Normal changes to the aging heart include decreased resting and maximal cardiac output,
decreased maximum heart rate, increased contraction and relaxation time of heart muscle, increased myocardial
stiffness during diastole, decreased myocyte number, and pigment accumulation in myocardial cells. Increased
ventricular wall stiffness contributes to the development of heart failure with preserved ejection failure (diastolic
dysfunction) but does not play a role in orthostatic hypotension.

(Choice D) The adrenal medulla continues to release catecholamines in response to stress (fight or flight)
throughout life. It does not participate in the fine control of blood pressure.

(Choice E) Although the glomerular filtration rate (GFR) tends to decline with old age, a decreased GFR would
promote sodium retention and expansion of the intravascular volume, counteracting orthostatic hypotension.

Educational objective:
Progressively decreasing baroreceptor sensitivity and defects in the myocardial response to this reflex are the main
reasons for the increased incidence of orthostatic hypotension in the elderly.
Question #66

A 43-year-old man with a 2-year history of type 2 diabetes mellitus comes to the office for a routine preventive
examination. He feels well and has no symptoms. Medications include metformin and aspirin. The patient works
as a computer programmer and has a sedentary lifestyle. He drinks 1 or 2 cans of beer on weekends and has a
20-pack-year smoking history. The patient is 183 cm (6 ft) tall and weighs 109 kg (240 lb); BMI is 32.5 kg/m2.
Blood pressure is 153/94 mm Hg and pulse is 82/min. The remainder of the physical examination is unremarkable.
Hemoglobin A1c is 7.6%. At a previous visit 3 months ago, blood pressure was 149/92 mm Hg. Which of the
following interventions would most likely cause the largest decrease in this patient's blood pressure?

A) 15-kg (33-lb) weight loss

B) Aerobic exercise

C) Cessation of alcohol intake

D) Dietary salt restriction

E) Smoking cessation
Explanation
Correct Answer:

A) 15-kg (33-lb) weight loss

Lifestyle interventions for hypertension

Approximate ↓
Modification Recommended plan
systolic BP (mm Hg)

DASH diet Diet high in fruits & vegetables & low in saturated & total fats 11

Weight loss Reduction of BMI to <25 kg/m2 6 per 10-kg loss

Aerobic exercise 30 minutes/day for 5+ days/week 7

Dietary sodium <1.5-2.3 g/day (response varies) 5-8

Alcohol limitation ≤2 drinks/day in men, ≤1 drink/day in women 5


DASH = Dietary Approaches to Stop Hypertension.

This patient has hypertension, defined as systolic blood pressure ≥130 mm Hg and/or diastolic blood pressure ≥80
mm Hg. Current guidelines recommend that all patients with hypertension be advised to get regular aerobic
exercise, avoid excessive alcohol intake, and follow a dietary program proven to lower blood pressure (eg,
Dietary Approaches to Stop Hypertension [DASH] diet).

In addition, patients who are overweight (BMI 25-29.9 kg/m2) or obese (BMI ≥30 kg/m2) should be advised to lose
weight to achieve and maintain a healthy weight. Obesity, particularly central/visceral obesity, is the single greatest
lifestyle risk factor for hypertension, with an attributable risk of approximately 60%-70%. In patients with obesity,
each 10 kg (22 lb) of weight loss can lower blood pressure by ~6 mm Hg; therefore, weight loss is one of the most
effective nonpharmacologic interventions for lowering blood pressure. However, additional lifestyle modifications,
including dietary changes and exercise, are typically implemented concurrently.

(Choice B) Moderate-intensity aerobic exercise can lower blood pressure in both hypertensive and normotensive
individuals and facilitates weight loss. However, weight loss has a greater impact on blood pressure than exercise
alone.

(Choice C) Daily heavy alcohol consumption (≥3 drinks per day in men, ≥2 in women) or binge drinking can raise
blood pressure and is a common cause of refractory hypertension (hypertension that remains uncontrolled despite
multiple medications). However, moderate alcohol intake, as in this patient, does not significantly affect blood
pressure.

(Choice D) Sodium restriction can lower blood pressure when implemented as part of the DASH diet (low in
sodium and saturated fat; high in potassium, calcium, and dietary fiber). Sodium restriction alone is somewhat less
effective and inconsistent, with the greatest effect in individuals age >65, the African American population, and
patients with chronic kidney disease.

(Choice E) Smoking cessation is an important component of lifestyle modification that reduces the risk of future
cardiovascular events and all-cause mortality. However, many studies indicate that chronic smoking is not clearly
related to an increased incidence of hypertension. Weight loss would be more effective in controlling this patient's
blood pressure.

Educational objective:
Obesity is the single greatest contributing lifestyle risk factor for hypertension; weight loss and the Dietary
Approaches to Stop Hypertension (DASH) diet are the most effective nonpharmacologic interventions for lowering
blood pressure.

Reference
• Non-pharmacological aspects of blood pressure management: what are the data?

• Long-term effects of weight-reducing diets in people with hypertension.


Question #67

A 44-year-old man comes to the office for follow-up after a recent hospitalization. The patient went to the
emergency department with palpitations 2 weeks ago and was found to have atrial fibrillation with rapid ventricular
response. He was admitted to the hospital, where he spontaneously converted to normal sinus rhythm overnight
and was discharged home the next day. Prior to this episode, the patient had gone on an alcohol drinking binge
during a friend's bachelor party. He otherwise rarely drinks alcohol. Medical history is unremarkable. He is a
lifetime nonsmoker. Blood pressure is 124/70 mm Hg and pulse is 78/min and regular. Estimated jugular venous
pressure is normal. Examination shows no abnormalities. Review of laboratory results from the hospital admission
shows normal creatinine level, liver function tests, thyroid studies, and lipid panel. Echocardiogram shows normal
left and right ventricular function and no valvular abnormalities. Which of the following is the best next step in
managing this patient?

A) Amiodarone

B) Aspirin and clopidogrel

C) No additional therapy

D) Rivaroxaban

E) Warfarin
Explanation
Correct Answer:

C) No additional therapy

CHA2DS2-VASc score for thromboembolic risk


in nonvalvular atrial fibrillation

Risk criteria Points

C Congestive heart failure 1

H Hypertension 1

A2 Age ≥75* 2

D Diabetes mellitus 1

S2 Stroke or TIA 2
V Vascular disease (eg, PAD, prior MI) 1

A Age 65-74* 1

Sc Sex category female 1

Maximum score 9

Total score
Generalized stroke risk Antithrombotic therapy
Male Female**

0 0 Low None

1 2 Moderate None or oral anticoagulant

≥2 ≥3 High Oral anticoagulant

*Patients are assigned to 1 of the 2 age categories.


**Different cutoffs are used for males & females. Female sex is considered a risk modifier that adds to the
CHA2DS2-VASc score only if other nonsex risk factors are present (female patients cannot have a total score of 1).

MI = myocardial infarction; PAD = peripheral artery disease; TIA = transient ischemic attack.

This patient had an episode of atrial fibrillation (AF) that spontaneously resolved within 24 hours. The
pathophysiology of AF involves aging and comorbidities (eg, hypertension, heart failure) that cause atrial
remodeling and create an underlying substrate for AF to develop. The substrate creates a lower threshold for
triggers (eg, binge alcohol intake, hyperthyroidism) to initiate and sustain AF; however, sometimes triggers can
cause AF in the absence of underlying substrate. After a first episode of AF, patients with risk factors for underlying
substrate typically develop recurrent (ie, paroxysmal) or permanent AF.

In contrast, patients without risk factors for underlying substrate (eg, age <50, no significant medical history)
are relatively unlikely to experience recurrent AF, especially when the trigger of the episode is removed (eg,
cessation of binge alcohol intake, treatment of hyperthyroidism). This is at least part of the reason these patients
have low thromboembolic risk associated with AF. They often have a CHA2DS2-Vasc score of 0, as in this
patient, for which chronic anticoagulation is not indicated. Men with a CHA2DS2-Vasc score of 1 (or women with a
score of 2) also have relatively low thromboembolic risk that may not indicate chronic anticoagulation. In men with
a CHA2DS2-Vasc score ≥2 (or women with a score ≥3), chronic anticoagulation is indicated and is accomplished via
the administration of a direct oral anticoagulant (eg, rivaroxaban, apixaban, dabigatran) or, less commonly, warfarin
(Choices D and E).

(Choice A) Amiodarone is used occasionally for maintenance of sinus rhythm in patients with recurrent,
symptomatic episodes of AF (ie, rhythm-control strategy). This patient has spontaneously converted to sinus
rhythm, and amiodarone is not indicated. First-line rate control agents (eg, metoprolol) are commonly used but are
not needed in patients with relatively low likelihood of recurrent AF.

(Choice B) Dual antiplatelet therapy with aspirin and clopidogrel has shown some benefit in lowering the
thromboembolic risk in patients with AF. However, the therapy is rarely used as anticoagulant agents (eg,
rivaroxaban, warfarin) provide greater benefit with roughly the same level of increase in bleeding risk.
Educational objective:
Patients without underlying risk factors who develop atrial fibrillation typically have low associated thromboembolic
risk. Those with a CHA2DS2-Vasc score of 0 have low thromboembolic risk, and chronic anticoagulation (eg,
rivaroxaban) is not indicated.

Reference
• 2014 AHA/ACC/HRS Guidelines for the management of patients with atrial fibrillation: a report of the
American College of Cardiology/American Heart Association Task Force on practice guidelines and the
Heart Rhythm Society.
Question #68

A 48-year-old woman comes to the office due to a lesion on her neck. The lesion has been present for the past 6
months and is slowly enlarging. Skin findings are shown in the image below.

Which of the following is the most likely diagnosis?


A) Basal cell carcinoma

B) Keratoacanthoma

C) Pyoderma gangrenosum

D) Pyogenic granuloma

E) Squamous cell carcinoma


Explanation
Correct Answer:

A) Basal cell carcinoma

Basal cell carcinoma

• Sun/ultraviolet light
Risk
• Fair skin
factors
• Ionizing radiation

• Skin-colored, pearly nodule ± rolled borders


Clinical
• Telangiectatic vessels
features
• ± Central ulceration, local invasion

Diagnosis • Shave, punch, or excisional biopsy

• First-line:
◦ Surgical excision with 4-mm margins
◦ Mohs micrographic surgery (face/high-risk tumors)
Treatment
• Second-line:
◦ Topical fluorouracil, topical imiquimod, C&E (low-risk
tumors only)
C&E = curettage & electrodesiccation.

This patient has a slow-growing, ulcerated, pearly nodule with a rolled border on sun-exposed skin, typical features
of basal cell carcinoma (BCC). BCC accounts for approximately 75% of all skin cancers in the United States; risk
factors include fair skin and history of heavy sun exposure. Alternate presentations include:

• Chronic ulcer that bleeds, oozes, or crusts


• Reddish patch or irritated area (superficial BCC)
• Pearly or translucent nodule that is pink, red, white, or skin toned (nodular BCC)
• Pale, scarlike area with poorly defined borders

BCC only rarely metastasizes but can invade nearby tissues (eg, nerves, bone). Nodular BCC on the trunk or
extremities is typically managed with standard surgical excision with 3- to 5-mm (narrow) margins. However, for the
face and other delicate or cosmetically sensitive areas, Mohs micrographic surgery (sequential removal of thin skin
layers with microscopic inspection to confirm that the margins are clear of malignant tissue) is more often
performed.

(Choice B) Keratoacanthoma is a rapidly growing, volcano-like nodule with a central keratotic plug. Although
these lesions may regress spontaneously, many are excised, especially if the lesion is near an important structure
(eg, eye).

(Choice C) Pyoderma gangrenosum presents as a painful, rapidly expanding ulcer with purple/dusky margins. It
typically occurs on the trunk or a lower extremity, most commonly in patients with systemic disease (eg, rheumatoid
arthritis, inflammatory bowel disease).

(Choice D) Pyogenic granuloma is a vascular tumor that presents as a red, beefy, friable nodule that grows rapidly
over weeks or months. The lesions can bleed with minor trauma but are not typically ulcerated.

(Choice E) Squamous cell carcinoma is much less common and grows faster than BCC. It often arises from a
precursor lesion, such as an actinic keratosis, and typically has overlying hyperkeratosis.
Educational objective:
Basal cell carcinoma is the most common skin malignancy in the United States. It typically presents as a slow-
growing nodule with pearly, rolled borders. Although it only rarely metastasizes, it may invade local tissues and
should be removed.

Reference
• Basal cell carcinoma.
Question #69

A 63-year-old woman comes to the office due to a rash on her face. She has had pustules and pink bumps in this
area for years; the rash appears to wax and wane but never fully resolves. The patient has also had a tingling
sensation in the involved areas following sun exposure, but there are no other associated symptoms. Skin
examination findings are shown in the exhibit. Which of the following is the best next step in management of this
patient's skin condition?

A) Topical clotrimazole

B) Topical corticosteroid

C) Topical metronidazole

D) Topical permethrin

E) Topical salicylic acid


Explanation
Correct Answer:

C) Topical metronidazole

Treatment of rosacea

• Avoidance of sun exposure, hot/spicy foods, alcohol


General measures
• Gentle cleansers & emollients

Erythematotelangiectatic rosacea • Topical brimonidine


(flushing, erythema, telangiectasia) • Laser/intense pulsed light therapy

• First line: topical metronidazole, azelaic acid,


Papulopustular rosacea
ivermectin
(small papules & pustules)
• Second line: oral tetracyclines

Phymatous rosacea • Oral isotretinoin


(irregular thickening of skin) • Laser therapy/surgery

Ocular rosacea • Lid scrubs & ocular lubricants


(burning/foreign body sensations, blepharitis, • Topical or systemic antibiotics (eg, metronidazole,
keratitis, conjunctivitis, corneal ulcers) macrolides)

Rosacea is a chronic erythematous rash that is most prominent on the convex areas of the face. The etiology is
unknown, but the rash may be due to a chronic inflammatory reaction to cutaneous microorganisms, ultraviolet light
damage, or vasomotor dysfunction. Symptoms (eg, flushing, skin sensitivity) are often precipitated by hot or spicy
foods, alcohol, sun exposure, or high ambient temperatures.

The initial management of rosacea depends on the general phenotype. Mild facial flushing and sensitivity can often
be managed with simple behavioral measures (eg, avoiding sun exposure). However, this patient has
papulopustular rosacea, which is characterized by overgrowth of the sebaceous glands manifesting as small
papules and pustules on the central face. Patients with these features typically require pharmacologic treatment.
First-line options include topical metronidazole, azelaic acid, or ivermectin. Oral tetracyclines (eg, doxycycline,
minocycline) are used for severe or refractory disease.

(Choice A) Topical clotrimazole is appropriate for the treatment of uncomplicated dermatophyte infections.
Dermatophyte infection of the face (ie, tinea facialis) is uncommon and typically presents as an annular lesion with
pruritus and a scaly border.

(Choice B) Topical corticosteroids (eg, triamcinolone) are not recommended as routine therapy for rosacea
because they can cause a rebound worsening of symptoms.

(Choice D) Permethrin is most commonly used to treat head lice and scabies. Although rosacea has been linked
to colonization by Demodex mites, the effectiveness of permethrin in rosacea is uncertain.

(Choice E) Topical salicylic acid is used for the management of acne vulgaris but is not the first-line treatment for
rosacea.

Educational objective:
Mild facial flushing and sensitivity due to rosacea can often be managed with simple behavioral measures (eg,
avoidance of sun exposure, hot/spicy foods, or alcohol). However, patients with papulopustular rosacea (ie,
papules and pustules on the central face) typically require pharmacologic treatment. First-line options include
topical metronidazole, azelaic acid, and ivermectin.

Reference
• Rosacea: diagnosis and treatment.

• Swiss S1 guideline for the treatment of rosacea.


Question #70

A 30-year-old man comes to the office due to a rash. For the last 3 weeks, he has had a slowly enlarging, pruritic
rash at the left groin. The patient had a similar rash 2 months ago that resolved after topical treatment with
miconazole; he has since been careful to dry his skin thoroughly after bathing. Medical history is unremarkable.
The patient has 1 long-term sexual partner and does not use tobacco, alcohol, or illicit drugs. Vital signs are
normal. BMI is 24 kg/m2. Examination shows an annular plaque with a scaly border in the left groin; scrotal skin is
normal. Potassium hydroxide microscopy of skin scrapings from the lesion reveals segmented hyphae. Which of
the following is the most appropriate next step in management of this patient?

A) Examine interdigital skin of toes

B) Obtain fungal blood cultures

C) Order serum immunoglobulin level

D) Prescribe hydrocortisone cream

E) Prescribe topical selenium sulfide


Explanation
Correct Answer:

A) Examine interdigital skin of toes

Risk factors for dermatophyte infection

• Warm, humid environments


Environmental exposures • Direct contact with infected person, fomites, or public showers
• Contact with animals (eg, kittens)

• Concurrent dermatophyte infection (autoinoculation)


• Occlusive clothing
Patient factors
• Obesity
• Peripheral artery disease

• Diabetes mellitus
Immune deficiency • HIV infection
• Systemic glucocorticoid therapy

This patient has a scaly, pruritic rash at the groin with sparing of the scrotum. Potassium hydroxide microscopy
showing fungal elements confirms the diagnosis of tinea cruris (TC). TC, often called jock itch, is a common
dermatophyte infection caused by Trichophyton rubrum and similar species.

Recurrent TC is common and suggests reexposure to an external source or autoinfection from a concurrent
dermatophyte infection elsewhere on the body (eg, tinea pedis, tinea corporis, onychomycosis). Therefore, the
patient should have a thorough skin inspection and have any other sites of infection treated; untreated foci of
dermatophyte infection can lead to continuing reinfection and potential secondary bacterial infection (eg, gram-
positive cellulitis).

Other measures for preventing recurrence include thorough drying of skin after bathing, use of absorbent natural-
fiber (eg, cotton) underwear, and avoidance of tight-fitting or occlusive clothing. Topical antifungals (eg, miconazole,
tolnaftate) are usually effective for eradication, and localized disease recurrence can be treated again with topical
agents.

(Choice B) Fungal blood cultures are most commonly used for evaluation of sepsis or fever of unknown origin in
patients with immune suppression, indwelling vascular access devices, or large-area burns. Tinea infections are
generally superficial, and fungemia and other invasive complications are rare.

(Choice C) Severe or widespread dermatophyte infection can be seen in patients with impaired immunity,
especially those with HIV infection or on systemic glucocorticoid therapy. Patients with immunoglobulin deficiencies
(eg, selective IgA deficiency, common variable immunodeficiency) typically develop recurrent sinopulmonary and
gastrointestinal bacterial infections rather than dermatophyte infection. Recurrence of uncomplicated TC is
common in healthy patients and does not suggest an underlying immune disorder.

(Choice D) Topical corticosteroids (eg, hydrocortisone) are not recommended in TC or other superficial
dermatophyte infections because they are associated with worsening of the infection.

(Choice E) Selenium sulfide is indicated for the treatment of tinea (pityriasis) versicolor (eg, hypo/hyperpigmented
macules on trunk and upper extremities) and seborrheic dermatitis of the scalp; these conditions are not
dermatophyte infections. Selenium sulfide is not recommended for TC.

Educational objective:
Recurrent tinea cruris is common and suggests reexposure to an external source or autoinfection from a concurrent
dermatophyte infection elsewhere on the body (eg, tinea pedis, tinea corporis, onychomycosis). Patients should
have a thorough skin inspection, and any other sites of infection should be treated.
Question #71

A 19-year-old college student comes to the office due to painful lesions on his right hand for the past several days.
He has also felt feverish and tired. The patient has a history of folliculitis and skin abscess requiring surgical
drainage. Over the past 6 months, he has had 3 sexual partners and uses condoms inconsistently. He began a
part-time landscaping job a month ago. Temperature is 37.4 C (99.3 F), blood pressure is 130/84 mm Hg, and
pulse is 86/min. Right hand examination findings are shown below.
He has no other skin rash and the remainder of the physical examination is normal. Which of the following is the
most likely causative organism of this patient's condition?

A) β-hemolytic streptococci

B) Coxsackievirus

C) Herpes simplex virus

D) Neisseria gonorrhoeae

E) Pseudomonas aeruginosa

F) Rickettsia rickettsii

G) Staphylococcus aureus

H) Treponema pallidum

I) Trichophyton rubrum
Explanation
Correct Answer:

C) Herpes simplex virus

This young patient with multiple sexual partners has mild systemic symptoms and a painful, vesicular hand rash
suggesting herpetic whitlow.

Herpetic whitlow arises when herpes simplex virus (HSV) inoculates a cutaneous defect of the hand. In adults,
infections typically occur after hand contact with an active genital herpes lesion (HSV-2). Risk is also increased in
health care workers who do not use adequate hand protection and are exposed to infected orotracheal secretions
(HSV-1).

Symptoms usually include a mild prodrome (fever, malaise), followed by the development of a focal area of
grouped vesicles on an erythematous base. Tingling, burning, and pain are common, and some patients may
have epitrochlear or axillary lymphadenopathy. Most lesions resolve spontaneously within 2-3 weeks, but patients
with immunocompromise may require antiviral medication (eg, acyclovir). Recurrent lesions occur in 20%-50% of
cases.

(Choices A and G) β-hemolytic streptococci and Staphylococcus aureus are common causes of erysipelas and
cellulitis. Manifestations typically include skin erythema, edema, and warmth; vesicular lesions would be unusual.

(Choice B) Coxsackievirus causes hand-foot-and-mouth disease and usually manifests with oral lesions and a
bilateral macular, maculopapular, or vesicular rash of the hands and feet. This patient with a localized, unilateral
rash on a small section of the hand is unlikely to have coxsackievirus.

(Choice D) Disseminated gonococcus may cause tenosynovitis, polyarthralgia, and cutaneous lesions on the distal
extremities that begin as macules and progress to papules, pustules, vesicles, or bullae. Lesions are usually
painless (not painful) and multiple (not single).

(Choice E) Pseudomonas aeruginosa may cause folliculitis after hot tub exposure or cellulitis after a nail puncture
through a shoe. Neither is likely to result in a small area of vesicular eruption.

(Choice F) Rocky Mountain spotted fever (RMSF) is a tick-borne rickettsial illness that usually manifests with
nonspecific signs (fever, headache, malaise, myalgias) and a diffuse macular rash that turns petechial. This patient
with a focal vesicular rash is unlikely to have RMSF.

(Choice H) Secondary syphilis causes a diffuse, symmetric macular/papular rash that includes the palms and
soles. A focal, unilateral, vesicular lesion is unlikely.

(Choice I) Trichophyton rubrum is a dermatophyte that causes ringworm, tinea pedis, and tinea manuum. Rashes
are typically pruritic and scaly (not vesicular).

Educational objective:
Herpetic whitlow is a viral infection of the hand caused by herpes simplex virus. Most adult infections are acquired
from contact with genital herpetic lesions or infected orotracheal secretions. Spontaneous resolution is the norm
but recurrences are common.
Question #72

A 28-year-old woman comes to the office due to painful nodular lesions under her arms for the past month. She
has also noticed a foul odor. The patient had similar lesions 4 months ago. Other medical conditions include
recurrent urinary tract infections. She takes postcoital nitrofurantoin prophylaxis and uses combined oral
contraceptives for birth control. The patient smokes a pack of cigarettes daily. Temperature is 36.1 C (97 F), blood
pressure is 111/77 mm Hg, and pulse is 78/min. Skin examination shows tender, fluctuant nodules with thick bands
of scar tissue at the axillae bilaterally, as shown in the exhibit. Which of the following is the greatest risk factor for
this patient's skin condition?

A) Combined oral contraceptive use

B) Deodorant application

C) Nitrofurantoin therapy

D) Poor body hygiene

E) Tobacco use
Explanation
Correct Answer:

E) Tobacco use

Hidradenitis suppurativa

• Disordered folliculopilosebaceous units:


◦ Ductal keratinocyte proliferation → follicular occlusion → follicular rupture →
Pathogenesis
inflammation
• Risk factors: smoking, metabolic syndrome (obesity, diabetes mellitus)

• Chronic & recurrent lesions in intertriginous areas


Clinical • Mild: painful nodules, draining abscesses
presentation • Moderate: sinus tracts & scarring
• Severe: extensive sinus tracts, widespread disease

• Mild: topical clindamycin


Treatment • Moderate: oral tetracycline
• Severe: tumor necrosis factor-α inhibitors (eg, adalimumab), surgical excision

• Depression & suicide


Complications
• Squamous cell carcinoma of skin
This patient with recurrent, painful nodules and thick, scarred plaques in the axillae has hidradenitis suppurativa
(HS). The pathogenesis of HS begins with the occlusion of folliculopilosebaceous units caused by keratinocyte
hyperproliferation and abnormal keratinocyte differentiation. Mechanical stress (eg, friction in intertriginous areas)
initiates an inflammatory response that can lead to follicular rupture and the release of additional proinflammatory
factors. Exacerbated by immune dysregulation and subsequent bacterial colonization or infection, the initial
inflammation often evolves into a chronic inflammatory response with a characteristic persistent/relapsing course.

A strong association exists between HS and tobacco use. Nicotine and other tobacco components are thought to
stimulate follicular occlusion through their effect on keratinocytes. In addition, these substances have
proinflammatory effects on neutrophils and T lymphocytes and promote the production of tumor necrosis factor-
alpha by keratinocytes. Smoking cessation is typically advised as part of management of HS and improvement of
the overall health of the patient. In addition to medical therapy (eg, antibiotics), surgery may be needed for
refractory disease and management of sinus tracts and scars.

(Choice A) Androgens are thought to stimulate HS development. Combined oral contraceptives containing
estrogen have antiandrogenic effects and are used as adjunctive therapy with antibiotics in HS management.

(Choice B) Fragrances and other chemicals in deodorants and other personal care products can cause allergic
contact dermatitis, which manifests acutely as pruritic, erythematous plaques and vesicles or chronically as scaling
and lichenification. Painful, fluctuant nodules with scars are not characteristic.

(Choice C) Although HS is not directly due to infection of the follicle, colonization by a broad range of pathogens
may exacerbate the inflammatory response. Long-term antibiotic therapy (eg, oral doxycycline, topical clindamycin)
is used to reduce the bacterial load and may have additional anti-inflammatory effects. However, nitrofurantoin is
rapidly cleared and concentrated in the urine and is unlikely to influence the development of HS.

(Choice D) Follicular injury due to excessive friction and mechanical pressure promotes the development of HS
and explains its propensity to affect intertriginous areas. However, poor body hygiene does not cause HS.

Educational objective:
Hidradenitis suppurativa (HS) is a chronic, relapsing condition characterized by inflammatory occlusion of
folliculopilosebaceous units. There is a strong association between tobacco use and HS.
Reference
• Hidradenitis suppurativa: rapid evidence review.
Question #73

A 17-year-old boy comes to the office due to a rash. He has had a progressive papular rash involving the trunk,
neck, upper arms, and genital area for 8 weeks. The lesions are mildly pruritic, but there are no other associated
symptoms. Past medical history is notable for mild intermittent asthma and childhood eczema. The patient also
was treated empirically a year ago for chlamydia after exposure to an infected sexual partner. He is up to date with
recommended vaccinations. Vital signs are normal. Examination shows normal growth and body habitus, with
appropriate sexual development for age. Dermatologic examination shows widespread firm, dome-shaped, flesh-
colored papules with central umbilication. This patient's rash is most commonly associated with which of the
following conditions?

A) Cellular immunodeficiency

B) Circulating autoantibodies

C) Complement deficiency

D) Impaired phagocytosis

E) Selective IgA deficiency


Explanation
Correct Answer:

A) Cellular immunodeficiency

Molluscum contagiosum (MC) is a skin infection caused by a poxvirus. It is characterized by small, pruritic, skin-
colored papules with umbilicated centers. Children are most commonly affected, but adolescents and adults also
can develop MC. Transmission occurs through skin-to-skin contact or via contaminated fomites, with subsequent
autoinoculation to additional sites. In children, lesions commonly appear on the extremities, face, or trunk; in adults,
sexual transmission may lead to lesions on the lower trunk and anogenital region. The diagnosis is primarily clinical
and based on the typical appearance of the lesions. MC is self-limited (resolution within 6-12 months), but
treatment with curettage, cryotherapy, or topical agents (eg, podophyllotoxin) may be considered to prevent further
spread, reduce symptoms, or improve cosmesis.

Patients with impaired cellular immunity (eg, HIV disease) may have a prolonged course with widely distributed
papules, facial involvement, and lesion counts numbering in the hundreds. HIV testing should be considered for
patients with MC, especially if they have large (>10 mm), numerous, or widespread lesions.

(Choice B) Hypersensitivity rashes caused by circulating autoantibodies may be of type II (antibody-dependent


cellular cytotoxicity) or type III (immune complex deposition) variety. Type II rashes are more likely to manifest as
blisters or bullae (eg, pemphigus vulgaris, bullous pemphigoid) than papules. Type III rashes tend to be more
erythematous and maculopapular.

(Choice C) The complement system is involved in defense against pathogenic bacteria. For example, C3
deficiency predisposes to pyogenic bacterial respiratory tract and sinus infections, and C5-C8 deficiencies
predispose to recurrent Neisseria infections. Complement deficiency does not increase the risk of MC or other viral
infections.

(Choice D) Disorders of phagocytosis (eg, chronic granulomatous disease, Chédiak-Higashi disease, Job
syndrome, defective leukocyte adhesion proteins) present with severe pyogenic bacterial infections. Innate
immunity against viruses is less affected.

(Choice E) Selective IgA deficiency can predispose to recurrent respiratory infections and chronic giardiasis. It will
not increase the risk of MC.

Educational objective:
Molluscum contagiosum is characterized by firm, flesh-colored, dome-shaped, umbilicated papules. Patients with
impaired cellular immunity (eg, HIV disease) are at risk for more severe, widespread disease.

Reference
• Molluscum contagiosum virus infection.
Question #74

An 89-year-old woman comes to the physician complaining of an episodic skin discoloration over the last several
months. She reports scattered dark purple areas on her hands and forearms without associated pain or itching.
She has no fever, chills, or recent weight loss. The patient's other medical problems include diet-controlled diabetes
and right knee osteoarthritis treated with acetaminophen. Physical examination shows thin, hyperpigmented skin
with several flat, dark purple ecchymotic areas over the dorsum of both forearms. Her abdomen is soft and
nontender. The liver span is 6 cm without a palpable spleen. Laboratory results are as follows:

Hematocrit 42%
Leukocytes 5,800/mm3
Platelets 220,000/mm3
Creatinine 0.6 mg/dL

350 mg/dL (normal 200-400 mg/


Fibrinogen
dL)

Prothrombin time 11 sec


International Normalized Ratio 1.0
Partial thromboplastin time 25 sec

Which of the following is the most likely cause of this patient's complaint?

A) Bone marrow failure

B) Lupus anticoagulant
C) Perivascular connective tissue atrophy

D) Poor platelet adhesion

E) Vitamin K deficiency
Explanation
Correct Answer:

C) Perivascular connective tissue atrophy

This patient's skin fragility, ecchymosis, and normal laboratory studies (blood counts and coagulation studies) are
consistent with senile purpura.

Senile purpura (solar or actinic purpura) is a noninflammatory disorder that is most common in the elderly but can
also be seen in middle-aged patients with extensive sunlight exposure. It is caused by loss of elastic fibers in
perivascular connective tissue. Minor abrasions that would merely stretch the skin in younger patients can rupture
superficial blood vessels in the elderly. The subsequent extravasation of blood leads to ecchymosis over
vulnerable areas, such as the dorsum of the hands and forearms. Patients can have residual brownish
discoloration from hemosiderin deposition. The incidence and severity are also increased in patients taking
anticoagulants, corticosteroids, or nonsteroidal anti-inflammatory drugs.

Senile purpura is usually not a dangerous condition and requires no further investigation. However, many older
patients with severe skin fragility may require careful wound care measures following even minor lacerations.

(Choice A) Bone marrow failure may present with ecchymosis by causing thrombocytopenia. Patients with bone
marrow failure may also have leukopenia and anemia. This patient's normal hematologic profile makes this unlikely.

(Choice B) The lupus anticoagulant is an IgM or IgG immunoglobulin that prolongs the activated partial
thromboplastin time (aPTT) by binding to phospholipids used in the assay. The prolonged aPTT is a laboratory
artifact; the lupus anticoagulant is not a physiologic anticoagulant but actually increases the risk of thrombosis.

(Choice D) Poor platelet adhesion can occur in a variety of settings, including anti-platelet therapy (eg, aspirin,
clopidogrel, eptifibatide), von Willebrand disease, or chronic kidney disease. This patient does not take any anti-
platelet medications. People with von Willebrand disease typically present at a much younger age and have a
family history of bleeding disorders. In addition, this patient's normal creatinine makes renal failure unlikely.
(Choice E) Vitamin K deficiency depletes vitamin K-dependent clotting factors (factors II, VII, IX, and X), leading to
an increase in prothrombin time (and aPTT to a lesser extent). This patient's coagulation studies are normal.

Educational objective:
Senile purpura usually presents with ecchymoses in elderly patients in areas exposed to repeated minor trauma
(eg, extensor surfaces of the hands and forearms). It is due to age-related loss of elastic fibers in perivascular
connective tissue. Senile purpura is not dangerous and requires no further evaluation.

Reference
• Management of the oral manifestations of senile purpura in an edentulous patient: a case report.
Question #75

A 33-year-old man comes to the office due to a rash. For the last several months, an increasing number of small
lesions have appeared on his hands. The spots are associated with intermittent, mild itching but are otherwise
asymptomatic. He uses electronic cigarettes regularly and drinks 3-5 alcoholic beverages on weekends. Vital signs
are within normal limits. BMI is 32 kg/m2. Skin examination findings are shown in the exhibit. Similar lesions are
visible in other locations, including the scalp. Which of the following is the most likely diagnosis?

A) Irritant contact dermatitis

B) Lichen planus

C) Molluscum contagiosum

D) Psoriasis

E) Scabies
Explanation
Correct Answer:

D) Psoriasis

Plaque psoriasis

• Well-defined, erythematous plaques with silvery scale


Skin lesions
• Extensor surfaces (knees, elbows), hands, scalp, back, nail plates

• Nail pitting
Extradermal
• Conjunctivitis, uveitis
manifestations
• Psoriatic arthritis

• Topical: high-potency glucocorticoids, vitamin D analogs, tar, retinoids, calcineurin


inhibitors, tazarotene
Treatment
• Ultraviolet light/phototherapy
• Systemic: methotrexate, calcineurin inhibitors, retinoids, apremilast, biologic agents

This patient has a chronic rash characterized by scaly, erythematous plaques consistent with psoriasis.
Psoriasis is classically found on the extensor surfaces of the knees and elbows (large plaque) but can also occur on
the scalp, hands, and trunk (eg, gluteal cleft) as small plaques. When psoriasis occurs on the hands, it is typically
found on the dorsal surface and likely caused by frequent minor trauma (Köbner phenomenon). Other possible
manifestations in the hands include nail changes (eg, pitting, subungual debris) and arthritic changes.
(Choice A) Irritant contact dermatitis, which commonly affects the hands, presents with erythema, scaling, and
fissuring. However, it does not form well-defined plaques and is unlikely to affect the scalp.

(Choice B) Lichen planus presents with pink or purple plaques and papules. It is usually associated with severe
pruritis and typically affects the volar surface of the wrists rather than the dorsum of the hands. Concurrent scalp
lesions would be unusual.

(Choice C) Molluscum contagiosum presents with small, skin-colored papules. The lesions are typically
umbilicated and smooth rather than scaly. Children are affected more commonly than adults.

(Choice E) Scabies commonly affects the hands but presents with intensely pruritic papules and burrows on the
interdigital spaces, palms, and flexor surface of the wrists.

Educational objective:
Psoriasis is characterized by erythematous, scaly plaques. It is classically found on the extensor surfaces of the
knees and elbows (large plaque) but can also occur on the scalp, hands, and trunk (eg, gluteal cleft) as small
plaques. When psoriasis occurs on the hands, it is typically found on the dorsal surface, likely due to frequent
minor trauma.
Question #76

A 60-year-old man comes to the office due to a sore on his lip. Four weeks ago, he noticed a painful lesion on his
lower lip; in the past week, the lesion became ulcerated. Medical history is unremarkable. The patient does not use
tobacco, alcohol, or illicit drugs. He works as a farmer and lives with his grandchildren. Vital signs are normal.
Examination shows a 6- × 7-mm scaly papule with a central ulcer located in the vermilion zone of the lower lip. The
oral mucosa and tongue are free of lesions. There are no palpable submental or submandibular lymph nodes, and
the remainder of the physical examination is unremarkable. What is the most likely diagnosis?

A) Aphthous stomatitis

B) Basal cell carcinoma

C) Herpes labialis

D) Squamous cell carcinoma

E) Verruca vulgaris
Explanation
Correct Answer:

D) Squamous cell carcinoma

Squamous cell carcinoma of skin

• Sun/ultraviolet, ionizing radiation exposure


Risk factors • Chronic scars/wounds/burn injuries
• Immunosuppression

• Scaly plaques/nodules
Clinical features • ± Ulceration
• SCC in situ: well-demarcated patches/plaques

• Punch, shave, or excisional biopsy


Diagnosis
• Dysplastic/anaplastic keratinocytes

SCC in situ
Invasive SCC
• Excision with 4- to 6-mm margins
Treatment • Excision with 4- to 6-mm margins
• C&E
• Mohs micrographic surgery
• Cryotherapy
• Topical 5-FU, imiquimod
5-FU = 5-fluorouracil; C&E = curettage & electrodesiccation; SCC = squamous cell carcinoma.

This patient has a persistent, ulcerated scaly papule on the lower lip, concerning for squamous cell carcinoma
(SCC). SCC is the most common malignancy of the lip, with 95% of cases occurring in the lower lip vermilion,
likely due to higher sunlight exposure.

SCC can affect any cutaneous surface. In fair-skinned patients, it is most common in sun-exposed areas. In dark-
skinned patients, SCC typically affects non–sun-exposed surfaces and areas with chronic inflammation (eg, venous
stasis ulcers) or chronic scarring (eg, burn scars). In addition, high-risk human papillomavirus infection can cause
genital and periungual SCC.

Well-differentiated SCC typically presents as firm, scaly papules, plaques, or nodules with or without ulceration,
ranging from 0.5-1.5 cm. Poorly differentiated SCC, in contrast, presents with beefy red papules or nodules that
may bleed or ulcerate. Most SCC lesions are asymptomatic, although some are painful or pruritic. Perineural
invasion can cause local paresthesia. Metastatic disease should be suspected when regional lymphadenopathy is
present.

(Choice A) Aphthous ulcers (ie, canker sores) are painful, shallow, round ulcerations with white/yellow bases.
They are recurrent and self-limiting and affect only the oral mucosa, not the keratinized epithelium of the lips.

(Choice B) Basal cell carcinoma classically presents as a shiny, pearly nodule with telangiectasia. Unlike SCC, it
is usually not scaly.

(Choice C) Herpes labialis typically presents at the vermilion border as painful vesicles that ulcerate, becoming
crusted over the next week. Lesions typically resolve within 2 weeks, not lasting a month as in this patient.

(Choice E) Verruca vulgaris (ie, common wart) is characterized as a firm papule that is hyperkeratotic and with
clefted surface. It does not ulcerate.

Educational objective:
Cutaneous squamous cell carcinoma (SCC), the most common malignancy of the lower lip, can affect any
cutaneous surface. Well-differentiated SCC presents as firm, scaly papules, plaques, or nodules with or without
ulceration. Poorly differentiated SCC presents with beefy red papules or nodules that may bleed or ulcerate.

Reference
• Basal cell and cutaneous squamous cell carcinomas: diagnosis and treatment
Question #77

A 16-year-old girl is brought to the office due to hair loss. Over the last few months, she has developed a few
patches of hair loss on the scalp. The lesions are associated with mild itching just before the loss of hair but are
otherwise asymptomatic. Medical history is unremarkable; the patient's only current medication is an oral
contraceptive. She does not use tobacco, alcohol, or illicit drugs. Vital signs are normal. Scalp examination
findings are shown in the exhibit. Which of the following is the most appropriate next step for this patient's hair
loss?

A) Cognitive behavioral therapy

B) Discontinue oral contraceptive

C) Intralesional triamcinolone

D) Oral griseofulvin

E) Reassurance and observation

F) Topical minoxidil
Explanation
Correct Answer:

C) Intralesional triamcinolone

Alopecia areata

• Autoimmune attack on hair bulb cells


Pathogenesis
• Genetic predisposition

• Painless, patchy, nonscarring hair loss


Clinical
• Narrowing of hair shafts close to skin surface (exclamation point hairs)
presentation
• Positive hair pull test (>5-6 hairs extracted)

• Mild/moderate hair loss: topical or intralesional corticosteroids


Management • Extensive hair loss: topical immunotherapy (eg, diphenylcyclopropenone), oral
corticosteroids

This patient has patchy, nonscarring hair loss with no scaling. This presentation is most consistent with alopecia
areata (AA), an autoimmune disorder involving the hair bulb cells that is often associated with other autoimmune
conditions (eg, vitiligo, hypothyroidism). The diagnosis of AA is usually based on visual inspection (often with the
assistance of dermatoscopy); close inspection may show narrowing of the hair shafts close to the skin surface
(exclamation point hairs).

Management of AA depends on the size and extent of the lesions. Intralesional corticosteroids (eg,
triamcinolone) are typically used for patients with limited areas of involvement, such as this patient; topical
corticosteroids can also be used for patients who wish to avoid injections (eg, children). Topical immunotherapy
(eg, diphenylcyclopropenone) can be considered for patients with severe, widespread hair loss (eg, alopecia
totalis). AA does not cause permanent damage to the hair follicle, and most patients have at least partial regrowth,
although recurrences are common.

(Choice A) Cognitive behavioral therapy is used in the management of trichotillomania, which causes patchy hair
loss but usually shows signs of irregular regrowth with hair of variable lengths.

(Choices B and E) Telogen effluvium is characterized by slowed hair growth and shedding of hair shafts. It can be
triggered by physiologic stressors (eg, severe illness, iron deficiency), hormonal changes (eg, initiation or
discontinuation of oral contraceptives), or emotional distress. It generally resolves spontaneously and is primarily
managed with reassurance and observation. However, telogen effluvium causes diffuse, not patchy, hair loss.

(Choice D) Griseofulvin is effective in the treatment of tinea capitis, a superficial dermatophyte infection that
presents with scaly, erythematous patches of hair loss, often with prominent itching and tender lymphadenopathy.

(Choice F) Minoxidil is a possible second-line treatment for AA but is more commonly used for female and male
(androgenetic) pattern hair loss. Female pattern hair loss presents with chronic, diffuse thinning of the hair at the
midline and vertex with sparing of the frontal hairline.

Educational objective:
Alopecia areata is characterized by patchy, nonscarring hair loss. It is an autoimmune disorder and is often
associated with other autoimmune conditions. Treatment includes topical or intralesional corticosteroids. A
recurring course is common, but most patients have regrowth over time.

Reference
• Alopecia areata.
Question #78

A 49-year-old man comes to the office due to a pruritic rash. For the last 6 months he has had an enlarging rash at
the groin that causes significant discomfort when he works outdoors in hot weather. The patient has tried over-the-
counter diphenhydramine cream, which only partially relieves the itching. Examination findings are shown in the
exhibit. The scrotum is not involved, and there is no inguinal adenopathy. Which of the following is the most likely
diagnosis of this patient's rash?

A) Candidal intertrigo

B) Erythrasma

C) Inverse psoriasis

D) Seborrheic dermatitis

E) Tinea cruris
Explanation
Correct Answer:

E) Tinea cruris

This patient has a pruritic rash at the groin with sparing of the scrotum. The lesions (annular plaques with a
raised, scaly border) are consistent with tinea cruris (TC) (ie, jock itch), a common dermatophyte infection caused
by Trichophyton rubrum and similar species. Risk of infection is higher in men and in individuals with diabetes or
other immune-suppressing conditions. The symptoms are frequently aggravated by heavy sweating and warm
ambient temperatures.

As with other dermatophyte infections, the diagnosis is often apparent based on historical features and examination
findings. It can be confirmed with potassium hydroxide preparation of skin scrapings, especially in severe or
atypical cases. Topical antifungals (eg, clotrimazole, tolnaftate) are usually effective, and oral antifungals (eg,
fluconazole) can be used for severe or refractory cases; however, nystatin is not effective. Measures recommended
to prevent recurrence include good hygiene with thorough drying (and/or desiccant powders) after bathing.
Concurrent dermatophyte infection elsewhere on the body (eg, tinea pedis, corporis) should be identified and
treated to prevent reinfection via autoinoculation.

(Choice A) Like TC, candidal intertrigo affects the warm, occluded skin inside skin folds. However, the rash
typically appears moist and macerated (rather than dry and scaly); lacks a clear, raised border; and often shows
satellite lesions.

(Choice B) Erythrasma is a superficial bacterial infection (Corynebacterium minutissimum) that presents as red or
brown patches in intertriginous areas. The border is typically flat, scaling (if present) is fine, and itching is mild.

(Choice C) Unlike common plaque psoriasis, which predominantly affects extensor surfaces, inverse psoriasis
affects intertriginous regions. However, this psoriasis variant presents as smooth plaques with minimal to no
scaling.

(Choice D) Although seborrheic dermatitis can involve the groin and axilla, it is much more common at the head
(eg, ears, scalp, eyebrows). The lesions show coarse scaling with a typically greasy appearance, and the plaques
usually have a salmon hue.

Educational objective:
Tinea cruris (ie, jock itch) presents as pruritic, annular plaques with a raised, scaly border. It occurs in the groin but
usually spares the scrotum. The diagnosis is often apparent based on clinical findings and can be confirmed with
potassium hydroxide preparation of skin scrapings. Topical antifungals (eg, clotrimazole, tolnaftate) are usually
effective.
Question #79

A 48-year-old woman with advanced-stage epithelial ovarian cancer undergoes surgery and adjuvant therapy with
paclitaxel and carboplatin. At a follow-up visit 2 weeks after her first cycle of chemotherapy, the patient reports
losing "clumps" of hair in the shower and while brushing her hair that morning. She feels moderately fatigued but
has no other symptoms. Examination shows thinning of the hair across the vertex and crown. The skin of the scalp
shows no erythema or scaling. Which of the following is the most likely explanation for this patient's hair loss?

A) Cell-mediated autoimmune reaction

B) Deposition of IgG and complement on epidermal keratinocytes

C) Increased androgen effect on hair follicle

D) Interruption of follicular keratinocyte proliferation

E) Prolonged telogen phase of hair cycle


Explanation
Correct Answer:

D) Interruption of follicular keratinocyte proliferation


Hair growth occurs in a cyclical pattern with 3 primary phases:

1. Anagen phase: Linear growth of the hair shaft associated with proliferation of hair follicle cells. At any
given time, 80%-90% of follicles are in anagen phase, which lasts up to 5 years.
2. Catagen phase: Transition phase characterized by regression and apoptosis of follicular cells. The hair
shaft may continue to be extruded from the follicle but is no longer growing. This phase lasts up to 2 weeks
and typically accounts for <1% of follicles.
3. Telogen phase: Resting phase of variable duration. The hair shaft is shed during this phase in
preparation for growth of a new hair.

This patient has acute, widespread hair loss following initiation of cytotoxic chemotherapy. Many
chemotherapeutic agents preferentially target rapidly dividing cells, including hair follicle cells in the anagen phase.
Impaired cellular reproduction can lead to cessation of hair growth and rapid shedding of hair shafts (anagen
effluvium). Chemotherapy-induced hair loss is especially common with antimicrotubule drugs (eg, paclitaxel),
alkylators (eg, cyclophosphamide), and antibiotic chemotherapy agents (eg, doxorubicin). The hair loss is usually
temporary and can be mitigated in some cases by scalp-cooling devices that reduce blood flow to the follicles.

(Choice A) Alopecia areata is a cell-mediated autoimmune disorder affecting hair follicles. It presents with patches
of complete hair loss, not diffuse or regional thinning.

(Choice B) Paclitaxel is associated with drug-induced, subacute cutaneous lupus erythematosus, which can cause
hair loss due to deposition of IgG and complement on epidermal keratinocytes. However, the typical lesions are
scaly papules and annular plaques that are distributed widely in sun-exposed areas.

(Choice C) Female pattern hair loss (ie, thinning at midline and vertex) can occur in patients with androgen-
secreting ovarian tumors or a decreased estrogen/androgen ratio due to aromatase inhibitor therapy for breast
cancer. However, this change occurs slowly over months, not days.

(Choice E) Telogen effluvium is characterized by widespread transition of hair follicles into the telogen phase. It
can occur in response to physiologic or emotional stressors (eg, pregnancy, severe illness) but presents with slowly
progressive, diffuse hair thinning.

Educational objective:
Many chemotherapeutic agents preferentially target rapidly dividing cells, including hair follicle cells in the anagen
phase. Impaired cellular reproduction in the anagen phase can lead to temporary, reversible cessation of hair
growth and rapid shedding of hair shafts (anagen effluvium).

Reference
• Anagen effluvium.
Question #80

An 82-year-old man comes to the office for evaluation of a spot on his back. He thinks the skin lesion has been
present for many months to a few years but is unsure. The lesion was asymptomatic until it recently started itching;
it is large and dark, but the patient is not sure whether it is evolving. The spot is shown in the image below:
Which of the following is the most appropriate next step in management of this patient's skin lesion?

A) Cryotherapy destruction of the lesion

B) Excisional biopsy of the lesion

C) Observation over 3-6 months

D) Punch biopsy from the lesion border

E) Punch biopsy from the lesion center


Explanation
Correct Answer:

B) Excisional biopsy of the lesion

Clinical features of melanoma (ABCDE)

• Asymmetry: when bisected, the 2 sides are not identical


• Border irregularities: uneven edges, pigment fading off
• Color variegation: variable mixtures of brown, tan, black & red
• Diameter: ≥6 mm
• Evolving: lesion changing in size, shape, or color; new lesion

The initial clinical assessment of pigmented skin lesions should emphasize identifying those that could be
melanoma and therefore warrant biopsy. Biopsy is appropriate in any of the following situations:

• The lesion has features that suggest melanoma using a validated predictive rule (eg, lesions displaying ≥1-2
of the ABCDE criteria).
• The lesion has inflammatory changes; itching, crusting, or bleeding; or sensory changes (eg,
paresthesia).
• The lesion is significantly different in appearance from other lesions on the same patient (ugly duckling
sign).
• The lesion develops thickening or nodularity, which may indicate vertical growth and increase metastatic
risk.

This patient's lesion is asymmetrical with a somewhat irregular border and notable color variegation (dark brown,
light brown, and pink). The associated itching further raises the risk, as does the apparent nodularity.
To rule out melanoma, full-thickness excisional biopsy with 1- to 3-mm margins of normal tissue is recommended.
Partial excision (eg, deep shave biopsy, central or peripheral punch biopsy) is generally only considered for very
large lesions or lesions in locations where complete excision may be problematic (eg, ear, face); partial excision
may not allow adequate sampling for diagnosis or accurate measurement of the lesion's depth (the most important
prognostic indicator in melanoma) (Choices D and E).

(Choice A) In-situ destruction via cryotherapy or topical fluorouracil is indicated for treatment of actinic keratosis.
Actinic keratosis presents as small, roughened papules in sun-exposed areas (eg, scalp, face, hands) and typically
is not pigmented.

(Choice C) Periodic surveillance is recommended for patients with numerous or atypical nevi that do not have
suspicious features. Suspected melanomas warrant tissue confirmation without delay.

Educational objective:
The assessment of pigmented skin lesions should include a predictive rule to estimate the risk of melanoma, such
as the ABCDE (Asymmetry, Border irregularities, Color variegation, Diameter, Evolving) rule; lesions with ≥1-2 of
the ABCDE criteria warrant excisional biopsy. Biopsy is also recommended for lesions with inflammation; itching,
crusting, or bleeding; or sensory changes and for lesions that are significantly different in appearance from other
pigmented spots on the same patient (ugly duckling sign).
Question #81

A 46-year-old man comes to the office due to discomfort in his mouth. For the last several months, he has had a
mild, but increasingly bothersome, burning sensation on the inside of his cheeks and back of his mouth. The
symptoms are worse when the patient eats hot or spicy foods or speaks for extended periods. Examination findings
are shown in the exhibit. Which of the following is the most likely diagnosis?

A) Aphthous stomatitis

B) Herpes simplex

C) Lichen planus

D) Oral candidiasis
Explanation
Correct Answer:

C) Lichen planus

Lichen planus

• 5 "Ps": pruritic, purple/pink, polygonal, papules & plaques


• Lacy, white network of lines (Wickham striae)
Clinical • Locations:
findings ◦ Skin (eg, ankles, wrists)
◦ Oral mucosa (white papules & plaques ± erythema, mucosal atrophy, ulcers)
◦ Genitalia

Disease • Hepatitis C
associations • Medications: ACE inhibitors, thiazide diuretics

• Chronic symptoms
Natural
• Formation of lesions at sites of trauma (Köbner phenomenon)
history
• Spontaneous resolution within 2 years

• Topical high-potency glucocorticoids (eg, betamethasone)


Treatment
• Widespread lesions: systemic glucocorticoids, phototherapy
This patient has chronic papules involving the oral mucosa consistent with lichen planus (LP). LP is an
autoimmune disorder driven by CD8+ T cells and may occur after exposure to a trigger, such as an infection, a
medication, or a contact allergen. LP occurs in a higher frequency in patients with hepatitis C, although the
relevance of this association is controversial.

Oral manifestations of LP may occur alone or in association with skin or genital lesions. Unlike cutaneous LP (eg,
pruritic purple/pink polygonal papules), oral LP lesions are most commonly characterized by white papules or
plaques connected by white, lacy markings known as Wickham striae; erythematous mucosal atrophy or
ulcerations can also occur. Lesions are typically bilateral and symmetric and can form along the lines of minor
trauma (Köbner phenomenon). Oral LP is often asymptomatic but can be associated with pain or irritation.

The diagnosis of LP is based primarily on examination, but biopsy should be considered for confirmation, especially
in patients with erythematous mucosal atrophy or ulceration. Treatment includes topical high-potency
glucocorticoids (eg, fluocinonide). Unlike cutaneous LP, which is self-limited and often resolves within 2 years,
oral LP often has a prolonged course with relapsing symptoms over many years.

(Choice A) Aphthous stomatitis (canker sores) presents acutely with painful ulcers. Although recurrences are
common, individual episodes usually last only a few days, and the mucosa returns to normal in the interim.

(Choice B) Primary oral herpes simplex infection typically presents as gingivostomatitis in children or pharyngitis in
adults, along with fever and lymphadenopathy. Recurrences present with clusters of ulcerating vesicles on the lips
and perioral skin.

(Choice D) Oral candidiasis (thrush) manifests as loosely adherent, fluffy, white exudates on the oral mucosa. It is
most commonly seen in patients with underlying immune suppression. Wickham striae make LP more likely.

Educational objective:
Oral lichen planus (LP) presents with white papules and plaques on the oral mucosa; variants may also show
erythematous mucosal atrophy or ulceration. It is often associated with hepatitis C. Treatment includes topical
high-potency glucocorticoids. Unlike cutaneous LP, which is self-limited, oral LP often has a prolonged course with
relapses over many years.

Reference
• Oral lichen planus: An update on pathogenesis and treatment.
Question #82

A 35-year-old man comes to the office due to a 2-month history of a skin bump on his lower back. He noticed a
similar bump in the same location 6 months ago, which spontaneously resolved after 3 weeks. The patient recalls
no trauma preceding either occurrence. The bump has grown slightly larger in the past month, but there is no
associated pruritus, pain, or other symptoms. His medical history is unremarkable, and he has no history of
tobacco or alcohol use. Vital signs are within normal limits. Skin examination shows a firm, mobile nodule, as
shown in the image below. There is no drainage from the lesion or tenderness to palpation. The lesion does not
change shape when pinched at the edges. The remainder of the physical examination is within normal limits.
Which of the following is the most likely diagnosis?

A) Basal cell carcinoma


B) Dermatofibroma

C) Epidermal inclusion cyst

D) Lipoma

E) Squamous cell carcinoma


Explanation
Correct Answer:

C) Epidermal inclusion cyst

This patient has an epidermal inclusion cyst (EIC), also known as an epidermal cyst, a discrete benign nodule
lined with squamous epithelium that contains a semisolid core of keratin and lipid. An EIC occurs when the
epidermis becomes lodged in the dermis due to trauma or comedones, or it can arise de novo. EICs can be seen
anywhere on the body, but are most common on the face, neck, scalp, or trunk. The lesion can gradually increase
in size and may intermittently produce a cheesy white discharge. Some patients may develop significant
inflammation with rupture and involvement of surrounding tissue. An EIC will usually resolve spontaneously but can
recur.

Diagnosis is made clinically with examination showing a dome-shaped, firm, and freely movable cyst or nodule with
a central punctum (small, dilated, pore-like opening). Excision is typically performed only for cosmetic reasons.
Incision and drainage are occasionally needed for infected and fluctuant cysts that are painful and erythematous.

(Choices A and E) Basal cell carcinoma is the most common type of skin cancer and usually presents as a pearly
papule or plaque with small telangiectasias on sun-exposed areas. Squamous cell carcinoma is the second most
common skin cancer and also occurs primarily on sun-exposed areas. The lesions are usually firm and scaly
papules, plaques, or nodules. Neither type of skin cancer will show a central punctum, and the skin overlying this
patient's nodule shows no signs of malignancy.

(Choice B) Dermatofibroma is a benign fibroblast proliferation that typically appears as a firm, hyperpigmented
nodule, most often on the lower extremities. Dermatofibromas have a fibrous component that causes dimpling in
the center when the area is pinched ("dimple" or "buttonhole" sign).

(Choice D) Lipomas are benign, painless subcutaneous masses with normal overlying epidermis. In contrast to
epidermal cysts, lipomas are usually soft to rubbery and irregular, and do not typically regress and recur.

Educational objective:
Epidermal inclusion cyst is a benign nodule containing squamous epithelium that produces keratin. It presents as a
dome-shaped, firm, freely movable cyst or nodule with a small central punctum. The lesion can remain stable or
gradually increase in size but may produce a cheesy white discharge; it usually resolves spontaneously.

Reference
• Epidermal inclusion cyst embedded in a recurrent benign fibrous histiocytoma.
Question #83

A 50-year-old man comes to the office for evaluation of an itchy rash on his lower abdomen. The rash has been
present for many months, and the patient is unable to obtain relief with moisturizers. He has no chronic medical
conditions and takes no medications. On examination, the affected skin is noted to be thickened and dry; the rash
is shown in the exhibit. The remainder of the examination reveals no abnormalities. What is the most likely cause
of the patient's symptoms?

A) Chronic plaque psoriasis

B) Exposure to an allergenic metal

C) Infection with Trichophyton rubrum

D) Infestation with mites

E) Xerosis
Explanation
Correct Answer:

B) Exposure to an allergenic metal

Allergic contact dermatitis

• Type IV hypersensitivity reaction


Pathophysiology • Common triggers: poison ivy/oak, nickel, dyes, topical
medications, skin care products

• Acute: pruritic red, indurated plaques with vesicles/bullae


Clinical
presentation
• Chronic: lichenification, fissuring

• Clinical findings
Diagnosis
• Patch testing for persistent cases

• Avoidance of suspected allergen


Treatment
• Topical or systemic corticosteroid

This patient has chronic allergic contact dermatitis (ACD) presenting with persistent itching and lichenification in
the affected skin. ACD represents a type IV (cell-mediated) hypersensitivity reaction, with initial sensitization
occurring within days of contact with the causative substance. On reexposure, the antigen is presented to
sensitized lymphocytes, leading to inflammatory skin changes.

Nickel (eg, belt buckles, watches, jewelry) is a common trigger for chronic ACD; corrosion of metal alloys by
electrolytes in sweat releases soluble metal ions that trigger the hypersensitivity reaction. Nickel-induced ACD
typically causes symptoms in characteristic locations, especially at the medial beltline (as in this patient), wrists,
and earlobes. Other common allergens that can induce ACD include clothing dyes, topical medications, leather-
and rubber-processing chemicals, and skin care products.

Acute ACD results when the skin is reexposed to high levels of an allergen (eg, poison ivy) and manifests with
intensely pruritic papules and plaques with erythema, vesicles, weeping, and crusting. In contrast, chronic, low-
level exposure, as is typical in nickel allergy, leads to lichenification and pruritus. Most cases of ACD respond to
application of topical corticosteroids and avoidance of the allergen.

(Choice A) Chronic plaque psoriasis presents with pruritic erythematous plaques covered with silvery scales on
multiple areas of the body. It is most common on extensor surfaces (eg, elbows, knees). This patient's lesion does
not have silvery scales and is isolated to the abdomen.

(Choice C) Infection with Trichophyton rubrum causes tinea corporis, which manifests as an expanding, scaly,
annular plaque. This patient's lesion is not annular (ring shaped), and occurrence at the location of clothing
fasteners is more suggestive of ACD.

(Choice D) Although scabies (Sarcoptes scabiei infestation) causes intense pruritus, the rash usually presents
acutely with excoriated red papules and burrows, not a lichenified plaque. The distribution of rash usually involves
>1 area (eg, hands, elbows, waist); rarely, scabies is localized to a single area.

(Choice E) Xerosis, or dry skin, is common in elderly persons and tends to occur when the environment is dry.
Symptoms often occur on the lower extremities. Severe xerosis can evolve into an eczematous plaque. However,
this patient's rash did not improve with moisturization, indicating a different etiology.

Educational objective:
Nickel can cause chronic allergic contact dermatitis (type IV hypersensitivity) that is typically triggered by belt
buckles, jewelry, or watches. Chronic, low-level nickel exposure can lead to itching and lichenification. Treatment
includes topical corticosteroids and allergen avoidance.
Reference
• Diagnosis and management of contact dermatitis.
Question #84

A 43-year-old man comes to the office due to a rash. He has a 1-week history of a blistering rash on his hands
associated with severe pruritus. The patient had a similar rash twice in the previous 2 months that resolved with
peeling of the skin on his hands. He has had no changes in household products and no occupational chemical
exposure. Medical history is unremarkable. Examination shows a vesicular rash on the hands as shown in the
exhibit. Which of the following is the most likely diagnosis of this patient's skin condition?

A) Dyshidrotic eczema

B) Herpetic whitlow

C) Nummular eczema

D) Psoriasis

E) Scabies

F) Tinea manuum
Explanation
Correct Answer:

A) Dyshidrotic eczema

Acute palmoplantar eczema (dyshidrotic eczema)

• Recurrent, acute episodes


Clinical features • Deep-seated, pruritic vesicles & bullae at hands & feet
• Complications: desquamation, chronic dermatitis, secondary infection

• Clinical features usually adequate for diagnosis


Diagnosis
• Biopsy: intraepidermal spongiosis, lymphocytic infiltrate

• Topical emollients
Treatment
• High/super high–potency topical corticosteroids

This patient has a recurrent, pruritic, vesicular rash consistent with dyshidrotic eczema (ie, acute palmoplantar
eczema, pompholyx). Dyshidrotic eczema is a common, although poorly understood, disorder that primarily affects
the hands and feet. The etiology is uncertain but likely multifactorial and variable; contributing factors may include
irritant or allergic contact dermatitis, ultraviolet radiation, and immune reactions to remote allergens or fungi.

The diagnosis is primarily based on clinical findings. A recurrent, pruritic rash characterized by deep-seated
vesicles that preferentially affect the palms, soles, and sides of the digits is typical. Biopsy can be performed for
confirmation but is not usually necessary. Patients with dyshidrotic eczema should be counseled on the use of
emollients, avoidance of irritants, and protection from cold and/or wet conditions. When additional intervention is
needed, topical corticosteroids are the first-line treatment; high-potency and super high–potency
corticosteroids (eg, betamethasone dipropionate) are preferred.

(Choice B) Herpetic whitlow presents with localized vesicles and pustules. The lesions are painful but not typically
pruritic and would not likely affect both hands.

(Choice C) Nummular eczema presents with chronic, round (coin-shaped), scaly plaques in regions of dry skin,
primarily on the lower extremities.

(Choice D) Psoriasis is characterized by chronic, well-demarcated scaly plaques. Vesicles and severe pruritus are
not common.

(Choice E) Scabies presents with papules, vesicles, and burrows, often on the hands (eg, web spaces, flexural
surfaces). Pruritus can be severe, but stereotyped recurrences are more consistent with dyshidrotic eczema.

(Choice F) Tinea manuum presents with chronic, scaly, irregular, or annular patches on the hands. It is usually
unilateral and typically occurs in association with tinea pedis. It is not typically vesicular and would not be expected
to cause repeated acute outbreaks.

Educational objective:
Dyshidrotic eczema (acute palmoplantar eczema) is characterized by a recurrent, pruritic, vesicular rash that
primarily affects the palms, soles, and sides of the digits. The diagnosis is based on clinical features, and treatment
includes high- and super high–potency topical corticosteroids.
Question #85

A 17-year-old girl comes to the office for follow-up due to worsening acne. The patient has scattered papules and
pustules with mild-to-moderate redness on the cheeks. The lesions are not painful, and there is no scarring. The
patient has been using topical tazarotene and benzoyl peroxide for 2 months without improvement and is concerned
that the acne is getting worse. She is sexually active. Medical history is otherwise negative. On examination, she
has moderate pustular acne with erythema on the cheeks. Which of the following is the best next step in
management of this patient?

A) Add oral spironolactone

B) Add topical clindamycin

C) Add topical hydrocortisone

D) Add topical salicylic acid

E) Switch to oral isotretinoin


Explanation
Correct Answer:

B) Add topical clindamycin


This patient has worsening inflammatory acne vulgaris despite appropriate initial treatment. Acne is
characterized by obstruction of the pilosebaceous follicle by the abnormal proliferation of keratinocytes (ie,
hyperkeratinization) in association with the excessive accumulation of sebum. The resulting comedone is
colonized by Cutibacterium acnes, which triggers an inflammatory response.
Initial management of inflammatory acne includes a topical retinoid (eg, tazarotene, tretinoin), which inhibits
comedogenesis, and benzoyl peroxide, which has bactericidal activity against C acnes. However, when this initial
regimen is ineffective, a topical antibiotic (eg, clindamycin) may be added. Oral antibiotics (eg, doxycycline) are
also effective and useful when topical agents are ineffective or impractical (eg, widespread inflammatory acne
affecting the back).

(Choice A) Androgens enhance sebum production and facilitate C acnes colonization. Spironolactone is an
antiandrogenic agent that can be used to treat women whose inflammatory acne does not respond to topical
treatment or who have androgen excess (eg, polycystic ovary syndrome). Because it can cause feminization of
male fetuses, it is contraindicated in sexually active women who do not use contraception. This patient is not on
maximum topical therapy; spironolactone is not indicated at this time.

(Choice C) Topical hydrocortisone is a corticosteroid used to treat inflammatory dermatoses (eg, eczema,
psoriasis) but is not indicated in acne. The inflammatory changes in acne are triggered by C acnes and usually
improve with antibiotic therapy.

(Choice D) Topical salicylic acid is a comedolytic agent that has mild anti-inflammatory properties. It can be
considered for inflammatory acne in patients who do not tolerate retinoids; however, retinoids are more effective and
generally preferred.

(Choice E) Oral isotretinoin is typically used to treat severe, inflammatory nodulocystic acne, but it is teratogenic
and can cause serious adverse effects (hypertriglyceridemia, pseudotumor cerebri); women who are sexually active
must be on 2 forms of contraception. This patient's acne is not severe enough to warrant isotretinoin.

Educational objective:
Inflammatory acne is treated with topical retinoids and benzoyl peroxide. When this regimen is inadequate, the
addition of topical or oral antibiotics is recommended.

Reference
• Acne vulgaris: diagnosis and treatment.
Question #86

A 52-year-old man comes to the office due to an enlarging rash at the left ankle and upper back. The rash began 6
weeks ago and is associated with significant itching. The patient has tried topical antihistamines and calamine
lotion without relief. Medical history is otherwise unremarkable. Temperature is 36.9 C (98.4 F), blood pressure is
135/82 mm Hg, pulse is 76/min, and respirations are 12/min. BMI is 25 kg/m2. Examination shows a rash at the
ankle, as seen in the image below, and a similar rash is noted at the upper back.
Which of the following is the most likely diagnosis in this patient?

A) Erythema marginatum

B) Erythema migrans

C) Nummular eczema

D) Pityriasis rosea

E) Tinea corporis
Explanation
Correct Answer:

E) Tinea corporis

Tinea corporis (ringworm)

• Athletes who have skin-to-skin contact


• Humid environment
Risk factors
• Contact with infected animals (eg,
rodents)

• Scaly, erythematous, pruritic patch with


centrifugal spread
Presentation
• Subsequent central clearing with raised,
annular border

• First-line/localized: topical antifungals


(eg, clotrimazole, terbinafine)
Treatment
• Second-line/extensive: oral antifungals
(eg, terbinafine, griseofulvin)

This patient has a pruritic rash with a scaly, erythematous border and partial central clearing consistent with tinea
corporis (ringworm). Tinea corporis is a cutaneous dermatophyte infection most commonly caused by
Trichophyton rubrum. Patches of tinea corporis are typically annular but may become confluent to form a "flower
petal" shape.

Patients with tinea corporis often have concurrent infection elsewhere on the body, hands (tinea manuum), groin
(tinea cruris), or between the toes or sides of the feet (tinea pedis). Those with immunocompromising conditions
(eg, HIV, diabetes mellitus) can have severe or widespread disease. Other risk factors include exposure to warm,
moist areas (eg, public showers, swimming pools), contact with infected animals or people (eg, during sports
participation), and wearing tight or occlusive clothing.

The diagnosis is confirmed using potassium hydroxide (KOH) preparation of skin scrapings. Most cases can be
treated with topical antifungals (eg, terbinafine, clotrimazole); patients who fail topical therapy or have extensive
disease may need systemic therapy (eg, oral terbinafine, fluconazole).

(Choice A) Erythema marginatum is a manifestation of acute rheumatic fever characterized by numerous,


widespread plaques with a sharp annular or serpiginous border and central clearing. It is transient, nonpruritic, and
most common in children.

(Choice B) Erythema migrans is a target-shaped rash that is the cardinal manifestation of Lyme disease. The
lesions are not scaly and do not have a raised border. Patients frequently have systemic symptoms (eg, myalgias,
fatigue) and history of exposure to ticks in an endemic area.

(Choice C) Nummular eczema is a chronic, relapsing/remitting condition that presents with erythematous, pruritic,
coin-shaped patches on the extremities. The lesions may initially have a scant exudate and are uncommon on the
back and upper trunk.

(Choice D) Pityriasis rosea is characterized by numerous small, oval, scaly plaques, often with an initial lesion
(herald patch) that is much larger than the later lesions. It is typically seen in children and young adults, and usually
occurs on the trunk and proximal (not distal) extremities. In addition, the rash typically resolves within 6 weeks of
presentation.

Educational objective:
Tinea corporis is a cutaneous dermatophyte infection that causes a pruritic rash with a scaly, erythematous border
and central clearing. Patients often have concurrent infection elsewhere on the body, hands, groin, or feet. Those
with immunocompromising conditions (eg, HIV, diabetes mellitus) can have severe or widespread disease.

Reference
• Mycology - an update. Part 1: dermatomycoses: causative agents, epidemiology and pathogenesis.
Question #87

A 35-year-old woman comes to the office due to a rash. For the past year, she has had progressive development of
multiple lesions on her face, neck, and central chest associated with mild pruritus. The patient otherwise feels well
and has no chronic medical conditions. Vital signs are normal. Skin examination reveals numerous scaly lesions
with pigment changes and shallow ulceration, as shown in the exhibit. Which of the following is the most likely
diagnosis for this patient's skin disorder?

A) Discoid lupus erythematosus

B) Necrobiosis lipoidica

C) Porphyria cutanea tarda

D) Systemic sclerosis
Explanation
Correct Answer:

A) Discoid lupus erythematosus

This patient has chronic, scaly, irregular, erythematous plaques with ulceration and central hypopigmentation
surrounded by hyperpigmentation. This presentation is consistent with discoid lupus erythematosus (DLE), the
most common form of chronic cutaneous lupus erythematosus. DLE can occur independently, although
systemic lupus erythematosus (SLE) eventually develops in up to 30% of patients. Risk factors for progression to
SLE include widespread lesions, concurrent arthralgias/arthritis, and high antinuclear antibody titers.

DLE most commonly affects sun-exposed regions of the head and neck but may also involve the chest and upper
extremities. The lesions tend to slowly expand over a period of months to years, eventually leading to dermal
atrophy and scarring. The rash often extends into the hair follicles, resulting in scarring alopecia.

(Choice B) Necrobiosis lipoidica is characterized by confluent annular lesions with a yellowish-brown hue, dilated
blood vessels, and epidermal atrophy. It typically affects pretibial skin and is most common in patients with diabetes
mellitus.

(Choice C) Like DLE, porphyria cutanea tarda affects sun-exposed areas (mainly the face and dorsum of the
hands). However, the lesions characteristically form fragile blisters and erosions rather than scaly plaques. Also,
although the disorder can lead to chronic scarring, individual lesions are often triggered acutely by minor trauma.

(Choice D) Systemic sclerosis (scleroderma) causes induration and hardening of the skin, most commonly at the
hands/fingers and face. Ulcerations are typically located at the fingertips.

Educational objective:
Discoid lupus erythematosus presents with scaly, erythematous plaques, leading to atrophy, hypopigmentation, and
scarring. It most commonly affects sun-exposed areas of the head and neck.
Reference
• Cutaneous lupus erythematosus: diagnosis and treatment.
Question #88

A 24-year-old woman comes to the office due to skin lesions on her trunk and proximal upper limbs. The patient
noticed the lesions after returning from a summer vacation at a Florida beach a week ago. During the vacation, she
had sun exposure on several occasions without prior application of sunscreen. The lesions are mildly itchy but not
painful. Medical history is notable for hypothyroidism, for which she takes levothyroxine. She drinks alcohol on
social occasions but does not use tobacco or illicit drugs. Examination shows hypopigmented lesions as shown in
the image below.
Which of the following is the most likely cause of this patient's current condition?

A) Autoimmune melanocyte injury

B) Candida albicans infection


C) Chemical irritant exposure

D) Dermatophyte skin infection

E) Epidermal hyperproliferation

F) Malassezia globosa infection

G) Ultraviolet skin damage


Explanation
Correct Answer:

F) Malassezia globosa infection

Tinea versicolor (pityriasis versicolor)

• Malassezia globosa skin flora grows in


Pathogenesis
exposure to hot & humid weather

• Hypopigmented, hyperpigmented, or mildly


erythematous lesions (face in children, trunk
Clinical
& upper extremities in adolescents & adults)
features
• ± Fine scale
• ± Pruritus

• KOH preparation shows hyphae & yeast


Diagnosis
cells in a "spaghetti & meatballs" pattern

• Topical ketoconazole, terbinafine, or


Treatment
selenium sulfide
KOH = potassium hydroxide.

Tinea versicolor (pityriasis versicolor) is a superficial fungal skin infection caused by Malassezia species
(nondermatophytic, lipid-dependent yeasts; eg, M globosa, M furfur). It is characterized by salmon-colored, hyper-
or hypopigmented macules that are sometimes covered by fine scales, most commonly on the upper trunk and
extremities. Hypopigmented areas are frequently noticed following sun exposure due to tanning of the surrounding
skin. Lesions may be itchy.

Potassium hydroxide (KOH) preparation of skin scrapings shows a "spaghetti and meatballs" appearance with
large, blunt hyphae (spaghetti) and thick-walled budding yeast (meatballs). Topical treatment with selenium
sulfide or ketoconazole is recommended, but the pigmentation changes can take months to resolve following
treatment.

(Choice A) Vitiligo is characterized by complete depigmentation (from melanocyte destruction, possibly due to
autoimmune mechanisms). However, it most commonly affects the face and hands and would not cause pruritus.

(Choice B) Cutaneous candidiasis presents with an erythematous, vesiculopapular rash in warm, moist areas (eg,
skinfolds). It is often associated with recent antibiotic use or immunocompromised states (eg, uncontrolled
diabetes).

(Choice C) Irritant contact dermatitis is caused by physical or chemical irritation, most commonly due to soaps,
solvents, or acid/alkali. It presents with erythema and fissuring, usually on the hands.

(Choice D) Tinea corporis (ringworm) presents as annular lesions with advancing scaly borders and central
clearing or as scaly patches over the trunk. Other dermatophyte infections include tinea pedis (athlete's foot), tinea
cruris (jock itch), and tinea capitis (scalp ringworm). Tinea versicolor is the only tinea infection that is not caused by
dermatophytes (which require keratin for growth).

(Choice E) Psoriasis is characterized by epidermal hyperproliferation and abnormal differentiation, and causes
erythematous plaques over extensor surfaces. Guttate psoriasis presents with scattered, scaly, erythematous
papules or small plaques, typically following an acute streptococcal infection.
(Choice G) Ultraviolet injury can lead to acute (eg, sunburn) or chronic (eg, photoaging) dermal injury. Sunburn
may show patchy involvement due to uneven application of sunscreen, but round hypopigmented macules would be
unusual.

Educational objective:
Tinea versicolor is a superficial nondermatophyte fungal skin infection caused by Malassezia species and
characterized by salmon-colored, hyper- or hypopigmented macules. The diagnosis is confirmed with potassium
hydroxide preparation of skin scrapings, and treatment is with topical antifungals.

Reference
• Pityriasis versicolor: an update on pharmacological treatment options.
Question #89

A 30-year-old woman comes to the office due to a 10-day history of a pruritic rash. The rash initially started on the
hands and quickly spread to the fingers, wrists, and elbows. The itching is particularly pronounced at night. The
patient works as a flight attendant and recently returned from an international trip, during which she stayed in a
hotel for 3 days. Her daughter also developed similar symptoms 2 days ago. She has 1 sexual partner and uses
oral contraceptive pills. Vital signs are normal. Skin examination shows small, erythematous papules on the palms
with scattered vesicles and pustules on the finger webs, palms, and wrist creases as shown below. There are
numerous excoriations on the wrists and elbows. The remainder of the examination is within normal limits.
Which of the following is the most likely diagnosis?

A) Allergic contact dermatitis

B) Bed bug infestation

C) Bullous impetigo
D) Hand-foot-and-mouth disease

E) Scabies

F) Secondary syphilis
Explanation
Correct Answer:

E) Scabies

Scabies

• Sarcoptes scabiei mite infestation


Pathogenesis
• Spread by direct person-to-person contact

• Extremely pruritic, small, erythematous papules


• Distribution: interdigital web spaces, flexor wrists,
Clinical features extensor elbows, axillae, feet, umbilicus & genitalia
• Burrows (thin serpiginous lines) may not be visible but
pathognomonic if present

• Clinical
Diagnosis
• Skin scraping with microscopy is confirmatory

• Topical 5% permethrin OR oral ivermectin


Treatment • Treat household members & close personal contacts
• Environmental measures (eg, launder clothing/bedding)

This patient with a pruritic rash involving the web spaces and palms most likely has scabies, or infestation by the
mite Sarcoptes scabiei. Scabies is highly contagious and spreads primarily through person-to-person contact,
although fomite transmission (eg, bedding) can occur because mites can survive off the skin for several days. This
patient's family member with similar symptoms supports the diagnosis.

Scabies presents with a rash that is intensely pruritic (worse at night) and comprised of small, crusted,
erythematous papules with extensive excoriations. Patients can also develop vesicles, pustules, and wheals, as in
this woman, and linear burrows are pathognomonic if visible. Common locations include the palms, interdigital web
spaces, and sides of the fingers. The flexor wrists and extensor elbows are often involved, as seen in this patient.

Diagnosis is usually clinical but can be confirmed by light microscopy of skin scrapings revealing mites, ova, and
feces.

(Choice A) Allergic contact dermatitis due to poison ivy causes an intensely pruritic rash that can be vesicular.
However, lesions are often linear, and palmar involvement is uncommon (due to skin thickness). In addition, this
patient's family member with a similar rash makes an infectious cause more likely.

(Choice B) Bed bug infestations present as pruritic, erythematous papules. However, bites often present in linear
tracks ("breakfast, lunch, and dinner" bites); and pustules and finger web involvement are not typical. Moreover,
bites on the palms are uncommon due to skin thickness, making this diagnosis less likely than scabies.

(Choice C) Bullous impetigo presents with flaccid bullae (which are not seen here) that rupture, leaving a brown-
yellow crust with a collarette of scale. This condition usually occurs in young children without palmar involvement.

(Choice D) Hand-foot-and-mouth disease (HFMD) presents with fever, oral lesions, and a maculopapular or
vesicular rash on the palms and soles that usually resolves within a week. Severe itching and pustules are not
typical, and this patient's lack of systemic findings makes HFMD unlikely.

(Choice F) Secondary syphilis causes a maculopapular rash involving the palms. However, the rash is typically
nonpruritic, and patients usually have constitutional symptoms (eg, fever) and lymphadenopathy, not seen here.

Educational objective:
Scabies causes an intensely pruritic rash that classically affects the hands (eg, palms, web spaces) and flexor
surfaces of the wrist. Erythematous papules with excoriations are typical, and a close contact with a similar rash
(person-to-person transmission) supports the diagnosis.

Reference
• Scabies: epidemiology, diagnosis, and treatment.
Question #90

A 66-year-old woman comes to the office due to rough, dry, and scaly skin. She has had these symptoms
intermittently for most of her life, but they worsen during the winter and have become progressively worse in the last
several years. The patient has associated mild itching but no erythema or exudates. She has attempted to treat
her symptoms with topical emollients but has been unable to eliminate the problem. Medical history is
unremarkable, and the patient does not smoke or drink alcohol. Vital signs are normal. An image of the patient's
skin is shown below.
Examination of the hands reveal increased major and minor lines in the palms. Which of the following is the most
likely diagnosis?

A) Atopic dermatitis

B) Dry skin

C) Ichthyosis vulgaris

D) Irritant contact dermatitis

E) Psoriasis
Explanation
Correct Answer:

C) Ichthyosis vulgaris

Ichthyosis vulgaris

• Loss of function in filaggrin gene


Pathophysiology ◦ Impaired epidermal barrier
◦ Reduced skin moisturization

• Onset in infancy/early childhood


Clinical features • Diffuse, scaly skin with mild pruritus
• Worse on extensor extremities, spares intertriginous areas

• Keratosis pilaris
Associated
• Palmar hyperlinearity
conditions
• Atopic disease

• Clinical findings
Diagnosis
• Biopsy if uncertain: reduced/absent granular layer in epidermis
• Long baths to remove scales
Therapy • Moisturization
• Keratolytics (eg, urea, alpha-hydroxy acid, salicylic acid)

This patient has ichthyosis vulgaris, a lifelong, inherited skin disorder characterized by diffuse cutaneous
scaling. It is caused by mutations of the filaggrin gene (FLG) that result in epidermal hyperplasia and defective
keratinocyte desquamation. Patients with homozygous mutations have worse clinical presentation compared to
those who are heterozygous.

Symptoms typically appear first in infancy or childhood and worsen during the dry seasons (eg, winter). The skin is
rough and dry, with fish-like scales ("ichthys" is the Greek word for "fish") that are white or light gray. There is no
associated erythema or other cutaneous lesions (eg, papules, vesicles) to indicate an alternate diagnosis (eg,
atopic dermatitis, psoriasis). As in this patient, ichthyosis vulgaris is more noticeable on the extensor surfaces of
the legs and spares the intertriginous areas (eg, axilla) and face. It is associated with palmar hyperlinearity and
atopic conditions (eg, allergic rhinitis, atopic dermatitis).

(Choice A) Although patients with ichthyosis may have concurrent atopic dermatitis, the latter has focal lesions
(eg, erythema, papules, lichenification) typically on flexural surfaces rather than diffuse scales.

(Choice B) Dry skin can occur during the winter, when ambient humidity is low; however, thick, fish-like scales and
palmar hyperlinearity are not characteristic of isolated dry skin.

(Choice D) Irritant contact dermatitis is caused by exposures to irritating chemicals (eg, soaps, detergents,
solvents) and typically presents with marked erythema, a burning sensation, or pruritus in a localized, not diffuse,
area.

(Choice E) Plaque psoriasis presents with well-demarcated, localized erythematous plaques with silvery scales
over the extensor surfaces of the elbows, knees, scalp, and trunk. These features are absent in this patient.

Educational objective:
Ichthyosis vulgaris is a lifelong, inherited disorder characterized by diffuse dry and rough skin with fish-like scales.
These features are most noticeable on the extensor legs and during the winter. There is no associated erythema or
other cutaneous lesions (eg, vesicles, papules) to indicate an alternate diagnosis.

Reference
• Ichthyosis vulgaris: the filaggrin mutation disease.
Question #91

A 29-year-old woman, gravida 1 para 0, at 20 weeks gestation comes to the office for evaluation of a lesion on her
hand. The patient first noticed the lesion a month ago after working in her garden and scratching her thumb. She
initially used a topical antibiotic ointment, but the lesion continued to grow and occasionally bled. There has been
no purulent drainage, and the patient has had no associated fever or chills. Her pregnancy has been
uncomplicated, and she has no chronic medical conditions. Temperature is 36.7 C (98.1 F), blood pressure is 120/
80 mm Hg, and pulse is 65/min. Fetal heart tones are 150/min. Examination of the hand is shown in the image
below:
Which of the following is the most likely diagnosis in this patient?

A) Bacillary angiomatosis

B) Basal cell carcinoma

C) Kaposi sarcoma
D) Keratoacanthoma

E) Pyogenic granuloma
Explanation
Correct Answer:

E) Pyogenic granuloma
This patient has a friable, dome-shaped nodule consistent with pyogenic granuloma (ie, lobular capillary
hemangioma). Pyogenic granulomas are small (<1 cm) vascular tumors composed of abnormal capillaries and
granulation tissue. They commonly occur in young adults, particularly pregnant women, likely due to the normal
pregnancy-mediated increase in vascular endothelial growth factor that may promote pyogenic granuloma growth.
Pyogenic granulomas are typically found on the hands (particularly the fingers), trunk, and oral mucosa/gingiva
(more common in pregnancy). They typically arise slowly over a few weeks to months and characteristically bleed
with minor trauma. Diagnosis is clinically based. Treatment options include surgical excision, laser therapy, and
topical treatments (eg, silver nitrate). However, lesions occurring during pregnancy usually regress postpartum
and may not require treatment.

(Choice A) Bacillary angiomatosis is a Bartonella infection seen primarily in patients with advanced HIV. It
presents as multiple small, reddish or purple papules that evolve into friable nodular lesions. Constitutional
symptoms (eg, fever, malaise, night sweats) are typically present.

(Choice B) Basal cell carcinoma presents as a slowly growing, ulcerating nodule with pearly borders and
telangiectatic vessels. It usually occurs in older individuals (age >55) and is uncommon in pregnancy.

(Choice C) Kaposi sarcoma is caused by human herpesvirus 8 and is most common in patients with advanced
HIV. In the skin, it typically presents as red or purple vascular papules.

(Choice D) Keratoacanthoma is a rapidly growing, volcano-like nodule with a central keratotic plug. It is most
common in hair-bearing skin in middle-aged and older adults and does not have a moist or friable surface.

Educational objective:
Pyogenic granulomas are small vascular tumors composed of abnormal capillaries and granulation tissue. They
typically present as friable, dome-shaped nodules on the hands, trunk, or oral mucosa/gingiva. Pyogenic
granulomas are common in pregnancy and usually regress postpartum.

Reference
• Pyogenic granuloma (lobular capillary hemangioma).
Question #92

An 80-year-old woman comes to the office for evaluation of a skin lesion. She has a black nodule on her back that
has been present "for quite a while." The nodule has occasional associated itching, but no pain, bleeding, or recent
change in appearance. Medical history is notable for hypertension, hypothyroidism, and mild dementia. The patient
lives in an assisted living facility and does not use tobacco or alcohol. On examination, there is a raised nodule with
a rubbery texture and oily surface, as shown in the image below.
What is the most likely diagnosis?
A) Acrochordon

B) Actinic keratosis

C) Basal cell carcinoma

D) Cutaneous wart

E) Melanoma

F) Seborrheic keratosis
Explanation
Correct Answer:

F) Seborrheic keratosis

Seborrheic keratosis (SK [plural, keratoses]) is a common epidermal tumor seen in middle-aged or elderly
individuals. SKs may be found almost anywhere on the body except the palms and soles. They can vary in
appearance from nearly flat macules to raised, wart-like lesions, and can be pink/white, brown, or black. They have
a velvety or greasy surface and well-demarcated border and are often described as having a "stuck-on"
appearance. SKs are usually asymptomatic but occasionally can be pruritic or tender. They are benign lesions,
although sudden onset of multiple SKs may indicate an occult internal malignancy (Leser-Trélat sign).

The diagnosis of SK is based on the characteristic presentation. Biopsy is not usually necessary but will show small
cells resembling basal cells, with variable pigmentation, hyperkeratosis, and keratin-containing cysts. Simple
observation is the preferred management, but bothersome lesions may be treated with excision, cryosurgery, or
electrodessication.

(Choice A) An acrochordon (skin tag) is a flesh-colored papule usually seen in regions of the body subjected to
friction, such as the neck, axillae, and inner thighs.

(Choice B) Actinic keratosis is a pre-malignant lesion characterized by dry, scaly, flat papules with an
erythematous base, seen most commonly in sun-exposed areas.

(Choice C) The most common presentation of basal cell carcinoma is a slow-growing papule or nodule with a
pearly, rolled border and overlying telangiectasias. Ulceration is common, as is bleeding following minor trauma.

(Choice D) Cutaneous warts are most common in children and young adults, and are usually seen on the hands,
elbows, and feet. They are not usually pigmented.

(Choice E) Biopsy is occasionally required to differentiate melanoma from SK. However, melanoma can usually be
distinguished by its indistinct or irregular border, smooth or nodular surface, changing appearance over time, and
predilection for sun-exposed areas. This lesion is located on the patient's back, has a rubbery texture, and has not
recently changed in appearance.

Educational objective:
Seborrheic keratosis is a benign epidermal tumor that presents in middle-aged or elderly individuals as a tan or
brown, round lesion with a well-demarcated border and "stuck-on" appearance. Diagnosis is based primarily on
clinical appearance, and treatment is usually not required.

Reference
• Seborrheic keratosis.
Question #93

A 28-year-old woman comes to the office for a periodic recheck of "moles." The patient has a history of multiple
irregular melanocytic nevi that have been monitored with regular surveillance, but none of the lesions have required
removal. She is concerned about a lesion on her abdomen, which is associated with intermittent itching and has
enlarged since her last visit. The patient has a history of recurrent sunburn in childhood. On examination, there are
numerous benign-appearing nevi scattered across the body surface. In addition, there is a 2 x 1 cm, pigmented
lesion at the lateral abdomen, as shown in the image, which has increased in length since her last visit 6 months
ago. Which of the following is the most likely diagnosis?

A) Café-au-lait macule

B) Discoid lupus

C) Lentigo

D) Melanocytic nevus

E) Melanoma

F) Seborrheic keratosis
Explanation
Correct Answer:

E) Melanoma

Malignant melanoma is most common in the skin, although it may occur wherever there are pigmented cells, such
as the eye and gastrointestinal mucosa. Sunlight exposure is an important factor in the pathogenesis of dermal
melanoma, and light-skinned individuals are at greatest risk. Melanomas frequently arise in preexistent dysplastic
nevi.

Gross findings that suggest melanoma rather than a benign melanocytic nevus include Asymmetric shape, irregular
or jagged Border, variability of Color (brown, black, red, blue, or unpigmented), Diameter >6 mm, and Evolution in
size and appearance over time (ABCDE criteria). In patients with multiple nevi, a lesion that is significantly
different from the patient's typical moles also warrants increased suspicion ("ugly duckling" sign). This patient has
risk factors for melanoma and has an enlarging, asymmetric lesion with irregular borders and color variability
(Choice D).

(Choice A) Café-au-lait spots are benign, lightly pigmented congenital macules. They may be an isolated finding,
but the presence of multiple lesions suggests underlying systemic disease (eg, neurofibromatosis). Café-au-lait
macules may enlarge somewhat over time, but color variegation, associated itching, and history of severe sunburn
are more consistent with melanoma.

(Choice B) Discoid lupus is characterized by erythematous, scaling plaques predominantly affecting the face, ears,
and scalp. Chronic lesions may develop depigmentation, but enlarging, pigmented lesions are more characteristic
of melanoma.

(Choice C) Lentigo is the result of intraepidermal melanocyte hyperplasia. It is most common in older individuals
and is characterized by a round or oval macule with even pigmentation.

(Choice F) Seborrheic keratosis is a benign pigmented lesion with a well-demarcated border and a velvety or
greasy surface. It may be nearly flat, but thickened lesions are often described as having a "stuck on" appearance.
Educational objective:
Malignant melanoma may occur wherever there are pigmented cells. Sunlight exposure, hereditary factors, fair
skin, and a history of preexistent dysplastic nevi are important risk factors. Findings that suggest melanoma include
asymmetric shape, irregular border, variability of color, diameter >6 mm, and evolution in size and appearance over
time.
Question #94

A 36-year-old man comes to the urgent care center due to a 2-day history of a pruritic rash on his hands. It began
with small papules and progressed to small blisters with oozing yellow fluid. A few days prior, the patient spent an
afternoon clearing bushes around his house but has had no recent travel or change in household products. Medical
history is unremarkable. Temperature is 36.6 C (97.9 F), blood pressure is 125/79 mm Hg, and pulse is 76/min.
Skin examination shows erythematous plaques with eroded vesicles and small bullae between the fingers of the left
hand and on the dorsum of the right hand, as shown in the exhibit. Which of the following is the most likely
diagnosis for this patient?

A) Bullous impetigo

B) Bullous pemphigoid

C) Contact dermatitis

D) Herpes simplex infection

E) Sporotrichosis
Explanation
Correct Answer:

C) Contact dermatitis

Allergic contact dermatitis

• Type IV hypersensitivity reaction


Pathophysiology • Common triggers: poison ivy/oak, nickel, dyes, topical
medications, skin care products

• Acute: pruritic red, indurated plaques with vesicles/bullae


Clinical
presentation
• Chronic: lichenification, fissuring

• Clinical findings
Diagnosis
• Patch testing for persistent cases

• Avoidance of suspected allergen


Treatment
• Topical or systemic corticosteroid

This patient with pruritic erythematous plaques, vesicles, and bullae that developed a few days after contact with
plants most likely has acute allergic contact dermatitis (ACD) due to Toxicodendron species (poison ivy/oak/
sumac). ACD is a type IV hypersensitivity reaction that can be triggered by a variety of allergens, including plants,
natural rubber, leather dyes, nickel, topical medications, and skin care products.

Toxicodendron produces urushiol, a highly allergenic resin. In sensitized individuals, the rash can develop within
4-96 hours of exposure; in previously unexposed individuals, it may not be seen for 3 weeks. The typical rash, as
seen in this patient, is usually limited to exposed skin, frequently forming linear streaks where skin has brushed
against plant leaves. However, diffuse or atypical patterns can be seen after exposure to contaminated clothes,
pets, or smoke from burning plants. Treatment typically involves high-potency topical or oral corticosteroids.

(Choice A) Bullous impetigo, a skin infection caused by Staphylococcus aureus, presents with rapidly enlarging
vesicles and bullae, which rupture to yield exudates and crusts. Pruritus can be present, but the surrounding skin
usually has little to no erythema. The condition is most common in children; occurrence in an adult is unusual
unless the patient is immunocompromised (eg, HIV infection).

(Choice B) Bullous pemphigoid is a chronic (not acute), pruritic, autoimmune, blistering disease that typically
affects people age >60. Areas affected include axillae, medial thighs, groin, abdomen, forearms, and lower legs.
Isolated involvement of the digits and hands is atypical.

(Choice D) Herpetic whitlow is an acute herpes simplex virus infection of the hand, often occurring in patients with
concurrent herpes labialis. It begins as tingling and pain, with the progressive appearance of small, clustered
vesicles on an erythematous base. It most commonly affects a single digit; bilateral involvement is rare.

(Choice E) Sporotrichosis is typically acquired by direct inoculation of the skin by a plant (eg, rose thorn)
containing the fungus. It presents with ulcerating, pustular nodules at the site of inoculation (eg, usually on 1 hand)
and associated lymphatic channels. The lesions are generally asymptomatic; vesicles and bullae are not
characteristic.

Educational objective:
Toxicodendron plants (poison ivy/oak/sumac) are a frequent cause of allergic contact dermatitis. The intensely
pruritic, erythematous, vesicular rash involves exposed skin, forming linear streaks where skin has brushed against
the plant leaves. However, diffuse or atypical patterns can be seen after exposure to contaminated clothes, pets, or
smoke from burning plants.
Reference
• Diagnosis and management of contact dermatitis.
Question #95

A 45-year-old woman comes to the office due to painful sores and blisters. Three weeks ago, the patient developed
painful sores inside her nose and mouth, which made it difficult for her to eat and swallow. Two weeks later, she
developed painful blisters on her trunk and extremities. The patient has no chronic medical conditions and takes no
medications. Temperature is 36.8 C (98.2 F), blood pressure is 115/76 mm Hg, pulse is 82/min, and respirations
are 14/min. On examination, the nasal, buccal, and palatine mucosa reveal several erosions, as shown in the
exhibit. The skin shows scattered, large erosions and a few bullae on the chest, back, and extremities, with a
background of normal-appearing skin. Heart, lung, and abdominal examinations are normal. Which of the following
is the most likely diagnosis?

A) Bullous pemphigoid

B) Crohn disease

C) Erosive lichen planus

D) Granulomatosis with polyangiitis

E) Pemphigus vulgaris
Explanation
Correct Answer:

E) Pemphigus vulgaris

Pemphigus vulgaris vs bullous pemphigoid

Pemphigus vulgaris Bullous pemphigoid

Age of onset • 40-60 • >60

• Painful • Pruritic
Clinical features • Flaccid bullae → erosions • Tense bullae
• Mucosal involvement common • Mucosal involvement rare

Histology • Intraepidermal cleavage • Subepidermal cleavage

• Net-like intercellular IgG • Linear IgG against hemidesmosomes along


Immunofluorescence
against desmosomes basement membrane

This patient with painful mucocutaneous bullae and erosions has typical features of pemphigus vulgaris (PV), a
life-threatening autoimmune blistering disease that results from a loss of cohesion between epidermal cells.
PV typically affects patients age 40-60. Lesions are characterized by painful, flaccid bullae that easily rupture to
form erosions, which may be the only lesions at the time of presentation. Mucosal surfaces are almost always
affected, with the oral mucosa being the most common initial site of involvement, as in this patient. Cutaneous
lesions typically follow and arise from a background of normal-appearing or erythematous skin.

The diagnosis of PV must be confirmed with a skin biopsy because therapy is often lifelong and involves agents
with significant toxicity (eg, systemic corticosteroids, rituximab). Serology for antibodies against desmosome
components (ie, desmogleins 1 and 3) can further support the diagnosis.

(Choice A) Bullous pemphigoid is an autoimmune blistering disease somewhat resembling PV. It can cause
blisters (which are tense vs flaccid in PV) and erosions; however, the lesions are pruritic rather than painful, patients
are usually age >60, and the mucosa is rarely involved.

(Choice B) Although oral aphthous ulcers can be a manifestation of Crohn disease, cutaneous manifestations of
Crohn disease (ie, erythema nodosum, pyoderma gangrenosum) do not typically include bullae.

(Choice C) Erosive oral lichen planus presents with painful ulcerations typically accompanied by white, reticular
lesions (ie, Wickham striae) and erythema, which are both absent in this patient. In addition, cutaneous lichen
planus presents as pruritic, purple, polygonal papules and plaques, not as bullae and erosions.

(Choice D) Granulomatosis with polyangiitis can cause oral and nasal ulcers; however, cutaneous findings consist
of purpura caused by leukocytoclastic angiitis.

Educational objective:
Pemphigus vulgaris is a mucocutaneous autoimmune blistering disease typically occurring in patients age 40-60.
Lesions consist of painful, flaccid blisters that easily rupture, forming erosions. The mucosa (eg, oral) is almost
always involved, and these lesions typically precede cutaneous lesions. A skin biopsy is required to confirm
diagnosis.

Reference
• Pemphigus.
Question #96

A 32-year-old woman comes to the office due to a "mole." The lesion is located on her back, and she believes it
has been present for years but does not recall when it first appeared. The patient thinks it might be enlarging and is
concerned due to a family history of skin cancer. The lesion is not painful and does not itch or bleed. She has fair
skin and a history of severe sunburns in childhood. Medical history is unremarkable, and she does not use tobacco,
alcohol, or illicit drugs. Vital signs are normal. Skin examination shows a few scattered benign-appearing nevi and
a single prominent lesion at her right scapular area. Which of the following additional skin examination findings
would be most concerning for malignancy?

A) Central keratin plug

B) Golf ball-size subcutaneous mass

C) Multicolored lesion

D) Peripheral scaling

E) "Stuck-on" appearance
Explanation
Correct Answer:

C) Multicolored lesion

Clinical features of melanoma (ABCDE)

• Asymmetry: when bisected, the 2 sides are not identical


• Border irregularities: uneven edges, pigment fading off
• Color variegation: variable mixtures of brown, tan, black & red
• Diameter: ≥6 mm
• Evolving: lesion changing in size, shape, or color; new lesion

Early diagnosis is critical in melanoma as survival rates rapidly decrease with increased tumor thickness and depth
of invasion. Historical and examination findings are used in the evaluation of moles to determine which lesions
require excision and which can be safely observed. Significant melanoma risk factors include prior personal or
family (≥2 members) history of melanoma, numerous (>100) nevi, prior atypical/dysplastic nevi, fair skin, and history
of severe sunburns.

Gross features favoring the diagnosis of melanoma include Asymmetry, Border irregularities, Color variegation (ie,
color differences within a single lesion), Diameter ≥6 mm, and change in appearance over time (Evolving lesion).
Coloration of melanoma can be highly variable. Lesions can be pigmented or unpigmented and various shades of
brown, black, red, blue, or pink. Color variegation in particular strongly suggests malignancy.

(Choice A) Keratoacanthomas are cutaneous tumors that usually present as dome-shaped nodules with a central
keratinous plug. They are generally benign, although rare cases with malignant transformation and metastasis
have been reported.
(Choice B) Lipomas present as slowly enlarging, mobile subcutaneous masses. They have a soft or rubbery
texture and are often golf-ball size or larger by the time they are brought to clinical attention. Lesions with the
typical appearance of a lipoma are rarely malignant and can be safely observed without biopsy.

(Choice D) Ring-shaped inflammatory skin lesions with peripheral scaling are characteristic of tinea corporis.

(Choice E) Seborrheic keratosis is a benign pigmented lesion with a well-demarcated border and a velvety or
greasy surface. It can be nearly flat, but thickened lesions are often described as having a "stuck on" appearance.
An explosive onset of multiple seborrheic keratoses (Leser-Trélat sign) can indicate internal malignancy, but
isolated lesions are benign.

Educational objective:
On clinical examination, features of a lesion that suggest melanoma include Asymmetry, Border irregularities, Color
variegation, Diameter ≥6 mm, and Evolution in color, size, or shape.

Reference
• Skills training to learn discrimination of ABCDE criteria by those at risk of developing melanoma.
Question #97

A 28-year-old woman comes to the office due to a painful leg ulcer that has been expanding over the last 2 months.
She has a history of Crohn disease, which has been well controlled for the past 2 years with azathioprine. The
patient works as a gardener. She does not smoke or use illicit drugs. Vital signs are within normal limits. Pulse
oximetry shows O2 saturation of 98% on room air. The leg ulcer is shown in the image below. The remainder of the
physical examination is normal.
Which of the following is the most likely diagnosis?
A) Ecthyma gangrenosum

B) Erythema nodosum

C) Hidradenitis suppurativa

D) Pyoderma gangrenosum

E) Sporotrichosis
Explanation
Correct Answer:

D) Pyoderma gangrenosum

Pyoderma gangrenosum

• Begins with small papule or pustule


Clinical • Rapidly progressive, painful ulcer with purulent base & violaceous border
features
• Precipitation of ulceration at site of injury (pathergy)

• Peak onset age 40-60


• Women > men
Epidemiology
• Association with inflammatory bowel disease, inflammatory (eg, rheumatoid) arthritis,
malignancy

• Exclusion of other causes of ulceration (eg, infection)


Diagnosis
• Skin biopsy: mixed inflammation (neutrophil predominant)

Treatment • Local or systemic glucocorticoids

This patient's skin lesion is consistent with pyoderma gangrenosum (PG). PG starts as an inflammatory papule,
pustule, or nodule and progresses to form an expanding ulcer with a purulent base and an irregular, violaceous
border. PG can present as single or multiple lesions, usually on the trunk or lower extremities. Nearly 30% of
cases are triggered by local trauma (pathergy).

Most patients with PG have an associated systemic disorder. Common associations include inflammatory bowel
disease, inflammatory arthropathies (eg, rheumatoid arthritis), and hematologic conditions (eg, acute myeloid
leukemia). PG is diagnosed clinically after excluding other diagnoses (eg, infection, venous ulcers, cutaneous
cancers). Skin biopsy of the ulcer margin is recommended and typically shows a neutrophil-predominant mixed
cellular infiltrate with dermal and epidermal necrosis. Surgical debridement is usually avoided due to the potential
of inducing pathergy; most lesions respond to treatment with local or systemic glucocorticoids.

(Choice A) Ecthyma gangrenosum is a skin infection most frequently due to Pseudomonas aeruginosa. It
presents as hemorrhagic pustules with surrounding erythema that evolve into necrotic ulcers and occurs most
commonly in the setting of profound neutropenia and bacteremia. Unlike in PG, patients typically have signs of
systemic infection (eg, fever), and pain is significantly less prominent.

(Choice B) Erythema nodosum is an acute panniculitis that presents as tender, erythematous nodules/plaques on
the lower extremities. It is also associated with systemic diseases; however, the lesions do not usually ulcerate,
and resolve without scarring after 2-8 weeks.

(Choice C) Hidradenitis suppurativa occurs at the axilla, groin, or scalp. It presents with inflammatory nodules or
abscesses with purulent drainage and chronic scarring.

(Choice E) Sporotrichosis is a fungal skin infection acquired through breaks in the skin, typically from rose bushes
or other plant materials. It presents as a small papule or nodule at the inoculation site, usually on exposed skin of
the hand or arm, which then may enlarge, ulcerate, and lead to additional lesions along the draining lymphatics.
Pain, if present, is mild.

Educational objective:
Pyoderma gangrenosum causes a rapidly progressive and painful ulcer with a purulent base and violaceous
border. Most patients have associated systemic disease (eg, inflammatory bowel disease). Diagnosis is made
clinically after excluding other etiologies, usually with skin biopsy.

Reference
• Clinical, serologic, and genetic factors associated with pyoderma gangrenosum and erythema nodosum in
inflammatory bowel disease patients.

• Characteristics and treatment of pyoderma gangrenosum in inflammatory bowel disease.


Question #98

A 15-year-old, previously healthy girl comes to the office due to "spots" on her face. About a year ago, the patient
started having tiny bumps on her forehead; they have progressively increased in number and size. She has used
several over-the-counter facial cleansing products without relief. The patient does not use tobacco, alcohol, or illicit
drugs and is not sexually active. Physical examination shows an oily complexion and the findings seen in the image
below:
Which of the following is the best next step in management of this patient?

A) Cessation of chocolate consumption

B) Cream-based cleansers

C) Oral contraceptives

D) Systemic antibiotics

E) Topical corticosteroid

F) Topical retinoids
Explanation
Correct Answer:

F) Topical retinoids
This patient has noninflammatory, comedonal acne on the forehead; the treatment of choice is a topical
retinoid. The pathogenesis of acne involves the following:

• Hyperkeratinization leads to keratin plug formation in the hair follicles, preventing sebum flow from the
follicles to the skin surface. When the keratin plug is hidden under the skin, it is called a closed comedone,
like those seen in this patient. In contrast, when the keratin plug is exposed at the follicular orifice, it is
called an open comedone; melanin and oxidized sebum give open comedones a dark color. Comedone
formation (ie, comedogenesis) is the initial step in acne pathogenesis.

• Cutibacterium acnes, which is part of the skin flora, relies on sebum as a nutrient source. As the
bacterium proliferates, it triggers an inflammatory response that leads to formation of the red papules and
pustules seen in inflammatory acne.

• Ongoing inflammation leads to follicle rupture, releasing additional proinflammatory substances and leading
to the formation of nodulocystic acne.

Topical retinoids inhibit comedogenesis by normalizing keratinization; decreasing epithelial cohesiveness, which
facilitates desquamation and prevents follicular plugging; and increasing epithelial turnover, which facilitates the
extrusion of comedones. Because retinoids inhibit comedogenesis, the first step in acne pathogenesis, they are
also used as part of a multiagent therapy for inflammatory acne.

(Choice A) Despite coverage in the lay media, chocolate has not been shown to promote acne formation.

(Choice B) Patients with oily skin are generally advised to use water- or gel-based cleansers. Cream-based
cleansers contain higher oil content and are comedogenic.

(Choice C) Estrogen-containing oral contraceptives have an antiandrogenic effect that inhibits sebum production.
They are commonly used for women with moderate/severe inflammatory acne, especially those who have
premenstrual acne flares. However, they are not recommended for noninflammatory comedonal acne.

(Choice D) Oral antibiotics (eg, tetracyclines) effective against C acnes are used to treat moderate/severe
inflammatory acne or acne that is widespread (eg, on the back), making the application of topical therapy
impractical.

(Choice E) Topical corticosteroids are used in a variety of inflammatory dermatoses (eg, contact dermatitis) but are
not indicated for acne. The inflammation seen in acne is triggered by C acnes; therefore, antibiotics are part of the
multiagent regimen for inflammatory acne.
Educational objective:
Topical retinoids are first-line treatment for noninflammatory, comedonal acne. They inhibit comedogenesis by
normalizing keratinization, decreasing epithelial cohesiveness, and increasing epithelial turnover.

Reference
• Acne vulgaris: diagnosis and treatment.
Question #99

A 37-year-old man comes to his primary care physician for the evaluation of slightly pruritic skin lesions on his penis
and around his anus. He has no fever, malaise, or anorexia. He is sexually active with multiple male partners and
uses condoms occasionally. The patient has never been tested for HIV or other sexually transmitted diseases. He
has no drug allergies. Examination shows skin-colored, verrucous, papilliform lesions as shown in the image below.
Which of the following is the most likely cause of this patient's skin condition?

A) Anatomical variation

B) Herpes simplex virus


C) Human papillomavirus

D) Poxvirus

E) Treponema pallidum
Explanation
Correct Answer:

C) Human papillomavirus

Condylomata acuminata (anogenital warts) are caused by the human papillomavirus (HPV), which is the most
common sexually transmitted disease in the United States. The characteristic lesions are verrucous, papilliform,
and either pink or skin-colored. The lesions are usually asymptomatic but may have mild itching or burning.
Systemic symptoms are usually absent.

The diagnosis of condyloma acuminata is primarily based on clinical presentation. Biopsy may be considered in
atypical cases. HPV infection (especially serotypes 16 and 18) is associated with increased risk for squamous cell
carcinoma of the anus, genital organs, and throat. HPV is also associated with increased risk for other sexually
transmitted diseases, especially HIV. HIV screening should be offered to patients with a new diagnosis of HPV.
The condition is self-limited in most cases. If specific treatment is desired, options include:

1. Chemical or physical agents (eg, trichloroacetic acid, podophyllin)


2. Immune therapy (eg, imiquimod)
3. Surgery (eg, cryosurgery, excision, laser treatment)

(Choice A) Pearly pink penile papules are a common anatomical variant, typically presenting as small papules
evenly distributed in a ring around the corona of the glans penis. They are a benign non-infectious condition,
though patients often request removal to avoid the appearance of having a sexually transmitted disease.

(Choice B) Herpes simplex is characterized by small grouped vesicles that erode to form small ulcers.

(Choice D) Molluscum contagiosum is a self-limited, localized skin infection caused by a poxvirus. It is


characterized by small pink or skin-colored papules with indented centers that may occur anywhere except the
palms and soles. These lesions may be widely scattered and may occur in a linear pattern due to spread of the
virus to adjacent skin via scratching.
(Choice E) Condyloma lata are a manifestation of secondary syphilis characterized by flattened pink or gray
velvety papules. These are seen most commonly at the mucous membranes and moist skin of the genital organs,
perineum, and mouth.

Educational objective:
Condylomata acuminata (anogenital warts) are verrucous papilliform lesions located in the anogenital region.
These lesions are caused by human papillomavirus, which is the most common sexually transmitted disease in the
United States. Certain serotypes (especially 16 and 18) are associated with squamous cell carcinoma of the anus,
genital organs, and throat.

Reference
• Genital HPV infection and related lesions in men.
Question #100

A 25-year-old woman comes to the physician with a rash on her buttocks. The lesions began 10 days ago and are
associated with intense itching and burning. She has attempted treatment with cool compresses and calamine
lotion without relief. The patient has had occasional diarrhea and unintentional weight loss over the past few
months. Her medical history is otherwise unremarkable. Her temperature is 37.1 C (98.8 F), blood pressure is 117/
76 mm Hg, pulse is 82/min, and respirations are 14/min. The skin lesions are shown in the image below.
Which of the following is the most likely diagnosis of this patient's rash?

A) Bullous pemphigoid

B) Dermatitis herpetiformis

C) Erythema multiforme
D) Herpes zoster

E) Pemphigus vulgaris

F) Psoriasis
Explanation
Correct Answer:

B) Dermatitis herpetiformis

Dermatitis herpetiformis (DH) causes intensely pruritic erythematous papules, vesicles, and bullae that occur
symmetrically in grouped ("herpetiform") clusters on the extensor surfaces of the elbows, knees, back, and
buttocks. DH represents an autoimmune dermal reaction due to dietary gluten and is commonly associated with
celiac disease (the likely cause of this patient's diarrhea and weight loss), although it may precede the
gastrointestinal manifestations.

Skin biopsy in DH shows subepidermal microabscesses (blisters) at the tips of the dermal papillae;
immunofluorescence studies show deposits of anti-epidermal transglutaminase IgA in the dermis. Initial treatment
includes dapsone, which has anti-inflammatory and immunomodulatory properties and provides rapid relief of
symptoms. Long-term management requires a gluten-free diet.

(Choice A) Bullous pemphigoid is an autoimmune disease that most commonly affects individuals age >60.
Patients have a variable prodrome of eczematous or urticarial lesions and subsequently develop tense bullae and
plaques affecting the flexural areas, groin, or axilla.

(Choice C) Erythema multiforme is an inflammatory disorder characterized by erythematous papules and plaques
that evolve into target lesions.

(Choice D) Reactivation of herpes zoster causes erythematous vesicles and ulcers in a dermatomal distribution,
and bilateral symptoms are rare.

(Choice E) Pemphigus vulgaris is an autoimmune disorder characterized by painful, flaccid bullae, mucosal
erosions, and separation of the epidermis from the dermis on light friction. The roof of the bullous lesions is fragile
and rapidly desquamates, leaving raw ulcers.

(Choice F) Psoriasis is a chronic inflammatory disorder that primarily affects the extensor surfaces of the elbows
and knees. Lesions appear as red, sharply demarcated, scaling papules that coalesce to form oval plaques.

Educational objective:
Dermatitis herpetiformis is an autoimmune dermal reaction due to dietary gluten and is commonly associated with
celiac disease. It presents with clusters of pruritic papules and vesicles on the elbows, knees, back, and buttocks.
Management includes dapsone and a gluten-free diet.

Reference
• The diagnosis and treatment of dermatitis herpetiformis.
Question #101

A 50-year-old man comes to the office due to a persistent ulcer on his left leg for the past 2 months. Ten years ago,
he sustained full-thickness burns to the left pretibial and ankle areas that required extensive wound care and skin
autografting and resulted in significant scarring. The patient had been doing well until 2 months ago, when he
developed an ulcer associated with pain and bloody drainage within the region of the prior skin graft. He has tried
local wound care with antibiotic ointment, but the lesion has not healed. On physical examination, there is a 3-cm,
irregularly shaped ulcer on the left lower leg just proximal to the medial malleolus. The ulcer has red granulation
tissue at the base that bleeds easily when touched. Pedal pulses are 2+ bilaterally. The legs are symmetric in girth;
there is no edema in the legs or feet. Which of the following is the most likely diagnosis?

A) Basal cell carcinoma

B) Cutaneous T-cell lymphoma

C) Delayed skin graft failure

D) Squamous cell carcinoma

E) Venous stasis ulcer


Explanation
Correct Answer:

D) Squamous cell carcinoma

This patient's nonhealing ulcer at the site of a chronic burn scar is most concerning for squamous cell carcinoma
(SCC). Although sun exposure is the most common cause of cutaneous SCC, SCC may also arise within chronic
wounds (eg, venous stasis ulcers) or scars (eg, healed burns). The exact mechanism of malignant degeneration
is unknown; however, contributing factors likely include chronic inflammation and vascular compromise (from
scarring) that shield the area from immune surveillance. When malignant transformation does occur, the average
latency period is 30 years, but it can occur early (eg, within a year of injury).

SCC from wounds or scars (Marjolin ulcer) typically presents as an enlarging, nonhealing ulcer; it may have
excessive granulation tissue that bleeds easily on contact and/or rolled or everted wound margins. Less commonly,
it may exhibit a nodular, a verrucous, or an exophytic appearance. This type of cancer tends to be aggressive, with
a high risk for recurrence, local invasion, and metastasis; skin biopsy should be obtained for timely diagnosis.
Treatment can include excision with wide margins, Mohs micrographic surgery, and amputation proximal to the
ulcer, depending on tumor characteristics.

(Choice A) By far, SCC is the most common cancer arising from chronic wounds or scars, although basal cell
carcinoma may also develop in a minority of cases.

(Choice B) Cutaneous T-cell lymphoma, sometimes referred to as mycosis fungoides, most commonly appears as
pruritic, eczematous patches or plaques. It does not typically arise from an old scar or burn wound.

(Choice C) Skin graft failure, in which a skin graft does not adhere to the wound bed and subsequently necroses,
is most commonly caused by factors (eg, mechanical friction, underlying seroma/hematoma) that impair the new
graft's ability to receive nutrients from the wound bed and undergo neovascularization. When failure occurs, it is
typically within the first week of the graft, not years later.

(Choice E) Although venous stasis ulcers frequently occur in the medial malleolar region, they are a manifestation
of late-stage venous insufficiency and are typically accompanied by pedal and/or leg edema, features absent in this
patient.

Educational objective:
A nonhealing ulcer arising from a chronic wound or scar is concerning for squamous cell carcinoma. These
neoplasms tend to be aggressive, with a high risk for recurrence, local invasion, and metastasis. Timely diagnosis
with biopsy is required.

Reference
• Marjolin's ulcer in chronic wounds—review of available literature.
Question #102

A 56-year-old man comes to the office due to a yearlong history of skin lesions on the cheeks, forehead, and scalp.
Except for their appearance, the lesions do not trouble the patient. Medical history includes androgenic alopecia
and poorly controlled type 2 diabetes mellitus. Vital signs are normal. Findings on the left cheek are shown in the
exhibit. Similar lesions appear on the right cheek, forehead, and scalp. Which of the following is the most likely
diagnosis in this patient?

A) Actinic keratosis

B) Psoriasis

C) Seborrheic dermatitis

D) Seborrheic keratosis

E) Tinea facialis
Explanation
Correct Answer:

A) Actinic keratosis

Actinic keratosis

Clinical • Erythematous, scaly papules; rough plaques


features • Sun-exposed areas

• Clinical appearance
Diagnosis • Biopsy if features of possible SCC (eg, size ≥1 cm, rapid growth,
ulceration, tenderness)

• Chronic/persistent
Prognosis • Progression to SCC
• Spontaneous resolution

• Isolated lesions: cryotherapy


Treatment
• Diffuse lesions: topical fluorouracil, imiquimod, tirbanibulin

SCC = squamous cell carcinoma.


This patient has actinic keratosis (AK), a precursor to squamous cell carcinoma (SCC). AK presents as chronic,
scaly papules or plaques with a characteristic sandpaper-like texture; it most commonly occurs in older, fair-
skinned patients. Chronic sun exposure is the major risk factor, with lesions primarily affecting sun-exposed areas
(eg, scalp, face); the surrounding skin often displays features of solar damage (eg, telangiectasias, deep wrinkles),
as in this patient.

AK is diagnosed based on appearance, but biopsy is indicated to rule out SCC for lesions exhibiting high-risk
features (eg, >1 cm, induration/ulceration, rapid growth, therapeutic failure). Isolated lesions can be treated with
focal cryotherapy (eg, liquid nitrogen), but involvement of a large area, as seen in this patient, requires field
therapy with topical agents (eg, 5-fluorouracil).

(Choice B) Psoriasis presents as erythematous plaques with silver scales. The lesions are pruritic and most
commonly occur at the extensor surfaces (eg, knees).

(Choice C) Seborrheic dermatitis typically presents with mildly pruritic, erythematous plaques with greasy, yellow
scales on the scalp, central face, and ears. Discrete, isolated scaly papules are atypical.

(Choice D) Seborrheic keratoses typically present as brown, stuck-on papules with rough surfaces.

(Choice E) Tinea facialis presents as an erythematous annular patch or plaque with light, diffuse scales.

Educational objective:
Actinic keratosis is characterized by scaly papules or plaques with sandpaper-like texture on sun-exposed areas
(eg, face, scalp). Lesions are precursors to squamous cell carcinoma and are treated with cryotherapy or topical
agents (eg, 5-fluorouracil), depending on distribution.

Reference
• Current perspectives on actinic keratosis: a review.
Question #103

A 65-year-old woman with Parkinson disease is hospitalized due to pneumonia. At admission, the patient is
incidentally found to have a rash involving the face, scalp, and area behind the ears that she says has been present
for several months. The rash is mildly itchy but not painful. The patient has not used new skin care or cosmetic
products. Other medical conditions include hypertension and type 2 diabetes mellitus. Physical examination
findings are as shown in the exhibit. Which of the following is the most appropriate management for this patient's
rash?

A) Oral terbinafine

B) Topical emollients only

C) Topical high-potency corticosteroid

D) Topical ketoconazole

E) Topical metronidazole
Explanation
Correct Answer:

D) Topical ketoconazole

Seborrheic dermatitis in adults

• Erythematous, pruritic plaques with greasy scales


Clinical features
• Scalp, central face, ears, chest

• CNS disease (eg, Parkinson disease)


Risk factors
• HIV

• Topical antifungals (eg, selenium sulfide, ketoconazole)


Treatment • Topical glucocorticoids
• Topical calcineurin inhibitors (eg, pimecrolimus)

This patient has seborrheic dermatitis (SD), which presents with erythematous patches and plaques with greasy
scales. SD most often affects the central face (eg, eyebrows, nasolabial folds), ears, and scalp (mild cases are
referred to as "dandruff"); less common locations include the chest, back, axillae, and inguinal area. SD is most
common in the first year of life and again in adulthood. Most cases are idiopathic, but it is more common or
extensive in patients with HIV and CNS disorders (eg, Parkinson disease, stroke). The diagnosis is usually
apparent based on clinical findings.

The pathogenesis of SD is unclear, but Malassezia species may play a role, possibly by altering the lipid
composition on the skin surface and promoting an inflammatory process. Topical antifungal agents (eg,
ketoconazole, selenium sulfide) are effective in treating this condition.

(Choice A) Oral antifungals (eg, terbinafine, itraconazole) are commonly used to treat extensive or refractory
fungal skin infections, such as tinea facialis, which presents with erythematous, scaly, annular plaques on the face.
They can be considered for patients with widespread or refractory SD, but topical agents are preferred due to
established effectiveness and lesser toxicity.

(Choice B) Topical emollients are most often used for xerosis (dry skin), which causes scaling but generally occurs
in indistinct patches on the extremities and trunk. Erythema is atypical, and symptoms on the face are uncommon.

(Choice C) Topical anti-inflammatory agents (eg, corticosteroids, calcineurin inhibitors [eg, pimecrolimus]) can be
used in SD to reduce erythema and itching. However, use of high-potency corticosteroids on the face is not
recommended due to possible dermal atrophy.

(Choice E) Topical metronidazole is appropriate for papulopustular rosacea, which also causes erythematous
patches and plaques on the central face. However, scaling is not seen.

Educational objective:
Seborrheic dermatitis presents with erythematous patches and plaques with greasy scales, most commonly on the
central face, ears, and scalp. It is more common in patients with HIV and CNS disorders (eg, Parkinson disease).
Diagnosis is made clinically, and topical antifungals (eg, ketoconazole, selenium sulfide) are effective for treatment.

Reference
• Diagnosis and treatment of seborrheic dermatitis.
Question #104

A 22-year-old woman comes to the office due to a skin disorder. The patient has a 5-year history of patchy
depigmentation primarily affecting the hands, feet, and face. Some of her lesions have resolved, but overall she
has experienced a slowly progressive course. The patient has no associated pain, itching, or erythema. Medical
history is unremarkable, and her only regular medication is an oral contraceptive. Vital signs are normal. Physical
examination findings are shown in the image below with similar lesions on the face and hands. Examination is
otherwise normal.
Which of the following diseases is most likely associated with this patient's skin condition?

A) Hashimoto thyroiditis

B) HIV infection

C) Hyperparathyroidism
D) Type 2 diabetes mellitus

E) Zollinger-Ellison syndrome
Explanation
Correct Answer:

A) Hashimoto thyroiditis

Vitiligo

Clinical • Depigmented macules on acral areas & extensor surfaces; face commonly affected
manifestations • Lesions may be symmetric, dermatomal, or unilateral

• Most cases progress gradually


• Repigmentation is spontaneous in 10%-20% of cases
Clinical course
• Increased incidence of other autoimmune disorders (eg, lupus, thyroid disease,
pernicious anemia, Addison disease)

• Limited disease: topical corticosteroid


Treatment • Extensive/unresponsive disease: oral corticosteroid, topical calcineurin inhibitor,
phototherapy

This patient has well-demarcated, depigmented macules involving the face and distal extremities consistent with
vitiligo. Vitiligo is caused by regional destruction of melanocytes, most likely due to an autoimmune etiology.
Genetic and environmental factors may also play a role.

Vitiligo can occur as an isolated disorder but is often associated with other autoimmune conditions (eg, pernicious
anemia, primary adrenal insufficiency, alopecia areata, rheumatoid arthritis, Sjogren syndrome). Autoimmune
thyroid disease (eg, chronic lymphocytic [Hashimoto] thyroiditis, Graves disease) is especially common, and many
experts advise patients with vitiligo be assessed for thyroid function.

(Choice B) HIV infection increases the risk for tinea versicolor, which typically presents with hypopigmented
macules. In contrast to vitiligo, which has completely depigmented lesions, the macules in tinea versicolor retain
partial pigmentation. They also have a predilection for the chest and upper back rather than the face and distal
extremities.

(Choices C and E) Hyperparathyroidism, gastrin-producing pancreatic tumors (Zollinger-Ellison syndrome), and


pituitary tumors can occur together in the multiple endocrine neoplasia type 1 syndrome. This disorder is due to
mutations in the MEN1 gene. It is not autoimmune and not associated with vitiligo.

(Choice D) Type 1 diabetes mellitus is due to autoimmune destruction of pancreatic beta cells and is associated
with vitiligo. Type 2 diabetes is not an autoimmune disorder and not associated with vitiligo.

Educational objective:
Vitiligo is an autoimmune condition characterized by areas of depigmentation due to destruction of melanocytes. It
can be associated with other autoimmune conditions, such as pernicious anemia, autoimmune thyroid disease, type
1 diabetes mellitus, primary adrenal insufficiency, hypopituitarism, and alopecia areata.

Reference
• Positivity rates of antithyroid antibody, antinuclear antibody and thyroid peroxidase antibody in different
types of vitiligo.
Question #105

A 28-year-old woman comes to the office due to worsening of a skin rash on her back and arms for the past 2
weeks. She has had no associated pain or pruritus. During her teenage years, the patient had comedonal acne on
her face, which resolved with topical medications. She was diagnosed with systemic lupus erythematosus 2 years
ago and is on oral prednisone due to a recent exacerbation with pain and swelling in the small joints of the hands.
The patient also takes hydroxychloroquine and as-needed nonsteroidal anti-inflammatory drugs. She does not use
tobacco, alcohol, or illicit drugs. She works in a dry-cleaning facility. The patient is sexually active with her
boyfriend and has an intrauterine device for contraception. Temperature is 37 C (98.6 F), blood pressure is 130/86
mm Hg, and pulse is 78/min. Skin examination shows uniform-appearing, 1- to 3-mm, erythematous papules
across her back, shoulders, and upper arms. There is mild symmetric synovitis in the hands and wrists, but the
remainder of the physical examination is unremarkable. Which of the following is the most likely cause of this
patient's skin rash?

A) Acne vulgaris

B) Chlorinated chemical exposure

C) Cutaneous manifestation of systemic lupus erythematosus

D) Disseminated gonococcal infection

E) Herpes simplex virus infection

F) Medication adverse effect


Explanation
Correct Answer:

F) Medication adverse effect


This patient has an acute papular inflammatory rash consistent with drug-induced acne, also called steroid-
induced folliculitis or steroid acne. Drug-induced acne is typically associated with systemic glucocorticoids but
can also be caused by topical glucocorticoids, glucocorticoid-sparing agents (eg, azathioprine, cyclosporine), and a
variety of other drugs (eg, anticonvulsants, antituberculous drugs).

In contrast to acne vulgaris (which displays lesions in various stages of development and typically occurs on the
face in adolescents), drug-induced acne can be seen in any age group, is characterized by monomorphic papules
without associated comedones, and commonly involves the upper back, shoulders, and upper arms (Choice A).
Some patients may have small pustules, but lesions are found in the same stage of development, and scarring
cystic and nodular lesions are not seen. Drug-induced acne does not respond to typical acne treatment but
improves rapidly on discontinuation of the offending agent.

(Choice B) Chloracne is a severe skin disorder caused by exposure to halogenated hydrocarbons (eg,
occupational exposure to the pollutant dioxin). It is characterized by inflammatory nodules and large comedones
affecting the head, neck, and axillae.

(Choice C) Common dermal manifestations of systemic lupus erythematosus include an erythematous malar rash,
discoid plaques and ulcers, and generalized photosensitivity.

(Choice D) Disseminated gonococcal infection typically presents with a vesiculopustular rash, tenosynovitis, and
migratory polyarthralgias. Most patients are febrile (unlike in this case), and the skin lesions are predominantly
found on the distal extremities and usually last only a few days.

(Choice E) Herpes simplex virus infection presents with clusters of vesicles and ulcers on an erythematous base,
typically in the perioral or genital areas.

Educational objective:
Drug-induced acne is a common side effect of systemic glucocorticoids and is characterized by monomorphic
papules without associated comedones, cysts, or nodules. Drug-induced acne does not respond to typical acne
treatment but improves rapidly on discontinuation of the offending agent.

Reference
• Drug-induced acneiform eruption.
Question #106

A 30-year-old, otherwise healthy man comes to the office due to a rash on the legs, chest, and abdomen. The rash
has been present for many years, and the patient is frustrated that he has not "outgrown" it. The affected skin is
dry, scaly, and slightly pruritic. Symptoms are worse in the winter, with improvement noted during the summer. The
patient has tried various over-the-counter lotions without relief. His mother and younger sister have the same
condition. Skin examination of the legs is shown in the exhibit, with the chest and abdomen showing similar
findings. Which of the following is the most appropriate next step in management of this patient's rash?

A) Oral corticosteroid

B) Oral retinoid

C) Topical alpha hydroxy acid

D) Topical calcipotriene
Explanation
Correct Answer:

C) Topical alpha hydroxy acid

Ichthyosis vulgaris

• Loss of function in filaggrin gene


Pathophysiology ◦ Impaired epidermal barrier
◦ Reduced skin moisturization

• Onset in infancy/early childhood


Clinical features • Diffuse, scaly skin with mild pruritus
• Worse on extensor extremities, spares intertriginous areas

• Keratosis pilaris
Associated
• Palmar hyperlinearity
conditions
• Atopic disease

• Clinical findings
Diagnosis
• Biopsy if uncertain: reduced/absent granular layer in epidermis
• Long baths to remove scales
Therapy • Moisturization
• Keratolytics (eg, urea, alpha-hydroxy acid, salicylic acid)

This patient has diffuse cutaneous scaling, a finding consistent with ichthyosis vulgaris. Ichthyosis vulgaris is a
chronic, inherited disorder that is caused by mutations of the filaggrin gene (FLG) and leads to epidermal
hyperplasia and defective keratinocyte desquamation. The skin is rough and dry, with fish-like scales. As in this
patient, ichthyosis vulgaris typically affects the trunk and extensor extremities, particularly the legs, while sparing the
flexures and face. Palmar hyperlinearity is also common. The symptoms typically first appear in infancy or
childhood and worsen during the winter due to decreased ambient humidity.

Treatment of ichthyosis vulgaris includes periodic long baths to remove the scales and frequent application of
moisturizers to smooth and hydrate the skin. Moisturizers containing keratolytics (eg, alpha hydroxy acid [eg,
lactic acid], urea, salicylic acid) not only soften keratin, but also loosen and facilitate the shedding of scales.

(Choice A) Oral corticosteroids are used temporarily to treat severe, refractory atopic dermatitis (chronic
inflammation reflecting skin barrier dysfunction) but are not recommended for ichthyosis vulgaris (epidermal
hyperplasia and defective keratinocyte desquamation). Atopic dermatitis presents acutely as intensely pruritic,
erythematous papules and vesicles; chronically, atopic dermatitis presents as lichenified, scaly plaques typically on
the flexural surfaces in adults. Diffuse, fish-like scales are not characteristic.

(Choice B) Oral retinoids (eg, isotretinoin) are used to treat nodulocystic acne; they reduce sebum production,
normalize follicular epithelial proliferation and desquamation, and reduce colonization by Cutibacterium acnes.
Because they primarily target the pilosebaceous unit, oral retinoids are not used to treat ichthyosis vulgaris.

(Choice D) Topical calcipotriene, a vitamin D analogue with possible immunomodulatory effects and
hypoproliferative effects on keratinocytes, is used to treat psoriasis either alone or with topical corticosteroids.
Psoriasis typically affects the extensor surfaces (eg, elbows, knees) and presents as well-defined, erythematous
plaques with silvery scales, features absent in this patient.

Educational objective:
Ichthyosis vulgaris is a chronic, inherited disorder characterized by rough and dry skin with fish-like scales.
Treatment includes topical keratolytics, such as alpha hydroxy acid (eg, lactic acid), urea, and salicylic acid to help
remove the scales. In addition, frequent bathing and application of moisturizers are also recommended.

Reference
• Ichthyosis vulgaris: the filaggrin mutation disease.
Question #107

A 34-year-old woman comes to the physician for evaluation of skin lesions. For the past year, she has noticed
darkening and thickening of the skin over her neck and groin areas. These areas occasionally feel itchy. Her blood
pressure is 130/80 mm Hg. Skin examination findings are shown in the image below.
Similar hyperpigmented, velvety lesions are found on the axilla and groin. This patient's condition is most likely
associated with which of the following?

A) Addison's disease

B) Hemochromatosis

C) Liver malignancy

D) Pellagra

E) Polycystic ovarian syndrome


Explanation
Correct Answer:

E) Polycystic ovarian syndrome

Skin conditions & associated diseases

Skin conditions Associated conditions

• Insulin resistance
• Acanthosis nigricans
• Gastrointestinal malignancy

• Insulin resistance
• Multiple skin tags • Pregnancy
• Crohn disease (perianal)

• Porphyria cutanea tarda


• Cutaneous leukocytoclastic vasculitis
• Hepatitis C
(palpable pupura) secondary
to cryoglobulinemia

• Dermatitis herpetiformis • Celiac disease


• Sudden-onset, severe psoriasis
• Recurrent herpes zoster • HIV infection
• Disseminated molluscum contagiosum

• HIV infection
• Severe seborrheic dermatitis
• Parkinson disease

• Explosive onset multiple, itchy


• Gastrointestinal malignancy
seborrheic keratoses

• Pyoderma gangrenosum • Inflammatory bowel disease

This image shows the typical presentation of acanthosis nigricans (AN), which is characterized by hyperkeratotic,
hyperpigmented plaques with a classic "velvety" texture. Flexural areas, particularly the axilla, groin, and posterior
neck, are the most common locations affected. Depending on the etiology, AN can be divided into benign and
malignant forms:

• Benign AN is typically seen in younger individuals and is associated with insulin-resistant states (eg,
diabetes mellitus, obesity, polycystic ovarian syndrome). Increased levels of insulin and/or insulin-like
growth factors are thought to stimulate epidermal and dermal proliferation. Similarly, skin tags
(acrochordons), which are pedunculated outgrowths of normal skin, are also commonly present on regions
affected by AN.
• Malignant AN (Choice C) is associated with underlying neoplasms, especially of the gastrointestinal and
genitourinary tracts. The sudden appearance of such skin changes in middle-aged or elderly patients is
suggestive of underlying malignancy. In addition, these patients are not obese (but instead may have lost
weight), and lesions can occur in uncommon areas (eg, mucous membranes, palms, soles).
(Choice A) Hyperpigmentation of the skin in primary adrenal insufficiency (Addison's disease) is generalized.
However, it is more prominent in areas exposed to friction (eg, elbows, knees) and light (eg, face, dorsum of
hands). The other findings of Addison's disease are vitiligo, dehydration, and hypotension. This patient's normal
blood pressure and characteristic skin findings are not suggestive of Addison's disease.

(Choice B) Diabetes mellitus and hyperpigmentation of the skin are features of hemochromatosis ("bronze
diabetes"), but the characteristic skin findings are different from those of acanthosis. The coloration in
hemochromatosis is typically brownish or bronze, and at times slate gray. The hyperpigmentation also tends to be
more prominent on sun-exposed skin, especially the face.

(Choice D) Niacin deficiency (pellagra) typically presents with photosensitive dermatitis, erythematous tongue,
diarrhea, vomiting, and neurologic symptoms (eg, insomnia, dementia, confusion).

Educational objective:
Acanthosis nigricans is characterized by symmetrical, hyperpigmented, velvety plaques in the axilla, groin, and
neck. It is associated with insulin resistance states (eg, diabetes mellitus, polycystic ovarian syndrome) in
younger patients and gastrointestinal malignancy in older individuals.
Question #108

A 23-year-old woman comes to the office due to hair loss. Two weeks ago, she suddenly developed loss of large
numbers of hair fibers when washing or brushing her hair. The patient has had 2 pregnancies; the first led to a
spontaneous abortion at 13 weeks, and the second led to a normal term vaginal delivery 4 months ago. She
subsequently experienced severe postpartum unipolar major depression requiring psychiatric hospitalization.
Current medications include fluoxetine and olanzapine. The patient is not breastfeeding. She does not use
tobacco, alcohol, or illicit drugs. Vital signs are normal. Examination shows diffuse hair loss with no erythema or
scaling in the scalp. When tracts of hair are pulled firmly, >20% of fibers are pulled out. Which of the following is
the most likely diagnosis for this patient's hair loss?

A) Alopecia areata

B) Androgenetic alopecia

C) Seborrheic dermatitis

D) Telogen effluvium

E) Tinea capitis

F) Trichorrhexis nodosa

G) Trichotillomania
Explanation
Correct Answer:

D) Telogen effluvium

Telogen effluvium

• Acute, diffuse, noninflammatory hair loss


Clinical findings • Scalp & hair fibers appear normal
• Hair shafts easily pulled out (hair pull test)

• Severe illness, fever, surgery


• Pregnancy, childbirth
Triggers
• Emotional distress
• Endocrine & nutritional disorders

• Address underlying cause


Management
• Reassurance (self-limited disorder)

This patient has acute, diffuse, noninflammatory hair loss consistent with telogen effluvium (TE), one of the
most common causes of hair loss in adults. Hair follicles pass through 3 phases:

• Growth phase (anagen; 90% of follicles)


• Transformative phase (catagen; <1%)
• Rest/shedding phase (telogen; 10%).
In TE, follicles undergo a widespread shift into the rest/shedding phase, with cessation of growth and subsequent
shedding. It is often triggered by a stressful event, such as weight loss, pregnancy, major illness or surgery, or
psychiatric trauma.

Patients with TE have widespread thinning of hair but the scalp and hair shafts appear normal. In the hair pull test,
small tracts of hair (50-60 fibers) are pulled firmly; extraction of >10%-15% of fibers is abnormal and suggests TE.
It is a self-limited disorder but may take up to a year to resolve completely.

(Choice A) Alopecia areata is an autoimmune disorder characterized by circumscribed patches of hair loss. Hair
shafts show narrowing close to the surface and may be broken off.

(Choice B) Androgenetic alopecia causes uneven hair loss in a characteristic pattern. Men have thinning at the
frontotemporal hairline and vertex; women predominantly have thinning at the vertex and sides with preservation of
the hairline.

(Choice C) Seborrheic dermatitis causes erythema and scaling at the ears, eyebrows, nasolabial folds, and scalp.

(Choice E) Tinea capitis is a superficial dermatophyte infection characterized by pruritic, scaly, erythematous
patches with hair loss. Proximal hair shafts may be visualized as small dots in the area of hair loss.

(Choice F) Trichorrhexis nodosa is characterized by fragility of hair with breaking of strands. It can be congenital
or acquired (eg, excessive heat, hair dyes, salt water). Close inspection shows fractured strands with splitting of
fibers.

(Choice G) Trichotillomania is a behavioral disorder characterized by compulsive pulling of hair. It presents with
irregular patches of hair loss and broken fibers, and typically affects the front and sides of the scalp, eyebrows, and
eyelashes.

Educational objective:
Telogen effluvium causes acute, diffuse, noninflammatory hair loss. It is often triggered by a stressful event, such
as weight loss, pregnancy, major illness or surgery, or psychiatric trauma. Patients have widespread thinning of hair
but the scalp and hair shafts appear normal. It is a self-limited disorder but may take up to a year to resolve
completely.
Reference
• Hair loss: common causes and treatment.
Question #109

A 27-year-old woman comes to the office due to a rash. Four weeks ago, she had an acute onset of numerous
painful, dark lesions on her legs. The patient has never had a similar rash in the past and otherwise feels well.
Medical history is unremarkable except for a recent episode of cystitis, which was treated with trimethoprim-
sulfamethoxazole. Examination shows multiple tender, indurated lesions on the lower extremities, as shown in the
exhibit. Which of the following is the most likely diagnosis of this patient's skin lesions?

A) Erythema nodosum

B) Leukocytoclastic vasculitis

C) Polyarteritis nodosa

D) Serum sickness

E) Stevens-Johnson syndrome

F) Thrombotic thrombocytopenic purpura


Explanation
Correct Answer:

A) Erythema nodosum

Erythema nodosum

• Tender, indurated, erythematous nodules


Clinical features
• Most common on anterior legs

• Infections (eg, Streptococcus)


• Inflammatory bowel disease
Etiology
• Sarcoidosis
• Medications (eg, antibiotics, oral contraceptives)

Pathologic findings • Septal panniculitis without vasculitis

• Spontaneous resolution (weeks)


Natural history
• Residual hyperpigmentation

This patient has tender, indurated lesions on the lower extremities consistent with erythema nodosum (EN). EN is
characterized by 2- to 3-cm, tender, erythematous or violaceous nodules. It usually occurs on the shins but can
develop elsewhere on the legs trunk, face, or the upper extremities. Women are more commonly affected.
EN results from a delayed-type hypersensitivity reaction to various antigens. Common triggers include infection
(eg, Streptococcus), inflammatory bowel disease (eg, Crohn disease), sarcoidosis, and malignancy. It can also
be triggered by medications, such as penicillins, sulfonamides (eg, trimethoprim-sulfamethoxazole in this patient),
and oral contraceptives. The diagnosis is based primarily on clinical findings, although, if necessary, biopsy of the
nodules can reveal septal panniculitis without vasculitis. EN is a self-limited condition that typically resolves within
weeks, but residual hyperpigmentation is common.

(Choice B) Cutaneous small-vessel vasculitis (ie, leukocytoclastic vasculitis) can be triggered by antibiotics but
presents as painful, raised, nonblanching, petechial or purpuric lesions (ie, palpable purpura).

(Choice C) Polyarteritis nodosa may cause nodular lesions resembling erythema nodosum, but it is usually
associated with systemic (eg, fever, arthralgias, weight loss) and extradermal (eg, renal insufficiency, abdominal
pain, mononeuritis multiplex) manifestations.

(Choice D) Serum sickness is an immune reaction against blood products or antigens from a nonhuman species
(eg, chimeric mouse antibodies [rituximab, infliximab]). Certain medications (including sulfonamides) can induce a
serum sickness-like reaction, but this typically manifests with fever, joint pain, and an urticarial or vasculitis-like rash.

(Choice E) Stevens-Johnson syndrome is characterized by erythematous bullae and desquamation, often


associated with mucosal involvement and a febrile prodrome.

(Choice F) Thrombotic thrombocytopenic purpura presents with a petechial rash associated with fever, renal
failure, abdominal pain, and neurologic manifestations.

Educational objective:
Erythema nodosum is a delayed hypersensitivity reaction characterized by 2- to 3-cm, tender, erythematous
nodules, most often on the shins. Common triggers include infection, inflammatory bowel disease, sarcoidosis, and
malignancy. It can also be triggered by medications, such as penicillins, sulfonamides, and oral contraceptives.

Reference
• Erythema nodosum.
Question #110

A 40-year-old man comes to the office due to a rash on his arms. The rash has been present for 3 weeks. The
lesions are very itchy; he has applied several over-the-counter lotions, but they did not help. The patient has not
participated in any unusual outdoor activity recently. He takes no medications on a regular basis. The patient has
used intravenous illicit drugs in the past but not in the last 15 years. Vital signs are normal. Skin findings are shown
in the exhibit. The remainder of the examination shows no abnormalities. Laboratory results are as follows:

Liver function studies


Total bilirubin 1.1 mg/dL
Alkaline phosphatase 70 U/L
Aspartate aminotransferase (SGOT) 87 U/L
Alanine aminotransferase (SGPT) 99 U/L

Which of the following is the most likely diagnosis?

A) Atopic dermatitis

B) Dermatitis herpetiformis

C) Erythema multiforme

D) Lichen planus

E) Tinea corporis
Explanation
Correct Answer:

D) Lichen planus

Lichen planus

• 5 "Ps": pruritic, purple/pink, polygonal papules & plaques


Clinical findings
• Lacy, white network of lines (Wickham striae)

• Hepatitis C
Disease associations
• Medications: ACE inhibitors, thiazide diuretics

• Chronic symptoms
Natural history • Formation of lesions at sites of trauma (Köbner reaction)
• Spontaneous resolution within 2 years

• Topical high-potency glucocorticoids (eg, betamethasone)


Treatment
• Widespread lesions: systemic glucocorticoids, phototherapy

This patient has lichen planus (LP), an immunologically mediated skin disorder affecting middle-aged adults. LP
typically develops symmetrically on the extremities, most commonly on the flexural surfaces of the wrists and
ankles; the nails, oral mucous membranes, and genitalia can also be involved. The skin lesions are described by
the 5 Ps: Pruritic, Purple/pink, Polygonal Papules and Plaques. They often have white, lacy markings known as
Wickham striae and can form along lines of minor trauma (Köbner reaction).

The diagnosis of LP is based primarily on examination, but biopsy can be performed for confirmation. LP is often
seen in association with hepatitis C; screening may be warranted in patients with a history of illicit drug use or
elevated hepatic aminotransferase levels. A similar disorder can be seen with a number of common medications
(eg, ACE inhibitors, thiazide diuretics). Although LP resolves spontaneously within 2 years, high-potency
topical glucocorticoids (eg, betamethasone) are often used to accelerate healing and alleviate pruritus.

(Choice A) Atopic dermatitis causes a pruritic rash affecting flexor surfaces but usually presents in childhood.
Most patients have other atopic disorders (eg, allergic rhinitis, asthma).

(Choice B) Dermatitis herpetiformis causes an intensely pruritic rash on the elbows, back, knees, and buttocks.
Lesions are typically herpetiform (ie, small, erythematous papules and vesicles); the purplish papules in this patient
are not typical of dermatitis herpetiformis.

(Choice C) Erythema multiforme presents with target-like plaques with central clearing that begin on the extensor
surfaces of the distal extremities and spread centripetally; such plaques are not seen in this patient. Oral lesions
and a febrile prodrome are also common.

(Choice E) Tinea corporis presents with round or ovoid, annular plaques with a scaly border and central clearing,
not the purple polygonal plaques seen in this patient.

Educational objective:
Lichen planus presents with pruritic, purple/pink, polygonal papules and plaques on the extremities, most commonly
on the flexural surfaces of the wrists and ankles. Lesions may also occur on the oral mucosa, nails, and genitalia.
Lichen planus is often associated with hepatitis C. Although this disorder resolves spontaneously, topical
glucocorticoids can be used to accelerate healing and alleviate pruritus.

Reference
• Dermatologic manifestations of chronic hepatitis C infection.
Question #111

A 62-year-old man comes to the office due to worsening itchy, dry skin on the hands. For 2 years, he has
experienced thickened, cracking skin at the dorsum of both hands that has extended to the fingers. The patient
tried moisturizing lotions, with partial relief, but the symptoms relapsed if he did not apply the lotions regularly.
Medical history is notable for seasonal allergies and an occasional herpes rash on his upper lip following upper
respiratory infections. He works as a janitor and does not use tobacco, alcohol, or illicit drugs. Examination shows
scaly skin and erythema on the fingers, finger webs, and dorsum of both hands, as shown in the exhibit. No other
skin lesions are seen. Which of the following is the most likely diagnosis?

A) Contact dermatitis

B) Herpetic whitlow

C) Psoriasis

D) Scabies

E) Tinea manuum
Explanation
Correct Answer:

A) Contact dermatitis
This patient has chronic irritant contact dermatitis (ICD), likely because of frequent handwashing and
occupational exposure to cleaning solutions. ICD can be triggered by water, detergents, solvents, and other
chemicals, as well as physical irritants such as metals, wood, and fiberglass. These irritants disrupt the epidermal
barrier through chemical or physical (eg, microtrauma) damage to keratinocytes, increasing epidermal permeability
and water loss. An inflammatory response follows at the site of exposure.

A single exposure to strong irritants can induce erythematous papules, edema, vesicles, and bullae (ie, acute ICD).
In contrast, repeated exposure to mild irritants (eg, water, dilute detergents) over time causes chronic ICD (as seen
in this patient), which presents with dry scales, lichenification, and fissuring.

Treatment of chronic ICD includes avoidance of irritants (eg, using barrier gloves), frequent application of
emollients to restore the skin barrier and reduce water loss, and topical corticosteroids to decrease inflammation.

(Choice B) Herpetic whitlow is an acute herpes simplex virus infection of the fingers caused by inoculation of the
virus (eg, from concurrent herpes labialis) into broken skin. Lesions (ie, painful vesicles on an erythematous base)
typically resolve spontaneously in 1-2 weeks, not years.

(Choice C) Psoriasis can affect the hands, presenting as red plaques with thick, silvery scales on the palms and
dorsal fingers. However, patients with hand psoriasis usually also have psoriatic nail disease or additional psoriatic
lesions elsewhere on the body.

(Choice D) Scabies can affect the hands but presents acutely as small, intensely pruritic papules and irregular
burrows. More diffuse skin involvement is seen in crusted scabies, but this typically occurs in immunocompromised
patients (eg, advanced HIV disease). Chronic scaling and improvement with emollients are not characteristic.

(Choice E) Tinea manuum is a dermatophyte infection of the hands that presents as pruritic, scaly patches on the
palms or as annular plaques with a raised border on the dorsum. It may resemble chronic ICD but does not
improve with emollients alone, is usually unilateral, and most commonly occurs in patients with concurrent tinea
pedis (via autoinoculation from scratching).

Educational objective:
Chronic irritant contact dermatitis is characterized by scaling, lichenification, and fissuring and commonly affects the
hands. It is caused by exposure to chemical (eg, detergents, solvents) or physical (eg, metals, fiberglass) irritants
that disrupt the epidermal barrier, triggering an inflammatory response. Treatment includes emollients, topical
corticosteroids, and irritant avoidance.
Reference
• Irritant contact dermatitis.
Question #112

A 35-year-old woman comes to the office due to a weeklong history of rash in the bilateral axilla. The rash is not
painful and does not itch. Two weeks ago, the patient fractured her right ankle while jogging and has since
ambulated with crutches. Medical history includes prediabetes. She smokes a pack of cigarettes a day. Her only
medication is acetaminophen with codeine as needed for ankle pain. BMI is 33 kg/m2. Vital signs are normal. On
examination, many red papules and pustules are present in both axillae. There is no axillary lymphadenopathy or
mass. The remainder of the skin examination is normal. Which of the following most likely contributed to this
patient's skin condition?

A) Allergic contact dermatitis

B) Candida albicans infection

C) Irritant contact dermatitis

D) Mechanical irritation of pilosebaceous follicles

E) Opiate intake
Explanation
Correct Answer:

D) Mechanical irritation of pilosebaceous follicles

This patient has nonpruritic papules and pustules indicative of an acneiform rash. Whereas acne vulgaris typically
occurs on the face and back, this patient's lesions are in the axillae. This presentation is most consistent with acne
mechanica, which can occur anywhere pressure or friction is applied. In addition to crutches and other medical
devices, common causes of acne mechanica include bra straps, turtlenecks, sports padding, helmets, and heavy
backpacks.

Recurrent mechanical pressure damages pilosebaceous follicles, leading to obstruction, comedone formation,
and acneiform lesions. Eliminating the source of pressure is usually adequate to allow clearing of acne
mechanica.

(Choice A) Allergic contact dermatitis is a type IV hypersensitivity reaction to external allergens (eg, poison ivy,
nickel). It manifests as erythema, swelling, vesicles, bullae, and lichenification (in chronic cases), typically with
significant pruritus. The asymptomatic papules and pustules in this patient suggest a different etiology.

(Choice B) Candidal intertrigo occurs in intertriginous areas where moisture and occlusion create an ideal
environment for Candida albicans infection. However, the infection typically causes beefy red patches with satellite
red papules or macules, not an acneiform eruption.

(Choice C) Irritant contact dermatitis is a nonimmunologic skin reaction to chemical agents (eg, solvents,
detergents). It usually presents with erythema, edema, vesicles, bullae, and lichenification (in chronic cases),
typically with a burning or stinging sensation. Irritant contact dermatitis typically affects exposed skin (eg, hands);
axillary involvement is uncommon.

(Choice E) Opiates can cause itching and urticaria through their effect on certain opioid receptors and through
non–IgE mediated mast cell activation. Papules and pustules without itching are not characteristic.
Educational objective:
Acne mechanica is an acneiform eruption caused by pressure-related damage to pilosebaceous follicles. It can
result from pressure from crutches, bra straps, turtlenecks, sports padding, helmets, and heavy backpacks.

Reference
• Inner thigh friction as a cause of acne mechanica.
Question #113

A 68-year-old man comes to the office due to a rash, which he first noticed several months ago. The rash is not
painful or itchy but is slowly enlarging. The patient has tried an over-the-counter, low-potency corticosteroid cream
without significant relief. Medical history includes hypertension and hypothyroidism. Vital signs are within normal
limits. Skin examination findings are shown in the exhibit. There are no other skin lesions or enlarged lymph
nodes. Which of the following is the best next step in management of this patient?

A) Punch biopsy of the lesion

B) Skin scraping for potassium hydroxide test

C) Topical high-potency corticosteroid therapy

D) Topical imiquimod therapy

E) Topical vitamin D analogue therapy


Explanation
Correct Answer:

A) Punch biopsy of the lesion

Squamous cell carcinoma of skin

• Sun/ultraviolet, ionizing radiation exposure


Risk factors • Chronic scars/wounds/burn injuries
• Immunosuppression

• Scaly plaques/nodules
Clinical features • ± Ulceration
• SCC in situ: well-demarcated patches/plaques

• Punch, shave, or excisional biopsy


Diagnosis
• Dysplastic/anaplastic keratinocytes

Invasive SCC SCC in situ

• Excision with 4- to 6-mm margins • Excision with 4- to 6-mm margins


Treatment
• Mohs micrographic surgery • C&E
• Cryotherapy
• Topical 5-FU, imiquimod
5-FU = 5-fluorouracil; C&E = curettage & electrodesiccation; SCC = squamous cell carcinoma.

This patient has an asymptomatic, scaly, well-demarcated, erythematous plaque that is slowly enlarging. Along
with its location on sun-exposed skin, the lesion is concerning for squamous cell carcinoma (SCC) in situ
(Bowen disease).

Unlike plaque psoriasis and chronic atopic dermatitis, which may have a similar appearance, SCC in situ is typically
asymptomatic, not pruritic. In addition, it often develops in regions with other signs of solar damage (eg, actinic
elastosis, mottled hyperpigmentation). Less commonly, it also may arise in areas with chronic inflammation,
scarring, or previous infection with high-risk human papillomavirus (eg, perianal skin, genitalia). SCC in situ is
confined to the epidermis, presenting as well-demarcated, scaly, red macules or plaques; when it becomes invasive
SCC, nodules and ulcerations are typically seen.

Timely diagnosis with a biopsy (eg, shave, punch) is important because SCC in situ can become invasive and
metastasize. The choice of biopsy is influenced by lesion characteristics (eg, flat vs nodular, size), anatomic
location, patient characteristics (eg, bleeding diathesis), and clinician preferences.

(Choice B) Potassium hydroxide microscopy of skin scrapings is used to confirm tinea corporis and often can
visualize fungal hyphae. However, tinea corporis typically presents as a pruritic, annular plaque with central
clearing (not present in this patient).

(Choice C) Topical high-potency corticosteroids are used to treat severe, chronic atopic dermatitis, which typically
manifests as pruritic, lichenified plaques. This patient's lesion is asymptomatic and has worsened despite topical
corticosteroid therapy, making atopic dermatitis unlikely.

(Choice D) Imiquimod is a topical immune response modifier that can be used to treat SCC in situ. However, a
biopsy to confirm the diagnosis is recommended prior to treatment.

(Choice E) Topical vitamin D analogues (eg, calcipotriene) are used to treat plaque psoriasis, which presents with
pruritic, erythematous plaques with silvery scales. This patient's lesion is asymptomatic and has worsened despite
topical corticosteroid application, making psoriasis unlikely.
Educational objective:
Squamous cell carcinoma in situ of the skin (Bowen disease) presents as a slowly enlarging, well-demarcated,
scaly, erythematous patch or plaque that is asymptomatic. It most commonly occurs on sun-exposed skin. Due to
its invasive and metastatic potential, timely diagnosis with a biopsy is recommended.

Reference
• Diagnosis and treatment of basal cell and squamous cell carcinoma.
Question #114

A 49-year-old man comes to the office for evaluation of a pruritic rash on his back. The rash has been present for a
few months and has been unrelenting. The itching disturbs his sleep, and over-the-counter antihistamines have not
helped. Medical history is significant for primary hypertension, for which he takes hydrochlorothiazide and lisinopril.
Vital signs are normal. The patient's skin findings are shown in the image below:
Which of the following is the most likely diagnosis?

A) Atopic dermatitis
B) Dermatitis herpetiformis

C) Lichen planus

D) Pityriasis rosea

E) Psoriasis

F) Tinea versicolor
Explanation
Correct Answer:

C) Lichen planus

Lichen planus

• 5 "Ps": pruritic, purple/pink, polygonal papules & plaques


Clinical findings
• Lacy, white network of lines (Wickham striae)

• Hepatitis C
Disease associations
• Medications: ACE inhibitors, thiazide diuretics

• Chronic symptoms
Natural history • Formation of lesions at sites of trauma (Köbner reaction)
• Spontaneous resolution within 2 years

• Topical high-potency glucocorticoids (eg, betamethasone)


Treatment
• Widespread lesions: systemic glucocorticoids, phototherapy

This patient has a pruritic rash consistent with lichen planus (LP). The skin lesions in LP are characterized by the
"5 P's": pruritic, purple/pink, polygonal papules and plaques. The lesions often show white, lacy markings known
as Wickham striae and can form along lines of minor trauma or scratching (Köbner phenomenon).
LP is usually idiopathic, although it is often seen in patients with hepatitis C. However, drug-induced LP (lichenoid
drug reaction) has been associated with a number of medications, including ACE inhibitors, thiazide diuretics,
beta blockers, and hydroxychloroquine. Drug-induced LP typically has a more diffuse presentation than idiopathic
LP, which is frequently limited to the flexor surfaces of the wrists and ankles, oral mucosa, and genitalia.

Treatment of drug-induced LP includes topical high-potency glucocorticoids and discontinuation of the offending
medication. Some patients may develop recurrent bouts despite drug discontinuation, and residual
hyperpigmentation is common.

(Choice A) Atopic dermatitis causes a pruritic rash affecting the flexor surfaces of the extremities. It usually
presents in childhood, and most patients have a history of other atopic disorders (eg, allergic rhinitis, asthma).

(Choice B) Dermatitis herpetiformis causes pruritic, erythematous papules, vesicles, and bullae on the extensor
surfaces of the elbows, back, knees, and buttocks. It is generally seen in patients with gluten-sensitive enteropathy
(celiac disease).

(Choice D) Pityriasis rosea begins with an erythematous annular lesion on the trunk ("herald patch"), followed by
an outbreak of macules and papules across the neck, trunk, and proximal limbs. It is often preceded by a
nonspecific viral prodrome. Pityriasis is most common in adolescents and young adults and usually resolves within
4-6 weeks.

(Choice E) Psoriasis is characterized by sharply demarcated, erythematous plaques with a thick, silvery scale
involving the scalp, extensor surfaces of the knees and elbows, neck, and back.

(Choice F) Tinea versicolor is a common superficial fungal dermatosis that usually presents with patches of altered
pigmentation and faint scaling. The lesions are generally flat, and pruritis is mild or absent.

Educational objective:
Drug-induced lichen planus (lichenoid drug reaction) is associated with a number of medications, including ACE
inhibitors, thiazide diuretics, beta blockers, and hydroxychloroquine. Whereas idiopathic lichen planus occurs most
commonly at the wrists and ankles, drug-induced lichen planus can have a more diffuse presentation. Treatment
includes topical glucocorticoids and discontinuation of the suspected medication.
Reference
• Severe drug-induced dermatoses.
Question #115

A 19-year-old woman comes to the office for evaluation of chronic acne. Over the last 6 months, the patient has
tried several over-the-counter acne treatments without improvement. She has a history of attention-deficit
hyperactivity disorder and mild intellectual disability. Her mother has a seizure disorder, and her maternal
grandmother died of kidney failure. Vital signs are within normal limits. Facial examination findings are shown
below.
There are several elliptical, hypopigmented macules on the trunk. Which of the following is the most likely
underlying cause of this patient's condition?

A) Congenital adrenal hyperplasia


B) Fabry disease

C) Neurofibromatosis type 1

D) Tuberous sclerosis

E) Von Hippel-Lindau disease


Explanation
Correct Answer:

D) Tuberous sclerosis

Tuberous sclerosis complex

• Mutation (inherited or de novo) in TSC1 or TSC2 gene


Pathophysiology
• Autosomal dominant

• Dermatologic
◦ Ash-leaf spots
◦ Angiofibromas of the malar region
◦ Shagreen patches
• Neurologic
Clinical features ◦ CNS lesions (eg, subependymal tumors)
◦ Epilepsy (eg, infantile spasms)
◦ Intellectual disability
◦ Autism & behavioral disorders (eg, hyperactivity)
• Cardiovascular: rhabdomyomas
• Renal: angiomyolipomas

• Tumor screening
Surveillance ◦ Regular skin & eye examinations
◦ Serial MRI of the brain & kidney
◦ Baseline echocardiography & serial ECG
• Baseline electroencephalography
• Neuropsychiatric screening

This patient has facial angiofibromas characteristic of tuberous sclerosis (TS), a neurocutaneous disorder in
which an autosomal dominant or de novo mutation in the TS complex gene results in benign tumors of the skin,
brain, and other organs. Features vary in severity and can present from infancy through adulthood.

Skin lesions occur in most patients with TS and may be the presenting sign. Angiofibromas are red or flesh-
colored, fibrous papules that are most commonly located in the malar region and often increase in number during
childhood. They may be mistaken for acne that is unresponsive to typical treatments, as seen here. Other common
skin findings include hypopigmented macules (ash-leaf spots), as seen in this patient, and shagreen patches
(thickened, leathery skin).

In patients with characteristic dermatologic findings of TS, a brain MRI is required to evaluate for associated CNS
lesions (eg, hamartomas). Brain hamartomas can cause seizures (as seen in this patient's mother) or manifest
more subtly with behavioral/cognitive disorders (eg, attention-deficit hyperactivity disorder, intellectual disability),
as seen in this patient. Diagnosis can be confirmed by genetic testing.

(Choice A) Late-onset, or nonclassic, congenital adrenal hyperplasia (partial 21-hydroxylase deficiency) can
present with hyperandrogenism (eg, severe acne, hirsutism) in adolescent girls. This patient's skin examination
shows angiofibromas rather than acne.

(Choice B) Fabry disease, a lysosomal storage disorder due to alpha-galactosidase A deficiency, typically causes
neuropathic pain and glomerular disease. Associated skin findings include telangiectasias and angiokeratomas,
which are not seen here.

(Choice C) Neurofibromatosis type 1 is an autosomal dominant neurocutaneous disorder characterized by axillary


freckling, hyperpigmented café-au-lait macules (blue arrow), and neurofibromas (black arrow). This patient's
hypopigmented macules and facial angiofibromas make TS more likely.
(Choice E) Von Hippel-Lindau syndrome is an autosomal dominant disease that causes various tumors, such as
hemangioblastoma, pheochromocytoma, renal cell carcinoma, and pancreatic tumors. Cutaneous findings are not
associated with von Hippel-Lindau syndrome.

Educational objective:
Tuberous sclerosis is an autosomal dominant neurocutaneous disorder characterized by typical skin (eg,
angiofibromas, ash-leaf spots) and neurologic (eg, seizures, behavioral/cognitive disorders) findings.

Reference
• Tuberous sclerosis.

• Cutaneous angiofibroma.
Question #116

A 48-year-old woman comes to the office due to a 4-day history of pruritic rash on her legs. The lesions initially
worsened rapidly over 1-2 hours, and some lesions resolved by the end of the day. The lesions recurred several
times over the past 2-3 days, with each episode lasting 6-8 hours. Temperature is 36.1 C (97 F), blood pressure is
138/77 mm Hg, and pulse is 78/min. Skin examination findings are shown in the image below. The remainder of
the physical examination is within normal limits.
Which of the following is the most likely diagnosis?

A) Atopic dermatitis

B) Contact dermatitis

C) Guttate psoriasis
D) Pityriasis rosea

E) Urticaria

F) Viral exanthem
Explanation
Correct Answer:

E) Urticaria

Acute urticaria

• Medications, insect stings


Etiologies • Infections, rheumatologic diseases
• Many cases are idiopathic

Pathophysiology • Mast cells release histamine → dermal edema

• Pruritic, erythematous plaques (ie, wheals)


Clinical features • Each wheal lasts <24 hr, urticaria duration <6 weeks
• Diffuse or localized

• No workup for 1st episode


• Allergy testing if specific trigger (eg, food) is
Evaluation suspected
• Laboratory testing/biopsy if systemic symptoms are
present
• 2nd-generation H1 antihistamines
Treatment & • Add H2 antihistamines or systemic corticosteroids if
prognosis severe
• Two-thirds of cases self-resolve

This patient has acute urticaria, intensely pruritic, well-circumscribed erythematous plaques (ie, wheals) caused
by mast cell activation in the superficial dermis. Urticaria lesions can be round or serpiginous and up to several
centimeters in diameter. Although individual lesions develop over minutes to hours and resolve within 24 hours,
additional lesions may continue to erupt for up to 6 weeks. Angioedema, which is caused by mast cell activation in
the deeper dermal and subcutaneous tissues (eg, face, lips, hands, buttocks), sometimes accompanies urticaria.

Acute urticaria usually represents an IgE-mediated allergic reaction (eg, medications, insect bites, food); additional
etiologies include infection (eg, bacterial, viral) and systemic illness (eg, rheumatologic disorder, mastocytosis);
many cases are idiopathic. Initial management includes a second-generation H1 blocker (eg, cetirizine). A single
episode typically requires no additional evaluation, although laboratory evaluation and biopsy are recommended for
episodes that are unresponsive to therapy or accompanied by systemic symptoms (eg, arthritis).

(Choice A) Acute atopic dermatitis presents with erythematous papules and vesicles, not wheals, that are typically
distributed over the flexural areas (eg, antecubital fossae) rather than extensor surfaces. Individual lesions persist
for days to weeks and are not transient.

(Choice B) Contact dermatitis is caused by contact with irritants or allergens that acutely cause pruritic,
erythematous papules or vesicles that last for days to weeks. Rapid resolution within a day is not typical.

(Choice C) Guttate (drop-like) psoriasis presents with an acute eruption of erythematous papules and 2- to 15-mm
plaques with fine scales. Individual lesions are persistent and do not resolve within 24 hours.

(Choice D) Pityriasis rosea is a self-limited condition likely caused by a viral etiology. It begins as a solitary herald
patch that appears pink on light skin and violet to brown on darker skin. The appearance of this patch is followed by
eruption of multiple ovoid papules and plaques with a collarette of scales on the trunk and proximal extremities.
Individual lesions usually take 4-6 weeks to resolve.

(Choice F) Viral exanthem is usually associated with fever and other symptoms (eg, cough, diarrhea) of a viral
illness. It presents with nonpruritic, erythematous macules and papules that last for days.

Educational objective:
Acute urticaria presents with intensely pruritic, well-circumscribed erythematous plaques (ie, wheals). Individual
lesions resolve within 24 hours, but additional lesions may erupt for up to 6 weeks. Angioedema sometimes
accompanies urticaria.

Reference
• Acute and chronic urticaria: evaluation and treatment.
Question #117

A 28-year-old man comes to the office asking for antibiotics to treat a "sinus infection." He reports recurrent
episodes of nasal congestion, rhinorrhea, and dry cough. The patient has used over-the-counter allergy medicines
with some relief but continues to feel uncomfortable and has difficulty concentrating at work. He does not have
shortness of breath, chest pain, or ocular symptoms. The patient has no prior history of allergies or asthma but had
eczema during childhood. He does not use tobacco, alcohol, or illicit drugs. His temperature is 37 C (98.6 F), blood
pressure is 120/78 mm Hg, and pulse is 76/min. Physical examination shows a transverse nasal crease, swollen
and pale nasal turbinates, and a clear nasal discharge. There is no maxillary sinus tenderness. The posterior
pharyngeal wall has a "cobblestone" appearance. Breath sounds are normal with no added sounds. Which of the
following is the most effective therapy for this patient's condition?

A) Inhaled beta agonist

B) Intranasal decongestant

C) Intranasal glucocorticoid

D) Oral antibiotics

E) Oral leukotriene modifier


Explanation
Correct Answer:

C) Intranasal glucocorticoid

Allergic rhinitis

• Rhinorrhea, nasal congestion, sneezing, nasal itching


Symptoms • Cough secondary to postnasal drip
• Ocular itching & tearing

• "Allergic shiners" (infraorbital edema & darkening)


• "Allergic salute" (transverse nasal crease)
Physical examination • Pale, bluish, enlarged turbinates
• Pharyngeal cobblestoning
• "Allergic facies" (high-arched palate, open-mouth breathing)

• Allergen avoidance
Treatment
• Intranasal corticosteroids

This patient has typical features of allergic rhinitis including nasal congestion, clear rhinorrhea, and pale,
edematous nasal mucosa. Patients may also have nasal creases, pharyngeal cobblestoning, conjunctival edema,
or thick, green nasal discharge. During peak allergy seasons, patients can experience systemic (eg, fever) or
neuropsychiatric (eg, fatigue, irritability) symptoms. Atopic disorders (eg, allergic rhinitis, "hay fever," asthma,
eczema) frequently cluster in families, but individual patients usually experience only a subset of these
manifestations.

Allergen avoidance can mitigate the symptoms of allergic rhinitis but is not always possible, and even low-level
exposure can trigger bothersome symptoms. Glucocorticoid nasal sprays (eg, fluticasone, mometasone) are the
most effective single agents. An initial response can be seen within several hours of administration, but maximal
benefits may require continuous treatment for several days or weeks. Nonsedating oral antihistamines (eg,
loratadine, cetirizine), antihistamine (eg, azelastine) or cromolyn nasal sprays, and leukotriene modifiers (eg,
montelukast) are less effective but can be considered based on the patient's symptoms and drug tolerances
(Choice E).

(Choice A) Inhaled beta agonists are indicated for patients with asthma. Patients with wheezing should be
evaluated for possible asthma, but a dry cough alone is a nonspecific symptom.

(Choice B) Nasal decongestant sprays can reduce mucosal edema but are less effective overall than
glucocorticoids and can cause rebound congestion (rhinitis medicamentosa).

(Choice D) This patient has no fever or sinus tenderness to suggest bacterial sinusitis.

Educational objective:
Glucocorticoid nasal sprays are the most effective single agent for allergic rhinitis, although maximal benefits may
require continuous treatment for several days or weeks. Oral antihistamines, antihistamine or cromolyn nasal
sprays, and leukotriene modifiers can be considered based on the patient's symptoms and drug tolerances.

Reference
• Mechanisms and clinical implications of glucocorticosteroids in the treatment of allergic rhinitis.
Question #118

A 70-year-old man is brought to the office by his wife. She reports that he "no longer listens to me" when she
speaks and that this has worsened over the past 2 years. The patient has moved his living room chair farther away
from her and closer to the TV. He does not like to join her for their weekly dinners with friends anymore. The
patient reports he has trouble understanding his wife, especially when the environment is noisy. He says he got
tired of asking his friends to repeat themselves and so does not enjoy eating out with them. Medications include
hydrochlorothiazide, lisinopril, and aspirin. The patient does not smoke or use alcohol. Which of the following is the
most likely cause of his condition?

A) Cochlear hair cell loss

B) Drug toxicity

C) Loss of dopaminergic neurons

D) Loss of mobility of the ossicles

E) Scarring of the tympanic membranes


Explanation
Correct Answer:

A) Cochlear hair cell loss

This elderly patient brought in for evaluation of social withdrawal has trouble understanding speech when there is
competing background noise. This is a common scenario for patients with presbycusis (age-related hearing loss),
a bilateral, progressive, sensorineural hearing loss (SNHL).

Presbycusis affects >50% of adults by age 75 and is likely related to cochlear hair cell loss and cochlear neuron
degeneration. Age-related brain atrophy also likely contributes (likely due to increased information processing
times) and may explain the disproportionate problems with speech discrimination in older compared with younger
patients with SNHL. High frequencies are affected first, making it more difficult for patients to understand higher-
pitched voices (eg, women, children). Tinnitus often develops and is typically described as continuous ringing,
rushing, or buzzing.

Patients with presbycusis often hear well in one-on-one conversations in a quiet room; however, even a small
amount of competing noise impairs hearing (eg, speech discrimination). Therefore, education of the patient and
family is imperative. In addition to limiting background noise, families should be counseled to look directly at
patients when speaking to them. Amplification (eg, hearing aids) may be beneficial.

(Choice B) Ototoxic medications (eg, aminoglycosides, loop diuretics) can also cause bilateral, symmetric SNHL
and may compound age-related hearing loss. This patient's medications (at typical doses) are not normally
implicated in hearing loss.

(Choice C) Parkinson disease leads to loss of dopaminergic neurons in the substantia nigra. It may present with
social isolation and withdrawal, but this patient's history of difficulty understanding speech is more consistent with
hearing loss.

(Choice D) Otosclerosis is caused by abnormal bone deposition, resulting in stiffening of the ossicular chain. It is
usually found in younger patients and presents with conductive hearing loss. In contrast to this patient, those with
conductive hearing loss may have paradoxically improved speech understanding in background noise (paracusis of
Willis).

(Choice E) Tympanosclerosis (scarring of the tympanic membrane) can cause stiffening that may lead to
conductive hearing loss in severe cases. However, patients typically have a history of otitis media (typically chronic)
or previous tympanostomy tubes. Chalky, white patches are often seen on the tympanic membrane.

Educational objective:
Sensorineural hearing loss that occurs with aging is called presbycusis. It presents with bilateral, high-frequency
hearing loss and is related to damage to sensorineural structures in the inner ear.
Question #119

A 25-year-old man comes to the office for evaluation of a painless mass in his mouth. The patient has had the
mass for many years and has had no growth or other changes in the mass over that time. He has had occasional
sinus infections and was in a motor vehicle accident several years ago in which he sustained a concussion, but he
is otherwise healthy. The patient smokes a pack of cigarettes a day and drinks 2 or 3 alcoholic beverages on the
weekends. Vital signs are normal. Examination of the mass is seen below:
The mass is immobile, nontender, and has a hard consistency. The maxillary and frontal sinuses are nontender,
and there is no cervical lymphadenopathy. Which of the following is the most likely cause of this patient's mass?

A) Congenital anomaly
B) Infectious sequelae

C) Malignant transformation

D) Palatal fracture

E) Vascular malformation
Explanation
Correct Answer:

A) Congenital anomaly

This patient has a chronic mass on his hard palate consistent with torus palatinus (TP), a benign bony growth (ie,
exostosis) located at the midline suture of the hard palate. It is thought to be caused by both genetic and
environmental factors and is more common in women and Asian individuals. TP can be congenital or develop
later in life. Similar lesions at the lingual surface of the mandible are termed "tori mandibulari."

TPs are usually <2 cm in size but can gradually enlarge over time. They are typically asymptomatic and are
frequently ignored by the patient but noted by clinicians (or family members) when examining the mouth for
unrelated reasons. However, the thin epithelium overlying the bony growth may ulcerate with minor trauma of the
oral cavity and heal slowly due to poor vascular supply. The diagnosis is usually obvious on clinical grounds.
Surgery is indicated for patients in whom the mass becomes symptomatic, interferes with speech or eating, or
causes problems with the fitting of dentures later in life.

(Choices B and C) Oral squamous cell carcinoma can be caused by smoking and excessive alcohol use; it can
also be a late complication of human papillomavirus infection. However, oral cancer more commonly presents as a
progressive ulcerating lesion located eccentrically on soft structures (eg, tongue, lips) rather than the center of the
hard palate. Rarely, paranasal sinus infections with atypical organisms (eg, blastomycosis, mucormycosis) can
erode into the palate, but this typically occurs in patients with immune suppression (eg, uncontrolled diabetes) and
would present acutely.

(Choice D) Chronic complications of palatal fracture include dental malocclusion, temporomandibular joint
dysfunction, and complicated wounds. However, palatal fractures are rare, are usually associated with other
obvious skull fractures, and are not a major cause of TP.

(Choice E) Maxillofacial vascular malformations are congenital abnormalities that are present at birth. They
present as soft, dusky lesions that grow with the patient and may bleed intermittently.
Educational objective:
Torus palatinus (TP) is a benign bony growth (exostosis) located on the midline suture of the hard palate. It can be
congenital or develop later in life. TP is typically chronic and asymptomatic, and the diagnosis is usually clinically
evident. Surgery is indicated if the mass becomes symptomatic, interferes with speech or eating, or causes
problems with the fitting of dentures.

Reference
• Current status of the torus palatinus and mandibularis.
Question #120

A 28-year-old woman comes to the office due to persistent nasal congestion and stuffiness. The patient has a
constant sensation of dripping in the back of her throat and states that food has tasted bland to her recently. A year
ago, she came to the emergency department due to severe wheezing after taking naproxen for menstrual
cramping. The patient has no history of head trauma. Family history is significant for asthma in her sister. She
does not smoke cigarettes but occasionally smokes marijuana. Which of the following is the most likely cause of
this patient's current symptoms?

A) Allergic rhinitis

B) Fungal rhinosinusitis

C) Nasal polyposis

D) Nasopharyngeal angiofibroma

E) Perforated nasal septum

F) Pyogenic granuloma
Explanation
Correct Answer:

C) Nasal polyposis

This patient has persistent nasal symptoms and previously had a severe wheezing reaction after taking naproxen, a
nonsteroidal anti-inflammatory drug (NSAID). This presentation is consistent with aspirin-exacerbated respiratory
disease (AERD), the clinical triad of:

• asthma (often severe and presenting in adulthood).


• bronchospasm or nasal congestion following the ingestion of aspirin or NSAIDs.
• chronic rhinosinusitis with nasal polyposis.

This patient's current symptoms of bland-tasting food (likely due to anosmia) and recurrent nasal discharge/
congestion are typical in patients with nasal polyps, which are identified on nasal examination as bilateral, gray,
glistening mucoid masses.

AERD is classified as a pseudoallergy because it is not mediated by IgE. Although the pathophysiology is not fully
elucidated, it seems to be related to overproduction of leukotrienes. Exposure to NSAIDs leads to acute
reactions. In addition to typical management of asthma (eg, inhalers) and chronic rhinosinusitis (eg, intranasal
saline and glucocorticoids), leukotriene-modifying agents are often prescribed for patients with AERD.

AERD is typically a clinical diagnosis but can be confirmed with an aspirin challenge. Aspirin desensitization is
used for patients with worsening disease or those who require aspirin or NSAID therapy for comorbid conditions
(eg, arthritis, coronary artery disease).

(Choices A and B) Allergic rhinitis and allergic fungal rhinosinusitis can both cause all the nasal symptoms this
patient is experiencing. However, nasal symptoms in combination with a respiratory reaction to an NSAID is
characteristic of AERD.

(Choice D) Juvenile nasal angiofibroma is a rare, benign tumor of the nasopharynx that can cause nasal
obstruction and nasal drainage but also usually results in epistaxis. In addition, it occurs almost exclusively in
teenage boys.

(Choice E) Patients with a perforated nasal septum often have nasal congestion with excess crusting. They may
also have noisy breathing (typically a whistling noise on inspiration). However, patients usually have bleeding and
typical risk factors (eg, nasal surgery, cocaine use) and do not have exacerbations after using NSAIDs.

(Choice F) Pyogenic granulomas are benign vascular tumors that can occur on the nasal septum and cause
congestion. However, bleeding is typically prominent because they are very friable, and there is no association with
NSAID use.

Educational objective:
Aspirin-exacerbated respiratory disease is a clinical condition defined by the triad of asthma, bronchospasm or
nasal congestion following the ingestion of aspirin or nonsteroidal anti-inflammatory drugs, and chronic
rhinosinusitis with nasal polyposis.

Reference
• Revision rates after endoscopic sinus surgery: a recurrence analysis.

• Pathogenesis of aspirin-exacerbated respiratory disease and reactions.


Question #121

A 35-year-old woman comes to the office due to persistent rhinorrhea. The patient has clear drainage from the right
side of her nose that increases when she has a bowel movement or bends over to pick something off the floor. On
examination, the external nose appears normal, the turbinates are pink, and the septum is straight. There is a small
amount of clear fluid from her right nostril. The remainder of the examination is normal. Which of the following
additional history is most important for establishing a diagnosis in this patient?

A) Family history

B) Illicit drug use

C) Occupation

D) Prior seasonal allergies

E) Recent head trauma

F) Use of decongestant sprays


Explanation
Correct Answer:

E) Recent head trauma

Cerebrospinal fluid rhinorrhea

• Accidental trauma (most common)


Etiology • Surgical trauma (eg, sinus surgery)
• Nontraumatic (eg, elevated intracranial pressure)

Clinical • Unilateral watery rhinorrhea with salty or metallic taste


presentation • Possible complication: meningitis

• Test for CSF-specific proteins (β-2 transferrin, β-trace protein)


Evaluation • Imaging (with intrathecal contrast)
• Endoscopy (± intrathecal fluorescein dye)

• Bed rest, head of bed elevation, avoidance of straining


Management • Lumbar drain placement
• Surgical repair
CSF = cerebral spinal fluid.

Clear, unilateral rhinorrhea that increases at times of relatively increased intracranial pressure (eg, bending over,
bowel movements) is suspicious for cerebrospinal fluid (CSF) rhinorrhea. Patients also often report a salty or
metallic taste. Most cases occur after head trauma, especially with fracture of the skull base (eg, cribriform plate,
temporal bone). CSF rhinorrhea can be evident immediately after the trauma or may have a delayed presentation
(days to months) in which rhinorrhea is noted after acute edema resolves.

Testing the nasal discharge for CSF-specific proteins (eg, beta-2 transferrin, beta-trace protein) is diagnostic.
Imaging or endoscopic nasal examination can be used to localize the precise site of the defect. Patients typically
require inpatient management for bed rest, head-of-bed elevation, and frequent neurologic evaluation because
they are at risk for meningitis due to nasal flora contamination of the CSF. If the CSF rhinorrhea does not resolve
with these measures, further interventions (eg, lumbar drain, operative repair) are necessary.

(Choices A and D) A personal or family history of atopy may suggest allergic rhinitis, which would present with
bilateral (rather than unilateral) rhinorrhea and pale or bluish, boggy, edematous turbinates. Accompanying
symptoms of sneezing, itching, and watery eyes are also typically present.

(Choices B and F) Intranasal cocaine and nasal decongestants cause vasoconstriction. Overuse can result in
bilateral (rather than unilateral) rhinorrhea and severe "rebound" nasal congestion (eg, rhinitis medicamentosa).
Examination often shows swollen, erythematous turbinates. Tissue destruction from vasoconstriction caused by
intranasal cocaine typically occurs at the septum (resulting in septal perforations) rather than at the skull base.

(Choice C) Patients can be exposed to a variety of substances at work that result in allergic, irritant-induced, or
corrosive rhinitis, leading to rhinorrhea. However, such rhinorrhea would be bilateral, and symptoms would increase
during times of exposure (ie, at work) rather than with head positioning or straining.

Educational objective:
Clear, unilateral rhinorrhea that increases at times of relatively increased intracranial pressure (eg, bending over,
bowel movements) is suspicious for cerebrospinal fluid rhinorrhea, which is most often caused by head trauma and
can result in meningitis.
Reference
• Traumatic CSF leaks of the anterior skull base.
Question #122

A 55-year-old man comes to the office for follow-up due to recurrent episodes of jaw pain. The patient's most
recent episode was a week ago. He had pain that worsened with eating, accompanied by a tender mass under the
left jaw and fever. All symptoms resolved within a few days with oral antibiotic treatment. This was the third
episode in the past year. Which of the following is the most likely underlying reason for this patient's recurrent
episodes of jaw pain?

A) Giant cell arteritis

B) Sialolithiasis

C) Sjögren syndrome

D) Temporomandibular joint disease

E) Trigeminal neuralgia
Explanation
Correct Answer:

B) Sialolithiasis

This patient with episodes of jaw pain and swelling (eg, tender mass) that are exacerbated by eating, are
associated with fever, and resolve with antibiotics likely has recurrent sialadenitis (salivary gland infection).
Sialadenitis is caused by salivary stasis that leads to retrograde seeding of bacteria (eg, Staphylococcus aureus,
oral flora) from the oral cavity. Salivary stasis can be seen in elderly postoperative patients or in those with
obstruction of the outflow duct. In this patient with no other known risk factors, salivary stasis is likely due to ductal
obstruction by salivary stones (sialolithiasis).

Salivary stones most often form in the submandibular, rather than the parotid, glands because saliva from the
submandibular glands has a higher mucus content and the duct travels against gravity, resulting in decreased
salivary flow. These stones are most commonly made of calcium and are visible on CT scan.

Sialolithiasis typically presents with pain and swelling exacerbated by eating. In some patients, management
with hydration, moist heat, milking of the gland, and sialagogues is sufficient to control symptoms and even flush out
the stone. However, in other patients (such as this one), ductal obstruction leads to secondary infections
(sialadenitis). Although these episodes typically respond to nonsteroidal anti-inflammatory drugs and antibiotics,
patients with recurrent infections should be referred to otolaryngology for stone removal.

(Choice A) Giant cell arteritis can present with jaw pain exacerbated by eating. However, pain due to jaw
claudication would worsen during chewing and improve shortly after cessation. In addition, pain would not resolve
with antibiotic treatment, and a tender submandibular mass would not be present.

(Choice C) Sjögren syndrome can present with salivary gland enlargement (typically painless) due to lymphocytic
infiltration of the glands. However, patients are typically women, it more often affects the parotid glands, and
secretions are significantly decreased, resulting in both xerostomia and dry eyes.

(Choice D) Temporomandibular joint disorder can present with episodic pain exacerbated by eating with
intervening asymptomatic periods. However, patients would not have fever or a submandibular mass.

(Choice E) Trigeminal neuralgia can present with recurrent pain in the jaw (V2 and V3 branches of the trigeminal
nerve) often triggered by minor stimuli like eating or brushing teeth. However, episodes last several seconds rather
than a few days, and fever and a submandibular mass would not be present.

Educational objective:
Salivary stones occur most often in the submandibular glands and can present with recurrent sialadenitis due to
obstruction of the duct.

Reference
• Human immunodeficiency virus and salivary gland pathology: an update.

• Algorithms for treatment of salivary gland obstructions.


Question #123

A 27-year-old woman, gravida 1 para 0 at 30 weeks gestation, comes to the office due to increasing difficulty
hearing, especially on the right side. She has had no ear pain or discharge. The patient has no prior medical
conditions; the pregnancy has been uncomplicated other than an episode of acute cystitis 8 weeks ago, which was
treated with antibiotics. She takes no medications aside from a multivitamin. The patient eats a balanced diet and
does not use tobacco or alcohol. Vital signs are within normal limits. Bilateral tympanic membranes are normal
with no middle ear effusion. A tuning fork held over the middle of the forehead sounds louder in the right ear.
Tuning fork sounds are better heard with bone conduction on the right side and with air conduction on the left. The
remainder of the neurologic examination shows no abnormalities. Which of the following is the most likely cause of
this patient's current symptoms?

A) Chronic otitis media

B) Medication ototoxicity

C) Ménière disease

D) Otosclerosis

E) Presbycusis

F) Vestibular schwannoma (acoustic neuroma)


Explanation
Correct Answer:

D) Otosclerosis

Interpretation of Rinne & Weber tests

Rinne result Weber result

Normal AC > BC in both ears Midline

Conductive BC > AC in affected ear,


Lateralizes to affected ear
hearing loss AC > BC in unaffected ear

Sensorineural Lateralizes to unaffected ear,


AC > BC in both ears
hearing loss away from affected ear

Mixed BC > AC in affected ear, Lateralizes to unaffected ear,


hearing loss AC > BC in unaffected ear away from affected ear

AC = air conduction; BC = bone conduction.


Hearing loss is classified as either conductive (impaired transmission of sound to the inner ear) or sensorineural
(involving the cochlea or auditory nerve), which can be distinguished with tuning fork examination. The Rinne test
compares air conduction (tuning fork near the ear) versus bone conduction (tuning fork on the mastoid bone). With
normal hearing, air-conducted sound is perceived as louder than bone-conducted sound. The Weber test is
performed by placing the tuning fork on the middle of the forehead equidistant from both ears. The sound carried
by bone conduction in this manner is normally heard equally in both ears; vibration heard louder in one ear is
abnormal.

• Sensorineural hearing loss causes lateralization to the unaffected ear because the inner ear is
unimpaired on that side and therefore can better hear the sound.
• Conductive hearing loss causes lateralization to the affected ear because the conductive deficit masks
the ambient noise in the room, allowing the sound to be better heard.

Conductive hearing loss can be caused by disorders of the external auditory canal (eg, cerumen impaction, otitis
externa), tympanic membrane (eg, perforation), middle ear space (eg, otitis media, cholesteatoma), or the ossicular
chain (eg, otosclerosis).

This young woman with conductive hearing loss and a normal otoscopic examination likely has otosclerosis.
Otosclerosis results from an imbalance of bone resorption and deposition that leads to stiffening and ultimately
fixation of the stapes, which dampens the transmission of sound energy from the tympanic membrane to the
cochlea. Otosclerosis may progress during pregnancy. It is inherited in an autosomal dominant pattern with
incomplete penetrance. Treatment involves hearing amplification or surgical reconstruction of the stapes.

(Choice A) Although chronic otitis media may cause conductive hearing loss, it is typically accompanied by ear
pain and/or fullness, which were not reported by this patient. Examination of the tympanic membranes would show
retraction of the tympanic membrane or fluid in the middle ear.

(Choice B) Aminoglycoside antibiotics can cause sensorineural (rather than conductive) hearing loss. In addition,
they are more commonly used in pyelonephritis rather than cystitis.

(Choices C, E, and F) Although Ménière disease (episodic vertigo, fluctuating hearing loss), presbycusis
(progressive hearing loss in advanced age), and vestibular schwannoma (CN VIII tumor) would all have normal
otoscopic examinations, they would cause sensorineural hearing loss rather than conductive.
Educational objective:
Otosclerosis causes fixation of the stapes, which results in conductive hearing loss. It often presents in young
women and may progress during pregnancy.

Reference
• The etiology of otosclerosis: a combination of genes and environment.

• Controversies in the evaluation and management of otosclerosis.


Question #124

A 28-year-old man comes to the office for evaluation of white lesions over the tongue that he recently noticed while
brushing his teeth. The lesions are not painful or itchy. Review of systems is negative for dysphagia and positive
for both fatigue and unintentional weight loss over the past 4 months. The patient smokes a pack of cigarettes daily
and does not drink alcohol. Vital signs are within normal limits. BMI is 22 kg/m2. Examination shows bilateral
corrugated, adherent plaques located on the lateral tongue surfaces; the lesions cannot be scraped off. Dentition is
normal without any caries or decayed tooth. There are multiple enlarged cervical lymph nodes bilaterally that are
mobile and nontender. What is the most appropriate next step in management?

A) Obtain HIV testing

B) Obtain lymph node biopsy

C) Perform laryngoscopy

D) Prescribe oral nystatin swish and swallow

E) Recommend tobacco cessation and reevaluate in 3 months


Explanation
Correct Answer:

A) Obtain HIV testing

This patient has multiple painless, white mucosal lesions on the lateral tongue surfaces that cannot be scraped off.
In addition, he has systemic symptoms including fatigue, unintentional weight loss, and cervical adenopathy. In a
patient his age (ie, young adult), this presentation is suspicious for oral hairy leukoplakia (OHL), a condition that is
caused by intense Epstein-Barr virus replication and that is highly associated with HIV infection.

OHL classically appears as multiple homogeneous lesions along the lateral tongue surfaces and has a
corrugated (ie, folded or wrinkled) appearance that was originally described as "hairy." When this classic
appearance is present (as in this patient), OHL can usually be distinguished from other white mucosal lesions
without the need for lesional biopsy. Because OHL occurs almost exclusively in patients with significant
immunodeficiency, especially HIV, workup should include HIV testing. OHL does not require specific treatment;
lesions often resolve with appropriate treatment of the underlying immunodeficiency (eg, antiretroviral treatment).

(Choices B and C) Cervical lymph node biopsy and laryngoscopy are frequent components of the workup of
mucosal head and neck cancer and would be appropriate if this patient's mucosal lesions were malignant.
However, his lesions are classic in appearance for OHL, and his lymphadenopathy is diffuse (multiple, mobile nodes
bilaterally). This is more likely related to systemic illness (ie, HIV); therefore, HIV testing should be performed first.
If HIV testing is negative, lesional biopsy (vs laryngoscopy or cervical lymph node biopsy) is a reasonable next step.

(Choice D) Nystatin is used to treat oral candidiasis (ie, thrush). Although thrush can present with multiple white
lesions in the oral cavity, they often have an erythematous base and can be scraped off with gauze or a tongue
depressor.

(Choice E) Tobacco cessation and close follow-up is appropriate for biopsy-proven benign oral leukoplakia, a
potentially premalignant lesion distinct from OHL despite similarities in the name. Leukoplakia is more common in
older patients with a long history of tobacco and alcohol use. This patient's young age; systemic symptoms; and
classic, corrugated lesions on the lateral tongue surfaces are more consistent with OHL.

Educational objective:
Oral hairy leukoplakia typically presents as multiple white lesions on the lateral tongue with a distinct corrugated
appearance. Because it occurs almost exclusively in patients with significant immunodeficiency, HIV testing should
be performed, especially in patients with signs of systemic illness.
Question #125

A 23-year-old woman comes to the office due to nasal congestion, rhinorrhea, and postnasal drip for more than a
year. Symptoms fluctuate in intensity and are worsened by cigarette smoke and strong fragrances. Although
symptoms do not change with the season, they worsen when the patient walks from indoors to outdoors on a cold
day. She has no eye or ear symptoms, itching, wheezing, or skin rash. The patient has been using over-the-
counter oral loratadine and pseudoephedrine without significant improvement. Medical history is unremarkable. On
inspection, the nasal mucosae appear boggy and erythematous, and rhinorrhea is clear. The conjunctivae are
normal. The lungs are clear to auscultation. Which of the following is the best next step in management of this
patient?

A) Imaging of the sinuses

B) Intranasal glucocorticoids

C) Nasal cytology

D) Radioallergosorbent test

E) Serum IgE level


Explanation
Correct Answer:

B) Intranasal glucocorticoids

Nonallergic rhinitis Allergic rhinitis

• Watery rhinorrhea, sneezing, eye


• Nasal congestion, rhinorrhea, sneezing,
symptoms
postnasal drainage
• Earlier age of onset
• Later onset (age >20) common
Clinical • Identifiable allergen or seasonal pattern
• No obvious allergic trigger
features • Pale/bluish nasal mucosa
• Perennial symptoms (may worsen with
• Associated with other allergic disorders
season changes)
(eg, eczema, asthma, eustachian tube
• Erythematous nasal mucosa
dysfunction)

• Mild: intranasal antihistamine or


• Intranasal glucocorticoids
Treatment glucocorticoids
• Antihistamines
• Moderate to severe: combination therapy

This patient has rhinitis triggered by strong smells and sudden exposure to cold air. She lacks eye symptoms or
triggers consistent with allergies (eg, seasonal variation), and the symptoms do not improve with oral
antihistamines. This presentation is consistent with nonallergic rhinitis (NAR), also known as vasomotor rhinitis.

In contrast to patients with allergic rhinitis, patients with NAR usually cannot identify specific allergic triggers (eg,
pollen counts), and they typically have negative testing for aeroallergens. However, many have very specific
behavioral triggers (eg, walking into cold air, eating). In addition, patients with NAR typically lack the sneezing
and allergic conjunctivitis (eg, itchy eyes, injected conjunctivae) that classically accompany allergic rhinitis.

First-line treatment for both allergic rhinitis and NAR is intranasal glucocorticoids. Intranasal antihistamines (eg,
azelastine, olopatadine) or intranasal ipratropium bromide can also be used. Combination therapy is often required
for patients with moderate to severe symptoms.

(Choice A) Imaging can be helpful for patients with chronic sinusitis, which can present with nasal congestion.
However, patients with chronic sinusitis also typically have mucopurulent drainage, facial pain or pressure, and a
decreased sense of smell.

(Choice C) Although nasal cytology in patients with NAR usually reveals predominant eosinophils and increased
numbers of mast cells, NAR is a clinical diagnosis, so nasal cytology is not required.

(Choice D) A serum radioallergosorbent test (RAST) can be helpful for identifying triggers in patients with allergic
rhinitis. However, it is not usually helpful in patients with NAR because the results are typically negative; therefore,
skin allergy or serum RAST testing is usually considered only if there is inadequate response to empiric treatment.

(Choice E) Serum IgE levels are usually elevated in allergic rhinitis but not in NAR.

Educational objective:
Nonallergic rhinitis usually presents with nasal congestion and rhinorrhea without specific allergic triggers. Patients
should be treated with intranasal glucocorticoids; intranasal antihistamines can be added if needed.

Reference
• Chapter 6: nonallergic rhinitis.
Question #126

A 27-year-old primigravida at 8 weeks gestation is found to have a thyroid nodule during her initial prenatal visit.
She has fatigue and frequent nausea with vomiting. The patient has no heat or cold intolerance and no skin
changes. She has no dysphagia to solids or liquids, although she has been eating more carbohydrates since
becoming pregnant. Medical history is otherwise not significant. The patient does not use tobacco, alcohol, or illicit
drugs. Temperature is 36.7 C (98.1 F), blood pressure is 110/70 mm Hg, pulse is 86/min, and respirations are 18/
min. Physical examination shows a small, 1.5-cm nodule in her right thyroid gland. Pelvic examination reveals a
slightly enlarged uterus consistent with 8 weeks gestation. Serum TSH is normal. Ultrasound of her thyroid reveals
a 1.5-cm hypoechoic nodule in her right thyroid lobe with irregular margins, internal microcalcifications, and internal
vascularity. Which of the following is the next most appropriate step in management of this patient?

A) Fine-needle aspiration biopsy

B) MRI of the neck

C) Radionuclide scan

D) Reassurance and follow-up after delivery

E) Serum thyroglobulin

F) Total thyroidectomy in the second trimester


Explanation
Correct Answer:

A) Fine-needle aspiration biopsy

Thyroid nodules are common and may be diagnosed on physical examination or noted incidentally when imaging
studies are obtained for other reasons. Once a thyroid nodule is found, cancer risk factors (eg, family history,
radiation exposure) should be assessed by history. Physical examination should evaluate the approximate size,
mobility, and firmness of the thyroid nodule and whether enlarged cervical lymph nodes are present. A serum TSH
should be obtained.

If serum TSH is normal, a thyroid ultrasound is the next step to determine nodule sonographic features and size.
Certain sonographic features (eg, microcalcifications, irregular margins, internal vascularity) carry a much higher
risk of malignancy than others (eg, cystic or spongiform lesions). Thyroid nodules >1 cm with these high-risk
sonographic features should undergo fine-needle aspiration (FNA) biopsy. Thyroid nodules >2 cm should all
undergo FNA (unless they are cystic, as they have a low risk of malignancy).

Pregnant women undergo the same evaluation but should not receive radioactive iodine (for either diagnosis or
treatment) because it can lead to congenital hypothyroidism, intellectual disability, and increased risk of malignancy
in the fetus. In this patient who has a >1-cm nodule with high-risk sonographic features, workup should proceed
with an FNA, which is safe during pregnancy.

(Choice B) Thyroid nodules may be found incidentally on an MRI scan; however, ultrasound is the initial imaging
modality of choice for workup of thyroid nodules.

(Choice C) Radionuclide scans may be done for thyroid nodules in the setting of a low TSH (hyperthyroid) level
because those nodules are often hot and less likely to be cancerous. In this patient with a normal TSH, an FNA is
the appropriate next step. In addition, radioactive iodine should never be used in pregnancy because of the risk to
the fetus.

(Choices D and F) If a thyroidectomy is needed (eg, due to cancer), it can often be delayed until after delivery.
However, should the workup reveal a more aggressive or rapidly growing thyroid cancer, the optimal timing of
surgery would be during the second trimester. Therefore, this patient should undergo an FNA to determine if (and
when) a thyroidectomy is needed.

(Choice E) Thyroglobulin is produced by both normal thyroid tissue and differentiated thyroid cancer cells.
Therefore, it is a useful tumor marker to monitor for recurrence after the thyroid gland has been completely
removed.

Educational objective:
Thyroid nodules that have suspicious sonographic features should undergo fine-needle aspiration biopsy, even if
the patient is pregnant.

Reference
• A review on thyroid cancer during pregnancy: multitasking is required.

• 2015 American Thyroid Association management guidelines for adult patients with thyroid nodules and
differentiated thyroid cancer: The American Thyroid Association Guidelines Task Force on thyroid nodules
and differentiated thyroid cancer.

• 2017 Guidelines of the American Thyroid Association for the diagnosis and management of thyroid disease
during pregnancy and the postpartum.
Question #127

A 48-year-old man comes to the office due to nasal congestion and rhinorrhea. Three months ago, the patient
developed symptoms of nasal congestion, rhinorrhea, cough, fatigue, and fever. He began using over-the-counter
acetaminophen and nasal decongestant spray. The fever resolved and symptoms initially improved, but since then,
the patient has had worsening nasal congestion and a decreased sense of smell despite continued medication use.
He has no other chronic medical problems. He works at a print shop. He does not use tobacco or illicit drugs. Vital
signs are within normal limits. The tympanic membranes are clear with no middle ear effusion. Nasal examination
shows swollen, beefy-red nasal turbinates obstructing the nasal airway. The oral cavity is clear; the tonsils are
small and not inflamed. There is no cervical adenopathy. Which of the following is the most appropriate next step
in management of this patient's current condition?

A) Allergen avoidance

B) CT scan of the paranasal sinuses

C) Measurement of serum IgE concentration

D) Serum antineutrophil cytoplasmic autoantibody assay

E) Withdrawal of nasal decongestant


Explanation
Correct Answer:

E) Withdrawal of nasal decongestant

This patient has rhinitis medicamentosa, nasal congestion that occurs after prolonged use of nasal
decongestant sprays. Oxymetazoline, the most commonly used over-the-counter nasal decongestant spray, leads
to rapid relief of nasal congestion by causing profound vasoconstriction. However, with continued use, it can injure
the vascular endothelium, leading to rebound congestion; patients then use it more frequently, setting up a vicious
cycle. This can occur in as few as 3-5 days. Physical examination often shows beefy-red nasal mucosa as
opposed to the edematous, pale mucosa often seen in allergic rhinitis.

Because they sometimes consider over-the-counter medications inconsequential, patients may not volunteer
information about the use of nasal decongestant sprays. The most important step in management is cessation of
the nasal decongestant spray, which is typically very uncomfortable for patients because of rebound nasal
congestion; significant counseling is required. Nasal glucocorticoids are often prescribed to aid cessation. A short
course of oral glucocorticoids can be added in severe cases.

(Choices A and C) Avoidance of allergens or measurement of serum IgE concentration is often recommended for
patients with allergic rhinitis, which can present with nasal congestion. However, it is typically associated with
sneezing and itching of the eyes. Physical examination would show edematous, pale nasal mucosa rather than the
beefy red mucosa that is classically associated with rhinitis medicamentosa.

(Choice B) CT scan of the paranasal sinuses can be used to evaluate for chronic rhinosinusitis, which can cause
nasal congestion and develop after upper respiratory symptoms. However, in addition to nasal congestion and a
loss of smell, patients with chronic sinusitis typically have mucopurulent drainage and facial pain or pressure. In
addition, this patient's continued use of nasal decongestant makes rhinitis medicamentosa much more likely.

(Choice D) Serum antineutrophil cytoplasmic autoantibody assay can be used to assess for some systemic
diseases that can sometimes present with nasal symptoms (eg, granulomatosis with polyangiitis). However, there
are clinical manifestations in other organ systems (eg, lower respiratory tract, kidneys), and this patient's history of
nasal decongestant spray use makes rhinitis medicamentosa much more likely.

Educational objective:
Over-the-counter decongestant sprays can cause rhinitis medicamentosa after only a few days. Cessation of the
medication is the most important step in management.

Reference
• Management of rhinitis medicamentosa: a systemic review.

• Rhinitis medicamentosa as a cause of increased intraoperative bleeding.

• Rhinitis medicamentosa: electron microscopic changes of human nasal mucosa.


Question #128

A 55-year-old man comes to the office due to left-sided facial numbness. His symptoms began a month ago with
tingling over his left mid-face and have progressively worsened to complete numbness on that side. The patient
has also had 2 months of headaches, nasal congestion with intermittent epistaxis, and left ear fullness. He has had
no fever, rhinorrhea, or purulent nasal discharge. The patient has taken over-the-counter allergy medications, but
these have provided no relief. He has no other medical problems and does not use tobacco, alcohol, or illicit
drugs. He recently immigrated to the United States from southern China to be closer to his son. Temperature is 37
C (98.6 F), blood pressure is 126/80 mm Hg, and pulse is 84/min. Examination reveals sensory loss to touch and
pain on the left side of the face. No facial muscle weakness or other neurological findings are present. Enlarged,
nontender, and mobile cervical lymph nodes are palpable bilaterally. No oropharyngeal or nasal lesions are
visualized. Otoscopy of the left ear demonstrates clear fluid behind the retracted tympanic membrane, but the right
ear is normal. Nasopharyngoscopy reveals a soft-tissue mass in the nasopharynx. Which of the following is the
most likely diagnosis?

A) Granulomatosis with polyangiitis

B) Mucormycosis

C) Nasal polyposis

D) Nasopharyngeal carcinoma

E) Tertiary syphilis
Explanation
Correct Answer:

D) Nasopharyngeal carcinoma

Nasopharyngeal carcinoma

• Endemic to Asia
Epidemiology • Linked with Epstein-Barr virus reactivation
• Risk factors: Diet (salty fish), smoking, genetics

• Obstruction: Nasal congestion, epistaxis, headache


Manifestations • Mass effect: Cranial nerve palsy, otitis media
• Spread: Neck mass (cervical lymphadenopathy)

Diagnosis • Endoscope-guided biopsy

• Radiation therapy
Treatment
• Chemotherapy

Nasopharyngeal carcinoma (NPC) is a tumor associated with Epstein-Barr virus reactivation that is rare in the
United States but endemic in southern China. Risk is likely increased in this region due to a nitrosamine-rich diet
(eg, salt-cured food, early exposure to salted fish) and genetic predisposition.
The nasopharynx links the nasal cavity and the oropharynx. NPC tumors obstruct this passage and invade
adjacent tissues, often resulting in nasal congestion with epistaxis, headache, facial numbness (para-cavernous
sinus tumor invasion), and/or serous otitis media (eustachian tube obstruction). Early metastasis to the bilateral
cervical lymph nodes is common.

Diagnosis is made with endoscope-guided biopsy of the primary tumor, and treatment usually involves a
combination of radiation therapy and chemotherapy. Survival is excellent if the neoplasm is discovered early, but
most patients present with advanced disease.

(Choice A) Granulomatosis with polyangiitis is a vasculitis that commonly causes ear (otitis) and nasal (rhinorrhea,
bloody discharge) symptoms. However, patients usually have systemic manifestations (eg, fever, malaise, weight
loss) and ulcerative lesions, not a soft-tissue mass.

(Choice B) Mucormycosis is a fungal infection primarily seen in patients with diabetes mellitus or significant
immunocompromise. Manifestations typically include a pan-sinusitis with fever and purulent rhinorrhea (unlike in
this patient). Necrotic destruction of bone may occur, and examination often reveals black eschars.

(Choice C) Nasal polyps are usually seen in patients with chronic sinusitis, asthma, or aspirin sensitivity and may
obstruct the nasal cavity, resulting in congestion and thick rhinorrhea. However, cranial nerve involvement is
unlikely and examination would reveal a grey mass with inflammatory material.

(Choice E) Tertiary syphilis may cause meningitis or meningovasculitis with headache and cranial nerve deficits;
however, nasal congestion and a soft-tissue mass are unlikely.

Educational objective:
Nasopharyngeal carcinoma is a tumor associated with Epstein-Barr virus reactivation that most commonly affects
individuals living in southern China. Manifestations are due to nasopharyngeal obstruction or invasion of adjacent
tissues and include nasal congestion with epistaxis, headache, diplopia, cranial nerve deficits (eg, facial numbness),
and otitis media. Early metastasis to the cervical lymph nodes is common.
Question #129

A 56-year-old man comes to the office for evaluation of a gradually enlarging, painless mass above the right angle
of the jaw. He has also recently experienced right-sided facial weakness. When he was in his 20s, the patient had
Hodgkin disease that was treated with radiation to the neck. Family history is significant for thyroid cancer in his
mother. The patient does not use tobacco, alcohol, or illicit drugs. He had several male and female sexual partners
in his 30s but has been sexually active with only one partner for the past 15 years. Vital signs are within normal
limits. Physical examination shows a firm, nontender mass in the right parotid gland and weakness of all right-sided
facial muscles. Which of the following is most concerning for malignancy in this patient?

A) Exposure to oncogenic virus

B) Family history of malignancy

C) Lack of salivary gland pain

D) Origin from parotid gland

E) Presence of facial droop


Explanation
Correct Answer:

E) Presence of facial droop


This patient has an enlarging parotid gland neoplasm. Most parotid neoplasms are benign, but exposure to
radiation, as in this patient, is a significant risk factor for malignant disease. Both the facial nerve (CN VII) and the
trigeminal nerve (CN V) are closely associated with the parotid gland. The presence of facial droop (CN VII
dysfunction) or facial numbness (CN V dysfunction) is very concerning for neural invasion due to malignant
disease.

As a result, all patients with parotid masses undergo detailed cranial nerve examination. Imaging (CT or MRI) is
typically performed. Ultrasound is often used to enable fine-needle aspiration biopsy. Parotid tumors represent a
diverse range of pathologies, including primary parotid neoplasms, lymphomas, or metastases (eg, squamous cell
carcinoma, melanoma). Surgical resection with sparing of the facial nerve is typically recommended, with possible
additional surgery or adjuvant treatment depending on the final histopathologic diagnosis.

(Choice A) Having multiple sexual partners is associated with a risk of infection with human papillomavirus, which
is an oncogenic virus implicated in head and neck mucosal squamous cell carcinoma. It is not associated with
salivary gland malignancies.

(Choice B) Several familial cancer syndromes are associated with thyroid malignancy (eg, multiple endocrine
neoplasia), but salivary gland malignancies are not seen in any of them.

(Choice C) Pain and firmness (or lack thereof) are not reliable indicators of malignancy in parotid tumors. Pain in
benign neoplasms can occur with associated infection, hemorrhage, or cystic enlargement. In malignant tumors,
pain usually indicates neural invasion.

(Choice D) Most tumors that originate in the parotid gland are benign (>80%) rather than malignant. Neoplasms
that originate in the submandibular gland or minor salivary glands have a higher likelihood of malignancy.

Educational objective:
Parotid masses are typically benign. Cranial nerve dysfunction (facial droop, facial numbness) increases concern
for malignancy.

Reference
• Salivary gland malignancies.
• Utility of preoperative fine needle aspiration in parotid lesions.

• Parotid gland tumors and the facial nerve.


Question #130

A 45-year-old man comes to the office due to loss of sexual desire and failure to attain satisfactory erections during
intercourse. He has no morning erections. The patient has no leg or buttock pain with exertion but has had
intermittent bilateral hand pain. Medical history is significant for type 2 diabetes mellitus and recent-onset
peripheral neuropathy. He does not use tobacco or alcohol. The patient has a 12-year-old child from a previous
marriage and now lives with his second wife. Blood pressure is 120/70 mm Hg and pulse is 70/min. BMI is 24
kg/m2. The neck is supple with no thyromegaly or lymphadenopathy. Cardiopulmonary examination is normal. The
abdomen is soft and nontender with palpable hepatomegaly. Genital examination shows small testes bilaterally but
otherwise normal secondary sexual characteristics. Peripheral pulses are 2+, and capillary refill in the feet is
normal. Neurologic examination demonstrates normal deep tendon reflexes and muscle strength; sensation is
mildly decreased in both ankles. Visual field testing is normal. What is the best next step in management of this
patient?

A) Karyotype analysis

B) Nocturnal polysomnography

C) Phosphodiesterase-5 inhibitor therapy

D) Scrotal ultrasound

E) Serum ferritin level

F) Testosterone replacement therapy


Explanation
Correct Answer:

E) Serum ferritin level

Hereditary hemochromatosis

• Hyperpigmentation
• Arthropathy
• Hepatomegaly, cirrhosis, hepatocellular carcinoma
Clinical manifestations
• Diabetes mellitus
• Hypopituitarism, secondary hypogonadism
• Cardiomyopathy

• Elevated liver transaminases


Diagnosis • Elevated serum ferritin, transferrin saturation
• HFE genetic mutations

Management • Therapeutic phlebotomy (urgent if ferritin >1,000 ng/mL)

This patient has hypogonadism presenting with decreased libido and erectile dysfunction (ED); the loss of morning
erections suggests an organic rather than psychogenic cause of ED. In addition, he has features suggestive of
hereditary hemochromatosis (HH), including diabetes mellitus (despite a normal BMI), joint pain, and
hepatomegaly. HH is due to excessive absorption of iron and deposition of excess iron in the tissues. HH can
cause secondary hypogonadism due to iron deposition in the pituitary, which impairs production of gonadotropins
(and often other pituitary hormones), leading to decreased testicular testosterone production and testicular atrophy.

Initial assessment of iron stores in patients with suspected HH should include serum iron, serum ferritin, and
transferrin saturation (ie, serum iron/total iron binding capacity × 100). The diagnosis is confirmed with
HFE genetic testing. Concurrently, hypogonadism can be assessed with serum testosterone, FSH, and LH, which
are characteristically low. Treatment includes therapeutic phlebotomy.

(Choice A) Klinefelter syndrome (ie, 47,XXY by karyotype analysis) can cause hypogonadism; however, patients
are typically infertile (this patient has a child). Also, Klinefelter syndrome does not cause joint pain or
hepatomegaly.

(Choice B) Nocturnal polysomnography is used as an initial test for obstructive sleep apnea, which can be
associated with erectile dysfunction. Obstructive sleep apnea is common in patients with diabetes but is typically
seen in those with significant obesity. This patient has a normal BMI and no specific symptoms (eg, morning
headache) of sleep apnea.

(Choices C and F) Testosterone replacement can treat hypogonadal symptoms (eg, decreased libido), and
phosphodiesterase-5 inhibitors (eg, sildenafil) can often improve erectile function. These interventions may be
useful for this patient later but may be unnecessary if phlebotomy restores gonadal function. Empiric treatment
without proper diagnostic evaluation can lead to serious complications of HH, such as heart failure and cirrhosis.
This patient has a normal BMI, which indicates that his diabetes (and neuropathy) could be due to underlying
undiagnosed HH rather than insulin resistance from obesity.

(Choice D) Scrotal ultrasound is typically performed in patients with possible testicular malignancy (eg, scrotal
mass) or other structural abnormalities (eg, hydrocele). It is unlikely to be helpful in evaluation of bilateral testicular
atrophy.

Educational objective:
Hereditary hemochromatosis can cause secondary hypogonadism due to iron deposition in the pituitary with
deficient production of gonadotropins. Initial assessment should include serum iron, serum ferritin, and transferrin
saturation. The diagnosis is confirmed with HFE genetic testing. Treatment includes therapeutic phlebotomy.
Question #131

A 52-year-old woman comes to the office for a routine follow-up. The patient takes hormone replacement therapy
for menopausal vasomotor symptoms. Her hormone dosage was increased a year ago after she began taking
carbamazepine for trigeminal neuralgia. She adheres to both treatments and reports no symptoms at present. The
patient drinks a glass of wine 2 or 3 times a week and does not use tobacco or recreational drugs. She works as a
tour boat operator and spends most of her time outdoors. The patient consumes a balanced diet but does not
exercise regularly. Family history is significant for osteoporosis in her mother. Vital signs are within normal limits.
BMI is 26 kg/m2. Physical examination shows no abnormalities. Laboratory results show a decreased serum
25-hydroxyvitamin D level. Which of the following is the greatest risk factor contributing to this patient's vitamin
deficiency?

A) Alcohol consumption

B) Carbamazepine therapy

C) Family history of osteoporosis

D) Hormone replacement therapy

E) Lack of regular exercise


Explanation
Correct Answer:

B) Carbamazepine therapy

Causes of vitamin D deficiency

• Lactose intolerance or milk avoidance


Inadequate intake
• Vegan diet (without fortification)

• Crohn disease
Malabsorption • Celiac disease
• Bariatric surgery (eg, gastric bypass), gastrectomy

• Extreme northern/southern latitudes


Inadequate sunlight exposure • Heavy skin pigmentation
• Sedentary/indoor lifestyle

• Chronic kidney disease


Metabolic disorders
• Chronic liver disease

Increased catabolism • Drugs: anticonvulsants (eg, phenytoin, carbamazepine),


isoniazid, rifampin

This patient has vitamin D deficiency, confirmed with a low serum 25-hydroxyvitamin D level. However, she has
no classic risk factors for dietary deficiency (eg, restricted diet, malabsorption) or inadequate dermal vitamin D
production (eg, nursing home residence).

Patients with adequate sources of vitamin D can develop deficiency due to excessive catabolism. Catabolism of
vitamin D is initiated by CYP24A1 (also called 24-hydroxylase), which acts on both the 25- and 1,25- hydroxylated
vitamin D forms, converting them to inactive forms. Carbamazepine and other anticonvulsants (eg, phenytoin)
induce expression of this enzyme, which can lead to increased breakdown of vitamin D and vitamin D deficiency.

(Choice A) Heavy alcohol use with secondary liver or pancreatic disease can lead to deficiency of fat-soluble
vitamins (eg, A, D, E, K). However, light alcohol intake, as in this patient, does not cause hepatic or pancreatic
dysfunction and is not associated with any significant nutritional deficiencies.

(Choice C) The etiology of osteoporosis in most patients is multifactorial; major contributing factors include age,
female sex (and menopausal status), body habitus, and lifestyle factors (eg, diet, smoking). Genetic factors are
important but usually lesser contributors, and inherited vitamin D disorders are rare.

(Choice D) Most (~85%) of vitamin D in circulation is bound to vitamin D–binding protein (DBP), providing a large
pool of vitamin D that is largely resistant to degradation. Supplemental estrogen (eg, oral contraceptives,
menopausal replacement) can increase synthesis of DBP, leading to a higher (not lower) measured vitamin D level.

(Choice E) Regular exercise promotes bone density maximization during childhood and slows bone density loss
during adulthood. Lack of regular exercise can increase the risk for osteoporosis but has little effect on vitamin D
levels. Although patients who do not exercise regularly also may spend little time outdoors, contributing to vitamin
D deficiency (due to inadequate sunlight exposure), this patient has an outdoor occupation.

Educational objective:
Patients with adequate dietary and dermal sources of vitamin D can develop deficiency due to increased catabolism
of vitamin D. Commonly used anticonvulsants (eg, carbamazepine, phenytoin) can induce CYP24A1, which
catabolizes vitamin D to inactive forms, leading to vitamin D deficiency.

Reference
• Effect of long-term carbamazepine therapy on bone health.
Question #132

A 24-year-old woman is evaluated for a lump in her neck. She discovered the lump a week ago while showering.
The patient feels well and has no associated pain, fever, night sweats, or weight changes. Her medical history is
unremarkable, although she has had irregular menses for a year since insertion of an intrauterine device. The
patient does not use tobacco, alcohol, or illicit drugs. Her mother died during surgery for thyroid cancer. Blood
pressure is 133/80 mm Hg and pulse is 78/min. On examination, there is a palpable 2-cm nodule in the right thyroid
lobe. The remainder of the examination, including chest, abdomen, extremities, and skin, is normal. Serum TSH
and calcium levels are normal and calcitonin is elevated. Ultrasound-guided aspiration biopsy reveals malignant
cells. Which of the following tests is the best next step in evaluation of this patient?

A) Aldosterone-renin activity ratio

B) Late-night salivary cortisol

C) Low-dose dexamethasone suppression test

D) Plasma fractionated metanephrine assay

E) Plasma insulin-like growth factor-1

F) Serum prolactin assay


Explanation
Correct Answer:

D) Plasma fractionated metanephrine assay

Classification of multiple endocrine neoplasia

• Primary hyperparathyroidism (parathyroid


adenomas or hyperplasia)
Type 1
• Pituitary tumors (prolactin, visual defects)
• Pancreatic tumors (especially gastrinomas)

• Medullary thyroid cancer (calcitonin)


• Pheochromocytoma
Type 2A
• Primary hyperparathyroidism (parathyroid
hyperplasia)

• Medullary thyroid cancer (calcitonin)


Type 2B • Pheochromocytoma
• Mucosal neuromas/marfanoid habitus

This patient with a malignant thyroid mass, elevated serum calcitonin level, and family history of thyroid malignancy
likely has an inherited form of medullary thyroid cancer (MTC). MTC is a calcitonin-producing tumor arising from
neuroendocrine parafollicular C cells of the thyroid gland. Although most MTC is sporadic, approximately one third
of MTC is inherited as a component of multiple endocrine neoplasia (MEN) types 2A and 2B.
MEN 2A and 2B (usually due to RET proto-oncogene mutations) are also associated with pheochromocytoma,
which can be asymptomatic at the time of diagnosis but cause life-threatening hypertensive crisis during surgical
procedures (eg, thyroidectomy). In light of this risk, most patients with MTC should undergo RET mutation testing
and screening for pheochromocytoma with a plasma fractionated metanephrine assay. If found,
pheochromocytoma should be resected prior to thyroidectomy.

(Choices A, B, and C) The aldosterone-renin activity ratio can help evaluate hyperaldosteronism (eg,
hypertension, hypernatremia, hypokalemia). The low-dose dexamethasone suppression test and the salivary
cortisol test are helpful in screening for Cushing syndrome (eg, obesity, facial plethora, easy bruising). This patient
has no manifestations of these conditions. In addition, adrenocortical tumors are uncommon in MEN syndromes,
with biochemical testing recommended only in patients with manifestations of excessive hormonal production.

(Choices E and F) Serum prolactin and insulin-like growth factor-1 are screening tests for prolactin and growth
hormone-secreting pituitary tumors. Pituitary tumors occur in MEN1 but are not a feature of MEN2.

Educational objective:
Medullary thyroid cancer (MTC) is a calcitonin-producing tumor of the thyroid parafollicular C cells. It often occurs
as a component of multiple endocrine neoplasia types 2A and 2B, which are also associated with
pheochromocytoma. Patients with MTC should be screened for pheochromocytoma prior to thyroidectomy with a
plasma fractionated metanephrine assay.

Reference
• Genetics, diagnosis, and management of medullary thyroid carcinoma and pheochromocytoma/
paraganglioma.
Question #133

A 40-year-old man with type 1 diabetes mellitus comes to the office due to recurrent hypoglycemia. A month ago,
the patient was seen in the emergency department due to unconsciousness associated with a blood glucose level
of 34 mg/dL. Since then, he has had several blood glucose readings <50 mg/dL that he corrected by drinking fruit
juice; however, he had no tremor, sweating, or palpitations during the episodes. Medications include insulin
glargine, insulin lispro, lisinopril, hydrochlorothiazide, and simvastatin. Vital signs and physical examination are
normal. Hemoglobin A1c is 8.1% and serum creatinine is 1.1 mg/dL. Which of the following is the most likely cause
of hypoglycemia unawareness in this patient?

A) Hydrochlorothiazide use

B) Lisinopril use

C) Previous severe hypoglycemia

D) Simvastatin use
Explanation
Correct Answer:

C) Previous severe hypoglycemia


Patients with type 1 diabetes mellitus can develop hypoglycemia because exogenous insulin continues to be
absorbed from the injection site despite falling glucose levels; those with long-standing diabetes (ie, >5 yr) often
also have alpha cell failure, which can lead to decreased glucagon secretion and exacerbate hypoglycemia.
Normally, hypoglycemia prompts a catecholamine (eg, epinephrine) surge, which increases hepatic glucose
production and triggers many of the characteristic neurogenic hypoglycemic symptoms (eg, arousal, tremor).
However, patients with long-standing diabetes often have a blunted autonomic response to hypoglycemia, with
reduced hypoglycemia awareness.

This blunted autonomic response is worse in patients who frequently have low circulating glucose levels. Such
recurrent or severe hypoglycemia reduces the glucose-raising effects of epinephrine and suppresses the
symptoms related to the catecholamine surge; in turn, this increases the risk for progressively worse hypoglycemic
episodes (ie, hypoglycemia-associated autonomic failure).

Management requires strict avoidance of hypoglycemia to restore awareness. Judicious timing of insulin
administration and carbohydrate ingestion (eg, in relation to physical activity) is necessary.

(Choices A, B, and D) Thiazide diuretics (eg, hydrochlorothiazide) and statins (eg, simvastatin) can raise blood
glucose levels, making glycemic control somewhat more difficult. In contrast, ACE inhibitors (eg, lisinopril) increase
insulin sensitivity and can occasionally induce hypoglycemia. However, these agents do not significantly affect
hypoglycemia awareness.

Educational objective:
Hypoglycemia normally prompts a catecholamine (eg, epinephrine) surge, which increases hepatic glucose
production and triggers hypoglycemic symptoms. Recurrent or severe hypoglycemia in patients with long-standing
diabetes reduces the glucose-raising effects of epinephrine and suppresses the symptoms related to the
catecholamine surge, increasing the risk for progressively worsening hypoglycemic episodes.

Reference
• Hypoglycemia-associated autonomic failure in diabetes.
Question #134

A 21-year-old man comes to the clinic for painless vision loss in the right eye and headaches. Blood pressure is
190/100 mm Hg and pulse is 96/min. Further evaluation reveals retinal and cerebellar hemangioblastomas. The
patient's father had very high blood pressure and significant hearing impairment and died of an intracranial
hemorrhage at age 48. Which of the following is the most likely cause of this patient's elevated blood pressure?

A) Aldosterone overproduction

B) Catecholamine overproduction

C) Cortisol overproduction

D) Polycystic kidney disease

E) Renal glomerular injury


Explanation
Correct Answer:

B) Catecholamine overproduction

Von Hippel-Lindau disease

• Mutation in the VHL tumor suppressor gene on chromosome 3


Etiology
• Autosomal dominant inheritance

• Cerebellar & retinal hemangioblastomas


Manifestations • Pheochromocytoma
• Renal cell carcinoma (clear cell subtype)

• Surveillance for associated malignancies


◦ Eye/retinal examination
◦ Plasma or urine metanephrines
Management
◦ MRI of the brain & spine
◦ MRI of the abdomen
• Tumor resection

This patient has vision loss and headaches due to retinal and CNS hemangioblastomas, which are benign but
heavily vascular tumors that can lead to hemorrhage or mass effect symptoms in the brain and spinal cord. Most
hemangioblastomas are sporadic; however, this patient has multiple hemangioblastomas and a family history of
intracranial hemorrhage, which suggests an inherited disorder, likely von Hippel-Lindau disease (vHL).
vHL is an autosomal dominant disorder associated with mutations in the VHL gene. Besides hemangioblastomas,
vHL is associated with clear cell renal cell carcinoma, pancreatic neuroendocrine tumors, endolymphatic sac tumors
of the middle ear (hearing loss), and pheochromocytomas. Pheochromocytomas (arising in the adrenal medulla)
and paragangliomas (in the sympathetic ganglia) are neuroendocrine tumors associated with excess production
of catecholamines, leading to headache, palpitations, and potentially severe hypertension.

As with hemangioblastomas, most pheochromocytomas and paragangliomas are sporadic. However, a significant
minority occur as a manifestation of a familial disorder. In addition to vHL, major genetic syndromes associated with
catecholamine-secreting tumors include multiple endocrine neoplasia type 2 (2A and 2B) and neurofibromatosis
type 1.

(Choice A) Primary hyperaldosteronism (Conn syndrome) can be caused by an adrenal adenoma or bilateral
adrenal hyperplasia. It typically presents with hypertension but is not associated with hemangioblastomas.

(Choice C) Hypercortisolism (Cushing syndrome) can cause hypertension but typically presents with central
obesity, easy bruising, proximal muscle weakness, and wide skin striae.

(Choice D) Autosomal dominant polycystic kidney disease can cause hypertension due to local renal ischemia and
increased renin release. It is associated with an increased risk of intracranial hemorrhage due to cerebral
aneurysms (not hemangioblastomas). Although vHL may be associated with the formation of renal cysts, this is not
a major contributor to hypertension.

(Choice E) Glomerulonephritis (eg, acute poststreptococcal glomerulonephritis) can cause hypertension but is
typically associated with gross hematuria (eg, tea-colored urine) and edema. Hemangioblastomas would not be
seen and are more suggestive of vHL.

Educational objective:
Von Hippel-Lindau disease (vHL) is an inherited disorder caused by mutations in the VHL gene. It is associated
with retinal and CNS hemangioblastomas, which are heavily vascular tumors that can lead to hemorrhage or mass
effect symptoms in the brain and spinal cord. Other manifestations include clear cell renal cell carcinoma,
pancreatic neuroendocrine tumors, endolymphatic sac tumors of the middle ear, and pheochromocytomas.

Reference
• Pheochromocytoma and paraganglioma: an endocrine society clinical practice guideline

• Genotype-phenotype correlations in pheochromocytoma and paraganglioma


Question #135

A 48-year-old woman comes to the office for evaluation of elevated blood pressure, which was first noted at her
dentist's office 4 weeks ago. Repeated home blood pressure measurements since then have remained around 150/
90 mm Hg. The patient has had no symptoms and has no history of hypertension or other medical conditions. She
takes no medications and does not use tobacco, alcohol, or illicit drugs. Family history is significant for
hypertension in her father. Blood pressure is 152/88 mm Hg and pulse is 76/min. A repeat blood pressure
measurement after 15 minutes shows a similar reading. BMI is 28.6 kg/m2. Physical examination reveals clear
lungs, normal heart sounds, a nontender abdomen with no organomegaly, and no lower extremity edema. There
are no abnormalities on ECG. Laboratory testing shows normal serum electrolytes and creatinine and no
proteinuria. Which of the following additional evaluations is most appropriate in this patient?

A) Assessment of left ventricular function

B) Home sleep apnea testing

C) Imaging for atherosclerotic renovascular disease

D) Measurement of plasma renin activity

E) Screening for diabetes mellitus


Explanation
Correct Answer:

E) Screening for diabetes mellitus

Laboratory evaluation of hypertension

• Serum electrolytes (Na, K, Ca)


• Serum creatinine
Renal function tests
• Urinalysis
• Urine albumin/creatinine ratio (optional)

• Fasting glucose or hemoglobin A1c


Endocrine tests • Lipid profile
• TSH

• ECG
Cardiac tests
• Echocardiography (optional)

• Complete blood count


Other tests
• Uric acid (optional)

This patient has hypertension, defined as a sustained blood pressure ≥130 mm Hg systolic or ≥80 mm Hg diastolic
(some guidelines recommend ≥140/90 mm Hg for diagnosis); given her age, mildly elevated blood pressure, and
family history of hypertension, this most likely represents primary (essential) hypertension. The initial evaluation
should identify complications of hypertension (eg, chronic kidney disease) or comorbid conditions (eg,
hyperlipidemia) that might influence management. Assessment should include a serum chemistry panel (including
electrolytes and creatinine), hemoglobin/hematocrit, lipid panel, and urinalysis.

In addition, patients with hypertension should be screened for diabetes mellitus with either fasting blood
glucose, which can be obtained as part of a serum chemistry panel, or hemoglobin A1c. Even in the absence of
hypertension, screening is advised for patients age 35-70 with a BMI ≥25 kg/m2 (ie, overweight or obesity).
Screening at younger ages should be considered for those with additional risk factors (eg, family history of
diabetes).

(Choice A) Many experts recommend ECG to identify signs of left ventricular hypertrophy in patients with
hypertension. Further assessment of ventricular function with echocardiography is more specific but is expensive
and would have low yield in this young patient with mild hypertension and a normal ECG.

(Choice B) Testing for sleep apnea is indicated for patients with symptoms (eg, daytime somnolence, witnessed
apneic episodes) and can be considered for those with refractory hypertension. However, it is not indicated for
asymptomatic patients with uncomplicated hypertension.

(Choice C) Duplex Doppler ultrasonography can identify renal artery stenosis, which typically occurs in older men
with diffuse atherosclerosis and causes severe or resistant hypertension requiring ≥3 medications. Other
characteristic findings include recurrent flash pulmonary edema and unexplained renal failure after starting an ACE
inhibitor.

(Choice D) Plasma renin activity is used to evaluate primary hyperaldosteronism. However, patients typically have
severe or resistant hypertension; hypokalemia is common. Primary hyperaldosteronism is unlikely in this patient
with mild hypertension and normal serum electrolytes.

Educational objective:
Patients with hypertension should be screened for diabetes mellitus with either fasting blood glucose or hemoglobin
A1c. Even in the absence of hypertension, screening for diabetes is advised for patients age 35-70 with a BMI ≥25.
Question #136

A 60-year-old man is brought to the emergency department by his daughter due to a 2-day history of confusion and
lethargy. According to the daughter, he has had a constant dry cough, fatigue, anorexia, polyuria, and constipation
for several weeks. In addition, the patient has lost 9.1 kg (20 lb) over the past 3 months. He has no associated
pain. The patient's medical history is significant for hypertension treated with chlorthalidone and for
gastroesophageal reflux disease treated with over-the-counter antacids. He smokes 2 packs of cigarettes daily and
consumes alcohol occasionally. Blood pressure is 130/90 mm Hg and pulse is 90/min. Temporal wasting is noted.
Chest and abdominal examinations are normal. Mental status examination shows somnolence and disorientation to
time. Neurologic examination shows decreased deep tendon reflexes. Laboratory results are as follows:

Serum chemistry
Sodium 140 mEq/L
Potassium 4 mEq/L
Chloride 104 mEq/L
Bicarbonate 24 mEq/L
Creatinine 1.6 mg/dL
Calcium 14.4 mg/dL
Glucose 100 mg/dL

Alkaline phosphatase, serum 130 U/L


Phosphorus (inorganic), serum 2.2 mg/dL

Which of the following is the most likely cause of this patient's symptoms?

A) Elevated parathyroid hormone


B) Elevated parathyroid hormone-related protein

C) Increased 1,25-dihydroxyvitamin D production

D) Local cytokine production

E) Metastatic osteolysis

F) Milk-alkali syndrome

G) Thiazide-induced hypercalcemia
Explanation
Correct Answer:

B) Elevated parathyroid hormone-related protein

Hypercalcemia of malignancy

Cause Tumor type Mechanism Diagnostic

• Squamous cell
• ↓ PTH
PTHrP* • Renal & bladder • PTH mimic
• ↑ PTHrP
• Breast & ovarian

• Breast • ↓ PTH & PTHrP


Bone metastases • ↑ Osteolysis
• Multiple myeloma • ↓ Vitamin D

• ↓ PTH
1,25-dihydroxyvitamin D • Lymphoma • ↑ Calcium absorption
• ↑ Vitamin D

*PTHrP causes approximately 80% of malignancy-associated hypercalcemia.

PTH = parathyroid hormone; PTHrP = parathyroid hormone-related protein.


This patient has polyuria and neuropsychiatric symptoms consistent with hypercalcemia. Severe hypercalcemia
can also cause impaired neuromuscular excitability, leading to weakness, diminished reflexes, and decreased
gastrointestinal motility. In light of this patient's weight loss, temporal wasting, dry cough, and smoking history, he
most likely has humoral hypercalcemia of malignancy (HHM) due to squamous cell carcinoma of the lung. HHM
is a paraneoplastic syndrome due to the release of parathyroid hormone-related protein (PTHrP) by malignant
cells, which is structurally similar to parathyroid hormone (PTH) and acts on the PTH-1 receptor. PTHrP causes
increased bone resorption and reabsorption of calcium in the distal renal tubule. Increased phosphate excretion
leads to hypophosphatemia.

HHM is characterized by severe (eg, >14 mg/dL) and rapid-onset hypercalcemia. The diagnosis is often
suspected on clinical grounds and can be confirmed by an elevated PTHrP level and suppressed PTH level. HHM
is usually seen in advanced malignancy and confers a poor prognosis.

(Choice A) Primary hyperparathyroidism causes gradual-onset, mild-to-moderate hypercalcemia (usually <14 mg/
dL). Symptoms (eg, renal stones, osteoporosis, constipation), if present, are typically chronic and nonspecific.

(Choice C) Granulomatous diseases (eg, sarcoidosis) and lymphoma can cause hypercalcemia due to extrarenal
production of 1,25-dihydroxyvitamin D. However, hypercalcemia due to excess 1,25-dihydroxyvitamin D would be
associated with hyperphosphatemia rather than hypophosphatemia. Also, sarcoidosis is more common at age <40.

(Choices D and E) Osteolytic malignancies (eg, breast cancer, multiple myeloma) cause hypercalcemia due to
stimulation of osteoclasts by local production of cytokines. This is significantly less common than HHM, and serum
phosphorus levels are usually normal. In addition, most patients have significant associated bone pain.

(Choice F) Milk-alkali syndrome is caused by excessive intake of calcium and absorbable alkali. It is characterized
by hypercalcemia, metabolic alkalosis, and acute kidney injury. Bicarbonate levels are elevated due to increased
intake and decreased renal excretion of bicarbonate.

(Choice G) Thiazide diuretics cause increased reabsorption of calcium in the distal tubule. The resulting
hypercalcemia is mild (usually <12 mg/dL) and rarely symptomatic.

Educational objective:
Humoral hypercalcemia of malignancy is characterized by severe, symptomatic hypercalcemia. It is due to the
release of parathyroid hormone-related protein by tumor cells, leading to increased bone resorption and
reabsorption of calcium in the distal renal tubule.

Reference
• Cancer-related hypercalcemia.
Question #137

A 50-year-old man comes to the office with a 2-day history of malaise, fever, fatigue, and pain in the front of his
neck. He recovered from an upper respiratory illness a week prior to the onset of his current symptoms, but he
previously had felt well. The patient was seen in the clinic occasionally in the remote past for sexually transmitted
infections, but his medical history is otherwise unremarkable. He lives alone and is sexually active with both men
and women. Temperature is 38.5 C (101.3 F), blood pressure is 150/70 mm Hg, and pulse is 110/min. The thyroid
gland is diffusely enlarged and tender. Laboratory results are as follows:

Complete blood count


Hemoglobin 13.9 g/dL
Hematocrit 44%
Platelets 200,000/mm3
Leukocytes 10,500/mm3

TSH <0.001 µU/mL


Free T4 4.6 ng/dL (0.9-2.4 ng/dL)
Erythrocyte sedimentation rate 100 mm/hr

Which of the following is the most likely diagnosis in this patient?

A) Acute retroviral syndrome

B) Disseminated gonococcal infection

C) Graves disease
D) Hashimoto thyroiditis

E) Infectious mononucleosis

F) Subacute thyroiditis

G) Suppurative thyroiditis
Explanation
Correct Answer:

F) Subacute thyroiditis

Thyroiditis

Clinical features Diagnostic testing

Chronic autoimmune
• TPO antibody
thyroiditis • Predominant hypothyroid features
• Variable radioiodine
• Diffuse goiter
uptake
(Hashimoto thyroiditis)

• Variant of chronic autoimmune


Painless thyroiditis thyroiditis
• Positive TPO antibody
• Mild, brief hyperthyroid phase
• Low radioiodine uptake
(silent thyroiditis) • Small, nontender goiter
• Spontaneous recovery

• Likely postviral inflammatory process


Subacute thyroiditis • Prominent fever & hyperthyroid • Elevated ESR & CRP
(de Quervain thyroiditis) symptoms • Low radioiodine uptake
• Painful/tender goiter
CRP = C-reactive protein; ESR = erythrocyte sedimentation rate; TPO = positive thyroid peroxidase.

This patient has primary hyperthyroidism (elevated free T4, suppressed TSH). In light of his associated fever,
neck pain, tender goiter, and elevated erythrocyte sedimentation rate, this is most likely due to subacute
thyroiditis. Subacute (de Quervain, subacute granulomatous) thyroiditis is thought to be due to a postviral
inflammatory process and is often preceded by an upper respiratory illness.

Thyrotoxicosis in subacute thyroiditis resolves spontaneously within a few weeks and may be followed by a
hypothyroid phase lasting a few months. Most patients eventually recover to a euthyroid state. Treatment is
symptomatic with beta blockers to control thyrotoxic symptoms and nonsteroidal anti-inflammatory drugs
(NSAIDs) for pain relief. Glucocorticoids are used for severe thyroid pain not responding to NSAIDs.

(Choices A, B, and E) Infectious mononucleosis is characterized by pharyngitis, fever and other systemic
symptoms, lymphadenopathy, and splenomegaly. Acute HIV infection can cause a mononucleosis-like illness with
fever, pharyngitis, and lymphadenopathy, often with a generalized macular rash and gastrointestinal symptoms.
Disseminated gonococcal infection typically presents with fever, chills, vesiculopustular rash, and polyarthralgias or
purulent monoarthritis. None of these disorders is associated with a goiter or hyperthyroid state.

(Choices C and D) Chronic lymphocytic (Hashimoto) thyroiditis typically presents with hypothyroidism, although
patients may develop a brief hyperthyroid phase. However, the goiter seen in both Hashimoto thyroiditis and
Graves disease is nontender, and fever is not common.

(Choice G) Suppurative infection of the thyroid gland (infectious thyroiditis) is a rare condition causing high-grade
fever and pain at the thyroid gland. The thyroid gland may be palpably enlarged due to abscess formation.
However, patients are usually euthyroid as the involvement of the thyroid gland is focal.

Educational objective:
Subacute (de Quervain) thyroiditis is characterized by fever, neck pain, and a tender goiter following an upper
respiratory illness. Patients have a self-limited thyrotoxic phase followed by hypothyroidism and eventual recovery
of thyroid function. Treatment is symptomatic with beta blockers and nonsteroidal anti-inflammatory drugs.

Reference
• Subacute, silent, and postpartum thyroiditis.
Question #138

A 50-year-old man comes to the office due to a 4-month history of achy joint pain in the lower back, shoulders, hips,
and knees. The pain increases with activity, and he has no morning stiffness. The patient also has excessive
sweating, polyuria, and erectile dysfunction. Medical history is unremarkable. The patient has a 25-pack-year
smoking history and drinks moderate amounts of alcohol on social occasions. Blood pressure is 150/90 mm Hg,
pulse is 80/min, and respirations are 16/min. Examination shows crepitus on passive movement of the knee joints.
There is increased interdental spacing and an enlarged tongue, and the remainder of the examination is normal.
Plain radiography of the hand reveals widening of the joint spaces, osteophytes, and thickened soft tissues.
Laboratory evaluation reveals low testosterone and hyperglycemia. What is the most likely diagnosis?

A) Acromegaly

B) Amyloidosis

C) Hemochromatosis

D) Paget disease of bone

E) Paraneoplastic syndrome
Explanation
Correct Answer:

A) Acromegaly

Clinical features of acromegaly

Local tumor effect Headache, visual field defects, cranial nerve defects

Gigantism, frontal bossing, malocclusion of jaw, macrognathia, arthritis, carpal tunnel


Musculoskeletal
syndrome, enlargement of hands/feet

Skin Skin thickening, hyperhidrosis (odor), skin tags

Cardiovascular Cardiomyopathy, hypertension, heart failure

Respiratory Sleep apnea

Gastrointestinal Colon polyps/cancer, diverticulosis


Endocrine Galactorrhea, hypogonadism, diabetes mellitus, hypertriglyceridemia

Additional
Enlarged tongue, thyroid, salivary glands, liver, spleen, kidney, prostate
features

This patient has characteristic features of acromegaly, including arthralgias, jaw widening (increased interdental
spaces), and macroglossia. Acromegaly is caused by excessive production of growth hormone (GH), usually by a
pituitary somatotroph adenoma. Enlarging adenomas can also cause decreased secretion of other pituitary
hormones (eg, gonadotropins, leading to erectile dysfunction and low testosterone). In addition to causing
hyperglycemia (eg, polyuria) and increasing metabolic rate (eg, hyperhidrosis), excess GH directly and indirectly
(via release of insulin-like growth factor-1 from the liver) leads to overgrowth of tissues, including bone, cartilage,
and visceral organs.

Joint involvement is common and often involves both axial and appendicular skeletons. GH causes hyperplasia of
articular chondrocytes and synovial hypertrophy, which may be visible on x-ray as joint space widening. In
later stages, cartilage degeneration typically occurs, leading to clinical and radiographic findings resembling
osteoarthritis. Spinal involvement may lead to kyphoscoliosis. Treatment (eg, transsphenoidal surgery) can
minimize arthropathy in the hypertrophic stages but will not reverse degenerative changes.

(Choice B) Macroglossia is a characteristic manifestation of amyloid light-chain amyloidosis in plasma cell


disorders. Patients may have polyarticular arthritis with widening of the joint spaces and thickening of soft tissues.
However, hyperhidrosis and dental malocclusion are more typical of acromegaly.

(Choice C) Hereditary hemochromatosis–associated arthropathy most commonly presents as degenerative


arthritis involving the second and third metacarpophalangeal joints. Periarticular osteophytes may be seen on x-ray,
but joint spaces are narrowed; close inspection may reveal deposition of calcium pyrophosphate dihydrate in
articular cartilage (chondrocalcinosis). Cranial abnormalities are atypical.

(Choice D) Paget disease of bone is characterized by excessive, disordered bone formation; affected bone
weakens, leading to bowing, fracture, and arthritis of adjacent joints. However, findings are focal and asymmetric;
x-rays reveal mixed lytic-sclerotic lesions and thickening of cortical and trabecular bone. Visceral enlargement
(enlarged tongue) and hyperhidrosis are not typical.

(Choice E) Hypertrophic osteoarthropathy is a paraneoplastic syndrome of pulmonary malignancy, primarily


adenocarcinoma. It is characterized by bone and soft tissue enlargement in the distal extremities. Manifestations
include clubbing, arthropathy, periostosis of long bones, and joint effusions; cranial abnormalities are not common.

Educational objective:
Joint involvement in acromegaly is common and often involves the axial and appendicular skeletons. Excessive
growth hormone causes hyperplasia of articular chondrocytes and synovial hypertrophy, which may be visible on x-
ray as joint space widening. In later stages, cartilage degeneration may lead to clinical and radiographic findings
resembling osteoarthritis.
Question #139

A 45-year-old woman comes to the clinic due to fatigue, weakness, and diffuse bone pain. She was diagnosed with
celiac disease 5 years ago but continues to consume gluten-containing foods on occasion. The patient takes over-
the-counter folic acid and iron supplements. Laboratory results are as follows:

Hemoglobin 12.2 g/dL


Calcium 9.0 mg/dL
Phosphate 2.6 mg/dL (normal: 3-4.5)
Parathyroid hormone 122 pg/mL (normal: 10-65)
Alkaline phosphatase 234 U/L (normal: 36-92)

Which of the following is directly responsible for this patient's bone pain?

A) Accelerated focal bone remodeling

B) Defective formation of collagen

C) Impaired osteoid matrix mineralization

D) Low bone mass with normal mineralization

E) Lytic lesions due to malignant cell infiltration


Explanation
Correct Answer:

C) Impaired osteoid matrix mineralization

Clinical features of osteomalacia

• Malabsorption
• Intestinal bypass surgery
Causes • Celiac disease
• Chronic liver disease
• Chronic kidney disease

• May be asymptomatic
Symptoms/ • Bone pain & muscle weakness
signs • Muscle cramps
• Difficulty walking, waddling gait

• ↑ Alkaline phosphatase, ↑ PTH


• ↓ Serum calcium & phosphorus, ↓ urinary
calcium
Diagnosis • ↓ 25(OH)D levels
• Imaging
◦ Cortical thinning & reduced bone density
◦ Bilateral symmetric pseudofractures
(Looser zones)

25(OH)D = 25-hydroxycholecalciferol; PTH = parathyroid hormone.

This patient has osteomalacia, presenting with fatigue, bone pain, and hypophosphatemia. Osteomalacia is a
potential complication of vitamin D deficiency and is frequently seen in patients with malabsorptive disorders (eg,
celiac disease). In addition to bone pain, manifestations may include muscle weakness, a waddling gait, and
increased risk of fracture. Bone density is typically decreased, and x-ray may reveal linear Looser zones.

1,25-dihydroxyvitamin D is required for optimal intestinal absorption of calcium and phosphorus. In vitamin D
deficiency, intestinal calcium absorption is impaired, leading to a series of metabolic alterations:

• Secretion of parathyroid hormone (PTH) is increased (ie, secondary hyperparathyroidism) to maintain


circulating calcium levels.

• PTH causes release of calcium from bone stores, increases renal calcium reabsorption, and suppresses
renal phosphate reabsorption.

• As a result, circulating calcium levels can be restored to normal or near normal, but phosphate levels are
reduced (as in this patient).

During new bone formation and remodeling, osteoclasts create a cavity at the bone surface that osteoblasts fill with
organic matrix (osteoid). Calcium and inorganic phosphate then deposit in the matrix to mineralize the newly
formed bone. However, calcium or phosphorus deficiency at the mineralization front will lead to reduced
mineralization of the osteoid matrix.

(Choice A) Paget disease of bone is characterized by accelerated focal bone remodeling. It starts with increased
osteoclastic activity, followed by rapid and disorganized new bone formation. Alkaline phosphatase is elevated
(indicating increased osteoblast activity). However, serum calcium, phosphorus, and PTH levels are typically
normal.
(Choice B) Osteogenesis imperfecta is due to defective formation of type 1 collagen, which is required for
development of bone, tendons, ligaments, skin, teeth, and sclera. It typically presents in childhood with fragility
fractures, and additional manifestations (eg, short stature, impaired hearing, blue sclera) are often present.

(Choice D) Osteoporosis is characterized by low bone mass, but the bone usually has adequate mineralization.
Laboratory evaluation shows normal serum calcium, phosphorus, PTH, and alkaline phosphatase.

(Choice E) Lytic bone lesions are due to malignant cell infiltration by solid tumors (eg, breast, renal cell) or multiple
myeloma, which induce activation of osteoclasts and local destruction of bone. Due to the release of bone mineral,
hypercalcemia is common, and PTH will be suppressed.

Educational objective:
Osteomalacia is due to defective mineralization of bone matrix. It is most commonly due to vitamin D deficiency,
which leads to decreased intestinal calcium and phosphorus absorption with resultant secondary
hyperparathyroidism. Clinical features include bone pain, muscle weakness, hypophosphatemia, and increased risk
of fracture.

Reference
• Vitamin D deficiency in adults: when to test and how to treat.

• Vitamin D deficiency presenting like hypophosphatemic osteomalacia.


Question #140

A 34-year-old woman comes to the office due to diarrhea, weight loss, and fatigue over the past year. The diarrhea
occurs 2 or 3 times daily and is accompanied by crampy abdominal pain. She has diarrhea occasionally at night
but no tenesmus or blood in the stool. The patient describes her stools as very foul smelling and floating.
Associated symptoms include diffuse bone pain. Physical examination is unremarkable. Laboratory results show
hemoglobin of 9.8 g/dL with a mean corpuscular volume of 72 µm3. Which of the following serum laboratory
findings would be expected in this patient?

Calcium Phosphate Parathyroid hormone

A)

Low Low High

B)

Normal Low Low

C)

Low High Low

D)

Low High High

E)

High Low High


Explanation
Correct Answer:

A)

Low Low High

Clinical manifestations of celiac disease

• Diarrhea, ± steatorrhea, weight loss


• Abdominal pain
Gastrointestinal
• Flatulence/bloating
• Late manifestations: ulcerative jejunitis, enteropathy-associated T-cell lymphoma

• Dermatitis herpetiformis
Mucocutaneous
• Atrophic glossitis

• Vitamin D deficiency
Endocrine
• Secondary hyperparathyroidism

• Osteomalacia/osteoporosis (adults)
Bone disorders
• Rickets (children)
Hematologic • Iron deficiency anemia

• Peripheral neuropathy
Neuropsychiatric
• Depression/anxiety

This patient has chronic diarrhea and abdominal pain; the associated weight loss and steatorrhea (ie, foul-smelling,
greasy, or floating stools) suggest a malabsorptive disorder, most likely celiac disease (CD) (ie, immune-
mediated hypersensitivity to gluten). CD causes villous atrophy in the small bowel, reducing its absorptive capacity.

CD is often complicated by nutritional deficiencies due to inadequate absorption. In particular, malabsorption of iron
is common and leads to iron deficiency (microcytic) anemia, as in this patient. Deficiencies of fat-soluble vitamins
(eg, A, D, E, K) are also common.

1,25-dihydroxyvitamin D (calcitriol, the active form of vitamin D) is required for optimal intestinal absorption of
calcium and phosphorus. In vitamin D deficiency, absorption of calcium is impaired, which stimulates secretion of
parathyroid hormone (PTH) (ie, secondary hyperparathyroidism). PTH causes release of calcium from bone,
increases renal calcium reabsorption, and suppresses renal phosphate reabsorption. Circulating calcium is
restored to normal or mildly low levels, but phosphate levels are significantly reduced. In the bone, vitamin D
deficiency can lead to osteomalacia (ie, abnormal mineralization with bone pain) and osteoporosis (ie, decreased
bone density).

(Choice B) In some patients with malabsorption due to CD, calcium may be restored to normal, but this requires
elevated levels of PTH (ie, PTH would not be low).

(Choice C) Hypoparathyroidism is characterized by hypocalcemia, hyperphosphatemia, and low PTH; it is most


often a complication of thyroid surgery with inadvertent damage to the parathyroid glands. Symptoms are primarily
related to hypocalcemia (eg, paresthesia, muscle cramps). Malabsorptive syndromes are typically associated with
secondary hyperparathyroidism, not hypoparathyroidism.
(Choice D) 25-hydroxyvitamin D is converted to 1,25-dihydroxyvitamin D by 1-alpha-hydroxylase in the kidneys. In
patients with chronic kidney disease (CKD), production of 1,25-dihydroxyvitamin D is reduced, leading to
hypocalcemia and secondary hyperparathyroidism. However, unlike in CD and other malabsorptive disorders,
serum phosphate is elevated due to decreased renal phosphate clearance.

(Choice E) Primary hyperparathyroidism (eg, PTH-secreting parathyroid adenoma) would lead to high PTH,
hypercalcemia, and hypophosphatemia. Calcium is not elevated in patients with secondary hyperparathyroidism
due to malabsorption.

Educational objective:
Celiac disease (ie, immune-mediated hypersensitivity to gluten) commonly presents with diarrhea, abdominal pain,
and weight loss. The associated villous atrophy leads to malabsorption, causing fat-soluble vitamin (eg, A, D, E, K)
deficiencies and iron deficiency anemia. Vitamin D deficiency can lead to hypocalcemia, hypophosphatemia, and
secondary hyperparathyroidism.

Reference
• Hypocalcemia: updates in diagnosis and management for primary care.
Question #141

A 30-year-old man comes to the office due to a 3-month history of progressive left breast enlargement. He has had
no breast pain or nipple discharge. The patient has Klinefelter syndrome and takes testosterone replacement
therapy. Vital signs are normal. BMI is 30 kg/m2. Physical examination shows a 2.5-cm, nontender, firm subareolar
mass on the left breast. The right breast is normal. The testes are atrophic, and body hair is sparse. Which of the
following is the best next step in management of this patient?

A) Check serum FSH and LH levels

B) Increase testosterone dose

C) Obtain breast imaging

D) Prescribe a selective estrogen receptor modulator

E) Provide reassurance and recommend weight loss


Explanation
Correct Answer:

C) Obtain breast imaging

Male breast cancer

• Family history; BRCA1 or BRCA2 mutation


Risk factors
• Abnormal estrogen/androgen ratio: Klinefelter syndrome, obesity, cirrhosis

• Subareolar mass
Presentation • Skin & nipple dimpling, induration, ulceration
• Often detected at advanced stage

• Mammography
Diagnosis
• Biopsy: invasive ductal carcinoma (hormone receptor positive) most common

This patient has a firm, unilateral breast mass concerning for breast cancer. Klinefelter syndrome (47,XXY) is
associated with testicular fibrosis and is a common cause of primary hypogonadism. Testosterone production is
reduced, and the compensatory increase in gonadotropins causes increased expression of aromatase, leading to
increased conversion of testosterone to estradiol and an elevated estrogen/androgen ratio. In the breast, this
causes gynecomastia and an elevated risk of breast cancer.

Initial evaluation of suspected breast cancer in men is similar to that for women and includes breast imaging (eg,
mammography). Characteristic features can distinguish benign gynecomastia from malignancy:
• Gynecomastia can have variable density but is typically symmetric and centrally located with respect to the
nipple; it may have an indistinct margin with surrounding fat.

• In breast cancer, the mass is more likely eccentric (ie, off center) to the nipple; it also is more likely to have
well-defined or spiculated margins and may contain calcifications.

The diagnosis can be confirmed with biopsy (eg, fine-needle aspiration, core-needle biopsy). Invasive ductal
carcinoma is the most common histologic type, as it is in women, and most tumors are hormone receptor positive.

(Choice A) Gonadotropin (ie, FSH, LH) levels are used to differentiate primary from secondary/central
hypogonadism, and the levels are elevated in most patients with Klinefelter syndrome. Serum FSH and LH levels
are not needed for this patient with an established diagnosis of Klinefelter syndrome.

(Choice B) Androgen therapy can alleviate the symptoms of hypogonadism (eg, decreased libido) but has minimal
benefit for established gynecomastia. Malignancy should be ruled out first in this patient.

(Choice D) Selective estrogen receptor modulators (eg, tamoxifen) are commonly used as adjuvant therapy for
estrogen receptor–positive breast cancer and are occasionally used to treat symptoms of gynecomastia in
adolescents with Klinefelter syndrome. However, initiating any therapy before the suspicious (eg, firm, unilateral)
breast mass is evaluated with imaging is inappropriate.

(Choice E) Weight loss can be helpful for patients with pseudogynecomastia due to excessive breast fat.
However, this patient has a firm breast mass suspicious for cancer and requires additional evaluation.

Educational objective:
Klinefelter syndrome is a common cause of primary hypogonadism and is associated with an elevated estrogen/
androgen ratio. This causes gynecomastia and increases the risk for breast cancer, which can present as a firm,
unilateral breast mass. The initial evaluation of suspected breast cancer includes breast imaging (eg,
mammography).

Reference
• The endocrinology of male breast cancer.
Question #142

A 48-year-old overweight man comes to the physician for evaluation of a right foot ulcer that he first noticed 3
weeks ago. He has a history of hypertension, type 2 diabetes mellitus, and hypercholesterolemia. His current
medications include ramipril, aspirin, metformin, sitagliptin, and rosuvastatin. He has a 20-pack-year smoking
history and occasionally drinks alcohol but does not use illicit drugs. The patient is afebrile. His blood pressure is
132/80 mm Hg and pulse is 76/min. Examination shows a 1.7 x 2 cm ulcer on the plantar surface of the great toe.
Which of the following tests will best assess this patient's risk of foot ulcers?

A) 6-minute walk test

B) Ankle-brachial index

C) Capillary refill time

D) Knee reflex assessment

E) Monofilament testing
Explanation
Correct Answer:

E) Monofilament testing
This patient has a diabetic foot ulcer, which can eventually lead to nontraumatic lower limb amputation. Risk factors
for diabetic foot ulcers include diabetic neuropathy, previous foot ulceration, vascular disease, and foot deformity.
Diabetic neuropathy is the most common underlying cause and is found in >80% of patients with diabetic foot
ulcers. Neuropathy decreases pain sensation and pressure perception, it causes muscle imbalance leading to foot
deformities, and impairs the microcirculation and integrity of the skin.

Neuropathic ulcers most commonly occur in the feet under bony prominences, such as the metatarsal heads. They
typically have a punched-out or undermined border. Peripheral sensory neuropathy can be documented by testing
for pressure sensation with a 10-g monofilament (placed on the plantar surface at a right angle with increasing
pressure until filament buckles). Patients with neuropathy have a higher pressure threshold and loss of
monofilament sensation, which are associated with an increased risk of foot ulcerations. Other sensory deficits may
include decreased vibratory sensation (tested with a tuning fork), decreased pinprick pain, or decreased
temperature sensation.

(Choice A) The 6-minute walk test is an assessment of functional status that measures how far a patient can walk
in a standardized time. It is typically used for patients with chronic lung or heart disease or with chronic pain
conditions (eg, fibromyalgia, osteoarthritis). It is not as useful for patients with diabetic neuropathy or foot ulcers.

(Choice B) Peripheral arterial disease (PAD) can be assessed by calculating the ankle-brachial index (ABI).
However, ABI is primarily a measure of large vessel PAD and does not accurately assess small vessel disease,
which often contributes to ulcers in diabetic patients. In addition, arterial ulcers are usually located on the tips of the
toes, rather than the plantar surface.

(Choice C) Delayed capillary refill time (>3 seconds) indicates impaired limb perfusion and may suggest underlying
peripheral arterial disease. However, delayed capillary refill is a nonspecific finding, and is also seen in hypotension
or volume depletion.

(Choice D) Diabetic neuropathy can affect the large nerve fibers in the lower extremities, causing decreased/
absent ankle reflexes. However, the knee reflex is usually not affected.

Educational objective:
Risk factors for diabetic foot ulcers include diabetic neuropathy, previous foot ulceration, vascular disease, and foot
deformity. Diabetic neuropathy is the most important contributing factor and is found in >80% of patients with
ulcers. Monofilament testing predicts the risk of future ulcers.

Reference
• The Semmes Weinstein monofilament examination as a screening tool for diabetic peripheral neuropathy.

• Prediction of diabetic foot ulcer occurrence using commonly available clinical information: the Seattle
Diabetic Foot Study.

• What you can't feel can hurt you.


Question #143

A 17-year-old girl comes to the office for evaluation of irregular menses. The patient reports irregular menses since
menarche at age 13, and her menstrual cycle has become increasingly unpredictable. Over the past year, she has
had 5 menstrual periods. Her most recent period was 6 weeks ago and it lasted 10 days, with heavy bleeding and
large clots. The patient has also gained 10 kg (22 lb) over the last year and has been unable to lose weight despite
changes in her diet. She has no medical problems and has had no surgeries. She does not use tobacco, alcohol,
or illicit drugs. Blood pressure is 130/80 mm Hg and pulse is 76/min. BMI is 28 kg/m2. Physical examination shows
coarse hair along the chin. There is no thyromegaly or palpable neck masses. The abdomen is soft and nontender,
with no striae or palpable masses. Deep tendon reflexes of the extremities are normal, and no pedal edema is
present. Hemoglobin is 10.2 g/dL. TSH and prolactin levels are normal. A urine pregnancy test is negative. Which
of the following is the best next step for addressing this patient's irregular menses?

A) Order CT scan of the adrenal gland

B) Prescribe combined oral contraceptives

C) Prescribe letrozole

D) Prescribe leuprolide

E) Prescribe levothyroxine

F) Prescribe spironolactone
Explanation
Correct Answer:

B) Prescribe combined oral contraceptives

Polycystic ovary syndrome

• Androgen excess (eg, acne, male pattern baldness, hirsutism)


• Oligoovulation or anovulation (eg, menstrual irregularities)
Clinical features
• Obesity
• Polycystic ovaries on ultrasound

• ↑ Testosterone levels
Pathophysiology • ↑ Estrogen levels
• LH/FSH imbalance

• Metabolic syndrome (eg, diabetes, hypertension)


• Obstructive sleep apnea
Comorbidities
• Nonalcoholic steatohepatitis
• Endometrial hyperplasia/cancer

• Weight loss (first-line)


Treatment options
• Oral contraceptives for menstrual regulation
• Letrozole for ovulation induction

This patient's irregular menses, hirsutism (eg, coarse hair along chin), and weight gain are consistent with
polycystic ovary syndrome (PCOS). Patients with PCOS often have hyperandrogenism, diagnosed either
clinically (eg, hirsutism, severe acne, androgenic alopecia) or biochemically (eg, elevated testosterone levels).
Hyperandrogenism, and the subsequent hyperestrogenism (due to peripheral androgen conversion), results in
chronic anovulation. Anovulatory cycles cause irregular menses, decreased progesterone secretion, and
uncontrolled endometrial proliferation from unopposed estrogen. When menstrual periods do occur, they are often
associated with heavy bleeding that may result in anemia.

The first-line therapy for menstrual regulation is a combination of weight loss and combined estrogen/progestin
oral contraceptives. Combined oral contraceptives contain progesterone to stimulate endometrial differentiation
(ie, limit continued proliferation) and estrogen to stabilize the uterine lining, which restores normal cycles. In
addition, combined oral contraceptives reduce hirsutism by blocking adrenal androgen secretion and increasing
production of sex hormone-binding globulin, which binds and decreases free testosterone.

(Choice A) A CT scan of the adrenal gland can be used in the evaluation of adrenocortical tumors as causes of
hyperandrogenism. Patients with adrenocortical tumors typically have virilization (eg, clitoromegaly, deepening
voice), abdominal striae, and elevated dehydroepiandrosterone sulfate levels.

(Choice C) Letrozole, an aromatase inhibitor, is administered for ovulation induction in PCOS to treat infertility. It
does not regulate menses.

(Choice D) Leuprolide, a gonadotropin-releasing hormone agonist, is used in the treatment of endometriosis.


Patients with endometriosis have dysmenorrhea, chronic pelvic pain, and no evidence of hyperandrogenism.

(Choice E) Levothyroxine is used to treat hypothyroidism, which can cause irregular menses, but patients also
have elevated TSH levels. This patient has a normal TSH level.

(Choice F) Spironolactone, an androgen receptor antagonist, is indicated for treatment of hirsutism; it does not
regulate menstrual cycles.
Educational objective:
Polycystic ovary syndrome may present with irregular menses and signs of hyperandrogenism (eg, hirsutism).
Treatment involves weight loss and oral contraceptives containing estrogen and progestin to regulate menstrual
cycles.

Reference
• Diagnosis and treatment of polycystic ovary syndrome: an Endocrine Society clinical practice guideline.

• Polycystic ovary syndrome and combined oral contraceptive use: a comparison of clinical practice in the
United States to treatment guidelines.

• Clinical review: insulin sensitizers for the treatment of hirsutism: a systematic review and metaanalyses of
randomized controlled trials.

• ACOG Practice Bulletin No. 108: polycystic ovary syndrome.


Question #144

A 55-year-old man presents to the office for a routine check-up. He has no present complaints. His past medical
history is significant for a long history of hypertension. He does not smoke or consume alcohol. His current
medications are enalapril and hydrochlorothiazide. His blood pressure is 140/90 mm Hg and heart rate is 80/min.
Physical examination reveals a moderately overweight man (BMI = 27 kg/m2) with a waist circumference of 41
inches. The laboratory studies show:

Fasting blood glucose 112 mg/dL


Total cholesterol 220 mg/dL
LDL cholesterol 140 mg/dL
Triglycerides 240 mg/dL

Which of the following is the most important pathogenic factor for this patient's condition?

A) Impaired secretion of insulin

B) Low absolute values of insulin

C) Insulin resistance

D) Sympathetic hyperactivity

E) Insulin-mediated vasodilatation
Explanation
Correct Answer:

C) Insulin resistance

This vignette describes a patient with the typical clinical presentation of metabolic syndrome, which
includes hypertension, impaired fasting glucose, and dyslipidemia. Patients are also characteristically overweight
(as seen in this case), with predominantly central (abdominal) fat distribution that is reflected by an increased waist-
to-hip ratio. Insulin resistance plays a central role in the pathogenesis of metabolic syndrome. Metabolic syndrome
is diagnosed when at least 3 of the 5 following criteria are met:

1. Abdominal obesity (Men: Waist circumference >40 inches; Women: Waist circumference >35 inches)
2. Fasting glucose >100 - 110 mg/dL
3. Blood pressure > 130/80 mm Hg
4. Triglycerides >150 mg/dL
5. HDL cholesterol (Men: <40 mg/dL; Women: <50 mg/dL)

(Choices A and B) Most health care providers believe that insulin resistance is the first event in the development of
type 2 diabetes mellitus (DM); however, type 2 DM does not develop without a beta cell secretory defect. Increased
insulin secretion from the beta cells is required to cope with increased insulin resistance. If beta cells are able to
compensate for the increased insulin resistance, subjects generally remain normoglycemic at the expense of very
high insulin levels; however, if beta cells are unable to compensate fully, glucose intolerance and type 2 DM results.

(Choices D and E) Aside from problems with glucose control, insulin resistance is associated with several other
systemic effects (e.g., dyslipidemia, endothelial dysfunction, procoagulable state, increased sympathetic activity,
increased markers of inflammation, decreased uric acid excretion, increased sodium absorption, disordered
breathing and increased testosterone production from the ovaries). Although sympathetic activity is increased in
insulin resistance it does not play a central role in pathogenesis of metabolic syndrome. The endothelial dependent
vasodilatation is impaired in insulin resistance.
Educational Objective:
Insulin resistance typical for patients with central-type obesity is the key pathogenic factor in the development of
type-2 diabetes mellitus and associated abnormalities (hypertension, dyslipidemia).

*Extremely high yield question for the USMLE!!!


Question #145

A 34-year-old man with no previous medical history comes to the clinic due to worsening headaches and dizziness.
The headaches tend to occur during periods of stress and are sometimes associated with blurry vision and nausea.
The patient does not use tobacco, alcohol, or illicit drugs. Blood pressure is 170/100 mm Hg supine and 150/90
mm Hg standing 2 minutes, and pulse is 90/min and regular. BMI is 24 kg/m2. The remainder of the physical
examination is unremarkable. Fasting laboratory results are as follows:

Hemoglobin 15.6 g/L


Sodium 144 mEq/L
Potassium 4.5 mEq/L
Creatinine 1.0 mg/dL
Glucose 196 mg/dL
Calcium 8.6 mg/dL

Which of the following investigations should be ordered next?

A) CT scan of the abdomen

B) CT scan of the head

C) Dexamethasone suppression test

D) Plasma metanephrines

E) Renal vascular imaging

F) Serum glucagon level


Explanation
Correct Answer:

D) Plasma metanephrines

Pheochromocytoma

• Classic triad: episodic headache, sweating & tachycardia


Indications for testing • Resistant HTN or HTN accompanied by unexplained ↑ glucose
• Family history or familial syndrome (eg, MEN2, NF1, VHL)

• Urine or plasma metanephrine levels


Diagnostic approach
• Confirmatory abdominal imaging for ↑ metanephrines

Notable features • 10% bilateral, 10% extraadrenal, 10% malignant

• Preoperative alpha blockade prior to beta blockade


Management
• Laparoscopic or open surgical resection

HTN = hypertension; MEN2 = multiple endocrine neoplasia type 2; NF1 = neurofibromatosis type 1; VHL = von
Hippel-Lindau syndrome.

This patient's hypertension, episodic headaches, and unexplained hyperglycemia should raise suspicion for
pheochromocytoma. Pheochromocytomas are rare neuroendocrine tumors that arise from chromaffin cells and
release excess catecholamines (ie, dopamine, epinephrine, norepinephrine) into the bloodstream, resulting in
characteristic manifestations. Headaches are often episodic and present in 90% of patients. Paroxysmal
hypertension occurs in approximately half of patients and may be associated with orthostasis (possibly reflecting
low plasma volume). Vision changes, coinciding with episodes of headache and hypertension, can also occur.

Because catecholamines inhibit insulin secretion, hyperglycemia is also common. Therefore, pheochromocytoma
should be considered in patients who develop hyperglycemia that is atypical for both type 1 (ie, usually presents
with diabetic ketoacidosis) and type 2 (ie, rarely occurs in patients age <35 with normal BMI, as in this patient)
diabetes mellitus, especially when accompanied by other suggestive features, such as onset of hypertension
(particularly at a young age). Other indications for screening include: the classic triad of headache, sweating, and
tachycardia; resistant hypertension; or a familial syndrome (eg, multiple endocrine neoplasia type 2).

The initial step in evaluation for pheochromocytoma involves testing for urine or plasma levels of metanephrines,
which are products of catecholamine breakdown. Normal levels typically rule out the disease; elevated levels are
nonspecific and warrant additional testing, such as CT scan of the abdomen (Choice A), to confirm the presence
and localization of the suspected pheochromocytoma (usually on one or both adrenal glands).

(Choice B) CT scan of the head is typically indicated for headaches only when red-flag features (eg, focal
neurologic deficits, fever, new-onset headache in patient age >50) are present.

(Choice C) Dexamethasone suppression testing is indicated for suspected hypercortisolism (ie, Cushing
syndrome). Although hypertension and hyperglycemia are common, patients are usually overweight and have
characteristic findings on physical examination (eg, purple abdominal striae, fat pad on the upper back).

(Choice E) Renal vascular imaging may help diagnose fibromuscular dysplasia, which typically presents with
headaches and can cause hypertension. However, fibromuscular dysplasia usually occurs in women and would not
explain this patient's hyperglycemia.

(Choice F) A serum glucagon level is indicated for suspected glucagonoma. Although a glucagonoma would
cause hyperglycemia, it would not explain hypertension, episodic headaches, and vision changes.

Educational objective:
Pheochromocytoma commonly presents with episodic headaches and hypertension and can cause unexplained
hyperglycemia. Measurement of urine or plasma metanephrines is the initial step in diagnostic evaluation.

Reference
• Prevalence of clinically unsuspected pheochromocytoma. Review of a 50-year autopsy series.
Question #146

A 31-year-old woman comes to the office due to a 6.8-kg (15-lb) weight gain over the last few months. She has
been unable to lose weight despite rigorous dieting and regular exercise. The patient also has experienced
weakness and cannot lift weights that she was able to lift before the onset of her symptoms. Her menstrual periods
have been irregular for the last few months, and she has had increasing anxiety and insomnia for which she has
started seeing a clinical psychologist. Medical history is unremarkable. She drinks wine only on rare occasions and
quit smoking 7 years ago after a 5-pack-year history. On examination, blood pressure is 160/100 mm Hg and pulse
is 88/min and regular. Neurologic examination shows proximal muscle weakness. Dark terminal hair is present on
the lower abdomen. Fasting laboratory results are as follows:

Glucose 130 mg/dL


Sodium 142 mEq/L
Potassium 3.6 mEq/L
Chloride 98 mEq/L
Bicarbonate 28 mEq/L
Calcium 9.2 mg/dL
TSH 2.2 mIU/L

Which of the following is the most appropriate next step in evaluating this patient's condition?

A) Early-morning cortisol level

B) Overnight low-dose dexamethasone suppression test

C) Serum ACTH level


D) Serum aldosterone to plasma renin activity ratio

E) Serum testosterone level


Explanation
Correct Answer:

B) Overnight low-dose dexamethasone suppression test

Features of Cushing syndrome

• Central obesity
• Skin atrophy & wide, purplish striae
• Proximal muscle weakness
Clinical
• Hypertension
manifestations
• Glucose intolerance
• Skin hyperpigmentation (if due to ACTH excess)
• Depression, anxiety

• 24-hour urinary cortisol excretion


Diagnosis • Late-night salivary cortisol assay
• Low-dose dexamethasone suppression test

This patient has weight gain, psychiatric symptoms, hirsutism, hypertension, and hyperglycemia. This constellation
of findings is consistent with hypercortisolism (Cushing syndrome). This condition is most commonly due to
exogenous administration of glucocorticoids, but can also be due to an ACTH-producing pituitary tumor (Cushing
disease), ectopic ACTH production (eg, small cell lung cancer), or primary adrenal disease.

The initial step in the evaluation is to confirm hypercortisolism with a late-night salivary cortisol assay, 24-hour
urine free cortisol measurement, and/or overnight low-dose dexamethasone suppression test. Two of these
first-line tests should be abnormal to establish the diagnosis. If hypercortisolism is confirmed, ACTH levels are
measured to differentiate ACTH-dependent (ie, Cushing disease, ectopic ACTH) from ACTH-independent (eg,
adrenal adenoma) causes (Choice C).

(Choice A) Early-morning cortisol levels will be low or low-normal in patients with primary adrenal insufficiency.
However, there is considerable overlap in cortisol levels in normal subjects and those with Cushing syndrome, and
this test has low utility in the evaluation of hypercortisolism.

(Choice D) The ratio of plasma aldosterone to plasma renin activity is useful to evaluate primary
hyperaldosteronism, which typically presents with hypertension and hypokalemia. This patient's hyperglycemia and
psychiatric and menstrual symptoms are more consistent with hypercortisolism.

(Choice E) Hyperandrogenism can be seen in Cushing syndrome due to ACTH-induced adrenal androgen
production or co-secretion of cortisol and testosterone by an adrenal tumor. Testosterone levels may be useful in
evaluating hirsutism, but would not explain this patient's constellation of findings consistent with Cushing syndrome.

Educational objective:
The initial step in the evaluation of Cushing syndrome is to confirm hypercortisolism with a late-night salivary
cortisol assay, 24-hour urine free cortisol measurement, and/or overnight low-dose dexamethasone test. If
hypercortisolism is confirmed, ACTH levels are measured to differentiate ACTH-dependent from ACTH-independent
causes.

Reference
• The diagnosis of Cushing syndrome: an Endocrine Society Practice Guideline.
Question #147

A 38-year-old woman comes to the office with a 3-week history of weight loss, nausea, abdominal pain, and
postural dizziness. She traveled to Thailand 6 months ago and has felt fatigued since then. Medical history is
notable for moderate persistent asthma treated with an inhaled beta-2 agonist and inhaled corticosteroid. Over the
last 2 years, the patient has had several asthma exacerbations requiring oral prednisone. She also has
hypothyroidism treated with levothyroxine. The patient is married and is a stay-at-home mother. Blood pressure is
90/60 mm Hg and pulse is 96/min. Pharyngeal examination shows bilateral tonsillar enlargement. Skin
examination shows increased pigmentation at the palmar creases and mucous membranes as well as a few
patches of vitiligo. Initial laboratory testing shows mild hyponatremia and hyperkalemia with normal renal function.
Complete blood count is normal, but differential shows moderate eosinophilia. Follow-up testing confirms a low 8
AM serum cortisol. Which of the following is the most likely cause of this patient's adrenal insufficiency?

A) Adrenal hemorrhage

B) Adrenal tumor

C) Autoimmune adrenalitis

D) Exogenous glucocorticoid use

E) HIV infection

F) Tuberculosis
Explanation
Correct Answer:

C) Autoimmune adrenalitis

Primary adrenal insufficiency

• Autoimmune adrenalitis (most common)


Etiology • Infection (eg, tuberculosis)
• Metastatic infiltration

• Fatigue, weakness, anorexia/weight loss


• Nausea, vomiting, abdominal pain
Clinical
• Salt craving, postural hypotension
features
• Hyperpigmentation
• Acute adrenal crisis: confusion, hypotension/shock

Laboratory • Hyponatremia, hyperkalemia, eosinophilia


findings • Low morning cortisol, high ACTH

• Glucocorticoids (eg, hydrocortisone, prednisone)


Treatment
• Mineralocorticoids (eg, fludrocortisone)

ACTH = adrenocorticotropic hormone.


This patient has several features of chronic primary adrenal insufficiency (PAI), including weight loss, abdominal
pain, and fatigue. Hyperpigmentation is a common finding and is due to cosecretion of melanocyte-stimulating
hormone with ACTH (both are derived from proopiomelanocortin), which is increased in response to cortisol
deficiency. In addition, loss of mineralocorticoid production can cause hypotension, hyponatremia, and
hyperkalemia. Eosinophilia and hyperplasia of lymphoid tissue (eg, tonsils) are common but nonspecific findings.

Autoimmune adrenalitis is responsible for >90% of cases of PAI in developed countries. It is due to
autoantibodies against adrenal enzymes that are responsible for corticosteroid synthesis. Autoimmune adrenalitis
can occur as an isolated disorder or in association with other autoimmune syndromes (eg, primary hypothyroidism,
vitiligo).

(Choice A) Bilateral adrenal hemorrhage due to sepsis (eg, Neisseria meningitidis) can lead to PAI; however, this
is less common, and the onset of clinical features is typically acute and dramatic.

(Choice B) Hyperfunctioning adrenal tumors are a potential cause of hypercortisolism, but not PAI. Bilateral
adrenal metastasis can cause PAI but this is a rare occurrence.

(Choice D) Exogenous glucocorticoids can suppress pituitary secretion of ACTH and hypothalamic production of
corticotropin-releasing hormone, leading to central (secondary) adrenal insufficiency. In contrast to PAI, secretion of
melanocyte-stimulating hormone is not increased so patients do not display hyperpigmentation. Also, because
mineralocorticoid production is controlled by the renin-angiotensin system, aldosterone levels are normal and
hyperkalemia is not seen with central adrenal insufficiency.

(Choice E) PAI in HIV disease can be due to opportunistic infections (eg, cytomegalovirus, atypical mycobacteria,
fungal infections) or inhibition of glucocorticoid synthesis by azole antifungal drugs. However, this patient has no
apparent opportunistic infections, and her other autoimmune findings make autoimmune adrenalitis more likely.

(Choice F) PAI due to tuberculosis (TB) is typically seen with miliary disease and is uncommon in developed
countries. It is unlikely that this patient contracted TB and so rapidly developed manifestations of TB-induced PAI
following her trip to Thailand 6 months ago.

Educational objective:
Autoimmune adrenalitis is the most common cause of primary adrenal insufficiency (PAI) in developed countries.
The key differentiating features of PAI from central adrenal insufficiency are the hyperpigmentation and
hyperkalemia seen in PAI.

Reference
• Diagnosis and treatment of primary adrenal insufficiency: an Endocrine Society Clinical Practice Guideline.

• Tuberculosis of the adrenal gland: a case report and review of the literature of infections of the adrenal
gland.
Question #148

A 31-year-old woman comes to the office with a 3-month history of palpitations and weight loss. She weighs 110 kg
(243 lb); 3 months ago, she weighed 118 kg (260 lb). There is no associated dysphagia, neck pain, or hoarseness,
and her menstrual periods have been regular. The patient was previously healthy and her family history is
unremarkable. She does not use tobacco or alcohol. The patient is sexually active with one partner and uses a
barrier method for contraception. Temperature is 36.7 C (98.1 F), blood pressure is 140/90 mm Hg, pulse is 102/
min, and respirations are 20/min. The thyroid gland cannot be clearly felt on examination due to body habitus.
Ocular examination is unremarkable. Laboratory tests show an elevated serum free triiodothyronine and thyroxine
and low TSH. A urine pregnancy test is negative. Radioactive iodine scan reveals uptake of tracer only in the right
thyroid lobe, as shown in the image below.

Which of the following processes is responsible for this patient's elevated thyroid hormone levels?

A) Antibody-stimulated thyroid hormone production

B) Autonomous thyroid hormone production


C) Exogenous thyroid hormone intake

D) Pituitary dysfunction

E) Release of preformed thyroid hormone


Explanation
Correct Answer:

B) Autonomous thyroid hormone production


This patient has clinical thyrotoxicosis (eg, palpitations, tachycardia, weight loss) with elevated thyroid hormone
levels and a suppressed TSH, consistent with primary hyperthyroidism. The radioactive iodine scan reveals
unicentric uptake only in the right lobe, thereby confirming the diagnosis of toxic adenoma. Both toxic adenoma
and multinodular goiter have a nodular pattern of uptake; however, in contrast to the focal uptake of radioactive
iodine in toxic adenoma, uptake in patients with multinodular goiter shows a patchy distribution.

Normally, TSH is the major driver for the production of thyroid hormones. However, the hyperplastic cells in toxic
adenoma and toxic multinodular goiter overproduce thyroid hormone autonomously without TSH stimulation.
Some of these patients have activating somatic mutations of the TSH receptors on the hyperplastic follicular cells,
leading to TSH-independent activation of adenylyl cyclase.

(Choice A) Hyperthyroidism in Graves disease results from thyroid-stimulating autoantibody-induced activation of


TSH receptors on thyroid follicular cells. Radioiodine uptake is diffuse rather than focal, and patients usually have
additional extrathyroidal manifestations (eg, exophthalmos, pretibial myxedema).

(Choice C) Surreptitious intake of thyroid hormone is occasionally seen in patients who are attempting to lose
weight. In such cases, radioiodine uptake is diffusely reduced.

(Choice D) TSH is elevated or inappropriately normal in hyperthyroidism resulting from TSH-secreting pituitary
tumors.

(Choice E) Release of preformed thyroid hormone is seen in various forms of thyroiditis, including subacute (de
Quervain) thyroiditis, silent thyroiditis, and the transient hyperthyroid phase seen in some patients with chronic
lymphocytic (Hashimoto) thyroiditis. Radioactive iodine uptake in these cases is markedly reduced.

Educational objective:
Hyperthyroidism from toxic adenoma is due to autonomous production of thyroid hormones from the hyperplastic
thyroid follicular cells. The radioactive iodine uptake scan reveals uptake only in the hyperactive nodule, with
suppression of uptake in the rest of the gland.
Question #149

A 62-year-old man comes to the office for a follow-up visit. He recently had episodes of sweating, headache,
tremor, and palpitation while working in his backyard; his wife has noticed that he seems confused during these
episodes. Medical history is notable for long-standing type 2 diabetes mellitus treated with insulin glargine and
glipizide, hypertension, peripheral vascular disease, and hyperlipidemia. The patient also has chronic kidney
disease, and his renal function has declined significantly over the past 2 years despite optimal glycemic control.
Which of the following is most likely responsible for this patient's symptoms?

A) Angina pectoris

B) Catecholamine-secreting tumor

C) Insulin excess

D) Orthostatic hypotension

E) Panic attacks

F) Thyroid hormone overproduction

G) Vasovagal reaction
Explanation
Correct Answer:

C) Insulin excess
This patient is likely experiencing recurrent hypoglycemia, presenting with both neurogenic (eg, sweating, tremor,
palpitation due to autonomic activation) and neuroglycopenic (eg, confusion, impaired consciousness due to
inadequate CNS glucose supply) symptoms. Hypoglycemia generally occurs when circulating insulin levels are
excessive for the associated glucose levels.

As blood glucose levels fall during activity or fasting, insulin secretion normally decreases. At the same time, the
production of counterregulatory hormones (eg, glucagon) increases to stimulate glycogenolysis and
gluconeogenesis in the liver. However, patients taking exogenous insulin are vulnerable to exercise-induced
hypoglycemia because insulin continues to be released from the injection site despite falling glucose levels. In
addition, strenuous exertion may cause changes in skin perfusion that can lead to increased insulin absorption.

Patients with chronic kidney disease are at increased risk of hypoglycemia due to delayed clearance of insulin by
the kidneys. These patients often also have additional factors, such as altered nutrition and autonomic dysfunction,
that can delay the normal physiologic compensation.

(Choice A) Patients with diabetes and coronary artery disease may experience atypical anginal symptoms such as
fatigue, dyspnea, and nausea. However, this patient's mental confusion is more consistent with the
neuroglycopenic effects of hypoglycemia.

(Choice B) Pheochromocytoma is an uncommon catecholamine-secreting tumor that presents with headache,


palpitations, diaphoresis, and severe hypertension. However, confusion is less typical and more characteristic of
hypoglycemia.

(Choices D and G) Orthostatic hypotension or a vasovagal reaction would cause symptoms of presyncope (eg,
light-headedness, darkening of vision) or syncope. Tremor is also uncommon.

(Choice E) Panic attacks can present with tachycardia and diaphoresis, but confusion is atypical. In addition,
working in the yard is not a common trigger for panic attacks.

(Choice F) Hyperthyroidism can cause tachycardia and sweating, but symptoms are typically chronic rather than
episodic and are often associated with goiter.

Educational objective:
Patients taking exogenous insulin for diabetes are vulnerable to exercise-induced hypoglycemia because insulin
continues to be released from the injection site despite falling glucose levels. Patients with chronic kidney disease
are at increased risk due to delayed clearance of insulin by the kidneys.

Reference
• Hypoglycemia in patients with diabetes and renal disease.
Question #150

A 45-year-old man comes to the office for follow-up. He was diagnosed with diabetes mellitus 5 years ago and has
had good control during most of that time on a program of long-acting and sliding-scale short-acting insulin
analogues. However, the patient was admitted to the hospital 6 months ago for a hypoglycemic seizure, and his
home glucose readings have been over 200 mg/dL frequently since. Medical history is notable for chronic
pancreatitis related to excessive alcohol use. Vital signs and physical examination are normal. Hemoglobin A1c is
8.6%; a year ago, it was 7.2%. When the physician asks about the reasons for the change in glycemic control, the
patient says, "I'm not sure, doctor. I feel pretty good right now." Which of the following is the most appropriate
statement for initiating a discussion with this patient?

A) "Heavy alcohol use can cause high blood glucose. I am concerned that you might be drinking excessively
again."

B) "It is normal for diabetes to worsen over time. Increasing your insulin dose should get your blood sugar
back under control."

C) "Some people who have experienced frighteningly low blood glucose often reduce their insulin dose. Do
you think this is happening with you?"

D) "The last 6 months must have been very stressful for you. It will probably help to talk to a counselor about
your emotions."

E) "You have had good control with your insulin in the past. Is it becoming hard to give yourself a shot so
frequently?"
Explanation
Correct Answer:

C) "Some people who have experienced frighteningly low blood glucose often reduce their insulin dose. Do
you think this is happening with you?"

Hypoglycemia in insulin therapy

• Long-standing type 1 diabetes


Risk factors
• Pancreatogenic diabetes

• Mild to moderate (neurogenic symptoms): anxiety, tremor, palpitations, sweating


Manifestations
• Severe (neuroglycopenic symptoms): confusion, seizure, loss of consciousness

• Query patients regarding hypoglycemic symptoms


Management • Individualized glycemic targets
strategies • Flexible insulin dosing regimen
• Emergency glucagon kits

This patient had good glycemic control on a multidose insulin regimen but experienced loss of control after a severe
episode of hypoglycemia. Patients with a diminished glucagon response, such as those with pancreatogenic
diabetes (eg, chronic pancreatitis with fibrosis of the islets), can develop rapid and severe hypoglycemia with little
warning. Such episodes can lead to confusion, seizure, and loss of consciousness and can be fatal. Even mild
hypoglycemia can be distressing due to the associated autonomic activation, which can induce anxiety, tremor,
palpitations, and sweating.
Frequent or severe hypoglycemia can encourage behavioral changes to prevent recurrence, such as increased
caloric intake, leading to weight gain, or reduced adherence to insulin management. In light of this, it helps to ask
patients in a nonjudgmental manner about behavioral changes (eg, reducing insulin dose) they might be making to
avoid hypoglycemia. Management strategies include less-stringent glycemic targets, flexible insulin regimens, and
prescription of emergency glucagon kits.

(Choice A) Although heavy alcohol use can impair glycemic control, there are no other indicators of current alcohol
intake in this patient, and the time course suggests overcompensation for hypoglycemia as the primary cause.
Once the behavioral factors that can affect diabetes control are addressed, discussions regarding continuing care
for alcohol use disorder could be considered, if appropriate.

(Choice B) Although this patient's home readings and hemoglobin A1c suggest that tighter glycemic control is
desirable, the reasons for loss of control should be better understood before any changes are made to his insulin
regimen.

(Choice D) The patient reports no anxiety or mood symptoms to warrant referral to a mental health professional.
The physician should first attempt to understand the patient's reasons for loss of glycemic control and consider
changes in diabetic management before making a referral to another provider.

(Choice E) Insulin therapy can be cumbersome and painful to administer, leading to nonadherence. However, this
patient's loss of control dates to his hospital stay for hypoglycemia.

Educational objective:
Severe hypoglycemia can cause confusion, seizure, loss of consciousness, and death. Even mild hypoglycemia
can be distressing due to the associated autonomic activation (eg, anxiety, palpitations, sweating). Frequent or
severe hypoglycemia can encourage behavioral changes to prevent recurrence, such as increased caloric intake or
less rigorous insulin management, which may impair glycemic control.

Reference
• Fear of hypoglycemia: relationship to physical and psychological symptoms in patients with insulin-
dependent diabetes mellitus.
• Fear of hypoglycemia in adults with type 1 diabetes: impact of therapeutic advances and strategies for
prevention—a review.
Question #151

A 34-year-old man comes to the office due to lack of sexual desire and erectile dysfunction for 3 months. Medical
history is significant for opioid use disorder, for which he has been taking methadone for 2 years. The patient has
gained 4.5 kg (10 lb) over the past 6 months. Vital signs are normal. BMI is 24.5 kg/m2. On examination, the
testes are small and soft. There is no gynecomastia. Visual field and thyroid examination findings are normal.
Laboratory results are as follows:

Total testosterone 200 ng/dL (normal: 264-916)


LH 4 mU/mL
Prolactin 14 ng/mL
TSH 3.1 µU/mL

Which of the following is the most likely cause of this patient's symptoms?

A) Klinefelter syndrome

B) Medication adverse effect

C) Pituitary tumor

D) Rapid weight gain


Explanation
Correct Answer:

B) Medication adverse effect

Male secondary hypogonadism

• Fatigue, decreased libido


Clinical features • Testicular atrophy
• Laboratory: low testosterone, low/normal LH

• Pituitary tumors, hyperprolactinemia


• Medications: opioids, glucocorticoids, exogenous androgens (withdrawal phase)
Common causes • Infiltrative disease (eg, hemochromatosis)
• Chronic/severe illness
• Eating disorders, severe weight loss

This patient has secondary hypogonadism presenting with low libido, erectile dysfunction, and a low serum
testosterone level. Secondary (ie, pituitary, hypothalamic) hypogonadism can be differentiated from primary
hypogonadism by a low or inappropriately normal LH. In primary hypogonadism, the elevated gonadotropin (eg,
LH) levels stimulate aromatization of estrogen precursors (eg, androstenedione) from the adrenal gland, leading to
gynecomastia; in contrast, breast enlargement in secondary hypogonadism is less prominent.

Secondary hypogonadism is a common adverse effect of opioids. Opioids suppress GnRH and LH secretion,
leading to reduced Leydig cell testosterone synthesis, decreased spermatogenesis, and testicular atrophy.
Additional manifestations can include depression, hot flashes, and osteoporosis. In women, menstrual irregularities
are common.

(Choice A) Klinefelter syndrome (47,XXY) is a common chromosomal disorder that causes primary
hypogonadism. It manifests with small, firm testes; low serum testosterone; and elevated LH levels. Gynecomastia
is often present.

(Choice C) Pituitary mass lesions can cause secondary hypogonadism due to disruption of the hypophyseal portal
vessels or direct injury to the LH-secreting gonadotrophs. However, these disorders typically cause a mild to
moderate elevation in prolactin (20-200 ng/mL) due to disruption of dopaminergic neurons in the pituitary stalk.

(Choice D) Rapid or severe weight loss (eg, anorexia nervosa) can cause secondary hypogonadism due to the
suppression of GnRH secretion. Although overweight and obese patients may have mildly low testosterone levels,
moderate weight gain in a patient with a normal BMI would not cause symptomatic hypogonadism.

Educational objective:
Secondary hypogonadism in men presents with low libido, erectile dysfunction, a low serum testosterone level, and
a low or inappropriately normal LH level. Secondary hypogonadism is a common adverse effect of opioids, which
suppress GnRH and LH secretion, leading to reduced Leydig cell testosterone synthesis.

Reference
• Testosterone deficiency in non-cancer opioid-treated patients.
Question #152

A 27-year-old woman comes to the office due to a 2-month history of dark hair growth on her cheeks, chin, and
upper lip. Her last menstrual period was 12 weeks ago, but she did not seek medical care because a home
pregnancy test was negative. The patient is gravida 0 para 0; her menstrual cycles began at age 11 and have
previously been regular. She has no associated weight changes, visual disturbances, skin striae, headaches, or
proximal muscle weakness. She takes no medications, including over-the-counter medications. Family history is
unremarkable. Physical examination shows a BMI of 24 kg/m2. Her skin is oily with acne on the forehead and
cheeks. Dark terminal hairs are noted on the upper lip, cheeks, chin, chest, and lower abdomen. Examination of
the genitals shows clitoromegaly. Abdominal examination is normal. Pelvic ultrasound reveals a normal uterus and
ovaries. CT scan of the abdomen reveals a left adrenal mass. Which of the following patterns of plasma hormone
levels is likely to be present in this patient?

Dehydroepiandrosterone sulfate Testosterone Luteinizing hormone

A)

High High Low

B)

Low Low Low

C)

Normal High Low

D)

Normal or high High High


E)

Normal Normal Normal


Explanation
Correct Answer:

A)

High High Low

Causes of hirsutism in women

Etiology Clinical features

• Oligomenorrhea, hyperandrogenism, obesity


Polycystic ovary syndrome
• Associated with type 2 diabetes, dyslipidemia, hypertension

• Normal menstruation
Idiopathic hirsutism
• Normal serum androgens

Nonclassic • Similar to polycystic ovary syndrome


21-hydroxylase deficiency • Elevated serum 17-hydroxyprogesterone

• More common in postmenopausal women


Androgen-secreting ovarian tumors,
• Rapidly progressive hirsutism with virilization
ovarian hyperthecosis
• Very high serum androgens
• Obesity (usually of the face, neck, trunk, abdomen)
Cushing syndrome
• Increased libido, virilization, irregular menses

Women normally produce a number of androgens, including testosterone (T), androstenedione (AS),
dehydroepiandrosterone (DHEA), and dehydroepiandrosterone sulfate (DHEAS). AS, DHEA, and T are
produced by both the ovaries and the adrenals. In contrast, DHEAS is produced predominantly in the adrenal
glands.

This patient has an adrenal mass with rapidly progressive (ie, over weeks) hirsutism (excess terminal hair growth)
and virilization (eg, clitoromegaly), suggesting very high androgen levels due to an androgen-producing
neoplasm. Most androgen-producing adrenal tumors overproduce DHEAS. Although DHEA and DHEAS are used
as diagnostic markers, they have negligible androgenic activity, and the clinical features are due to the conversion
of DHEA and DHEAS to more potent androgens (ie, AS and T). Therefore, this patient would most likely have
elevated DHEAS and T levels. LH would be low due to negative feedback by T.

(Choice B) Low androgen with low LH levels would be seen in hypopituitarism with hypogonadotropic
hypogonadism, which can cause amenorrhea but would have other manifestations of ovarian insufficiency (eg, hot
flashes, vaginal dryness) rather than hirsutism and virilization.

(Choice C) Virilizing ovarian tumors can release large amounts of T, leading to suppression of LH release. DHEAS
would be normal as adrenal androgen secretion is regulated primarily by ACTH rather than LH. However, this
patient's pelvic ultrasound does not reveal an ovarian tumor.

(Choice D) Polycystic ovary syndrome (PCOS) can cause hyperandrogenism with ovulatory dysfunction. T and LH
would be increased, and some patients have increased adrenal androgen production as well. However, this
patient's ovaries have normal morphology, and rapidly progressive hirsutism with virilization would be unusual in
PCOS.

(Choice E) Patients with idiopathic hirsutism have normal androgen and LH levels. However, symptoms would be
more slowly progressive, and menstrual cycles would be normal.
Educational objective:
Rapidly progressive hirsutism with virilization suggests very high androgen levels due to an androgen-producing
neoplasm. Elevated dehydroepiandrosterone sulfate levels are seen in androgen-producing adrenal tumors.
Question #153

A 52-year-old woman comes to the office for follow-up of type 2 diabetes mellitus. The patient has had diabetes for
10 years, and it is currently managed with metformin, canagliflozin, dulaglutide, and insulin glargine. Home morning
fasting blood glucose levels have ranged between 85 and 115 mg/dL over the past 3 months. Blood pressure is
128/72 mm Hg and pulse is 79/min. BMI is 41 kg/m2. Laboratory results are normal except for a hemoglobin A1c of
8.1% and a nonfasting glucose of 290 mg/dL. Which of the following best explains the suboptimal glycemic control
in this patient?

A) Dawn phenomenon

B) Inadequate basal insulin coverage

C) Insulin-neutralizing antibodies

D) Postprandial hyperglycemia
Explanation
Correct Answer:

D) Postprandial hyperglycemia

Hemoglobin A1c reflects average glucose levels over the life-span of hemoglobin (about 3 months) and should be
≤7% for most patients with type 2 diabetes mellitus.
Hemoglobin A1c is influenced by both fasting and postprandial glucose concentrations. This patient's hemoglobin
A1c is elevated despite an acceptable morning fasting glucose (target range: 80-130 mg/dL). She also has a
markedly elevated nonfasting glucose (290 mg/dL). Therefore, this patient likely has frequent postprandial
hyperglycemia, leading to impaired overall glycemic control.

Management requires better control of postprandial hyperglycemia without inducing fasting hypoglycemia. This
goal can generally be achieved through a basal-bolus insulin regimen:

• Basal insulin (ie, insulin glargine) supplies a steady influx of insulin (ie, without a true peak), which
maintains fasting glucose control through suppression of hepatic gluconeogenesis.

• Rapid-acting insulin (eg, insulin aspart) is typically given just before meals; it peaks within 15-30 minutes
and controls postprandial hyperglycemia.

This patient's already controlled fasting glucose (eg, 85-115 mg/dL) suggests adequate basal insulin coverage. To
adequately treat her postprandial hyperglycemia, rapid-acting insulin should be added (Choice B).

(Choice A) The dawn phenomenon causes early-morning (2-8 AM) fasting hyperglycemia due to the circadian
increase in growth hormone and cortisol secretion; in many patients, it can be managed by increasing the basal
insulin dose. This patient's suboptimal glycemic control results from inadequate postprandial control rather than
early-morning hyperglycemia (ie, her fasting glucose taken in the morning is within normal levels).

(Choice C) Insulin-neutralizing antibodies against exogenous insulin may be suspected when a patient has fasting
and nonfasting hyperglycemia despite very high insulin doses; however, this patient's fasting glucose is adequately
controlled with basal insulin. In general, insulin-neutralizing antibodies are rare when a recombinant insulin
analogue (eg, glargine) is used but was more common in the past with bovine and porcine insulin.

Educational objective:
Patients with diabetes mellitus who have an elevated hemoglobin A1c despite normal fasting glucose levels may
have postprandial hyperglycemia. A combined regimen, including a long-acting basal insulin (to control fasting
glucose) and a rapid-acting premeal insulin (to control postprandial glucose), can improve glycemic control.

Reference
• Diagnosis and management of diabetes: synopsis of the 2016 American Diabetes Association standards of
medical care in diabetes.
Question #154

A 24-year-old woman comes to the office due to episodic sweating, palpitation, and tremor for the past 3 weeks.
The symptoms typically occur after she skips meals and are relieved by eating carbohydrate-containing foods. The
patient has had no abdominal pain, weight change, or excessive urination. Three months ago, she was diagnosed
with diabetes mellitus, and treatment with insulin degludec was started. She refuses to monitor glucose levels at
home. Medical history is notable for chronic pancreatitis resulting from adult-onset cystic fibrosis; the patient has no
other organ involvement from this disease. Her only other medication is oral lipase for pancreatic exocrine
insufficiency. Examination shows no abnormalities. BMI is 24 kg/m2. Laboratory results show a hemoglobin A1c of
7.5% and serum creatinine of 0.75 mg/dL. Which of the following is the primary contributor to this patient's
symptoms?

A) Decreased clearance of insulin

B) Depleted hepatic glycogen stores

C) Glucagon deficiency

D) Malabsorption of nutrients

E) Upregulation of insulin receptors in the peripheral tissues


Explanation
Correct Answer:

C) Glucagon deficiency

Abnormalities of the cystic fibrosis transmembrane conductance regulator (CFTR) gene cause formation of viscous
mucus, which can lead to obstruction of the pancreatic duct with inflammation and fibrosis of the surrounding gland.
In addition to exocrine pancreatic insufficiency, destruction of the insulin-secreting beta cells in the pancreatic
islets can develop, resulting in pancreatogenic diabetes. In such cases, exogenous insulin is required.
Pancreatogenic diabetes is also common in patients with pancreatectomy or chronic pancreatitis of other causes
(eg, alcohol).

Patients with pancreatogenic diabetes typically also lose glucagon-secreting alpha cells. Because glucagon is
the primary counterregulatory hormone to insulin, these patients are at risk for insulin-induced hypoglycemia,
which may be rapid and severe. Loss of alpha cell reserve is also seen in patients with long-standing type 1
(idiopathic) diabetes.

This patient is likely experiencing recurrent hypoglycemia due to unopposed insulin analogue therapy. Management
requires a conservative insulin regimen, and she should be advised that careful monitoring of home glucose is
necessary. In addition, she should be issued an emergency glucagon kit for use in severe hypoglycemia.

(Choice A) Insulin degludec is an insulin analogue that forms polymers at the injection site, causing delayed
uptake from the site and a prolonged duration of action. The insulin is cleared primarily by the kidneys; this patient
has normal renal function and clearance is likely normal.

(Choices B and D) Malabsorption in patients with cystic fibrosis primarily affects fats and proteins, not
carbohydrates. At least 24-48 hours of fasting is typically required to deplete hepatic glycogen stores, and this
patient has no other indication of reduced carbohydrate reserve. In patients with intact counterregulatory systems
(ie, glucagon, epinephrine), as glycogen stores are depleted, hepatic glucose production continues via
gluconeogenesis. However, this effect is inadequate in this patient due to loss of glucagon.

(Choice E) Excessive circulating insulin can cause hypoglycemia and leads to down-regulation, not upregulation,
of insulin receptor expression.

Educational objective:
Chronic pancreatitis or pancreatic resection can cause loss of insulin-producing beta cells, leading to
pancreatogenic diabetes; exogenous insulin is required. Patients typically also lose glucagon-secreting alpha cells
and are at risk for rapid and severe hypoglycemia.
Question #155

A 52-year-old woman comes to the clinic for follow-up of type 2 diabetes mellitus that was diagnosed 6 months ago
after she was hospitalized for cellulitis of the right lower leg. The patient's diabetes is managed with insulin in
addition to diet and exercise. Her only other medical condition is hypertension, for which she takes antihypertensive
medication. The patient's mother had systemic lupus erythematosus and died at age 60. Serum creatinine is 1.7
mg/dL. Urine albumin/creatinine ratio is elevated at 190 mg/g and was also elevated 3 months ago. Which of the
following additional findings would most strongly support a diagnosis of diabetic nephropathy in this patient?

A) Discrepancy in right and left kidney size

B) Left ventricular hypertrophy

C) Microscopic hematuria

D) Rapidly progressive renal dysfunction

E) Retinal neovascularization
Explanation
Correct Answer:

E) Retinal neovascularization

Diabetic nephropathy (DN) is exceedingly common in patients with both type 1 and type 2 diabetes mellitus and is
characterized by persistent albuminuria (≥30 mg/g) and/or decreased glomerular filtration rate (GFR) (eg, elevated
creatinine). DN is typically slowly progressive for years before clinical signs occur.

The diagnosis is usually made clinically, with renal biopsy reserved for patients with an unclear presentation. In the
absence of signs suggesting another etiology (eg, casts in the urine, rapid progression of renal dysfunction), the
diagnosis can be presumed in patients who have persistent albuminuria and/or decreased GFR and either of the
following:

• Prolonged history of diabetes: Type 1 diabetes is typically clinically obvious at onset; therefore, DN may
be assumed those with a disease duration of ≥5 years. In contrast, type 2 diabetes is often asymptomatic,
so patients may have this disorder for years before the diagnosis, making the duration of the disease difficult
to calculate.

• Proliferative diabetic retinopathy (PDR): Both PDR (ie, retinal neovascularization) and DN are chronic
microvascular complications of diabetes due to persistent hyperglycemia, so the presence of PDR
correlates with diabetic kidney disease.

Therefore, patients may be diagnosed with DN if there is concomitant PDR, regardless of the length of time since
the diagnosis.

(Choice A) Renal size discrepancy is seen in patients with unilateral chronic kidney disease (eg, renal artery
stenosis, ureteral obstruction). With prolonged DN, both kidneys would likely be shrunken and atrophic.

(Choice B) Left ventricular hypertrophy correlates with chronic, uncontrolled hypertension. Although uncontrolled
hypertension may be seen in patients with chronic kidney disease, it is not specific for DN.
(Choice C) Although microscopic hematuria may occur with DN, it is nonspecific and may be seen in a variety of
diseases (eg, malignancy, nephritic syndromes).

(Choice D) DN usually develops over a prolonged period (years). Rapidly progressive renal dysfunction suggests
an alternate diagnosis (eg, lupus nephritis) and further workup. Renal biopsy may be indicated.

Educational objective:
Diabetic nephropathy (DN) is characterized by persistent albuminuria (≥30 mg/g) and/or decreased glomerular
filtration rate. In the absence of signs suggestive of another etiology, the diagnosis can be presumed in patients
with renal dysfunction and either a prolonged history of diabetes (≥5 years for type 1 diabetes) or proliferative
diabetic retinopathy, which correlates with the presence of DN.
Question #156

A 40-year-old man comes to the office for follow-up of hypertension. He was diagnosed with mild hypertension 6
months ago and has had good control with regular exercise and weight loss. The patient feels well and takes no
medications. He does not smoke and drinks a few glasses of wine per week. The patient's medical history is
otherwise unremarkable. His mother has osteoporosis that is treated with oral alendronate. On examination,
inspection and palpation of the neck show no masses. Laboratory results are as follows:

Sodium 140 mEq/L


Potassium 4.0 mEq/L
Bicarbonate 25 mEq/L
Chloride 101 mEq/L
Calcium 11.8 mg/dL
Albumin 4.0 g/dL
Phosphorus 2.2 mg/dL
Creatinine 1.2 mg/dL
25-hydroxyvitamin D 38 ng/mL (normal: 25-80 ng/mL)

Serum parathyroid hormone level is 814 pg/mL. Further testing shows a high 24-hour urine calcium excretion of
325 mg. DXA scan reveals normal bone mineral density. Renal ultrasound shows multiple small (<5 mm) stones in
both kidneys. Which of the following is the most appropriate next step in management of this patient?

A) Initiate bisphosphonate therapy

B) Obtain CT scan of the chest, abdomen, and pelvis

C) Refer for parathyroid imaging and parathyroidectomy


D) Repeat serum calcium measurement in 3-6 months

E) Start vitamin D supplementation


Explanation
Correct Answer:

C) Refer for parathyroid imaging and parathyroidectomy

Primary hyperparathyroidism

• Parathyroid adenoma (most common), hyperplasia,


Etiology carcinoma
• Increased risk in MEN types 1 & 2A

• Asymptomatic (most common)


• Mild, nonspecific symptoms (eg, fatigue, constipation)
Symptoms
• Abdominal pain, renal stones, bone pain, neuropsychiatric
symptoms

• Hypercalcemia
Diagnostic
• Elevated or inappropriately normal PTH
findings
• Elevated 24-hour urinary calcium excretion

• Age <50
Indications for
• Symptomatic hypercalcemia
parathyroidectomy
• Complications: Osteoporosis (T-score <−2.5, fragility
fracture), nephrolithiasis/calcinosis, CKD (GFR <60 mL/min)
• Elevated risk of complications: Calcium >1 mg/dL above
normal, urinary calcium excretion >400 mg/day

CKD = chronic kidney disease; GFR = glomerular filtration rate; MEN = multiple
endocrine neoplasia; PTH = parathyroid hormone.

This patient has hypercalcemia, hypophosphatemia, and an elevated parathyroid hormone (PTH) level
consistent with primary hyperparathyroidism (PHPT). PTH causes increased reabsorption of calcium from the
distal tubule, but net urinary calcium excretion is increased due to excess resorption of calcium from bones. By
contrast, familial hypocalciuric hypercalcemia is characterized by hypercalcemia with an elevated PTH but low
urinary calcium excretion (often <100 mg/24 hr).

Parathyroidectomy is recommended for patients with symptomatic hypercalcemia. It is also recommended for
patients, such as this one, with complications (eg, osteoporosis, nephrocalcinosis, nephrolithiasis, impaired renal
function) or at significant risk for complications (eg, serum calcium ≥1 mg/dL above normal, urinary calcium
excretion >400 mg/24 hr). In addition, younger patients (age <50) such as this one are likely to have complications
during their lifetime and should be offered surgery.

This patient is relatively young and has nephrolithiasis (likely as a complication of PHPT); he should therefore be
referred for parathyroidectomy. Parathyroid imaging (eg, sestamibi scan, ultrasonography) helps optimize the
surgical approach by potentially determining the affected side and evaluating for the possibility of a minimally
invasive intervention.

(Choice A) Parathyroidectomy is the treatment of choice for PHPT. However, bisphosphonate therapy can be
used in patients who decline surgery and have osteopenia or osteoporosis.

(Choice B) Extended CT imaging would be indicated for suspected hypercalcemia of malignancy, which typically
presents with severe, symptomatic hypercalcemia, low PTH, and elevated PTH-related protein. Imaging to evaluate
for multiple endocrine neoplasia syndromes is recommended only if there is evidence of multiglandular
hyperparathyroidism or additional endocrine tumors.

(Choice D) Continued observation is appropriate in older asymptomatic patients with near-normal calcium levels
and normal bone density.

(Choice E) Vitamin D supplementation is not recommended in patients with PHPT who do not have overt vitamin D
deficiency as it may exacerbate hypercalcemia and hypercalciuria. This patient has a normal vitamin D level.

Educational objective:
The most common presentation of primary hyperparathyroidism is asymptomatic hypercalcemia with an elevated
parathyroid hormone level. Parathyroidectomy is recommended for patients with symptomatic hypercalcemia and
those with complications (eg, nephrolithiasis) or at increased risk for complications. Younger patients (age <50) are
likely to have complications during their lifetime and should be offered surgery.

Reference
• Primary hyperparathyroidism: an update.

• Guidelines for the management of asymptomatic primary hyperparathyroidism: summary statement from the
third international workshop.

• Sestamibi (99mTc) scan as a single localization modality in primary hyperparathyroidism and factors
impacting its accuracy.
Question #157

A 65-year-old woman comes to the office due to concerns about her risk for osteoporotic fractures. She reports that
her sister takes medication for osteoporosis. Medical history is notable for hypothyroidism treated with
levothyroxine and hypertension treated with hydrochlorothiazide. The patient underwent hysterectomy with
salpingo-oophorectomy at age 51. She has a 10-pack-year history of cigarette smoking but quit at age 40, and she
drinks 2 or 3 glasses of wine a week. Vital signs are normal. BMI is 23 kg/m2. Examination of the heart, lungs, and
abdomen is normal. Pelvic examination shows vaginal atrophy. Laboratory evaluation is normal. DXA scan
reveals a T-score of −2.5 at the lumbar spine and −1.9 at the femoral neck, which is consistent with osteoporosis.
In addition to calcium and vitamin D supplementation, which of the following is the most appropriate treatment for
this patient?

A) Nasal calcitonin

B) No additional treatment, repeat DXA in a year

C) Oral alendronate

D) Oral raloxifene

E) Transdermal estrogen
Explanation
Correct Answer:

C) Oral alendronate

Medications for osteoporosis

• First-line treatment for most patients


• Taken with water on an empty stomach an hour before food & other
Bisphosphonates
medications
(eg, alendronate,
• Not recommended for patients with renal impairment
risedronate)
• Atypical fractures possible with prolonged use
• Oral & parenteral options available

• Risk of infection & skin reactions


Denosumab
• Close monitoring for hypocalcemia needed

Anabolic agents • Useful in severe osteoporosis


(eg, teriparatide) • Monitor serum calcium, uric acid & renal function

• Modest reduction in fracture risk


Nasal calcitonin
• Reduces pain from fracture
Selective estrogen • Less effective than bisphosphonates
receptor modulators • May lower risk of breast cancer
(eg, raloxifene) • Increased risk of DVT

DVT = deep vein thrombosis.

This woman has multiple risk factors for osteoporosis, including age ≥65, postmenopausal status, smoking history,
and family history of osteoporosis. Her screening DXA scan shows a T-score of −2.5, which is consistent with
osteoporosis. Initial management should address lifestyle factors, including regular weight-bearing exercise,
avoidance of tobacco and excessive alcohol use, and adequate intake of calcium and vitamin D (using
supplements as needed).

In addition, women with osteoporosis confirmed by DXA (ie, T-score ≤−2.5) or with an osteoporotic fragility fracture
warrant pharmacologic antiresorptive therapy. Antiresorptive therapy is also appropriate for patients with
osteopenia (T-score between −1 and −2.5) who are at high risk for fragility fracture.

First-line treatment for most patients with postmenopausal osteoporosis is a bisphosphonate (eg, alendronate,
risedronate). Bisphosphonates preserve bone density and decrease the risk of vertebral and hip fracture. After
initiation of bisphosphonate therapy, bone density is typically remeasured after 2 years to assess the response.
Most experts recommend discontinuation after 5 years due to a risk of atypical fracture with prolonged use.

(Choice A) Nasal calcitonin has relatively modest antiresorptive properties and is not recommended as first-line
treatment for osteoporosis. However, it often has benefits for pain relief in patients with osteoporotic fracture.

(Choice B) This patient has confirmed osteoporosis, warranting bisphosphonate therapy. Follow-up testing is
typically performed within 2-5 years; shorter testing intervals (eg, 1 year) are used primarily for patients with
accelerated bone loss (eg, glucocorticoid therapy).

(Choice D) Raloxifene is a selective estrogen receptor modulator. It is less effective than bisphosphonates for
treating osteoporosis and is associated with an increased risk for venous thromboembolism. However, raloxifene is
associated with a reduced risk for invasive breast cancer and may be useful for women with high breast cancer risk.

(Choice E) Estrogen replacement therapy has moderate benefits in maintaining bone density but is associated with
an increased risk for breast cancer. It is recommended primarily for women with severe vasomotor symptoms (eg,
hot flashes) and low risk for breast cancer. This patient's vaginal mucosal atrophy is a common finding in
postmenopausal women and does not warrant estrogen therapy in the absence of significant symptoms.

Educational objective:
Bisphosphonates (eg, alendronate, risedronate) are the first-line treatment for most women with postmenopausal
osteoporosis. After initiation of bisphosphonate therapy, bone density is typically remeasured after 2 years to
assess response. Treatment is usually discontinued after 5 years due to a risk of atypical fracture with prolonged
use.
Question #158

A 55-year-old man comes to the office for follow-up. The patient has a 5-year history of type 2 diabetes mellitus and
hypertension and had an acute myocardial infarction 2 years ago. He reports no symptoms but says he is
frustrated at his inability to lose weight despite lifestyle modifications. The patient is on the maximum dose
of metformin. He does not use tobacco, alcohol, or recreational drugs. Blood pressure is 126/70 mm Hg and pulse
is 74/min. BMI is 32 kg/m2. Physical examination is otherwise unremarkable. Laboratory results are as follows:

Blood urea nitrogen 14 mg/dL


Serum creatinine 1.0 mg/dL
Fasting blood glucose 156 mg/dL
Hemoglobin A1c 8.3%

An additional medication is prescribed to improve glycemic control, assist with weight loss, and reduce
cardiovascular mortality. Which of the following medications was most likely prescribed for this patient?

A) Dipeptidyl peptidase-4 inhibitor

B) Glucagon-like peptide-1 agonist

C) Long-acting insulin

D) Sulfonylurea

E) Thiazolidinedione
Explanation
Correct Answer:

B) Glucagon-like peptide-1 agonist

Type 2 diabetes and comorbid cardiovascular disease

• Smoking cessation, regular exercise


Lifestyle
• Reduced saturated fat, refined sugar intake

• GLP-1 receptor agonists (eg, liraglutide)


Cardioprotective antidiabetic agents
• SGLT-2 inhibitors (eg, empagliflozin)

• Statins
Lipid/antiplatelet therapy
• Low-dose aspirin

• Goal BP <130-140/80-90 mm Hg
Blood pressure control
• ACE inhibitor/ARB preferred

ACE = angiotensin converting enzyme; ARB = angiotensin receptor blocker; BP = blood pressure; GLP-1 =
glucagon-like peptide-1; SGLT-2 = sodium-glucose cotransporter-2.

Obesity and cardiovascular disease are common comorbidities in patients with type 2 diabetes mellitus. This
patient has suboptimal glycemic control on metformin (a first-line diabetic treatment) and requires additional
therapy; he should ideally be given an add-on diabetic agent that reduces cardiovascular mortality, induces
weight loss, and carries a low risk for hypoglycemia (which increases coronary ischemia–related mortality).
Suitable agents include glucagon-like peptide-1 receptor agonists (GLP-1RAs) and certain sodium-glucose
cotransporter-2 (SGLT-2) inhibitors:

• GLP-1RAs (eg, exenatide, liraglutide) regulate glucose by increasing glucose-dependent insulin release
(ie, low risk for hypoglycemia) and suppressing glucagon. They also slow gastric emptying, which
decreases appetite and causes weight loss.

• SGLT-2 inhibitors (eg, canagliflozin, empagliflozin) increase renal glucose and sodium excretion, leading to
mild weight loss and mild diuresis. Because renal glucose filtration is proportional to serum glucose level,
hypoglycemia is rare. These agents also reduce heart failure–related mortality and progression of diabetic
nephropathy.

(Choice A) Dipeptidyl peptidase-4 inhibitors (eg, sitagliptin) prevent degradation of GLP-1 and therefore function
like GLP-1RAs. However, they are less potent than GLP-1RAs and are not associated with weight loss or
cardioprotective effects.

(Choices C and D) Adding insulin or a sulfonylurea (eg, glimepiride) would improve glycemic control but is
associated with significant weight gain and risk for hypoglycemia. Compared to GLP-1RAs or SGLT2 inhibitors,
these agents are less preferred for patients with obesity and coronary artery disease.

(Choice E) Thiazolidinediones (eg, pioglitazone) are associated with weight gain, fluid retention, and increased risk
for heart failure in patients with underlying heart disease. Pioglitazone should be avoided in this patient.

Educational objective:
Obesity and cardiovascular disease are common comorbidities in patients with type 2 diabetes mellitus. In these
patients, glucagon-like peptide-1 receptor agonists and/or certain sodium-glucose cotransporter-2 inhibitors can be
added to decrease cardiovascular mortality, induce weight loss, and minimize the risk for hypoglycemia.

Reference
• Dulaglutide and cardiovascular outcomes in type 2 diabetes (REWIND): a double-blind, randomised
placebo-controlled trial.

• Effect of glucagon-like peptide-1 receptor agonists on all-cause mortality and cardiovascular outcomes: a
meta-analysis.

• Cardiovascular and renal outcomes with empagliflozin in heart failure.


Question #159

A 55-year-old man comes to the office for follow-up of hypertension. The diagnosis was made 3 months ago after
the patient experienced a transient ischemic attack. He has occasional headaches and general fatigue but
otherwise feels well. Current medications include aspirin, lisinopril, and low-dose chlorthalidone. Blood pressure is
157/95 mm Hg and pulse is 69/min. BMI is 26 kg/m2. Cardiac auscultation reveals an S4 heart sound. There are
no carotid or abdominal bruits on examination. Pulses are 2+ in all extremities. Laboratory results are as follows:

Potassium 3.1 mEq/L


Creatinine 0.9 mg/dL
Plasma renin activity low

Which of the following best explains this patient's laboratory findings?

A) Chlorthalidone adverse effect

B) Lisinopril adverse effect

C) Malignant hypertension

D) Primary hyperaldosteronism

E) Renovascular hypertension
Explanation
Correct Answer:

D) Primary hyperaldosteronism

Adrenocortical causes of hypertension

Plasma Plasma renin Aldosterone/


Clinical examples
aldosterone activity renin ratio

Primary • Bilateral adrenal hyperplasia


↑ ↓ ↑
hyperaldosteronism • Adrenal adenoma

• Renovascular HTN
Secondary
↑ ↑ ↔ • Malignant HTN
hyperaldosteronism
• Renin-secreting tumor

• Cushing syndrome
Nonaldosterone • Exogenous mineralocorticoids
↓ ↓ ↔
mediated (eg, fludrocortisone)
• Deoxycorticosterone excess*
*Includes deoxycorticosterone-producing adrenal tumors (rare) & relatively uncommon forms of
congenital adrenal hyperplasia (eg, 11ꞵ-hydroxylase deficiency).

HTN = hypertension.

This patient with low plasma renin likely has secondary hypertension due to primary hyperaldosteronism, which is
typically caused by an adrenal adenoma or bilateral adrenal hyperplasia. Primary hyperaldosteronism is
characterized by autonomous aldosterone secretion that leads to increased intravascular volume and
hypertension. Hypertension can cause headaches if severe and left ventricular hypertrophy (eg, S4 heart sound) if
prolonged.

The elevated serum aldosterone causes suppressed plasma renin activity (PRA) due to feedback inhibition of the
renin-angiotensin-aldosterone system. Electrolyte abnormalities such as hypokalemia and metabolic alkalosis are
often present due to aldosterone-mediated K+ and H+ secretion, respectively. The addition of a thiazide diuretic (eg,
chlorthalidone) for treatment of hypertension can substantially worsen these electrolyte abnormalities (as in this
patient).

Despite increased Na+ reabsorption, extravascular volume overload (eg, peripheral edema) is rarely seen due to
aldosterone escape.

(Choice A) Thiazides (eg, chlorthalidone) inhibit the Na+-Cl− cotransporter in the distal convoluted tubule, which
results in natriuresis and potassium wasting. Although thiazide use explains this patient's hypokalemia (which can
cause fatigue), by itself it would be expected to cause a rise in PRA due to reduced intravascular volume and
decreased Na+ delivery to the macula densa (which stimulates renin release).

(Choice B) ACE inhibitors (eg, lisinopril) decrease conversion of angiotensin I to angiotensin II, which results in a
compensatory rise in PRA in addition to reduced glomerular filtration rate due to efferent arteriole vasodilation. A
mild rise in serum creatinine and an increase in serum potassium are often seen.

(Choice C) Malignant hypertension is characterized by markedly elevated blood pressure with end-organ damage
to the eye (eg, retinal hemorrhages, papilledema), CNS (eg, hemorrhage, encephalopathy), and kidneys (eg,
nephrosclerosis). Kidney involvement results in creatinine elevation (normal in this patient) and dysregulated renin
release that leads to a paradoxically high PRA.

(Choice E) Renovascular hypertension is characterized by decreased renal perfusion (most often due to unilateral
renal artery stenosis) that results in an inappropriately high PRA that leads to hypertension. An abdominal bruit is
typically auscultated.

Educational objective:
Primary hyperaldosteronism results in hypertension and predisposes to electrolyte abnormalities (ie, hypokalemia,
metabolic alkalosis). Low plasma renin activity is expected.

Reference
• Primary aldosteronism.
Question #160

A 26-year-old woman calls the office with a question about management of hypothyroidism during pregnancy. The
patient is not pregnant but wishes to become pregnant soon. She has a history of hypothyroidism for several years
and has been on a stable dose of levothyroxine. TSH checked 3 months ago was 2.0 µU/mL. The patient currently
has mild fatigue but is otherwise asymptomatic. Family history is notable for primary hypothyroidism in her mother,
who is doing well on a regimen of liothyronine (T3) and levothyroxine (T4). Which of the following is the most
appropriate recommendation for this patient?

A) Continue the current levothyroxine dose through pregnancy

B) Decrease the levothyroxine dose when the patient becomes pregnant

C) Increase the levothyroxine dose prior to pregnancy

D) Increase the levothyroxine dose when the patient becomes pregnant

E) Switch to combination treatment with liothyronine and levothyroxine


Explanation
Correct Answer:

D) Increase the levothyroxine dose when the patient becomes pregnant

Maternal thyroid testing in pregnancy, first trimester

Hormone Change Mechanism

Total T4 Increased
• β-hCG stimulates thyroid hormone production in first trimester
• Estrogen stimulates TBG; thyroid increases hormone production to
Unchanged or mildly maintain steady free T4 levels
Free T4
increased

TSH Decreased • Increased β-hCG & thyroid hormone suppress TSH secretion

β-hCG = β-human chorionic gonadotropin; TBG = thyroxine-binding globulin.

During pregnancy, estrogen induces an increase in serum thyroxine-binding globulin levels, requiring an increase
in the amount of thyroid hormone needed to saturate the binding sites. Thyroxine production is also increased due
to the stimulatory effects of hCG on TSH receptors (as hCG and TSH have a similar alpha subunit). These changes
lead to an increase in total thyroid hormone levels, with a minimal increase in free hormone levels and a decrease
in TSH. However, patients with pre-existing hypothyroidism are unable to increase thyroxine production
appropriately and are at risk for a worsening hypothyroid state and adverse fetal and maternal effects (eg,
gestational hypertension, preeclampsia, premature delivery, postpartum hemorrhage).

Individuals such as this patient on a stable dose of thyroid replacement should have their dose increased by
approximately 30% at the time the pregnancy is detected (Choices A and B). The dose should be adjusted
subsequently (typically in 4-week increments) based on TSH using pregnancy-specific norms. Total T4 should
also be followed and maintained within pregnancy-specific norms.

(Choice C) This patient has been on a stable dose of levothyroxine and has a TSH in the low-normal range (<2.5
µU/mL). She does not require dosing adjustments at this time.

(Choice E) T3 is the active form of thyroid hormone and is produced predominantly by conversion from T4 in
peripheral tissues. However, T3 has a short half-life, and supplementation with liothyronine (oral T3) is associated
with wide fluctuations in blood levels. T3 supplementation is not recommended for most patients with primary
hypothyroidism, and those who are taking T3 should be changed to levothyroxine alone if considering pregnancy.

Educational objective:
Levothyroxine requirements increase during pregnancy. Patients with hypothyroidism should increase their
levothyroxine dose at the time pregnancy is detected, with subsequent dose adjustments based on TSH and total
T4 using pregnancy-specific norms.

Reference
• Management of thyroid dysfunction during pregnancy and postpartum: an Endocrine Society clinical practice
guideline.
Question #161

A 60-year-old man comes to the office due to edema of his face and ankles for the past 2 weeks. He has had no
chest pain or breathlessness. The patient has a 15-year history of type 2 diabetes mellitus, which is currently
managed with exercise, dietary modification, and oral medications. A month ago, his hemoglobin A1c was 6.9%.
Temperature is 37 C (98.6 F), blood pressure is 146/87 mm Hg, pulse is 75/min, and respirations are 15/min.
Physical examination shows periorbital edema and bilateral pitting edema around the ankles. The remainder of the
examination is normal. Laboratory results are as follows:

Serum chemistry
Sodium 140 mEq/L
Potassium 4.3 mEq/L
Bicarbonate 20 mEq/L
Blood glucose 120 mg/dL
Blood urea nitrogen 37 mg/dL
Creatinine 2.4 mg/dL
Total cholesterol 300 mg/dL

ECG is normal. Urine protein is 3,700 mg/24 hr. Estimated glomerular filtration rate is 28 mL/min/1.73 m2. Which
of the following measures would have the greatest impact in slowing the progression of this patient's kidney
disease?

A) Aggressive lipid management

B) Aspirin therapy

C) Intensive blood pressure control


D) Intensive glycemic control with insulin

E) Very low-protein diet


Explanation
Correct Answer:

C) Intensive blood pressure control

Diabetic kidney disease

• Persistent albuminuria (>3 months apart)


Clinical findings • Initial hyperfiltration followed by progressive decline in GFR
• Hypertension usually present

• Screen at the time of diagnosis in type 2 DM


• Screen 5 years after the diagnosis in type 1 DM
Evaluation • Serum creatinine
• Urine spot albumin to creatinine ratio (or 24-hour urine protein)
• Urinalysis/urine microscopy (to exclude other causes)

• Intensive glycemic control


◦ Target hemoglobin A1c ≤7% (for most patients)
◦ SGLT2 inhibitor preferred; GLP-1 agonist
• Blood pressure control
Management/prevention
◦ Target blood pressure <130/80 mm Hg
◦ ACE inhibitor preferred (or angiotensin II receptor blocker)
• General cardiovascular risk management
◦ Smoking cessation
◦ Lipid management

DM = diabetes mellitus; GFR = glomerular filtration rate; GLP-1 = glucagon-like peptide-1; SGLT2 = sodium-
glucose cotransporter-2.

This patient has long-standing type 2 diabetes mellitus complicated by significant diabetic kidney disease (DKD).
Initially, DKD is characterized by hyperfiltration (ie, increased glomerular filtration rate [GFR]); however, as the
disease progresses, GFR decreases, manifesting as a rising serum creatinine level. Concurrently, glomerular
protein losses lead to moderately increased albuminuria (urine protein excretion 30-300 mg/24 hr; previously
termed microalbuminuria) and eventually severely increased albuminuria (>300 mg/24 hr). As in this patient,
advanced DKD can present with frank proteinuria and nephrotic syndrome.

In patients with type 2 diabetes, intensive blood pressure control is associated with reduced progression of DKD;
a blood pressure (BP) target of <130/80 mm Hg is recommended for most patients. ACE inhibitors or angiotensin II
receptor blockers are the preferred antihypertensive drugs; these agents lower systemic and intraglomerular
pressures, which may be renoprotective.

(Choices A and B) Intensive lipid control with statins can reduce the risk for coronary heart disease; however, lipid-
lowering therapy has not clearly been shown to reduce the progression of nephropathy. Low-dose aspirin also may
reduce the incidence of cardiovascular events but would not slow the progression of nephropathy.

(Choice D) Intensive glycemic management to a target hemoglobin A1c <7% can slow the progression of DKD;
however, this patient has good overall glycemic control (A1c of 6.9%), and further lowering is associated with an
increased risk for hypoglycemia and may actually increase the risk for cardiac events. Although sodium-glucose
cotransporter 2 inhibitors also reduce the risk of DKD progression in patients with type 2 diabetes, these drugs are
more effective when used in patients with an estimated GFR of ≥30 mL/min/1.73 m2.

(Choice E) A moderate restriction in protein intake (eg, <1 g/kg/day) in patients with type 2 diabetes may slow the
progression of DKD, but the benefits are small. Very low protein intake is not advised because patients with
diabetes often have some degree of protein wasting at baseline and additional, complex dietary requirements.

Educational objective:
Strict blood pressure control, particularly using ACE inhibitors or angiotensin II receptor blockers, can reduce the
progression of diabetic kidney disease (DKD). Tight glycemic control with a target hemoglobin A1c <7% also can
prevent progression of DKD; further lowering is associated with an increased risk of hypoglycemia and possibly
cardiac events.

Reference
• Blood pressure, hypertension, RAAS blockade, and drug therapy in diabetic kidney disease.
Question #162

A 19-year-old woman comes to the office due to severe acne. She had expected the acne to improve after high
school, but instead, it is worsening. The patient has tried multiple remedies without significant improvement. She is
concerned that the acne is scarring her face. Since the patient started college last year, she has gained 13.6 kg (30
lb). She sings in a local church choir and has not noticed any voice changes. The patient has no chronic medical
conditions, has had no surgeries, and takes no medications. BMI is 29 kg/m2. Physical examination shows
nodulocystic acne on the face, arms, back, and forehead. The scalp is normal, with no alopecia. Tanner stage is
5. There is no skin discoloration or striae. No masses are palpable on abdominal examination. A urine pregnancy
test is negative. Which of the following additional information is required to establish the most likely diagnosis?

A) Family history of hypothyroidism

B) Food intake diary

C) History of an eating disorder

D) Intravenous drug use

E) Menstrual history

F) Number of lifetime sexual partners


Explanation
Correct Answer:

E) Menstrual history

Hyperandrogenism

• Hirsutism
• Nodulocystic acne
Clinical features
• Androgenic alopecia
• ↑ Serum testosterone

• Polycystic ovary syndrome


• Androgen-secreting tumor
Differential diagnosis
• Cushing syndrome
• Nonclassical CAH

CAH = congenital adrenal hyperplasia.

Severe nodulocystic acne occurring on the arms and upper back is a sign of hyperandrogenism. A common
cause of hyperandrogenism in young women, particularly in conjunction with obesity, is polycystic ovary
syndrome (PCOS). In addition to either clinical (eg, severe acne, hirsutism, androgenic alopecia) or laboratory (eg,
elevated serum testosterone) evidence of hyperandrogenism, the diagnosis of PCOS requires a history of irregular
menses indicating anovulation and/or polycystic ovaries on ultrasound. When 2 of these 3 criteria are met and
other causes of hyperandrogenism (eg, Cushing syndrome, nonclassical congenital adrenal hyperplasia) are
excluded, the diagnosis of PCOS is established.

(Choice A) Autoimmune hypothyroidism is linked to family history and may cause irregular menses and weight
gain. Typical skin findings are coarse hair, skin discoloration, and, occasionally, alopecia, not nodulocystic acne.

(Choices B and C) Patients with eating disorders do not typically have signs of hyperandrogenism. In addition,
there is no established causal relationship between certain foods and acne.

(Choice D) Intravenous drug use may cause localized skin infections (eg, folliculitis) but not diffuse nodulocystic
acne.

(Choice F) The number of lifetime sexual partners is used to assess the risk for cervical dysplasia and sexually
transmitted infections; it is not relevant in the differential diagnosis of acne.

Educational objective:
Severe nodulocystic acne may be the initial sign of hyperandrogenism in patients with polycystic ovary syndrome.
The diagnosis is based on the presence of 2 of 3 manifestations: clinical or laboratory evidence of
hyperandrogenism, menstrual irregularities, and polycystic ovaries on ultrasound.

Reference
• Genetic, hormonal and metabolic aspects of PCOS: an update.
Question #163

A 25-year-old woman comes to the office due to "chest pain" that began 2 days ago. The pain is described as a
constant soreness across the chest, and the patient is unable to sleep prone because of the pain. She is an avid
runner and kickboxer and has been unable to maintain her normal exercise routine due to discomfort wearing a
sports bra. The patient is sexually active with her husband and recently stopped taking oral contraceptives because
the couple are trying to have a child. Her last menstrual period was 3 weeks ago; menses are regular, occur
approximately every 28 days, and last for 6 days. The patient takes a daily folic acid supplement and no other
medications. Vitals signs are normal. Palpation demonstrates bilateral, nonfocal chest tenderness and diffusely
nodular, dense breasts. Which of the following is the most likely diagnosis in this patient?

A) Costochondritis

B) Cyclic mastalgia

C) Fat necrosis

D) Fibroadenoma

E) Fibrocystic changes

F) Phyllodes tumor
Explanation
Correct Answer:

E) Fibrocystic changes

Benign breast disease

Diagnosis Clinical features

• Solitary, well-circumscribed & mobile mass


Breast cyst
• ± Tenderness

• Multiple, diffuse nodulocystic masses


Fibrocystic changes
• Cyclic premenstrual tenderness

• Solitary, firm, well-circumscribed & mobile mass


Fibroadenoma
• Cyclic premenstrual tenderness

• After trauma or surgery


Fat necrosis • Firm, irregular mass
• ± Ecchymosis, skin/nipple retraction

In women, the breasts are a common source of bilateral, nonfocal chest pain because breast tissue is
heterogeneous (ie, composed of glands, stroma, and ducts), and pain can occur with normal tissue proliferation.
Breast tissue proliferation varies with hormonal fluctuations (eg, estrogen, progesterone) during the menstrual
cycle. In some women (typically premenopausal and of reproductive age), glandular tissue proliferation can lead
to diffusely nodular breasts and breast pain known as fibrocystic changes of the breast.

Patients with fibrocystic breast changes typically have cyclic, premenstrual breast tenderness (as in this patient)
and improvement in pain symptoms during or after menstruation. On examination, both breasts typically have
multiple nodulocystic masses. Because fibrocystic changes are benign, patients can be reassured and offered
symptomatic relief with nonsteroidal anti-inflammatory drugs and/or oral contraceptives.

(Choice A) Costochondritis is inflammation of the rib cartilage, which can cause parasternal chest pain that
worsens with chest movement (eg, coughing, deep inhalation). In contrast to this patient's bilateral, nonfocal pain,
costochondritis is characterized by pain that is typically unilateral, sharp or pressure-like, and associated with focal
tenderness to palpation of the chest wall. It is not associated with diffuse breast nodularity.

(Choice B) Cyclic mastalgia is a benign breast disorder diagnosed in women with breast pain related to the
menstrual cycle (ie, due to fluctuations in estrogen and progesterone). Although patients also have bilateral,
premenstrual breast tenderness, they have no associated breast nodularity or masses.

(Choice C) Fat necrosis of the breast is an uncommon complication of breast trauma (typically invasive breast
surgery) and presents as a unilateral, firm, irregular mass. In contrast, this patient has bilateral breast nodularity.

(Choice D) Although fibroadenomas are also estrogen sensitive and vary in size and tenderness depending on the
menstrual cycle, they typically present as a solitary, unilateral, well-circumscribed, and mobile breast mass.

(Choice F) A phyllodes tumor is a rare, rapidly growing breast tumor that presents as a unilateral, smooth, firm,
and painless breast mass of variable size.

Educational objective:
Fibrocystic breast changes are a common cause of cyclic, premenstrual breast tenderness in women of
reproductive age. Patients typically have diffusely nodular breasts and bilateral, nonfocal breast or chest pain.

Reference
• The role of E2/P ratio in the etiology of fibrocystic breast disease, mastalgia and mastodynia.
Question #164

A 58-year-old woman, gravida 2 para 2, comes to the office for evaluation of increased urinary frequency and
painful urination. The patient developed these symptoms 2 days ago, and they did not improve despite increasing
fluid intake and voiding regularly. She has had 2 other episodes of acute simple cystitis in the past 8 months, which
resolved with antibiotics. The patient underwent menopause at age 53 and has had no episodes of
postmenopausal bleeding. BMI is 28 kg/m2. Vital signs are normal. Physical examination shows mild suprapubic
tenderness and no costovertebral tenderness. Pelvic examination shows thin vulvar tissue but no excoriations or
lesions. Urinalysis is positive for leukocyte esterase and nitrites. The patient is prescribed a course of antibiotics
and is asymptomatic a week later. Which of the following is the best next step in management of this patient's
symptoms?

A) Cystoscopy

B) Daily probiotic consumption

C) Pelvic ultrasound

D) Topical corticosteroids

E) Urine cytology

F) Vaginal estrogen
Explanation
Correct Answer:

F) Vaginal estrogen

Recurrent urinary tract infection

• ≥2 infections in 6 months
Definition
• ≥3 infections in 1 year

• History of cystitis at age ≤15


• Spermicide use
Risk factors
• New sexual partner
• Postmenopausal status

• Urinalysis
Evaluation
• Urine culture

• Behavior modification
Prevention • Postcoital or daily antibiotic prophylaxis
• Topical vaginal estrogen for postmenopausal patients

This patient has recurrent urinary tract infections (UTI), defined as ≥3 episodes in a year (or ≥2 episodes in 6
months) of simple cystitis (eg, leukocyte esterase and nitrites on urinalysis with no costovertebral tenderness).
Women are at increased risk for cystitis due to the proximity of the urethra to the vagina and rectum. In
postmenopausal women, the risk increases further due to estrogen deficiency, which causes:

• vulvovaginal atrophy (eg, thin vulvar tissue).

• decreased bulk and elasticity of the bladder trigone and urethra, resulting in an increased risk for ascending
infection.

• decreased vaginal lactobacilli levels and an elevated vaginal pH, leading to an increased rate of vaginal
Escherichia coli colonization.

Therefore, in addition to behavioral changes (eg, increased fluid intake), treatment of postmenopausal women with
recurrent UTI is with topical vaginal estrogen. Topical estrogen can improve genitourinary atrophy and prevent
future UTI episodes. In addition to vaginal estrogen, antibiotic prophylaxis (daily or postcoital) can be considered.

(Choices A and E) Cystoscopy and urine cytology can evaluate for bladder cancer, which may present with
recurrent UTIs; however, bladder cancer is typically associated with gross hematuria or red blood cells on urinalysis,
which are not seen in this patient. In addition, she has no significant risk factors (eg, tobacco use, prior pelvic
radiation).

(Choice B) Daily probiotic consumption has not been shown to increase vaginal Lactobacillus colonization or
decrease the risk of recurrent cystitis.

(Choice C) Pelvic ultrasonography may be indicated in patients with abnormal uterine bleeding (eg,
postmenopausal bleeding) or a pelvic mass; it is not used to evaluate recurrent cystitis.

(Choice D) Topical corticosteroids can treat vulvar lichen planus and lichen sclerosus, which may present in
postmenopausal patients with dyspareunia and atrophic vulvar tissue. However, patients typically have visible
lesions (ie, plaques) or excoriations due to vulvar pruritus, which are not seen in this patient.

Educational objective:
Recurrent urinary tract infections (≥3 episodes in a year or ≥2 episodes in 6 months) commonly occur in
postmenopausal patients due to estrogen deficiency, which causes vulvovaginal atrophy, decreased bulk and
elasticity of the bladder trigone and urethra, and increased vaginal pH. Treatment is with vaginal estrogen.

Reference
• Genitourinary syndrome of menopause: an overview of clinical manifestations, pathophysiology, etiology,
evaluation, and management.
Question #165

A 65-year-old woman comes to the office to follow up for osteoporosis, which was diagnosed by a screening bone
mineral density scan. The patient was prescribed alendronate, but she stopped taking it due to intense stomach
pain from the medication. She saw a television advertisement about raloxifene and is interested in this treatment
option. The patient has a history of deep venous thrombosis in her left leg while on an oral contraceptive at age 38
that was treated with several months of heparin. She currently takes medications for hypertension and
hyperlipidemia diagnosed after a minor heart attack at age 63. Her mother had breast cancer at age 52 and died
from ovarian cancer at age 61. A maternal aunt died from endometrial cancer at age 72. Blood pressure is 125/80
mm Hg, and physical examination is normal. Which of the following is a contraindication to raloxifene in this
patient?

A) History of breast cancer in her mother

B) History of endometrial cancer in her maternal aunt

C) History of myocardial infarction

D) History of ovarian cancer in her mother

E) History of venous thrombosis


Explanation
Correct Answer:

E) History of venous thrombosis

Selective estrogen receptor modulators

• Tamoxifen
Drugs
• Raloxifene

• Competitive inhibitor of estrogen binding


Mechanism of action
• Mixed agonist/antagonist action

• Prevention of breast cancer in high-risk patients


Indications • Tamoxifen: adjuvant treatment of breast cancer
• Raloxifene: postmenopausal osteoporosis

• Hot flashes
• Venous thromboembolism
Adverse effects
• Endometrial hyperplasia & carcinoma (tamoxifen only)
• Uterine sarcoma (tamoxifen only)

Selective estrogen receptor modulators (SERMs) are a class of nonsteroidal compounds that exhibit estrogen
agonist and antagonist properties in a tissue-specific fashion. The 2 most frequently used SERMs are raloxifene
and tamoxifen. Raloxifene exhibits estrogen agonist activity on the bone and decreases post-menopausal
osteoporosis. Although less effective than alendronate, raloxifene is frequently used for osteoporosis
management in postmenopausal women who cannot tolerate bisphosphonates or are at high risk for invasive
breast cancer.

All medicines with estrogen agonist activity, including oral contraceptives, hormone replacement therapy, and all
SERMs, increase the risk for venous thromboembolism (VTE). Consequently, current or prior VTE disorders (eg,
pulmonary embolism, deep vein thrombosis, retinal vein thrombosis) are contraindications to both raloxifene and
tamoxifen use.

(Choices A and C) SERMs have estrogen antagonist activity in the breast and decrease the incidence of
malignancy. A family history of breast cancer is a potential indication, rather than a contraindication, for raloxifene
use. Raloxifene also decreases total and low-density lipoprotein cholesterol levels, although it neither increases nor
decreases the risk of coronary heart disease. In contrast, a history of coronary artery disease or estrogen-receptor-
positive breast cancer is a contraindication for treatment of menopausal symptoms with hormone replacement
therapy (due to unopposed estrogen activity).

(Choice B) Raloxifene has estrogen antagonist activity on the uterus and does not increase the risk for endometrial
hyperplasia or cancer. As a result, a family history of uterine cancer is not a contraindication to raloxifene use. In
contrast, tamoxifen is associated with an increased risk of uterine hyperplasia and cancer.

(Choice D) SERMs are not associated with an increased incidence of ovarian cancer. Family history of ovarian
cancer is not a contraindication to raloxifene or tamoxifen use. Women at high risk for ovarian cancer can be
offered oral contraceptives or, after childbearing is finished, a prophylactic oophorectomy.

Educational objective:
Raloxifene is a selective estrogen receptor modulator with estrogen antagonist activity in the breast and uterus and
agonist activity in the bone. It is used to treat osteoporosis in women at high risk for breast cancer.
Contraindications include a history of venous thromboembolism.

Reference
• Selective estrogen receptor modulators in clinical practice: a safety overview.
Question #166

A 70-year-old woman comes to the office for worsening constipation. The patient has chronic constipation for which
she takes over-the-counter fiber supplements and stool softeners; however, for the past 3 months, the patient has
had to push 2 fingers into her vagina to defecate. She has had increased bloating but no nausea, diarrhea, or
abdominal pain. The patient has 1 son. Medical history includes hypothyroidism and type 2 diabetes mellitus for
which she takes levothyroxine and metformin. A colonoscopy last year was normal. BMI is 32 kg/m2. Vital signs
are normal. The abdomen is soft and nontender with normoactive bowel sounds. On rectal examination, the
resting sphincter tone is normal and there are no palpable masses. With Valsalva maneuver, the posterior vaginal
wall extends outside the hymenal ring. There is no fecal incontinence with Valsalva maneuver. Anocutaneous
reflex is intact bilaterally. TSH is 3.9 mU/L and serum hemoglobin A1c is 7.5%. Fecal occult blood testing is
negative. Which of the following is the most likely underlying cause of this patient's presentation?

A) Autonomic dysfunction due to neuropathy

B) External anal sphincter dysfunction

C) Levator ani muscle complex injury

D) Overuse of fiber supplementation

E) Subtherapeutic levothyroxine dosing


Explanation
Correct Answer:

C) Levator ani muscle complex injury


This patient's chronic constipation is caused by pelvic organ prolapse, the herniation of pelvic organs (eg, bladder,
rectum) into the vaginal wall due to levator ani muscle complex damage. The levator ani muscle complex forms
most of the pelvic floor and functions to hold the pelvic organs in a stable position. When this complex is damaged,
such as with increased intraabdominal pressure (eg, pregnancy, obesity) or obstetric trauma (eg, forceps-
assisted vaginal delivery), there is increased pelvic floor laxity, resulting in decreased pelvic organ support.

The decreased pelvic floor support causes an anatomic change to normal pelvic organ positions; for the rectum, this
leads to a change in rectal angle during defecation, which can cause incomplete defecation and constipation.
With accumulation of fecal material, the rectum expands and herniates through the rectovaginal septum, causing
prolapse of the rectum into the posterior vaginal wall (ie, rectocele). The prolapse is exacerbated by Valsalva
maneuvers (eg, vaginal bulge extending past the hymen while straining), which cause further difficulty with
defecation. Therefore, some patients apply digital pressure against the prolapse (ie, splinting) to help improve
bowel movements. Treatment is with pelvic floor muscle exercises, vaginal pessary placement, or possible surgical
correction.

(Choice A) Diabetic autonomic neuropathy can occasionally cause constipation but is more commonly associated
with gastroparesis and chronic diarrhea (usually due to decreased rectal sphincter tone). This patient has a normal
rectal sphincter tone, making this diagnosis unlikely.

(Choice B) External anal sphincter dysfunction can occur with prior perineal laceration, a common complication of
vaginal delivery. In contrast to this patient, those with external anal sphincter dysfunction typically have decreased
sphincter tone and anal incontinence (ie, involuntary loss of flatus or fecal material).

(Choice D) Fiber supplementation is used to help treat constipation, and overuse can cause abdominal bloating
and flatulence but typically not worsening constipation.

(Choice E) Hypothyroidism can cause chronic constipation; however, this patient's TSH level is normal, making
subtherapeutic levothyroxine dosing an unlikely cause of this patient's symptoms.

Educational objective:
Pelvic organ prolapse, the herniation of pelvic organs (eg, rectum) into the vagina, occurs due to damage to the
levator ani muscle complex. Patients with prolapse of the posterior vaginal wall (ie, rectocele) classically have
chronic constipation and a vaginal bulge.

Reference
• Functional disorders: rectocele.
Question #167

A 31-year-old woman, gravida 3 aborta 3, comes to the office for an annual examination and discussion of
contraceptive options. Seven months ago, she experienced her third spontaneous miscarriage and underwent a
recurrent miscarriage workup. Results were consistent with antiphospholipid antibody syndrome, and the patient
was informed that she is at increased risk for another miscarriage. Due to these risks, the patient and her husband
have elected to adopt. She takes no medications and does not use tobacco, alcohol, or illicit drugs. Her blood
pressure is 115/60 mm Hg and pulse is 88/min. BMI is 22 kg/m2. Physical examination is normal. Which of the
following is the best contraceptive option for this patient?

A) Combined hormonal patch

B) Combined oral contraceptive pills

C) Condom with spermicide

D) Copper intrauterine device

E) Medroxyprogesterone injection
Explanation
Correct Answer:

D) Copper intrauterine device

Absolute contraindications to
combined hormonal contraceptive pills

• Active breast cancer


• Migraines with aura
• Uncontrolled hypertension
• Active hepatitis, severe cirrhosis, liver
cancer
• Age ≥35 & ≥15 cigarettes/day
• Ischemic heart disease, stroke
• <3 weeks postpartum
• Prolonged immobilization
• Thrombophilia (eg, factor V Leiden,
antiphospholipid antibody syndrome)
• Venous thromboembolism

Thromboembolic events occur due to a combination of hypercoagulability, endothelial injury, and venous stasis (eg,
Virchow triad) with patients typically having more than one risk factor for their development. Women with a pre-
existing risk factor for thromboembolism are at increased risk for clot development during pregnancy and with
exogenous estrogen exposure due to its hypercoagulable properties.

Antiphospholipid antibody syndrome (APS) is an autoimmune disorder characterized by vascular thrombosis


and pregnancy complications (eg, recurrent miscarriage, preeclampsia, fetal growth restriction). APS is an absolute
contraindication to combined (estrogen/progestin) hormonal contraceptives due to the increased risk of arterial and
venous thrombosis (Choices A and B). Patients who desire pregnancy prevention should be offered hormone-
free options: the copper intrauterine device or permanent sterilization (eg, tubal ligation).

(Choice C) Condoms with spermicide are a safe, hormone-free option but have the lowest efficacy rate of all
contraceptive options. Patients should be placed on the most efficacious contraceptive possible.

(Choice E) Depo medroxyprogesterone acetate is a systemic progestin-only contraceptive. Progestin-only


contraceptives have less risk for thromboembolism compared to combined hormonal options, but systemic forms
are still not recommended in high-risk patients.

Educational objective:
Absolute contraindications to combination hormonal contraceptives include a history of antiphospholipid antibody
syndrome or thromboembolism. Patients with these conditions should be placed on hormone-free contraceptives
(eg, copper intrauterine device).

Reference
• Association of venous thromboembolism with hormonal contraception and thrombophilic genotypes.

• Antiphospholipid antibodies and the placenta: a systematic review of their in vitro effects and modulation by
treatment.
Question #168

A 23-year-old woman comes to the office for "trouble" with her bowel movements. Despite experiencing urgency to
use the bathroom, the patient frequently strains on the toilet with minimal production of stool. She describes a
whitish-appearing anal discharge and itchiness but has no pain, fever, or unexpected weight loss. The patient
reports several new male sexual partners in the past year. Aside from oral contraceptives, she does not take any
medications. Temperature is 37.7 C (99.9 F), blood pressure is 132/65 mm Hg, pulse is 70/min, and respirations
are 14/min. The abdomen is nontender and nondistended without hepatosplenomegaly. There is no palpable
lymphadenopathy. External appearance of the genitalia is unremarkable. Rectal examination reveals mucopurulent
anal discharge without perianal abnormalities. Which of the following is the most likely diagnosis?

A) Crohn disease

B) Cryptosporidium parvum infection

C) Escherichia coli O157:H7 infection

D) Giardiasis

E) Neisseria gonorrhoeae infection


Explanation
Correct Answer:

E) Neisseria gonorrhoeae infection

Gonococcal proctitis

• Receptive anal intercourse


Transmission
• Direct spread from the vagina

• Mucopurulent anal discharge, occasional rectal bleeding


Manifestations • Tenesmus, constipation
• Pruritus, rectal pain, rectal fullness

Diagnosis • Nucleic acid amplification test of rectal swab

Treatment • Ceftriaxone + doxycycline (to cover Chlamydia)

This patient with new sexual partners has developed tenesmus (eg, straining with minimal stool production) and
mucopurulent rectal discharge, a presentation concerning for gonococcal proctitis. Gonorrhea is caused by the
gram-negative diplococci bacterium Neisseria gonorrhoeae and occurs most commonly in adolescents and young
adults who have unprotected sexual intercourse. Rectal infection can occur through receptive anal intercourse
and via proximal spread from the vagina.
Manifestations of gonococcal proctitis typically include tenesmus, mucopurulent anal discharge, and pruritus;
rectal fullness, bleeding, anorectal pain, and constipation can also occur. The diagnosis is confirmed with nucleic
acid amplification testing of a rectal swab. Because patients are often coinfected with Chlamydia trachomatis, they
should be treated with dual antibiotic therapy (ie, ceftriaxone + doxycycline) to cover both pathogens
(azithromycin has not been studied for treatment of proctitis).

(Choice A) Crohn disease (CD) causes anal fistula and perirectal abscess, which can cause mucopurulent anal
discharge; however, both conditions are typically visible on examination. In addition, because the rectum is typically
spared, CD usually does not cause tenesmus. Typical manifestations of CD include abdominal pain, weight loss,
and diarrhea, possibly with hematochezia.

(Choices B and D) Cryptosporidium parvum and giardiasis both cause profuse watery diarrhea, although
steatorrhea is sometimes seen in giardiasis. Tenesmus and mucopurulent anal discharge are not consistent with
either infection.

(Choice C) Escherichia coli O157:H7 produces Shiga toxin, which leads to hematochezia and sometimes
hemolytic uremic syndrome (eg, renal failure, hemolytic anemia, thrombocytopenia). Abdominal pain is typical, and
tenesmus and mucopurulent anal discharge are unexpected.

Educational objective:
Gonococcal proctitis typically occurs in adolescents or young adults practicing unprotected sexual intercourse.
Rectal infection can occur via receptive anal intercourse or proximal spread from the vagina. Manifestations include
tenesmus, mucopurulent anal discharge, and pruritus; rectal fullness, bleeding, anorectal pain, and constipation
may also occur.

Reference
• Sexually transmitted proctitis.
Question #169

A 38-year-old woman comes to the office due to jaundice. Her eyes have been yellow for the past 8 days, and she
has also had pruritus, fever, nausea, and vomiting. The patient has had no unexpected weight loss or bloody
stools. Medical history is significant for hypertension; current medications include amlodipine and an oral
contraceptive. The patient works in a day care center, where some of the children were recently ill. She drinks
alcohol but does not use illicit drugs and is in a monogamous relationship with her husband. Family history is
unremarkable. Temperature is 38.7 C (101.7 F), blood pressure is 125/80 mm Hg, and pulse is 80/min. Scleral
icterus is present. Oropharyngeal and cardiopulmonary examinations are unremarkable. The neck is supple
without lymphadenopathy. The abdomen is nontender and nondistended. The liver edge is smooth and palpable
below the right costal margin. Laboratory evaluation reveals the following:

Complete blood count


Hemoglobin 13.7 g/dL
Platelets 290,000/mm3
Leukocytes 12,000/mm3

Liver function studies


Albumin 3.9 g/dL
Total bilirubin 4.2 mg/dL
Total protein 6.8 g/dL
Alkaline phosphatase 130 U/L
Aspartate aminotransferase (SGOT) 1534 U/L
Alanine aminotransferase (SGPT) 1722 U/L

Which of the following is the most likely diagnosis?


A) Alcoholic hepatitis

B) Bacterial liver abscess

C) Budd-Chiari syndrome

D) Hepatitis A infection

E) Infectious mononucleosis

F) Primary biliary cholangitis

G) Ruptured hepatic adenoma


Explanation
Correct Answer:

D) Hepatitis A infection

Hepatitis A

• Fecal-oral transmission
Transmission
• Poor sanitation/hygiene, travel to endemic areas
& risk factors
• Contaminated food & water

Clinical • Fever, nausea, right upper quadrant pain


presentation • Jaundice, hepatomegaly

Diagnostic • Elevated liver transaminases (>1,000 U/L)


testing • Anti–hepatitis A serology

• Supportive/expectant
Management
• Spontaneous recovery in most patients

This patient most likely has hepatitis A, which typically begins with fever, vomiting, and mild right upper quadrant
(RUQ) pain. Jaundice develops a few days later, often with pruritus, dark urine, and acholic stools. Hepatitis A
virus (HAV) undergoes fecal-oral transmission and is commonly acquired from unsanitary food (eg, shellfish) and
water supplies; outbreaks may occur in facilities with poor sanitation, such as homeless shelters and day care
centers (children often do not have jaundice and may go undiagnosed). Unvaccinated persons are at increased
risk, as are international travelers and men who have sex with men.

Hepatomegaly (eg, liver edge palpable below right costal margin) is common in hepatitis A. Liver transaminases
(eg, aspartate aminotransferase [AST], alanine aminotransferase [ALT]) rise early, often to >1,000 U/L. Direct
hyperbilirubinemia and mildly elevated alkaline phosphatase are seen soon thereafter and correlate with jaundice.
Anti-HAV IgM serology confirms the diagnosis. Treatment is supportive; complete recovery typically occurs within
2-3 months. Fulminant liver failure necessitating liver transplant is rare.

(Choice A) Alcoholic hepatitis can cause moderate elevations in liver transaminases, but AST and ALT are rarely
>500 U/L; the AST/ALT ratio is usually >2:1. Although this patient uses alcohol, the profound elevations in
transaminases are atypical for alcoholic hepatitis.

(Choice B) Pyogenic liver abscess can cause fever, hepatomegaly, and elevated liver transaminases, but patients
are acutely ill (eg, shaking chills) and have significant leukocytosis. Examination shows marked hepatic
tenderness, often with guarding and rocking tenderness (pain caused by gently rocking the abdomen); rebound
tenderness may be present.

(Choice C) Acute hepatic vein thrombosis (eg, Budd-Chiari syndrome) causes severe RUQ pain, ascites, and
hepatosplenomegaly. Absence of ascites and significant RUQ tenderness makes this less likely in this patient.

(Choice E) Infectious mononucleosis can cause fever and sometimes jaundice, but splenomegaly is more common
than hepatomegaly; patients typically have pharyngitis and cervical lymphadenopathy.

(Choice F) Primary biliary cholangitis is characterized by autoimmune destruction of intralobular bile ducts. Typical
findings include profound elevations of alkaline phosphatase and bilirubin but near-normal aminotransferase levels.

(Choice G) A ruptured hepatic adenoma, which is associated with oral contraceptive use, causes severe RUQ pain
and hemorrhagic shock. Although fever, and elevated liver enzymes are common, hypotension, tachycardia, and
anemia would also be present.

Educational objective:
Hepatitis A causes fever, vomiting, jaundice, hepatomegaly, and severe elevations in hepatic transaminases (eg,
aminotransferase levels >1,000 U/L). Unvaccinated individuals are at increased risk, as are day care workers,
homeless shelter residents, international travelers, and men who have sex with men.
Question #170

A 58-year-old man comes to the office due to yellowish discoloration of the skin, anorexia, and unintentional weight
loss of 6 kg (13.2 lb) over the past 3 months. He also reports dark urine and pale stools. The patient has had no
fever, abdominal pain, constipation, or diarrhea. He has a history of hypertension and hyperlipidemia. The patient
does not use tobacco, alcohol, or illicit drugs. Vital signs are within normal limits. BMI is 32 kg/m2. Scleral icterus
is present. An enlarged, nontender gallbladder is palpated below the right costal margin. There is no ascites.
Abdominal imaging in this patient is most likely to reveal which of the following findings?

A) Cystic lesion at the tail of the pancreas

B) Gallstone obstructing the cystic duct

C) Intra- and extra-hepatic biliary duct dilation

D) Multiloculated hypoechoic hepatic lesions

E) Peripancreatic fluid collection

F) Thrombus within the portal vein


Explanation
Correct Answer:

C) Intra- and extra-hepatic biliary duct dilation


This patient with painless jaundice, weight loss, anorexia, and a palpable gallbladder most likely has pancreatic
cancer. Most (60%-70%) pancreatic tumors occur in the head of the pancreas, which is in close proximity to the
biliary tree. As these tumors expand, they can compress the common bile duct (CBD), resulting in backup of bile
with subsequent intra- and extrahepatic biliary duct dilation. When the tumor also compresses the pancreatic
duct, a double duct sign (ie, dilation of both the common bile duct and the pancreatic duct) can be seen
radiographically, which is further suggestive of advanced neoplasia.

Symptoms of pancreatic cancer occurring at the head of the pancreas include painless jaundice (eg, yellowish
skin, pruritus, pale stools, dark urine) and a nontender, distended gallbladder at the right costal margin
(Courvoisier sign). In contrast, cancers in the body or tail of the pancreas usually present with abdominal pain but
without jaundice since they do not typically obstruct the biliary system.

(Choice A) Pancreatic cystic lesions are usually asymptomatic. Jaundice is unexpected in a pancreatic tail lesion
due to the distance from the biliary tree.

(Choice B) Gallstone obstruction of the cystic duct can cause biliary colic (ie, intermittent right upper quadrant
[RUQ] pain) or cholecystitis (ie, RUQ pain, leukocytosis, fever). However, because the CBD is not obstructed,
jaundice does not occur. In addition, gallbladder tenderness is expected, and weight loss would be atypical.

(Choice D) Pyogenic liver abscesses are identified radiographically as multiloculated, hypoechoic hepatic lesions
that present with jaundice and weight loss. However, fevers and leukocytosis are expected, and an enlarged
gallbladder would be atypical.

(Choice E) Encapsulated peripancreatic fluid collections (eg, pseudocysts) occur in patients with a history of
pancreatitis and are usually asymptomatic but can cause epigastric pain and early satiety. Pseudocysts are almost
never located close enough to the biliary tree to cause obstruction; therefore, jaundice and gallbladder enlargement
are unexpected.

(Choice F) Portal vein thrombosis (PVT) is most often associated with cirrhosis. Acute PVT usually presents with
abdominal pain, whereas chronic PVT most often presents with variceal bleeding. This patient's weight loss,
jaundice, and palpable gallbladder is more consistent with pancreatic cancer.

Educational objective:
Tumors in the head of the pancreas can present with weight loss, jaundice, and a nontender, distended gallbladder
(eg, Courvoisier sign) on examination. Imaging can demonstrate dilation of both the intra- and extrahepatic bile
ducts as well as the pancreatic duct (ie, double duct sign).

Reference
• Imaging diagnosis of pancreatic cancer: a state-of-the-art review.
Question #171

A 25-year-old woman comes to the office due to 6 weeks of postprandial abdominal discomfort, flatulence, and
nausea. The patient has tried both omeprazole and fiber supplementation without improvement of her symptoms.
Although her bowel movements are currently regular, she had an acute diarrheal illness 2 months ago that resolved
spontaneously. The patient has no vomiting, fever, bloody stools, early satiety, trouble swallowing, or unexpected
weight loss. She has had no sick contacts, and her family history is insignificant. The patient does not use tobacco,
alcohol, or illicit drugs. Temperature is 37.4 C (99.3 F), blood pressure is 120/80 mm Hg, and pulse is 80/min. She
is well-appearing. Mucous membranes are moist, and cardiopulmonary examination is normal. The abdomen is
moderately distended but nontender with no rebound or guarding. Complete blood count and serum electrolytes
are within normal limits. Which of the following is the best next step in management of this patient?

A) Bacterial stool cultures

B) Colonoscopy

C) Esophagogastroduodenoscopy

D) Lactose hydrogen breath test

E) Short course of rifaximin

F) Trial of amitriptyline
Explanation
Correct Answer:

D) Lactose hydrogen breath test

Lactose intolerance

• Congenital or developmental lactase deficiency


Etiology/risk factors
• Small intestinal infection or inflammation

• Gastrointestinal distress after dairy intake


◦ Abdominal pain
Clinical features
◦ Flatulence/bloating
◦ ± Watery diarrhea

• Resolution of symptoms on dairy-restricted diet


Diagnosis
• Lactose breath hydrogen test

• Dietary restriction of lactose


Management
• Lactase replacement if dairy ingested

This patient likely has lactose intolerance (LI), presenting with postprandial gastrointestinal distress (eg,
abdominal pain, bloating), benign abdominal examination, and normal laboratory studies. LI is caused by deficiency
of intestinal lactase, an enzyme in the brush border that metabolizes dietary lactose. Undigested lactose is
metabolized by colonic bacteria, releasing hydrogen gas and other byproducts.

LI has been described in many regions in the world, with some variation across groups. It may be precipitated by
inflammatory disorders affecting the brush border, such as infectious gastroenteritis (as is likely in this patient
who had a recent intestinal illness), celiac disease, and Crohn disease.

The symptoms of LI in adults are often nonspecific and may vary with underlying visceral hypersensitivity. Diarrhea
is typically not prominent. The relation to lactose intake may not be readily apparent because a broad variety of
foods contain lactose; clarification may require a dedicated trial of dietary lactose exclusion. Lactose breath
testing, which detects hydrogen released from a standardized dose of oral lactose, can be diagnostic.

(Choice A) Stool cultures can diagnose bacterial gastroenteritis and are indicated in severe acute diarrhea (eg,
hypovolemia, hematochezia, fever). They are not required in this individual since her diarrhea has resolved.

(Choices B and C) Esophagogastroduodenoscopy is indicated to evaluate upper gastrointestinal symptoms


associated with features suggesting bleeding (eg, iron deficiency anemia) or malignancy (eg, progressive
dysphagia, odynophagia). Likewise, colonoscopy is indicated for lower gastrointestinal symptoms associated with
bleeding (eg, hematochezia) or risk factors for malignancy (eg, age >50, unexpected weight loss). This young
patient has no alarm features, and invasive studies should be deferred.

(Choice E) Rifaximin is indicated for small intestinal bacterial overgrowth (SIBO), which causes bloating and
diarrhea in susceptible patients (eg, intestinal surgery, scleroderma). SIBO is unlikely in a patient with regular bowel
movements and no relevant risk factors.

(Choice F) Tricyclic antidepressants (TCAs) (eg, amitriptyline) are sometimes used to treat functional
gastrointestinal disorders (eg, irritable bowel syndrome). Although this patient's symptoms resemble a functional
bowel disorder, TCAs have significant potential adverse effects, and LI should be ruled out first.

Educational objective:
Lactose intolerance is caused by deficiency of intestinal lactase and presents with postprandial abdominal pain,
bloating, and nausea. It may be precipitated by inflammatory disorders that affect the intestinal brush border (eg,
infectious gastroenteritis, celiac disease, Crohn disease). Lactose breath testing, which detects hydrogen released
from a standardized dose of oral lactose, can be diagnostic.
Question #172

A 62-year-old man comes to the office due to anorexia, fatigue, and a 9 kg (20 lb) weight loss over the past 6
months. He rarely drinks alcohol and does not use illicit drugs, although he has smoked one pack of cigarettes
daily for the past 40 years. The patient has previously been diagnosed with hypertension, diverticulosis, external
hemorrhoids, and osteoarthritis. Current medications are atenolol and aspirin, and he has no known drug allergies.
Temperature is 37 C (98.6 F), blood pressure is 132/88 mm Hg, pulse is 70/min, and respirations are 12/min.
Physical examination reveals a soft abdomen that is mildly tender to deep palpation in the midepigastric region.
The sclerae are icteric, and the skin is jaundiced. Laboratory results show:

Complete blood count


Hemoglobin 13.5 g/L
Platelets 220,000/mm3
Leukocytes 5,100/mm3

Liver function studies


Albumin 3.9 g/dL
Total bilirubin 6.7 mg/dL
Alkaline phosphatase 647 U/L
Aspartate aminotransferase (SGOT) 110 U/L
Alanine aminotransferase (SGPT) 102 U/L

Antimitochondrial antibodies are negative. Abdominal ultrasound demonstrates mild dilation of the common bile
duct; there are no stones present in the gallbladder and no evidence of gallbladder wall thickening. Which of the
following is the best next step in the management of this patient's condition?
A) Abdominal CT scan

B) Endoscopic retrograde pancreatography (ERCP)

C) Percutaneous transhepatic cholangiogram (PTC)

D) Serum amylase and lipase

E) Serum CA 19-9
Explanation
Correct Answer:

A) Abdominal CT scan

Pancreatic adenocarcinoma

• Smoking
• Hereditary pancreatitis
Risk factors
• Nonhereditary chronic pancreatitis
• Obesity & lack of physical activity

• Systemic symptoms (eg, weight loss, anorexia) (>85%)


• Abdominal pain/back pain (80%)
Clinical • Jaundice (56%)
presentation • Recent-onset atypical diabetes mellitus
• Unexplained migratory superficial thrombophlebitis
• Hepatomegaly & ascites with metastasis

• Cholestasis (↑ alkaline phosphatase & direct bilirubin)


Laboratory • ↑ CA-19-9 (not as a screening test)
studies
• Abdominal ultrasonography (if jaundiced) or CT scan (if no jaundice)

This patient with a history of tobacco use now has weight loss, fatigue, anorexia, and painless jaundice with
dilation of the common bile duct on ultrasound. This presentation is concerning for pancreatic cancer, the 4th
leading cause of cancer death in the United States.

The presentation of pancreatic cancer varies with tumor location. Tumors in the pancreatic head often cause
compression of the pancreatic duct and common bile duct, resulting in painless obstructive jaundice with elevations
in serum bilirubin and alkaline phosphatase. Tumors in the pancreatic body or tail often result in progressive
abdominal pain without jaundice.

Abdominal ultrasound is usually the initial imaging performed on patients with jaundice. When the ultrasound is
nondiagnostic (as in this case) or when there is clinical suspicion for pancreatic cancer in a nonjaundiced patient, an
abdominal CT scan should be obtained. This imaging modality has a sensitivity of up to 97% in the detection of
pancreatic cancer and can be used for staging purposes and preoperative planning.

(Choice B) Endoscopic retrograde pancreatography is used to visualize the biliary tree and pancreatic ducts and
allows for tissue sampling. However, it is invasive and has a lower sensitivity than CT scan for tumors located in the
uncinate process or tail of the pancreas. It is typically recommended for patients in which other modalities were
inconclusive and can sometimes be used to relieve obstruction (eg, via biliary stent placement).

(Choice C) Percutaneous transhepatic cholangiogram is an invasive procedure used to evaluate the biliary tree of
patients who have had a less invasive imaging fail. It can be used for stent placement across malignant
obstructions but is not a first-line diagnostic technique.

(Choice D) Serum amylase and lipase are elevated in pancreatitis, which typically presents with severe abdominal
pain radiating into the back with nausea and vomiting, not painless jaundice. They are not helpful in diagnosing
pancreatic malignancy.

(Choice E) Serum CA 19-9 is a tumor marker that is often elevated in pancreatic cancer, but sensitivity is related to
tumor size and is limited in small cancers. In addition, it has low specificity and may be elevated in multiple
conditions (eg, hepatocellular cancer, cholangitis). It can be used to monitor disease progression but is less useful
for diagnosis than CT scan.

Educational objective:
The presentation of pancreatic cancer is dependent on tumor location; tumors in the pancreatic head often present
with painless obstructive jaundice, whereas those in the body or tail often cause abdominal pain without jaundice.
Although ultrasound is the imaging test of choice to evaluate a jaundiced patient, a CT scan of the abdomen is
indicated for those with a nondiagnostic ultrasound or to evaluate for suspected malignancy in a nonjaundiced
patient.

Reference
• Diagnosis and detection of pancreatic cancer.
Question #173

A 34-year-old woman who recently immigrated to the United States from China has laboratory tests performed prior
to entering nursing school. The patient has no known past medical history and takes no medications. She is
currently in a monogamous relationship and does not use illicit drugs. The patient believes she is up-to-date with all
of her immunizations. She brings the following report with her to the clinic.

HBsAg Negative
Anti-HBs Positive
Anti-HBc Positive
HBeAg Negative
Anti-HBe Negative

Which of the following is the appropriate interpretation of these serological test results?

A) Acute hepatitis B infection

B) Chronic hepatitis B infection

C) Recovery phase of hepatitis B infection

D) Resolved hepatitis B infection

E) Vaccination against hepatitis B infection


Explanation
Correct Answer:

D) Resolved hepatitis B infection

This patient's serologic test results are consistent with resolved hepatitis B virus (HBV) infection. Individuals
who are immune to HBV due to natural infection are positive for anti-HBs and negative for HBsAg. They are also
positive for IgG anti-HBc because they form antibodies directed against the HBV core antigen. The HBV vaccine
contains HBsAg, which stimulates production of anti-HBs and confers immunity in the host. However, the vaccine
does not contain the core antigen so antibodies are not made against it and patients are consequently anti-HBc
negative (Choice E).

Serological markers for the hepatitis B virus include the following:

• HBsAg: The first serologic marker detected in the serum after inoculation. It precedes the onset of clinical
symptoms and elevation of serum aminotransferases. It also remains detectable during the entire
symptomatic phase of acute hepatitis B and suggests infectivity.

• Anti-HBs: Appears in the serum after either successful HBV vaccination or clearance of HBsAg, and
remains detectable for life in most patients. It indicates non-infectivity and immunity. However, there is a
time lag between the disappearance of HBsAg and the appearance of anti-HBs; this is termed the "window
period."

• HBcAg: This marker is not detectable in serum as it is normally sequestered within the HBsAg coat.

• Anti-HBc: IgM appears shortly after the emergence of HBsAg and may be the only diagnostic marker for
acute HBV infection during the "window period." IgG remains detectable during recovery from acute HBV
infection or progression to chronic infection.

• HBeAg: This antigen is detectable shortly after the appearance of HBsAg and indicates active viral
replication/infectivity. It is associated with the presence of HBV DNA. HBeAg tends to disappear shortly
after aminotransferase levels peak and before HBsAg is eliminated; it is followed by the appearance of anti-
HBe.

• Anti-HBe: This marker suggests cessation of active viral replication and low infectivity.

(Choices A and B) Acute hepatitis B infection is characterized by the presence of HBsAg and IgM anti-HBc. In
contrast, chronic hepatitis B infection is defined by the presence of HBsAg in the serum for >6 months.

(Choice C) The recovery phase of hepatitis B infection is characterized by the presence of anti-HBs, anti-HBc
(predominantly IgG), and anti-HBe.

Educational objective:
Individuals with resolved hepatitis B infection are positive for anti-HBs and anti-HBc but negative for HBsAg.
Individuals who are immunized with the hepatitis B vaccine are positive for anti-HBs but negative for HBsAg and
anti-HBc.

Reference
• Hepatitis B: diagnosis and treatment.

• Hepatitis B virus infection.


Question #174

A 59-year-old woman comes to the office for a routine visit. She reports no symptoms. She has no medical
problems and takes no medications. She grew up in the United States and has not traveled outside the country.
The patient does not use tobacco, alcohol, or illicit drugs. Her blood pressure is 120/70 mm Hg, pulse is 88/min,
and respirations are 14/min. Physical examination shows a firm, nontender mass in the right upper quadrant. The
remainder of the examination is within normal limits. Laboratory testing is unremarkable. Abdominal CT scan is
shown in the image below.
This patient is at greatest risk for which of the following conditions?

A) Anaphylactic shock

B) Gallbladder adenocarcinoma
C) Hepatic abscess

D) Intestinal perforation

E) Liver cirrhosis

F) Renal cell carcinoma


Explanation
Correct Answer:

B) Gallbladder adenocarcinoma
This patient's abdominal CT scan shows a porcelain gallbladder, a term used to describe the calcium-laden
gallbladder wall with bluish color and brittle consistency often associated with chronic cholecystitis. The
pathogenesis of the condition remains unclear, but it is thought that calcium salts are deposited intramurally due to
the natural progression of chronic inflammation or chronic irritation from gallstones. Patients can be asymptomatic,
have right upper quadrant pain, or have a firm and nontender right upper quadrant mass on examination.

Plain x-rays can show a rimlike calcification in the area of the gallbladder, and CT scan typically reveals a calcified
rim in the gallbladder wall with a central bile-filled dark area. Porcelain gallbladder has been associated with
increased risk for gallbladder adenocarcinoma (2%-5% in some studies). Cholecystectomy is typically
considered for patients with porcelain gallbladder, particularly if they are symptomatic or have incomplete mural
calcification.

(Choice A) Ruptured echinococcal cyst can cause anaphylaxis with eventual shock. However, an echinococcal
cyst usually appears on abdominal CT as a cystic liver lesion (sometimes with calcifications) without gallbladder
involvement. It would be unlikely in a patient who has never traveled outside the United States.

(Choice C) Liver abscess is typically amebic or bacterial in origin (not a complication of porcelain gallbladder) and
is caused by portal vein bacteremia, systemic bacteremia, ascending cholangitis, or trauma. Ultrasound or CT scan
is usually diagnostic.

(Choices D and E) Porcelain gallbladder is typically not directly associated with intestinal perforation or cirrhosis of
the liver. Gallstone ileus can be caused by a fistula between the gallbladder and intestine, which allows gallstones
to enter and obstruct the intestine.

(Choice F) Risk factors associated with renal cell carcinoma (RCC) include smoking, hypertension, and obesity.
However, porcelain gallbladder is not usually associated with increased risk for RCC.

Educational objective:
Porcelain gallbladder is usually diagnosed on abdominal imaging showing a calcified rim in the gallbladder wall with
a central bile-filled dark area. It is associated with an increased risk for gallbladder adenocarcinoma and usually
requires cholecystectomy.

Reference
• Porcelain gallbladder: often an overlooked entity.

• Management of porcelain gallbladder, its risk factors, and complications: a review.


Question #175

A 40-year-old man comes to the office due to anal discharge and pruritus. The patient had an episode of severe
anal pain several months ago, which resolved spontaneously after 2 days. Since then, he has had an intermittent,
foul-smelling discharge on his undergarments and occasional pain during defecation. The patient has had no
abdominal pain, diarrhea, or blood in the stool. He has no prior medical conditions and takes no medications. Vital
signs are within normal limits. Physical examination shows a soft and nontender abdomen. The perianal skin
appears inflamed. There is an indurated pustule-like lesion close to the anal verge. Digital rectal examination
reveals mild tenderness but no mass. Which of the following is the most appropriate next step in management of
this patient?

A) Anorectal manometry

B) Colonoscopy

C) Empiric antibiotics

D) Stool test for ova and parasites

E) Surgical evaluation
Explanation
Correct Answer:

E) Surgical evaluation

Anorectal fistula (fistula in ano)

• Perianal abscess
• Crohn disease
Causes
• Malignancy, radiation proctitis
• Infection (eg, lymphogranuloma venereum)

• Perirectal pain, discharge


Clinical manifestations • Inflammatory papule/pustule
• Palpable fistula tract

• Assess extent of fistula


◦ Gentle probe
Management
◦ Imaging (endosonography, fistulogram, MRI)
• Surgery (eg, fistulotomy)

This patient with an inflamed perianal lesion associated with pain on defecation and chronic discharge has an
anorectal fistula (fistula in ano). Anorectal fistulas are most often due to rupture of a perianal abscess with
formation of a residual sinus tract. They may also occur as a complication of Crohn disease, radiation proctitis,
atypical infections (eg, lymphogranuloma venereum), or trauma.
Examination of an anorectal fistula often reveals an external terminus and an indurated tract leading to the rectum.
An internal terminus can sometimes be identified on anoscopy or by cautiously passing a probe through the fistula
from the external opening. The diagnosis is often apparent on clinical grounds, but endoscopic sonography, MRI, or
fistulogram can be used in complex cases to assess the extent of fistula formation.

Management of an anorectal fistula requires surgical intervention (eg, fistulotomy). Fistulas can often be
assessed more fully (eg, exploration with a soft probe) while patients are under anesthesia at the time of surgery to
delineate the extent of the fistula. The entirety of the fistula must be addressed because residual fistula tracts can
lead to persistent symptoms and fecal incontinence.

(Choice A) Anorectal manometry can provide information regarding anorectal neurologic dysfunction (eg,
decreased relaxation of puborectalis and anal sphincter) and is indicated for evaluation of chronic constipation and
fecal incontinence. Patients with anorectal fistula can have leakage of stool, but neurologic function is typically
normal.

(Choice B) Flexible sigmoidoscopy is occasionally needed to identify the internal fistula opening if it is not apparent
on anoscopy. However, colonoscopy is typically only necessary for complex, recurrent, or atypical fistulas or fistulas
associated with inflammatory bowel disease.

(Choice C) Some perianal fistulas associated with Crohn disease can be managed with antibiotics and medical
treatment of the underlying inflammatory bowel disease. For fistulas arising from a perianal abscess, nonsurgical
treatment has a high failure rate.

(Choice D) Stool test for ova and parasites is typically performed in the evaluation of diarrhea. The most common
organisms detected include Giardia intestinalis and Cryptosporidium species. This patient has no diarrhea (or
abdominal pain or blood in the stool) and is unlikely to have an intestinal infection.

Educational objective:
Anorectal fistulas are most often due to rupture of a perianal abscess with formation of a persistent sinus tract.
Symptoms include pain with defecation and chronic discharge. Management requires surgical intervention.

Reference
• Anorectal conditions: anal fissure and anorectal fistula.
Question #176

A 55-year-old man comes to the office due to progressive abdominal distension for the past 2 months. He reports
that his abdomen has become so large that his clothes no longer fit. Temperature is 37.5 C (99.5 F), blood
pressure is 152/87 mm Hg, pulse is 80/min, and respirations are 18/min. The abdomen is grossly enlarged and
nontender. Shifting dullness is present. Trace pitting edema is identified in the bilateral lower extremities.
Paracentesis reveals bloody ascitic fluid. Repeat paracentesis from another site yields similar bloody fluid. Which
of the following is most likely responsible for this patient's presentation?

A) Hepatocellular carcinoma

B) Nephrotic syndrome

C) Portal vein thrombosis

D) Spontaneous bacterial peritonitis

E) Tuberculosis
Explanation
Correct Answer:

A) Hepatocellular carcinoma

This patient has new-onset ascites (abdominal distension, shifting dullness) that is persistently bloody on multiple
paracenteses. Bloody ascites is caused by red blood cells leaking into the intraperitoneal fluid; ascites appears
frankly bloody when the peritoneal red blood cell count is >50,000 mm3. Most cases occur due to localized trauma
from a paracentesis; however, these bleeds usually resolve without intervention. Persistently bloody ascites found
on multiple paracenteses suggests an underlying malignancy.

Hepatocellular carcinoma (HCC) is the most common malignancy to present with bloody ascites due to tumor
growth disrupting and eroding nearby blood vessels. Less frequently, peritoneal metastases from distant primary
sites (eg, ovaries, prostate) can cause bloody ascites. Therefore, patients with bloody ascites should
undergo abdominal imaging, measurement of alpha-fetoprotein blood levels (elevated with HCC), and cytologic
analysis of the ascitic fluid to identify the primary tumor. Unless the patient is hemodynamically compromised, no
specific acute intervention is otherwise required for nontraumatic bloody ascites; in the rare case of hemodynamic
compromise, angiography with embolization should be considered.

(Choice B) Nephrotic syndrome can result in ascites in association with diffuse anasarca. It typically causes non-
bloody straw-yellow ascites, and most patients also have severe lower extremity edema. This patient's trace pitting
edema may be related to ascites and underlying cirrhosis (eg, hypoalbuminemia) but is unlikely to reflect nephrotic
syndrome.

(Choice C) Portal vein thrombosis does not commonly cause ascites because the obstruction occurs proximal to
the hepatic sinusoids. Hepatic vein thrombosis (ie, Budd-Chiari syndrome) can cause ascites; however, it typically
causes acute onset of non-bloody, straw-yellow ascites in association with fever and abdominal pain.

(Choice D) Spontaneous bacterial peritonitis typically causes fever, abdominal pain, and/or altered mental status
associated with cloudy or turbid ascitic fluid. A formal diagnosis requires an increase in ascitic fluid
polymorphonuclear cells (≥250 cells/mm3).

(Choice E) Abdominal tuberculosis can lead to ascites, but the fluid is generally straw-colored; bloody ascites is
rare. In addition, tuberculous ascites is typically associated with abdominal pain as well as fever and night sweats,
reflecting tuberculosis reactivation.

Educational objective:
Persistently bloody ascites after multiple diagnostic paracenteses is concerning for an underlying malignancy.
Hepatocellular carcinoma is the most common cause, although bloody ascites can also occur with peritoneal
metastases from distant primary sites (eg, ovaries, prostate). Cytologic analysis of the ascitic fluid can help identify
the primary tumor.
Question #177

A 22-year-old man comes to the office due to worsening burning sensations in his hands and feet for the past
several months. He also reports soreness in his mouth and has had intermittent episodes of a postprandial bloating
sensation and diarrhea. The patient drinks alcohol occasionally but does not use tobacco or illicit drugs. He is
sexually active and does not use condoms consistently. Vital signs are within normal limits. BMI is 20 kg/m2. On
physical examination, the tongue appears red and smooth. The neck is supple and has no lymphadenopathy.
Heart and lung sounds are normal. The abdomen is soft and nontender. Neurologic examination shows decreased
pinprick and vibration sensation in the bilateral fingers and toes. Muscle strength and deep tendon reflexes are
normal. Laboratory testing reveals microcytic anemia. Which of the following is the most likely underlying cause of
this patient's symptoms?

A) Alcohol-induced toxic neuropathy

B) Autoimmune process

C) Prolonged hyperglycemia

D) Retroviral infection

E) Thyroid dysfunction
Explanation
Correct Answer:

B) Autoimmune process

Clinical manifestations of celiac disease

• Diarrhea, ± steatorrhea, weight loss


• Abdominal pain
Gastrointestinal
• Flatulence/bloating
• Late manifestations: ulcerative jejunitis, enteropathy-associated T-cell lymphoma

• Dermatitis herpetiformis
Mucocutaneous
• Atrophic glossitis

• Vitamin D deficiency
Endocrine
• Secondary hyperparathyroidism

• Osteomalacia/osteoporosis (adults)
Bone disorders
• Rickets (children)

Hematologic • Iron deficiency anemia


• Peripheral neuropathy
Neuropsychiatric
• Depression/anxiety

This young patient has developed progressive paresthesia and sensory loss in a distal, symmetric, stocking-glove
distribution. This is characteristic of a length-dependent axonal polyneuropathy, which raises concern for an
underlying systemic disease. The findings of atrophic glossitis (ie, smooth, red tongue with burning pain),
microcytic anemia, and gastrointestinal symptoms (eg, bloating, diarrhea) are suspicious for celiac disease.

Celiac disease is an autoimmune disorder triggered by gluten ingestion. Classically, an immune response to
gliadin causes inflammation in the small intestine, resulting in atrophy of the villi, leading to malabsorption. In
addition to gastrointestinal symptoms, malabsorption can result in other clinical manifestations, including iron
deficiency anemia and bone mineral disorders related to vitamin D deficiency (eg, osteomalacia, osteoporosis,
rickets).

Other extraintestinal manifestations of celiac disease may be related to an overlapping autoimmune response.
Peripheral neuropathy, seen in up to 50% of patients, and other neuropsychiatric manifestations may be due to
autoantibody production, rather than nutritional deficiency, because they often precede symptoms of malabsorption.
Although atrophic glossitis may be caused by vitamin deficiencies, it might be due to autoantibodies, given that it
manifests early in the disease course.

(Choice A) Chronic alcohol use can cause polyneuropathy. However, the severity typically correlates with lifetime
alcohol consumption and therefore would be unlikely in a young patient who drinks occasionally. Postprandial
diarrhea is uncommon with alcohol use.

(Choice C) Prolonged hyperglycemia in uncontrolled diabetes can cause distal symmetric polyneuropathy.
However, this patient's atrophic glossitis, microcytic anemia, bloating, and diarrhea in the absence symptoms of
diabetes mellitus type 1 (eg, polyuria, polydipsia) or risk factors for diabetes mellitus type 2 (eg, obesity) make
celiac disease more likely.

(Choice D) Peripheral neuropathy can occur in patients with HIV either due to the neurotoxic effects of
antiretroviral medications or to mitochondrial dysfunction related to virus activity. Sore throat and painful
mucocutaneous ulceration, rather than atrophic glossitis, can be seen. Normocytic, rather than microcytic, anemia
is common with HIV. In addition, most patients with HIV have constitutional symptoms and lymphadenopathy.

(Choice E) Hypothyroidism can cause polyneuropathy. However, in addition to sensory loss, decreased deep
tendon reflexes are typically present. Constipation, rather than diarrhea, and normocytic anemia (due to decreased
proliferation of erythrocyte precursors), rather than microcytic anemia, would also be expected.

Educational objective:
Celiac disease can present with a diverse array of symptoms related to malabsorption (eg, diarrhea, microcytic
anemia, vitamin D deficiency) or autoimmune inflammation (eg, peripheral neuropathy, atrophic glossitis).

Reference
• Neurologic presentation of celiac disease.

• Neurological manifestations of neuropathy and ataxia in celiac disease: a systematic review.

• Celiac neuropathy.
Question #178

A 45-year-old man from Guatemala comes to the office due to persistent nausea and vomiting of partially digested
food. He has had these symptoms for the past 1 month. He has also lost 2.3 kg (5 lb) during this period of time.
His appetite is good, but he mentions early satiety. He has not had any hematemesis, black stools, difficulty
swallowing, and chest pain. Other medical problems include type 2 diabetes for the past 1 year and a suicide
attempt 3 months ago in which he ingested acid. He has a history of peptic ulcer disease and often takes antacids
for heartburn. He drinks alcohol and smokes one pack of cigarettes daily. His temperature is 36.8 C (98.2 F), blood
pressure is 110/65 mm Hg, pulse is 110/min, and respirations are 16/min. Mucous membranes are dry. Abdominal
examination shows succussion splash on the epigastrium. Which of the following is the most likely diagnosis?

A) Achalasia

B) Chronic pancreatitis

C) Diabetic gastroparesis

D) Esophageal stricture

E) Portal hypertension

F) Pyloric stricture
Explanation
Correct Answer:

F) Pyloric stricture

This patient presents with signs/symptoms consistent with gastric outlet obstruction caused by mechanical
obstruction, leading to postprandial pain and vomiting with early satiety. Common causes of gastric outlet
obstruction include gastric malignancy, peptic ulcer disease, Crohn disease, strictures (with pyloric stenosis)
secondary to ingestion of caustic agents, and gastric bezoars.

Physical examination can show an abdominal succussion splash, which is elicited by placing the stethoscope over
the upper abdomen and rocking the patient back and forth at the hips: Retained gastric material >3 hours after a
meal will generate a splash sound and indicates the presence of a hollow viscus filled with both fluid and gas.

This patient ingested acid 3 months ago, which is a risk factor for the development of a pyloric stricture. Acid
ingestion causes fibrosis 6-12 weeks after the resolution of the acute injury. Upper endoscopy is usually required to
confirm the diagnosis, and treatment is primarily surgical.

(Choices A and D) Esophageal stricture and dysmotility (eg, achalasia) tend to present with dysphagia, which is
not this patient's presenting symptom. In addition, abdominal succussion splash is not a typical finding in
esophageal stricture or achalasia.

(Choice B) Chronic pancreatitis can lead to inflammation and fibrosis in adjacent structures (e.g., duodenum,
jejunum, and transverse colon) that can rarely lead to obstruction. However, gastric obstruction is not usually
caused by pancreatitis.

(Choice C) Diabetic gastroparesis tends to occur in patients who have had diabetes for longer than a decade. This
patient was diagnosed with diabetes only 1 year ago and is less likely to have gastroparesis.

(Choice E) Portal hypertension typically does not lead to gastric obstruction or an abdominal succussion splash on
physical examination.
Educational objective:
Gastric outlet obstruction can be caused by many disease processes and is characterized by early satiety, nausea,
nonbilious vomiting, and weight loss. In a patient with a history of acid ingestion, pyloric stricture is the most likely
cause.

Reference
• Pyloric and antral strictures following corrosive acid ingestion: A report of four cases.

• Caustic injury of the upper gastrointestinal tract in adults: a clinical and endoscopic study.
Question #179

A 65-year-old man comes to the office due to a several month history of difficulty swallowing and frequent coughing
during meals. His wife reports that his breath odor has changed and his voice sounds "gurgly." The patient
occasionally regurgitates food or medications taken earlier in the day. Medical history is significant for hypertension,
gastroesophageal reflux disease, and osteoarthritis of the right knee. Medications include hydrochlorothiazide,
famotidine, and, occasionally, naproxen as needed. Which of the following mechanisms leads to the development
of this patient's condition?

A) Abnormal cellular proliferation

B) Acid reflux

C) Inflammation

D) Metabolic abnormalities

E) Motor dysfunction
Explanation
Correct Answer:

E) Motor dysfunction
This patient with dysphagia, halitosis, and a history of regurgitating undigested food likely has a Zenker
(pharyngoesophageal) diverticulum. Patients are typically age >60 and have an insidious onset of progressive
dysphagia. Retained food trapped in the pouch causes halitosis and sometimes a "gurgling" sound. This food is
often regurgitated later and appears undigested because it has not been exposed to gastric enzymes. Aspiration
of the regurgitated food may lead to recurrent aspiration pneumonia.

Abnormal spasm or diminished relaxation of the cricopharyngeal muscles during swallowing (cricopharyngeal
motor dysfunction) is the underlying mechanism of the formation of a Zenker diverticulum. Increased intraluminal
pressure above the cricopharyngeus muscle eventually causes herniation of the mucosa, resulting in a pulsion (ie,
pushing) pseudodiverticulum. This occurs through a zone of muscle weakness between inferior pharyngeal
constrictor fibers and the cricopharyngeus muscle.

Diagnosis is confirmed with a contrast swallow study. Treatment is surgical. The cricopharyngeus muscle is divided
(cricopharyngeal myotomy), and the diverticulum may be removed (ie, diverticulectomy) or combined with the
esophageal lumen (ie, diverticulotomy).

(Choice A) Abnormal cellular proliferation is the underlying pathophysiologic defect in neoplasia. Esophageal
cancer may cause dysphagia, but not typically regurgitation of undigested food.

(Choice B) Gastroesophageal reflux disease (GERD) is frequently associated with Zenker diverticula, but it is not
causative. GERD can cause dysphagia and regurgitation of acid, but it does not typically cause regurgitation of
undigested food or medications that were consumed several hours before.

(Choice C) Chronic inflammation in the mediastinum (eg, due to tuberculosis or fungal infections) can lead to the
formation of midesophageal diverticula due to the pull (traction) of adjacent scar tissue. In contrast to pulsion
diverticula, these traction diverticula are true diverticula (ie, involve the mucosal, submucosal, and muscular
layers). Traction diverticula are very rare, and this patient has no signs of tuberculosis or other mediastinal
inflammatory processes.

(Choice D) Metabolic abnormalities (eg, iron deficiency) may be associated with upper esophageal webs
(Plummer-Vinson syndrome) that can cause an insidious onset of dysphagia. However, halitosis and regurgitation
of undigested food are not typical.
Educational objective:
Diminished relaxation of the cricopharyngeus muscle during swallowing results in increased intraluminal pressure in
the hypopharynx. This may cause the mucosa to herniate, forming a Zenker (pharyngoesophageal) diverticulum,
which presents in patients age >60 with dysphagia, halitosis, and regurgitation of undigested food.

Reference
• Zenker's diverticula: pathophysiology, clinical presentation, and flexible endoscopic management.
Question #180

A 45-year-old man comes to the office due to progressive shortness of breath on exertion and fatigue for the past
several weeks. Two years ago, the patient was diagnosed with alcoholic cirrhosis and esophageal varices after
presenting with hematemesis. He underwent banding of the varices. Several months ago, the patient came to the
office with massive ascites and a large-volume paracentesis was performed. His current medications include
furosemide, spironolactone, and nadolol. He has had no alcohol for the past 2 years. The patient does not use
tobacco or illicit drugs. Blood pressure is 114/72 mm Hg and pulse is 63/min. Pulse oximetry shows 95% on room
air. There is no change in dyspnea on lying down or moving into an upright position. His neck veins are flat. Heart
sounds are normal with no murmur or gallop. Dullness and decreased breath sounds are present on the right. Left-
sided breath sounds are normal with no added sounds. The abdomen is moderately distended with shifting
dullness. Bowel sounds are normal. Stool guaiac is negative. He has 1+ bilateral lower extremity pitting edema.
Which of the following is the most likely cause of this patient's current symptoms?

A) Alcohol-induced cardiac dysfunction

B) Decreased diaphragmatic excursion

C) Fluid passage through diaphragmatic defects

D) Intrapulmonary vascular dilations

E) Medication adverse effects

F) Recurrent variceal bleeding


Explanation
Correct Answer:

C) Fluid passage through diaphragmatic defects

Patients with cirrhosis and portal hypertension frequently have abdominal ascites and peripheral edema due to low
albumin levels and abnormal extracellular fluid volume regulation. A small number of these patients may also
develop hepatic hydrothorax, a pleural effusion not due to underlying cardiac or pulmonary abnormalities.

Hepatic hydrothorax generally results in transudative pleural effusions and is thought to occur due to small
defects in the diaphragm. These defects permit peritoneal fluid to pass into the pleural space, which occurs much
more commonly on the right side due to the less muscular hemidiaphragm. Patients have dyspnea, cough,
pleuritic chest pain, and hypoxemia. Diagnosis involves documentation of the effusion (eg, chest x-ray) and testing
to exclude other causes (eg, thoracentesis, echocardiogram).

Treatment involves salt restriction and diuretic administration. Therapeutic thoracentesis could be attempted in
patients with prominent symptoms. Chest tube placement should be avoided as it can result in large-volume
protein, fluid, and electrolyte losses as well as other severe complications (eg, renal failure). The definitive option
for treatment is liver transplantation, although this may not be appropriate for all patients depending on other
factors.

(Choice A) This patient's lack of other findings suggestive of heart failure (eg, increased jugular venous pressure,
S3, bilateral crackles) makes alcoholic cardiomyopathy much less likely.

(Choice B) Tense ascites can result in dyspnea and decrease the range of diaphragmatic excursion due to
increased intraabdominal pressure, but this is unlikely to cause right-sided dullness and decreased breath sounds.

(Choice D) Hepatopulmonary syndrome results from intrapulmonary vascular dilations in the setting of chronic liver
disease. Patients frequently have evidence of platypnea (increased dyspnea while upright) or orthodeoxia (oxygen
desaturation while upright).
(Choices E and F) Although beta blockers (eg, nadolol) or chronic anemia due to recurrent bleeding can result in
increased fatigue and dyspnea, this does not explain the patient's pulmonary examination findings. Moreover,
variceal bleeding is usually large volume and more clinically evident (eg, hematemesis, positive stool guaiac).

Educational objective:
Hepatic hydrothorax generally results in transudative pleural effusions and is thought to occur due to small defects
in the diaphragm. Diagnosis involves documentation of the effusion and testing to exclude other causes.

Reference
• Pulmonary complications of hepatic diseases.
Question #181

A 38-year-old man with moderate intellectual disability from Down syndrome comes to the office with his caregiver
for an annual medical evaluation. The caregiver states that the patient previously had bowel movements almost
every day, but over the past several months, he has had them only every 3 or 4 days. He now has hard stools with
straining during defecation, but there is no blood in the bowel movements. The patient reports no abdominal pain,
nausea, or vomiting, and his appetite is normal. His mobility is limited, and he takes no medications. Blood
pressure is 132/88 mm Hg and pulse is 70/min. BMI is 30 kg/m2. Physical examination shows an obese, soft
abdomen with no tenderness, palpable masses, or organomegaly. Digital rectal examination reveals no tenderness
or mass lesion, and fecal occult blood testing is negative. In addition to increased dietary fiber and water intake,
which of the following is the most appropriate next step in management of this patient?

A) Abdominal CT scan with oral contrast

B) Anorectal manometry

C) Barium enema study

D) No additional diagnostic testing

E) Thyroid function studies


Explanation
Correct Answer:

E) Thyroid function studies

Common secondary causes of constipation

Lifestyle • Low-fiber diet, dehydration, inactivity

Functional • Irritable bowel syndrome, depression

Medications • Opioids, iron supplements, anticholinergics*, CCBs

Metabolic • Hypothyroidism, diabetes mellitus, hypercalcemia

Obstruction • Colon cancer, inflammatory/ischemic/surgical stricture

Neurologic • Spinal cord injury, multiple sclerosis, Parkinson disease

*Anticholinergic medication classes: antipsychotics, antidepressants, antihistamines, antispasmodics.


CCBs = calcium channel blockers.

This patient with moderate intellectual disability from Down syndrome has had a recent change in bowel habits
consistent with constipation (eg, straining, hard stools, frequency <3/week). Constipation is common in patients
with intellectual disabilities and is often secondary to limited mobility or an adverse effect of medications typically
used in this population (eg, antipsychotics, antidepressants). However, recent-onset constipation may be the first
manifestation of an underlying medical condition and requires further evaluation for a secondary cause.

Patients with Down syndrome are at increased risk for endocrine abnormalities (possibly due to defects in
organogenesis or regulatory genetic pathways). Hypothyroidism is a common metabolic cause of constipation
(due to decreased motility), and thyroid function studies should be performed. Other metabolic causes (eg,
diabetes mellitus, hypercalcemia) of constipation should also be ruled out.

(Choice A) Abdominal CT scan with oral contrast can identify secondary causes of constipation due to mechanical
obstruction (eg, colon cancer, colonic stricture). It is not initially indicated in patients, such as this one, with a
negative physical examination and no alarm features. However, abdominal CT scan should be considered if
symptoms change or remain unexplained.

(Choices B and C) Anorectal manometry and barium enema are useful in diagnosing congenital aganglionic
megacolon (Hirschsprung disease) as a cause of constipation. Although Hirschsprung disease is more common in
patients with Down syndrome, it would be highly unusual to acutely emerge in this 38-year-old patient; it typically
presents in the neonatal period with abdominal distension, bilious emesis, and delayed passage of meconium.

(Choice D) In patients with an unrevealing history, negative physical examination, and absence of alarm features
(eg, weight loss, hematochezia), empiric treatment (eg, increased fiber and fluid intake) is a reasonable initial
approach without further diagnostic testing. However, this patient's recent onset of constipation, young age, and
risk factor for endocrine abnormalities warrants additional evaluation for secondary causes.

Educational objective:
Recent-onset constipation may indicate an underlying medical condition. In patients with Down syndrome who are
at risk for endocrine abnormalities, evaluation for secondary causes should include laboratory assessment for
underlying metabolic conditions (eg, hypothyroidism, diabetes mellitus, hypercalcemia).
Question #182

A 36-year-old man comes to the office after experiencing intermittent episodes of rectal bleeding over the past 2
months. Initially, he noted only small streaks of blood on the tissue when wiping, but yesterday he dripped bright
red blood after defecating and the stool was coated with blood. He has had no anal or abdominal pain, nausea,
vomiting, anorexia, or weight loss. The patient has been having bowel movements every 2 or 3 days, which is
unchanged from his previous routine. He has no other medical concerns and takes no medications. The patient's
father was diagnosed with colon cancer at age 54. Blood pressure is 130/70 mm Hg, and pulse is 80/min with no
orthostatic changes. Physical examination shows a soft and nontender abdomen with no masses or
organomegaly. Inspection of the perianal skin shows no abnormalities. No mass or tenderness is palpable on
digital rectal examination. Anoscopy reveals purplish mucosal bulges. Which of the following is the best next step
in management of this patient?

A) Biopsy of the anal lesions

B) High-fiber diet and follow-up

C) Referral for colonoscopy

D) Referral for surgical evaluation

E) Topical nifedipine therapy


Explanation
Correct Answer:

B) High-fiber diet and follow-up

Initial management of hemorrhoids

• Increased fluid intake


• Increased fiber intake (foods, fiber supplements)
Dietary factors
• Reduced fat intake
• Moderation of alcohol intake

• Limit time sitting on toilet (eg, 3 minutes)


Behavioral factors • Limit defecation to once daily
• Avoid straining during defecation

• Analgesics (eg, benzocaine)


Topical agents • Astringents (eg, witch hazel)
• Hydrocortisone

This patient has painless, bright red rectal bleeding and mucosal bulges on anoscopy consistent with internal
hemorrhoids. Hemorrhoids are the most common cause of low-volume rectal bleeding, especially in patients who
do not have red-flag features (eg, passage of dark blood or blood mixed with stool, fever, abdominal pain, change in
bowel habits) to suggest more serious causes. The diagnosis is usually obvious based on clinical findings.
Anoscopy is useful for those whose hemorrhoids are not palpable on examination, but further diagnostic testing is
not usually required for young, otherwise healthy patients with typical symptoms, such as this patient.

Initial management of uncomplicated hemorrhoids includes increased intake of fluid and fiber (eg, psyllium husk),
reduction in fat and alcohol intake, and regular exercise. Additional measures may include phlebotonics (eg,
calcium dobesilate), topical hydrocortisone, astringents (eg, witch hazel), and local anesthetics (eg, benzocaine,
lidocaine). More aggressive management, including rubber band ligation and surgical hemorrhoidectomy, is
generally advised only for patients with refractory symptoms or prolapsed hemorrhoids that cannot be reduced
manually (ie, grade IV hemorrhoids) (Choice D).

(Choice A) Biopsy is warranted for pedunculated or sessile colorectal polyps, which typically present as pink, red,
or flesh-colored papules or masses. The purplish bulges in this patient are typical for hemorrhoids, and biopsy
would lead only to unnecessary bleeding.

(Choice C) Colonoscopy is recommended for patients who have profuse rectal bleeding or who are at high risk for
colorectal cancer. High risk features include prior adenomatous colon polyps, age ≥50 (unless the patient has had
a negative colonoscopy within the last 2-3 years), age 40-49 with a first-degree relative with colorectal cancer at
age <60, or a familial colon cancer syndrome (eg, Lynch syndrome). Younger patients, such as this patient, with
low-volume bleeding (eg, blood on the surface of the stool, drips in the toilet) and a benign explanation (eg,
hemorrhoids) may be monitored without colonoscopy.

(Choice E) Topical nifedipine is used to reduce anal sphincter pressure in patients with anal fissures. Fissures can
cause bright red rectal bleeding, but are typically associated with severe pain and would not present as a purplish
mucosal bulge.

Educational objective:
Initial management of uncomplicated hemorrhoids includes increased intake of fluid and fiber, reduction of fat and
alcohol intake, and regular exercise. Additional measures may include phlebotonics, topical hydrocortisone,
astringents, and local anesthetics. Rubber band ligation and surgical hemorrhoidectomy are advised only for
patients with refractory symptoms or prolapsed hemorrhoids that cannot be reduced manually.

Reference
• Adequate dietary fiber supplement and TONE can help avoid surgery in most patients with advanced
hemorrhoids.
Question #183

A 72-year-old man comes to the hospital due to several episodes of bright red blood per rectum. He also has
dizziness and severe fatigue. The patient has never had prior episodes of gastrointestinal bleeding. He takes
lisinopril and amlodipine for hypertension. The patient is a lifetime nonsmoker. Laboratory evaluation reveals
decreased levels of von Willebrand factor multimers but a normal platelet count. Upper endoscopy and
colonoscopy do not identify a source of bleeding. Which of the following is most likely associated with this patient's
condition?

A) Calcific aortic stenosis

B) Chronic liver disease

C) Glucose intolerance

D) Uncontrolled hypertension

E) Vitamin B12 deficiency


Explanation
Correct Answer:

A) Calcific aortic stenosis

This patient's brisk gastrointestinal (GI) bleeding with no identifiable source on upper or lower endoscopy suggests
bleeding in the distal duodenum, jejunum, or ileum, which are not well visualized with standard endoscopy. Small
bowel bleeding is often due to vascular malformations in the bowel wall, particularly angiodysplasias.

Angiodysplasias are tortuous, dilated, thin-walled vessels lined by a layer of endothelium with or without a small
amount of smooth muscle. They are thought to form due to recurrent, intermittent obstruction of distal venules in
the muscularis propria, which leads to proximal arterial damage. Symptomatic angiodysplasias are most common
in the GI tract and primarily occur in individuals age >60. Risk of bleeding from angiodysplasias is increased with
end-stage renal disease and aortic stenosis (AS).

AS is thought to trigger bleeding from angiodysplasias due to destruction of circulating von Willebrand factor (VWF)
multimers when they pass through the damaged valve. Because VWF multimers play a crucial role in primary
hemostasis by binding platelets to sites of endothelial injury, reduction in circulating levels of VWF increases the risk
of bleeding significantly.

(Choice B) Chronic liver disease is associated with portal hypertension, which can cause bleeding from gastric or
esophageal varices. However, varices are generally seen on upper endoscopy. In addition, chronic liver disease is
often associated with elevated INR due to deficiency in coagulation factors; VWF is not typically affected because it
is primarily produced by the endothelium.

(Choice C) Glucose intolerance is associated with the development of type 2 diabetes mellitus, which is
characterized by excessive elevations in blood glucose following meals due to insulin resistance and impaired
insulin secretion.

(Choice D) Uncontrolled, resistant hypertension may be seen in patients with obstructive sleep apnea, renal artery
stenosis, and primary aldosteronism. Uncontrolled hypertension is not a common cause of low VWF multimers and
GI bleeding. Severe hypertension can cause anemia due to mechanical destruction of erythrocytes (eg,
schistocytes).

(Choice E) Vitamin B12 deficiency is often caused by disorders of the GI tract, such as atrophic gastritis, celiac
disease, and bacterial overgrowth in the small bowel. Although vitamin B12 deficiency commonly causes
megaloblastic anemia and is sometimes associated with mild thrombocytopenia, it is not tightly linked to low VWF
multimers or GI bleeding.

Educational objective:
Angiodysplasias in the gastrointestinal (GI) tract are a common source of GI bleeding. Bleeding from
angiodysplasias may be triggered by underlying aortic stenosis, which is associated with low levels of von
Willebrand factor multimers; this glycoprotein is often destroyed when it passes through the damaged valve at high
velocity.
Question #184

A 68-year-old woman comes to the office due to food getting "stuck" in her throat, which has worsened over the
past 2 years. She occasionally has to get out of bed to vomit, although she does not vomit during the day. She has
not changed her diet or lost weight. Medical history includes obesity, hypertension, and diabetes. A barium
esophagogram is shown in the image below. Which of the following is the most appropriate next step in
management of this patient?
A) Cricopharyngeal myotomy

B) Esophageal balloon dilation

C) Ligamentum arteriosum ligation

D) Oral metoclopramide

E) Oral omeprazole

F) Swallowing rehabilitation
Explanation
Correct Answer:

A) Cricopharyngeal myotomy
This patient's lateral view of a contrast esophagogram shows a posterior outpouching of the cervical esophagus
that is connected to the true lumen. This is consistent with a Zenker (pharyngoesophageal) diverticulum (ZD). The
diverticular pouch can trap food and medication. Patients often regurgitate the undigested food or medications
several hours after eating. This can be more pronounced when the individual lies supine, as in this patient.

Abnormal spasm or diminished relaxation of the cricopharyngeal muscles during swallowing (cricopharyngeal
motor dysfunction) is thought to be the underlying mechanism of ZD formation. Increased intraluminal pressure
above the cricopharyngeus muscle eventually results in herniation of the mucosa through an area of weakness.
Therefore, ZD is classified as a pulsion (resulting from increased intraluminal forces), rather than a traction
(resulting from external tugging forces), pseudodiverticulum.

Treatment of symptomatic patients involves surgical division of the cricopharyngeus muscle (cricopharyngeal
myotomy). The diverticular pouch can be removed or combined with the esophageal lumen.

(Choice B) Esophageal balloon dilation is used to treat benign esophageal strictures, which present with
dysphagia limited to solid foods. However, on swallow study, strictures appear as a smoothly tapering, concentric
narrowing of the esophageal lumen.

(Choice C) Ligamentum arteriosum ligation is used to treat some patients with vascular ring, a congenital
malformation of the aortic arch system that encircles the trachea and/or esophagus and can cause compressive
symptoms. A contrasted swallow study would show external compression of the esophagus rather than a
diverticular pouch.

(Choices D and E) Oral metoclopramide may be used for gastroparesis to improve motility but is rarely used for
esophageal disease. Oral omeprazole is used to treat gastroesophageal reflux disease, which often coexists with
ZD. Medical therapy would not improve the underlying cricopharyngeal dysfunction or the diverticular pouch.

(Choice F) Swallowing rehabilitation is used to treat patients with oropharyngeal dysphagia due to neurologic
disease, trauma, or surgery. It typically strengthens oropharyngeal musculature but would not relax the
cricopharyngeus muscle.

Educational objective:
Impaired relaxation of the cricopharyngeus muscle during swallowing may lead to the formation of a Zenker
diverticulum, which is seen on contrast swallow study as a pouch posterior to the esophagus. Treatment is surgical
with cricopharyngeal myotomy with or without diverticulectomy.

Reference
• Zenker's diverticulum: exploring treatment options.
Question #185

A 52-year-old man comes to the office due to abnormal liver chemistry results found during a life insurance medical
examination. He occasionally feels fatigued and has pedal edema after prolonged standing but otherwise feels
well. The patient has no medical history and takes no medications. He drinks 1 or 2 cans of beer on weekends and
does not smoke. He used intravenous drugs in his 20s but has not since then. Vital signs are within normal range.
Physical examination shows palmar erythema and multiple spider angiomas. Laboratory results are as follows:

Complete blood count


Hemoglobin 11.2 g/dL
Platelets 120,000/mm3

Serum chemistry
Creatinine 0.8 mg/dL

Liver function studies


Albumin 3.4 g/dL
Total bilirubin 1.3 mg/dL
Aspartate aminotransferase 62 U/L
Alanine aminotransferase 99 U/L

Coagulation studies
INR 1.4

Immunologic and rheumatologic studies


Hepatitis C virus antibody positive
Hepatitis C RNA 1 million copies/mL

Abdominal ultrasonography reveals a coarse, nodular-appearing liver with no masses, mild ascites, and
splenomegaly. Upper gastrointestinal endoscopy demonstrates medium-sized but nonbleeding esophageal
varices. He is referred for a hepatitis C evaluation with a gastroenterologist. Which of the following is the best
management for this patient at this time?

A) ACE inhibitors

B) Beta-adrenergic receptor blockers

C) Endoscopic sclerotherapy

D) H2 histamine receptor blockers

E) Octreotide injections
Explanation
Correct Answer:

B) Beta-adrenergic receptor blockers

This patient has evidence of cirrhosis (eg, spider angioma, nodular coarse liver, varices), likely from chronic
hepatitis C. He is found to have medium-sized, nonbleeding esophageal varices. Variceal hemorrhage develops
in approximately one third of all patients with varices and is a major cause of morbidity and mortality. Most cirrhotic
patients should undergo diagnostic upper endoscopy to assess for varices and to determine their risk of
hemorrhage. Those with (medium- or large-sized) varices, such as this patient, generally should be started on a
nonselective beta blocker.

Nonselective beta blockers (eg, propranolol, nadolol) are recommended to decrease progression to large varices
and the risk of variceal hemorrhage. They are thought to act by decreasing adrenergic tone in mesenteric
arterioles, which results in unopposed alpha-mediated vasoconstriction and decreased portal venous flow.
Endoscopic variceal ligation is an alternate primary preventive therapy in patients with contraindications to beta
blocker therapy.

(Choice A) ACE inhibitors have not been shown to reduce variceal bleeding or improve outcomes in cirrhosis, and
they have no role in the management of patients with esophageal varices.

(Choice C) Endoscopic sclerotherapy is an effective treatment for actively bleeding esophageal varices. It is not
recommended for primary prophylaxis of variceal hemorrhage.

(Choice D) H2 histamine receptor blockers have not been shown to reduce progression or bleeding complications
in patients with cirrhosis and varices.

(Choice E) Octreotide is a long-acting somatostatin analogue that causes splanchnic vasoconstriction and reduced
portal blood flow by inhibiting the release of glucagon. It is used in the treatment of active variceal bleeding and has
no role in primary prophylaxis.
Educational objective:
In most patients with nonbleeding varices, prophylactic treatment with nonselective beta blockers (eg, propranolol,
nadolol) is recommended to reduce the likelihood of progression to large varices as well as the risk of variceal
hemorrhage. Endoscopic variceal ligation can be used as an alternate primary preventive therapy in patients with
contraindications to beta blocker therapy.

Reference
• Expanding consensus in portal hypertension: report of the Baveno VI Consensus Workshop: stratifying risk
and individualizing care for portal hypertension.
Question #186

A 62-year-old man comes to the office due to fatigue and weight loss. Over the last 6 months, he has lost 9 kg (20
lb) and can no longer walk long distances without sitting down to rest. The patient has not seen a physician in 15
years. He has smoked a pack of cigarettes daily for the last 35 years but does not use alcohol or illicit drugs. He
has not recently traveled. Temperature is 36.9 C (98.4 F), blood pressure is 130/80 mm Hg, pulse is 88/min, and
respirations are 12/min. Lungs are clear to auscultation, and abdominal examination shows a liver span of 14 cm
with no tenderness. Rectal examination reveals normal sphincter tone but a slightly enlarged prostate that is
nontender to palpation. Chest x-ray is unremarkable. CT scan of the abdomen with contrast is shown in the image
below.
Which of the following is most likely to establish the diagnosis in this patient?

A) Alpha-fetoprotein measurement

B) Aspiration and culture of the liver lesions

C) Bone marrow biopsy

D) Colonoscopy

E) Prostate-specific antigen measurement

F) Stool test for ova and parasites


Explanation
Correct Answer:

D) Colonoscopy

This patient with weight loss and fatigue has multiple liver lesions on CT scan, suggesting metastatic disease to
the liver rather than primary liver cancer. Colorectal cancer is the most common source of liver metastases (as
blood from the colon moves through the portal circulation directly to the liver) and should be excluded in this patient,
even in the absence of specific symptoms (eg, alteration in bowel habits, gastrointestinal bleeding). Abdominal CT
is a useful screening test but can often miss primary intraluminal tumors. Colonoscopy is the most appropriate
next diagnostic step as it both localizes the tumor and provides a tissue diagnosis.

Lung and breast cancers also metastasize to the liver, although less commonly than gastrointestinal malignancies.
This patient is a smoker; however, he has a normal chest x-ray and no respiratory symptoms (eg, cough), making
metastatic lung cancer unlikely.

(Choice A) Alpha-fetoprotein measurement is used along with imaging studies in the diagnosis of hepatocellular
carcinoma, which most commonly occurs in the setting of chronic liver disease and presents as a solitary mass.

(Choice B) With pyogenic liver abscesses, aspiration and culture can guide antibiotic therapy. However,
abscesses typically present acutely with fever, right upper quadrant pain, anorexia, and weight loss. Serologic
studies can distinguish between pyogenic and amebic liver abscesses.

(Choice C) Bone marrow biopsy is useful in establishing a diagnosis of myeloproliferative or lymphoproliferative


disorders. It would be uncommon for these disorders to cause multiple liver masses.

(Choice E) Prostate-specific antigen measurement is a useful marker for prostate cancer. However, prostate
cancer tends to be an indolent malignancy that commonly metastasizes to the pelvic lymph nodes and bones rather
than the liver.

(Choice F) Liver flukes can be diagnosed with ova and parasite stool tests. However, they tend to invade the
biliary tree rather than the liver parenchyma, and the patient has no history of recent travel abroad, making this
diagnosis unlikely.

Educational objective:
Multiple liver masses are much more likely to be the result of metastatic disease than infectious causes or primary
liver malignancy. Primary tumors of the gastrointestinal tract, lung, and breast are the most common diseases
causing liver metastases.
Question #187

A 22-year-old man comes to the office due to yellow eyes lasting a day. Over the past 3 days, the patient has had
fever, fatigue, nasal discharge, and sore throat. He has no chronic medical conditions. His only medication is
acetaminophen as needed for current symptoms. The patient drinks 4 or 5 alcoholic beverages a week, and the
last drink was 3 days ago. Temperature is 38 C (100.4 F), blood pressure is 120/78 mm Hg, pulse is 80/min, and
respirations are 16/min. Examination shows scleral icterus; the oropharynx is erythematous without exudates.
There is no pallor or lymphadenopathy. Liver span is 11 cm, and a firm, nontender liver edge is palpable. There is
no splenomegaly. Laboratory results are as follows:

Complete blood count


Hemoglobin 13 g/dL
Reticulocytes 1%
Leukocytes 7,500/mm3
Neutrophils 50%
Lymphocytes 40%

Liver function studies


Total bilirubin 3.5 mg/dL
Direct bilirubin 0.2 mg/dL
Alkaline phosphatase 78 U/L
Aspartate aminotransferase (SGOT) 16 U/L
Alanine aminotransferase (SGPT) 20 U/L

What is the most likely cause of jaundice in this patient?


A) Alcohol-mediated hepatocellular injury

B) Decreased intracellular storage of bilirubin

C) Genetically mediated decreased enzyme activity

D) Infection-induced hemolytic anemia

E) Virus-mediated hepatocellular injury


Explanation
Correct Answer:

C) Genetically mediated decreased enzyme activity

Gilbert syndrome

Epidemiology • Most common inherited disorder of bilirubin metabolism

Pathogenesis • ↓ Hepatic UDP glucuronosyltransferase activity → ↓ conjugation of bilirubin

• Recurrent episodes of mild jaundice


Clinical
• Provoked by stress (eg, febrile illness, fasting, dehydration, vigorous exercise,
findings
menstruation, surgery)

• ↑ Unconjugated bilirubin (ie, indirect hyperbilirubinemia)


Diagnosis • Normal CBC, blood smear, reticulocyte count
• Normal AST, ALT, alkaline phosphatase

Treatment • Benign; no treatment required


ALT = alanine aminotransferase; AST = aspartate aminotransferase; CBC = complete blood count; UDP = uridine
diphosphogluconurate.

This patient has scleral icterus and indirect (unconjugated) hyperbilirubinemia in the setting of an upper respiratory
tract infection (eg, fever, fatigue, nasal discharge, sore throat). This presentation is consistent with Gilbert
syndrome, the most common inherited disorder of bilirubin glucuronidation.

Gilbert syndrome is characterized by episodic jaundice due to decreased activity of UDP glucuronosyltransferase,
the hepatic enzyme responsible for bilirubin conjugation. These benign episodes can be precipitated by periods of
stress, such as febrile illness (as seen in this patient), dehydration, fasting, vigorous exercise, menstruation, or
surgery.

A presumptive diagnosis of Gilbert syndrome is made based on the presence of persistent indirect
hyperbilirubinemia in the absence of other laboratory abnormalities (eg, hepatic aminotransferases, complete
blood count, reticulocyte count). This condition is benign, and reassurance should be provided.

(Choice A) Excessive alcohol consumption (typically >6 drinks/day) can lead to alcoholic hepatitis, which causes
fever and jaundice. However, aspartate aminotransferase (AST) and alanine aminotransferase (ALT) would be
elevated in a ≥2:1 ratio. In addition, tender hepatomegaly is typical and in contrast to this patient's nontender,
normal-sized (≤15 cm) liver.

(Choice B) Rotor syndrome, a rare autosomal recessive disorder, can cause episodic jaundice due to impaired
intrahepatic storage of conjugated bilirubin. However, a predominantly direct (conjugated) hyperbilirubinemia would
be expected.

(Choice D) Hemolytic anemia, which may be triggered by certain viral infections (eg, Epstein-Barr virus, HIV),
increases unconjugated bilirubin production due to red blood cell destruction. In contrast to this patient, a low
hemoglobin level and an increased reticulocyte count would be expected in a patient with hemolytic anemia.

(Choice E) Acute viral hepatitis can cause fever, jaundice, and hyperbilirubinemia. However, significant elevations
in AST and ALT, sometimes exceeding 1,000 U/L, would be expected, making this diagnosis very unlikely. In
addition, predominant indirect hyperbilirubinemia in the absence of hemolysis is not common with viral infections.

Educational objective:
Gilbert syndrome is characterized by intermittent jaundice and indirect (unconjugated) hyperbilirubinemia in the
absence of other laboratory abnormalities. Jaundice is often triggered by periods of stress, such as febrile illness,
dehydration, fasting, vigorous exercise, menses, and surgery.

Reference
• Gilbert syndrome.
Question #188

A 60-year-old woman comes to the office due to progressive fatigue over the past 6 months. She has no weight
loss, joint pain, hematochezia, vomiting, dyspnea, cold intolerance, anxiety, or depression. Medical history is
significant for hyperlipidemia, for which the patient has been taking simvastatin for 5 years. Family history is
unremarkable. She does not use tobacco, alcohol, or illicit drugs. Temperature is 37.6 C (99.7 F), blood pressure is
142/80 mm Hg, pulse is 80/min, and respirations are 12/min. Sclera are anicteric and mucosal membranes are
moist. Heart and lung sounds are unremarkable. The abdomen is nontender and nondistended. Hepatomegaly is
present. Skin and musculoskeletal examinations are unremarkable. Laboratory results are as follows:

Complete blood count


Hemoglobin 14.1 g/dL
Platelets 435,000/mm3
Leukocytes 7,000/mm3

Liver function studies


Albumin 3.9 g/dL
Total bilirubin 0.7 mg/dL
Alkaline phosphatase 90 U/L
Aspartate aminotransferase (SGOT) 175 U/L
Alanine aminotransferase (SGPT) 192 U/L

Blood, plasma, and serum


Ferritin 1625 ng/mL

Serology for viral hepatitis is negative. Right upper quadrant ultrasound reveals an enlarged liver with normal
echotexture and no evidence of cirrhosis. This patient would most benefit from which of the following?
A) Discontinue simvastatin

B) Oral prednisone

C) Penicillamine

D) Phlebotomy

E) Repeat blood tests in 1 year

F) Ursodeoxycholic acid
Explanation
Correct Answer:

D) Phlebotomy

Hereditary hemochromatosis

• Hyperpigmentation
• Arthropathy
• Hepatomegaly, cirrhosis, hepatocellular carcinoma
Clinical manifestations
• Diabetes mellitus
• Hypopituitarism, secondary hypogonadism
• Cardiomyopathy

• Elevated liver transaminases


Diagnosis • Elevated serum ferritin, transferrin saturation
• HFE genetic mutations

Management • Therapeutic phlebotomy (urgent if ferritin >1,000 ng/mL)

This patient has clinical features suggestive of hereditary hemochromatosis (HH), including hepatomegaly,
elevated liver transaminases, and a striking elevation in serum ferritin. HH is an autosomal recessive disorder
caused by an HFE genetic mutation and is characterized by increased intestinal iron absorption, tissue iron
deposition, and ultimately multisystem end-organ damage. HH is most common in men and postmenopausal
women; premenopausal women are less severely affected due to menstrual blood (and iron) loss.
Excessive iron stores in HH can be demonstrated by an elevated transferrin saturation (ie, serum iron/total iron
binding capacity × 100). The diagnosis is confirmed by testing for HFE genetic mutations.

The treatment of choice is therapeutic phlebotomy, which helps keep ferritin levels low enough to avoid
complications. Patients with extreme hyperferritinemia (eg, serum ferritin >1000 ng/mL) must not wait for genetic
testing results before receiving urgent phlebotomy because they are at exceptionally high risk for immediate and
irreversible end-organ damage (eg, cardiomyopathy, cirrhosis, arthritis, diabetes).

(Choice A) Statins (eg, simvastatin) can cause hepatotoxicity with elevated liver transaminases, but serum ferritin
is not affected. Although this patient's statin may be suspended temporarily, either now or in the future if liver
enzymes do not normalize, phlebotomy must be initiated immediately.

(Choice B) Autoimmune hepatitis can be treated with prednisone. Although it can cause mild elevations in ferritin,
levels typically do not exceed 1000 ng/mL.

(Choice C) Penicillamine is used to treat Wilson disease. This inherited disorder can cause elevated liver
enzymes, but clinical manifestations are generally apparent at age <35, and most patients have neuropsychiatric
symptoms (eg, psychosis, tremors).

(Choice E) Patients with HH who are at low risk for end-organ damage (eg, ferritin <500 ng/mL, normal liver
function tests) may be closely followed with periodic laboratory testing; phlebotomy is initiated if laboratory values
worsen or the patient develops symptoms of end-organ damage.

(Choice F) Ursodeoxycholic acid is used to treat primary biliary cholangitis, which presents with fatigue and
pruritis. Mild elevations in transaminases may occur, but profound alkaline phosphatase elevations and direct
hyperbilirubinemia are more typical findings.

Educational objective:
Hereditary hemochromatosis is characterized by increased intestinal iron absorption, tissue iron deposition, and
multisystem end-organ damage (eg, hepatotoxicity). The diagnosis is confirmed by testing for HFE genetic
mutations, but patients with extreme hyperferritinemia (eg, serum ferritin >1000 ng/mL) should receive urgent
therapeutic phlebotomy without waiting for genetic testing results.
Reference
• Hereditary hemochromatosis: missed diagnosis or misdiagnosis?
Question #189

A 52-year-old woman comes to the office due to intense itching and fatigue. She is unable to specify when her
symptoms started as they developed gradually. Past medical history is significant for hypothyroidism and carpal
tunnel syndrome. Current medications include levothyroxine. The patient lives with her husband and 3 children.
She does not smoke and drinks wine on social occasions. Vital signs are normal. Cardiopulmonary examination
shows no abnormalities. The abdomen is soft with normal bowel sounds. Hepatomegaly is present. There is no
scleral icterus or jaundice, but bilateral xanthelasma and skin excoriations are evident. Laboratory results are as
follows:

Serum creatinine 1.0 mg/dL


Total cholesterol 503 mg/dL

Total bilirubin 1.5 mg/dL


Alkaline phosphatase 410 U/L
Aspartate aminotransferase (AST, SGOT) 42 U/L
Alanine aminotransferase (ALT, SGPT) 44 U/L

A right upper quadrant ultrasound shows a normal common bile duct. Which of the following is the most
appropriate next step in management of this patient?

A) Check anti-mitochondrial antibodies

B) Check anti-smooth muscle antibodies

C) Discontinue levothyroxine

D) Obtain MRI scan of the abdomen


E) Prescribe oral glucocorticoids
Explanation
Correct Answer:

A) Check anti-mitochondrial antibodies

This patient has several features of cholestasis (impaired biliary flow), including fatigue, pruritus, and elevated
alkaline phosphatase. A right upper quadrant ultrasound distinguishes intrahepatic (no biliary tract dilation) from
extrahepatic (biliary tract dilation; eg, due to gallstones) cholestasis. If ultrasound suggests intrahepatic cholestasis
(as with this patient), the next step is to obtain serum anti-mitochondrial antibody titers, which have high
sensitivity and specificity for primary biliary cholangitis (PBC, previously termed primary biliary cirrhosis).

PBC is a chronic liver disease characterized by autoimmune destruction of the intrahepatic bile ducts with resulting
cholestasis. It presents most commonly in middle-aged women and is insidious in onset. As the disease
progresses, jaundice, hepatomegaly, steatorrhea, and portal hypertension may develop. Additional complications
can include severe hyperlipidemia (with xanthelasma) and metabolic bone disease. PBC is often associated with
other autoimmune disorders (eg, autoimmune thyroid disease).

(Choices B and E) Autoimmune hepatitis is associated with elevated titers of antinuclear antibodies and anti-
smooth muscle antibodies. It is characterized by fluctuating hepatocellular injury (ie, elevated transaminases)
rather than cholestasis. First-line treatment includes oral glucocorticoids.

(Choice C) A number of medications can cause intrahepatic cholestasis, including certain antibiotics (eg,
macrolides), anabolic steroids, and oral contraceptives. Levothyroxine is not associated with cholestasis.

(Choice D) Abdominal MRI is primarily used to evaluate abdominal masses or clarify nonspecific abnormalities on
other imaging tests. It is not needed in the evaluation of PBC.

Educational objective:
Primary biliary cholangitis (previously termed primary biliary cirrhosis) is a chronic liver disease characterized by
intrahepatic cholestasis due to autoimmune destruction of small bile ducts. It presents in middle-aged women with
fatigue, pruritus, hepatomegaly, and elevated alkaline phosphatase. The diagnosis is confirmed with serum anti-
mitochondrial antibody titers.

Reference
• Pathophysiology and diseases of the proximal pathways of the biliary system.
Question #190

A 54-year-old woman comes to the office due to fatigue, anorexia, and weight loss. Over the past 3 months, she
has lost 10 kg (22 lb) but has not changed her diet or activity level. The patient has a history of hypertension and
type 2 diabetes mellitus. She does not use tobacco, alcohol, or illicit drugs. Temperature is 36.9 C (98.4 F), blood
pressure is 120/80 mm Hg, pulse is 80/min, and respirations are 16/min. BMI is 31 kg/m2. Mild temporal wasting is
noted. The abdomen is soft, the flanks are full, and shifting dullness is present. There is hepatomegaly and a
single palpable liver nodule. Trace edema is present in the bilateral lower extremities. Laboratory results are as
follows:

Complete blood count


Hemoglobin 11.6 g/dL
Platelets 125,000/mm3
Leukocytes 9,000/mm3

Liver function studies


Albumin 2.5 g/dL
Total bilirubin 2.3 mg/dL
Alkaline phosphatase 370 U/L
Aspartate aminotransferase (SGOT) 134 U/L
Alanine aminotransferase (SGPT) 124 U/L

Blood, plasma, and serum


Alpha-fetoprotein, serum 12 ng/mL (normal: 0-20)

Which of the following is the most likely diagnosis?


A) Focal nodular hyperplasia

B) Hepatic adenoma

C) Hepatocellular carcinoma

D) Hydatid cyst

E) Pyogenic liver abscess


Explanation
Correct Answer:

C) Hepatocellular carcinoma

This patient's ascites (shifting abdominal dullness), hypoalbuminemia, mildly elevated liver function tests,
thrombocytopenia, and hyperbilirubinemia are consistent with underlying cirrhosis. Most cases are caused by
alcohol abuse, chronic viral hepatitis, or nonalcoholic fatty liver disease (as is likely in this patient with a history of
diabetes mellitus and obesity).

Patients with cirrhosis are often unaware of the diagnosis until they develop complications such as variceal bleeding
or hepatocellular carcinoma (HCC), a primary liver tumor. Risk of HCC in patients with cirrhosis is 1%-8% per
year. In these patients, HCC usually presents with decompensated liver failure (eg, ascites, jaundice,
hypoalbuminemia), weight loss, cachexia (eg, temporal wasting), possible hepatomegaly, and a palpable liver
nodule.

Alpha-fetoprotein (AFP), a glycoprotein produced by the fetal liver and yolk sac, is elevated in ~50% of cases;
therefore, AFP can serve as an important diagnostic clue (when elevated) but cannot be used to rule out the
diagnosis. Triple phase arterial contrast CT scan of the abdomen is diagnostic in most cases.

(Choice A) Focal nodular hyperplasia is a benign liver lesion that usually arises in young women. Although a
minority develop a palpable liver nodule, most are asymptomatic; the lesion does not usually rupture, undergo
malignant transformation, cause liver failure, or result in weight loss/cachexia.

(Choice B) Hepatic adenoma is a benign liver lesion common in young women on oral contraception. Some
lesions undergo malignant transformation (to HCC), which may result in cachexia, weight loss, and anorexia.
However, decompensated liver failure is uncommon (because the rest of the liver has preserved function).

(Choice D) Hydatid cyst is an Echinococcus tapeworm infection. Most cases are asymptomatic, but large hydatid
cysts can cause hepatomegaly, right upper quadrant pain, or nausea/vomiting. Decompensated liver failure and
cachexia/weight loss would be uncommon.
(Choice E) Pyogenic liver abscess is most common in the setting of biliary infection or bowel perforation. Although
weight loss, anorexia, fatigue, transaminitis, hepatomegaly, and hyperbilirubinemia commonly occur, almost all
patients have fever, abdominal pain, and leukocytosis.

Educational objective:
Patients with cirrhosis are often unaware of the condition until they develop complications. One major complication
is hepatocellular carcinoma, a primary liver tumor that often presents with decompensated liver failure, weight loss,
and a palpable liver lesion. Alpha-fetoprotein is elevated in ~50% of cases; therefore, it can be a useful diagnostic
clue but cannot be used to rule out the disease.
Question #191

A 68-year-old man comes to the office due to fatigue. It is difficult for him to get up in the morning and go to work.
He tires easily at work and goes to bed early due to feeling tired and sleepy. He also describes recent upper
abdominal pain that is constant and gnawing and interferes with his sleep. The patient's appetite is poor, and he
has lost 7 kg (15 lb) during the last month. He has no other medical problems. His medications include over-the-
counter famotidine. The patient works as a police officer. Two months ago, while investigating a serious crime
scene, he slipped on blood, fell, and hit his head. He is a former smoker with a 30-pack-year history. Physical
examination is significant for tenderness and fullness in the epigastrium. Which of the following is the most likely
diagnosis?

A) Chronic subdural hematoma

B) Duodenal ulcer

C) Major depressive episode

D) Pancreatic cancer

E) Post-traumatic stress disorder


Explanation
Correct Answer:

D) Pancreatic cancer

Pancreatic adenocarcinoma

• Smoking
• Hereditary pancreatitis
Risk factors
• Nonhereditary chronic pancreatitis
• Obesity & lack of physical activity

• Systemic symptoms (eg, weight loss, anorexia) (>85%)


• Abdominal pain/back pain (80%)
Clinical • Jaundice (56%)
presentation • Recent-onset atypical diabetes mellitus
• Unexplained migratory superficial thrombophlebitis
• Hepatomegaly & ascites with metastasis

• Cholestasis (↑ alkaline phosphatase & direct bilirubin)


Laboratory • ↑ CA-19-9 (not as a screening test)
studies
• Abdominal ultrasonography (if jaundiced) or CT scan (if no jaundice)

This patient has epigastric pain/tenderness and weight loss in the setting of nonspecific systemic symptoms and a
significant smoking history. This combination strongly suggests a malignancy affecting the upper gastrointestinal
tract or associated solid organs, such as the liver, gallbladder, or pancreas. Among the choices listed, pancreatic
adenocarcinoma is the most likely diagnosis.

Adenocarcinoma of the pancreas is the fourth leading cause of cancer-related death in the United States; peak
incidence is in the late seventh (for men) and eighth (for women) decade. Smoking is an important risk factor.
Pancreatic cancer has a very high mortality rate as it is frequently diagnosed at relatively late stages (early-stage
disease usually causes only mild, nonspecific symptoms). Symptoms of pancreatic cancer vary and depend
primarily on the location of the tumor.

Most (60%-70%) pancreatic cancers originate in the head of the pancreas. These cancers are more likely to
present with jaundice (common bile duct obstruction) and steatorrhea (inability to secrete fat-digesting enzymes or
blockage in the main pancreatic duct). However, jaundice may appear at a later stage if the tumor arises from the
tail or body of the pancreas. Most patients describe epigastric abdominal pain that is usually insidious, gnawing,
and worse at night. However, some patients may not have pain and simply present with painless jaundice.
Systemic symptoms such as weight loss and fatigue are common. Another classic association is migratory
thrombophlebitis (Trousseau syndrome).

Diagnosis is established with either ultrasound (less expensive) in patients with jaundice (head tumors) or CT scan
(highly sensitive) in patients without jaundice (body and tail tumors).

(Choice A) A chronic subdural hematoma from trauma would be more likely in an elderly person and typically
presents with headache, personality changes, seizures, confusion, or hemiparesis. Abdominal pain and tenderness
with anorexia and weight loss are not expected. Although neurogenic gastric mucosal injury (Cushing ulcer) can be
associated with traumatic brain injury, this condition is characterized by acute onset, usually with prominent gastric
bleeding and obvious neurologic impairment.

(Choice B) Duodenal ulcers can also cause burning epigastric pain. However, the pain is typically periodic and
relieved by food, as it is generally a result of acid secretion in the absence of a food buffer. Anorexia and weight
loss are less common. Antisecretory therapy with proton pump inhibitors or H2 receptor antagonists generally
provides some relief. This patient describes constant pain accompanied by anorexia, weight loss, and systemic
symptoms, red flags that should raise suspicion for a malignancy.
(Choice C) A major depressive episode could be responsible for fatigue, anorexia, and weight loss, but abdominal
pain and tenderness would not be typical. Even if a primary mood disorder is suspected, localized pain with weight
loss usually warrants evaluation for occult malignancy.

(Choice E) Post-traumatic stress disorder (PTSD) consists of persistent reexperiencing of a previous traumatic
event with recurrent attacks of anxiety, hyperarousal, nightmares, and/or flashbacks. This patient may have
experienced a stressful event during his criminal investigation, but he does not describe recurrent distressing
memories or nightmares related to the event. In addition, PTSD disturbs social and occupational functioning but
does not typically cause anorexia, weight loss, or signs of organic disease such as abdominal pain and tenderness.

Educational objective:
Pancreatic cancer classically presents insidiously with a combination of constant and gnawing epigastric pain that is
frequently worse at night, anorexia with weight loss, and jaundice due to extrahepatic biliary obstruction. A peptic
duodenal ulcer typically causes periodic epigastric pain relieved by meals.

Reference
• Pancreatic cancer: diagnosis and management.
Question #192

A 32-year-old woman comes to the emergency department with her boyfriend for evaluation of diarrhea for the past
3 weeks. The patient works as a schoolteacher and says, "There was a stomach bug going around recently, but no
one else has had symptoms for this long." She reports voluminous and watery bowel movements that occur 10-12
times a day. She feels fatigued and weak and states that her sleep has been disrupted a few times due to nocturnal
bowel movements. The patient has no abdominal pain, melena, hematochezia, vomiting, or unexplained weight
changes. Her boyfriend reports that she had a similar episode of diarrhea 2 months ago; laboratory evaluation and
colonoscopy showed no abnormalities, and the patient's symptoms resolved a few days later. Temperature is 37 C
(98.6 F), blood pressure is 120/80 mm Hg, and pulse is 84/min. There are no orthostatic changes. Mucous
membranes are moist, and capillary refill is normal. Abdominal examination shows mild tenderness to palpation in
all 4 quadrants, with no rebound tenderness, guarding, or rigidity. Complete blood count, serum chemistry, liver
function tests, and urinalysis are normal. HIV testing is negative. Fecal occult blood is negative. Which of the
following is the most appropriate next step in management of this patient?

A) Assay for tissue transglutaminase IgA antibodies

B) CT scan of the abdomen

C) Lactose hydrogen breath test

D) Measurement of stool osmolality

E) Norovirus stool PCR testing


Explanation
Correct Answer:

D) Measurement of stool osmolality

This patient has normal vital signs, physical examination, laboratory results, and colonoscopy findings but reports
persistent, voluminous diarrhea, raising suspicion for factitious diarrhea. Factitious disorder is characterized by
intentional falsification of illness in the absence of external reward (eg, financial compensation, disability benefits).
Patients with factitious diarrhea purposely cause large, voluminous stools, most commonly by improper use of
laxatives; however, they can also create the appearance of diarrhea by adding fluid to the stool. Therefore,
helpful tests in evaluating factitious diarrhea include:

• Stool osmolality: Stool osmolality is in equilibrium with plasma osmolality and typically remains constant
(eg, 290 mOsm/kg) in organic gastrointestinal disease. Hypoosmolality suggests addition of water or other
dilute fluid; hyperosmolality suggests addition of a concentrated fluid (eg, urine).
• Stool electrolytes: Elevated stool magnesium or phosphate levels suggest overuse of saline osmotic (ie,
magnesium- or phosphate-containing) laxatives.
• Stool osmotic gap: Osmotic laxatives (eg, lactulose, polyethylene glycol) cause a high osmotic gap
diarrhea, whereas senna and bisacodyl produce a low osmotic gap secretory diarrhea.

(Choice A) Tissue transglutaminase IgA antibodies are seen in celiac disease, which is a common cause of
chronic diarrhea. However, patients typically have additional findings, such as weight loss and signs of
malabsorption (eg, iron deficiency anemia, vitamin D deficiency with hypocalcemia).

(Choice B) An abdominal CT scan may identify colitis (eg, colonic wall thickening, fat stranding) in patients with
diarrhea; however, it is less sensitive than a colonoscopy, which was unremarkable in this patient.

(Choice C) The lactose hydrogen breath test can diagnose lactose intolerance, which causes postprandial
abdominal discomfort and diarrhea. However, the bowel movements would be triggered by meals and do not
typically awaken patients from sleep.
(Choice E) Norovirus can cause chronic diarrhea in immunocompromised patients (eg, HIV, solid-organ
transplant), but symptoms are typically brief (eg, 1-2 days) and self-limited in immunocompetent individuals.

Educational objective:
Diarrhea that is not explained after extensive evaluation suggests possible factitious diarrhea. Helpful studies
include stool osmolality (hypoosmolality suggests addition of water or dilute fluid; hyperosmolality suggests addition
of concentrated fluid [eg, urine]), stool electrolytes (elevated magnesium or phosphate suggests use of saline
osmotic laxatives), and stool osmotic gap (osmotic laxatives cause a high osmotic gap diarrhea).

Reference
• Features associated with laxative abuse in individuals with eating disorders.

• Brief report: factitious diarrhea detected by measurement of stool osmolality.

• Evaluating the patient with diarrhea: a case-based approach.


Question #193

A 42-year-old man with a history of Crohn disease comes to the office for follow-up. He had a partial ileal resection
due to a stricture and also had multiple surgeries to treat an enterocutaneous fistula. The patient has received
parenteral nutrition for the past several weeks and recently restarted oral feeding. He reports nonbloody diarrhea
but no fever or abdominal pain. The patient says that food does not taste the same as before. On examination, he
has patchy alopecia and a pustular, crusting skin rash with scaling and erythema around the mouth and on the
extremities. No abdominal tenderness is present and bowel sounds are normal. The jugular venous pulse is
normal and there is no lower extremity edema. Which of the following is most likely to improve this patient's current
condition?

A) Elemental copper

B) Gluten avoidance

C) Niacin therapy

D) Selenium supplement

E) Zinc supplement
Explanation
Correct Answer:

E) Zinc supplement

Clinical manifestations of trace mineral


deficiencies

• Impaired glucose control in


Chromium
diabetes

• Brittle hair
• Skin depigmentation
• Neurologic dysfunction (eg, ataxia,
Copper
peripheral neuropathy)
• Anemia
• Osteoporosis

Iron • Microcytic anemia

• Thyroid dysfunction
Selenium • Cardiomyopathy
• Immune dysfunction
• Alopecia
• Pustular skin rash (perioral
region & extremities)
Zinc • Hypogonadism
• Impaired wound healing
• Impaired taste
• Immune dysfunction

This patient most likely has zinc deficiency. Zinc is a trace mineral that is obtained from meat, nuts, and fortified
cereal in the diet. It is an essential component of numerous enzymes in the body and plays an important role in
gene transcription and cell division. Clinical manifestations of zinc deficiency in adults include hypogonadism,
impaired wound healing, impaired taste, and immune dysfunction. Characteristic physical examination findings
include alopecia as well as a skin rash consisting of erythematous pustules around body orifices (eg, mouth) and
on the extremities.

Zinc is absorbed mostly in the duodenum and jejunum, and deficiency can be due to malabsorption (eg, Crohn
disease, celiac disease), bowel resection, gastric bypass, or poor nutritional intake. In addition, patients dependent
on parenteral nutrition (PN) are at risk for trace mineral deficiency (eg, zinc, selenium, copper, chromium) due to
inadequate supplementation of parenteral intake. Comorbid diarrhea and malabsorption further increase the risk of
trace mineral deficiency in patients on PN due to excessive gastrointestinal losses.

(Choice A) Clinical manifestations of copper deficiency include fragile hair, skin depigmentation, neurologic
dysfunction that can mimic vitamin B12 deficiency, and sideroblastic anemia. Impaired taste and a pustular skin rash
would be unusual.

(Choice B) Gluten avoidance is used to treat celiac disease, which can lead to malabsorption and deficiency of
zinc and other vitamins and minerals. Celiac disease is associated with dermatitis herpetiformis, an autoimmune-
mediated vesicular skin rash that typically appears on the extremities. Perioral involvement would be unusual. In
addition, celiac disease is less likely in this patient with alternate explanations for zinc deficiency (eg, Crohn
disease, recent PN).
(Choice C) Niacin (vitamin B3) deficiency (rare in developed countries) causes pellagra, which is characterized by
dermatitis, diarrhea, and dementia. The rash associated with pellagra is hyperpigmented, occurs symmetrically in
sun-exposed areas, and is not pustular.

(Choice D) Selenium deficiency can lead to thyroid dysfunction as well as cardiomyopathy. The absence of heart
failure signs (eg, jugular venous distension, lower extremity edema) in this patient makes selenium deficiency less
likely.

Educational objective:
Risk factors for trace mineral deficiency include malabsorption, bowel resection, poor nutritional intake, and
dependence on parenteral nutrition. Clinical manifestations of zinc deficiency include hypogonadism, impaired
taste, impaired wound healing, alopecia, and skin rash with perioral involvement.

Reference
• Zinc: physiology, deficiency, and parenteral nutrition.
Question #194

A 36-year-old woman comes to the office due to intermittent abdominal pain. She has had 5 episodes of pain over
the past several weeks. The pain is dull, is located in the right upper quadrant and epigastrium, and often radiates
to her back. The pain occurs only after eating and lasts 30-60 minutes. It is not relieved by antacids. The pain is
occasionally associated with nausea and vomiting, but the patient has no hematemesis, melena, or weight loss.
She does not use nonsteroidal anti-inflammatory drugs, tobacco, or alcohol. Her father died of pancreatic cancer at
age 64. Temperature is 37.5 C (99.5 F), blood pressure is 142/89 mm Hg, pulse is 67/min, and respirations are 12/
min. BMI is 32 kg/m2. The abdomen is tender to palpation in the right upper quadrant and epigastrium but is
nondistended and without rebound or guarding. Laboratory results are as follows:

Complete blood count


Hemoglobin 14.4 g/dL
Platelets 280,000/mm3
Leukocytes 7500/mm3

Liver function studies


Total bilirubin 0.2 mg/dL
Alkaline phosphatase 73 U/L
Aspartate aminotransferase (SGOT) 12 U/L
Alanine aminotransferase (SGPT) 24 U/L

Which of the following is the best next step in management of this patient?

A) Abdominal ultrasound

B) Abdominal x-ray
C) CT scan of the abdomen

D) Helicobacter pylori stool antigen test

E) HIDA scan

F) Upper gastrointestinal endoscopy


Explanation
Correct Answer:

A) Abdominal ultrasound

This patient's presentation suggests biliary colic, which occurs when the gallbladder contracts against a gallstone
(or sludge) that is temporarily blocking the cystic duct. Risk factors for gallstone disease include female sex, age
>40, hypertriglyceridemia, and obesity. Patients classically develop dull postprandial right upper quadrant
(RUQ) or epigastric pain that worsens over an hour and subsides with gallbladder relaxation. Nausea, vomiting,
and diaphoresis may occur; however, vital signs, white blood cell count, and liver function tests (LFTs) should
remain normal. The best test to evaluate biliary colic is a RUQ abdominal ultrasound, which can confirm the
presence of gallstones.

Acute cholecystitis, a common complication of gallstone disease, can occur with complete obstruction of the cystic
duct. In contrast to biliary colic, acute cholecystitis typically presents with severe, unremitting RUQ pain and signs
of systemic illness (eg, fever, tachycardia, leukocytosis). LFTs typically remain normal, and elevations should raise
suspicion for biliary obstruction (eg, choledocholithiasis, cholangitis). Characteristic ultrasound findings of
cholecystitis include gallbladder wall thickening, edema, and pericholecystic fluid.

(Choices B and C) Most gallstones are radiolucent and may not be detected on abdominal x-ray or CT scan. CT
scan can be helpful in ruling out pancreatic cancer, which can also cause epigastric pain. However, despite this
patient's family history, pancreatic cancer is exceedingly rare in young individuals and would typically present with
weight loss and progressive pain.

(Choice D) Testing for Helicobacter pylori is ideal in patients with dyspepsia. However, although postprandial
epigastric pain is common, it is usually described as burning (as opposed to dull) and relieved by antacids.
Importantly, dyspepsia is not associated with vomiting.

(Choice E) A HIDA scan is used to diagnose acute cholecystitis in patients with an equivocal abdominal
ultrasound. It is not indicated as first-line imaging. Additionally, this patient does not have signs of cholecystitis (eg,
fever, leukocytosis).

(Choice F) Upper gastrointestinal endoscopy is useful for ruling out peptic ulcers, but the postprandial abdominal
pain is often described as burning and relieved by antacids. Major risk factors are nonsteroidal anti-inflammatory
drug use and H pylori infections.

Educational objective:
Biliary colic occurs when the gallbladder contracts against a gallstone that temporarily blocks the cystic duct.
Classic symptoms include episodic postprandial right upper quadrant or epigastric pain, nausea, and vomiting;
however, vital signs, white blood cell count, and liver function studies remain normal. The diagnosis is confirmed
with an abdominal ultrasound demonstrating the presence of gallstones.
Question #195

A 40-year-old man comes to the office for an annual checkup. The patient has gained weight over the past year,
which he attributes to a diet consisting primarily of processed food and red meat. He is otherwise asymptomatic
and feels well. Medical history is significant for celiac disease, type 2 diabetes mellitus, hyperlipidemia, and a
Wilms tumor treated 35 years ago with chemoradiation and surgical resection. Medications include insulin, aspirin,
and simvastatin. There is no family history of colon cancer. Vital signs are normal. BMI is 48 kg/m2. Examination
shows acanthosis nigricans but is otherwise normal. Which aspect of this patient's history warrants colon cancer
screening earlier than what is typically recommended for the general population?

A) Aspirin use

B) Celiac disease

C) Diabetes mellitus

D) History of abdominal radiation therapy

E) Obesity

F) Red meat consumption


Explanation
Correct Answer:

D) History of abdominal radiation therapy

Risk factors for colon cancer

Lifestyle • Frequent consumption of red/processed meat


factors • Tobacco, alcohol use

• Personal/family history of adenomatous polyps or colon cancer


• Inherited colon cancer syndromes (eg, familial adenomatous
Medical/
polyposis, Lynch syndrome)
family
• Ulcerative colitis
history
• Diabetes/obesity
• Prior abdominopelvic radiation

Protective • High-fiber diet


factors • Aspirin/NSAID use

NSAID = nonsteroidal anti-inflammatory drug.

Abdominopelvic radiation is used to treat several childhood cancers, including lymphoma, neuroblastoma,
sarcoma, and Wilms tumor (nephroblastoma). However, exposure to abdominopelvic radiation is associated with a
significant risk of developing colon adenocarcinoma in adulthood, with an estimated risk >4 times higher than that
of the general population. The mechanism is variable and multifactorial, with contributing effects including
increased mutations in colonic mucosa, chronic inflammatory changes, and underlying genetic susceptibility.

For individuals with a history of abdominopelvic radiation treatment, colon cancer screening should commence at an
earlier age (eg, age 30-40) than that recommended for average-risk patients (ie, age 45). Colonoscopy is
preferred, although acceptable alternatives include fecal occult blood testing and fecal DNA testing.

Patients with a family history of adenomatous polyps or colon cancer should also undergo screening at an earlier
age. Other risk factors that warrant early screening include ulcerative colitis and certain hereditary colorectal
cancer syndromes (eg, hereditary nonpolyposis colorectal cancer [Lynch syndrome], familial adenomatous
polyposis).

(Choice A) Although the mechanism is incompletely understood, long-term use of aspirin and nonsteroidal anti-
inflammatory drugs is associated with a decreased (not increased) risk for colon cancer.

(Choice B) Even though celiac disease (ie, gluten-sensitive enteropathy) predisposes patients to small bowel
adenocarcinoma and lymphoma, the overall risk is low; regular screening of the small bowel for these cancers is not
required. Celiac disease does not predispose patients to colorectal cancer and therefore would not influence the
timing of colon cancer screening.

(Choices C, E, and F) Diabetes, obesity, and daily consumption of red meat can all increase a person's chance of
developing colorectal cancer, but the overall risk is small compared to that of prior abdominopelvic radiation
therapy. These factors do not necessitate earlier colon cancer screening.

Educational objective:
Childhood cancer survivors treated with abdominopelvic radiation are at increased risk for developing colorectal
adenocarcinoma. Also at higher risk are patients with inherited colon cancer syndromes and patients with a history
of inflammatory bowel disease. All should start colon cancer screening at an earlier age than the general
population.

Reference
• Secondary gastrointestinal cancer in childhood cancer survivors: a cohort study.

• Predictors of colorectal cancer surveillance among survivors of childhood cancer treated with radiation: a
report from the Childhood Cancer Survivor Study.
Question #196

A 76-year-old man comes to the office due to constipation. He has infrequent bowel movements associated with
straining during defecation. In addition, the patient has intermittent fecal leakage and hematochezia but no melena
or unexpected weight loss. Medical history is notable for prostate cancer treated with external beam radiation and
brachytherapy 18 months ago. Follow-up CT scans and prostate-specific antigen levels show no evidence of
cancer recurrence. Temperature is 37.5 C (99.5 F), blood pressure is 132/80 mm Hg, pulse is 80/min, and
respirations are 12/min. Conjunctival pallor is present. Cardiopulmonary examination is unremarkable. The
abdomen is soft and nontender. Colonoscopy reveals rectal pallor with areas of mucosal hemorrhage and
telangiectasias; the rest of the colon is unremarkable. Which of the following is most likely contributing to this
patient's symptoms?

A) Chronic inflammation of the mucosa and submucosa

B) Electrolyte abnormalities

C) Parasympathetic nervous system dysregulation

D) Pelvic floor dyssynergia

E) Progressive rectal fibrosis


Explanation
Correct Answer:

E) Progressive rectal fibrosis

Radiation proctitis

Acute radiation proctitis Chronic radiation proctitis

Postradiation onset • ≤8 weeks • >3 months to years

• Obliterative endarteritis & chronic mucosal


Pathogenesis • Direct mucosal damage ischemia
• Submucosal fibrosis

• Diarrhea, mucus discharge,


• Severe bleeding
Manifestations tenesmus
• ± Strictures with constipation & rectal pain
• Minimal bleeding

Endoscopic • Severe erythema • Multiple telangiectasias


appearance • Edema, ulcerations • Mucosal pallor & friability
• Antidiarrheals (eg, loperamide) • Endoscopic thermal coagulation
Management
• Butyrate enemas • Sucralfate or glucocorticoid enemas

This patient has chronic radiation proctitis (RP), presenting with constipation, fecal incontinence, and
hematochezia. RP is due to injury to the rectal epithelium from pelvic radiation therapy, most commonly in the
treatment for prostate, colon, ovarian, or cervical cancer. RP can have both acute and chronic manifestations:

• Acute RP typically occurs in the first 8 weeks of therapy; damage to the rectal epithelium causes diarrhea
and mucoid discharge.

• Chronic RP, which develops >3 months after radiation exposure, is characterized by obliterative endarteritis
and submucosal fibrosis.

Progressive fibrosis in chronic RP can impair rectal compliance and cause anorectal stricture formation. Typical
symptoms include constipation, fecal incontinence, and fecal impaction. Chronic tissue hypoxia results in
neovascularization, which can lead to hemorrhage and secondary anemia (eg, conjunctival pallor). Colonoscopy is
diagnostic and typically reveals mucosal pallor, telangiectasias, mucosal hemorrhages, and tissue friability.

(Choice A) Ulcerative colitis causes chronic inflammation of the colonic mucosa and submucosa, leading to
tenesmus, hematochezia, and abdominal pain but rarely constipation. Colonoscopy findings include erythema and
superficial ulcerations; mucosal pallor and telangiectasias are more consistent with RP.

(Choice B) Hypercalcemia can occur in advanced prostate cancer due to bone metastasis or the secretion of
parathyroid hormone–related protein but would be unlikely in this patient with no evidence of active malignancy.
Although hypercalcemia can cause constipation, it would not explain this patient's hematochezia or colonoscopy
findings.

(Choices C and D) Parasympathetic dysregulation can cause constipation and fecal incontinence in patients with
underlying neurologic disorders (eg, Parkinson disease). Dyssynergic defecation occurs when muscles of the
pelvic floor fail to relax during bowel movements, preventing stool expulsion from the rectal vault. However, in these
conditions, colonoscopy findings are typically normal, and hematochezia would be unexpected.
Educational objective:
Chronic radiation proctitis is characterized by obliterative endarteritis and submucosal fibrosis, which leads to
anorectal stricture formation, reduced rectal compliance, constipation, and fecal incontinence. Chronic tissue
hypoxia results in neovascularization and telangiectasia formation, which can lead to hemorrhage.

Reference
• Radiation proctopathy.
Question #197

A 36-year-old man comes to the office for a routine preventive visit. The patient feels well, but for the past 2 days
he has occasionally noticed blood on the tissue paper after defecation. Otherwise, his bowel movements have
remained normal in consistency and frequency. He has no melena, vomiting, abdominal pain, or unexpected weight
changes. Medical history is unremarkable, and the patient has no family history of malignancy. Vital signs are
normal. The abdomen is soft and nontender with normal bowel sounds and no hepatosplenomegaly or masses.
Digital rectal examination is unremarkable except for traces of red blood visible on the glove. Which of the following
is the most appropriate next step in management?

A) Anoscopy

B) Colonoscopy

C) CT colonography

D) Fecal DNA testing

E) Reassurance and periodic follow-up


Explanation
Correct Answer:

A) Anoscopy

This patient has minimal bright red blood per rectum (BRBPR). This symptom, characterized by small amounts
of bright red blood on the toilet paper after wiping or a few drops of blood in the toilet bowl after defecation, most
often is due to benign disorders such as hemorrhoids or anal fissures. However, more serious disorders (eg,
proctitis, ulcers, colorectal polyps, cancer) are possible.

Clinical factors associated with an increased risk of serious disease in patients with BRBPR include a large amount
of blood mixed within stool, systemic symptoms (eg, fever, weight loss), and changes in bowel habits or stool
caliber; these factors generally warrant direct visualization of the entire colon with colonoscopy (Choice B).
Colonoscopy should also be considered for those age ≥45, for whom colorectal cancer screening via colonoscopy is
appropriate regardless of symptoms.

However, for younger patients, initial testing with less invasive methods to visualize the anal canal (eg, anoscopy)
generally should be performed first. If a benign cause of bleeding (eg, hemorrhoid) is confirmed, conservative
management (eg, stool softeners) and interval follow-up is appropriate (Choice E).

(Choices C and D) Fecal DNA testing and CT colonography are appropriate screening options for colorectal
cancer beginning at age 45 for asymptomatic patients at average risk. They have low sensitivity for small lesions
and nonmalignant causes (eg, hemorrhoids, inflammatory bowel disease) of BRBPR and would not be indicated for
this patient.

Educational objective:
Minimal rectal bleeding usually is due to hemorrhoids or other benign conditions. In patients with no clinical
features suggesting a serious cause (eg, blood mixed with stool, fever, weight loss, change in bowel habits or stool
caliber) and no other indication for colonoscopy (eg, age ≥45), initial evaluation with anoscopy is appropriate.
Question #198

An 84-year-old woman comes to the office due to 2 months of bright red bleeding from the rectum. The patient also
has intermittent crampy abdominal pain and a 6-kg (13.2-lb) unintentional weight loss. She reports no fevers,
tenesmus, chest pain, palpitations, or vomiting. Medical history is significant for obesity and chronic kidney
disease. Temperature is 37.6 C (99.7 F), blood pressure is 130/80 mm Hg, and pulse is 70/min. The patient
appears cachectic. There is no palpable lymphadenopathy. Cardiac auscultation reveals normal rate and rhythm
and no heart murmurs. Lung sounds are normal. The abdomen is nontender and nondistended. Which of the
following is the most likely diagnosis?

A) Angiodysplasia

B) Cecal adenocarcinoma

C) Diverticular bleeding

D) Internal hemorrhoids

E) Sigmoid colon adenocarcinoma


Explanation
Correct Answer:

E) Sigmoid colon adenocarcinoma

This patient has clinical features suspicious for colon cancer. Adenocarcinoma of the colon often presents with
fatigue, abdominal pain, and unexpected weight loss, but more specific symptoms are variable and can correlate
to the site of the tumor:

• Right-sided colon tumors: In the right colon, the lumen is wide, and stool is mostly liquid; obstructive
symptoms are uncommon unless the tumor is very large and far advanced. Visible bleeding (ie,
hematochezia) can occur but is atypical because blood is diluted by stool before it passes the anal verge.
Therefore, adenocarcinoma in the cecum and ascending colon typically presents with occult bleeding and
iron-deficiency anemia (Choice B).

• Left-sided colon tumors: In the descending and sigmoid colon, tumors may obstruct the flow of stool,
leading to crampy or colicky pain and/or a change in bowel habits (eg, constipation). Due to the proximity
of these tumors to the anal verge, visible red/maroon hematochezia is common.

• Rectal tumors: Bleeding is typically visible as hematochezia or frank red blood. Other symptoms include
narrowed stools, tenesmus, and a sensation of a mass in the rectum.

(Choice A) Angiodysplasia is characterized by abnormal dilated and tortuous blood vessels in the intestines. It is
most common at age >60 and can present with painless hematochezia or occult bleeding with iron deficiency
anemia. This patient's crampy pain and unexpected weight loss are atypical for angiodysplasia.

(Choice C) Diverticular hemorrhage causes acute, self-limited hematochezia that generally lasts up to several days
(not months). It is usually painless and does not cause weight loss.

(Choice D) Internal hemorrhoids present with anal pruritus and bright red rectal bleeding, typically on defecation.
Crampy pain and weight loss would not be seen.
Educational objective:
Adenocarcinoma in the left side of the colon may obstruct the flow of stool, leading to altered bowel habits; visible
hematochezia is common. In contrast, in the right side of the colon, obstructive symptoms are uncommon, and any
associated bleeding can be diluted by stool and is less likely to be visible; therefore, right-sided colon cancer often
presents with occult bleeding and iron-deficiency anemia.

Reference
• Clinical assessment to determine the risk of bowel cancer using symptoms, age, mass and iron deficiency
anaemia (SAMI).
Question #199

A 42-year-old man who recently emigrated from Northern Africa comes to the clinic due to a 1-month history of
abdominal pain and watery diarrhea. He has also had a skin rash for the last 2 months that gets worse with sun
exposure. In addition, the patient has felt depressed recently and has loss of appetite with mild weight loss.
Medical history is notable for latent tuberculosis, for which he is currently taking daily isoniazid. The patient takes
no other medications and does not use tobacco, alcohol, or recreational drugs. He is vegetarian, and his diet has
consisted mostly of corn and other cereal grains. On examination, there is a pigmented scaly skin rash in the malar
distribution of his face, on his neck, and on the back of his hands. He also has mild, diffuse abdominal tenderness.
Neurologic examination, including distal sensation and gait, is normal. Liver function tests are normal. Which of the
following is the most likely diagnosis?

A) Acute intermittent porphyria

B) Isoniazid hypersensitivity

C) Pellagra

D) Seborrheic dermatitis

E) Systemic lupus erythematosus

F) Ulcerative colitis
Explanation
Correct Answer:

C) Pellagra
Pellagra ("rough skin" in Italian vernacular) is due to niacin (vitamin B3) deficiency and is characterized by the "3
Ds": dermatitis, diarrhea, and dementia:

• Dermatitis is primarily on sun-exposed areas of the body and is characterized by rough, hyperpigmented,
scaly skin.
• Diarrhea is often associated with abdominal pain, nausea, and loss of appetite.
• Dementia is due to neuronal degeneration in the brain and spinal cord and can lead to memory loss,
affective symptoms (eg, depressed mood in this patient), and psychosis.

Niacin is present in a broad variety of foods and can be synthesized endogenously from tryptophan. In developing
countries, niacin deficiency is seen in populations that subsist primarily on corn products (niacin in corn occurs in a
bound, unabsorbable form). In developed countries, it is primarily seen in patients with impaired nutritional intake
(eg, heavy alcohol use, chronic illness). Pellagra can also be seen occasionally in those with carcinoid syndrome
(due to depletion of tryptophan) or Hartnup disease (congenital disorder of tryptophan absorption). Prolonged
isoniazid therapy without concomitant use of pyridoxine (vitamin B6), which is needed for niacin synthesis, can
interfere with metabolism of tryptophan and occasionally lead to pellagra.

(Choice A) Acute intermittent porphyria (AIP) causes abdominal pain, vomiting, and diarrhea, often with neurologic
symptoms (eg, agitation, paresthesias, confusion). Although AIP may be triggered by isoniazid, the symptoms are
episodic rather than chronic, chronic transaminase elevation is common, and it is more frequent in women than in
men; in addition, unlike porphyrea cutanea tarda, AIP is not typically associated with a rash.

(Choice B) Isoniazid hypersensitivity can cause hives (maculopapular rash) with pruritus, fever, and hepatitis. This
patient's chronic scaly rash is not consistent with a hypersensitivity rash. The most common side effects of
isoniazid therapy are hepatitis and peripheral neuropathy.

(Choice D) Seborrheic dermatitis is characterized by erythematous, scaly plaques affecting the scalp, face, chest,
and intertriginous areas. It can be associated with dementia (eg, Parkinson disease). However, the hands are not
typically affected, and it does not cause gastrointestinal symptoms.

(Choice E) Systemic lupus erythematosus causes a photosensitive rash in a malar distribution. CNS
manifestations (eg, psychosis) can occur. Although isoniazid can cause drug-induced lupus (eg, positive
antihistone antibodies), a rash is uncommon and would make this diagnosis less likely. With lupus, gastrointestinal
involvement typically causes impaired motility, and diarrhea is not typical. In addition, this patient's diet makes
pellagra more likely.

(Choice F) Ulcerative colitis (UC) causes bloody diarrhea as opposed to the watery diarrhea seen with pellagra.
Although UC can have extraintestinal manifestations, associated skin findings include erythema nodosum and
pyoderma gangrenosum, not scaly dermatitis.

Educational objective:
Pellagra is due to niacin deficiency and is characterized by the "3 Ds": dermatitis, diarrhea, and dementia.
Prolonged isoniazid therapy can interfere with niacin metabolism and occasionally cause pellagra.
Question #200

A 61-year-old woman comes to the office due to 3 months of constipation and pain in the left lower abdomen. The
pain is crampy, continuous, and nonradiating and is not affected by bowel movements or meals. The patient reports
hard, pelletlike stools and early satiety but no vomiting, hematochezia, melena, or weight changes. The patient
does not have a history of chronic constipation. She is postmenopausal and has not had a period in >5 years.
Medical history is significant for hypertension. Family history is unremarkable, and the patient does not use tobacco
or alcohol. Temperature is 37.5 C (99.5 F), blood pressure is 145/87 mm Hg, pulse is 80/min, and respirations are
12/min. No scleral icterus or palpable lymphadenopathy is present. Cardiopulmonary examination is
unremarkable. The abdomen is mildly distended and tender to palpation in the left lower quadrant without rebound
or guarding. Rectal examination is unremarkable and negative for fecal occult blood. Colonoscopy results are
unremarkable. Which of the following is the best next step in management?

A) Abdominal x-ray

B) Barium enema

C) Biofeedback therapy

D) Fiber supplementation

E) Pelvic floor muscle exercises

F) Pelvic ultrasound
Explanation
Correct Answer:

F) Pelvic ultrasound

Epithelial ovarian carcinoma

• Acute: shortness of breath, obstipation/constipation with vomiting, abdominal distension


Clinical
• Subacute: pelvic/abdominal pain, bloating, early satiety
presentation
• Asymptomatic adnexal mass

Laboratory
• ↑ CA-125
findings

• Solid mass
Ultrasound
• Thick septations
findings
• Ascites

Management • Exploratory laparotomy

Constipation is common and is often benign, but it can be associated with more concerning etiologies (eg,
malignancy). Onset of constipation at an older age (eg, age ≥50), particularly when accompanied by early satiety
or pain unrelated to bowel movements (as in this patient), is an alarm feature that should raise suspicion for
malignancy. Other alarm features (not seen in this patient) include weight loss, hematochezia, and a family history
of malignancy or inflammatory bowel disease. Colon cancer should be ruled out with colonoscopy, as occurred in
this case. Given the unremarkable colonoscopy, this patient should be evaluated for ovarian cancer.

Ovarian cancer, the second most common gynecologic malignancy in the United States, often presents
subacutely with subtle, nonspecific symptoms (eg, lower abdominal pain, bloating, early satiety, constipation).
Examination often demonstrates abdominal distension or an adnexal mass but may remain normal until late in the
disease course. Because of the vague presentation and high mortality associated with ovarian cancer, pelvic
examination and imaging should be considered in older women (eg, postmenopausal, age ≥50) with new-onset
abdominal symptoms, particularly in the absence of any other obvious cause (eg, colon cancer). Pelvic
ultrasound is inexpensive, rapidly performed, and has high specificity for the detection of malignant sonographic
features (eg, thick septations, solid components).

(Choice A) Abdominal x-ray can be used to assess stool burden in patients with constipation; however, it cannot
detect ovarian cancer and would likely delay the diagnosis.

(Choice B) Barium enema is sometimes used to evaluate diverticular disease or polyps. However, this patient has
normal colonoscopy results and should undergo pelvic imaging to rule out ovarian cancer.

(Choices C, D, and E) Biofeedback therapy and pelvic floor muscle exercises are used to treat dyssynergic
defecation, a common cause of functional constipation in the elderly due to the inability to relax pelvic floor muscles
during bowel movements. Fiber supplementation is often used to treat constipation-predominant irritable bowel
syndrome (IBS). Although both dyssynergic defecation and IBS could be considered if ovarian cancer is ruled out,
initiating therapy before performing pelvic imaging may delay the diagnosis, potentially increasing morbidity and
mortality.

Educational objective:
Alarm features for constipation include acute onset at an older age, weight loss, and hematochezia. Ovarian cancer
has nonspecific symptoms (eg, lower abdominal pain, bloating, early satiety, constipation). Therefore, older women
(eg, postmenopausal, age ≥50) with new-onset abdominal pain and/or concerning gastrointestinal
symptoms, ovarian cancer should be considered.

Reference
• Symptoms associated with ovarian cancer.
Question #201

A 45-year-old Asian-American woman comes to the physician due to bloating, flatulence, abdominal cramps and
explosive watery diarrhea. These symptoms occur after ingesting dairy products. She has not had any weight
loss. She has not had bone pain or easy bruising. Physical examination shows abdominal distention and
borborygmi. You decide to investigate the patient further. Which of the following test results is most likely to be
observed?

A) Positive urine test for reducing substances

B) Decreased stool osmotic gap

C) Positive hydrogen breath test

D) Alkaline stool pH

E) Positive acid steatocrit test


Explanation
Correct Answer:

C) Positive hydrogen breath test

Lactase is a brush border enzyme that hydrolyses lactose. Its concentration declines steadily as one ages into
adulthood, especially in people of non-European ancestry. When there is an inability to absorb lactose found in milk
and dairy products, the condition is called lactose intolerance. Acquired deficiency is most commonly seen in
Asian (90%) and African populations. Patients typically manifest with osmotic diarrhea, abdominal cramps,
bloating and flatulence after ingestion of such products.

Previously, the lactose tolerance test was used to aid in the diagnosis of patients. This test is based on
measurement of the blood glucose level after oral lactose administration. The lactose tolerance test is cumbersome
and time consuming. The lactose hydrogen breath test, a noninvasive test, can confirm the diagnosis if unclear.
A positive hydrogen breath test is characterized by a rise in the measured breath hydrogen level after the ingestion
of lactose, thus indicating bacterial carbohydrate metabolism (ie, impaired malabsorption).

(Choice A) Urine test for reducing substances is positive in patients with glucosuria, galactosuria, and so on.
Those are not characteristic features of lactose intolerance, which is characterized by a positive stool (rather than
urine) test for reducing substances due to carbohydrate malabsorption.

(Choice B) The diarrhea secondary to lactase deficiency has a high osmotic gap (eg, >125 mOsm/kg), due to the
unmetabolized lactose and organic acids. The osmotic gap is calculated as 290 - [2 (stool Na + stool K)] and is >75
mOsm/kg in all forms of osmotic diarrhea.

(Choice D) The stool pH is acidic in lactase deficiency due to the fermentation products.

(Choice E) Acid steatocrit is a test for fat malabsorption, not carbohydrate malabsorption.

Educational objective:
Lactose intolerance is characterized by a positive hydrogen breath test, positive stool test for reducing substances,
low stool pH and increased stool osmotic gap. There is no steatorrhea.
Question #202

A 42-year-old man comes to the office to discuss abnormal blood work results that were obtained at a local health
fair. The patient currently feels well and reports no symptoms. Temperature is 37.6 C (99.7 F), blood pressure is
125/72 mm Hg, pulse is 82/min, and respirations are 13/min. Examination shows no scleral icterus. Heart and lung
sounds are normal; skin and abdominal examination are normal. Laboratory results are as follows:

Liver function studies


Albumin 3.8 g/dL
Total bilirubin 0.7 mg/dL
Alkaline phosphatase 521 U/L
Aspartate aminotransferase (SGOT) 21 U/L
Alanine aminotransferase (SGPT) 31 U/L
Gamma-glutamyl transpeptidase 390 U/L

Right upper quadrant ultrasound is normal. The patient undergoes magnetic resonance cholangiopancreatography,
which reveals luminal irregularities with mild focal dilatations within both intrahepatic and extrahepatic biliary ducts.
Which of the following additional testing should be obtained from this patient?

A) Colonoscopy

B) Echocardiogram

C) Ocular slit-lamp examination

D) Pulmonary function tests


E) Serum ferritin levels
Explanation
Correct Answer:

A) Colonoscopy

Primary sclerosing cholangitis

• Asymptomatic
• Fatigue & pruritus
Clinical features
• Associated with IBD, particularly UC (~90% of
patients)

• Cholestatic liver injury (↑↑ alkaline


phosphatase, ↑ bilirubin)
Laboratory/imaging • Multifocal stricturing/dilation of intrahepatic
&/or extrahepatic bile ducts on
cholangiography

• Fibrous obliteration of small bile ducts, with


Liver biopsy concentric replacement by connective tissue
in onion-skin pattern

Complications • Biliary stricture


• Cholangitis or cholelithiasis
• Cholangiocarcinoma, colon cancer, biliary
cancer
• Cholestasis (eg, ↓ fat-soluble vitamins,
osteoporosis)

IBD = inflammatory bowel disease; UC = ulcerative colitis.

This patient has laboratory and magnetic resonance cholangiopancreatography (MRCP) findings consistent with
primary sclerosing cholangitis (PSC), a chronic disorder that causes progressive biliary fibrosis and stricturing.
Many patients are asymptomatic at the time of diagnosis and have only elevations in alkaline phosphatase and
gamma-glutamyl transpeptidase (ie, cholestatic pattern). As the disease progresses, obstruction of the bile ducts
leads to jaundice, pruritus, and direct hyperbilirubinemia. Ultrasound may remain normal but can show bile duct
wall thickening and dilations. However, MRCP findings of multifocal intrahepatic and extrahepatic biliary
strictures and segmental dilations are considered diagnostic.

Up to 90% of patients with PSC also have inflammatory bowel disease (IBD), with ulcerative colitis being more
common than Crohn disease. To rule out underlying IBD, a colonoscopy should be performed in all patients with a
new diagnosis of PSC. Patients with both PSC and IBD should have annual colonoscopies because they are at a
markedly elevated risk for colon cancer (~5 times higher than those with IBD alone).

(Choices B and E) Hereditary hemochromatosis also causes chronic liver disease and cirrhosis; however, patients
demonstrate a hepatocellular pattern of liver injury, skin hyperpigmentation and diabetes mellitus is typical, and
biliary stricturing is unexpected. Although echocardiograms (to assess for heart failure) and serum ferritin levels are
indicated in these patients, they are not required in PSC.

(Choice C) Wilson disease, which occurs from impaired intracellular copper transportation, can also cause liver
disease; however, patients have a hepatocellular pattern of liver injury, most demonstrate neuropsychiatric
symptoms, and biliary stricturing is unexpected. Slit-lamp demonstrates Kayser-Fleischer rings.
(Choice D) Alpha-1 antitrypsin deficiency presents with early onset emphysema (diagnosed with pulmonary
function studies) and hepatitis. Although cholestasis can occur, segmental dilations and strictures are more
consistent with PSC.

Educational objective:
Primary sclerosing cholangitis is often diagnosed in asymptomatic individuals with high alkaline phosphatase and
gamma-glutamyl transpeptidase levels (cholestatic pattern). Magnetic resonance cholangiopancreatography
findings of multifocal intrahepatic and extrahepatic biliary strictures with segmental dilations are diagnostic. A
colonoscopy is recommended at the time of diagnosis because many patients also have inflammatory bowel
disease.

Reference
• Radiologic manifestations of sclerosing cholangitis with emphasis on MR cholangiopancreatography.

• ACG clinical guideline: primary sclerosing cholangitis.


Question #203

A 54-year-old man comes to the office after a year of progressive fatigue and frequent loose stools. He describes
voluminous, foul-smelling stools that float and are difficult to flush. The patient has lost about 9 kg (20 lbs) in the
past 6 months. He was hospitalized on multiple occasions several years ago for epigastric pain radiating to the
back, nausea, and vomiting. Lately, the patient has had intermittent episodes of similar pain lasting for 15-30
minutes after meals. He has had no hematemesis, hematochezia, or melena. The patient stopped drinking alcohol
several years ago and does not use tobacco or recreational drugs. Temperature is 37 C (98.6 F), blood pressure is
118/80 mm Hg, and pulse is 78/min. BMI is 19.5 kg/m2. Abdominal examination shows epigastric tenderness on
deep palpation. Bowel sounds are normal. Which of the following is most likely to improve this patient's
symptoms?

A) Antimicrobial therapy

B) Cholecystectomy and stone removal

C) Gluten-free diet

D) Helicobacter pylori eradication

E) Mesenteric angioplasty

F) Pancreatic enzyme supplementation

G) Restriction of dairy products


Explanation
Correct Answer:

F) Pancreatic enzyme supplementation

Common causes of steatorrhea

• Chronic pancreatitis due to alcohol abuse, cystic fibrosis, or autoimmune/


Pancreatic insufficiency hereditary pancreatitis
• Pancreatic cancer

• Small-bowel Crohn disease


• Bacterial overgrowth
Bile salt–related • Primary biliary cirrhosis
• Primary sclerosing cholangitis
• Surgical resection of ileum (at least 60-100 cm)

• Celiac disease
Impaired intestinal surface
• AIDS enteropathy
epithelium
• Giardiasis

• Whipple disease
Other rare causes
• Zollinger-Ellison syndrome
• Medication induced

This patient's presentation of weight loss and loose, greasy, malodorous stools that float in the toilet and are
difficult to flush is consistent with fat malabsorption (steatorrhea), which can be caused by various conditions.

This patient's steatorrhea is most likely due to chronic pancreatitis related to long-standing alcohol abuse. His
previous hospitalizations for vomiting and epigastric pain that radiates to the back raise suspicion for recurrent
episodes of acute alcoholic pancreatitis. Repeated pancreatic injury causes progressive inflammation and fibrosis
and results in impaired endocrine and exocrine (loss of digestive enzymes) functions. Patients with chronic
pancreatitis may initially be asymptomatic, but disease progression ultimately leads to malabsorption, steatorrhea,
and postprandial epigastric pain that can become continuous. Symptomatic improvement can be achieved with
alcohol cessation and pancreatic enzyme supplementation.

(Choice A) Antimicrobial therapy (eg, rifaximin) should be considered in patients with small intestinal bacterial
overgrowth syndrome, which can present with abdominal pain/bloating and symptoms of malabsorption. However,
this condition is typically associated with anatomical abnormalities (eg, surgical blind loop) or motility disorders (eg,
scleroderma) of the gastrointestinal tract.

(Choice B) Cholecystectomy and stone removal should be considered in patients with biliary colic, which typically
presents with postprandial right upper quadrant/epigastric pain associated with nausea/vomiting. Steatorrhea and
malabsorption are not characteristic.

(Choice C) A gluten-free diet is appropriate for individuals with celiac disease, which can present with diarrhea,
malabsorption, and nutritional deficiencies. However, this patient's history of alcohol use and epigastric abdominal
pain that radiates to the back is more consistent with chronic pancreatitis.

(Choice D) Helicobacter pylori infection is a common cause of peptic ulcer disease, which can present with
postprandial epigastric pain, nausea/vomiting, and weight loss due to food aversion. However, peptic ulcer disease
does not usually cause steatorrhea.

(Choice E) Mesenteric angioplasty should be considered in patients with chronic mesenteric ischemia, which can
present with postprandial abdominal pain (intestinal angina) and weight loss due to food aversion. However,
steatorrhea is not characteristic.

(Choice G) Restriction of dairy products may be considered in patients with lactose intolerance, which typically
presents with abdominal pain/cramping, bloating, flatulence, and frothy/watery diarrhea after dairy product
consumption. However, steatorrhea and malabsorption are not characteristic.

Educational objective:
Steatorrhea occurs due to fat malabsorption and generally presents with voluminous, greasy, and foul-smelling
stools that are difficult to flush. It is commonly caused by pancreatic exocrine insufficiency (loss of digestive
enzymes) in patients with chronic alcoholic pancreatitis. Alcohol cessation and pancreatic enzyme supplementation
can improve symptoms in such patients.

Reference
• Chronic pancreatitis.

• Diagnosis and treatment of pancreatic exocrine insufficiency.


Question #204

A 23-year-old woman comes to the office with her husband because of nausea, vomiting, and a 7-kg (15.4-lb)
weight loss over the last 3 months. According to her husband, she seems "stressed out" about eating food
and spends time researching her diet. The patient says, "Just after eating a couple bites, I feel like I've had
Thanksgiving dinner. I get relief after throwing up, but I'm worried about my weight loss." She has had no changes
in bowel movements. The patient's only medical condition is type 1 diabetes mellitus, diagnosed when she was age
5. She uses bolus-prandial insulin. BMI is 20 kg/m2. Temperature is 37 C (98.6 F), blood pressure is 104/70 mm
Hg, pulse is 80/min, and respirations are 14/min. On examination, the abdomen is soft with active bowel sounds;
there is no distension. The remainder of the examination demonstrates no abnormalities. Complete blood count,
serum electrolytes, and serum TSH are normal. Her hemoglobin A1c is 8.3%. Which of the following is the most
appropriate next step in management of this patient?

A) ACTH stimulation test

B) Cognitive-behavioral therapy

C) Gastric-emptying study

D) Trial of a selective serotonin reuptake inhibitor


Explanation
Correct Answer:

C) Gastric-emptying study

Gastroparesis

• Diabetes mellitus (autonomic neuropathy)


• Medications (eg, opioids, anticholinergic drugs)
Causes • Traumatic/postsurgical injury (ie, vagus nerve injury)
• Neurologic (eg, multiple sclerosis, spinal cord injury)
• Idiopathic/postviral

• Nausea & vomiting, epigastric abdominal pain


• Early satiety, bloating, weight loss
Clinical features
• Labile glucose (diabetes mellitus)
• Epigastric distension & succussion splash

• Exclude obstruction: upper endoscopy ± CT/MR enterography


Diagnosis
• Assess motility: nuclear gastric-emptying study

• Frequent small meals (low fat, soluble fiber only)


Treatment
• Promotility drugs (eg, metoclopramide, erythromycin)
• Gastric electrical stimulation &/or jejunal feeding tube (refractory symptoms)

This patient's nausea, postprandial fullness, fear of eating, and significant weight loss could be explained by an
eating disorder. Anorexia would seem unlikely because she has a normal BMI (18.5 - 24.9 kg/m2) and she is
concerned about her weight loss (anorexia involves an intense fear of weight gain). Bulimia nervosa involves binge
eating (not present in this patient) followed by compensatory measures (eg, purging) and somewhat fits with her
presentation. However, psychiatric disorders are usually a diagnosis of exclusion that is only made after underlying
medical disorders have been ruled out.

Long-standing diabetes mellitus (especially type 1) is often complicated by autonomic neuropathy; therefore,
this patient is at increased risk for gastroparesis. Gastroparesis typically results from disruption of neurologic
signaling within the vagus nerve and the myenteric plexus. Common signs include postprandial emesis, early
satiety, weight loss, and abdominal pain, and many patients develop food aversion (eg, restricting food intake)
because eating exacerbates the symptoms. The diagnosis is best evaluated with a gastric-emptying scan and an
investigation for mechanical obstruction (eg, upper endoscopy ± CT/MR enterography). Management options
include dietary modification (eg, low fat, small and frequent meals) and gastric prokinetic agents (eg,
metoclopramide, erythromycin).

(Choice A) ACTH stimulation testing can help diagnose primary adrenal insufficiency, a condition that causes
vomiting and weight loss. Patients may have decreased appetite but would not be fearful of eating. In addition,
fatigue (90%), hyperpigmentation (40-70%), and electrolyte abnormalities such as hyponatremia (70-80%) and
hyperkalemia (30-40%) would be expected.

(Choice B) Cognitive-behavioral therapy (CBT) is commonly used in the management of eating disorders.
However, an eating disorder is less likely in this patient, and CBT should not be initiated until a medical cause of her
symptoms has been appropriately investigated.

(Choice D) Selective serotonin reuptake inhibitors can be used to treat major depressive disorder, generalized
anxiety disorder, and some eating disorders. This patient's signs of psychosocial distress are most likely secondary
to gastroparesis, which requires diagnosis and treatment.
Educational objective:
Long-standing diabetes mellitus (especially type 1) is a common cause of gastroparesis. Patients typically have
postprandial vomiting, early satiety, weight loss, and abdominal pain, and many develop food aversion. The
diagnosis is best evaluated with a gastric-emptying scan and an investigation for mechanical obstruction (eg, upper
endoscopy ± CT/MR enterography).

Reference
• Gastroparesis.
Question #205

A 46-year-old man comes to the office due to a sensation of right upper quadrant fullness. The patient has had
obesity since early adolescence. Medical history includes type 2 diabetes mellitus and hypertension. Current
medications include metformin, candesartan, and hydrochlorothiazide. The patient does not use tobacco, alcohol,
or recreational drugs. His father has type 2 diabetes mellitus. Temperature is 36.7 C (98 F), blood pressure is 138/
90 mm Hg, pulse is 72/min, and respirations are 16/min. BMI is 36 kg/m2. Physical examination is notable for
acanthosis nigricans over the neck creases and hepatomegaly. Laboratory results are as follows:

Albumin 4.0 mg/dL


Total bilirubin 1.0 mg/dL
Direct bilirubin 0.3 mg/dL
Alkaline phosphatase 100 U/L
Aspartate aminotransferase (SGOT) 122 U/L
Alanine aminotransferase (SGPT) 131 U/L
Blood glucose (fasting) 168 mg/dL

Viral hepatitis serologies, antimitochondrial antibodies, antinuclear antibodies, serum ceruloplasmin, and transferrin
saturation are normal. Which of the following is the most likely pathophysiologic mechanism responsible for this
patient's increased liver enzymes?

A) Decreased peripheral lipolysis

B) Glucagon overproduction

C) Increased glucocorticoid production


D) Increased hepatic glycogen synthesis

E) Insulin resistance
Explanation
Correct Answer:

E) Insulin resistance

Nonalcoholic fatty liver disease

• Hepatic steatosis on imaging or biopsy


Definition
• Exclusion of other etiologies (eg, alcohol, hepatitis C, glucocorticoids)

• Mostly asymptomatic
• Metabolic syndrome
Clinical features
• AST/ALT ratio <1
• Hyperechoic texture on ultrasound examination

• Weight loss (eg, diet modification, exercise)


Treatment
• Consider bariatric surgery if BMI ≥35

Prognosis • Hepatic fibrosis associated with increased risk for cirrhosis & liver-related death

ALT = alanine aminotransferase; AST = aspartate aminotransferase.

This patient's presentation (hepatocellular injury in the absence of viral hepatitis, heavy alcohol use, and abnormal
iron level) is most consistent with nonalcoholic fatty liver disease (NAFLD). Affected patients typically have
coexisting metabolic syndrome (eg, central obesity, diabetes mellitus, hyperlipidemia, hypertension). NAFLD can
range from bland steatosis to inflammation and hepatocyte injury (steatohepatitis) to fibrosis and cirrhosis.

NAFLD is likely due to toxic lipid accumulation in the liver. The pathogenesis appears to involve insulin
resistance, which increases peripheral lipolysis (in adipocytes) and triglyceride synthesis (Choice A), leading to
increased hepatic accumulation of fatty acids and triglycerides. On histologic examination, hepatocytes show
macrovesicular fat deposition with peripherally displaced nuclei. In addition, hepatic free fatty acid (FFA)
accumulation:

• increases free radical and proinflammatory cytokine generation, which can lead to inflammation and
hepatocyte injury (steatohepatitis).

• worsens insulin resistance by impairing insulin-dependent glucose uptake and increasing hepatic
gluconeogenesis.

Insulin resistance is also associated with impaired FFA oxidation (in the liver) and clearance (from the liver, due to
decreased VLDL production).

(Choice B) Some patients with type 2 diabetes have increased glucagon production. Although glucagon increases
gluconeogenesis and glycogenolysis, it does not have a role in hepatic fat accumulation.

(Choice C) Cushing syndrome leads to increased endogenous glucocorticoid production and can contribute to
insulin resistance and NAFLD, but it would be unlikely in this patient with near life-long obesity.

(Choice D) Hepatic glycogen synthesis is usually decreased in type 2 diabetes mellitus and does not have a role in
hepatic steatosis.

Educational objective:
Insulin resistance is thought to have a major role in nonalcoholic fatty liver disease, which frequently coexists with
metabolic syndrome. Insulin resistance increases peripheral lipolysis in adipocytes, leading to increased hepatic
accumulation of free fatty acids. Hepatic fatty acid accumulation increases free radical and proinflammatory
cytokine generation, resulting in tissue damage (eg, steatohepatitis).
Reference
• Epidemiology, pathogenesis, diagnosis and emerging treatment of nonalcoholic fatty liver disease.
Question #206

A 32-year-old woman comes to the office due to intermittent abdominal pain and nonbloody diarrhea for the past 3-4
months. She describes the pain as crampy and located in the mid-abdomen and right lower quadrant. The patient
thinks she has lost some weight during this period. She modified her diet several times in an attempt to decrease
the symptoms, believing that heavy meals exacerbate the pain and diarrhea. The patient underwent appendectomy
with abscess debridement one year ago. She has no other medical issues and takes no medication other than oral
contraceptives. She has not traveled outside the United States and does not use tobacco, alcohol, or recreational
drugs. Temperature is 36.7 C (98 F), blood pressure is 120/70 mm Hg, pulse is 85/min, and respirations are 14/
min. A few shallow ulcers are present in her mouth. Abdominal examination shows mild tenderness in the right
lower quadrant without rebound. Laboratory results are as follows:

Hemoglobin 10.2 g/dL


Leukocytes 14,500/mm3
Platelets 530,000/mm3
Erythrocyte sedimentation rate 48 mm/hr

Which of the following is the most likely diagnosis for this patient?

A) Celiac disease

B) Crohn disease

C) Giardia infection

D) Irritable bowel syndrome

E) Lactose intolerance
F) Ulcerative colitis

G) Yersinia infection
Explanation
Correct Answer:

B) Crohn disease

Crohn disease

• Gastrointestinal: abdominal pain, nonbloody diarrhea, oral ulcers, malabsorption, weight loss,
fistula/abscess formation, perianal disease
Clinical • Extraintestinal:
findings ◦ Musculoskeletal: arthritis
◦ Ophthalmic: uveitis, scleritis, episcleritis
◦ Skin: erythema nodosum, pyoderma gangrenosum (less common)

• Leukocytosis, iron deficiency, ↑ inflammatory markers


• B12 deficiency if terminal ileum involved
• Endoscopy:
Diagnosis ◦ Focal ulcerations next to normal mucosa (eg, cobblestoning)
◦ Skip lesions, rectal sparing

• Radiography: strictures, bowel wall thickening

Treatment • Corticosteroids, azathioprine, anti-TNF therapies (eg, infliximab)


TNF = tumor necrosis factor.

This patient with chronic abdominal pain, diarrhea, weight loss, and evidence of inflammation (elevated
erythrocyte sedimentation rate [ESR]) likely has Crohn disease. Other frequent gastrointestinal manifestations of
Crohn disease include microscopic bleeding, fistula and stricture formation, abdominal abscesses, and
malabsorption. It can involve any part of the gastrointestinal tract from mouth (eg, aphthous ulcers) to anus.
Common extraintestinal symptoms include arthritis, uveitis, scleritis, and erythema nodosum. Laboratory findings
are similar to other chronic inflammatory diseases and can include leukocytosis, anemia, reactive thrombocytosis,
and elevated inflammatory markers (eg, ESR). Diagnosis is confirmed with endoscopic or radiographic studies,
and treatment involves immunosuppressive therapy.

(Choice A) Although celiac disease can also present with chronic diarrhea, abdominal pain, weight loss, and
anemia from malabsorption, elevated inflammatory markers are uncommon, as are oral manifestations.

(Choice C) Chronic giardiasis commonly presents with loose, greasy stools, significant weight loss, and abdominal
cramping; however, oral involvement, anemia, and elevated inflammatory markers are not typical. Moreover, this
patient does not have clear risk factors for Giardia infection (eg, international travel, hiking).

(Choices D and E) Irritable bowel syndrome and lactose intolerance are not commonly associated with weight
loss, anemia, leukocytosis, or elevated inflammatory markers. Such findings should prompt evaluation for other
causes of gastrointestinal complaints.

(Choice F) Ulcerative colitis more frequently presents as bloody (rather than nonbloody) diarrhea with tenesmus
and incontinence, and does not involve the oral mucosa.

(Choice G) Infectious colitis due to a number of bacteria (including Yersinia), parasites, or amoebae can closely
mimic Crohn disease, especially if it presents with ileitis. However, infectious colitis is a more acute process and
does not involve the oral mucosa.

Educational objective:
Crohn disease is characterized by chronic abdominal pain, diarrhea, weight loss, and evidence of inflammation (eg,
anemia, elevated inflammatory markers). It can involve any part of the gastrointestinal tract from mouth to anus.

Reference
• Diagnosis and management of Crohn disease.
Question #207

A 27-year-old man comes to the office due to episodic abdominal pain. The pain is concentrated in the epigastrium
and is gnawing in quality. It wakes him up during the night and is promptly relieved by a glass of water and a piece
of bread. The patient has no associated vomiting or diarrhea but has experienced occasional "dark stools." He has
no significant past medical history and takes no medications. Family history is significant for biliary disease in his
mother. He smokes a pack of cigarettes daily and consumes a can of beer most days. Vital signs are within normal
limits. Physical examination shows mild epigastric discomfort on deep palpation. After the diagnosis is confirmed,
which of the following is the most appropriate intervention to provide relief of this patient's symptoms?

A) 4-week course of omeprazole

B) Abstinence from alcohol

C) Antibiotics and pantoprazole

D) Cholecystectomy

E) Naproxen as needed

F) Selective vagotomy

G) Smoking cessation
Explanation
Correct Answer:

C) Antibiotics and pantoprazole

This patient with epigastric pain and intermittent melena most likely has a duodenal ulcer (DU). The pain of DU
is often worse on an empty stomach (possibly due to unopposed gastric acid emptying into the duodenum) and
improves with food (due to alkaline fluid secretion into the duodenum). By contrast, the pain of gastric ulcers is
often worse after eating (due to increased acid secretion).

The majority of DUs are caused by either Helicobacter pylori infection or nonsteroidal anti-inflammatory drugs
(NSAIDs). Malignant ulceration should be considered with gastric ulcers but would be very unlikely in this young
patient with DU. As this patient has no history of NSAID use, H pylori infection is the most likely etiology and can be
confirmed with endoscopic biopsy or urea breath test. Management of DU due to H pylori requires the following:

• Antisecretory therapy, preferably a proton pump inhibitor (PPI) (eg, omeprazole, pantoprazole), and
• Antibiotic eradication (eg, amoxicillin plus clarithromycin)

(Choice A) Compared to antisecretory therapy with antibiotic eradication, antisecretory therapy alone leads to
reduced healing rates and a higher risk of recurrent ulcer in patients with H pylori-associated DU. Antisecretory
therapy without antibiotics would be recommended for DU attributable to causes other than H pylori.

(Choices B and G) Heavy alcohol intake can contribute to hemorrhagic gastritis, but light alcohol intake likely does
not affect the course of DU. Smoking appears to increase the risk of peptic ulcer in patients infected with H pylori
but does not increase relapse following H pylori eradication.

(Choice D) Laparoscopic cholecystectomy is recommended for biliary colic (intermittent, severe right upper
quadrant pain with nausea) or acute cholecystitis. This patient's epigastric pain (relieved by food) and melena are
unlikely to be due to symptomatic gallbladder disease.

(Choice E) NSAIDs can contribute to the risk of DU. Therefore, they are contraindicated during the acute
management of DU and should be used with caution following confirmed ulcer resolution.

(Choice F) Selective vagotomy decreases gastric acid production by removing vagal input to the stomach. With
antibiotic regimens for H pylori and the advent of PPIs, selective vagotomy is used only in refractory cases.

Educational objective:
Duodenal ulcer (DU) is characterized by epigastric pain that improves with eating. The majority of DUs are caused
by Helicobacter pylori or nonsteroidal anti-inflammatory drugs. Management of H pylori-associated ulcer requires
acid suppression and antibiotic eradication therapy.

Reference
• Helicobacter pylori and gastric or duodenal ulcer.

• A prospective study of alcohol, smoking, caffeine, and the risk of duodenal ulcer in men.

• Does smoking predispose to peptic ulcer relapse after eradication of Helicobacter pylori?
Question #208

A 54-year-old woman comes to the emergency department due to bright red blood per rectum. She reports passing
3 large, bloody stools over the past 2 days, which have been associated with mild abdominal cramps. The patient
has no melena, fever, vomiting, or unexpected weight changes. She underwent a radical hysterectomy and
adjuvant radiation therapy for cervical cancer over 12 months ago. The patient has no other medical conditions and
takes no medication. Temperature is 37.5 C (99.5 F), blood pressure is 140/80 mm Hg, pulse is 80/min, and
respirations are 12/min. Cardiopulmonary examination is unremarkable. The abdomen is nontender and
nondistended. Extremities are warm and well perfused. Laboratory results are as follows:

Complete blood count


Hemoglobin 9.8 g/dL
Platelets 325,000/mm3
Leukocytes 6,200/mm3

Serum chemistry
Blood urea nitrogen 17 mg/dL
Creatinine 0.8 mg/dL

A colonoscopy is performed and shows mucosal pallor, friability, and multiple telangiectasias, all of which are
confined within the rectum. Scattered diverticula are present but limited to the descending and transverse colon.
Which of the following is the most likely diagnosis?

A) Angiodysplasia

B) Clostridioides difficile infection

C) Crohn disease
D) Diverticular colitis

E) Ischemic colitis

F) Radiation proctitis
Explanation
Correct Answer:

F) Radiation proctitis

Radiation proctitis

Acute radiation proctitis Chronic radiation proctitis

Postradiation onset • ≤8 weeks • >3 months to years

• Obliterative endarteritis & chronic mucosal


Pathogenesis • Direct mucosal damage ischemia
• Submucosal fibrosis

• Diarrhea, mucus discharge,


• Severe bleeding
Manifestations tenesmus
• ± Strictures with constipation & rectal pain
• Minimal bleeding

Endoscopic • Severe erythema • Multiple telangiectasias


appearance • Edema, ulcerations • Mucosal pallor & friability
• Antidiarrheals (eg, loperamide) • Endoscopic thermal coagulation
Management
• Butyrate enemas • Sucralfate or glucocorticoid enemas

This patient with a history of cervical cancer treated with radiation therapy has hematochezia. In association with
the characteristic rectal findings on colonoscopy, this presentation suggests chronic radiation proctitis (RP). RP
occurs due to damage to the rectal epithelium associated with pelvic radiation therapy. Risk is related to intensity,
duration, and type of radiation exposure.

Manifestations of RP vary based on mechanism of mucosal injury. Acute RP occurs due to direct mucosal damage
and presents ≤8 weeks postradiation with diarrhea, tenesmus, and mucus discharge. Chronic RP, which can arise
months to years after radiation, occurs from submucosal fibrosis and obliterative endarteritis. The resulting tissue
hypoxia promotes neovascularization and telangiectasia formation, which are prone to bleeding and often result in
significant hematochezia and blood loss anemia; stricturing may occur.

Colonoscopy in chronic RP demonstrates mucosal pallor, friability, and multiple telangiectasias confined to the
rectum. Management includes local thermal coagulation or sucralfate and glucocorticoid enemas.

(Choice A) Angiodysplasia is an arteriovenous malformation that usually occurs in elderly patients as well as those
with chronic kidney disease and severe aortic stenosis. Although it morphologically resembles telangiectasia, the
presence of colonic mucosal pallor and friability and the history of pelvic radiation is more consistent with RP.

(Choice B) Clostridioides difficile typically presents with watery diarrhea, fever, and leukocytosis in individuals with
recent antibiotic exposure. White pseudomembranes are expected on colonoscopy.

(Choice C) Crohn disease can cause hematochezia but typically presents with abdominal pain, diarrhea, and
perianal lesions. Colonoscopy typically demonstrates deep colonic ulcerations and skip lesions. In addition,
although isolated rectal inflammation can occur in ulcerative colitis, it is extremely uncommon in Crohn disease.

(Choice D) Diverticular colitis refers to mucosal inflammation immediately adjacent to diverticula. This patient's
inflammatory changes are confined to the rectum, separate from the diverticula located in the descending and
transverse colons.
(Choice E) Ischemic colitis results from colonic hypoperfusion and typically affects intestinal watershed areas (eg,
rectosigmoid junction, splenic flexure). Although hematochezia and abdominal cramps are common, ischemic
colitis usually occurs in the setting of profound hypotension (eg, septic shock, acute heart failure).

Educational objective:
Radiation proctitis (RP) is caused by mucosal damage associated with pelvic radiation therapy. Acute RP presents
≤8 weeks postradiation with diarrhea, tenesmus, and mucus discharge, whereas chronic RP occurs months to years
after radiation, resulting in hematochezia, anemia, and possibly strictures. Colonoscopy demonstrates mucosal
pallor, friability, and telangiectasias confined to the rectum.

Reference
• Chronic haemorrhagic radiation proctitis: a review.
Question #209

A 53-year-old woman comes to the office due to progressive exertional dyspnea over the past several weeks. The
patient's abdominal girth has also increased during this time, which has been associated with discomfort and early
satiety. Medical history is significant for bacterial endocarditis due to intravenous drug use. Temperature is 36.7 C
(98 F), blood pressure is 120/70 mm Hg, pulse is 85/min, and respirations are 18/min. Physical examination shows
no jugular venous distension or hepatojugular reflux. Chest is clear to auscultation. The abdomen is distended and
nontender. Both shifting dullness and a fluid wave are present. Hepatosplenomegaly is present. There is trace
bilateral lower extremity edema. Which of the following best explains this patient's presentation?

A) Chronic liver disease

B) Constrictive pericarditis

C) Nephrotic syndrome

D) Ovarian cancer

E) Right heart failure


Explanation
Correct Answer:

A) Chronic liver disease

This patient with progressive dyspnea has ascites (ie, abdominal distension, shifting dullness, fluid wave).
Although ascites can occur from a variety of diseases, over 80% of cases are due to cirrhosis. In the United States,
cirrhosis is most commonly due to chronic alcohol misuse, nonalcoholic steatohepatitis, and hepatitis C. Given this
patient's hepatosplenomegaly and history of intravenous drug use, hepatitis C cirrhosis is the most likely diagnosis.

Cirrhosis promotes the formation of ascites due to portal hypertension, which causes hemodynamic change (ie,
splanchnic vasodilation) that leads to salt and water retention. Patients with ascites have increased abdominal
girth and discomfort, as well as weight gain, dyspnea (from increased abdominal pressure), and early satiety.
Physical examination shows shifting dullness and a fluid wave. In the setting of cirrhosis, stigmata of chronic
liver disease (eg, spider angiomas, palmar erythema, asterixis, jaundice), hepatomegaly (cirrhotic livers can be
small, normal sized, or enlarged), and splenomegaly (typically reflecting congestion from portal hypertension) can
also be seen. Laboratory abnormalities in cirrhosis include abnormal liver enzymes, thrombocytopenia, and
evidence of synthetic dysfunction, such as hypoalbuminemia (which can cause trace bilateral edema) and elevated
international normalized ratio.

Abdominal ultrasound can confirm the presence of ascites, and a diagnostic paracentesis should be performed in all
patients to confirm the underlying etiology.

(Choices B and E) Both constrictive pericarditis and right heart failure can cause ascites, hepatomegaly (due to
congestive hepatopathy), and peripheral edema. However, jugular venous distension (JVD) is expected (eg,
positive hepatojugular reflex with right heart failure), and Kussmaul sign (ie, rise in JVD with inspiration) may be
present. In addition, constrictive pericarditis (which can be idiopathic/viral) usually presents with a pericardial knock
(accentuated heart sound before S3, seen in ~50% of cases) and pulsus paradoxus.

(Choice C) Nephrotic syndrome often presents with anasarca and ascites due to profound hypoalbuminemia.
However, patients typically have massive lower extremity edema (as opposed to this patient's trace edema); in
addition, hepatosplenomegaly would be atypical.

(Choice D) Metastatic ovarian cancer with peritoneal carcinomatosis can cause malignant ascites. However,
because it occurs in advanced cancer, patients usually have abdominal pain (from tumor invasion) and a history of
significant weight loss. Splenomegaly is unexpected.

Educational objective:
Ascites presents with increased abdominal girth and findings of shifting dullness and fluid wave. It is most
commonly caused by cirrhosis due to chronic alcohol misuse, nonalcoholic steatohepatitis, or hepatitis C. In the
setting of cirrhosis, stigmata of chronic liver disease (eg, spider angiomas, palmar erythema, asterixis, jaundice)
and hepatosplenomegaly are also common.

Reference
• Nonalcoholic steatohepatitis is the second leading etiology of liver disease among adults awaiting liver
transplantation in the United States.

• Ascites: diagnosis and management.

• Differential diagnosis of ascites.


Question #210

A 57-year-old man comes to the office due to several months of persistent tingling in both legs. The patient also
says he feels clumsy and often bumps into furniture. He has a history of hip osteoarthritis, but the pain is well
controlled with acetaminophen and occasional ibuprofen and does not limit his mobility. The patient also has a
history of gastroesophageal reflux disease and Barrett esophagus; he has used omeprazole for the past several
years and has had no recent heartburn or dysphagia. He does not use tobacco, alcohol, or illicit drugs. The patient
consumes a balanced diet and walks 1 or 2 miles every other day. Vital signs are within normal limits. Lower
extremity muscle strength is intact. Sensation to light touch and vibration are diminished in the feet bilaterally. The
remainder of the physical examination shows no abnormalities. Which of the following is the most appropriate next
step in evaluation of this patient's symptoms?

A) Serologic testing for HIV

B) Serum protein electrophoresis

C) Serum vitamin B12 level test

D) Urine analysis for heavy metals

E) Viral hepatitis serology


Explanation
Correct Answer:

C) Serum vitamin B12 level test


This patient's omeprazole use, lower extremity paresthesia, and diminished light touch/vibration sensation in the
feet raise strong suspicion for vitamin B12 deficiency. Because vitamin B12 is an essential cofactor for DNA
synthesis and myelin formation, deficiency usually manifests with hematologic (eg, megaloblastic anemia) and/or
neurologic abnormalities. Common neurologic findings include subacute combined degeneration of the dorsal
columns (eg, impaired proprioception/vibrioception) and the lateral corticospinal tracts (eg, positive Babinski
reflex), but patients often first develop symmetric lower extremity paresthesia due to myelinated peripheral nerve
damage.

Vitamin B12 is ingested bound to animal protein and must be liberated in the stomach by pepsin, which is activated
from pepsinogen by gastric acid. Therefore, patients on long-term proton pump inhibitor therapy (eg,
omeprazole) sometimes develop vitamin B12 deficiency due to achlorhydria. Older patients are particularly at risk.
The diagnosis is usually made with serum vitamin B12 level, but methylmalonic acid or homocysteine testing may
be necessary if this test is inconclusive.

(Choice A) HIV can cause a distal symmetric polyneuropathy that generally begins with tingling and numbness in
the bilateral feet. However, all sensory modalities are typically diminished (not just light touch and vibration),
reflexes are generally impaired, and neuropathic pain is usually prominent.

(Choice B) Neurologic manifestations (eg, peripheral neuropathy, paresthesias) can develop in multiple myeloma
(MM), which is diagnosed by serum protein electrophoresis. However, they are typically due to plasmacytoma
infiltration or paraneoplastic effects and are not usually present at initial diagnosis. MM is unlikely in this patient with
no other suggestive features (eg, bone pain, anemia). Most patients with neurologic manifestations of MM have
significant neuropathic pain and mixed sensory and motor abnormalities.

(Choice D) Urine heavy metal testing can diagnose lead poisoning, which is often associated with peripheral
neuropathy. However, peripheral motor neurons are primarily affected; therefore, most patients have motor
weakness (eg, wrist or ankle drop).

(Choice E) Chronic hepatitis C virus infection can cause mixed cryoglobulinemia, which is associated with
peripheral neuropathy. However, most cases are marked by both sensory and motor abnormalities; and patients
often have palpable purpura and arthritis or arthralgias. This patient's acid suppression and signs of dorsal spinal
cord injury make vitamin B12 deficiency more likely.
Educational objective:
Older patients with a history of chronic antacid use are at risk for vitamin B12 deficiency due to achlorhydria. These
patients frequently have subtle neurologic findings, including lower extremity paresthesias and signs of dorsal
column injury (eg, diminished light touch/vibration sensation). A serum vitamin B12 test is usually diagnostic, but
methylmalonic acid or homocysteine testing may be required for confirmation in inconclusive cases.
Question #211

A 42-year-old woman comes to the office for follow-up due to abnormal liver function studies. The patient reports
no symptoms. Medical history is significant for primary hypertension, and she has been taking amlodipine for >5
years. Family history is negative for liver disease. The patient drinks a glass of wine 3 or 4 times a week but
denies smoking or illicit drug use. Temperature is 37.5 C (99.5 F), blood pressure is 146/80 mm Hg, and heart rate
is 80/min. BMI is 28 kg/m2. Scleral icterus and palmar erythema are absent. Cardiopulmonary and abdominal
examinations are both unremarkable. Laboratory studies are as follows:

Complete blood count


Hemoglobin 13.8 g/dL
Platelets 390,000/mm3
Leukocytes 6,000/mm3

Liver function studies


Total protein 11.2 g/dL
Albumin 4.2 g/dL
Total bilirubin 0.9 g/dL
Alkaline phosphatase 90 U/L
Aspartate aminotransferase (SGOT) 452 U/L
Alanine aminotransferase (SGPT) 512 U/L

Coagulation studies
PT time 12 sec
Activated PTT 29 sec
Initial testing 3 months ago showed similar results. Which of the following most likely explains this patient's
persistent laboratory abnormalities?

A) Alcoholic hepatitis

B) Alpha-1-antitrypsin deficiency

C) Autoimmune hepatitis

D) Drug-induced liver injury

E) Hepatitis A

F) Primary biliary cholangitis


Explanation
Correct Answer:

C) Autoimmune hepatitis

Autoimmune hepatitis

• Asymptomatic
◦ Identified by abnormal LFTs
• Symptomatic
Presentation ◦ Fatigue, anorexia, nausea, jaundice
◦ Can progress to fulminant liver failure &/or cirrhosis
• Often associated with other autoimmune disorders (eg, vitiligo, autoimmune
thyroiditis, celiac disease)

• Hepatocellular pattern (↑↑ AST & ALT)


• Hypergammaglobulinemia
Laboratory • Elevated autoantibodies
findings ◦ Anti–smooth muscle
◦ Anti–liver/kidney microsomal type 1
◦ Antinuclear (nonspecific)

Histology • Portal & periportal lymphoplasmacytic infiltration


Treatment • Prednisone ± azathioprine

ALT = alanine aminotransferase; AST = aspartate aminotransferase; LFTs = liver function tests.

This woman with persistent asymptomatic elevations in aspartate aminotransferase (AST) and alanine
aminotransferase (ALT) and a large gamma gap (total protein − albumin = >4 g/dL) has autoimmune hepatitis
(AIH). AIH is thought to be triggered by environmental exposure in susceptible individuals; it occurs most
commonly in women with comorbid autoimmune disease (eg, autoimmune thyroiditis, vitiligo).

The initial presentation is variable, with most patients presenting with constitutional symptoms (eg, fatigue, weight
loss), abdominal pain, pruritus, and, possibly, cirrhosis (eg, jaundice, ascites). However, approximately 25% are
asymptomatic and identified by routine blood work demonstrating a hepatocellular pattern of liver injury, with
predominant elevations in AST and ALT and normal or mildly elevated alkaline phosphatase and bilirubin. High
levels of autoantibodies (typically IgG) result in hypergammaglobulinemia and a gamma gap, a helpful
characteristic feature (not seen in all cases). Positive serology (eg, anti–smooth muscle, anti–liver/kidney
microsomal type 1, antinuclear antibodies) or hypergammaglobulinemia confirms the diagnosis.

(Choice A) Alcoholic hepatitis also causes a hepatocellular pattern of injury, but an AST/ALT ratio of >2:1 is
expected, and transaminases rarely exceed 300 U/L. In addition, patients typically have heavy alcohol use (5 or 6
drinks/day), fever, and jaundice. A gamma gap would be atypical.

(Choice B) Alpha1-antitrypsin deficiency (AATD) is an autosomal codominant condition leading to emphysema


(typically presenting in patients in their 40s) and liver disease (eg, chronic hepatitis, cirrhosis). This patient does not
have manifestations of emphysema (eg, cough, dyspnea) or a family history of liver disease, and a protein gap
would not occur with AATD.

(Choice D) Although amlodipine can cause hepatotoxicity, this adverse effect is exceedingly rare, typically
develops within weeks to months (rather than after >5 years), and is generally associated with a cholestatic pattern.
An elevated gamma gap would not occur.
(Choice E) Hepatitis A causes a hepatocellular pattern of injury, but it typically presents with vomiting, fever, and
abdominal pain; transaminases are generally >1,000 U/L and would be expected to normalize after 3 months.

(Choice F) Primary biliary cholangitis is an autoimmune disorder characterized by elevated antimitochondrial


antibodies. Although PBC can be asymptomatic and mild AST and ALT elevation can be seen, a cholestatic picture
is expected, with marked elevations in alkaline phosphatase (≥1.5x the upper limit of normal).

Educational objective:
Autoimmune hepatitis is characterized by a hepatocellular pattern of liver injury (elevations in aspartate
aminotransferase and alanine aminotransferase with normal alkaline phosphatase and bilirubin), as well as high
levels of autoantibodies, resulting in elevated serum globulins and a gamma gap. Positive serology (eg,
anti–smooth muscle, anti–liver/kidney microsomal type1, antinuclear antibodies) or hypergammaglobulinemia
confirms the diagnosis.

Reference
• Diagnosis and management of autoimmune hepatitis: current status and future direction
Question #212

A 66-year-old woman comes to the office due to 4 days of watery diarrhea and abdominal cramps. The patient has
had 7-10 loose stools daily with no blood or mucus. She has a history of constipation requiring frequent laxative
use but last took a laxative 6 days ago. The patient received oral antibiotics for acute sinusitis 3 weeks ago. She
has a history of gastroesophageal reflux disease and type 2 diabetes mellitus. The patient takes omeprazole and
metformin daily. She smokes a pack of cigarettes daily. Temperature is 38 C (100.4 F). The abdomen is soft with
mild distension and mild diffuse tenderness. No guarding or rebound tenderness is present. Stool testing for
Clostridioides difficile is positive. In addition to recent antibiotic use, which of the following most likely predisposed
this patient to her current condition?

A) Chronic laxative use

B) Chronic metformin use

C) Cigarette smoking

D) Gastric acid suppression

E) Small bowel dysmotility


Explanation
Correct Answer:

D) Gastric acid suppression

Clostridioides difficile colitis

• Recent antibiotic use or hospitalization


• Advanced age (>65)
• Gastric acid suppression (eg, PPI, H2
Risk factors
blocker)
• Underlying inflammatory bowel disease
• Chemotherapy

• Profuse watery diarrhea


Clinical
• Leukocytosis (~15,000/mm3)
presentation
• Fulminant colitis or toxic megacolon

• Stool PCR for C difficile genes*


Diagnosis • Stool EIA for C difficile toxin & glutamate
dehydrogenase antigen

• Hand hygiene with soap & water


Infection
• Contact isolation
control
• Sporicidal disinfectants (eg, bleach)
*Genes specific to toxigenic strains are assessed.

EIA = enzyme immunoassay; H2 = histamine-2 receptor; PPI =


proton pump inhibitor.

Clostridioides difficile is a gram-positive, anaerobic bacterium that causes infectious diarrhea in nosocomial and
outpatient settings. Hardy, antibiotic-resistant spores are ingested and convert to fully functional bacilli in the
colon. Although the organism is noninvasive, pathogenic strains produce exotoxins (enterotoxin A, cytotoxin B)
that penetrate colonic epithelial cells, resulting in apoptosis and loss of tight junctions.

The greatest risk factor for C difficile–associated diarrhea is recent antibiotic use (eg, fluoroquinolones,
clindamycin, cephalosporins, penicillins). Antibiotics disrupt the normal colonic flora (eg, Bacteroides), eliminating a
crucial epithelial barrier and freeing nutrients that C difficile uses to proliferate. Other common risk factors include
the following:

• Gastric acid suppression: C difficile spores are acid resistant, but proton pump inhibitors (PPIs) are
thought to alter the colonic microbiome, which increases the risk for C difficile proliferation.

• Recent hospitalization: C difficile transmission is most common in the hospital setting, particularly when
patients are severely ill.

• Advanced age (eg, >65): Older individuals often have diminished colonic immunity and greater exposure
to antibiotics, PPIs, and hospitals.

Less common risk factors include inflammatory bowel disease and cancer chemotherapy, both of which can alter
the gut microbiome.

(Choice A) This patient discontinued laxatives 6 days ago; they are unlikely to cause diarrhea several days later.
Laxative use is not typically associated with increased risk for C difficile–associated diarrhea; gastric acid
suppression is a much stronger risk factor.
(Choice B) Metformin can cause gastrointestinal adverse effects, including watery diarrhea; however, these
adverse effects more commonly occur at the start of therapy, rather than from chronic use. Metformin use is not
associated with an increased risk of C difficile infection.

(Choice C) Cigarette smoking is not considered a major risk factor for the development of C difficile infection. In
contrast, it has been shown to negatively impact the development, severity, and progression of Crohn disease.

(Choice E) Small bowel dysmotility (eg, due to diabetes mellitus) and gastric hypochlorhydria (eg, PPI use) can
lead to small intestinal bacterial overgrowth, which can manifest as chronic watery diarrhea. However, bloating and
flatulence are also commonly present, and fever is unexpected.

Educational objective:
Clostridioides difficile is a common cause of nosocomial and outpatient infectious diarrhea. Major risk factors
include recent antibiotic use, age >65, and gastric acid suppression (eg, proton pump inhibitors). Less common risk
factors include underlying inflammatory bowel disease and cancer chemotherapy.

Reference
• Proton pump inhibitors therapy and risk of Clostridium difficile infection: systematic review and meta-
analysis.

• Association of gastric acid suppression with recurrent Clostridium difficile infection: a systematic review and
meta-analysis.

• Acute diarrhea.
Question #213

A 43-year-old woman comes to the office due to elevated aminotransferase levels, which were identified during
routine blood work. She feels well and reports no symptoms. She has no known medical conditions, and her family
history is unremarkable. She used heroin 10 years ago but completed rehabilitation and has not used it since. She
does not use tobacco or alcohol. Temperature is 37.5 C (99.5 F), blood pressure is 120/80 mm Hg, and pulse is 87/
min. BMI is 24.5 kg/m2. There is no scleral icterus or asterixis. Skin and cardiopulmonary examinations are
unremarkable. The abdomen is soft and nontender; there is no ascites. Laboratory results are as follows:

Hematocrit 38%
Platelets 145,000/mm3
Creatinine 0.8 mg/dL
Liver function studies
Albumin 2.8 g/dL
Total bilirubin 1.2 mg/dL
Alkaline phosphatase 120 U/L
Aspartate aminotransferase (SGOT) 120 U/L
Alanine aminotransferase (SGPT) 134 U/L
INR 1.7
Hepatitis panel
Hepatitis A virus IgG antibody negative
Hepatitis A virus IgM antibody negative
Hepatitis B surface IgG antibody positive
Hepatitis B surface antigen negative
Hepatitis C virus antibody positive
Hepatitis C virus RNA 1,978,677 IU/mL

Abdominal ultrasonography reveals a nodular, shrunken liver without evidence of ascites. Upper endoscopy is
negative for esophageal or gastric varices. In addition to providing antiviral therapy to treat this patient's hepatitis C
infection, which of the following would be recommended at this time?

A) Antiviral therapy for hepatitis B

B) Daily lactulose

C) Daily spironolactone

D) Hepatitis A vaccination

E) Hepatitis B vaccination

F) No additional intervention
Explanation
Correct Answer:

D) Hepatitis A vaccination

Overview of cirrhosis management

Treat
• HCV, HBV: antiviral therapy
underlying
• NASH: weight loss
liver disease

Provide
• Avoid alcohol & hepatotoxic medications
preventive
• Vaccinate for HAV & HBV (unless already immune)
care

• Screen for esophageal varices (endoscopy)


Manage • Screen for HCC: ultrasound ± serum AFP every 6-12 months
complications • Frequent clinical assessment for ascites and encephalopathy (prophylactic treatment not
recommended)

AFP = alpha fetoprotein; HAV = hepatitis A virus; HBV = hepatitis B virus; HCC = hepatocellular carcinoma; HCV =
hepatitis C virus; NASH = nonalcoholic steatohepatitis.

This patient has hepatitis C virus (HCV) cirrhosis confirmed by positive HCV antibodies, elevated HCV RNA
levels, and a nodular, shrunken liver on ultrasound. Although chronic HCV infection is often asymptomatic and
may cause only mild elevations in aminotransferases, some infected individuals (~20%) develop cirrhosis.
Laboratory findings suggestive of cirrhosis include thrombocytopenia, hypoalbuminemia, and coagulopathy (eg,
elevated PT/INR). Liver biopsy showing extensive fibrosis and regenerative nodules can confirm the diagnosis if
necessary.

A priority of cirrhosis management is to protect patients from further hepatic insult. Patients should be vaccinated
against both hepatitis A virus (HAV) and hepatitis B virus (HBV) if there is no evidence of immunity (eg, positive
HAV IgG, positive HBV surface antibody). Acute HAV or HBV infection in unvaccinated/nonimmune patients with
cirrhosis can cause severe acute or chronic liver failure. This patient has evidence of immunity to HBV but not HAV
and therefore should be vaccinated against HAV.

(Choices A and E) This patient does not require vaccination against or treatment for HBV because she does not
have active infection (eg, negative HBV surface antigen) and is already immune (eg, positive HBV surface IgG
antibody).

(Choice B) Patients with cirrhosis should be frequently assessed for features of hepatic encephalopathy (eg,
abnormal mental status, sleep disturbance, asterixis), but asymptomatic patients do not require treatment (eg,
lactulose, rifaximin).

(Choice C) Cirrhosis predisposes to ascites, which is treated with diuretics (eg, spironolactone, furosemide), but
prophylactic therapy is not recommended for patients without ascites.

(Choice F) This patient with cirrhosis requires vaccination against HAV.

Educational objective:
Patients with cirrhosis from any cause, including chronic hepatitis C virus, should be vaccinated against both
hepatitis A virus (HAV) and hepatitis B (HBV) virus unless they have serologic evidence of immunity. Acute HAV or
HBV infection in unvaccinated/nonimmune patients with cirrhosis can cause severe acute or chronic liver failure.

Reference
• Diagnosis and management of hepatitis C.
Question #214

A 37-year-old man comes to the office for a routine health maintenance examination. He has been feeling well and
has no significant symptoms. The patient does not use tobacco, alcohol, or illicit drugs. His father was diagnosed
with rectal cancer at age 50 and underwent partial colectomy. Vital signs and physical examination show no
abnormalities. Which of the following is the most appropriate initial screening test for this patient?

A) Colonoscopy at age 40

B) Colonoscopy at age 45

C) Colonoscopy at age 50

D) Colonoscopy now

E) Yearly fecal occult blood test starting at age 40

F) Yearly fecal occult blood test starting at age 45


Explanation
Correct Answer:

A) Colonoscopy at age 40

Colon cancer screening

• Start at age 45:


◦ Colonoscopy every 10 years
◦ gFOBT or FIT every year
Patients at ◦ FIT-DNA every 1-3 years
average risk ◦ CT colonography every 5 years

◦ Flexible sigmoidoscopy every 5 years (or every 10 years


with annual FIT)

• Colonoscopy at age 40 (or 10 years prior to age of diagnosis


Patients with FDR with CRC or high-risk in FDR, whichever comes first)
adenomatous polyp* • Repeat every 5 years (every 10 years if FDR diagnosed at
age >60)

• Start screening 8-10 years after diagnosis


Patients with ulcerative colitis
• Colonoscopy every 1-3 years
*Adenomatous polyp ≥10 mm, high-grade dysplasia, villous elements (for example).

CRC = colorectal cancer; FDR = first-degree relative; FIT = fecal immunochemical test; FIT-DNA = multitarget
stool DNA test; gFOBT = guaiac-based fecal occult blood test.

For average-risk adults, screening for colorectal cancer (CRC) is recommended beginning at age 45. Commonly
used modalities include annual fecal occult blood testing, multitarget stool DNA testing every 1-3 years, and
colonoscopy every 10 years. However, patients with a first-degree family history of CRC (as in this case) or
advanced adenomatous polyps (eg, ≥10 mm, high-grade dysplasia, villous elements) are at increased risk and
should undergo earlier testing.

Of the available screening methods, colonoscopy has the greatest sensitivity and is the recommended option for
individuals at increased risk. Patients with a first-degree family history of CRC should undergo initial colonoscopy at
age 40 or 10 years prior to the age of diagnosis of that relative, whichever is earlier (Choices B, C, and D).

If the relative's CRC was diagnosed at an early age (ie, <60), the patient's screening colonoscopy should be
repeated every 5 years. If the relative's CRC was diagnosed at a later age (ie, ≥60) and the patient has a normal
initial colonoscopy, the patient has only a nominally increased risk for CRC and may undergo repeat screening as
for an average-risk patient.

(Choices E and F) Yearly fecal occult blood testing is not recommended for individuals at increased risk of CRC
because it has low sensitivity compared to colonoscopy; visualization of the entire colon with colonoscopy is
preferred. If the individual is not willing to undergo colonoscopy, yearly multitarget stool DNA test is reasonable
given its high sensitivity.

Educational objective:
For average-risk adults, screening for colorectal cancer (CRC) is recommended beginning at age 45. Patients with
a first-degree family history of CRC or advanced adenomatous polyps are at increased risk and should undergo
screening colonoscopy beginning at age 40 or 10 years prior to the age of diagnosis of the first-degree relative,
whichever comes first.
Reference
• Screening for colorectal cancer: US Preventive Services Task Force recommendation statement.

• ACG clinical guidelines: colorectal cancer screening 2021.


Question #215

A 40-year-old man comes to the office due to a 2-day history of retrosternal chest pain, pain with swallowing, and
epigastric burning. The pain is severe and the patient is afraid to swallow food. He has never had similar
symptoms before. The patient has no associated shortness of breath, vomiting, melena, or blood in the stool. Past
medical history is notable for nonischemic cardiomyopathy, for which he takes furosemide, carvedilol,
spironolactone, lisinopril, and potassium chloride. Temperature is 37.2 C (99 F), blood pressure is 110/65 mm Hg,
and pulse is 67/min. Examination shows a soft holosystolic murmur at the cardiac apex, similar to that in prior
examinations. The rest of the examination shows no abnormalities. The patient's ECG shows sinus rhythm with no
acute ischemic changes. Endoscopy reveals circumferential deep ulceration with relatively normal surrounding
mucosa at the middle third of the esophagus. Which of the following is most likely diagnosis?

A) Candida esophagitis

B) Diffuse esophageal spasm

C) Esophageal carcinoma

D) Gastroesophageal reflux disease

E) Pill-induced esophagitis

F) Viral esophagitis
Explanation
Correct Answer:

E) Pill-induced esophagitis

Medication-induced esophagitis

Drug class Drug

Antibiotics Tetracyclines

Anti-inflammatory Aspirin & many nonsteroidal


agents anti-inflammatory drugs

Bisphosphonates Alendronate, risedronate

Others Potassium chloride, iron

This patient has abrupt-onset retrosternal pain and severe odynophagia, suggestive of medication-induced
esophagitis (pill esophagitis). Pill esophagitis is due to a direct effect of certain medications on esophageal
mucosa. Mucosal injury in pill esophagitis can be due to acid effect (eg, tetracyclines), osmotic tissue injury (eg,
potassium chloride), or disruption of normal gastroesophageal protection (eg, nonsteroidal anti-inflammatory
drugs). Patients usually do not have prior esophageal disease, although pill esophagitis can be worse in those with
concurrent gastroesophageal reflux.

Typical symptoms of pill esophagitis include sudden-onset odynophagia and retrosternal pain that can sometimes
cause difficulty swallowing. It is most common in the mid-esophagus due to compression by the aortic arch or an
enlarged left atrium. The diagnosis is usually made clinically but can be confirmed on endoscopy, which shows
discrete ulcers with relatively normal-appearing surrounding mucosa. Treatment includes primarily stopping the
offending medication to prevent future injury.

(Choices A and F) Endoscopy in Candida esophagitis is characterized by white plaques, and most patients will
also have oral thrush. Ulcerating lesions are seen in esophagitis due to herpes simplex (vesicles and round/ovoid
ulcers) or cytomegalovirus (large linear ulcers). These conditions are most common in immunocompromised
patients.

(Choice B) Diffuse esophageal spasm is characterized by uncoordinated, simultaneous contractions of the lower
esophagus. Patients typically have recurrent episodes of liquid/solid dysphagia and chest pain.

(Choice C) Esophageal cancer typically presents with progressive solid food dysphagia and unintentional weight
loss. Risk factors for squamous cell carcinoma include alcohol and tobacco use, whereas factors for
adenocarcinoma include Barrett esophagus, gastroesophageal reflux disease, smoking, and obesity.

(Choice D) Gastroesophageal reflux disease typically causes recurrent or persistent burning pain in the upper
abdomen and chest. Symptoms are worse following large meals or certain foods (eg, chocolate, peppermint) or
when lying down. Symptoms are usually subacute to chronic rather than abrupt.

Educational objective:
Pill esophagitis is due to a direct effect of certain medications on esophageal mucosa. Tetracyclines, potassium
chloride, bisphosphonates, and nonsteroidal anti-inflammatory drugs are common causes. Patients experience
sudden-onset odynophagia and retrosternal pain that can sometimes cause difficulty swallowing.

Reference
• Drug-induced esophagitis.
Question #216

A 32-year-old woman comes to the office with rectal pain. The pain is crampy and episodic, occurring at least once
every week and lasting about 5 minutes each time before completely resolving. It is not associated with defecation,
sexual intercourse, sitting, or standing. The patient reports no rectal bleeding, pruritus, constipation, diarrhea, fecal
or urinary incontinence, or unexplained weight loss. She is sexually active. Vital signs are normal. Conjunctival
pallor is absent, and mucous membranes are moist. Heart and lung sounds are normal. The abdomen is
nontender and nondistended. Pelvic and rectal examinations show no abnormalities. Which of the following is the
most likely diagnosis?

A) Coccydynia

B) Endometriosis

C) Fibromyalgia

D) Irritable bowel syndrome

E) Proctalgia fugax

F) Sacral nerve compression


Explanation
Correct Answer:

E) Proctalgia fugax

Proctalgia fugax

• Spastic contraction of the anal sphincter


Pathophysiology
• Pudendal nerve compression

• Female sex
Risk factors • Other functional pathologies (eg, irritable bowel syndrome)
• Psychosocial stress, anxiety

• Recurrent rectal pain unrelated to defecation


Manifestations • Episodes lasting seconds to minutes (≤30 min)
• No pain between episodes

• Normal physical examination (eg, rectal, pelvic, prostate)


Evaluation
• No laboratory abnormalities

Management • Reassurance
• Nitroglycerin cream ± biofeedback therapy for refractory symptoms

This patient with recurrent episodes of rectal pain most likely has proctalgia fugax (PF). PF is a functional
anorectal disorder characterized by recurrent, brief episodes (seconds to minutes) of rectal pain unrelated to
defecation. Patients are pain-free between episodes. Attacks may be precipitated by stress, sexual intercourse,
and/or sitting but often occur without an obvious stimulus, as in this patient. Although the pathophysiology is poorly
understood, it likely involves uncontrolled spasms of the anal sphincter and possibly intermittent pudendal nerve
compression. Women are affected more commonly than men; other risk factors include functional pathologies (eg,
irritable bowel syndrome) and anxiety disorders.

PF is a diagnosis of exclusion; abnormalities in the physical examination (eg, digital rectal, prostate, pelvic) or
laboratory evaluation (eg, complete blood count, electrolytes, inflammatory markers) suggest an alternate
diagnosis. Most patients can be reassured of the benign nature of this condition. Because symptoms are often
brief and no preventive measure is effective, most patients do not require specific treatment. Topical nitroglycerin or
biofeedback therapy may be useful for frequent or prolonged symptoms.

(Choice A) Coccydynia (pain at the coccyx) is most commonly diagnosed in women with obesity and often results
from traumatic injury. The pain is exacerbated by sitting, and a rectal examination elicits tenderness of the coccyx
bone during palpation.

(Choice B) Endometriosis sometimes presents with rectal pain but is cyclical typically and related to the menstrual
cycle. In addition, it usually causes dyspareunia and dysmenorrhea.

(Choice C) Fibromyalgia causes widespread musculoskeletal pain as well as fatigue and sleep disruption. This is
unlikely in this patient because her pain is limited to the rectum.

(Choice D) Irritable bowel syndrome causes recurrent abdominal pain related to defecation as well as constipation
and/or diarrhea. This patient's isolated rectal pain unrelated to defecation is more consistent with PF.

(Choice F) Compression of the sacral nerves S2-S4 can lead to sacral and buttock pain; however, pain often
radiates down the leg. Urinary and fecal incontinence and sexual dysfunction are common. In addition, symptoms
would not be expected to occur intermittently.

Educational objective:
Proctalgia fugax is a functional anorectal disorder characterized by recurrent episodes of rectal pain unrelated to
defecation. Attacks may be precipitated by stress, sexual intercourse, and/or sitting but often occur without an
obvious stimulus. Proctalgia fugax is a diagnosis of exclusion; physical examination (eg, digital rectal, prostate,
pelvic) and laboratory analysis are normal.

Reference
• Functional anorectal disorders.
Question #217

A 46-year-old woman comes to the clinic due to left lower extremity swelling for the past 2 years. She has had
several episodes of cellulitis involving the left leg. During the most recent episode 6 months ago, she underwent
treatment with intravenous antibiotics that was complicated by a catheter-related axillary vein thrombosis requiring 3
months of anticoagulation therapy. Her medical history is also significant for diet-controlled type 2 diabetes
mellitus. BMI is 34 kg/m2. Physical examination shows firm edema of the left lower extremity. There is no
erythema or warmth. The examiner cannot lift the skin from the dorsum of the toes on the left foot but is able to do
so with the toes on the right foot. Which of the following is the most likely cause of this patient's symptoms?

A) Albuminuria

B) Disruption of lymphatics

C) Increased central venous pressure

D) Systemic sclerosis

E) Venous valve incompetence


Explanation
Correct Answer:

B) Disruption of lymphatics

Chronic lymphedema

• Physical disruption of lymphatic drainage


◦ Lymphadenectomy, radiation
◦ Malignant obstruction
Etiology
◦ Chronic inflammation (eg, recurrent cellulitis)
◦ Parasitic infection (eg, filariasis)
◦ Congenital (eg, Turner syndrome)

• Swelling, heaviness, discomfort


Clinical presentation • Early: soft skin, pitting edema
• Late: firm, dry & thickened skin; nonpitting edema

• Weight loss, limb elevation


Treatment
• Compression bandages & physiotherapy
• Diuretics contraindicated

This patient most likely has chronic lymphedema of the left leg. Lymphedema most commonly results from an
acquired disruption of the lymphatic system that leads to accumulation of lymphatic fluid in the interstitium.
Common causes include malignancy and its treatment (eg, radiation, lymph node dissection), chronic inflammation
(eg, recurrent cellulitis, connective tissue disease), and severe chronic venous insufficiency. Obesity is often a
strong contributing factor.

Chronic lymphedema typically presents with pain, swelling, and heaviness in one or more extremities. Patients with
early disease usually demonstrate soft skin with pitting edema. However, progressive deposition of
subcutaneous collagen and adipose tissue occurs, leading to later development of firm, thickened skin and
nonpitting edema. In advanced cases, the skin may develop a warty appearance. Inability to lift the skin on the
dorsum of the second toe (positive Stemmer sign) is highly specific for lymphedema.

The diagnosis is usually based on clinical presentation, but imaging (eg, duplex ultrasound) may be useful in some
cases. Treatment is challenging and may involve weight loss, limb elevation, compression bandages, and
physiotherapy (eg, lymphatic drainage by massage).

(Choices A and C) Severe albuminuria (eg, diabetic nephropathy) can lead to peripheral edema due to low oncotic
pressure of the blood, and increased central venous pressure (eg, heart failure) can lead to lower extremity edema
due to elevated hydrostatic pressure in the deep veins of the legs. Lymphedema, evidenced by thick skin and a
positive Stemmer sign, is not typical in either setting.

(Choice D) Systemic sclerosis often leads to peripheral edema due to inflammatory disruption of blood vessel
walls; lymphedema due to inflammatory disruption of lymphatics can occur but is less common. If systemic
sclerosis were present in this patient, other suggestive features (eg, sclerodactyly, Raynaud phenomenon) would be
expected.

(Choice E) Venous valve incompetence is the major cause of lower extremity edema due to chronic venous
insufficiency. When severe, lymphedema may occur due to lymphatic obstruction. However, this patient's unilateral
symptoms and history of recurrent cellulitis make venous valve incompetence an unlikely underlying cause.

Educational objective:
Chronic lymphedema is most commonly caused by an acquired disruption of the lymphatic system (eg, chronic
inflammation, malignancy) and typically presents with pain and swelling in one or more extremities. Patients usually
have pitting edema that later progresses to nonpitting edema accompanied by firm, thickened skin.
Reference
• Prevalence and characteristics of lymphoedema at a wound-care clinic.
Question #218

A 68-year-old man comes to the office due to 2 weeks of worsening fatigue and exertional dyspnea. The patient
was diagnosed with early-stage chronic lymphocytic leukemia a year ago but has not required treatment because
he has not had significant symptoms. Vital signs are within normal limits. Physical examination shows generalized
lymphadenopathy and hepatosplenomegaly. Laboratory results are as follows:

Complete blood count


Hemoglobin 7.1 g/dL
Reticulocytes 10%
Platelets 210,000/mm3
Leukocytes 44,800/mm3 (80% lymphocytes)

Hemoglobin was within normal limits a month ago. Which of the following is the most likely underlying cause of this
patient's anemia?

A) Cytokine-mediated iron dysregulation

B) Folate consumption by leukemic cells

C) Immune-mediated hemolysis

D) Leukemic bone marrow infiltration

E) Selective suppression of erythroid precursors


Explanation
Correct Answer:

C) Immune-mediated hemolysis
This patient's fatigue, exertional dyspnea, and low hemoglobin suggest acute symptomatic anemia. Although
anemia can be triggered by a wide range of conditions, concurrent clinical data often narrows the differential
diagnosis.

In this case, a high reticulocyte count suggests that there are adequate levels of iron, folate, and vitamin B12, to
generate new erythrocytes, which makes cytokine-mediated iron dysregulation (eg, anemia of chronic disease) and
folate deficiency (eg, leukemic folate consumption) unlikely (Choices A and B). It also suggests that the bone
marrow is responding appropriately to the anemia by increasing the production of erythrocytes, which (along with
the normal platelet count) makes bone marrow infiltration with leukemic cells less likely (Choice D).

Most cases of anemia with a high reticulocyte count are caused by acute bleeding or hemolysis. In this case,
hemolysis is most likely because patients with chronic lymphocytic leukemia often have significant immune
dysregulation, which triggers the formation of IgG autoantibodies against the erythrocyte membrane (warm
agglutinins). Red blood cells (RBCs) coated with IgG are subsequently identified by the Fc receptor on splenic
macrophages and partially or wholly phagocytized, leading to extravascular, immune-mediated hemolysis (ie,
autoimmune hemolytic anemia). Patients typically present with signs of acute anemia and RBC lysis (eg, jaundice,
dark urine, elevated indirect bilirubin). The diagnosis is confirmed with a direct antiglobulin (Coombs) test, which
detects the presence of IgG or C3 (a complement fragment) on erythrocytes.

(Choice E) Autoantibodies against erythroid precursors can lead to pure red cell aplasia, which is marked by
anemia with a paucity of reticulocytes (not reticulocytosis).

Educational objective:
Anemia with reticulocytosis suggests that the bone marrow is responding appropriately to the anemia by generating
new erythrocytes and that sufficient levels of folate, vitamin B12, and iron are available for erythrocytosis. Anemia
with reticulocytosis is commonly seen in acute bleeding conditions and hemolysis. Chronic lymphocytic leukemia is
associated with warm autoimmune hemolytic anemia.
Question #219

A 53-year-old man comes to the office due to a 3-day history of right leg swelling and pain. He has had no chest
pain or dyspnea. The patient describes himself as healthy, and his last visit to the doctor was 10 years ago. He has
had no weight loss or abdominal pain. The patient smokes and has a 30-pack-year history. He has an active
lifestyle and has not recently traveled. The patient's mother died of breast cancer, and his father has congestive
heart failure. Temperature is 37 C (98.6 F), blood pressure is 140/80 mm Hg, pulse is 70/min, and respirations are
14/min. Oxygen saturation is 97% on room air. Normal vesicular breath sounds and cardiac sounds are heard on
chest auscultation. Abdominal examination is unremarkable. The right leg is swollen and tender up to midthigh.
Results of complete blood cell count and coagulation studies are within normal limits. Duplex ultrasonography
demonstrates incompressible popliteal and femoral veins, and anticoagulation is started immediately. Chest x-ray is
unremarkable. Which of the following is most appropriate in evaluation of this patient's current condition?

A) Age-appropriate cancer screening

B) CT scan of the abdomen

C) Positron emission tomography of the chest

D) Protein C, protein S, and antithrombin tests

E) Serum carbohydrate antigen (CA) 19-9 and carcinoembryonic antigen tests


Explanation
Correct Answer:

A) Age-appropriate cancer screening

This patient has deep venous thrombosis (DVT) of the popliteal and femoral veins. Risk factors for venous
thromboembolism (VTE) include inherited (eg, factor V Leiden, prothrombin gene mutation, protein C deficiency)
and acquired (eg, immobilization, surgery, malignancy, medications) factors. Initial evaluation is directed at
identifying major causes of VTE or factors that can increase the risks of anticoagulation therapy; evaluation should
include careful history, physical examination, and limited diagnostic testing (eg, blood counts, serum chemistries,
coagulation studies, fecal occult blood testing, chest x-ray).

In the absence of any clear provoking factors (eg, recent surgery, immobilization), patients with a first episode of
VTE, such as this man, should be referred for age-appropriate cancer screening (eg, colonoscopy) to evaluate
for malignancy as a potential risk factor for VTE. In addition, any suspicious symptoms of malignancy (eg, weight
loss, unexplained pain) warrant consideration for more extensive cancer evaluation (eg, CT scan of the abdomen)
(Choice B).

(Choice C) Because of this patient's smoking history, he should be offered screening for lung cancer with low-dose
CT scan of the chest. However, he has a normal chest x-ray and no symptoms concerning for lung cancer (eg,
hemoptysis, cough), so positron emission tomography of the chest would not be appropriate.

(Choice D) Testing for an inherited thrombophilia (eg, factor V Leiden; deficiency of protein C, protein S, or
antithrombin III) is generally considered only if the patient's history is suggestive of a genetic predisposition (eg,
recurrent VTE; multiple or unusual sites of thrombosis; family history of VTE, particularly at a young age).

(Choice E) Carbohydrate antigen (CA) 19-9 and carcinoembryonic antigen are tumor markers generally used to
trend disease progression in patients with diagnosed pancreatic and gastrointestinal tract cancer, respectively.
They are not effective screening tests.

Educational objective:
In the absence of any clear provoking factors (eg, recent surgery, immobilization), patients with a first episode of
venous thromboembolism should be referred for age-appropriate cancer screening (eg, colonoscopy) to evaluate
for malignancy as a potential risk factor for this disease.

Reference
• Screening for occult cancer in unprovoked venous thromboembolism.
Question #220

A 42-year-old woman comes to the physician with bilateral knee pain that severely limits her mobility. She also
complains of diffuse morning joint stiffness that takes several hours to improve. She has been taking over-the-
counter ibuprofen and aspirin but has experienced little relief of symptoms. She has no other medical problems and
does not use tobacco, alcohol, or illicit drugs. Her vital signs are within normal limits. Physical examination shows
tenderness and swelling of multiple metacarpophalangeal joints, as well as both wrists and knees. Laboratory
results are as follows:

Hemoglobin 8.4 g/dL


Serum iron 30 μg/dL (normal 50-150 μg/dL)
Total iron-binding capacity 230 μg/dL (normal 250-460 μg/dL)
Ferritin 300 ng/mL (normal 15-200 ng/mL)

Which of the following is the most appropriate next step in management of this patient's anemia?

A) Cyanocobalamin supplementation

B) Erythropoietin

C) Folic acid supplementation

D) Iron supplementation

E) Methotrexate

F) Packed red blood cell transfusion


G) Splenectomy
Explanation
Correct Answer:

E) Methotrexate

Iron studies in microcytic anemia

Transferrin
saturation
Cause MCV Iron TIBC Ferritin
(Iron/TIBC)

Iron deficiency ↓ ↓ ↑ ↓ ↓

Thalassemia ↓↓ ↑ ↓ ↑ ↑↑

Anemia of chronic disease


Normal/↓ ↓ ↓ Normal/↑ Normal/↓
(inflammation)

MCV = mean corpuscular volume; TIBC = total iron binding capacity.

This patient has inflammatory arthritis, which is most consistent with early rheumatoid arthritis. She has moderate
anemia with low serum iron and total iron-binding capacity (TIBC) consistent with anemia of chronic disease
(ACD). Other characteristic laboratory findings of ACD include normochromic normocytic red blood cells, mildly
decreased transferrin saturation, and normal to elevated ferritin. In contrast, iron-deficiency anemia features
elevated TIBC and low ferritin. Erythropoietin levels are variable, but ACD patients usually have a low reticulocyte
count relative to anemia severity, suggesting impaired red blood cell production. ACD pathophysiology is thought to
involve iron trapping within macrophages, leading to reduced serum iron concentrations and poor iron availability
for hemoglobin synthesis. Decreased erythropoietin production and poor marrow response to erythropoietin may
also play a role.

ACD is commonly associated with chronic inflammatory diseases (eg, infections, cancer, autoimmune disorders) but
can also be observed in heart disease, diabetes mellitus, and acute inflammation. Treating the underlying
inflammatory disorder will often improve the anemia. Common agents used in rheumatoid arthritis treatment
include methotrexate, hydroxychloroquine, and tumor necrosis factor inhibitors (eg, infliximab, etanercept).
Erythropoietin or darbepoetin treatment may benefit ACD patients unresponsive to underlying inflammatory disorder
treatment (Choice B). Packed red blood cell transfusions are rarely required but can be considered after all other
treatment interventions have failed (Choice F).

(Choice D) Serum iron concentrations are low in ACD patients. However, iron supplementation is not beneficial
due to impaired iron utilization in ACD.

(Choices A and C) B-complex vitamins (pyridoxine, folic acid, cyanocobalamin, nicotinic acid) are not deficient in
ACD; supplementation will not improve the anemia of ACD.

(Choice G) Hereditary spherocytosis is treated via splenectomy; this can be occasionally used for treating immune
thrombocytopenic purpura and chronic idiopathic myelofibrosis with refractory anemia. However, splenectomy will
not improve this patient's anemia.

Educational objective:
Anemia of chronic disease is a disorder of iron utilization that most commonly occurs in the setting of chronic
inflammation. It is characterized by a normocytic anemia with decreased serum iron, decreased total iron-binding
capacity, decreased iron saturation, and normal/elevated serum ferritin. Treating the underlying inflammatory
disorder will often improve the anemia.
Reference
• Anemia of chronic disease.
Question #221

A 24-year-old woman comes to the office due to a month of progressive exertional dyspnea. The patient does not
have chest pain or palpitations. She has no chronic medical conditions and takes no medications regularly.
Temperature is 37 C (98.6 F), blood pressure is 114/68 mm Hg, pulse is 90/min, and respirations are 12/min. BMI is
23 kg/m2. Examination shows mucosal pallor and bilateral cervical lymphadenopathy. Cardiopulmonary
auscultation reveals clear lung fields and no murmurs. Laboratory results are as follows:

Hemoglobin 9 g/dL
Mean corpuscular volume 76 µm3
Reticulocytes 0.5%
Platelets 240,000/mm3
Leukocytes 7,500/mm3

Creatinine 0.8 mg/dL

Chest x-ray reveals significant bilateral mediastinal lymphadenopathy. What is the most likely cause of this patient's
anemia?

A) Immune-mediated hemolysis

B) Increased inflammatory cytokines

C) Increased iron recycling by macrophages

D) Intratumor hemorrhage

E) Thrombotic microangiopathy
Explanation
Correct Answer:

B) Increased inflammatory cytokines

Anemia of chronic disease

• ↑ inflammatory cytokines (eg, hepcidin)


• Inhibition of ferroportin on enterocytes and macrophages
Pathogenesis
• ↓ iron absorption and ↑ iron sequestration
• Reduced circulating iron → impaired erythropoiesis

• Malignancy
• Chronic infection
• Rheumatic disease
Common Etiologies
• Obesity
• Diabetes mellitus
• Congestive heart failure

• Normocytic/slightly microcytic anemia


• ↓ serum iron, iron-binding capacity, reticulocyte count
Laboratory findings
• ↑ bone marrow iron
• Lower than expected erythropoietin for degree of anemia
Treatment • Treat underlying condition causing inflammation

Microcytic anemia with an inappropriately low reticulocyte response is primarily seen in iron deficiency anemia and
anemia of chronic disease (ACD). In this case, the presence of a significant cervical and mediastinal
lymphadenopathy (likely compressing the bronchus and causing dyspnea) in an otherwise young, healthy patient
raises suspicion for ACD due to underlying malignancy (eg, Hodgkin lymphoma).

ACD develops due to long-term elevations in serum inflammatory cytokines; most cases are triggered by
underlying rheumatologic disease, chronic infection, obesity, diabetes mellitus, or malignancy. Although a wide
range of inflammatory cytokines (eg, IL-1, IL-6, tumor necrosis factor-alpha, interferon gamma) contribute to the
hematologic changes associated with ACD, the primary mediator is hepcidin, a small peptide produced by the liver
in response to inflammation or bacterial lipopolysaccharide.

Hepcidin binds to and destroys iron channels (ferroportin) on enterocytes and reticuloendothelial macrophages,
leading to reduced iron absorption in the gut and reduced iron release from the reticuloendothelial system (which
scavenges senescent erythrocytes and provides 95% of daily iron). This dramatically reduces iron availability for
the generation of new erythrocytes, which causes a normocytic or slightly microcytic anemia with low
reticulocyte response.

(Choice A) Immune-mediated hemolysis is marked by rapid-onset normocytic anemia with a high reticulocyte
response. Indirect bilirubin and lactate dehydrogenase are usually elevated and haptoglobin is reduced. Although
thymoma can cause pure red cell aplasia due to immune-mediated causes, it is typically marked by a mediastinal
mass with no lymphadenopathy.

(Choice C) Macrophages in the reticuloendothelial system process senescent erythrocytes and provide
approximately 95% of the iron needed for erythrocytosis. ACD is marked by reduced (not increased) erythrocyte
recycling due to the destruction of iron channels on the plasma membrane of reticuloendothelial macrophages,
which reduces iron release.

(Choice D) Intratumor hemorrhage is uncommon but would be marked by acute pain and reticulocytosis due to
acute blood-loss anemia.
(Choice E) Patients with malignancy are at increased risk for thrombotic microangiopathy, which causes
microangiopathic hemolytic anemia. In this condition, reticulocytes are increased (due to stimulation of
erythrocytosis) and severe thrombocytopenia is present (due to the consumption of platelets in the
microvasculature).

Educational objective:
Anemia of chronic disease is caused by long-term elevation in inflammatory cytokines. It is primarily mediated by
hepcidin, a small peptide produced by the liver that destroys iron channels on enterocytes and reticuloendothelial
macrophages. This limits iron availability for erythropoiesis, which causes a normocytic or slightly microcytic
anemia with a low reticulocyte response.
Question #222

A 67-year-old woman comes to the office due to progressive fatigue and anorexia over the last 6 months. She has
lost 4 kg (8.8 lb), which she attributes to early satiety. She has had no abdominal pain, night sweats, or fevers. The
patient's past medical history is significant for hypothyroidism, hypertension, and gout. She does not use tobacco,
alcohol, or illicit drugs and is compliant with her medications. Temperature is 37.3 C (99.2 F), blood pressure is
128/82 mm Hg, and pulse is 88/min. The patient is slightly thin but well nourished. Mild mucosal pallor is present.
There is no lymphadenopathy in the cervical or supraclavicular chains. Cardiopulmonary examination is normal. A
spleen tip is palpable with deep exhalation. Laboratory results are as follows:

Hemoglobin 8.6 g/dL


Platelets 285,000/mm3
Leukocytes 42,800 /mm3

Florescence in situ hybridization reveals an abnormality in chromosome 22. Which of the following is the most
important target in treating this patient's disease?

A) DNA methylation mutations

B) Folic acid metabolism

C) Retinoic acid receptor

D) Thymidine synthesis

E) Tyrosine kinase
Explanation
Correct Answer:

E) Tyrosine kinase

Chronic myeloid leukemia (CML) is a clonal myeloproliferative disorder driven by the abnormal fusion gene BCR-
ABL (due to a translocation between chromosomes 9 and 22). This gene causes leukemogenesis due to a
constitutively active tyrosine kinase. First-line treatment for most CML patients involves tyrosine kinase
inhibitors such as imatinib. Although these drugs are not curative, they often can induce long-term remission.

Patients with CML may present asymptomatically, although fatigue, night sweats, weight loss, and abdominal
fullness (due to splenomegaly) are not uncommon. Patients may experience weight loss and early satiety from
splenomegaly. Peripheral blood smear in CML reveals dramatic leukocytosis (often >100,000/mm3) with absolute
basophilia and a shift toward very early neutrophil precursors (promyelocytes, myelocytes).

(Choice A) DNA methylation mutations are found in many hematologic malignancies including acute myelogenous
leukemia and certain myelodysplastic syndromes. Drugs are available to counter the effects of these mutations, but
they are not currently used in CML.

(Choices B and D) Methotrexate alters folic acid metabolism and reduces the ability of the body to generate
thymidine as folate is an important component of thymidine synthesis. Methotrexate is used in many autoimmune
conditions and neoplasms but is not used in the treatment of CML.

(Choice C) The retinoic acid receptor is involved in the pathogenesis of acute promyelocytic leukemia (APML), a
subtype of acute myeloid leukemia. All-trans retinoic acid is a crucial component of therapy for APML, which often
presents with complications due to pancytopenia.

Educational objective:
Chronic myeloid leukemia (CML) is driven by a reciprocal translocation of chromosomes 9 and 22 forming the BCR-
ABL fusion gene. This gene creates a constitutively active tyrosine kinase. Tyrosine kinase inhibitors such as
imatinib are a key therapy in the treatment of CML.
Question #223

A 58-year-old man comes to the office due to 2 months of exertional shortness of breath and easy fatigability. The
patient has no chronic medical problems and takes no medications. He eats a balanced diet. He does not use
tobacco, alcohol, or illicit drugs. Vital signs are within normal limits. Physical examination shows conjunctival pallor
and areas of depigmentation on the arms suggestive of vitiligo. The tongue appears shiny. The spleen is not
palpable. The remainder of the examination is normal. Laboratory results are as follows:

Complete blood count


Hemoglobin 7.5 g/dL
Mean corpuscular volume 110 µm3
Platelets 135,000/mm3
Leukocytes 4,100/mm3

Which of the following is the most likely cause of this patient's current symptoms?

A) Autoantibodies to intrinsic factor

B) Autoimmune (Hashimoto) thyroiditis

C) Dietary folic acid deficiency

D) Dietary vitamin B12 deficiency

E) Replacement of bone marrow with fat cells


Explanation
Correct Answer:

A) Autoantibodies to intrinsic factor

Common causes of macrocytic anemia*

• Folate deficiency
• Vitamin B12 deficiency
• Myelodysplastic syndromes
• Acute myeloid leukemias
• Drug-induced (eg, hydroxyurea, zidovudine, chemotherapy agents)
• Liver disease
• Alcohol abuse
• Hypothyroidism

*Mean corpuscular volume >100 μm3.

This patient's macrocytic anemia, mild thrombocytopenia/leukopenia, and glossitis (smooth, shiny tongue) indicate
likely vitamin B12 deficiency. The leading cause of vitamin B12 deficiency is pernicious anemia, an autoimmune
condition associated with antibodies against intrinsic factor (a protein carrier crucial for vitamin B12 absorption).
Most cases occur in older patients who have other autoimmune conditions (eg, vitiligo, autoimmune thyroid
disease).

Because vitamin B12 serves as a crucial cofactor for DNA synthesis, deficiency often disproportionately affects
rapidly dividing cells; common manifestations include:
• Megaloblastic anemia (eg, macrocytosis, hypersegmented neutrophils, anemia, mild leukopenia/
thrombocytopenia) due to impaired hematopoiesis
• Glossitis due to impaired replication of gastrointestinal epithelium

The diagnosis of pernicious anemia can be confirmed by the detection of anti–intrinsic factor antibodies; treatment
with vitamin B12 supplementation is generally curative.

(Choice B) Patients with hypothyroidism from autoimmune thyroiditis usually develop normocytic, normochromic
anemia (due to bone marrow hypoproliferation). Occasionally, mild macrocytic anemia can occur, but no
megaloblastic changes or other signs of vitamin B12 deficiency (eg, glossitis) would be present. Most cases arise in
young women.

(Choice C) Nutritional folate deficiency, which can also cause megaloblastic anemia, is quite rare in developed
countries due to the fortification of grain with folate. However, folate body stores are lower than those of vitamin B12
and can be depleted in a few months when nutrition is poor (eg, severe alcohol abuse). This patient eats a
balanced diet and is unlikely to have folate deficiency.

(Choice D) Dietary vitamin B12 deficiency is uncommon in those who eat a balanced diet because the vitamin is
found in a wide range of assessable animal proteins (eg, meat, dairy, eggs) and total body stores are generally
sufficient for >1 year (total stores are 2-5 mg and daily requirement is ~2 µg/day). However, patients with long-
standing poor nutrition (eg, chronic alcoholic use) or vegan dietary habits may be at risk.

(Choice E) Aplastic anemia is associated with the destruction of hematopoietic stem cells, leading to the
replacement of bone marrow with fat. Although pancytopenia is common, macrocytosis is rarely seen and glossitis
would be atypical.

Educational objective:
Pernicious anemia, an autoimmune condition associated with antibodies against intrinsic factor, is the most
common cause of vitamin B12 deficiency. It occurs most often in older adults with other autoimmune conditions.
Patients generally have megaloblastic anemia, atrophic glossitis (smooth, shiny tongue), and/or neurologic
abnormalities (eg, lower extremity paresthesia, subacute combined degeneration).
Question #224

A 70-year-old man comes to the office due to 6 months of progressive fatigue and dyspnea. He has had no fever,
night sweats, or weight loss. The patient's medical history is significant for benign prostatic hyperplasia,
hypertension, and mechanical aortic valve replacement for severe aortic stenosis. The patient currently takes
warfarin, tamsulosin, and amlodipine. He does not drink alcohol. Temperature is 36.4 C (97.5 F), blood pressure is
150/99 mm Hg, and pulse is 68/min. BMI is 27 kg/m2. The patient appears well nourished and fit. No scleral
icterus is present, but the mucosa is pale. There is no jugular venous distension or lymphadenopathy. Cardiac
examination reveals a normal S1 and a mechanical S2. Pulmonary examination is normal. Both the liver and the
spleen are palpable. The remainder of the examination shows no abnormalities. Laboratory results are as follows:

Complete blood count


Hematocrit 26%
Reticulocytes 10%
Platelets 90,000/mm3
Leukocytes 7,500/mm3

Serum chemistry
Creatinine 1.2 mg/dL

Coagulation studies
INR 3

Which of the following is the best next step in management?

A) ADAMTS-13 assay
B) Bone marrow biopsy

C) Colonoscopy

D) Echocardiogram

E) Liver biopsy
Explanation
Correct Answer:

D) Echocardiogram

This patient with a mechanical aortic valve developed symptomatic anemia and reticulocytosis, raising strong
suspicion for traumatic macrovascular hemolysis. Most cases arise in the setting of a dysfunctional mechanical
or severely calcified aortic valve; high pressure gradients across the irregular valve cause shearing of passing
erythrocytes, leading to intravascular hemolysis. Damaged erythrocytes that are not lysed in the intravascular
space are generally destroyed by splenic macrophages, leading to concurrent extravascular hemolysis.

Mechanical aortic valves often cause mild, asymptomatic hemolysis. However, valve deterioration can trigger
progressive symptomatic anemia (eg, fatigue, dyspnea with exertion) with signs of erythrocyte lysis (eg, jaundice,
dark urine) and reticulocytosis. Thrombocytopenia can also occur because platelets may be damaged as they
pass through the rigid valve opening. Hepatosplenomegaly can develop as the fragmented erythrocytes are
cleared in the macrophages and monocytes within the liver and spleen. Peripheral blood smear generally shows
schistocytes (helmet cells). A transthoracic echocardiogram should be performed to visualize valve function and
determine valvular pressure gradients.

(Choice A) ADAMTS-13, a plasma protease that cleaves ultralarge von Willebrand fibers, is deficient in thrombotic
thrombocytopenic purpura (TTP). Patients with TTP frequently have acute (not chronic) anemia due to
microangiopathic hemolysis. Severe thrombocytopenia (<10,000/mm3) is also present due to platelet consumption
in the microvasculature. Most patients also have altered mental status or renal insufficiency.

(Choice B) Bone marrow suppression (eg, tumor infiltration) can cause chronic anemia and thrombocytopenia.
However, significant reticulocytosis is uncommon because the bone marrow cannot efficiently generate cells.

(Choice C) Colon cancer is a common cause of chronic anemia in older individuals who have not undergone
screening colonoscopy. Reticulocytosis would be unusual (it is more common with acute, rather than chronic, blood
loss anemia), and splenomegaly and thrombocytopenia are not typically present.
(Choice E) Cirrhosis can cause hepatosplenomegaly with thrombocytopenia (splenic platelet trapping), as well as
anemia (eg, bleeding varices, chronic inflammation). However, this patient has no risk factors for cirrhosis (eg,
alcohol use, hepatitis); his elevated INR likely reflects therapeutic warfarin use rather than liver disease; and his
mechanical valve makes traumatic hemolysis much more likely. Finally, because liver biopsy is an invasive
procedure, noninvasive tests (eg, echocardiogram evaluating valve function) are generally performed first.

Educational objective:
A mechanical or severely calcified aortic valve can cause hemolytic anemia due to erythrocyte shearing. Patients
also frequently have mildly low platelets due to concurrent mechanical platelet injury. An echocardiogram should be
performed to assess the valvular function.
Question #225

A 65-year-old woman comes to the office due to a month of back pain. The pain is primarily in her lumbar and
thoracic spine and is partially relieved with acetaminophen. It is worse with activity and has limited her ability to
take restorative yoga classes. The patient has a history of hypertension, type 2 diabetes mellitus, and osteopenia.
She takes over-the-counter calcium and vitamin D supplements in addition to prescription medications. The patient
had her yearly physical examination 4 months ago; physical and laboratory examinations were normal at that time.
Temperature is 36.6 C (97.8 F), blood pressure is 148/82 mm Hg, and pulse is 94/min. The patient is thin but
appears well. Mucous membranes are moist. There is no lymphadenopathy in the cervical or supraclavicular
chains. Cardiopulmonary and abdominal examinations are normal. There is no focal tenderness over the spine.
Muscle strength is 5/5 in all 4 extremities. Laboratory results are as follows:

Complete blood count


Hemoglobin 10.2 g/dL
Platelets 220,000/mm3
Leukocytes 8,800/mm3

Serum chemistry
Sodium 138 mEq/L
Potassium 4.2 mEq/L
Chloride 102 mEq/L
Bicarbonate 26 mEq/L
Blood urea nitrogen 30 mg/dL
Creatinine 2.5 mg/dL
Calcium 10.9 mg/dL
Glucose 118 mg/dL
Urine dipstick is negative and urine sediment is bland except for a few granular casts. Which of the following is the
most likely cause of this patient's current condition?

A) Acetaminophen toxicity

B) Diabetes mellitus

C) Monoclonal protein

D) Primary hyperparathyroidism

E) Systemic lupus erythematosus

F) Vitamin D toxicity
Explanation
Correct Answer:

C) Monoclonal protein

Multiple myeloma (MM) is a plasma cell neoplasm. Patients with MM often present with constitutional symptoms
(fatigue, weight loss) or bone pain (back, chest). MM is characterized by the proliferation of neoplastic cells in the
bone marrow (bone destruction, hypercalcemia, anemia) and the excessive production of a single immunoglobulin
(monoclonal protein). The latter can accumulate at high levels in the serum and deposit in the renal tubules. This
can result in renal insufficiency, often with a bland urinalysis (urine dipstick protein typically detects albumin, not
immunoglobulins) that may show evidence of granular casts. The monoclonal protein can also damage the
glomeruli (amyloidosis, monoclonal immunoglobulin deposition disease), leading to nephrotic syndrome.

(Choice A) Acetaminophen toxicity may occur at high doses. However, it typically affects the liver (hepatitis,
jaundice) and would not account for this patient's hypercalcemia or anemia.

(Choice B) Diabetic nephropathy usually develops over many years and is associated with albuminuria. With
normal laboratory studies 4 months ago and a bland urinalysis, this patient is unlikely to have diabetic nephropathy.

(Choice D) Primary hyperparathyroidism can cause hypercalcemia. However, most patients are asymptomatic or
are diagnosed after developing calcium nephrolithiasis. This patient who has hypercalcemia associated with bone
pain and anemia is more likely to have multiple myeloma.

(Choice E) Systemic lupus erythematosus typically causes glomerular injury resulting in proteinuria and hematuria,
often with an increase in creatinine. This patient has a bland urinalysis, making lupus nephritis unlikely.

(Choice F) Vitamin D toxicity tends to occur only at exceedingly high doses and is associated with vomiting,
confusion, polyuria, and polydipsia. Anemia and renal failure would be uncommon.

Educational objective:
Multiple myeloma is a plasma cell neoplasm. It is characterized by the excessive production of monoclonal protein
that can accumulate in the kidneys and cause renal insufficiency. Approximately 50% of patients with multiple
myeloma have renal insufficiency at diagnosis.
Question #226

A 62-year-old man comes to the office due to several months of fatigue. The patient has no chest pain, vomiting,
diarrhea, weight loss, or back pain. Review of systems is significant for occasional leg cramps. He takes lisinopril
for hypertension and atorvastatin for hyperlipidemia. The patient no longer smokes cigarettes but has a 30-pack-
year history. He does not consume alcohol. Blood pressure is 132/80 mm Hg and pulse is 95/min. Conjunctiva are
pale. Cardiopulmonary auscultation reveals clear lungs and a soft 2/6 ejection murmur at the right upper sternal
border. The abdomen is soft and nontender without palpable masses. There are no skin rashes or peripheral
edema. Laboratory results are as follows:

Complete blood count


Hemoglobin 7.7 g/dL
Mean corpuscular volume 72 fL
Platelets 180,000/mm3
Leukocytes 8,200/mm3

Serum chemistry
Blood urea nitrogen 12 mg/dL
Creatinine 0.8 mg/dL

Blood, plasma, and serum


Ferritin, serum 7 ng/mL

Which of the following is the most appropriate next step in management of this patient?

A) Bone marrow biopsy


B) Discontinuation of atorvastatin

C) Endoscopic evaluation

D) Serum C-reactive protein

E) Serum erythropoietin level

F) Serum free monoclonal light chains


Explanation
Correct Answer:

C) Endoscopic evaluation

This patient has clinical findings (fatigue, pallor, ejection murmur) consistent with anemia and laboratory evidence
(low mean corpuscular volume and ferritin) of iron deficiency.

Iron is derived from dietary sources (eg, animal meat) and is mostly absorbed in the duodenum. Body iron is
distributed to red blood cells (hemoglobin), other iron-containing proteins (eg, myoglobin), and ferritin/hemosiderin
in the reticuloendothelial system. In developed countries, iron deficiency rarely stems from dietary or absorption
inadequacies. Most cases are due to overt or occult blood loss.

History and physical examination can often identify an overt source of bleeding (eg, menorrhagia); otherwise,
referral to a gastroenterologist for endoscopic evaluation (eg, colonoscopy, esophagogastroduodenoscopy) is
usually required (particularly in those age >50) as patients may have gastritis, telangiectasia, or malignancy (eg,
colon cancer).

(Choice A) Although bone marrow biopsy is considered the gold standard for assessing iron stores, less invasive
testing (eg, ferritin) has largely supplanted this diagnostic modality. Bone marrow biopsy is used primarily to
evaluate for hematologic malignancies (eg, myelodysplastic syndrome) and stem cell abnormalities (eg, aplastic
anemia).

(Choice B) Atorvastatin may cause hepatotoxicity and myopathy but is not associated with iron deficiency anemia.

(Choice D) C-reactive protein is usually elevated in infection or inflammatory conditions. Patients with these
illnesses often have anemia of chronic disease due to poor iron utilization and impaired erythrocytosis. Anemia of
chronic disease is usually associated with normal or high ferritin levels.

(Choice E) Erythropoietin (EPO) stimulates erythrocytosis. Low levels of EPO can be seen in many chronic
diseases (eg, renal insufficiency). EPO is not often measured, and conditions that result in low EPO are usually
associated with anemia of chronic disease (normal or high ferritin levels).

(Choice F) Monoclonal light chains can be elevated in plasma cell disorders such as multiple myeloma. Although
anemia is common, it is usually normocytic and normochromic (not microcytic).

Educational objective:
Gastrointestinal blood loss is the most common occult cause of iron deficiency anemia, especially in patients age
>50. Referral to a gastroenterologist for endoscopy is usually required.
Question #227

A 56-year-old man comes to the office due to fatigue. He has a 20-year history of diabetes mellitus treated with
daily insulin therapy. The patient also has end-stage renal disease and receives hemodialysis 3 times a week. He
has been receiving erythropoietin therapy for anemia (pretreatment hemoglobin: 8.0 g/dL). Physical examination
shows pale conjunctiva. Laboratory results are as follows:

Hemoglobin 9.2 g/dL


Mean corpuscular volume 77 μm3
Mean corpuscular hemoglobin concentration 30% hemoglobin/cell
White blood cell count 7,000/mm3
Platelets 150,000/mm3
Hemoglobin A1c 7.5%
Erythrocyte sedimentation rate 15 mm/hr

Which of the following is the most appropriate intervention to improve this patient's symptoms?

A) Blood transfusion

B) Change erythropoietin to darbepoetin

C) Folic acid supplementation

D) Iron supplementation

E) Tighter blood glucose control


Explanation
Correct Answer:

D) Iron supplementation

Patients with advanced chronic kidney disease and end-stage renal disease frequently develop a hypoproliferative
(normochromic, normocytic) anemia due to decreased erythropoietin production by the failing kidneys. The primary
treatment is erythropoiesis-stimulating agents (ESAs), such as erythropoietin or darbepoetin, which increase
production of red blood cells in the bone marrow. Goal hemoglobin is typically 10-11.5 g/dL.

However, this patient has a hypochromic (low mean corpuscular hemoglobin concentration) and microcytic (low
mean corpuscular volume) anemia that has not responded to ESA therapy. This suggests concurrent iron
deficiency, which is the most common cause of inadequate response to ESAs in patients on dialysis. Iron stores
can be rapidly depleted during hematopoiesis after ESA administration, especially in chronically ill patients whose
iron stores may already be low. Iron stores should therefore be evaluated prior to starting ESAs and frequently
thereafter. The treatment of choice for iron deficiency in patients on dialysis is intravenous iron.

(Choice A) Blood transfusions are associated with significant complications (eg, volume overload, transfusion
reactions) and should be used only for more severe anemia that cannot be corrected by other means. Iron
supplementation in this patient is likely to restore responsiveness to ESAs and correct his anemia.

(Choice B) Darbepoetin is an ESA with a longer half-life than erythropoietin and can be dosed less frequently;
however, they both have the same effects on hematopoiesis. Both require sufficient iron stores to induce red blood
cell production.

(Choice C) Folic acid supplementation is beneficial for folate deficiency and hereditary spherocytosis. Folate
deficiency causes a macrocytic anemia, not a microcytic anemia as seen in this patient.

(Choice E) Tight blood glucose control decreases the incidence of microvascular complications of diabetes (eg,
retinopathy, neuropathy, nephropathy). Although improved glucose control could reduce this patient's risk of renal
complications, it will not help his anemia.
Educational objective:
Anemia of chronic kidney disease is due to erythropoietin deficiency; treatment is with erythropoiesis-stimulating
agents (ESAs) (eg, erythropoietin, darbepoetin). Iron deficiency is the most common cause of inadequate response
to ESAs in patients on dialysis.

Reference
• Erythropoiesis-stimulating agent hyporesponsiveness in end-stage renal disease patients.
Question #228

A 45-year-old man comes to the office due to several months of easy fatigability and exertional dyspnea. The
patient had a subtotal gastrectomy for a nonhealing gastric ulcer 5 years ago. He takes daily iron supplementation.
Blood pressure is 120/70 mm Hg, pulse is 80/min, and respirations are 16/min. Physical examination shows a shiny
tongue and pale palmar creases. No lymphadenopathy, hepatomegaly, or splenomegaly is present. Laboratory
results are as follows:

Hemoglobin 7.8 g/dL


Leukocytes 3,800/µL
Bilirubin, total 2.3 mg/dL
Bilirubin, direct 0.4 mg/dL
Alkaline phosphatase 20 U/L
Aspartate aminotransferase 12 U/L
Alanine aminotransferase 24 U/L
Lactate dehydrogenase, serum 190 U/L

Stool tests for occult blood are repeatedly negative. This patient's condition involves which of the following
pathophysiologic mechanisms?

A) Immune-related red blood cell injury

B) Impaired DNA synthesis

C) Impaired globin chain synthesis

D) Impaired glutathione synthesis


E) Mechanical red blood cell injury

F) Red blood cell membrane instability


Explanation
Correct Answer:

B) Impaired DNA synthesis

This patient, with chronic anemia following subtotal gastrectomy, has characteristic features of vitamin B12
deficiency. Loss of intrinsic factor, whether due to gastric resection or autoimmune gastritis, is a common cause
of B12 deficiency. Vitamin B12 is a required cofactor for the formation of thymidylate and purine molecules for DNA
synthesis. B12 deficiency therefore causes ineffective erythropoiesis due to delayed nuclear maturation, resulting in
decreased transition to mature red blood cell (RBC) forms and high numbers of immature megaloblasts in the bone
marrow. Increased intramedullary hemolysis of these megaloblasts releases heme, causing indirect
hyperbilirubinemia, which may manifest as jaundice. Hemolysis also releases the intracellular enzyme lactate
dehydrogenase (LDH), raising serum levels. Total RBC count and reticulocyte count will be low. Patients may also
develop thrombocytopenia and leukopenia with hypersegmented polymorphonuclear cells.

(Choices A, D, E, and F) Hyperbilirubinemia and high LDH may be seen in any cause of hemolysis, such as the
following:

• Autoimmune hemolytic anemia


• Glucose-6-phosphate dehydrogenase deficiency due to impaired glutathione synthesis
• Mechanical injury to RBCs (artificial heart valves/foreign material within vasculature)
• Hereditary spherocytosis (increased membrane fragility)

However, history of gastrectomy and presence of glossitis are more suggestive of B12 deficiency in this patient.
Glossitis may also be seen in deficiency of other micronutrients such as riboflavin, niacin, folic acid, or iron.

(Choice C) Defective globin chain synthesis is seen in diseases such as sickle cell anemia and α- and β-
thalassemia.

Educational objective:
Vitamin B12 deficiency is common after a total or partial gastrectomy. Vitamin B12 is a necessary cofactor in purine
synthesis, and its deficiency causes defective DNA synthesis. This results in ineffective erythropoiesis, presenting
as megaloblastic anemia.

Reference
• Folate (vitamin B9) and vitamin B12 and their function in the maintenance of nuclear and mitochondrial
genome integrity.

• New insights into erythropoiesis: the roles of folate, vitamin B12, and iron.
Question #229

A 55-year-old man comes to the office for an annual visit. Medical history includes hypertension and
hypercholesterolemia, for which he takes amlodipine and atorvastatin. He is adherent to his medication regimen.
The patient feels well in general, but over the past few months, he has had difficulty achieving and maintaining an
erection, despite having a normal libido. He drinks 2 or 3 glasses of wine daily and does not smoke. He works for a
lawn care company, frequently driving the grass cutter. Blood pressure is 160/100 mm Hg and pulse is 80/min. BMI
is 45 kg/m2. Cardiopulmonary examination is normal. The abdomen is soft and nontender. The liver and spleen
are not palpable. Laboratory results are as follows:

Complete blood count


Hemoglobin 19 g/dL
Hematocrit 57%
Platelets 240,000/mm3
Leukocytes 5,100/mm3

Serum chemistry
Blood urea nitrogen 14 mg/dL
Creatinine 0.9 mg/dL

Which of the following is most likely responsible for this patient's increased hematocrit?

A) Carboxyhemoglobinemia

B) Clonal proliferation of myeloid cells

C) Increased erythropoietin production


D) Plasma volume contraction

E) Testosterone deficiency
Explanation
Correct Answer:

C) Increased erythropoietin production

Obstructive sleep apnea

• Relaxation of pharyngeal muscles, leading to closure of airway


Pathophysiology
• Loud snoring with periods of apnea

• Daytime somnolence
• Nonrestorative sleep with frequent awakenings
Symptoms
• Morning headaches
• Affective & cognitive symptoms

• Systemic hypertension
Sequelae
• Pulmonary hypertension & right-sided heart failure

This patient's presentation is most consistent with obstructive sleep apnea (OSA). Obesity (particularly BMI >35
kg/m2) is the strongest risk factor for OSA. Not all patients have obvious respiratory symptoms (eg, snoring,
gasping for air); the sequelae of OSA (eg, erectile dysfunction, arterial hypertension) may be what prompts the
initial evaluation.

In patients with OSA, recurrent collapse of the pharynx during sleep results in transient airway obstruction. This
causes short periods (20-40 seconds) of apnea or hypopnea, which reduce blood oxygen levels. The kidneys
respond to the hypoxemia by increasing erythropoietin, which stimulates the bone marrow to differentiate more
red blood cells (RBCs). Therefore, it is quite common for patients with OSA to have elevated hematocrit levels
(polycythemia).

The terms polycythemia and erythrocytosis are often used interchangeably. Polycythemia is a laboratory finding of
elevated RBC count and hematocrit. Relative polycythemia is generally due to reduced plasma volume, whereas
absolute polycythemia (ie, erythrocytosis) is due to increased RBC mass and can be primary (eg, polycythemia
vera) or secondary (eg, due to chronic hypoxia or erythropoietin-producing tumors).

(Choice A) Carbon monoxide (CO) poisoning increases carboxyhemoglobin levels, shifting the oxygen dissociation
curve to the left and resulting in tissue hypoxia. The kidneys respond by increasing erythropoietin, which may
cause erythrocytosis. However, patients with CO poisoning typically have dizziness, headaches, nausea, and
confusion. This patient has obesity, hypertension, and erectile dysfunction, making OSA far more likely.

(Choice B) The clonal proliferation of myeloid cells can cause erythrocytosis in polycythemia vera. Patients are
often asymptomatic, but signs of increased blood viscosity (eg, transient neurologic symptoms, thrombosis),
aquagenic pruritus, and gout are common.

(Choice D) Significant plasma volume contraction causes relative polycythemia because RBCs are concentrated
due to lower intravascular volume. This patient's normal blood urea nitrogen and creatinine levels make significant
volume contraction very unlikely.

(Choice E) Testosterone deficiency can result in erectile dysfunction. However, erythrocytosis can be caused by
testosterone excess (eg, androgen use) rather than testosterone deficiency. OSA is an independent risk factor for
erectile dysfunction, and treatment with continuous positive airway pressure may improve sexual function. Notably,
testosterone supplementation (even to normal levels) can worsen OSA.

Educational objective:
Obstructive sleep apnea (OSA) can present without symptoms of snoring or witnessed apneic events. OSA causes
transient periods of hypoxemia. The kidneys respond by increasing erythropoietin, which can result in
erythrocytosis.

Reference
• The effect of CPAP and PDE5i on erectile function in men with obstructive sleep apnea and erectile
dysfunction: a systematic review and meta-analysis.
Question #230

A 21-year-old man comes to the office due to recent irritability and aggressive behavior. He is a star college football
linebacker and is training for his senior season. The patient does not use tobacco or alcohol. His mother has
diabetes and his father has basal cell skin cancer. Blood pressure is 132/84 mm Hg and pulse is 62/min. The
patient is muscular and well appearing. There is no lymphadenopathy in the cervical or supraclavicular chains.
Cardiopulmonary examination is normal. Mild gynecomastia is present. The abdomen is soft and nontender. Liver
span is 8 cm, and the spleen is not palpable. Laboratory results are as follows:

Hematocrit 58%
175,000/
Platelets
mm3
7800/
Leukocytes
mm3
Erythrocyte
15 mm/h
sedimentation rate

Which of the following is the best single explanation for this patient's presentation?

A) Anabolic steroid abuse

B) Autologous blood transfusion

C) Erythropoietin abuse

D) Intensive exercise schedule

E) Polycythemia vera
Explanation
Correct Answer:

A) Anabolic steroid abuse

Clinical features of androgen abuse

• Exogenous (eg, testosterone replacement therapy)


Types of androgens • Synthetic (eg, stanozolol, nandrolone)
• Androgen precursors (eg, DHEA)

• Reproductive
◦ Men: decreased testicular function & sperm production, gynecomastia
Side effects/ ◦ Women: acne, hirsutism, voice deepening, menstrual irregularities
clinical presentation • Cardiovascular: left ventricular hypertrophy, possible ↓ HDL & ↑ LDL
• Psychiatric: aggressive behavior (men), mood disturbances
• Hematologic: polycythemia, possible hypercoagulability

DHEA = dehydroepiandrostenedione.

Androgen abuse is common amongst high-performance athletes. Androgens are anabolic; they increase muscle
mass and maximum voluntary strength. They are taken directly (natural or synthetic androgens) or induced to rise
indirectly (estrogen blockers, androgen precursors, gonadotropins). Adverse effects are common. In men,
androgen abuse can cause testicular atrophy, reduced spermatogenesis, gynecomastia, mood disturbances, and
aggressive behavior. In women, androgens can cause acne, hirsutism, deepening of the voice, and menstrual
irregularities. Laboratory findings include erythrocytosis, hepatotoxicity, and dyslipidemia. The mechanism for
erythrocytosis in androgen abuse remains unclear, but increases in hematocrit and hemoglobin are dose
dependent. This patient is likely abusing androgens in preparation for his upcoming football season.

(Choice B) An autologous blood transfusion involves removing blood several weeks before a competitive event
and then transfusing the blood back into the athlete prior to the event. The net effect is increased hematocrit,
oxygen-carrying capacity, and performance. However, this process typically does not cause gynecomastia or mood
changes, as seen in this patient.

(Choice C) Self-injection of erythropoietin (EPO) stimulates the bone marrow to produce more red blood cells,
thereby increasing the hematocrit. Gynecomastia, aggression, and mood changes are atypical in EPO abuse; they
are much more common with androgens. EPO is primarily abused by athletes participating in endurance sports.

(Choice D) An intensive exercise schedule usually increases muscle mass, improves muscle function, and
enhances exercise capacity. Erythrocytosis is not a typical consequence of a strenuous exercise regimen.

(Choice E) Polycythemia vera (PV) is a chronic myeloproliferative disorder that causes erythrocytosis. It is often
asymptomatic, but aquagenic pruritus (itching after a warm bath or shower), bleeding, transient neurologic
symptoms, and erythromelalgia (burning cyanosis of the hands or feet) are common presenting symptoms.
Gynecomastia, irritability, and aggression are more suggestive of androgen abuse than PV.

Educational objective:
Competitive athletes using exogenous androgens, autologous blood transfusions, or erythropoietin to enhance
athletic performance can develop an elevated hematocrit. Clinical findings suggesting exogenous androgens
include gynecomastia, testicular atrophy, mood disturbances, and hepatotoxicity.

Reference
• Androgen abuse in athletes: detection and consequences.
Question #231

A 25-year-old African American woman comes to the office after experiencing increasingly heavy menstrual cycles
for 3 months. The patient has also noticed that her gums frequently bleed when she brushes her teeth. Review of
systems is positive for ongoing pain and stiffness in her hands and wrists that is partially responsive to ibuprofen.
She is not taking any medications and has no known medical conditions. The patient is afebrile, blood pressure is
149/79 mm Hg, and pulse is 87/min. A nonpainful oral ulcer is present. Multiple petechiae are present on the upper
arms and shins. The wrists and hands are tender to palpation and demonstrate reduced range of motion.
Laboratory results are as follows:

Hemoglobin 10.8 g/dL


Platelets 40,000/mm3
Leukocytes 2,500/mm3
Serum creatinine 1.7 mg/dL

Which of the following is the most likely cause of thrombocytopenia in this patient?

A) Aberrant splenic sequestration

B) Bone marrow invasion by malignancy

C) Dilutional cytopenia

D) Ineffective hematopoiesis

E) Peripheral destruction
Explanation
Correct Answer:

E) Peripheral destruction

Hematologic manifestations of SLE

Common mechanisms Uncommon mechanisms

• Anemia of chronic disease


• Medications
• SLE nephritis
Anemia • MAHA
• Iron deficiency anemia (gastrointestinal loss)
• Aplastic anemia
• Immune hemolytic anemia

• Medications
Leukopenia • Immune-mediated destruction
• Bone marrow dysfunction

• Medications
Thrombocytopenia • Immune-mediated destruction
• TTP (↑ consumption)

MAHA = microangiopathic autoimmune hemolytic anemia; SLE = systemic lupus erythematosus; TTP =
thrombotic thrombocytopenic purpura.
This patient's presentation (joint pains, oral ulcer, hypertension, pancytopenia, impaired renal function) suggests
possible systemic lupus erythematosus (SLE), a systemic autoimmune disorder. Her severe thrombocytopenia
(platelet count <50,000/mm3), manifesting with common associated features of bleeding gums and heavy menstrual
periods, is likely due to SLE. In the United States, SLE occurs most commonly in African American, Hispanic, and
Asian women. Hematologic abnormalities occur in over half of SLE patients and include:

• Anemia, resulting from immune destruction, but also from coexisting renal disease or chronic inflammation
• Leukopenia (eg, neutropenia), often immune-mediated and correlating with disease activity
• Thrombocytopenia, most commonly due to immune thrombocytopenic purpura (ITP)

The presence of autoantibodies is the primary mechanism underlying the most common cytopenias in SLE.
Thrombocytopenia is most often caused by antiplatelet antibodies, which trigger increased destruction of
platelets via immunoglobulin binding in peripheral blood vessels and subsequent macrophage phagocytosis. Less
commonly, thrombocytopenia can occur in SLE-associated hypercoagulable states in which platelets are consumed
within thrombi (eg, thrombotic thrombocytopenic purpura, associated with schistocytes; disseminated intravascular
coagulation, which can occur in antiphospholipid antibody syndrome).

(Choice A) Although splenic sequestration of immunoglobulin-bound platelets may occur downstream, the primary
initiating mechanism for platelet destruction in SLE occurs in peripheral blood vessels via the abnormal formation of
immune complexes. In contrast, aberrant splenic sequestration occurs in conditions that increase portal pressure
and splenic congestion, such as liver disease, hepatic vein thrombosis, or sickle cell anemia. Typically, the
thrombocytopenia associated with these conditions is mild (60,000-150,000/mm3).

(Choices B and D) Malignant invasion of bone marrow, seen in leukemias and lymphomas, may cause
pancytopenia. Similarly, pancytopenia can be caused by ineffective hematopoiesis resulting from primary bone
marrow disorders (eg, myelodysplastic syndrome, aplastic anemia). However, these conditions are unlikely to
explain this patient's new kidney disease, hypertension, nonpainful oral ulcer, and arthralgias.

(Choice C) Dilutional thrombocytopenia can occur following massive transfusion of packed red blood cells or large-
volume intravenous fluid resuscitation, in which plasma volume is increased disproportionately to platelet count.
This patient has no history of either procedure.

Educational objective:
Pancytopenia (decreased red blood cells, white blood cells, and platelets) is common in patients with systemic
lupus erythematosus. It usually indicates concurrent peripheral, immune-mediated destruction of all 3 cell lines.

Reference
• Hematological manifestations of systemic lupus erythematosus.
Question #232

A 62-year-old man comes to the office for evaluation of a lump in his neck. The patient first noticed the lump under
the right side of his jaw about 4 months ago when he cut himself while shaving. The lump is slowly getting larger.
He has also had occasional deep-seated right ear pain. The patient has had no fevers, chills, cough, or shortness
of breath. He has had no change in his diet and no weight loss. Medical history is significant for hypertension and
type 2 diabetes mellitus. The patient smokes 2 packs of cigarettes daily and uses alcohol occasionally. Physical
examination shows a firm, nontender, right-sided submandibular mass that is 3 cm in diameter. Ear examination is
normal. There are no abnormal skin lesions. Oral cavity examination shows poor dentition and no mucosal
lesions. The tonsils are small and soft with no lesions. Chest examination is unremarkable. The abdomen is soft
and nontender with no hepatosplenomegaly. There is no other lymphadenopathy. Complete blood count is within
normal limits. Which of the following is the most likely cause of this patient's condition?

A) Chronic lymphocytic leukemia

B) Granulomatous polyangiitis

C) Hodgkin lymphoma

D) Infectious mononucleosis

E) Medullary thyroid carcinoma

F) Mycobacterial lymphadenitis

G) Squamous cell carcinoma


Explanation
Correct Answer:

G) Squamous cell carcinoma

Malignancy is extremely likely in a patient with a persistent (>2 weeks), palpable (>1.5 cm), firm neck mass; a
smoking history; and no preceding infection. By far the most common malignancy in an upper cervical node is
mucosal head and neck squamous cell carcinoma (SCC). Indeed, the first (and only) apparent manifestation
may be a palpable cervical lymph node, representing regional nodal metastasis. Referred otalgia is another
common presenting symptom, facilitated by either the glossopharyngeal nerve (CN IX) (innervates both the base of
tongue and the external auditory canal [EAC]) or the vagus nerve (CN X; innervates parts of the larynx/hypopharynx
and the EAC).

Identification of the primary source of head and neck SCC is essential to direct treatment. Thorough examination
includes endoscopic visualization using laryngopharyngoscopy as well as neck imaging (CT with contrast) to
evaluate the primary site and characterize the cervical nodal disease. Fine-needle aspiration of the lymph node is
advised over open biopsy to avoid tumor seeding.

(Choice A) Chronic lymphocytic leukemia (CLL) may present in older patients with painless cervical
lymphadenopathy. However, it typically causes waxing and waning lymphadenopathy. It would not cause ear pain,
and leukocytosis would also be expected.

(Choice B) Granulomatous polyangiitis can present with head and neck symptoms, including ear pain and
subcutaneous nodules. However, it is a systemic disease, often with life-threatening renal (eg, glomerulonephritis)
and pulmonary (eg, nodules, hemorrhage) manifestations. A solitary lymph node in the neck with no constitutional
symptoms would be unlikely.

(Choice C) Hodgkin lymphoma can also cause painless, cervical lymphadenopathy. However, many patients have
constitutional "B symptoms" (eg, fever, night sweats, weight loss). Hodgkin lymphoma would not explain the ear
pain in this patient, and head and neck SCC is much more common in adult smokers.
(Choice D) Infectious mononucleosis can cause adenopathy, but it is typically bilateral with multiple lymph nodes
involved. Additionally, it is typically seen in adolescents or young adults and presents with fever, pharyngitis, and
fatigue.

(Choice E) Medullary thyroid carcinoma (MTC) is a rare neuroendocrine carcinoma of the thyroid parafollicular C
cells that can be sporadic or associated with a germline RET mutation (multiple endocrine neoplasia type 2).
Although MTC can metastasize to lateral neck nodes, a thyroid nodule would also be expected.

(Choice F) Mycobacterial lymphadenitis, often caused by Mycobacterium avium complex, presents as a solitary,
enlarging cervical lymph node. However, it typically presents in children age 1-5 years, and the lymph node
becomes fluctuant with an overlying violaceous skin discoloration. Tuberculous lymphadenitis is unlikely in a patient
with no risk factors. Cervical lymphadenitis is unlikely to cause ear pain.

Educational objective:
Nontender, solitary cervical lymph nodes are concerning for mucosal head and neck squamous cell carcinoma,
especially in an adult patient with a smoking history.

Reference
• Clinical practice guideline: evaluation of the neck mass in adults.
Question #233

A 28-year-old man comes to the office for follow-up. Two years ago, he was diagnosed with early-stage nodular
sclerosis Hodgkin lymphoma after a chest x-ray revealed a mediastinal mass. The patient underwent combination
chemotherapy and radiation treatment. Follow-up imaging has revealed no evidence of disease, and he currently
feels well. The patient has no other medical conditions and takes no medications. He does not use tobacco,
alcohol, or illicit drugs. Vital signs and physical examination are normal. Compared to the general population, this
patient is at greatest risk for which of the following?

A) Diabetes mellitus

B) Osteoporosis

C) Primary sclerosing cholangitis

D) Rheumatoid arthritis

E) Solid organ malignancy


Explanation
Correct Answer:

E) Solid organ malignancy

Patients with Hodgkin lymphoma (HL) are generally treated with combination chemotherapy and radiation
therapy, which cures >75% of cases. However, these treatments are associated with serious long-term
complications, including:

• Secondary malignancy: Radiation exposure increases the risk of solid organ (eg, breast, lung)
malignancy, and chemotherapy increases the risk of hematologic malignancy. Most cases arise >5 years
after treatment. Secondary malignancies are the leading cause of death in those who have been cured of
HL.
• Cardiovascular disease: Risk of coronary artery disease, valve damage, peripheral vascular disease, and
cardiomyopathy are increased in those who have been treated for HL. Cardiovascular disease is the
leading nonmalignant cause of death in long-term HL survivors.

Other common treatment-related complications include pulmonary disease (eg, fibrosis, bronchiectasis) and
hypothyroidism from chest radiation and neuropathy from chemotherapy.

(Choice A) Survivors of HL do not have an increased risk of diabetes mellitus (DM). However, patients who have
concomitant DM are at increased risk for cardiovascular complications of treatment.

(Choice B) Early osteoporosis is often seen in those who take glucocorticoids, anticonvulsants, and
antidepressants; risk is also increased in those who have disorders of calcium metabolism due to inflammatory
bowel disease, celiac disease, hyperthyroidism, and hyperparathyroidism. It is not linked to treatment for HL.

(Choice C) Primary sclerosing cholangitis often occurs in patients with ulcerative colitis (and to a lesser extent,
Crohn disease). It is not linked to treatment for HL.

(Choice D) Rheumatoid arthritis can increase the risk of developing lymphoma due to chronic immune activation
and treatment with alkylating agents (eg, azathioprine, methotrexate). However, lymphoma does not increase the
risk of developing rheumatoid arthritis.

Educational objective:
Hodgkin lymphoma is generally treated with combination chemotherapy and radiation therapy. Although >75% of
patients are cured, treatment can lead to long-term complications, most notably secondary malignancy (eg, solid
organ, hematologic) and cardiovascular disease.
Question #234

A 66-year-old man comes to the office due to increasing pain in his left arm. The patient works as a furniture
transporter and initially attributed the pain to a muscular strain, but it has slowly worsened and stopped responding
to over-the-counter pain medications. The pain started 3 months ago and is mainly near the left elbow. He also
notes back pain. He has not had fever or chills. His other medical problems include hypertension and
hypercholesterolemia. The patient has never smoked. Temperature is 36.8 C (98.2 F), blood pressure is 132/86
mm Hg, pulse is 88/min, and respirations are 14/min. He appears comfortable. His mucous membranes are pale.
There is no lymphadenopathy. Cardiopulmonary examination is normal. The abdomen is soft without
organomegaly. The prostate is smooth, nontender, and moderately enlarged. Neurologic examination is
unremarkable. Laboratory results are as follows:

Complete blood count


Hemoglobin 9.8 g/dL
Mean corpuscular volume 89 µm3
Platelets 254,000/mm3
Leukocytes 6,800/mm3
Neutrophils 60%
Lymphocytes 32%
Monocytes 8%

X-ray of the left arm is shown in the exhibit. Which of the following tests would be the most useful next step in the
management of this patient?

A) Blood cultures

B) Bone marrow biopsy


C) Bone scan

D) Prostate biopsy

E) Serum parathyroid hormone levels

F) Serum protein electrophoresis


Explanation
Correct Answer:

F) Serum protein electrophoresis


This patient has back pain, normocytic anemia, and arm pain with an x-ray showing multiple osteolytic (lucent)
lesions. In an elderly patient, this raises suspicion for multiple myeloma (MM).
MM is a plasma cell neoplasm. Pathology stems from bone marrow infiltration (fractures, hypercalcemia, anemia)
and monoclonal protein production (renal insufficiency). Serum protein electrophoresis (SPEP) is a common
screening test for MM that detects elevated serum monoclonal protein (M-spike). Bone marrow biopsy, a more
invasive procedure, can then confirm diagnosis (>10% clonal plasma cells).

(Choice A) Blood cultures are useful for diagnosing infections. This patient has symptoms of bone pain and
evidence of osteolytic lesions. He has not had fevers, chills, or other systemic symptoms. Infection is much less
likely than malignancy.

(Choice B) Bone marrow biopsy is an invasive test and performed only after gathering more information with
peripheral smear (rouleaux), serum free light chain analysis, SPEP, and urine protein electrophoresis.

(Choice C) Bone scans are not useful in the diagnosis of MM as they detect only osteoblastic activity (as is often
seen in metastatic solid tumors). In MM, the lesions are osteoclastic (lytic), so whole body cross-sectional bone
imaging (eg, low dose CT scan), is needed to identify bone involvement.

(Choice D) Prostate cancer can cause bone lesions, but they tend to be osteoblastic (sclerotic) not osteolytic. In a
patient with osteolytic lesions alone, MM is far more likely.

(Choice E) Hyperparathyroidism can cause osteopenia with subsequent fractures but is unlikely to cause osteolytic
lesions. Osteitis fibrosa cystica (eg, brown tumors of the long bones) is associated with hyperparathyroidism;
however, it is very rare in the United States, lesions are typically well-defined (vs moth-eaten appearance), and
other radiographic abnormalities are common (eg, subperiosteal bone resorption of the middle phalanx, highly
specific for hyperparathyroidism). MM is far more common and more likely.

Educational objective:
Bone pain is a common presenting symptom in multiple myeloma (MM). In an elderly patient with evidence of
osteolytic lesions on x-ray, MM should always be suspected. The screening tests of choice for MM are serum
protein electrophoresis, urine protein electrophoresis, and free light chain analysis. Diagnosis can be confirmed
with bone marrow biopsy.
Question #235

A 62-year-old man comes to the office due to an abnormal chest x-ray. Two weeks ago, the patient was seen at an
emergency department after a motor vehicle collision. Evaluation showed no significant trauma, but a chest x-ray
revealed enlarged mediastinal lymph nodes. The patient has had no fever, cough, or dyspnea but has had
nonpainful nodules in his axillae for the past several months. Medical history includes hypertension, for which he
takes chlorthalidone. He is lifelong nonsmoker and drinks alcohol occasionally. Vital signs are within normal limits.
Physical examination reveals clear lungs and normal heart sounds. Bilateral axillary and inguinal lymph nodes are
enlarged and nontender. There is no rash or extremity edema. Blood cell counts and serum chemistry studies are
within normal limits. A chest x-ray from a year ago also shows mediastinal lymphadenopathy. Which of the
following is the most likely underlying cause of this patient's current condition?

A) Antinuclear antibody formation

B) BCL-2 gene overexpression

C) Disseminated fungal infection

D) Drug hypersensitivity reaction

E) Metastasis of a lung neoplasm


Explanation
Correct Answer:

B) BCL-2 gene overexpression

This patient's long-standing lymphadenopathy raises suspicion for follicular lymphoma, a common form of non-
Hodgkin lymphoma (NHL) that often presents in elderly patients. Most cases present in an indolent fashion with
months or years of painless peripheral lymphadenopathy in the cervical, axillary, or inguinal region. Although
hilar and mediastinal lymphadenopathy occurs, large mediastinal masses are rare. The lymphadenopathy can wax
and wane. B symptoms (eg, night sweats, fever, weight loss) and laboratory abnormalities (eg, elevated lactate
dehydrogenase, cytopenia) are generally absent.

Follicular lymphoma is usually diagnosed with excisional lymph node biopsy. Histopathology typically shows
nodular growth of follicular lymphocytes, and immunophenotyping most often reveals (in ~85% of cases) a
translocation between chromosomes 14 and 18 that leads to the overexpression of BCL-2, an oncogene that
prevents apoptosis.

(Choice A) Antinuclear antibody is a diagnostic marker for systemic lupus erythematosus (SLE). Although patients
with uncontrolled SLE sometimes have peripheral lymphadenopathy, other diagnostic manifestations (eg, arthralgia,
rash, serositis, cytopenia) are usually present.

(Choice C) Disseminated histoplasmosis can cause peripheral lymphadenopathy, but almost all cases are
accompanied by fever, fatigue, weight loss, and laboratory abnormalities (eg, transaminase elevation, anemia of
chronic disease).

(Choice D) Certain drugs (eg, phenytoin) can cause painless peripheral lymphadenopathy without other
symptoms. However, chlorthalidone is not a common culprit.

(Choice E) Lung cancer can cause hilar and mediastinal lymphadenopathy, but chest x-ray would likely show a
pulmonary mass. In addition, progressive symptoms (eg, worsened chest x-ray, weight loss, pulmonary
manifestations) would be expected over the course of a year. Furthermore, spread to the inguinal lymph nodes
would be atypical.

Educational objective:
Follicular lymphoma is a common form of non-Hodgkin lymphoma. It generally presents in an older patient with
months or years of waxing and waning lymphadenopathy. B symptoms and laboratory abnormalities are rare, but
mediastinal/hilar lymphadenopathy is sometimes seen.
Question #236

A 50-year-old man comes to the office due to feeling fatigued and weak for a year. He becomes tired early in the
day and also finds it difficult to remember things. He is currently unemployed and lives on Social Security benefits.
The patient says that he drinks 2-3 beers a day but has been charged multiple times with driving under the
influence. His parents died of old age. He lives with 4 friends. Vital signs are within normal limits. Laboratory
results are as follows:

Leukocytes 5,500/mm3
Hemoglobin 7.0 g/dL
Hematocrit 22%
Platelets 196,000/mm3
1.7 million cells/
Red blood cells
µm3
Mean corpuscular volume 114 µm3
Mean corpuscular hemoglobin
36 pg/cell
(MCH)
Reticulocytes 0.4%

Peripheral smear shows anisocytosis, poikilocytosis, and basophilic stippling. Which of the following is the best
next step in management of this patient?

A) Blood lead levels

B) Bone marrow biopsy

C) Direct antiglobulin test


D) Iron studies

E) Serum B12 and folate levels


Explanation
Correct Answer:

E) Serum B12 and folate levels

Common causes of macrocytic anemia*

• Folate deficiency
• Vitamin B12 deficiency
• Myelodysplastic syndromes
• Acute myeloid leukemias
• Drug-induced (eg, hydroxyurea, zidovudine, chemotherapy agents)
• Liver disease
• Alcohol abuse
• Hypothyroidism

*Mean corpuscular volume >100 μm3.

This patient's presentation is consistent with macrocytic anemia, which is usually suggested by abnormal red
blood cell (RBC) indices, including an elevated (ie, >100 µm3) mean corpuscular volume (MCV), elevated mean
corpuscular hemoglobin (amount of hemoglobin in cell), and normal mean corpuscular hemoglobin concentration
(low values are seen with microcytosis, elevated values are only seen with spherocytosis or some hemolytic
anemias). Macrocytic anemia is due to 2 primary etiologies:

• Megaloblastic (eg, impaired DNA synthesis due to B12 or folate deficiency): Typically associated with MCV
>110 µm3, low reticulocyte count, macroovalocyte RBCs, hypersegmented neutrophils (mean lobe count
>5), anisocytosis, and poikilocytosis.

• Nonmegaloblastic (eg, alcohol use, hypothyroidism, drugs, liver disease): Associated with increased,
normal, or decreased reticulocyte count depending on the etiology.

Therefore, the peripheral blood smear and reticulocyte count are helpful in distinguishing between the 2 etiologies.

This patient's smear findings and MCV >110 µm3 favor megaloblastic anemia. Measuring B12 and folate levels
can aid diagnosis and distinguish between B12 and folate deficiency. Patients with low levels usually require further
testing to elucidate the underlying disorder. Pernicious anemia is the most important cause of B12 deficiency, and
patients may also develop neurologic deficits (eg, memory loss) and achlorhydria.

(Choice A) Lead poisoning can be associated with basophilic stippling (ribosomal precipitates that form varying
sizes of blue granules in the RBC cytoplasm); however, lead poisoning would cause microcytic (MCV <80 µm3)
rather than macrocytic anemia. Basophilic stippling is nonspecific and can also be due to thalassemias or to
alcohol use (as is likely in this patient).

(Choice B) Bone marrow biopsy is helpful for diagnosing myelodysplasia, which usually causes cytopenias in ≥1
cell lines. Although patients can develop macrocytosis, bone marrow biopsy is usually reserved for evaluating
patients with normal B12 and folate levels.

(Choice C) Direct antiglobulin test can help diagnose autoimmune hemolytic anemias. Patients typically have
hemoglobin of 7-10 g/dL with elevated reticulocyte count, bilirubin, and lactate dehydrogenase. The haptoglobin is
also markedly decreased, and peripheral smear usually shows spherocytes. However, it would not cause the
significantly elevated MCV seen in this patient.

(Choice D) Measuring iron levels is useful for evaluating patients with microcytic anemia (MCV <80 µm3).

Educational objective:
Macrocytic anemia can be due to megaloblastic (eg, impaired DNA synthesis due to B12 or folate deficiency) or
nonmegaloblastic (eg, alcohol use, hypothyroidism, drugs, liver disease) etiologies. Peripheral smear, reticulocyte
count, and B12 and folate levels can aid diagnosis and distinguish between B12 and folate deficiency.
Reference
• Macrocytic anemia.
Question #237

A 32-year-old Italian-American man comes to the office for a routine checkup. The patient is a business executive
and has been under significant stress recently. He drinks alcohol occasionally and has a 10-pack-year smoking
history. Laboratory results are as follows:

Hemoglobin 10.8 g/dL


Red blood cell count 5.7 million cells/mm3
Mean corpuscular volume 61 µm3
Leukocytes 5,500/mm3
Platelets 170,000/mm3

Serial fecal occult blood tests are negative. Peripheral blood smear is shown in the image below.
Which of the following is the best treatment for this patient?

A) Cobalamin

B) Deferoxamine
C) Erythropoietin

D) Folic acid

E) Hydroxyurea

F) Iron

G) Prednisone

H) Reassurance with no specific therapy required

I) Splenectomy
Explanation
Correct Answer:

H) Reassurance with no specific therapy required

Iron deficiency anemia & thalassemias

Iron
Parameter deficiency Alpha-thalassemia minor Beta-thalassemia minor
anemia

MCV ↓ ↓ ↓

RDW ↑ Normal Normal

RBCs ↓ Normal Normal

Peripheral Microcytosis,
Target cells Target cells
smear hypochromia

Serum iron ↓ Iron & ferritin Normal/↑ iron & ferritin (RBC Normal/↑ iron & ferritin (RBC
studies ↑ TIBC turnover) turnover)

Response to iron
↑ Hemoglobin No improvement No improvement
supplementation

Hemoglobin
Normal Normal ↑ Hemoglobin A2
electrophoresis

MCV = mean corpuscular volume; RBCs = red blood cells; RDW = red blood cell distribution width; TIBC = total
iron-binding capacity.

Microcytic anemia (eg, mean corpuscular volume [MCV] <80 µm3) is most often due to iron deficiency or
thalassemia minor. Although both can cause hypochromia and target cells, significant microcytosis in the setting
of mild anemia and a normal (rather than low) erythrocyte count suggests thalassemia (abnormal alpha- or beta-
globin synthesis).

Thalassemia is classified based on the number and type of globin chains affected. Mutations to 3 or 4 globin genes
are usually incompatible with life or require lifelong blood transfusions. In contrast, patients with defects in 1 or 2 of
the alpha-globin chains (alpha-thalassemia minima or minor) or 1 beta-globin chain (beta-thalassemia minor) are
usually asymptomatic. Patients of Mediterranean descent (as in this case) are most likely to have beta-
thalassemia, whereas patients from Southeast Asia are more likely to have alpha-thalassemia; electrophoresis can
confirm the diagnosis.

Laboratory findings in thalassemia minor typically demonstrate mild hematocrit reduction (>28%), significantly low
MCV (55-75 µm3), and disproportionally elevated red blood cell (RBC) counts. The disproportionately elevated
RBC count (eg, normal or high rather than low) and decreased MCV help distinguish thalassemia minor from iron
deficiency anemia, which does not typically cause microcytosis until hemoglobin is <10 g/dL (Choice F). Patients
with thalassemia minor show no improvement in hemoglobin with iron supplementation and do not require
specific treatment.

(Choices A and D) Cobalamin (vitamin B12) and folic acid deficiency cause macrocytosis rather than microcytosis.

(Choices B and E) Deferoxamine is used to treat iron overload in those with transfusion-dependent thalassemia
major or sickle cell disease (eg, sickle cells on smear). Hydroxyurea decreases the frequency and severity of pain
crises in patients with sickle cell anemia by increasing hemoglobin F levels.

(Choice C) Erythropoietin (EPO) can be used to treat anemia associated with chronic kidney disease or transient
bone marrow failure after chemotherapy. EPO level elevations may occur in thalassemia (in response to anemia)
but cannot compensate for the hemoglobin defect.

(Choices G and I) Prednisone is the treatment of choice for autoimmune hemolytic anemia (eg, spherocytes on
smear). Splenectomy can reduce erythrocyte or platelet sequestration in certain hematologic conditions (eg,
hereditary spherocytosis, immune thrombocytopenia).

Educational objective:
Thalassemia minor is usually asymptomatic and no treatment is required. Patients typically have mild anemia, low
mean corpuscular volume (55-75 µm3), target cells on peripheral smear, and disproportionately high red blood cell
(RBC) count. Iron deficiency anemia is associated with low RBC count and no microcytosis until hemoglobin is <10
g/dL.

Reference
• Beta-thalassemia.
Question #238

A 36-year-old woman comes to the office for a tuberculin skin test prior to volunteering at her daughter's school,
which is required by state law. The patient lives in the southern United States. She feels well and has never been
tested for tuberculosis. The patient has no ill contacts and no underlying medical conditions. She was born in the
United States, is not a health care worker, and has never been incarcerated. Two days after intradermal injection of
the Mycobacterium tuberculosis purified protein derivative, there is a 12-mm induration at the injection site. What is
the best next step in management of this patient?

A) Chest x-ray

B) Isoniazid, rifampin, ethambutol, and pyrazinamide for 8 weeks

C) Isoniazid with pyridoxine for 9 months

D) No additional intervention is required

E) Rifampin for 4 months


Explanation
Correct Answer:

D) No additional intervention is required

Tuberculin skin test (TST) thresholds

Induration
Patient population
size

• Individuals with HIV infection


• Recent contacts of patient with known TB
≥5 mm
• Persons with nodular or fibrotic changes on chest x-ray consistent with previously healed TB
• Organ transplant recipients & other immunosuppressed persons

• Recent immigrants (<5 years) from TB-endemic areas


• Injection drug users
• Residents & employees in high-risk settings (eg, prisons, nursing homes, hospitals, homeless
shelters)
≥10 mm
• Mycobacteriology laboratory personnel
• Persons with conditions that result in higher risk for TB reactivation (eg, diabetes mellitus,
leukemia, end-stage renal disease, chronic malabsorption syndromes, low body weight)
• Children age <5 or those exposed to adults in high-risk categories
≥15 mm • All of the above plus healthy persons

PPD = purified protein derivative; TB = tuberculosis.

Tuberculin skin testing (TST) is used to identify patients who have been exposed to Mycobacterium tuberculosis
and may have latent tuberculosis infection (LTBI). This test injects M tuberculosis purified protein derivative (PPD)
into the dermis on the inner surface of the forearm (Mantoux technique). Patients that have been exposed to M
tuberculosis will develop a delayed hypersensitivity reaction in response to PPD exposure, leading to induration at
the injection site 48-72 hours after administration.

The cutoff for a positive TST depends on patient risk factors. In those who are healthy with no risk factors who live
in the United States (as in this case), the threshold for a positive TST is ≥15 mm induration. This threshold is higher
than in some other countries (eg, the Canadian threshold for this population is ≥10 mm) due to greater likelihood of
exposure to nontuberculous mycobacteria in the United States, which increases false-positive results at lower cutoff
values.

LTBI testing is not generally recommended for patients at low risk for exposure but may be required by law (as in
this case). In these individuals, the following is recommended:

• When TST is negative (eg, <15 mm), no further management or work-up is recommended.

• When TST is positive, patients should generally undergo repeat TST or an interferon-gamma release assay
(IGRA) to exclude a false-positive result. Repeat testing is warranted in low-risk, low-prevalence settings; it
is not recommended in high-risk or high-prevalence settings.

IGRA can be used for initial or repeat LTBI screening but may not always be available.

(Choice A) A detailed history, physical examination, and chest x-ray should be obtained in patients with a positive
TST to rule out active TB. Patients with a positive TST and no signs of active TB infection (negative x-ray) have
LTBI. Given her low pretest probability of TB, this patient's TST result <15 mm is considered negative.
(Choice B) Active pulmonary TB can be treated with isoniazid, rifampin, ethambutol, pyrazinamide for 8 weeks
followed by isoniazid and rifampin for an additional 16 weeks. This patient has negative LTBI screening; she does
not require treatment.

(Choices C and E) Isoniazid monotherapy (with pyridoxine added for those at high risk for neuropathy) and
rifamycin-based regimens are first-line treatments for LTBI. Rifamycin-based regimens (eg, daily rifampin for 4
months) are generally preferred due to fewer adverse effects and shorter treatment duration, which improves
adherence. This patient with a negative TST does not require further evaluation or treatment.

Educational objective:
Tuberculin skin testing can be used to identify patients with latent tuberculosis infection. In the United States, an
induration size <15 mm is considered negative (ie, ≥15 mm is considered positive) in healthy patients with a low
likelihood of tuberculosis infection.
Question #239

A 26-year-old man comes to the physician with a 1-week history of dysuria and increased urinary frequency. He
has had multiple sexual partners in the past month and is inconsistent with condom use. His temperature is 37.1 C
(98.9 F), blood pressure is 110/70 mm Hg, and pulse is 68/min. Mucopurulent discharge is seen at the urethral
meatus. Urinalysis shows the following:

Blood Negative
Glucose Negative
Ketones Negative
Leukocyte esterase Positive
Nitrites Negative
White blood cells 50-100/hpf
Red blood cells 0-1/hpf
Bacteria None

Gram stain of the discharge shows no bacteria. Culture of the discharge and urine show no growth after 48 hours
of incubation. Which of the following is the most likely diagnosis?

A) Acute bacterial cystitis

B) Acute pyelonephritis

C) Chlamydial urethritis

D) Gonococcal urethritis
E) Trichomonal urethritis
Explanation
Correct Answer:

C) Chlamydial urethritis

Infectious urethritis in men

• Neisseria gonorrhoeae
• Chlamydia trachomatis
Etiology
• Mycoplasma genitalium
• Trichomonas vaginalis

• Dysuria
Manifestations
• Discharge

• Gram stain & culture


Diagnosis
• Nucleic acid amplification testing

• GU alone: ceftriaxone
• GU + Chlamydia*: ceftriaxone +
Treatment doxycycline
• Chlamydia/Mycoplasma: azithromycin
• Trichomonas: metronidazole
*GU + uncertain Chlamydia status is also treated with ceftriaxone
+ doxycycline.

GU = gonococcal urethritis.

This patient is sexually active and has dysuria, pyuria (white blood cells >10/hpf), urinary frequency, and
urethral discharge; these symptoms are concerning for urethritis. There was no growth on the Gram stain and
urine culture ("culture-negative" urethritis), which should raise concern for chlamydial urethritis. Chlamydia
trachomatis is a common cause of urethritis in men. It cannot be visualized in Gram-stained material or recovered
in conventional culture. The diagnosis can be made with nucleic acid amplification testing of a first-catch urine
sample without pre-cleaning the genital area. The treatment of choice is azithromycin or doxycycline.

(Choices A and B) Patients with acute bacterial cystitis have dysuria, urinary frequency, suprapubic discomfort,
bacteriuria, and pyuria. Fever, flank pain, and costovertebral tenderness are present if the infection spreads beyond
the bladder (pyelonephritis). Mucopurulent urethral discharge is not found. Urine culture is the gold standard for
diagnosing urinary tract infection.

(Choice D) Gonococcal urethritis presents in a similar fashion to chlamydial urethritis. Neisseria gonorrhoeae is
also very common and infection often occurs simultaneously with C trachomatis. Gram stain shows the gram-
negative cocci of gonococcus in 95% of cases of gonococcal urethritis, making this diagnosis unlikely.

(Choice E) Trichomoniasis is usually asymptomatic in men and is an uncommon cause of urethritis. Although
motile trichomonads are typically seen on the wet mount prep in women, microscopy is insensitive in men.

Educational objective:
In the absence of identifiable bacteria on culture or Gram stain, a mucopurulent urethral discharge in a patient who
is sexually active suggests chlamydial urethritis. Nucleic acid amplification testing confirms the diagnosis.

Reference
• Urethritis and cervicitis in adolescents
• Laboratory-confirmed gonorrhea and/or chlamydia rates in clinically diagnosed pelvic inflammatory disease
and cervicitis
Question #240

A 40-year-old man is evaluated prior to being discharged from the hospital to a physical rehabilitation center. The
patient was brought to the emergency department 2 weeks ago after his motorcycle collided with a truck. He was
thrown 3 m (10 ft) away and sustained multiple rib and thoracic vertebrae fractures. The patient had no sensation
or motor function below the T10 spinal level. Surgical stabilization of the fractures was performed, but there was no
neurologic recovery. The spinal injury is also complicated by neurogenic bladder requiring urinary catheterization.
Which of the following is most likely to reduce the risk of infection associated with urinary catheter use in this
patient?

A) Bladder irrigation with antibacterial solution

B) Clean intermittent catheterization

C) Daily antibacterial application to the urethral meatus

D) Frequent antiseptic cleaning of the external genitalia

E) Prophylactic systemic antibiotics


Explanation
Correct Answer:

B) Clean intermittent catheterization

Catheter-associated urinary tract infection (CA-UTI) is a common complication of urinary catheter use and can
be caused by:

• Extraluminal ascent of microorganisms due to the ability of some pathogens (eg, Escherichia coli,
Enterococcus spp, Candida spp, Pseudomonas aeruginosa) to form biofilm (slime-enclosed bacterial
aggregates) along the catheter wall, allowing them to reach the bladder within 24 hours of catheter insertion
• Intraluminal infection (less common) due to impaired urinary catheter drainage or contamination of a urinary
collection bag

CA-UTI is most effectively prevented by avoiding unnecessary catheter use and minimizing the duration of
catheterization. However, in patients with neurogenic bladder, long-term catheter use is required. In these
patients, clean intermittent catheterization (CIC), which involves periodic insertion and removal (eg, every 4-6
hours) of a clean urinary catheter and can often be performed by the patient, is usually the initial treatment. CIC
interrupts the extraluminal and intraluminal mechanisms of infection and, compared with the use of indwelling
catheters, is associated with a significantly lower risk of CA-UTI. Indwelling catheters (changed monthly) can be
considered if patients or their caregivers cannot perform CIC but are associated with an increased risk of UTI,
stricture, and bladder spasm; suprapubic tube placement is another option.

(Choice A) Bladder irrigation with antibacterial solutions can lead to the emergence of resistant bacteria and may
increase the risk of UTI due to the killing of nonpathogenic bacteria.

(Choices C and D) The application of antibacterial creams to the urethral meatus or antibacterial washes to the
external genitalia has not been found to be helpful in decreasing the risk of CA-UTI.

(Choice E) In patients using a urinary catheter, antibiotics should be administered only in the setting of a proven
UTI. Prophylactic antibiotics may increase the risk of development of resistant organisms and have not been
reliably demonstrated to decrease the risk of CA-UTI.

Educational objective:
Clean intermittent catheterization is an effective measure for reducing the risk of catheter-associated urinary tract
infection in patients with neurogenic bladder.

Reference
• Pathogenesis of bacteriuria and infection in the spinal cord injured patient.
Question #241

A 54-year-old woman comes to the office as a new patient. She immigrated to the United States 3 weeks ago for
political asylum. The patient has a history of hypertension and type 2 diabetes mellitus. She was also diagnosed
with active pulmonary tuberculosis 4 months ago. She completed 2 months of intensive antituberculosis therapy
with 4 drugs and now is on isoniazid and rifampin alone. Repeat sputum testings for acid-fast bacillus are
negative. The patient feels well overall but does have tingling and numbness of the bilateral hands and feet that
started a few weeks ago; she has no cough, fever, chills, or night sweats. She does not use tobacco, alcohol, or
illicit drugs. Temperature is 37 C (98.6 F), blood pressure is 126/84 mm Hg, and pulse is 84/min. Lung auscultation
reveals fine crackles in the left upper lung field. Heart sounds are normal and regular. Neurologic examination
shows normal motor strength but decreased touch and pain sensation in the bilateral upper and lower extremities.
Romberg sign is positive. Skin examination shows no abnormalities. Hemoglobin A1C level is 7%. Chest x-ray
reveals fibrotic changes in the left upper lung. Which of the following is the most likely cause of this patient's current
symptoms?

A) Degeneration of the dorsal and lateral spinal columns

B) Inflammatory demyelination of axons

C) Loss of motor neurons

D) Medication adverse effect

E) Microvascular nerve injury

F) Paraproteinemia neuropathy
Explanation
Correct Answer:

D) Medication adverse effect

Active tuberculosis is typically treated with 2 months of isoniazid (INH), rifampin, ethambutol, and pyrazinamide.
Most patients then transition to several months of INH and rifampin alone. Drug side effects are common with
treatment, most notably INH-induced hepatotoxicity and peripheral neuropathy.

INH-induced peripheral neuropathy is caused by pyridoxine (vitamin B6) deficiency. INH binds the active form
of pyridoxine, resulting in renal excretion. Most patients have large enough stores of pyridoxine to tolerate
increased excretion; however, those with malnourishment, pregnancy, or certain comorbid illnesses (eg, diabetes
mellitus) may develop a deficiency. Because pyridoxine serves as a crucial cofactor in the synthesis of
neurotransmitters, deficiencies typically manifest with neurologic symptoms. Most commonly, patients develop
numbness and tingling in a "stocking-glove" distribution. Physical examination usually reveals deficits in
proprioception and vibration; over time, touch, pain, and temperature sensation may be affected.

Those at high risk for pyridoxine deficiency (such as this patient with diabetes mellitus) should be given prophylactic
pyridoxine supplementation while on INH.

(Choice A) Vitamin B12 deficiency affects the dorsal/lateral spinal columns, resulting in paresthesias, ataxia, and
loss of vibratory sensation/proprioception. This patient with diabetes mellitus on INH is at high risk for pyridoxine
deficiency and most likely has INH-induced peripheral neuropathy.

(Choice B) Guillain-Barré syndrome causes acute inflammatory demyelinating polyneuropathy that usually
manifests with paresthesias of the toes and fingertips followed by ascending motor weakness. This patient has had
numbness and tingling for a few weeks without motor weakness; INH-induced peripheral neuropathy is far more
likely.

(Choice C) Amyotrophic lateral sclerosis is a progressive, neurodegenerative disorder that causes motor neuron
loss. Patients usually have significant muscle weakness or incoordination. This patient with sensory changes and
no motor weakness is unlikely to have this diagnosis.

(Choices E and F) Neuropathy caused by diabetes mellitus (microvascular nerve injury) or monoclonal plasma cell
disorders (paraprotein neuropathy) usually begins in the feet with alterations in proprioception/vibratory sensation.
Paraprotein neuropathy may progress to motor weakness. This patient is at high risk for INH-induced pyridoxine
deficiency due to diabetes mellitus; with a few weeks of sensory symptoms in both the hands and feet with no motor
symptoms, INH-induced peripheral neuropathy is more likely.

Educational objective:
Patients with malnutrition, pregnancy, or certain comorbid conditions (eg, diabetes mellitus) should be started on
pyridoxine supplementation when treated for latent or active tuberculosis with isoniazid (INH). This helps prevent
INH-induced peripheral neuropathy, which is due to INH-mediated pyridoxine deficiency.
Question #242

A 56-year-old man comes to the office due to a right foot ulcer that is not healing. He first noticed a small wound on
the right sole 2 weeks ago but does not remember any trauma. The patient applied over-the-counter antibiotic
cream, but the wound gradually enlarged and began draining foul-smelling material. He has had no fever or
significant pain. The patient has a history of hypertension and type 2 diabetes mellitus. Temperature is 37.2 C (99
F), blood pressure is 140/80 mm Hg, and pulse is 86/min. Right foot examination shows a 2×3 cm plantar ulcer
under the first metatarsal head. There is purulent exudate and the bone can be palpated with a sterile metal probe.
Distal pulses are present, but lower-extremity sensation to light touch and pain is decreased. A recent hemoglobin
A1c level is 8.9%. Which of the following is the best next step in management of this patient's foot ulcer?

A) Amputation of the foot

B) Erythrocyte sedimentation rate

C) Fungal blood cultures

D) Metatarsal bone biopsy

E) Swab and culture from the wound base


Explanation
Correct Answer:

D) Metatarsal bone biopsy

Patients with diabetes have multiple risk factors for the development of infected foot ulcers, including:

• peripheral neuropathy (eg, decreased lower extremity sensation, as in this patient): leading to impaired
recognition of minor damage
• hyperglycemia (eg, elevated hemoglobin A1c, as in this patient): leading to impaired immunity
• peripheral arterial disease: further contributing to impaired healing once a wound is present

Contiguous (nonhematogenous) extension of infection from the ulcer to underlying bone causes osteomyelitis.
Because this complication may be asymptomatic, patients with diabetic foot ulcers should be assessed for extent of
infection. As seen in this patient, the presence of a positive probe-to-bone test (palpating bone with a sterile, blunt
wooden or metal tool), a large ulcer (eg, >2 cm2), or an ulcer lasting ≥1 week increase the likelihood of
osteomyelitis. Fever, pain, an elevated erythrocyte sedimentation rate (ESR), and sinus tract drainage (eg,
purulent drainage) may be present.

The most specific diagnostic test for osteomyelitis is bone biopsy and culture. Bone biopsy, preferably performed
before antibiotic initiation, allows for identification of the causative pathogens and tailoring of antibiotic therapy.
Positive superficial wound cultures do not reliably predict the culprit organisms; even among patients with diabetes
and foot ulcers who do not have osteomyelitis, wound base culture should be performed following debridement and
curettage (ie, aseptically scraping the wound base with a blade or curette) rather than following a simple swab
(which can be contaminated) (Choice E).

(Choice A) Management of a foot ulcer complicated by osteomyelitis includes surgical debridement of necrotic
material and prolonged (ie, multiple weeks) antimicrobial therapy. Amputation is the option of last resort and is used
in instances of significant complications (eg, ischemia, necrosis) or when medical management fails.
(Choice B) Although an elevated ESR (eg, >70 mm/hr) is suggestive of osteomyelitis in the appropriate clinical
setting, it can be increased by any significant inflammatory process and therefore is nonspecific. In osteomyelitis,
an ESR is helpful in following the response to antibiotic treatment; however, the diagnosis should ideally be
established via tissue biopsy and the culprit organism identified via microbiologic examination.

(Choice C) Although patients with diabetes are at increased risk for superficial fungal infections, invasive fungal
infections are an uncommon cause of osteomyelitis, so fungal cultures are not a part of the standard evaluation.
Routine blood cultures can be helpful in patients with osteomyelitis due to hematogenous spread (eg, infective
endocarditis) but are less helpful in disease due to direct extension of local infection.

Educational objective:
Bacterial infection of a chronic diabetic foot ulcer may be minimally symptomatic and requires specific assessment.
When the bone can be palpated with a probe, the risk of underlying osteomyelitis is greatly increased. Biopsy and
culture of affected bone is critical to confirming the diagnosis and guiding management.

Reference
• Osteomyelitis in diabetic foot: a comprehensive overview.
Question #243

A 39-year-old previously healthy man comes to the office due to 4 days of nausea, loss of appetite, malaise, and
mild upper abdominal discomfort. He has had no vomiting or diarrhea. The patient returned from a trip to the
Philippines with his wife 2 weeks ago, but she has had no symptoms. He did not receive any vaccinations or
chemoprophylaxis prior to travel and reports that he did not consume bottled water consistently during the trip. The
patient does not use tobacco, alcohol, or illicit drugs. Temperature is 37.9 C (100.2 F), blood pressure is 118/72 mm
Hg, and pulse is 96/min. Physical examination reveals scleral icterus and tender hepatomegaly. Laboratory results
are as follows:

Hematocrit 36%
Platelets 180,000/mm3
Leukocytes 13,000/mm3
Liver function studies
Total bilirubin 4.3 mg/dL
Alkaline phosphatase 120 U/L
Aspartate aminotransferase 7,207 U/L
Alanine aminotransferase 8,180 U/L

Which of the following is the most likely sequela of this patient's condition?

A) Complete recovery

B) Fulminant hepatic failure

C) Hepatocellular carcinoma
D) Hepatorenal syndrome

E) Liver cirrhosis

F) Portal vein occlusion


Explanation
Correct Answer:

A) Complete recovery

This patient's presentation is consistent with acute hepatitis A virus (HAV) infection. HAV infection commonly
presents with fever, nausea, vomiting, and abdominal pain followed by jaundice and pruritus. Patients may also
report dark urine (bilirubinuria) and pale stools. Physical examination and laboratory analysis reveal tender
hepatomegaly and significant elevations in transaminases (>1,000 U/L), bilirubin, and alkaline phosphatase.
Diagnosis involves identifying anti-HAV IgM antibodies in the serum. Transmission occurs via the fecal-oral route
and is common in areas with overcrowding and poor sanitation. A number of groups are at increased risk for HAV
infection, including international travelers, men who have sex with men, illicit drug users, and those with
household or sexual contact with infected persons.

Treatment is largely supportive, and most patients completely recover in 3-6 weeks. A small subset of patients
will have a more prolonged course complicated by cholestatic or relapsing hepatitis. Fulminant hepatic failure is
rare; it can occur in patients age >50 or with preexisting chronic liver disease (Choice B). Post-exposure
prophylaxis with either HAV vaccine or HAV immune globulin should be considered for close contacts of patients.
HAV vaccination has now been incorporated into routine childhood vaccination schedules in the United States.

(Choices C and E) There are few significant long-term consequences of HAV infection; however, both hepatitis B
virus (HBV) and hepatitis C virus (HCV) infections are associated with an increased risk for hepatocellular
carcinoma and cirrhosis. This patient's travel history without prior HAV vaccination makes acute HAV more likely
than acute HBV or HCV, which are more commonly transmitted through intravenous drug use. In addition, acute
HAV is symptomatic in >70% of patients, whereas acute HBV and HCV are most often asymptomatic.

(Choice D) Patients with portal hypertension from chronic liver disease (eg, cirrhosis) are at increased risk for
hepatorenal syndrome (acute kidney injury in setting of liver disease). Hepatorenal syndrome can also rarely occur
in patients with fulminant hepatic failure from other causes (eg, hepatitis E). However, development of chronic liver
disease or fulminant hepatic failure is extremely rare in a young patient with HAV.
(Choice F) Portal vein occlusion can occur due to chronic hepatosplenic schistosomiasis, which can be found in
large parts of Africa and Asia. However, acute schistosomiasis (Katayama fever) presents with fever, urticaria,
angioedema, dry cough, and eosinophilia, and would not explain the patient's abnormal liver studies.

Educational objective:
Hepatitis A virus infection commonly presents with fever, nausea, vomiting, and abdominal pain followed by
jaundice and pruritus. Physical examination and laboratory analysis reveal tender hepatomegaly and significant
elevations in transaminases, bilirubin, and alkaline phosphatase. Most patients completely recover in 3-6 weeks.

Reference
• Hepatitis A.
Question #244

A 24-year-old man comes to the office due to 3 days of urethral discharge and a burning sensation with urination.
He had sexual intercourse with a new female partner a week ago and has had 4 female sexual partners over the
past 6 years. The patient has no other medical conditions and takes no medication. He does not use tobacco,
alcohol, or illicit drugs. Vital signs are within normal limits. Examination shows no skin lesions or rash. The
abdomen is soft and nontender. Genital examination shows copious, purulent discharge from the urethral orifice.
Nucleic acid amplification testing (NAAT) result from the urine specimen is positive for Neisseria gonorrhoeae. HIV
testing is negative. Antibiotic therapy is prescribed. Which additional test is indicated in this patient?

A) Herpes simplex virus PCR

B) Human papillomavirus PCR

C) NAAT for Trichomonas vaginalis

D) No additional testing

E) Syphilis serology
Explanation
Correct Answer:

E) Syphilis serology

Sexually transmitted infection screening*

• Neisseria gonorrhoeae (eg, NAAT)


• Chlamydia trachomatis (eg, NAAT)
All patients
• Syphilis (eg, RPR)
• HIV (eg, 4th-generation antigen/antibody)

• Women only: Trichomonas vaginalis (eg, wet mount)


Additional testing for certain
• Herpes simplex virus screening (eg, serology) only when history of
populations
characteristic lesions

*For patients with active sexually transmitted infection or those who request screening.

NAAT = nucleic acid amplification testing; RPR = rapid plasma reagin.

This patient with a new sexual partner and copious urethral discharge has Neisseria gonorrhoeae infection. N
gonorrhoeae and Chlamydia trachomatis are the most commonly reported communicable diseases in the United
States, and coinfection occurs in up to 40% of cases. Patients diagnosed with a sexually transmitted infection
(STI) and those who ask for screening for an STI should undergo evaluation for the following:
• N gonorrhoeae (eg, NAAT)
• C trachomatis (eg, NAAT)
• HIV (eg, HIV antigen/antibody testing)
• Syphilis (eg, rapid plasma reagin)

Women who request STI screening should also be tested for Trichomonas vaginalis using microscopy with wet
mount or NAAT (if suspicion is high and wet mount is negative); however, routine screening in men is not
recommended because the infection is often asymptomatic and transient (<10 days) (Choice C). Herpes simplex
virus screening is generally reserved for those who have recurrent vesicular, papular, or ulcerative genital lesions
characteristic of the disease (Choice A).

Patients with an STI should be counseled on safe sexual practices and encouraged to inform recent partners of
infection (with a recommendation to get tested).

(Choice B) Human papillomavirus screening is generally reserved for women (eg, cervical) or men who have sex
with men (eg, anal). Screening is focused on identifying oncogenic strains or cytologic abnormalities associated
with those strains to reduce patients' risk of cancer. Routine screening in heterosexual men is not recommended.

(Choice D) Additional screening is recommended to identify common coinfections to prevent progression (eg,
neurosyphilis) and limit spread to others.

Educational objective:
Patients with Neisseria gonorrhoeae infection are at risk for coinfection with other sexually transmitted pathogens,
including Chlamydia trachomatis, HIV, and Treponema pallidum (syphilis). Patients should be screened for these
infections, counseled on safe sexual practices, and given appropriate antibiotics.
Question #245

A 73-year-old man comes to the office due to 3 days of fever, facial pain over his right cheek, and bloody nasal
discharge. Last night he was able to see without difficulty, but this morning, he developed double vision. He has
had no recent trauma to the face or dental procedures. The patient has a 30-year history of type 2 diabetes mellitus
complicated by nephropathy and retinopathy. His medical history also includes hypertension and hyperlipidemia.
He is widowed and lives alone. He does not use tobacco or alcohol. Temperature is 39 C (102.2 F), blood pressure
is 130/76 mm Hg, and pulse is 102/min. The patient appears ill. Chemosis and mild proptosis of the right eye are
present. There is tenderness over the right maxillary sinus. Nasal examination shows right nasal congestion and
necrosis of the right nasal turbinate. Necrosis of the hard palate is present. Cardiopulmonary examination is
normal. The patient has no skin rash. CT scan reveals opacification and bony erosion of the right maxillary sinus.
Hemoglobin A1c is 11%. Which of the following is the most likely causative organism?

A) Actinomyces

B) Clostridium septicum

C) Oral anaerobic bacteria

D) Pseudomonas aeruginosa

E) Rhizopus species

F) Staphylococcus aureus

G) Streptococcus pneumoniae
Explanation
Correct Answer:

E) Rhizopus species

Rhino-orbital-cerebral mucormycosis*

• Diabetes mellitus (ketoacidosis)


Risk factors • Hematologic malignancy
• Solid organ or stem cell transplant

• Acute/aggressive
Manifestations • Fever, nasal congestion, purulent nasal discharge, headache, sinus pain
• Necrotic invasion of palate, orbit, brain

Diagnosis • Sinus endoscopy with biopsy & culture

• Surgical debridement
Treatment • Liposomal amphotericin B
• Elimination of risk factors (eg, ↑ glucose, acidosis)

*Largely due to Rhizopus species.


Mucormycosis is a highly destructive fungal infection caused by a collection of molds ubiquitous in the
environment. Spores are inhaled and convert to hyphae in the nasal turbinates and respiratory tree.
Immunocompetent individuals rapidly clear the organism, but individuals with significant immunocompromise are
at high risk for invasive disease. Patients with poorly controlled diabetes mellitus (especially with ketoacidosis)
are most likely to develop mucormycosis.

Rhino-orbital-cerebral mucormycosis is usually caused by Rhizopus species. Symptoms are acute and include
fever, nasal congestion, purulent nasal discharge, headache, and sinus pain. Local, necrotic spread (due to
infarction of infected tissues) to the palate, orbit, and brain is common. Early diagnosis is critical as mortality rates
are as high as 60%; any patient with significant immunocompromise and sinusitis requires sinus endoscopy with
cultures and biopsy (histopathologic identification of the organism) to evaluate for mucormycosis. Treatment
involves surgical debridement, antifungal therapy (eg, liposomal amphotericin B), and elimination of the
predisposing factor (eg, hyperglycemia).

(Choice A) Actinomyces is a commensal bacterium of the oral cavity that may cause a painless, slow-growing
mass with draining sinus tracts (sulfur granules) on or near the jaw. Infections occur mainly in patients with
immunocompromise (eg, diabetes mellitus) after dental procedures. This patient has an acute, painful infection with
prominent necrosis, making mucormycosis more likely.

(Choice B) Clostridium septicum causes spontaneous gas gangrene with acute fever, severe muscle pain, and
bullous, taut skin lesions. Infections primarily involve the extremities.

(Choice C) Oral anaerobic bacteria are less likely to cause disease in the aerobic sinus environment. This patient
with poorly controlled diabetes mellitus, palate/turbinate necrosis, and sinus drainage likely has mucormycosis.

(Choice D) Pseudomonas aeruginosa is an aerobic gram-negative organism that can cause a variety of infections,
including endophthalmitis. This infection usually manifests with acute ocular pain and decreased visual acuity after
eye trauma or surgery. Palate/turbinate necrosis and nasal discharge are atypical.

(Choices F and G) Staphylococcus aureus and Streptococcus pneumoniae are common causes of bacterial
sinusitis. Severe cases may result in complications, including osteomyelitis; however, symptoms typically arise over
days or weeks and would not be associated with necrosis of the hard palate and turbinate.
Educational objective:
Mucormycosis is an invasive fungal infection that most commonly manifests as rhino-orbital-cerebral disease in
patients with immunocompromise (particularly poorly controlled diabetes mellitus). Symptoms include acute fever,
nasal congestion, purulent nasal discharge, headache, and sinus pain. Local, necrotic spread is common.
Diagnosis requires sinus endoscopy with tissue sampling. Rhizopus is the most common cause.
Question #246

A 35-year-old woman comes to the office with a 3-day history of sore throat. A day after the sore throat started, she
developed runny nose and cough. The patient has had difficulty sleeping due to the severity of the cough. She has
smoked a pack of cigarettes daily for the last 10 years. Temperature is 37.1 C (98.8 F) and blood pressure is 115/
65 mm Hg. Oxygen saturation is 98% on room air. The tympanic membranes are clear and intact. The tonsils are
red and without exudate. The uvula is midline. There is no cervical lymphadenopathy. The lungs are clear to
auscultation, and the abdomen is nontender. Which of the following is the best next step in management of this
patient?

A) Amoxicillin

B) Chest x-ray

C) Streptococcal rapid antigen detection test

D) Symptomatic treatment only

E) Throat culture
Explanation
Correct Answer:

D) Symptomatic treatment only


This patient has acute pharyngitis, characterized by tonsillar and/or oropharyngeal pain and redness. Viral
pharyngitis is more common than bacterial pharyngitis and is caused by common cold viruses (eg, adenovirus,
rhinovirus, coronavirus), which produce upper respiratory tract symptoms (eg, rhinorrhea, nasal congestion,
cough), occasionally accompanied by conjunctival injection (ie, viral conjunctivitis). Viral pharyngitis typically
lacks exudates, fever is typically low-grade, and cervical lymphadenopathy is minimal or absent. Symptoms
generally self-resolve within a week, and symptomatic treatment (eg, nonsteroidal anti-inflammatory drugs) alone
is often sufficient; antibiotics are not indicated (Choice A).

In contrast, bacterial pharyngitis, most commonly caused by group A Streptococcus (GAS), typically presents with
isolated exudative pharyngitis (ie, absence of cough, rhinorrhea, conjunctivitis), along with fever and anterior
cervical lymphadenopathy. Although GAS pharyngitis also self-resolves, it can cause rare but serious
complications such as acute rheumatic fever, rheumatic heart disease, and post-streptococcal glomerulonephritis,
especially in children. To reduce the risk of these complications, treatment with a penicillin is recommended.

Streptococcal rapid antigen detection test (RADT) can diagnose pharyngitis due to GAS but is not needed if clinical
suspicion is low. In adults, the Centor criteria are used to determine the need for RADT:

• Score 0-1: low likelihood of GAS pharyngitis; no testing or antibiotic therapy

• Score 2-3: intermediate likelihood of GAS; RADT with antibiotic therapy if positive

• Score 4: high likelihood of GAS; consider empiric antibiotic therapy without testing

This patient has a Centor criteria score of 0; neither RADT nor antibiotic therapy is indicated (Choices A and C).

(Choice B) Although viral respiratory infections are occasionally complicated by secondary bacterial pneumonia,
this patient has no concerning features (eg, fever, crackles on lung examination) for which a chest x-ray is
warranted.

(Choice E) Throat culture is more sensitive than RADT (90%-95% versus 77%-92%) but is not the first-line test to
evaluate for GAS because it takes 24-48 hours (versus 10 minutes for RADT). Given the relatively low incidence of
GAS pharyngitis in adults, throat culture is generally reserved to confirm a negative RADT in those in whom GAS
infection carries a high risk of complications (eg, history of rheumatic fever, caring for infants).

Educational objective:
Group A Streptococcus (GAS) pharyngitis is characterized by exudative pharyngitis, fever, tender anterior cervical
lymphadenopathy, and absence of cough, features that are less common in viral pharyngitis. Patients at low risk for
GAS pharyngitis based on these clinical criteria do not require rapid antigen testing or empiric antibiotic treatment.

Reference
• Poststreptococcal illness: recognition and management.

• Common questions about streptococcal pharyngitis.


Question #247

A 42-year-old woman comes to the physician due to 3 weeks of low-grade fever, weight loss of 4.5 kg (10 lb), and
malaise. She has a history of mitral valve prolapse but is otherwise healthy. The patient underwent a tooth
extraction a month ago. She does not use tobacco, alcohol, or illicit drugs. Temperature is 38.5 C (101.3 F), blood
pressure is 145/76 mm Hg, pulse is 90/min, and respirations are 18/min. The lungs are clear to auscultation and
percussion. A 3/6 holosystolic murmur is present at the apex. Chest x-ray is normal. Urinalysis is unremarkable.
Blood is drawn for cultures and empiric antibiotics are started. Echocardiography shows moderate mitral
regurgitation and a 6-mm mobile mass attached to the mitral valve. Which of the following organisms is the most
likely cause of this patient's condition?

A) Enterococcus species

B) Staphylococcus aureus

C) Staphylococcus epidermidis

D) Staphylococcus saprophyticus

E) Streptococcus gallolyticus (formerly S bovis)

F) Streptococcus sanguinis
Explanation
Correct Answer:

F) Streptococcus sanguinis

Culture-positive infective endocarditis

• Prosthetic valves
• Intravascular catheters
Staphylococcus aureus
• Implanted devices (eg, pacemaker/defibrillator)
• Intravenous drug users

• Gingival manipulation
Viridans streptococci
• Respiratory tract incision or biopsy

• Prosthetic valves
• Intravascular catheters
Staphylococcus epidermidis
• Implanted devices

Enterococci • Nosocomial urinary tract infections

Streptococcus gallolyticus • Colon carcinoma


(formerly S bovis) • Inflammatory bowel disease

• Immunocompromised host
Fungi
• Intravascular catheters
(eg, Candida)
• Prolonged antibiotic therapy

This patient with a history of mitral valve prolapse (MVP) had a dental extraction and subsequently developed
subacute fever, mitral regurgitation, and a mitral valve vegetation, indicating infective endocarditis (IE).
Procedures associated with bacteremia (eg, tooth extraction) can occasionally trigger IE, particularly in patients with
valvular disease. Features suggestive of IE include subacute, nonspecific systemic signs/symptoms (eg, malaise,
weight loss); new cardiac murmur; signs of embolism; bacteremia with a common IE pathogen; and/or evidence of
valvular vegetation (eg, mobile mass) on echocardiography.

The most likely underlying organism can often be inferred based upon the preceding procedure, as follows:

• Viridans streptococci, a group of commensal bacteria in the mouth and upper respiratory tract, are the
most likely organisms to cause IE following dental manipulation (eg, tooth extraction) or respiratory
mucosa penetration. This group includes Streptococcus sanguinis, Streptococcus mitis, Streptococcus
mutans, and Streptococcus milleri.

• Enterococcus, a gram-positive organism, is most likely to cause IE following gastrointestinal or genitourinary


procedures involving an infected area of mucosa (Choice A).

• Staphylococcus aureus and coagulase-negative Staphylococcus (eg, S epidermis) are the most common
causes of IE following skin and soft tissue procedures in areas of infection (Choices B and C). These skin
commensals are also the most common causes of IE in users of intravenous drugs.

Although MVP is associated with increased risk of native valve endocarditis, the overall risk of IE in this population
is quite low (particularly compared to the relatively high prevalence of MVP). In addition, preprocedural antibiotic
prophylaxis has not decreased IE incidence, and many cases of IE develop following transient, daily episodes of
bacteremia. Therefore, guidelines do not recommend prophylactic antibiotics for patients with MVP prior to
procedures associated with bacteremia.

(Choice D) Staphylococcus saprophyticus is a common cause of urinary tract infections in young women; it is not a
common cause of IE.

(Choice E) Streptococcus gallolyticus (formerly S bovis) IE is seen primarily in patients with ulcerative lesions of
the colon due to colon cancer or inflammatory bowel disease. It is not common following dental procedures.

Educational objective:
Infectious endocarditis (IE) following dental or respiratory tract procedures is most often caused by viridans
streptococci. Enterococcus is the most common cause of IE following manipulation of infected areas of the
gastrointestinal or genitourinary tract. IE following skin or soft tissue infection or in users of intravenous drugs is
most often due to Staphylococcus aureus or coagulase-negative Staphylococcus.

Reference
• Preeminence of Staphylococcus aureus in infective endocarditis: a 1-year population-based survey.
Question #248

A 34-year-old man is being discharged from the hospital after treatment for splenic rupture and intraabdominal
bleeding following blunt abdominal trauma. His postoperative course was uncomplicated. The patient has no other
chronic medical conditions and previously took no medications. He has no known drug allergies. Vital signs are
normal. Abdominal examination shows a well healing incision. Appropriate vaccinations are planned for the first
follow-up appointment in 2 weeks. Which of the following antibiotics should be prescribed for this patient to take
immediately if he develops a fever?

A) Amoxicillin-clavulanate

B) Clindamycin

C) Metronidazole

D) Rifampin

E) Trimethoprim-sulfamethoxazole
Explanation
Correct Answer:

A) Amoxicillin-clavulanate

This patient with splenic rupture likely had an emergency splenectomy. Because the spleen plays a vital role in
generating antibodies, storing monocytes, and removing antibody-coated bacteria, patients with splenectomy are
at significant risk for fulminant bacterial infection. Risk is greatest with encapsulated bacteria (eg,
Streptococcus pneumoniae, Hemophilus influenzae, Neisseria meningitidis) because these organisms are
destroyed primarily by splenic monocytes following opsonization in the intravascular or reticuloendothelial space.

Fulminant bacterial infection can lead to death within hours of symptoms. Therefore, patients who undergo
splenectomy are prescribed an antibiotic to take immediately when they develop fever. Amoxicillin-clavulanate is
used most often due to its broad spectrum of activity against common encapsulated pathogens, but those with
penicillin allergy can take levofloxacin, which has similar broad efficacy. Patients should be instructed to proceed to
the nearest emergency department for evaluation of fever after they take the antibiotic.

Patients who undergo splenectomy are also generally vaccinated against common encapsulated organisms.
Vaccinations are typically administered >2 weeks after urgent splenectomy to ensure adequate humoral response.
Most patients receive the 13-valent and 23-valent pneumococcal vaccines, the H influenzae type B vaccine, and the
quadrivalent and monovalent meningococcal vaccines. Patients should also be given the yearly seasonal influenza
vaccination to reduce the risk of secondary bacterial infection with S pneumoniae following influenza infection.

(Choice B) Clindamycin is used to treat some gram-positive aerobic and anaerobic bacteria but has almost no
efficacy against gram-negative bacteria such as N meningitidis and H influenzae.

(Choice C) Metronidazole effectively covers gram-negative anaerobic bacteria and some protozoans but has
minimal efficacy against aerobic and gram-positive organisms (eg, S pneumoniae).

(Choice D) Rifampin has activity against intracellular pathogens such as Mycobacterium tuberculosis. Although it
is sometimes used for chemoprophylaxis against N meningitidis in high-risk patients, it is not generally used for the
treatment of this organism or other common encapsulated pathogens.

(Choice E) Trimethoprim-sulfamethoxazole has broad efficacy against many gram-positive and gram-negative
bacteria but is not typically used for splenectomy patients with fever due to delayed bactericidal activity and
resistance patterns.

Educational objective:
Patients who undergo splenectomy are at high risk for fulminant sepsis with encapsulated organisms. They should
be prescribed amoxicillin-clavulanate to take immediately in the setting of fever and then should be advised to
proceed directly to the nearest emergency department.
Question #249

A 70-year-old man who lives in a group home is brought to the emergency department due to 3 months of
persistent cough and anorexia. The cough occurs mainly in the early morning and is often productive. He has a
history of chronic kidney disease, type 2 diabetes mellitus, and an ischemic stroke that resulted in right-sided
residual paralysis. He has never smoked cigarettes. Temperature is 38.2 C (100.8 F), blood pressure is 124/80
mm Hg, pulse is 90/min, and respirations are 16/min. BMI is 22 kg/m2. Examination shows cervical
lymphadenopathy and muscle wasting. Cardiopulmonary examination shows no abnormalities. Laboratory results
are as follows:

Complete blood count


Hemoglobin 10 g/dL
Leukocytes 7,800/mm3
Neutrophils 70%
Lymphocytes 20%
Monocytes 10%

Serum chemistry
Creatinine 2.8 mg/dL
Total protein 8 g/dL
Albumin 2.8 g/dL

Chest x-ray is shown in the exhibit. Tuberculin skin testing is negative. What is the most likely diagnosis?

A) Infectious mononucleosis

B) Granulomatosis with polyangiitis


C) Small cell lung cancer

D) Tuberculosis
Explanation
Correct Answer:

D) Tuberculosis

Despite his negative tuberculin skin test (TST) result, several features of this elderly patient's presentation are
concerning for pulmonary tuberculosis (TB). Risk of reactivation TB is significantly increased (6- to 50-fold) in
chronic kidney disease (CKD) due to impairments in cell-mediated immunity (CMI), which usually serves as a
protective mechanism against intracellular organisms (eg, latent Mycobacterium tuberculosis). CMI is also required
to generate induration against the TST. Therefore, in patients with CKD, TST anergy (false-negative result) is
common and a negative TST cannot rule out active TB infection.

Younger patients with active pulmonary TB frequently have fever and cough. In contrast, older patients such as
this one often have nonspecific symptoms such as weeks or months of fatigue, anorexia, muscle wasting, and
weight loss. Mild, productive cough may occur, particularly in the mornings (overnight pooling of secretions).
Lymphadenopathy, particularly hilar (seen on this patient's x-ray) and cervical (reflecting lymphatic spread), is
common. Laboratory findings suggestive of TB in this patient include:

• anemia: likely anemia of chronic disease in the setting of inflammation

• monocytosis (late finding): monocytes differentiate into macrophages, which are important for the
granulomatous antimycobacterial response

• hypergammaglobulinemia (eg, elevated total protein in this patient): reflecting the heterogeneous antibody
response against M tuberculosis

• hypoalbuminemia (late finding, particularly in the elderly): possibly due to inflammatory cytokines
stimulating production of acute phase reactants at the expense of albumin; in addition, although not
obviously present in this patient, malnutrition often coexists with TB, compounding hypoalbuminemia.

Upper lobe cavitation and hilar lymphadenopathy (as seen in this patient) are classic, although older individuals
frequently have multifocal radiographic infiltrates. Due to all these differences, the diagnosis of TB in older patients
can easily be missed or delayed.

(Choice A) Infectious mononucleosis can cause adenopathy, fever, and systemic symptoms (eg, anorexia, fatigue),
some of which can last for a prolonged time. However, it usually affects young adults and would be uncommon in
this patient's age group; symptoms typically last a few weeks rather than months; and productive cough and
cavitary lesions are not seen.

(Choice B) Granulomatosis with polyangiitis presents with upper/lower respiratory symptoms and renal
insufficiency (glomerulonephritis). Radiographic findings may include multiple nodules that may be cavitary; a
single large cavitary lesion with associated hilar lymphadenopathy is uncommon. In addition, most patients have
upper airway symptoms (eg, otitis media, rhinosinusitis).

(Choice C) Although small cell lung cancer can cause cough and anorexia, it typically occurs exclusively in
smokers and presents with a large hilar mass. Cavitary lesions are more common with squamous cell carcinoma.

Educational objective:
Pulmonary tuberculosis is common in older individuals with comorbidities (eg, renal failure) due to waning cell-
mediated immunity. Older patients often have fatigue, weight loss, and anorexia; cavitation is seen less often on
chest x-ray.
Question #250

A 20-year-old woman comes to the office due to headache, fatigue, malaise, myalgias, and fever. She has no
known sick contacts. The patient recently returned from a summer internship as an arborist at a New Jersey nature
preserve. She has a monogamous sexual relationship with her partner and uses an intrauterine device for
contraception. Temperature is 37.9 C (100.2 F), blood pressure is 124/78 mm Hg, and pulse is 98/min.
Oropharyngeal examination is normal, and no cervical lymphadenopathy is present. There is a 4-cm, flat, annular
lesion on her right lower abdomen with no scaling, raised borders, induration, or purulence. Which of the following
is the best next step in management of this patient?

A) Biopsy of the rash

B) Borrelia burgdorferi serology

C) Oral doxycycline

D) Oral fluconazole

E) Reassurance and symptomatic care


Explanation
Correct Answer:

C) Oral doxycycline

Stage Lyme disease clinical features

• Erythema migrans
Early localized
• Fatigue, headache
(days to 1 month)
• Myalgia, arthralgia

• Multiple erythema migrans


• Unilateral/bilateral cranial nerve palsy (eg, CN VII)
Early disseminated
• Meningitis
(weeks to months)
• Carditis (eg, AV block)
• Migratory arthralgia

• Arthritis
Late
• Encephalitis
(months to years)
• Peripheral neuropathy

AV = atrioventricular.

This patient who worked outdoors in a Lyme-endemic area and developed systemic symptoms and an annular rash
likely has early localized Lyme disease due to Borrelia burgdorferi, a spirochete transmitted by the Ixodes
scapularis tick during blood feeding.

Symptoms of early localized Lyme disease develop 7-14 days after inoculation. The hallmark feature is erythema
migrans (EM), a flat, annular macule at the tick bite site that slowly expands outward as the spirochete moves
through the dermis. Areas of central clearing form over time, leading to the classic bull's-eye appearance.
Systemic manifestations (eg, headache, myalgia, fever, malaise) also commonly occur.

Diagnosis is based on characteristic symptoms because B burgdorferi serology is generally negative at this early
stage of illness (the humoral response is still being formed). Empiric treatment with a 14-day course of oral
doxycycline is curative in most patients.

(Choice A) Biopsy of a skin lesion is generally reserved for rashes that do not improve with standard therapy or
when malignancy is suspected. This patient's systemic symptoms and flat, annular lesion with no scaling or raised
borders is classic for Lyme disease.

(Choice B) B burgdorferi serology is diagnostic for patients with early disseminated (eg, multiple EM lesions,
cranial nerve paralysis, carditis) and late (eg, monoarticular arthritis) Lyme disease. However, it is not used for early
localized Lyme disease because it is often falsely negative.

(Choice D) Tinea corporis is a common cause of an annular-shaped skin lesion on the trunk. However, it is
generally scaly with raised borders and is not associated with systemic symptoms.

(Choice E) Reassurance and symptomatic care are appropriate for most viral upper respiratory infections; most
patients have rhinorrhea, nasal congestion, and/or cough.

Educational objective:
Early localized Lyme disease should be suspected in individuals in Lyme-endemic areas who develop a flat, annular
rash with or without central clearing. The diagnosis is made clinically because serology is negative in early disease;
doxycycline is usually curative.

Reference
• The clinical assessment, treatment, and prevention of Lyme disease, human granulocytic anaplasmosis, and
babesiosis: clinical practice guidelines by the Infectious Diseases Society of America.

• Laboratory evaluation in the diagnosis of Lyme disease.


Question #251

A 42-year-old woman comes to the office in October for a follow-up visit 3 weeks after being hospitalized for
abdominal swelling. The patient reports that she has occasional fatigue but is otherwise feeling well. She has been
taking lactulose, spironolactone, and furosemide with some improvement in her symptoms. The patient drank 12
beers daily for 15 years but has been sober for the past 3 months. She does not use tobacco or recreational
drugs. When asked about her immunization history, the patient recalls getting all appropriate childhood
vaccinations and a tetanus-diphtheria toxoids booster following a traumatic injury at age 31; she does not recall
receiving any vaccines since then. Physical examination reveals mild jaundice, palmar erythema, a slightly
distended abdomen, and spider angiomas. Laboratory results are as follows:

Hepatitis panel
Hepatitis A IgM negative
Hepatitis A IgG negative
Hepatitis B surface antigen negative
Hepatitis B surface antibody negative
Hepatitis C IgM negative

Which of the following vaccinations are recommended for this patient?

PCV20 = 20-valent pneumococcal conjugate vaccine


Tdap = tetanus–reduced diphtheria–acellular pertussis vaccine

A) Hepatitis A, hepatitis B, and influenza

B) Hepatitis A, hepatitis B, and Tdap

C) Hepatitis A, hepatitis B, influenza, and PCV20


D) Hepatitis A, hepatitis B, influenza, PCV20, and Tdap

E) Influenza, PCV20, and Tdap


Explanation
Correct Answer:

D) Hepatitis A, hepatitis B, influenza, PCV20, and Tdap

Recommended vaccines for patients with chronic liver disease

Tetanus Every 10 years

Influenza Annually

Pneumococcus At diagnosis

Hepatitis A Initiate series if not immune

Hepatitis B Initiate series if not immune

This patient has clinical features consistent with cirrhosis due to alcohol use. Patients with chronic liver disease
require the same immunizations as the general population in addition to disease-specific vaccines.

The influenza vaccine is recommended annually for all adults (Choice B). The intramuscular inactivated
influenza vaccine appears to be more effective than the live attenuated intranasal vaccine and is preferred.
All adults should receive a tetanus vaccine every 10 years. Generally, the tetanus-diphtheria toxoids (Td) is given,
but this should be replaced by the tetanus–reduced diphtheria–acellular pertussis (Tdap) vaccine once. If Tdap has
not been previously administered, it may be given at any time regardless of the time since the previous Td.

Vaccination against pneumococcus with the 20-valent pneumococcal conjugate vaccine (PCV20)—or with the
15-valent pneumococcal conjugate vaccine (PCV15) followed by the 23-valent pneumococcal polysaccharide
vaccine (PPSV23)—is recommended for all adults age ≥65. Vaccination is also recommended for those age <65
who have certain comorbid conditions that increase their risk for pneumococcal disease (eg, chronic liver, lung,
heart disease; diabetes mellitus; smoking). Because this patient has cirrhosis (ie, chronic liver disease), she should
be vaccinated with PCV20.

Patients with chronic liver disease who contract viral hepatitis are at increased risk for hepatic decompensation. For
this reason, they should be vaccinated against hepatitis A and B unless they have documented evidence of
immunity. Because this patient's hepatitis serologies do not indicate immunity, both vaccines should be
administered (Choice E).

(Choices A and C) Given that this patient's last tetanus vaccination was ≥10 years ago, she is due for one now.
The pneumococcus vaccine should be added because she has cirrhosis.

Educational objective:
Patients with chronic liver disease require the same immunizations as the general population plus vaccinations for
hepatitis A and B and pneumococcus.

Reference
• Cirrhosis: diagnosis and management.
Question #252

A 46-year-old woman with chronic HIV infection comes to the office for a health maintenance visit. The patient
recently moved to the state for a job with an architecture firm. Currently, she is sexually active with 2 male partners
who use condoms consistently. The patient's records show that vaccinations through age 18 are up to date and
that she was vaccinated against meningococcus as a freshman in college. She also received a vaccine for tetanus-
diphtheria-acellular pertussis 3 years ago following a motor vehicle collision. The patient has no history of
opportunistic infections. She is on antiretroviral therapy. CD4 is 670/mm3. Screening for hepatitis A, B, and C
shows a positive hepatitis B surface antibody. In addition to influenza and pneumococcus vaccinations, which of the
following vaccines is most appropriate for this patient?

A) Hepatitis A

B) Hepatitis B

C) Human papillomavirus

D) Measles, mumps, and rubella

E) Tetanus-diphtheria
Explanation
Correct Answer:

A) Hepatitis A

Vaccines for adults with HIV infection

Vaccine Indications

HAV • All patients without documented immunity to HAV

HBV • All patients without documented immunity to HBV

• All patients age 11-26


HPV
• Consider with shared decision-making in patients age 27-45

Influenza • Annually for all patients (inactivated formulation)

Meningococcus • All patients age 11-18


• Large groups living in close proximity (eg, college students,
(serogroups A, C, W, Y) members of military services, incarcerated individuals)
• Asplenia or complement deficiency

• PCV20

Pneumococcus OR

• PCV15 followed by PPSV23

• Tdap once (repeat with each pregnancy in women)


Tetanus, diphtheria & pertussis
• Td every 10 years

Live vaccines (eg, MMR, varicella) are contraindicated if CD4+ cell count <200/mm3

HAV = hepatitis A virus; HBV = hepatitis B virus; HCV = hepatitis C virus; HPV = human papillomavirus; MMR =
measles, mumps & rubella; PCV15 = 15-valent pneumococcal conjugate vaccine; PCV20 = 20-valent
pneumococcal conjugate vaccine; PPSV23 = 23-valent pneumococcal polysaccharide vaccine; Td = tetanus-
diphtheria toxoids booster; Tdap = tetanus–reduced diphtheria–acellular pertussis.

Patients with HIV infection who have a CD4 ≥200/mm3 should receive all vaccines (eg, seasonal influenza, tetanus-
diphtheria-acellular pertussis) that an otherwise healthy person requires. They should also receive the 20-valent
pneumococcal conjugate vaccine (or 15-valent conjugate vaccine followed by the 23-valent pneumococcal
polysaccharide vaccine).

In addition, patients with HIV are at increased risk for serious infection and prolonged viremia from hepatitis A
virus (HAV). Therefore, HAV vaccination is recommended for patients with HIV who do not have evidence of
immunity (ie, negative serum anti-HAV), regardless of other risk factors. This is particularly relevant for those with
longstanding HIV infection or other HAV risk factors (eg, men who have sex with men). Although HAV vaccines are
provided in childhood, protective antibody levels may rise slowly and wane over time, particularly in patients with
HIV. Serologic response should be confirmed with a serum anti-HAV assay after 1 month; if seroconversion is not
achieved, revaccination is advised.

(Choice B) Hepatitis B vaccine is recommended for all patients with HIV who do not have documented immunity to
hepatitis B. This patient has a positive serology for hepatitis B surface antibody.

(Choice C) The human papillomavirus (HPV) vaccine is generally administered for patients age 11-12, with catch-
up vaccination recommended up to age 26. Although HPV vaccination is considered safe up to age 45, the benefits
in patients age ≥46, including those with HIV, are likely to be small and vaccination is therefore not recommended.

(Choice D) Certain live attenuated vaccines (eg, measles, mumps, and rubella [MMR]) may be given to patients
with HIV and CD4 ≥200/mm3. However, this patient has already received her childhood vaccines and does not
require MMR.

(Choice E) Tetanus-diphtheria-acellular pertussis (Tdap) is recommended as a one-time vaccine for all adults, with
revaccination provided to women at each pregnancy. Subsequently, tetanus-diphtheria toxoid (Td) booster is
recommended at 10-year intervals, or after 5 years in the event of high-risk trauma. This patient received Tdap
vaccination 3 years ago.

Educational objective:
Patients with HIV are at increased risk for serious infection and prolonged viremia from hepatitis A virus (HAV), and
therefore HAV vaccination is recommended.

Reference
• Vaccination against sexually transmitted infections.

• Vaccination against viral hepatitis of HIV-1 infected patients.


Question #253

A 27-year-old woman comes to the office due to joint pain. Her symptoms began 10 days ago and consist of
bilateral pain in the metacarpophalangeal joints, proximal interphalangeal joints, wrists, knees, and ankles. She
describes joint stiffness lasting 10-15 minutes on awakening in the morning. The patient has also had associated
fatigue and a few episodes of loose bowel movements associated with mild skin itching and patchy redness. She
has no fever, weight loss, or lymphadenopathy. She has no other medical conditions and takes no medications.
The patient is married and has 2 children. She works as an elementary school teacher. On examination, there is
tenderness of the involved joints without swelling or redness. The remainder of the physical examination is
unremarkable. Which of the following is most likely elevated in this patient?

A) Anti-cyclic citrullinated peptide antibodies

B) Anti-double-stranded DNA antibodies

C) Antinuclear antibodies

D) Anti-parvovirus B19 IgM antibodies

E) Anti-streptolysin titer

F) Cryoglobulin levels

G) Rheumatoid factor
Explanation
Correct Answer:

D) Anti-parvovirus B19 IgM antibodies

Parvovirus B19 infection

• Most patients are asymptomatic or have flulike symptoms


• Erythema infectiosum (fifth disease): Fever, nausea & “slapped cheek” rash (more
Signs & common in children)
symptoms • Acute, symmetric arthralgia/arthritis: Hands, wrists, knees & feet (resembles RA)
• Transient pure red cell aplasia; aplastic crisis in patients with underlying hematologic
disease (eg, sickle cell)

• Acute infection
◦ B19 IgM antibodies in immunocompetent patients
Diagnosis ◦ NAAT for B19 DNA in immunocompromised patients
• Previous infection: B19 IgG antibodies (documents immunity)
• Reactivation of previous infection: NAAT for B19 DNA

NAAT = nucleic acid amplification testing; RA = rheumatoid arthritis.

This patient's acute symmetric arthralgias most likely represent viral arthritis secondary to parvovirus B19.
Parvovirus is among the most common causes of acute viral arthritis (others include hepatitis B and C, HIV, and
rubella). Parvovirus infection most commonly affects adults who have frequent contact with children (eg, school
teachers, daycare workers). Children with parvovirus infection often develop a "slapped cheek" rash as part of
erythema infectiosum. Nearly 75% of adults with parvovirus infection develop a nonspecific rash, but <20% have
the characteristic erythema infectiosum rash.

Adults most commonly develop a polyarticular, symmetric arthritis (60% of cases) involving peripheral joints,
including the hands (metacarpophalangeal [MCP], proximal interphalangeal [PIP], and wrist), knees, and ankles.
Arthritis due to parvovirus B19 typically does not cause joint destruction or chronic arthritis. Other possible
nonspecific findings include fever, fatigue, and diarrhea. Parvovirus B19 infection is diagnosed by detecting anti-
B19 IgM antibodies in the serum, which develop within 10-15 days of infection and usually remain positive for 1-6
months. The symptoms typically resolve spontaneously in 2-3 weeks without the need for specific treatment.

(Choices A and G) Anti-cyclic citrullinated peptide antibodies and rheumatoid factor are usually associated with
rheumatoid arthritis (RA). RA presents with joint stiffness ≥1 hour in the morning, joint swelling on examination,
and symptoms for >6 weeks. Although parvovirus can affect similar joints (eg, MCP, PIP, wrist), this patient's
acute symptom onset (10 days) and absence of both joint swelling and prolonged joint stiffness in the morning make
RA less likely. In addition, RA would be unusual at age 27.

(Choices B and C) Antinuclear antibody testing is highly sensitive for systemic lupus erythematosus (SLE); anti-
double-stranded DNA antibodies are highly specific for the diagnosis. There is significant overlap in the
appearance of parvovirus-related arthritis and the arthritis of SLE, but SLE most often causes chronic rather than
acute arthritis. Other findings of SLE include malar or discoid rash, hematologic abnormalities (eg, anemia), renal
disease, fever, malaise, and weight loss. This patient's short duration of joint pain and absence of other findings
make SLE unlikely.

(Choice E) Anti-streptolysin titers are positive in patients with acute rheumatic fever and indicate recent
streptococcal infection. Typical findings include migratory polyarthritis, pancarditis, Sydenham's chorea, erythema
marginatum, and subcutaneous nodules. However, the arthritis of rheumatic fever is usually migratory and initially
affects lower-extremity joints.

(Choice F) Mixed cryoglobulinemia is usually associated with chronic hepatitis C and can cause arthralgias. The
arthralgias of mixed cryoglobulinemia usually occur in association with a chronic vasculitic syndrome characterized
by palpable purpura, lymphadenopathy, nephropathy, and neuropathy.
Educational objective:
Parvovirus B19 can cause an acute symmetric arthritis of the hands (metacarpophalangeal, proximal
interphalangeal, and wrist), knees, and ankle joints. The diagnosis can be confirmed with anti-parvovirus B19 IgM.
The syndrome is usually self-limited and does not require specific treatment.

Reference
• Parvovirus B19: its role in chronic arthritis.
Question #254

A 46-year-old man comes to the urgent care clinic due to genital lesions. Three days ago, the patient noticed a
papule on the shaft of his penis, which has since evolved into an ulcer. Two similar lesions have developed on the
glans penis in the last 24 hours. The patient has had multiple female sexual partners in the last 3 months and uses
condoms inconsistently. He has no other symptoms. Examination reveals a 2-cm, nonpainful, ulcerated lesion on
the shaft of the penis with indurated borders and no purulent drainage. There are 2 smaller, similar lesions on the
glans penis. Mild, bilateral, nontender inguinal lymphadenopathy is present. Which of the following is the most
likely diagnosis?

A) Behçet disease

B) Chancroid

C) Herpes simplex infection

D) Lymphogranuloma venereum

E) Syphilis
Explanation
Correct Answer:

E) Syphilis

Infectious genital ulcers

• Pustules, vesicles, or small ulcers on


erythematous base
Herpes simplex virus
• Tender lymphadenopathy
• Systemic symptoms common

Painful
• Larger, deep ulcers with gray/yellow
exudate
Haemophilus ducreyi • Well-demarcated borders & soft,
(chancroid) friable base
• Severe lymphadenopathy that may
suppurate

• Usually single ulcer (chancre)


Treponema pallidum
Painless • Indurated borders & hard,
(syphilis)
nonpurulent base
Chlamydia trachomatis • Initial small, shallow ulcers (often
serovars L1-L3 missed)
(lymphogranuloma • Then painful & fluctuant adenitis
venereum) (buboes)

Genital ulcers are typically caused by sexually transmitted infection. Although diagnostic confirmation requires
testing, the causative etiology is often inferred based on patient history, the presence or absence of pain, and visual
inspection of the lesions, as follows:

• History: Genital ulcers due to infectious organisms are most common in those who have unprotected sex
with multiple partners. Genital ulcers in the United States are usually caused by herpes simplex virus
(HSV) or syphilis, whereas patients who have recently returned from travel overseas are at increased risk for
tropical and subtropical infections such as chancroid or lymphogranuloma venereum (LGV).

• Pain: Painless ulcers usually indicate syphilis or LGV whereas pain is typically seen with HSV or
chancroid.

• Appearance: The number and size of lesions, the presence/absence of painful lymphadenopathy, and the
presence/absence of purulence informs the likely diagnosis.

This patient's painless ulcer with indurated borders, a nonpurulent base, and bilateral, painless
lymphadenopathy is consistent with syphilis (chancre). Although patients with primary syphilis typically have a
single chancre at the site of inoculation, multiple chancres occasionally occur, particularly in those with underlying
HIV.

(Choice A) Behçet disease is marked by recurrent mucocutaneous ulcers; oral ulcers are most common, but
genital ulcers can occur. However, the ulcers are painful, round, and have a white-yellow necrotic base with
surrounding erythema. In addition, men usually develop Behçet ulcers on the scrotum.

(Choice B) Chancroid is caused by Haemophilus ducreyi and is marked by one or more painful ulcers with
purulence and an erythematous base. Painful inguinal lymphadenitis may occur, leading to buboes. Most cases
arise in the developing world; it is very uncommon in the United States.

(Choice C) HSV can cause multiple small, painful ulcers with an erythematous base. Most patients with initial
infection have systemic, nonspecific symptoms and tender lymphadenopathy.

(Choice D) LGV is an infection with serovars L1-L3 of Chlamydia trachomatis. Although it causes multiple painless
ulcers, they tend to be very small (<6 mm) and disappear within a few days. In addition, LGV is very uncommon in
the United States.

Educational objective:
In the United States, genital ulcers are most often due to herpes simplex virus or syphilis. In developing regions,
ulcers can also be caused by chancroid or lymphogranuloma venereum. Painless ulcers with indurated borders
and a clean base are most often due to primary syphilis (chancre); although most cases are marked by a single
chancre, multiple chancres can occur, particularly in those with HIV.
Question #255

A 22-year-old woman, gravida 1 para 0, at 12 weeks gestation comes to the office due to 3 days of rash, fever, and
malaise. The rash seems to be getting larger but does not hurt or itch. Two weeks ago, the patient went camping
in northern Massachusetts. She has no known drug allergies. Temperature is 38.3 C (100.9 F), blood pressure is
110/80 mm Hg, pulse is 88/min, and respirations are 16/min. The rash is shown below.
The remainder of the examination is normal. Which of the following is the most appropriate next step in
management of this patient?
A) Amoxicillin

B) Azithromycin

C) Ciprofloxacin

D) Clindamycin

E) Observation
Explanation
Correct Answer:

A) Amoxicillin

This patient camped in a Lyme-endemic area and subsequently developed nonspecific systemic symptoms and a
target-like rash, raising strong suspicion for early localized Lyme disease. Lyme disease is a spirochetal infection
(Borrelia burgdorferi) transmitted by the deer tick Ixodes scapularis. Most patients develop manifestations of early,
localized Lyme disease 7-14 days after transmission. Spirochetal replication in the dermis generates the rash of
erythema migrans—a slowly expanding, macular lesion that eventually forms areas of central clearing (bull's-eye
rash). Systemic symptoms (eg, fever, headache, myalgia, fatigue) commonly occur.

First-line treatment for early localized Lyme disease in nonpregnant adults is doxycycline due to excellent systemic
penetration and efficacy against other pathogens (eg, Anaplasma phagocytophilum) commonly transmitted by I
scapularis. In pregnant women, the use of doxycycline is more controversial (eg, possible risk for fetal tooth
discoloration and retardation of skeletal development) and is generally considered on a case-by-case basis (eg,
short therapy duration, gestational age). In this patient population, oral amoxicillin is most often used in lieu of
doxycycline for the treatment of Lyme disease. Azithromycin is less effective than amoxicillin and doxycycline for
Lyme disease and is generally reserved for patients with allergies to both of these medications (Choice B). Lyme
disease during pregnancy does not appear to cause harm to the fetus.

(Choice C) Ciprofloxacin is used primarily to treat gram-negative enteric pathogens. It is not effective against
Lyme disease.

(Choice D) Clindamycin can be used for pyogenic skin infections but is not a first-line agent for Lyme disease.
This patient who recently went camping in a Lyme disease–endemic area and now has systemic symptoms and a
target-like rash likely has early, localized Lyme disease.

(Choice E) Observation would lead to spirochetal dissemination and a high risk for later complications (eg, facial
nerve palsy, aseptic meningitis, heart block, arthritis).
Educational objective:
Pregnant patients with early, localized Lyme disease are typically treated with amoxicillin. This treatment is
generally curative and does not pose a risk to the fetus.
Question #256

A 42-year-old man comes to the office after attempting to donate blood, when he was found to have abnormal viral
serologic results for hepatitis. The patient feels well and takes no medications. He has no history of liver disease,
but his father died of complications of alcoholic cirrhosis. The patient drinks 1 or 2 beers on weekdays and 5 or 6
on the weekends. He used injection drugs for several years in his 20s but not recently. The patient does not use
tobacco. He has had several lifetime sexual partners but is now in a monogamous relationship. Temperature is
37.1 C (98.8 F), blood pressure is 136/85 mm Hg, and pulse is 84/min. BMI is 34.2 kg/m2. A smooth, nontender
liver edge is palpable 3 cm below the right costal margin; otherwise, the physical examination is unremarkable.
Laboratory results are as follows:

Aspartate aminotransferase (SGOT) 156 U/L


Alanine aminotransferase (SGPT) 214 U/L

Hepatitis panel
Hepatitis B surface antibody positive
Hepatitis B surface antigen negative
Hepatitis C virus antibody positive

Which of the following is the most appropriate next step in management of this patient?

A) Abdominal CT scan

B) Hepatitis B vaccination

C) Hepatitis C virus RNA PCR testing

D) Ledipasvir-sofosbuvir therapy
E) Liver biopsy
Explanation
Correct Answer:

C) Hepatitis C virus RNA PCR testing

This patient has elevated transaminases, hepatomegaly, and a positive serologic test for hepatitis C virus (HCV)
antibody. This suggests exposure to HCV and warrants testing to confirm the diagnosis of chronic HCV infection.
Because the virus may spontaneously clear in some patients, the diagnosis of chronic HCV infection is a 2-step
process requiring both a positive serologic test for the anti-HCV IgG antibody and a confirmatory molecular test
for circulating HCV RNA.

Once the diagnosis has been confirmed, patients should undergo further evaluation to determine candidacy for
treatment with direct-acting antiviral agents (eg, ledipasvir-sofosbuvir) (Choice D). This often involves
noninvasive tests (eg, laboratory tests, ultrasound-based transient elastography) to assess the extent of liver
fibrosis and, sometimes, HCV genotype testing. Pregnancy testing is recommended if a ribavirin-based regimen is
under consideration.

(Choice A) In patients with established chronic HCV (ie, HCV RNA positive), ultrasound-based transient
elastography, rather than CT scan, can be used to assess for fibrosis. Ultrasound is also preferred over CT scan for
screening for hepatocellular carcinoma in patients who have advanced HCV and cirrhosis.

(Choice B) This patient shows evidence of immunity to hepatitis B virus (positive hepatitis B surface antibody),
making vaccination unnecessary.

(Choice E) Although liver biopsy was once used to assess the extent of fibrosis in patients with confirmed chronic
HCV, noninvasive laboratory tests and ultrasound-based transient elastography are now preferred. There is no
need to assess for liver fibrosis in this patient unless HCV RNA testing is positive.

Educational objective:
Because hepatitis C virus may spontaneously clear in some patients, the diagnosis of chronic infection requires
both a positive serologic antibody test and a confirmatory molecular test for circulating hepatitis C virus RNA.
Reference
• Diagnosis and management of hepatitis C.
Question #257

A 36-year-old woman comes to the office due to a right lower extremity rash. The patient developed a large
abrasion over her right ankle after falling off her mountain bike 3 days ago. Yesterday, she noticed progressive,
painful red streaks near the wound. She has no chronic medical conditions and takes no medications.
Temperature is 38.2 C (100.8 F), blood pressure is 126/78 mm Hg, and pulse is 92/min. Examination reveals a
4-cm abrasion just superior to the right lateral malleolus; no purulence or induration is present, but there are several
tender, erythematous streaks extending proximally from the wound toward the knee. Tender, enlarged lymph nodes
are also present in the right popliteal fossa. Which of the following is the most appropriate treatment for this
patient?

A) Azithromycin

B) Cephalexin

C) Ciprofloxacin

D) Griseofulvin

E) Itraconazole

F) Penicillin V
Explanation
Correct Answer:

B) Cephalexin

Lymphangitis

• Cutaneous injury → pathogen invasion of lymphatics in deep dermis


Epidemiology
• Streptococcus pyogenes & MSSA

• Tender, erythematous streaks proximal to wound


Manifestations • Regional tender lymphadenopathy (lymphadenitis)
• Systemic symptoms (eg, fever, tachycardia)

Treatment • Cephalexin

MSSA = methicillin-sensitive Staphylococcus aureus.

This patient had a skin abrasion and subsequently developed tender, erythematous streaks extending towards her
knee and regional lymphadenopathy, raising strong suspicion for acute infectious lymphangitis. Most cases
develop in the setting of cutaneous wound, which creates a portal for skin pathogens to enter the deep dermis and
invade lymphatic channels. Patients generally have tender, erythematous streaks extending from the wound
toward the draining lymph nodes; regional tender lymphadenopathy (lymphadenitis) and systemic symptoms (eg,
fever) are often present.
The leading causes of lymphangitis are Streptococcus pyogenes and methicillin-sensitive Staphylococcus
aureus (MSSA); therefore, empiric treatment with cephalexin is generally curative.

(Choice A) Azithromycin is often used for upper respiratory infection due to excellent activity against a number of
gram-positive upper respiratory pathogens, including S pyogenes. However, it is not regularly used for skin and soft
tissue infection due to limited activity against MSSA.

(Choice C) Ciprofloxacin is primarily used for gram-negative bacterial infections, particularly those that arise below
the level of the diaphragm (eg, urinary tract, gastrointestinal). It has limited activity against gram-positive bacteria
and is not commonly employed for skin and soft tissue infections.

(Choice D) Griseofulvin is an antifungal agent that is primarily used to treat cutaneous fungal infections (eg,
ringworm). This patient with tender erythematous streaks and regional lymphadenitis following a cutaneous injury is
far more likely to have a bacterial infection.

(Choice E) Itraconazole is an antifungal agent that is often used to treat sporotrichosis, the leading cause of
nodular lymphangitis. In contrast to acute infectious lymphangitis, nodular lymphangitis is marked by the formation
of nodular swellings along the proximal lymphatic channel over the course of weeks (not days). Gardeners are
classically affected.

(Choice F) Although penicillin V has activity against S pyogenes, it is not typically employed for skin and soft tissue
infections because many strains of MSSA are resistant.

Educational objective:
Acute infectious lymphangitis is marked by the formation of proximal, tender, erythematous streaks at the site of
skin wound with regional lymphadenopathy and systemic symptoms (eg, fever). Most cases are caused by
Streptococcus pyogenes or methicillin-sensitive Staphylococcus aureus; therefore, treatment with cephalexin is
generally curative.
Question #258

A 26-year-old woman comes to the office due to a flulike illness with low-grade fever, malaise, nausea, and
anorexia. She came back from a trip to Honduras a week ago, and her symptoms started 2 days prior to her
return. The patient has no significant medical history and takes no medications. She occasionally smokes
marijuana and has used injection drugs in the past. She has had several sexual partners and currently lives with
her boyfriend. Temperature is 37.9 C (100.2 F), blood pressure is 121/78 mm Hg, and pulse is 86/min. The patient
has scleral icterus. Cardiopulmonary examination is unremarkable. The liver edge, palpated 4 cm below the right
costal margin, is smooth and tender. Laboratory results are as follows:

Total bilirubin 44.8 mg/dL


Aspartate aminotransferase 11,222 U/L
Alanine aminotransferase 21,184 U/L
Hepatitis panel
Hepatitis B surface antibody positive
Hepatitis B core antibody positive
Hepatitis B surface antigen negative
Hepatitis A IgM antibody positive
Hepatitis A IgG antibody negative
Hepatitis C antibody negative

After discussing the findings, the patient says she is worried that she may transmit the disease to her boyfriend.
Which of the following interventions for the boyfriend is most likely to decrease his risk of disease acquisition?

A) Barrier contraceptive use


B) Hepatitis A vaccination

C) Hepatitis B immune globulin

D) Hepatitis B vaccination

E) Hepatitis B vaccine and immune globulin

F) No intervention necessary
Explanation
Correct Answer:

B) Hepatitis A vaccination

This patient's presentation and laboratory findings are consistent with both acute hepatitis A virus infection
(markedly elevated transaminases, positive hepatitis A IgM antibody [Ab], negative hepatitis A IgG Ab, recent travel
to Honduras) and prior hepatitis B virus infection (positive hepatitis B core Ab, positive hepatitis B surface Ab,
history of intravenous drug use). Since the introduction of routine hepatitis A vaccination in the United States,
infection rates have fallen significantly, but disease transmission remains a risk for a number of groups, including
international travelers, men who have sex with men, illicit drug users, and those with household or sexual contact
with infected persons. Transmission occurs most frequently via the fecal-oral route, although sexual and blood-
borne transmission can also occur.

Postexposure prophylaxis with hepatitis A vaccine or hepatitis A immune globulin should be considered in a
number of groups, including:

• Close personal contacts (eg, sexual contacts, household contacts) of hepatitis A-infected patients
• Child care center contacts (eg, staff and children) where staff, children, or household contacts of attendees
have been infected
• Food preparation workers whose coworkers have been infected

Prophylaxis should be given within 2 weeks of exposure; in general, younger patients (age <40) should receive
hepatitis A vaccine, whereas older patients (age >41) should receive hepatitis A immune globulin.

(Choice A) Although sexual transmission of hepatitis A virus may occur occasionally, barrier contraceptive use
alone would not provide adequate protection against the most common route of transmission (fecal-oral).

(Choices C, D, and E) This patient's viral markers imply previous hepatitis B infection (core Ab) with resolution
(surface Ab), making further vaccination and treatment for hepatitis B unnecessary. Postexposure prophylaxis with
hepatitis B immune globulin and vaccine would be administered to a patient with a percutaneous or sexual
exposure to an individual with positive hepatitis B surface antigen.

Educational objective:
Postexposure prophylaxis with either hepatitis A vaccine or hepatitis A immune globulin should be considered in a
number of groups, including those with close personal contact (eg, sexual contacts, household contacts) with
infected patients.

Reference
• Hepatitis A.
Question #259

A 25-year-old man comes to the office due to right hand pain and swelling. He injured the skin over his right
knuckles when he punched a man in the teeth during a bar fight 3 days ago. The patient ignored the wound initially,
but his hand became more painful and swollen yesterday. He has no medical history and takes no medications.
The patient received a tetanus toxoid vaccination at age 18. Temperature is 37.3 C (99.1 F), blood pressure is 120/
70 mm Hg, and pulse is 78/min. There are small lacerations over the right third and fourth metacarpophalangeal
joints with surrounding erythema, swelling, and tenderness. No discharge or crepitus is present. Hand radiographs
reveal no foreign body or bone/joint changes. Cultures are obtained. Which of the following empiric antibiotics is
most appropriate for this patient?

A) Amoxicillin-clavulanate

B) Ampicillin

C) Ciprofloxacin

D) Clindamycin

E) Erythromycin
Explanation
Correct Answer:

A) Amoxicillin-clavulanate

This patient experienced a bite wound from a clenched fist ("fight-bite") injury. Human bite wounds are prone to
polymicrobial infection with aerobic and anaerobic oral flora. The most common organisms include streptococci,
Staphylococcus aureus, Eikenella corrodens, Haemophilus influenzae, and beta-lactamase–producing anaerobic
bacteria.

Patients often initially ignore a wound until pain, swelling, or purulent discharge develops. Blood and wound
cultures should be sent; empiric antibiotics are required. Although large studies on empiric antibiotic selection for
human bite wounds are scarce, amoxicillin-clavulanate is often the treatment of choice due to excellent coverage
of gram-positive, gram-negative, and beta-lactamase–producing oral anaerobic organisms (as clavulanate is a beta-
lactamase inhibitor). Surgical debridement is usually necessary, and wounds are typically left open to drain and
heal by secondary intention (due to high infection risk with closure). Tetanus vaccination should be administered
to those who are not up to date.

(Choice B) Ampicillin can be used for upper respiratory tract infections (eg, pharyngitis) and is effective against
many gram-positive and gram-negative organisms. However, it does not provide coverage for beta-
lactamase–producing flora and is often combined with the beta-lactamase inhibitor sulbactam in an intravenous
formulation.

(Choice C) Ciprofloxacin covers many gram-negative organisms and some gram-positive organisms (excluding
most streptococci), but it does not provide reliable coverage for anaerobic organisms. It is often used for
genitourinary and gastrointestinal infections.

(Choice D) Clindamycin is effective against gram-positive bacteria and anaerobes but not against most gram-
negative organisms (eg, Eikenella corrodens). It is used for some lung abscesses, skin and soft-tissue infections,
and female upper reproductive tract infections (anaerobic coverage in combination with other agents).
(Choice E) Erythromycin is sometimes used as a second-line agent in the treatment of gonococcal or chlamydial
urethritis. Erythromycin covers some atypical organisms (eg, Mycoplasma), but overall it does not provide
adequate gram-negative or anaerobic coverage.

Educational objective:
Human bite wounds often result in polymicrobial infections with aerobic and anaerobic oral organisms. Empiric
treatment with amoxicillin-clavulanate provides adequate coverage for the majority of virulent oral bacteria.
Question #260

A 24-year-old woman comes to the office due to 3 days of left eye redness and discharge, with crusting of the eyelid
and difficulty opening it in the morning. Today, the patient's right eye also became red. She has had no pain,
itching, photophobia, or visual difficulty. Her boyfriend has had similar symptoms. The patient has no chronic
medical conditions and takes no medications. Physical examination shows bilateral conjunctival erythema. There
is thick, yellowish discharge at the corner of the eyes that quickly reaccumulates after wiping. The cornea is
normal, and no ocular tenderness is present. Which of the following pathogens is most likely responsible for this
patient's condition?

A) Adenovirus

B) Chlamydia trachomatis

C) Herpes simplex virus

D) Neisseria gonorrhoeae

E) Pseudomonas aeruginosa

F) Staphylococcus aureus

G) Staphylococcus epidermidis
Explanation
Correct Answer:

F) Staphylococcus aureus

Acute conjunctivitis*

Viral Bacterial Allergic

Distribution Unilateral or bilateral Unilateral or bilateral Bilateral

Discharge Watery/mucoid Purulent Watery

Conjunctival Diffuse injection;


Diffuse injection; nonfollicular Diffuse injection; follicular ("bumpy")
appearance follicular ("bumpy")

Associated Unremitting discharge Ocular pruritus, history of atopy (eg,


Viral prodrome
findings (reaccumulates within minutes) allergic rhinitis, asthma)

<30 minutes (often sudden onset) to


Duration 1-2 weeks 1-2 weeks
perennial
*Red flags suggestive of alternate etiology: decreased visual acuity, photophobia, pain with extraocular
movement & fixed/distorted pupil.

This patient with conjunctival erythema and thick eye discharge likely has bacterial conjunctivitis. Most cases
occur after direct contact with secretions of an infected individual. Although bacterial conjunctivitis in children is
caused by a wide range of pathogens (eg, Haemophilus influenzae, Moraxella catarrhalis, Streptococcus
pneumoniae), most cases in adults are caused by Staphylococcus aureus.

Bacterial and viral conjunctivitis frequently affect both eyes and cause conjunctival erythema, discharge, and a
"stuck shut" eye when waking. However, differentiation can often be made based on the quality of the eye
discharge, as follows:

• Bacterial conjunctivitis causes thick, purulent eye discharge that continues throughout the day and night;
the discharge is typically present at the corners of the eye and reaccumulates within minutes after wiping.
• Viral conjunctivitis (eg, adenovirus) is usually marked by scant, stringy, and watery discharge that causes a
gritty sensation in the eye. Viral conjunctivitis is also frequently associated with other upper respiratory
manifestations (eg, rhinorrhea, sore throat, cough) (Choice A).

(Choice B) Chlamydia trachomatis can cause bacterial conjunctivitis but is a much less common cause than S
aureus. In addition, most cases are marked by long-standing symptoms (eg, weeks or months) and concurrent
urogenital infection.

(Choice C) Herpes simplex virus can cause a vision-threatening infection of the cornea. Although patients
frequently have corneal injection, most cases are marked by eye pain, blurred vision, watery discharge, and
characteristic dendritic lesions seen on the cornea by slit lamp.

(Choice D) Neisseria gonorrhoeae causes a rapidly progressive conjunctivitis with copious purulent discharge.
Eye tenderness and a concomitant urogenital infection are typically present. It can also be seen in neonates when
the mother is infected.

(Choice E) Pseudomonas aeruginosa eye infection is classically seen in patients who wear extended wear contact
lenses. It typically causes an ulcerative keratitis associated with eye pain.

(Choice G) Staphylococcus epidermidis is not a common cause of bacterial conjunctivitis but can occasionally
cause endophthalmitis in those with other nidi of infection (eg, infective endocarditis). This is generally marked by
blurred vision and eye pain, not conjunctival erythema and discharge.

Educational objective:
Bacterial conjunctivitis should be suspected in patients with conjunctival erythema and thick, purulent eye discharge
that reaccumulates within a few minutes after wiping. Most cases occur after direct contact with an infected
individual, and one or both eyes may be affected. S aureus is the most common etiology in adults.
Question #261

A 54-year-old man comes to the office due to 2 weeks of progressive fever, exertional dyspnea, and nonproductive
cough. The patient has a history of HIV infection but does not comply with antiretroviral therapy. Temperature is
38.9 C (102 F), blood pressure is 120/80 mm Hg, pulse is 100/min, and respirations are 28/min. Pulse oximetry is
80% on room air. Oxygen saturation increases to 92% with the use of a 100% nonrebreather mask. Laboratory
results are as follows:

Complete blood count


Hemoglobin 9.6 g/dL
Platelets 120,000/mm3
Leukocytes 8,000/mm3 (no bands)

Arterial blood gases on room air


pH 7.45
PaO2 54 mm Hg
PaCO2 44 mm Hg

CD4 count is 170/mm3 and lactate dehydrogenase level is 480 U/L. Chest x-ray reveals diffuse bilateral interstitial
infiltrates. What is the most appropriate next step in management of this patient?

A) Ceftriaxone

B) Initiation of antiretroviral treatment

C) Pentamidine and corticosteroids


D) Trimethoprim-sulfamethoxazole and corticosteroids

E) Trimethoprim-sulfamethoxazole only
Explanation
Correct Answer:

D) Trimethoprim-sulfamethoxazole and corticosteroids

This patient with HIV has subacute dyspnea, fever, dry cough, elevated lactate dehydrogenase (LDH), and diffuse
bilateral pulmonary infiltrates, raising strong suspicion for Pneumocystis pneumonia (PCP). Most cases arise in
those with untreated AIDS who have CD4 counts <200/mm3. Manifestations are generally indolent and include
exertional dyspnea, dry cough, and fever. Laboratory evaluation frequently reveals elevated LDH, and chest x-ray
usually shows bilateral reticulonodular interstitial infiltrates. The diagnosis is confirmed when Pneumocystis jirovecii
is identified in induced sputum or bronchoalveolar lavage. Treatment with trimethoprim-sulfamethoxazole (TMP-
SMX) is generally curative.

Because P jirovecii proliferates within the alveoli and triggers a strong inflammatory response, patients with PCP
often have significant hypoxia and a large alveolar-arterial oxygen gradient. Hypoxia often worsens with the
initiation of antimicrobial treatment due to the release of pro-inflammatory intracellular macromolecules during lysis
of the organism. Therefore, concomittant corticosteroids are generally administered to those with severe PCP to
reduce the inflammatory response. Indications include:

• a PaO2 ≤70 mm Hg
• an alveolar-arterial (A-a) gradient ≥35 mm Hg, or
• pulse oximetry <92% on room air.

This patient's PaO2 of 54 mm Hg on room air indicates a need for concomitant corticosteroids, not TMP-SMX alone
(Choice E).

Alternate oral regimens for mild/moderate PCP include dapsone-TMP, primaquine with clindamycin, or atovaquone
suspension. Alternate regimens for moderate/severe disease include intravenous (IV) pentamidine or primaquine
with IV clindamycin. Because pentamidine has high rates of adverse effects (eg, hypotension, hypoglycemia,
nephrotoxicity, arrhythmias), it is generally reserved for patients with severe PCP who cannot tolerate TMP-SMX
(Choice C). There is no evidence that this patient is unable to tolerate TMP-SMX.

(Choice A) Patients with bacterial pneumonia usually have productive cough, lobar infiltrate on chest x-ray, and an
elevated circulating bands. Community-acquired bacterial pneumonia is often treated with ceftriaxone, but
concomitant azithromycin or doxycycline is usually added to cover atypical organisms.

(Choice B) Antiretroviral therapy initiation is usually delayed 1-2 weeks in patients with PCP (until hypoxia is
improved) due to the risk of worsened respiratory status from immune reconstitution syndrome. A transient delay in
antiretroviral therapy also helps limit drug interactions and ensures that patients are tolerating PCP treatment.

Educational objective:
Manifestations of Pneumocystis pneumonia (PCP) in patients with HIV are usually indolent and include
nonproductive cough, exertional dyspnea, fever, hypoxia, elevated lactate dehydrogenase, and bilateral interstitial
infiltrates. Trimethoprim-sulfamethoxazole is the first-line treatment. Concomitant corticosteroids are administered
for those with PaO2 ≤70 mm Hg, alveolar-arterial gradient ≥35 mm Hg, or pulse oximetry <92% on room air to
reduce risk of respiratory decompensation.

Reference
• Prevention and treatment of opportunistic infections in HIV-infected adults and adolescents: updated
guidelines from the Centers for Disease Control and Prevention, National Institutes of Health, and HIV
Medicine Association of the Infectious Diseases Society of America.
Question #262

A 26-year-old man with a history of Crohn disease is being evaluated for initiation of anti–tumor necrosis factor
therapy. The patient has had progressively severe ileocolitis despite taking glucocorticoids for several weeks.
Interferon-gamma release assay is performed. During the test, the interferon-gamma level in the patient's blood
sample is measured at baseline and after incubating the sample with control antigens and mycobacterial antigens.
The results are as follows:

Sample Interferon-gamma level Reference range


Baseline 0.05 IU/mL <8 IU/mL
With control antigens 0.07 IU/mL >0.5 IU/mL above baseline
With mycobacterial antigens 0.00 IU/mL

HIV testing is negative. Which of the following best explains the results of this patient's interferon-gamma release
assay?

A) Active tuberculosis

B) Bacille Calmette-Guérin vaccination

C) Concurrent inflammation

D) Latent tuberculosis

E) Medication effect
Explanation
Correct Answer:

E) Medication effect

Exposure of this patient's blood to a control agent does not significantly stimulate interferon-gamma release above
baseline, indicating a lack of lymphocyte response to a strong stimulatory mitogen. This is considered an
indeterminate result because lymphocytes are unable to release significant quantities of interferon-gamma
whether or not they have been previously exposed to Mycobacterium tuberculosis antigens.

A minimal response to control antigens is most often seen with lymphocyte immunosuppression due to HIV or
the use of immunomodulatory medications (eg, glucocorticoids). In patients with HIV, testing should be repeated
when CD4 counts improve with antiretroviral therapy; in those taking immunomodulatory medications, testing
should be repeated after these medications have been discontinued for a couple of weeks. This usually restores
the ability of lymphocytes to generate interferon-gamma to a control mitogen, thereby allowing interpretation of
interferon-gamma release in response to M tuberculosis antigens.

Evaluation for latent tuberculosis infection with either the interferon-gamma release assay (IGRA) or tuberculin
skin test (TST) should be performed prior to the initiation of anti–tumor necrosis factor therapy (which increases the
risk of reactivation tuberculosis). Because IGRA uses antigens fairly specific for tuberculous mycobacteria, it is
more sensitive and specific than TST. Furthermore, because IGRA identifies antigens that are not present in
Mycobacterium bovis, test results are not typically affected by previous vaccination with bacille Calmette-Guérin (a
live strain of M bovis sometimes used as a vaccine against tuberculosis) (Choice B).

(Choices A and D) Active and latent tuberculosis are both marked by elevated levels of interferon-gamma in
response to mycobacterial antigens (positive IGRA); definitive diagnosis of active TB also requires microbial
diagnosis (identification of the organism in culture). In this case, however, the lack of response to control antigens
makes the IGRA results to mycobacterial antigens impossible to interpret.

(Choice C) Indeterminate IGRA results can also be seen with concurrent inflammation or ongoing non-TB
infections that provoke interferon-gamma release. However, in such situations, patients would have high baseline
levels of interferon-gamma (unlike in this patient).

Educational objective:
Interferon-gamma release assay measures lymphocyte response to tuberculous antigens. An indeterminate result
can occur when patients have high background interferon-gamma levels due to ongoing inflammation or concurrent
infection; it can also occur when there is minimal response to control mitogens due to lymphocyte suppression from
HIV or immunomodulatory medications (eg, glucocorticoids).
Question #263

A 32-year-old previously healthy woman comes to the clinic for evaluation of a rash on the soles of her feet and
palms of her hands for 2 days. She also has had intermittent fever, chills, decreased appetite, and malaise. Three
days before the rash appeared, the patient visited a children's water park with her 2-year-old niece. She has no
known chronic medical conditions. Temperature is 37.8 C (100 F), blood pressure is 113/78 mm Hg, pulse is 65/
min, and respirations are 14/min. Oropharyngeal examination shows several ulcers on the soft palate, buccal
mucosa, and posterior oropharynx; gingiva and dentition are normal. Examination of the skin on her sole, foot, and
hands is shown in the exhibits. The remainder of the examination is unremarkable. Which of the following is the
best next step in management of this patient?

A) Antinuclear antibodies

B) Blood cultures and echocardiogram

C) No additional testing

D) Skin biopsy with direct immunofluorescence

E) Throat culture
Explanation
Correct Answer:

C) No additional testing

Hand-foot-and-mouth disease

• Coxsackievirus infection
• Any age, usually age <7
Pathogenesis &
• Transmission: direct contact with respiratory, oral, vesicular secretions or
epidemiology
fecal-oral spread
• Peaks during summer/fall

• Painful vesicles/ulcers on anterior oral mucosa


Clinical features • Macules/papules/vesicles on palms, soles, buttocks
• ± Systemic symptoms (eg, fever, malaise)

• Myopericarditis
Complications • Aseptic meningitis
• Nail dystrophy (1-2 months later)

Management • Supportive (eg, pain control, hydration)

This patient has oral ulcers and a rash on her palms and soles, which are findings characteristic of hand-foot-and-
mouth disease (HFMD) caused by coxsackievirus. Transmission is via fecal-oral spread or contact with
respiratory droplets, oral secretions, or vesicular fluid, and infection often occurs in outbreaks involving communal
settings (eg, water parks). HFMD usually affects children age <7 but can present in adolescents and adults, as in
this patient.

HFMD presents with a painful oral enanthem consisting of vesicles/ulcers usually affecting the tongue, buccal
mucosa, or soft palate as well as a maculopapular or vesicular exanthem on the hands and feet (including the
palms and soles); the rash is typically nonpruritic but may be painful. Systemic symptoms (eg, fever, decreased
appetite, malaise) can also occur, as seen here.

Diagnosis of HFMD is clinical (ie, no additional testing required), and management is supportive care (eg,
hydration, analgesics) because the illness is typically self-limited.

(Choice A) Antinuclear antibodies are usually positive in systemic lupus erythematosus (SLE), which can present
with systemic symptoms (eg, fever, malaise), rash, and oral ulcers. However, unlike this case, the malar rash in
SLE involves the cheeks/nasal bridge, not the palms and soles, and the ulcers are painless.

(Choices B) Valvular vegetations on echocardiogram and positive blood cultures are findings in infective
endocarditis (IE), which may present with Janeway lesions (painless, erythematous macules) or Osler nodes
(painful, violaceous nodules) on the palms and soles. However, most patients have associated risk factors (eg,
structural heart disease, poor dentition) and a murmur, findings not seen in this patient. In addition, oral ulcerations
do not occur.

(Choice D) Skin biopsy with direct immunofluorescence can diagnose IgA vasculitis (Henoch-Schönlein purpura),
which causes palpable purpura on gravity-dependent areas, such as the buttocks and lower extremities. However,
maculopapular lesions of the palms and oral ulcerations would not be expected.

(Choice E) Acute rheumatic fever due to untreated group A streptococcal pharyngitis may cause a truncal rash, or
erythema marginatum, and subcutaneous nodules over bony areas, not erythematous lesions on the palms and
soles. In addition, preceding infection (which is not associated with discrete oral lesions) is typically resolved, so
oropharyngeal examination would be normal and throat culture would be negative.

Educational objective:
Hand-foot-and-mouth disease caused by coxsackievirus presents with oral vesicles and ulcers in addition to a
maculopapular or vesicular exanthem classically involving the palms and soles. Diagnosis is clinical, and treatment
is supportive.

Reference
• Hand-foot-and-mouth disease: rapid evidence review.
Question #264

A 42-year-old man comes to the office due to a week of subjective fever, sore throat, malaise, headache, and skin
rash. The rash began on his trunk a week ago and has now spread to his entire body. He has had no chest pain,
shortness of breath, diarrhea, or urethral discharge. The patient was diagnosed with hypertension 2 months ago
and started on lisinopril. He has had 3 new female sexual partners over the last year. He drinks alcohol
occasionally but does not smoke or use illicit drugs. The patient has not participated in any unusual outdoor
activities. Temperature is 37.2 C (99 F), blood pressure is 130/80 mm Hg, pulse is 78/min, and respirations are 16/
min. Examination shows a full-body maculopapular rash without evidence of excoriations. Several raised, grey
mucosal patches are seen in the mouth. Cervical, axillary, inguinal, and epitrochlear lymphadenopathy is present.
HIV testing is negative. Which of the following is the most likely cause of this patient's current symptoms?

A) Bacterial endocarditis

B) Drug eruption

C) Epstein-Barr virus infection

D) Gonococcal infection

E) Rickettsial infection

F) Syphilis
Explanation
Correct Answer:

F) Syphilis

Syphilis manifestations

Primary • Painless genital ulcer (chancre)

• Diffuse rash (palms & soles)


• Lymphadenopathy (epitrochlear)
Secondary • Condyloma latum
• Oral lesions
• Hepatitis

Latent • Asymptomatic

• CNS (tabes dorsalis, dementia)


Tertiary • Cardiovascular (aortic aneurysm/insufficiency)
• Cutaneous (gummas)

CNS = central nervous system.


Secondary syphilis typically occurs weeks to months after patients are exposed to Treponema pallidum and
develop primary syphilis (chancre). Secondary syphilis is characterized by systemic symptoms (fever, malaise,
sore throat, headache), widespread lymphadenopathy (LAD), grey mucous patches, raised grey genital papules
(condylomata lata), and a diffuse maculopapular rash that begins on the trunk, extends to the extremities, and
involves the palms and soles. The presence of epitrochlear LAD is particularly characteristic of secondary syphilis;
it is rumored that sailors would routinely perform a 2-handed "sailor's handshake" (with one hand on the elbow) to
determine if potential partners had epitrochlear nodes prior to engaging their company.

Syphilis is diagnosed using a combination of nontreponemal (eg, rapid plasma reagin) and treponemal-specific
(eg, T pallidum enzyme immunoassay) serologic tests. Treatment for secondary syphilis is the same as for primary
syphilis—one dose of intramuscular penicillin G benzathine—which provides up to 3 weeks of treatment-dose
penicillin. Adequate treatment is confirmed by a 4-fold decrease in serologic titers at 6-12 months.

(Choice A) Bacterial endocarditis often causes nonspecific symptoms and cutaneous findings (eg, petechiae,
splinter hemorrhages). A diffuse maculopapular rash and LAD would be uncommon.

(Choice B) Drug reactions may cause maculopapular rash but not sore throat or grey mucous patches.

(Choice C) Epstein-Barr virus (EBV) causes sore throat, fever, fatigue, and LAD (infectious mononucleosis).
Maculopapular rash may occur but is uncommon. Grey mucous patches are not a common feature of adult EBV
infection.

(Choice D) Disseminated gonococcus presents with tenosynovitis, polyarthralgia, and a skin rash that usually
consists of a few pustules. This patient has a diffuse rash, LAD, and sore throat, making syphilis far more likely.

(Choice E) Rocky Mountain spotted fever (RMSF) is a tick-borne rickettsial infection marked by high fever,
headache, malaise, and a maculopapular rash that usually spreads centripetally toward the trunk (unlike in this
patient), includes the palms and soles, and becomes petechial over time. Epidemic typhus is a rare, louse-borne
rickettsial infection associated with the abrupt onset of fever, severe headache, and malaise; a centrifugally-
spreading macular or maculopapular rash (typically sparing the palms and soles) develops several days later. This
patient's grey mucous patches, extensive LAD, multiple new sexual partners, and lack of outdoor exposure make
syphilis more likely.
Educational objective:
Secondary syphilis is characterized by systemic symptoms (fever, malaise), widespread lymphadenopathy
(particularly epitrochlear), and a diffuse maculopapular rash that begins on the trunk and extends to the extremities,
including the palms and soles. Diagnosis is made with serology (using both a treponemal and a nontreponemal
test). One intramuscular dose of penicillin G benzathine is the standard treatment.

Reference
• Prevention and treatment of sexually transmitted diseases: an update.

• Primary and secondary syphilis – United States, 2005-2013.


Question #265

A 22-year-old man comes to the office due to several small, pruritic skin pustules on his lower extremities. The
patient first noticed the lesions a week ago during a trip to New Mexico, where he hiked in the mountains and swam
in a heated hotel pool. He was given trimethoprim-sulfamethoxazole at an urgent care center, but the pustules have
persisted despite taking the medication as prescribed for 3 days. The patient has no chronic medical conditions
and takes no other medications. Temperature is 37.9 C (100.2), blood pressure is 132/68 mm Hg, and pulse is 96/
min. Examination shows several small, tender papulopustules on the bilateral lower extremities. Which of the
following pathogens is the most likely cause of this patient's symptoms?

A) Group A Streptococcus

B) Methicillin-sensitive Staphylococcus aureus

C) Pseudomonas aeruginosa

D) Sporothrix schenckii

E) Vibrio vulnificus
Explanation
Correct Answer:

C) Pseudomonas aeruginosa

This patient developed a tender papulopustular eruption on his lower extremities after swimming in a heated hotel
pool, raising strong suspicion for Pseudomonas aeruginosa folliculitis ("hot-tub" folliculitis). Most cases develop
within hours or a few days after swimming in an inadequately chlorinated swimming pool or hot tub. The rash
that forms is generally tender and may appear as papules, pustules, or nodules. Low-grade fever is often
present. The eruption is usually self-limited and does not require treatment. However, patients should be advised
to avoid the contaminated water that led to this condition. If lesions persist, an oral fluoroquinolone may be
considered.

Although Staphylococcus aureus is the leading cause of purulent skin infection (and folliculitis), most cases are
sensitive to trimethoprim-sulfamethoxazole (TMP-SMX) and are marked by a purulent lesion with surrounding
erythema and induration. The presence of several papulopustular lesions that did not respond to TMP-SMX make
this pathogen less likely (Choice B).

(Choice A) Group A Streptococcus (eg, Streptococcus pyogenes) is the leading cause of cellulitis. Most cases
present with spreading erythema, warmth, and tenderness at a site of skin breakdown (eg, fungal infection,
wound). Purulent lesions are less common and generally indicate an alternate pathogen.

(Choice D) Sporothrix schenckii is a dimorphic fungus found in soil that can occasionally cause cutaneous infection
after being introduced into the skin during gardening or other outdoor activities. However, infections usually take
weeks to develop and generally present with an ulcerative papule at the site of inoculation; subsequent lesions form
along the proximal lymphatic chain. This patient's bilateral papulopustular eruption on the lower extremities makes
Pseudomonas folliculitis far more likely.

(Choice E) Vibrio vulnificus is a estuarine bacteria that causes cellulitis following inoculation into a wound. This
organism lives in marine environments, not hotel pools, and usually causes a spreading cellulitis similar to S
pyogenes.

Educational objective:
Pseudomonas folliculitis is a self-limited cutaneous eruption that develops within hours or a few days following
exposure to inadequately chlorinated pools or hot tubs. Patients generally have tender papules, pustules, or
nodules and low-grade fever. No treatment is usually necessary, but swimming in the contaminated water should
be avoided.
Question #266

A 73-year-old man comes to the emergency department saying, "I can't pee!" The patient has not been able to
produce any urine over the past 24 hours and has had lower abdominal discomfort. He reports that his urine flow
has been weak for the past year. He has had no fever, weakness, numbness, dysuria, back pain, or hematuria.
The patient's other medical conditions include myelodysplastic syndrome, hypertension, and lumbar strain. He
does not use tobacco, alcohol, or illicit drugs. Temperature is 36.9 C (98.4 F), blood pressure is 150/110 mm Hg,
pulse is 94/min, and respirations are 16/min. The abdomen is soft with lower abdominal tenderness and dullness to
percussion. Neurologic examination shows absent Achilles tendon reflexes bilaterally but is otherwise
unremarkable. Straight catheterization of the bladder produces 800 mL of urine. Which of the following is most
likely responsible for this patient's current condition?

A) Acute inflammation of the prostate

B) Bladder outflow obstruction

C) Detrusor sphincter dyssynergia

D) Poor urethral sphincter function

E) Spinal cord compression


Explanation
Correct Answer:

B) Bladder outflow obstruction

This patient with a history of a weakened urinary stream now has acute urinary retention, characterized by the
inability to voluntarily micturate, with resulting bladder distension and suprapubic pain. Although the differential for
acute urinary retention is broad (eg, neurologic disorder, medications, infection), the most common etiology is
bladder outflow obstruction (BOO).

In men, BOO is most often due to benign prostatic hyperplasia (BPH); other etiologies include malignancy (eg,
prostate, rectal), urethral stricture, and urolithiasis. BPH is more common with increasing age; up to 70% of men
age >70 have lower urinary tract symptoms attributable to the disorder. Common symptoms include slow urinary
stream, postvoid dribbling, hesitancy, urgency, and nocturia. Physical examination reveals an enlarged, nontender
prostate. Management of BPH causing BOO includes acute decompression with urethral or suprapubic
catheterization. Definitive therapies include alpha blockers (eg, terazosin, tamsulosin) and 5-alpha reductase
inhibitors (eg, finasteride); surgical intervention (eg, transurethral resection of the prostate [TURP]) may be
indicated.

Although spinal cord compression can cause acute urinary retention (or bowel or bladder incontinence) and,
occasionally, decreased reflexes (although hyperreflexia can also be seen), patients typically have severe back
pain, decreased rectal tone, and reduced strength or sensation in the lower extremities. Achilles tendon reflex
decreases with age, and its absence is common in older individuals (Choice E).

(Choice A) Acute prostatitis can cause abdominal pain and acute urinary retention, but patients are typically
acutely ill with signs of systemic illness (eg, fever, chills). In addition, they typically have dysuria and urinary
urgency.

(Choice C) Detrusor sphincter dyssynergia results from simultaneous activation of the detrusor muscle and the
urethral sphincter, resulting in BOO. It typically occurs in patients with neurologic disorders (eg, spinal cord injury,
multiple sclerosis) and would be unexpected in this patient.

(Choice D) Poor urethral sphincter function would result in stress incontinence with urinary leakage, not urinary
retention. Stress incontinence is most frequently seen in men after prostate surgeries (eg, TURP).

Educational objective:
Benign prostatic hyperplasia causes slow voiding, postvoid dribbling, hesitancy, urgency, or nocturia; complications
include bladder outlet obstruction with acute urinary retention. Management of acute retention includes bladder
decompression with urethral or suprapubic catheterization; definitive treatment includes alpha blockers (eg,
terazosin, tamsulosin), 5-alpha reductase inhibitors (eg, finasteride), and possibly surgery.

Reference
• An evidence-based approach to emergency department management of acute urinary retention.
Question #267

A 56-year-old man comes to the office due to progressive urinary frequency, urgency, and hesitancy for the past
several months. The patient also reports new-onset low back pain and perineal pain during ejaculation. He has
had no dysuria or hematuria. The patient has smoked a pack of cigarettes daily for 30 years. Temperature is 37 C
(98.6 F), blood pressure is 130/76 mm Hg, and pulse is 80/min. There is no suprapubic or costovertebral angle
tenderness. Examination of the external genitalia is normal. Rectal examination shows increased anal sphincter
tone and a smooth, slightly enlarged prostate. Deep tendon reflexes are normal. Urinalysis shows many
leukocytes and no erythrocytes. Urine culture grows no organisms. Serum prostate-specific antigen is 2 ng/mL
(age-specific reference value is <3.5 ng/mL). Which of the following is the most likely cause of this patient's current
condition?

A) Bacterial sequestration in the epididymis

B) Hyperplasia of prostatic stromal and epithelial cells

C) Malignancy of prostatic glandular elements

D) Noninfectious chronic prostate inflammation

E) Resistant bacterial infection of the prostate

F) Urothelial neoplasm arising from the bladder


Explanation
Correct Answer:

D) Noninfectious chronic prostate inflammation

Chronic prostatitis/chronic pelvic pain syndrome

• Pain in pelvis, perineum, genitalia


Symptoms • Irritative voiding symptoms (eg, urgency, hesitancy)
• Hematospermia, pain with ejaculation

• No or mild prostate tenderness


Diagnosis
• Sterile urine culture

• Alpha blockers (eg, tamsulosin)


Management • Antibiotics (eg, ciprofloxacin), especially if history of UTI
• 5-alpha-reductase inhibitors (eg, finasteride)

UTI = urinary tract infection.

Chronic prostatitis/chronic pelvic pain syndrome (CP/CPPS), typically defined as chronic pelvic pain (including
pain in the perineum and testes, which can radiate to the back) for >3 months without an identifiable cause, can
present as voiding difficulties, irritative voiding symptoms (eg, frequency, urgency, hesitancy), pain with
ejaculation, or blood in the semen. Patients are generally afebrile and have little or no prostate tenderness
(despite the term "prostatitis"), with a normal urinalysis and negative urine culture results. Prostate-specific antigen
levels are often normal. There is usually no history of urinary tract infection. CP/CPPS can be categorized as
inflammatory or noninflammatory based on the presence of leukocytes in urine and prostatic secretions, but this
distinction has uncertain clinical significance. The exact etiology of CP/CPPS is unclear, but it is thought to be due
to noninfectious chronic prostate inflammation.

CP/CPPS is a diagnosis of exclusion, and additional testing (eg, scrotal ultrasound, cystoscopy) is individualized
based on the patient's particular clinical features. Even though a bacterial cause is not known, antibiotics (eg,
fluoroquinolones) are often helpful in symptom relief. Other interventions include alpha-adrenergic inhibitors (eg,
tamsulosin) and 5-alpha-reductase inhibitors (eg, finasteride).

(Choice A) Epididymitis can also present with irritative voiding symptoms, but patients usually have scrotal pain,
swelling, and tenderness as well as a purulent urethral discharge.

(Choice B) Benign prostatic hyperplasia may cause irritative voiding symptoms, but patients more typically have
slowly progressive bladder outlet obstruction (eg, decreased force of stream, incomplete voiding, dribbling, nocturia)
and do not typically have back pain.

(Choice C) Prostate cancer is unlikely in a young patient with a normal prostate-specific antigen level and no
nodules or asymmetry on prostate examination. Elevated prostate-specific antigen levels should not be attributed
to CP/CPPS.

(Choice E) Chronic bacterial prostatitis (not to be confused with CP/CPPS) presents with dysuria and urinary
frequency, but patients often have a history of urinary tract infection, prostatic tenderness or induration, and a
positive urine culture. Acute bacterial prostatitis is characterized by fever, perineal pain, and severe tenderness on
prostate examination.

(Choice F) Bladder cancer most commonly presents with painless gross hematuria, with or without irritative voiding
symptoms.

Educational objective:
Chronic prostatitis/chronic pelvic pain syndrome is a common disorder of uncertain etiology, characterized by
irritative voiding symptoms (eg, frequency, urgency, hesitancy); perineal or genital pain; and pain or blood on
ejaculation. Urine culture is sterile. Treatment includes antibiotics, alpha blockers, and 5-alpha-reductase
inhibitors.

Reference
• Contemporary management of chronic prostatitis/chronic pelvic pain syndrome.
Question #268

A 74-year-old man comes to the office due to urinary frequency over the last 2 months. He wakes up to use the
bathroom several times during the night. He also has had mild straining during urination. Over the last few weeks,
he has had 2 episodes of bloody urine that cleared spontaneously. The patient had right knee replacement for
severe osteoarthritis 7 years ago. His father died of colon cancer. He has a 30-pack-year smoking history but quit
15 years ago. Rectal examination shows an enlarged and smooth prostate with no nodules, and normal rectal
sphincter tone. Serum creatinine is 0.8 mg/dL and prostate-specific antigen level is 3.8 ng/mL (normal <4 ng/mL).
Urinalysis shows 0-1 white blood cells/hpf and 3-5 red blood cells/hpf, with no proteinuria, bacteriuria, casts,
or dysmorphic red blood cells. Which of the following is the best next step in management of this patient?

A) Cystoscopy

B) Finasteride

C) Prostate biopsy

D) Urine culture

E) Urine flow rate testing


Explanation
Correct Answer:

A) Cystoscopy

Indications for cystoscopy

• Gross hematuria with no evidence of glomerular disease or infection


• Microscopic hematuria with no evidence of glomerular disease or infection
but increased risk for malignancy
• Recurrent urinary tract infections refractory to antibiotic therapy
• Obstructive symptoms with suspicion for stricture, stone
• Irritative symptoms without urinary infection
• Abnormal bladder imaging or urine cytology

This patient has benign prostatic hyperplasia (BPH), which usually affects men age >50, can cause lower urinary
tract symptoms (eg, increased urinary frequency, hesitation, dribbling, nocturia) and a feeling of incomplete voiding,
and is associated with a smooth and symmetrically enlarged prostate without nodules. However, he also has gross
(eg, episodes of bloody urine) and microscopic (eg, >3 red blood cells/hpf) hematuria. Causes of microscopic
hematuria include renal (eg, renal cell cancer, IgA nephropathy), ureteral (eg, stricture, stone), bladder (eg, cancer,
cystitis), and prostate/urethral (eg, BPH, prostate cancer, urethritis) abnormalities.

In the absence of evidence of glomerular disease (eg, dysmorphic red blood cells) or infection (eg, pyuria,
bacteriuria), cystoscopy is recommended for all patients who have gross hematuria or for those with microscopic
hematuria associated with other risk factors for bladder cancer, such as this patient. Cigarette smoking is the
most important risk factor for bladder cancer. Although smoking cessation decreases this risk, patients still have a
higher risk than nonsmokers even up to 20 years after smoking cessation. Other risk factors include certain
occupational exposures (eg, painters, metal workers), chronic cystitis, iatrogenic causes (eg, cyclophosphamide),
and pelvic radiation exposure.

Although BPH can cause hematuria, this patient should first be evaluated with cystoscopy to exclude bladder
cancer. If negative, he can receive BPH treatment with alpha adrenergic antagonists (eg, terazosin, tamsulosin)
with or without 5-alpha reductase inhibitors (eg, finasteride) (Choice B).

(Choice C) Prostate biopsy is generally indicated to evaluate for prostate cancer. Although prostate cancer can
present with obstructive voiding symptoms and hematuria, rectal examination usually shows an asymmetrically
enlarged prostate that can be firm or have nodules. In this patient with a normal PSA and benign features on
prostate examination, biopsy is not needed.

(Choice D) Urine culture is generally recommended to rule out infections (eg, cystitis) as the cause of microscopic
hematuria. Cystitis usually presents with acute onset of hematuria, lower urinary tract symptoms, and abnormal
urinalysis (eg, pyuria, red blood cells). However, this patient's symptom duration (2 months) and absence of
significant pyuria or bacteriuria make this less likely.

(Choice E) Maximal urinary flow rate testing can also diagnose BPH. Flow rates >15 mL/sec usually exclude
significant bladder outlet obstruction. This is considered an optional test for most patients.

Educational objective:
Cystoscopy is recommended for patients with gross hematuria or with microscopic hematuria and other risk factors
for bladder cancer. Risk factors for bladder cancer include cigarette smoking, certain occupational exposures (eg,
painters, metal workers), chronic cystitis, iatrogenic causes (eg, cyclophosphamide), and pelvic radiation exposure.

Reference
• Indication of cystoscopy in patients with asymptomatic microscopic haematuria.

• Diagnosis and treatment of bladder cancer: how can we improve?


Question #269

A 28-year-old man comes to the physician for evaluation of infertility. He is healthy and has no other complaints.
He eats a high-protein diet and exercises daily in order to be muscular. He weighs 85 kg (187 lb) and is 175 cm
(5'8") tall. His temperature is 37.2° C (98.9° F) and blood pressure is 130/82 mm Hg. Physical examination shows
small testes. The remainder of the examination shows no abnormalities. Initial laboratory studies show:

Hemoglobin 16.0 g/dL


Platelet count 200,000/mm3
Leukocyte count 4500/mm3
Serum creatinine 1.4 mg/dL
Serum LH low
Serum testosterone normal

Which of the following is the most likely cause of his infertility?

A) Klinefelter syndrome

B) Mumps orchitis

C) Exogenous steroid use

D) Myotonic dystrophy

E) Varicocele

F) Cryptorchidism
G) Testicular torsion

H) Chronic medical illness


Explanation
Correct Answer:

C) Exogenous steroid use

This patient presents for evaluation of infertility likely due to anabolic steroid (e.g., testosterone analog) abuse.
Anabolic steroids are commonly used by many bodybuilders and athletes to enhance performance and muscle
mass. Anabolic steroids contain exogenous androgens that inhibit GnRH release by the hypothalamus (due to
feedback inhibition), which causes decreased LH and FSH release by the pituitary gland and leads to decreased
sperm and testosterone production by the testes. The exogenous androgen suppresses native testosterone
production but is detected as testosterone by current assays, so patients can have normal serum testosterone
levels. Other adverse effects include acne, gynecomastia (due to conversion of the androgens to estrone),
decreased testicular size, aggressiveness, and psychotic symptoms. Laboratory findings can show erythrocytosis
(due to increased erythropoiesis) with elevated hemoglobin, cholestasis, hepatic failure, dyslipidemia, and slightly
elevated creatinine (due to increased muscle mass).

(Choice A) Klinefelter syndrome (i.e., XXY seminiferous tubule dysgenesis) is an inherited disorder characterized
by testicular fibrosis (primary hypogonadism), azoospermia, gynecomastia, decreased intelligence, increased axial
skeletal growth, and high FSH and LH levels.

(Choice B) Mumps orchitis is a potential cause of infertility that is characterized by acute testicular pain and
inflammation during the acute viral illness.

(Choice D) Myotonic dystrophy is characterized by testicular atrophy as well as widespread muscular atrophy,
weakness, lower-than-normal testosterone level, and high FSH and LH levels.

(Choice E) Varicocele results in scrotal swelling, with a "bag of worms" sensation on palpation.

(Choice F) Cryptorchidism manifests in infancy, with a failure to palpate two testes in the scrotum on examination.
Patients are at an increased risk of testicular cancer.
(Choice G) Testicular torsion produces acute testicular pain.

(Choice H) Chronic medical illness typically results in anemia of chronic disease rather than erythrocytosis.

Educational objective:
Anabolic steroid use by a man can produce infertility by suppressing the production of GnRH, LH, and FSH.

Reference
• Anabolic steroids and semen parameters in bodybuilders
Question #270

A 60-year-old man comes to the office for evaluation of erectile dysfunction. For the last 6 months, the patient has
had progressive difficulty maintaining an erection long enough to have satisfying sexual intercourse. In addition, he
has had a decrease in early morning erections. There is no decrease in libido. He is happily married and states he
is not depressed or anxious. Medical history is significant for hypertension, hyperlipidemia, and coronary artery
disease; the patient was admitted for an acute myocardial infarction 2 years ago, and a drug-eluting stent was
placed in the right coronary artery. He is able to walk briskly for 30 minutes daily without developing angina or other
symptoms. Current medications include metoprolol, atorvastatin, lisinopril, aspirin, and ticagrelor. He does not use
alcohol. Blood pressure is 122/70 mm Hg and pulse is 66/min. BMI is 27 kg/m2. Cardiopulmonary examination is
unremarkable. Testicular size and secondary sexual characteristics are normal, and there is no gynecomastia.
Laboratory results are as follows:

Hemoglobin 14 g/dL
Creatinine 0.8 mg/dL
Glucose 80 mg/dL
LDL 60 mg/dL
TSH 2.7 µU/m
Total testosterone 455 ng/dL (normal: 300-800)

Which of the following is the best next step in treating this patient's erectile dysfunction?

A) Oral sildenafil

B) Oral trazodone

C) Stop lisinopril
D) Stop metoprolol

E) Testosterone gel
Explanation
Correct Answer:

A) Oral sildenafil

Sexual activity can increase the risk of myocardial infarction due to tachycardia and the acute rise in blood
pressure. However, most patients with uncomplicated myocardial infarction (ie, no recurrent angina, no
arrhythmias, normal ventricular function) and successful revascularization are at low risk of recurrent cardiac events
and may resume normal sexual activity at least 2 weeks following the initial cardiac event.

Erectile dysfunction (ED) is common in patients with cardiovascular disease due to decreased penile blood flow,
endothelial dysfunction, and vasoactive medications. First-line treatment for most patients with ED, including those
with cardiovascular disease, is a phosphodiesterase-5 (PDE-5) inhibitor (eg, sildenafil). These medications are
safe and effective in treating ED in low-risk patients with cardiovascular disease. However, PDE-5 inhibitors are
contraindicated in patients taking nitrates because concurrent use can cause a precipitous drop in blood
pressure, leading to syncope. They also should be used cautiously in patients who take alpha blockers because
this combination also can cause severe hypotension.

(Choice B) Trazodone is somewhat effective for some men with ED but primarily for those with psychogenic rather
than physiologic ED. Sildenafil is more likely to be helpful for this patient.

(Choice C) ACE inhibitors (eg, lisinopril) and angiotensin II receptor blockers decrease mortality for most patients
with coronary artery disease, particularly those with reduced ejection fraction or comorbid hypertension, diabetes, or
chronic kidney disease. These drugs are associated with a very low risk of ED and should be continued in this
patient.

(Choice D) Cardioselective beta blockers (eg, metoprolol) can cause ED, but a large systemic review of
randomized controlled trials confirmed that the overall incidence is low (<1%/yr). Given that these medications
decrease mortality in patients with a history of acute MI, they should be continued unless the patient is completely
intolerant or experiences severe side effects.
(Choice E) Testosterone therapy may be useful in ED associated with low libido and low serum testosterone
levels. This patient's libido and testosterone level are normal.

Educational objective:
Erectile dysfunction is common in patients with cardiovascular disease. First-line treatment includes
phosphodiesterase-5 (PDE-5) inhibitors (eg, sildenafil). PDE-5 inhibitors are contraindicated in patients taking
nitrates and used cautiously in patients who take alpha blockers because concurrent use can cause severe
hypotension.

Reference
• Cardiovascular safety of phosphodiesterase type 5 inhibitors after nearly 2 decades on the market.

• Beta-blocker therapy and symptoms of depression, fatigue, and sexual dysfunction.


Question #271

A 45-year-old man comes to the office due to an itchy rash on his penis. He first noted the lesions several months
ago. Findings from examination of the penile skin are shown in the exhibit. The remainder of the physical
examination is normal. Which of the following is the most likely diagnosis?

A) Herpes simplex

B) Lichen planus

C) Molluscum contagiosum

D) Penile cancer

E) Primary syphilis
Explanation
Correct Answer:

B) Lichen planus

Lichen planus

• 5 "Ps": pruritic, purple/pink, polygonal, papules & plaques


• Lacy, white network of lines (Wickham striae)
Clinical • Locations:
findings ◦ Skin (eg, ankles, wrists)
◦ Oral mucosa (white papules & plaques ± erythema, mucosal atrophy, ulcers)
◦ Genitalia

Disease • Hepatitis C
associations • Medications: ACE inhibitors, thiazide diuretics

• Chronic symptoms
Natural
• Formation of lesions at sites of trauma (Köbner phenomenon)
history
• Spontaneous resolution within 2 years

• Topical high-potency glucocorticoids (eg, betamethasone)


Treatment
• Widespread lesions: systemic glucocorticoids, phototherapy
This patient has a chronic rash characterized by pruritic plaques consistent with lichen planus (LP). LP most
commonly occurs as pruritic, purple/pink, polygonal papules and plaques on the skin of the extremities and trunk
(cutaneous LP), but lesions may also appear on the genitalia (genital LP) or oral mucosa (oral LP). The lesions
often have white, lacy markings known as Wickham striae and can form along the lines of minor trauma (Köbner
phenomenon).

The diagnosis of LP is based primarily on examination, but biopsy can be performed for confirmation. Although the
etiology is unknown, LP is often associated with hepatitis C. Treatment includes topical high-potency
glucocorticoids (eg, betamethasone). The disorder is self-limited and typically resolves within 2 years.

(Choice A) Herpes simplex virus commonly affects the penis but presents with painful, ulcerating vesicles on an
erythematous base. The symptoms are acute and episodic rather than chronic.

(Choice C) Molluscum contagiosum can affect the penis, especially when acquired by sexual contact. Like the
lesions in LP, the lesions in molluscum contagiosum can be chronic and pruritic but typically appear as umbilicated
nodules rather than plaques.

(Choice D) Bowen disease (cutaneous squamous cell carcinoma in situ) may develop on the penis but typically
presents as a painless, erythematous plaque. It may progress to penile cancer, which is most common in men age
>60, but usually presents as a painless ulcer/nodule with inguinal adenopathy.

(Choice E) Primary syphilis typically presents with a single painless, nonpruritic ulcer with induration (ie, chancre)
and bilateral, nontender lymphadenopathy.

Educational objective:
Lichen planus most commonly occurs as pruritic, purple/pink, polygonal papules and plaques on the skin of the
extremities and trunk, but lesions may also appear on the genitalia or oral mucosa. The diagnosis is based
primarily on examination, but biopsy can be performed for confirmation. Treatment includes topical high-potency
glucocorticoids. The disorder is self-limited and typically resolves within 2 years.

Reference
• Genital lichen planus: update on diagnosis and treatment.
Question #272

An 80-year-old man comes to the office for evaluation of fatigue and 4.5 kg (10 lb) weight loss. Medical history is
notable for benign prostatic hyperplasia (which is treated with tamsulosin), coronary artery disease, and
hypertension. The patient does not use tobacco and drinks 2 shots of whiskey daily. He eats a vegetarian diet.
Vital signs are normal. Physical examination shows a nodular prostate and inguinal lymphadenopathy. Serum
prostate-specific antigen level is 25 ng/mL (normal: <4.5 ng/mL). Biopsy of the prostate reveals adenocarcinoma.
Which of the following risk factors is most strongly associated with development of cancer in this patient?

A) Advanced age

B) Alcohol use

C) Benign prostatic hyperplasia

D) Use of tamsulosin

E) Vegetarian diet
Explanation
Correct Answer:

A) Advanced age

Comparison of benign prostatic hyperplasia & prostate cancer

BPH Prostate cancer

Risk factors • Age >50 • Age >40, African American & family history

Affected • Usually peripheral zone of prostate but can be


• Central portion (transitional zone)
part anywhere

• Symmetrically enlarged & smooth • Asymmetrically enlarged, nodules & firm


Examination prostate prostate
• Can have elevated PSA • Markedly elevated PSA

BPH = benign prostatic hyperplasia; PSA = prostate-specific antigen.

Prostate adenocarcinoma is the second most common malignancy in men worldwide. Although most cases are
asymptomatic, prostate cancer can present with systemic symptoms (eg, fatigue, weight loss), bone pain (due to
metastases), and/or lower urinary tract symptoms. Prostatic nodules or asymmetry on digital rectal examination,
inguinal lymphadenopathy, and significant elevation of prostate-specific antigen (PSA) support the diagnosis but
are not present in all cases. Confirmation generally requires transrectal biopsy.

The greatest risk factor for prostate cancer is advanced age. Men age <40 rarely have prostate cancer, but
prevalence steadily increases as age advances; 30%-80% of men age >70 have histologic evidence of prostate
cancer. Other risk factors for prostate cancer are less potent, including:

• Ethnicity: Black men have the greatest risk of prostate cancer and tend to have more aggressive tumors and
present at a later stage of disease. Risk is thought to be increased in this population due to genetic
predisposition.

• Diet: Diets high in animal fat (eg, alpha-linoleic acid) and low in fruits and vegetables cause a mild increase
in risk for prostate cancer. Vegetarian and fruit-heavy diets do not increase risk (Choice E).

(Choice B) Alcohol intake does not seem to increase the overall risk of prostate cancer. However, patients who
consume excessive alcohol and develop prostate cancer may have more aggressive tumors.

(Choices C and D) Most studies indicate minimal or no elevation in the risk of prostate cancer in patients with
underlying benign prostatic hypertrophy (BPH). This is supported by the fact that BPH typically affects the central/
transitional zone of the prostate, but prostate cancer primarily occurs in the periphery of the gland. Treatment of
BPH with alpha-1-adrenergic antagonists (eg, tamsulosin) does not seem to have a strong effect on the
development of prostate cancer. In contrast, 5-alpha reductase inhibitors (eg, finasteride, dutasteride), which block
peripheral conversion of testosterone to dihydrotestosterone (the more active form), reduce risk.

Educational objective:
The greatest risk factor for prostate cancer is advanced age; approximately 30%-80% of men age >70 have
histologic evidence of prostate cancer. Less-prominent risk factors include black ethnicity and a diet high in meat
and low in fruits and vegetables.

Reference
• Epidemiology, etiology, diagnosis and treatment of prostate cancer.
Question #273

A 27-year-old woman comes to the office for follow-up. The patient was last seen 2 months ago due to generalized
musculoskeletal pain and several months of fatigue. She was diagnosed with fibromyalgia and prescribed a low-
impact exercise program and pharmacotherapy. Her pain and fatigue have improved, but she now notes
intermittent episodes of "dizziness." Most of the episodes are short-lived, but she occasionally needs to brace
herself against a wall or sit until the dizziness resolves. She also notes worsened lethargy and frequent dry mouth.
Other medical history includes migraines and irritable bowel syndrome. The patient currently takes daily
amitriptyline for fibromyalgia, occasional acetaminophen for muscular pains, and polyethylene glycol as needed for
constipation. Temperature is 36.8 C (98.2 F), blood pressure is 110/70 mm Hg, pulse is 70/min, and respirations
are 14/min. BMI is 21 kg/m2. Mucous membranes are pink and moist, and there is no jugular venous distension.
Cardiopulmonary auscultation is normal. Neurologic examination demonstrates intact cranial nerves and normal
muscle strength in all extremities. Which of the following would be most helpful in determining the cause of this
patient's dizziness?

A) Carotid sinus massage

B) Dix-Hallpike testing

C) Orthostatic blood pressure measurements

D) Romberg testing with eyes closed

E) Tandem gait testing


Explanation
Correct Answer:

C) Orthostatic blood pressure measurements

This patient with fibromyalgia was recently initiated on amitriptyline. Amitriptyline exerts its therapeutic effects by
inhibiting the reuptake of norepinephrine and serotonin, but its use is often limited due to interaction with other
receptors, including:

• Muscarinic receptors, leading to anticholinergic symptoms (eg, dry mouth, constipation, urinary retention)
• Histamine receptors, leading to lethargy
• Alpha-adrenergic receptors, leading to orthostatic hypotension

These side effects often limit treatment and result in medication discontinuation.

Orthostatic hypotension is particularly common, even at low-dose therapeutic levels of amitriptyline. Manifestations
may be subtle and typically include transient episodes described as "dizziness" or lightheadedness with sudden
postural change, and patients often have a history of bracing themselves against a wall or sitting until the episode
resolves. Diagnosis is made when orthostatic blood pressure measurement reveals a systolic blood pressure
decline ≥20 mm Hg or diastolic blood pressure decline ≥10 mm Hg. Discontinuation of the medication usually
resolves the symptoms.

(Choice A) Carotid sinus massage can be used to diagnose/treat tachyarrhythmias (eg, paroxysmal
supraventricular tachycardia). Although tachyarrhythmias can present with dizziness, most cases are also
associated with palpitations and shortness of breath.

(Choice B) The Dix-Hallpike maneuver can be used to diagnose benign paroxysmal positional vertigo. This
condition is characterized by short episodes of vertigo with or without nausea/vomiting due to specific head
movements (eg, rolling over in bed, looking up). Because this patient developed symptoms after the initiation of
amitriptyline, a medication side effect is more likely.
(Choice D) Sensory ataxia is often suspected when patients have a positive Romberg test. Sensory ataxia can be
associated with balance dysregulation but is not typically characterized by episodic dizziness.

(Choice E) Tandem gait testing can be used to assess for cerebellar ataxia, which generally causes disequilibrium
and balance issues. Patients do not typically have episodic dizziness that improves with leaning against a wall or
sitting.

Educational objective:
Amitriptyline is commonly used for depression, insomnia, and pain disorders but is frequently associated with side
effects, including orthostatic hypotension, lethargy, and anticholinergic symptoms (eg, dry mouth, constipation,
urinary retention). Discontinuing the medication usually resolves these symptoms.
Question #274

A 65-year-old man comes to the office for evaluation of slowly progressive right-hand weakness for the past 3
months. The patient has no pain or numbness. Medical history is significant for degenerative joint disease in both
his hips and hypertension. Vital signs are normal. Physical examination shows atrophy of right thenar eminence
and first and second web spaces. Muscle strength testing reveals decreased right grip strength and moderate
weakness of the finger abduction. Tendon reflexes are 3+ in the right arm and 1+ in the left arm and lower limbs.
Sensory examination shows no abnormalities. Electrophysiologic studies reveal widespread fibrillations and
positive sharp waves in bilateral upper and lower extremity muscle groups. The underlying mechanism of this
patient's disease most likely involves an abnormality in which of the following?

A) Anterior horn cell

B) Dorsal spinal root

C) Neuromuscular junction

D) Peripheral nerve

E) Skeletal muscle
Explanation
Correct Answer:

A) Anterior horn cell


This patient has hand weakness associated with hyperreflexia, an upper motor neuron (UMN) sign, and atrophy, a
lower motor neuron (LMN) sign. This presentation is strongly suggestive of amyotrophic lateral sclerosis (ALS), a
progressive, fatal neurodegenerative disease. Symptoms result from deterioration of UMNs (eg, corticospinal
neurons located in the primary motor cortex) and LMNs (eg, cranial nerves, anterior horn cells).
The diagnosis of ALS is typically made clinically in a patient who displays both UMN and LMN signs. However,
early in the disease process, only a few muscle groups may be affected (as in this patient), and electrodiagnostic
studies can be helpful to rule out other neuromuscular diseases. Electrodiagnostic studies typically demonstrate
signs of LMN dysfunction, with denervation in multiple muscle groups (even asymptomatic muscle groups). The
denervated fibers upregulate acetylcholine receptors (as they no longer receive input from the neuromuscular
junction), leading them to become hypersensitive to acetylcholine and spontaneously discharge; this manifests as
widespread fibrillations and positive sharp waves (reflecting spontaneous depolarization) on electrophysiologic
studies. Classically, sensory nerve testing is normal.

(Choice B) Degeneration of dorsal spinal roots would lead to sensory dysfunctions, not motor dysfunctions, as
seen in this patient.

(Choice C) Diseases of the neuromuscular junction such as myasthenia gravis (MG) or Lambert-Eaton myasthenic
syndrome (LEMS) are characterized by weakness that either worsens (MG) or improves (LEMS) with repetition;
UMN signs are unexpected. Repetitive nerve stimulation tests demonstrate progressive decline (or improvement) in
the amplitude of complex motor action potentials.

(Choice D) Although peripheral nerve disorders (eg, hereditary motor and sensor neuropathy) may present with
weakness and widespread electrodiagnostic changes consistent with denervation, patients would not have
hyperreflexia (an UMN sign).

(Choice E) Diseases affecting the skeletal muscle include dermatomyositis and polymyositis. Although fibrillation
potentials can be seen on insertion of the needles for electromyography (EMG), inflammatory myopathies typically
affect symmetric proximal skeletal muscles, and EMG changes are limited to muscles that are affected rather than
the widespread changes seen in this patient. In addition, these myopathies do not cause UMN signs.

Educational objective:
Amyotrophic lateral sclerosis leads to degeneration of cells in the anterior horn of the spinal cord. It is characterized
by often asymmetric weakness with upper and lower motor neuron signs. Electrodiagnostic studies often show
characteristics of both acute and chronic denervation.

Reference
• Electrodiagnosis in persons with amyotrophic lateral sclerosis.
Question #275

A 42-year-old woman comes to the office due to left lower jaw pain for the past 10 days. The patient describes brief
attacks of severe, stabbing pain that shoots from near her left ear down along her jawline. The pain often occurs
while chewing food and lasts several seconds, and she has had no episodes while asleep. The patient has no prior
medical history other than postpartum depression after the birth of her second child 6 years ago. She has had no
trauma or recent surgery. Vital signs are within normal limits. The patient has no facial rash or tenderness, but
during the examination, she develops another episode of pain when the left cheek is lightly touched. Ear, nose, and
oral cavity examination is normal. Which of the following is the most appropriate first-line pharmacotherapy for this
patient?

A) Acyclovir

B) Naproxen

C) Oxcarbazepine

D) Prednisone

E) Sumatriptan
Explanation
Correct Answer:

C) Oxcarbazepine

Trigeminal neuralgia

• Compression of the trigeminal nerve (CN V) root (eg, vascular loop, multiple sclerosis
Pathophysiology
plaque)

• Paroxysmal attacks of unilateral, severe, stabbing pain lasting seconds to 2 min


Clinical features • Limited to CN V distribution (mainly V2 & V3)
• Triggered by innocuous stimuli (eg, chewing, talking, brushing teeth, light touch)

• MRI/MRA of the brain with contrast


Workup
• Nerve conduction (eg, trigeminal reflex) testing if unable to obtain MRI

• Carbamazepine or oxcarbazepine
Treatment
• Surgery (eg, decompression) only for severe, medically refractory cases

MRA = MR angiography.

This patient likely has trigeminal neuralgia (TN), as evidenced by the unilateral, intermittent, stabbing ear pain
that radiates down the jaw, lasts several seconds, and is triggered by minor stimuli (eg, chewing, light touch). TN
usually causes severe neuropathic (burning, electric shock–like) pain along the V2 (maxillary) and V3
(mandibular) branches of the trigeminal nerve (CN V).

TN is most often caused by vascular compression of the trigeminal nerve root as it enters the pons. Because of the
risks associated with neurosurgical microvascular decompression, control of symptoms with medication is preferred;
surgery is reserved for patients who fail medical management.

First-line therapy is usually carbamazepine or oxcarbazepine, antiepileptic drugs that inhibit action potentials by
modulating sodium channels. Oxcarbazepine is often better tolerated (eg, less nausea/vomiting, less risk for
leukopenia), but patients are at risk for hyponatremia due to increased sensitivity to antidiuretic hormone.

(Choice A) Acyclovir can treat reactivation of herpes viruses, which can cause neuralgia (eg, zoster) or neuropathy
(eg, Bell palsy). Like TN, herpetic neuralgia has a dermatomal distribution and is associated with allodynia (ie, pain
from light touch). Pain can precede the vesicular rash by several days. However, seconds-long paroxysms of pain
that do not occur at night and are not accompanied by a rash after >1 week are more consistent with TN. Acyclovir
would not improve TN, which is caused by nerve compression.

(Choice B) Naproxen is used for temporomandibular joint disorders, which can present with severe ear, face, and/
or jaw pain aggravated by chewing. Morning headaches (due to night bruxism) are common; examination often
shows tenderness of the mastication muscles, tooth wear, and crepitus of the temporomandibular joint with motion,
none of which are seen in this patient. Neuropathic pain does not typically respond to NSAIDs.

(Choice D) Prednisone is given for clinically suspected giant cell arteritis, which causes a unilateral headache with
tenderness to palpation of the temporal artery (vs allodynia in TN). However, most patients are age >50 and have
systemic symptoms (eg, fatigue, fever) and jaw claudication (ie, pain or fatigue with chewing, relieved with rest)
versus short episodes of neuropathic pain.

(Choice E) Sumatriptan (classically indicated for migraine) is also used for cluster headaches, which can cause
severe, paroxysmal, unilateral head and facial pain. However, episodes often occur at night, last approximately 90
minutes (vs seconds), predominantly affect the upper face, and are often accompanied by autonomic symptoms (ie,
miosis, lacrimation). Sumatriptan use in TN is rare, with limited data supporting it.
Educational objective:
Trigeminal neuralgia is characterized by short, episodic, severe pain usually along the V2 (maxillary) and V3
(mandibular) branches of the trigeminal nerve (CN V), often triggered by minor stimuli. Oxcarbazepine or
carbamazepine is first-line therapy.

Reference
• Trigeminal neuralgia—a prospective systematic study of clinical characteristics in 158 patients.

• Increased risk of trigeminal neuralgia after hypertension: a population-based study.


Question #276

A 65-year-old woman comes to the office for evaluation of progressively worsening weakness in her legs and
difficulty walking for 6 months. The patient also reports a constant dull headache and occasional urinary
incontinence. Medical history is significant for rheumatoid arthritis for which she takes methotrexate. Vital signs are
within normal limits. BMI is 34 kg/m2. Examination of the lower extremities shows mild spasticity and brisk deep
tendon reflexes on both sides. Babinski sign is present bilaterally; sensations are normal. Upper extremity
neurologic examination shows no abnormalities. Gait is spastic. Musculoskeletal examination shows no active
synovitis. What is the most likely cause of this patient's condition?

A) Amyotrophic lateral sclerosis

B) Cervical myelopathy

C) Folic acid deficiency

D) Parasagittal meningioma

E) Mononeuritis multiplex
Explanation
Correct Answer:

D) Parasagittal meningioma
This patient's deficits include the following:
• Weakness of bilateral, symmetric lower limbs
• Sparing of the upper limbs
• Upper motor neuron signs (eg, hyperreflexia, upgoing Babinski, spastic gait)
• Urinary incontinence (suggesting dysfunction of the adjacent frontal lobe)

These deficits could be due to a lesion in the lower spinal cord. However, the patient's constant headache and lack
of sensory findings suggest that the lesion is located in the brain. A mass in the anterior midline of the brain
attached to the falx cerebri is one possible explanation. Therefore, this woman's symptoms are likely caused by a
parasagittal (ie, parafalcine) meningioma, a benign brain tumor that usually grows very slowly.

(Choice A) Amyotrophic lateral sclerosis often presents with spastic weakness with no sensory deficits. However,
mixed upper and lower motor neuron signs would be present and constant headache would be unusual.

(Choice B) Patients with rheumatoid arthritis are at increased risk of compression of the spinal cord due to cervical
myelopathy from degenerative joint disease. This can cause weakness with upper motor neuron signs. However, a
lesion in the cervical cord would likely impact the upper extremities.

(Choice C) Methotrexate can cause folic acid deficiency. However, folic acid deficiency does not typically lead to
neurologic manifestations, although it can occasionally cause a length-dependent polyneuropathy. In addition, it is
quite rare due to food fortification and does not lead to constant headache.

(Choice E) Mononeuritis multiplex, a neuropathy of ≥2 noncontiguous peripheral nerves most commonly caused by
vasculitis, can occur in patients with rheumatoid arthritis. However, it would not cause upper motor neuron signs or
headaches.

Educational objective:
Meningiomas often occur at dural reflections and can cause headaches and progressive neurologic defects that
localize to the brain area compressed by the mass.

Reference
• Diagnostic challenges in meningioma.
Question #277

A 65-year-old woman comes to the office reporting progressively increasing orthopnea, daytime fatigue, and
morning headache for the last month. She has had no chest pain or problems with swallowing or speech. Medical
conditions include myocardial infarction, type 2 diabetes, hypertension, and amyotrophic lateral sclerosis.
Temperature is 37 C (98.6 F), blood pressure is 130/70 mm Hg, pulse is 88/min, and respirations are 16/min. Pulse
oximetry shows 95% on room air. BMI is 24 kg/m2. Examination shows normal jugular venous pressure, clear
lungs, and normal heart sounds. There is no peripheral edema. Neurologic examination shows 3/5 power in right
lower extremity and 4/5 power in left lower extremity and muscle atrophy, unchanged from her last examination 2
months ago. Laboratory results are as follows:

Hemoglobin 12 g/dL
Creatinine 0.8 mg/dL
B-type natriuretic peptide (BNP), plasma 90 pg/mL (normal: <100, but variable)

Which of the following is the most appropriate next step in management?

A) Furosemide

B) Home oxygen therapy

C) Modafinil

D) Noninvasive positive-pressure ventilation

E) Tiotropium

F) Tracheostomy
Explanation
Correct Answer:

D) Noninvasive positive-pressure ventilation

This patient with amyotrophic lateral sclerosis (ALS) (asymmetric muscle strength, muscle atrophy) has
manifestations of early respiratory insufficiency. Patients with ALS, a terminal neurodegenerative disease that
affects both upper and lower motor neuron function, most often die of respiratory failure, which is caused by
multiple factors:

• Inspiratory muscle (eg, diaphragm) atrophy and weakness, which leads to poor inspiratory strength
• Expiratory muscle weakness, which leads to an ineffective cough and poor bronchial clearance
• Bulbar muscle weakness, which leads to dysphagia and chronic aspiration

Because gravity is not assisting with lung expansion, respiratory symptoms are often more pronounced when
patients are supine. In addition, sleep further contributes to respiratory insufficiency because the accessory
respiratory muscles are less active during REM sleep, and relaxation of the weakened upper airway musculature
results in airway collapse and obstruction. Therefore, early respiratory insufficiency may manifest as orthopnea
and appear similar to obstructive sleep apnea (eg, daytime fatigue, morning headache), as seen in this patient.

Treatment focuses on improving respiratory function, which both prolongs survival and improves quality of life.
Noninvasive positive-pressure ventilation (NIPPV) improves respiratory function by opening the upper airway
and providing positive end-expiratory pressure to improve atelectasis. NIPPV is considered first-line therapy for
ALS patients with respiratory insufficiency; it can be used intermittently at night but may need to be used more
frequently as the disease progresses. Hypoxia and hypercarbia are late signs of respiratory failure; treatment
should be initiated prior to the development of abnormal vital signs or CO2 retention.

(Choice A) Furosemide is used to treat heart failure, which can present with orthopnea and fatigue. However, this
patient has no jugular venous distension, no peripheral edema, and a normal b-type natriuretic peptide level,
making heart failure unlikely.
(Choice B) Home oxygen therapy is not recommended in patients with ALS due to the risk of decreasing the
ventilatory drive, which may worsen carbon dioxide retention and potentially lead to hypercapnic coma.

(Choice C) Modafinil is sometimes used to treat fatigue associated with ALS. However, this patient's symptoms of
orthopnea, fatigue, and headache are more likely related to respiratory insufficiency, which is better treated with
NIPPV.

(Choice E) Tiotropium is a long-acting bronchodilator used in patients with chronic obstructive pulmonary disease
or asthma, but this patient's symptoms are due to respiratory muscle weakness rather than bronchoconstriction.

(Choice F) Tracheostomy and invasive mechanical ventilation can be considered for patients with contraindications
to NIPPV (eg, inability to clear secretions, severe bulbar dysfunction) or those with progressive respiratory failure
despite NIPPV. Most patients with advanced disease choose to forgo these options given the associated
complications (eg, increased caregiver burden) and terminal nature of ALS.

Educational objective:
Amyotrophic lateral sclerosis can cause respiratory insufficiency due to respiratory muscle weakness. Noninvasive
positive-pressure ventilation prolongs survival and improves quality of life.

Reference
• Respiratory failure in amyotrophic lateral sclerosis.
Question #278

A 67-year-old man comes to the office for a new patient evaluation after recently relocating to a retirement
community. The patient has a history of hypertension treated with losartan. He has a 40-pack-year history but quit
smoking 5 years ago. Blood pressure is 130/80 mm Hg and pulse is 80/min. Examination reveals a bruit in the
right side of the neck; the remainder of the examination is normal. Carotid duplex ultrasonography reveals 45%
stenosis at the right common carotid artery bifurcation. The left common carotid artery has a lesion causing 40%
stenosis. Hemoglobin A1c is 5.3% and LDL cholesterol is 120 mg/dL. Which of the following is the best next step
in management of this patient's arterial abnormalities?

A) Antiplatelet and statin therapy alone

B) Bilateral carotid artery revascularization

C) CT angiography of the neck

D) Periodic repeat imaging without additional therapy

E) Right carotid artery revascularization


Explanation
Correct Answer:

A) Antiplatelet and statin therapy alone


This patient with multiple atherosclerotic risk factors (eg, hypertension, prior smoking) has bilateral carotid
atherosclerotic disease, which was discovered when carotid duplex ultrasonography was performed to evaluate a
carotid bruit. Because this patient's carotid disease has not caused transient ischemic attack or stroke symptoms, it
is considered asymptomatic; in contrast, disease that has caused focal neurologic symptoms within the previous 6
months would be considered symptomatic.

The classification of carotid atherosclerotic disease as asymptomatic or symptomatic, along with its severity,
impacts surgical management. For asymptomatic lesions, such as in this patient, the following is recommended:

• >80% stenosis: Revascularization with carotid endarterectomy (CEA) is generally recommended.

• 50%-79% stenosis: Revascularization with CEA is generally not recommended but may be considered in
select patients with low perioperative risk (eg, <3%).

• <50% stenosis (such as this patient): Revascularization with CEA has no proven benefit, so it is not
recommended; lesions are managed with intensive medical therapy alone (Choices B and E).

Intensive medical therapy, which is recommended for all patients with carotid atherosclerotic disease (both
asymptomatic and symptomatic), includes an antiplatelet agent (eg, aspirin), a statin, and strict blood pressure
control. Patients should also receive counseling about lifestyle changes (eg, exercise, smoking cessation), as well
as rigorous management of other comorbidities that increase cardiovascular risk (eg, diabetes, obesity). Finally,
patients with stenosis <80% should receive periodic (eg, annual) carotid duplex surveillance to detect any
progression.

(Choice C) Although CT angiography is sometimes obtained prior to CEA for surgical planning, surgical
revascularization is not indicated in this patient with <50% stenosis. CT angiography is not routinely indicated as a
follow-up for an abnormal carotid ultrasound, especially when the stenosis is low grade (eg, <50%) and
asymptomatic.

(Choice D) Periodic (eg, annual) surveillance with carotid duplex ultrasonography is recommended to detect
potential progression of stenosis in lesions causing <80% stenosis (as seen in this patient). However, surveillance
alone is insufficient; intensive medical therapy (eg, aspirin, statin) should also be initiated.

Educational objective:
The surgical management of carotid atherosclerotic disease depends on whether it is asymptomatic or
symptomatic, as well as the degree of stenosis. Patients with asymptomatic disease and stenosis <50% do not
benefit from carotid revascularization. Instead, they are managed with intensive medical therapy (eg, antiplatelet
agent, statin) and periodic carotid duplex ultrasonography to assess for progression of lesions.

Reference
• Evaluation and management of atherosclerotic carotid stenosis.

• Update on surgical management for asymptomatic carotid stenosis.

• Efficacy of contemporary medical management for asymptomatic carotid artery stenosis.

• A risk prediction model for determining appropriateness of CEA in patients with asymptomatic carotid artery
stenosis.
Question #279

A 76-year-old woman is brought to the office by her daughter due to progressively worsening memory and language
difficulties. The patient first started having memory problems and word-finding difficulties 5 years ago. She then
began having difficulty balancing her checkbook and buying groceries. The patient became lost while driving to
church last year and no longer drives. She is now dependent on her daughter for cooking and cleaning. Over the
past 6 months, the patient's personality has changed from seeming apathetic to becoming more paranoid and
agitated; she frequently claims to have seen her niece stealing from her purse. The patient recently developed
urinary incontinence. She has a history of hyperlipidemia and osteoarthritis of the left hip and both knees. Blood
pressure is 130/80 mm Hg and pulse is 90/min. Deep tendon reflexes are 2+ throughout, and she has preserved
motor strength. The patient recalls none of 3 objects on memory testing and cannot draw a clock. Laboratory
studies show normal electrolytes, lipid panel, TSH, vitamin B12, and complete blood count. Which of the following is
the most likely diagnosis?

A) Alzheimer disease

B) Dementia with Lewy bodies

C) Frontotemporal dementia

D) Normal pressure hydrocephalus

E) Vascular dementia
Explanation
Correct Answer:

A) Alzheimer disease

Clinical features of Alzheimer disease

• Anterograde memory loss (ie, immediate recall affected, distant memories preserved)
Early • Visuospatial deficits (eg, lost in own neighborhood)
findings • Language difficulties (eg, difficulty finding words)
• Cognitive impairment with progressive decline

• Neuropsychiatric (eg, hallucinations, wandering)


• Dyspraxia (eg, difficulty performing learned motor tasks)
Late
• Lack of insight regarding deficits
findings
• Noncognitive neurologic deficits (eg, pyramidal & extrapyramidal motor, myoclonus, seizures)
• Urinary incontinence

This patient has evidence of dementia (ie, major neurocognitive disorder), including cognitive deficits that have led
to impairment in activities of daily living (eg, preparing meals, cleaning). Her early and prominent memory
symptoms, age of onset, and symptom evolution are classic for Alzheimer disease (AD).

AD is the most common cause of dementia in the United States and usually occurs after age 65. Death usually
occurs within 10 years of diagnosis regardless of the treatment offered. Patients experience a progressive
functional decline over the course of their disease. Later findings usually include personality and behavioral
changes (eg, apathy, agitation), neuropsychiatric changes (eg, delusions, paranoia), neurologic manifestations
(eg, myoclonus, seizures), and apraxia (eg, difficulty with motor tasks). Urinary incontinence may develop in later
stages of AD, secondary to severe cognitive dysfunction.

(Choice B) In addition to the presence of dementia, a diagnosis of dementia with Lewy bodies requires the
presence of ≥2 of the following clinical features: parkinsonism, fluctuating cognition, visual hallucinations, and rapid
eye movement sleep behavior disorder.

(Choice C) Frontotemporal dementia typically presents with early and prominent behavioral/personality change
and only later with prominent memory deficits. It typically manifests around age 60, demonstrably earlier than AD.

(Choice D) Normal pressure hydrocephalus (NPH) is characterized by abnormal gait, cognitive impairment, and
urinary incontinence. Gait impairment is the most prominent clinical feature of NPH and appears early in its course;
the gait is broad-based and shuffling. Gait impairment is not a classic characteristic of early AD (although it is
commonly seen in older patients due to other conditions [eg, osteoarthritis]).

(Choice E) Vascular dementia typically presents with cortical- or subcortical-predominant symptom clusters,
depending on the location of the precipitating vascular pathology. A stepwise decline is classic; however, deficits in
executive function are much more severe than memory deficits early in the course of the disease. Vascular
dementia is less likely in this patient due to the disease course (eg, memory deficits) and the absence of previous
cerebral vascular disease.

Educational objective:
Alzheimer disease, the most common type of dementia in the United States, is characterized by early and
prominent memory impairment. The differential includes dementia with Lewy bodies, frontotemporal dementia, and
vascular dementia, as well as nondementing syndromes such as normal pressure hydrocephalus.

Reference
• Alzheimer's disease.

• Alzheimer disease: epidemiology, diagnostic criteria, risk factors and biomarkers.


Question #280

A 38-year-old woman comes to the office because of chronic aching pain and stiffness around her neck, shoulders,
lower back, and hips. Even minor exertion worsens the pain. The patient also has difficulty sleeping and frequently
wakes up in the middle of the night. Even after a good night's sleep, she always feels fatigued in the morning. The
patient has no muscle weakness, fever, malaise, weight loss, or rash. She tried water aerobic exercise 3 times a
week but had no symptom improvement. She has a history of irritable bowel syndrome. Vital signs are within
normal limits. Examination shows multiple tender spots at muscle insertion points and bony prominences. Her
muscle strength is 5/5 in all extremities, and deep-tendon reflexes are 2+ throughout. No sensory abnormalities are
noted. Laboratory results are as follows:

Hematocrit 43%
Leukocytes 7,000/mm3
Platelets 200,000/mm3
Creatine kinase, serum 60 U/L
TSH 3.0 µU/mL
Erythrocyte sedimentation rate 9 mm/h

Which of the following is the most appropriate therapy for this patient?

A) Amitriptyline

B) Colchicine

C) Methotrexate

D) Naproxen
E) Oxycodone

F) Prednisone
Explanation
Correct Answer:

A) Amitriptyline

This patient's presentation is consistent with likely fibromyalgia (FM). FM presents most commonly in young to
middle-age women with widespread pain, fatigue, and cognitive/mood disturbances. Patients tend to have fairly
normal physical examination findings except for point tenderness in areas such as the mid trapezius, lateral
epicondyle, costochondral junction in the chest, and greater trochanter. FM has no specific diagnostic laboratory
findings. Revised 2010 American College of Rheumatology criteria suggest using the widespread pain index and
symptom severity scale rather than trigger points for FM diagnosis. The index and severity scale better emphasize
cognitive problems, fatigue, and severity of somatic symptoms.

Initial FM treatment should emphasize patient education, regular aerobic exercise, and good sleep hygiene.
Patients who do not respond to conservative measures may require medications. Tricyclic antidepressants (eg,
amitriptyline) are preferred first-line drugs. Serotonin and norepinephrine reuptake inhibitors (eg, duloxetine,
milnacipran) and pregabalin are alternate therapies that may be useful for patients not responding to tricyclic
antidepressants. Patients with persistent symptoms may require combination drug therapy, referral for supervised
rehabilitation, pain management consultation, or cognitive-behavioral therapy.

(Choices B and C) Colchicine is used to treat gout, which typically presents with acute-onset monoarthritis.
Methotrexate can effectively treat inflammatory arthritides (eg, rheumatoid arthritis, psoriatic arthritis), which are
usually associated with elevated inflammatory markers (eg, erythrocyte sedimentation rate [ESR], rheumatoid
factor) and joint pain. However, neither drug is effective for FM.

(Choices D and F) Glucocorticoids (eg, prednisone) and nonsteroidal anti-inflammatory drugs (eg, naproxen) can
treat inflammatory conditions that typically have elevated markers (eg, ESR, C-reactive protein). However, FM is
not an inflammatory condition and does not usually respond to glucocorticoids or nonsteroidal anti-inflammatory
drugs.
(Choice E) Opioids (eg, oxycodone), except for tramadol, are usually not recommended for FM as there is little
evidence that they provide significant relief, and the risks (eg, abuse, diversion) outweigh the benefits.

Educational objective:
Fibromyalgia is a pain syndrome that occurs most commonly in young to middle-aged women and is characterized
by fatigue, widespread pain, and cognitive/mood disturbances. Amitriptyline is an effective initial therapy.
Pregabalin, duloxetine, and milnacipran are alternate therapies for patients not responding to tricyclic
antidepressants.

Reference
• The American College of Rheumatology preliminary diagnostic criteria for fibromyalgia and measurement of
symptom severity.

• Comparative efficacy and acceptability of amitriptyline, duloxetine and milnacipran in fibromyalgia syndrome:
a systematic review with meta-analysis.

• The role of antidepressants in the management of fibromyalgia syndrome: a systematic review and meta-
analysis.
Question #281

A 30-year-old man comes to the office with fatigue and lethargy that has worsened over the last 2 weeks. He has
been forgetful lately and feels "exhausted" at the end of the day. The patient works as a contractor and is currently
renovating old houses for sale. He describes feeling "clumsy" and dropping things at work, as well as tripping
multiple times while climbing stairs. He also has abdominal pain that he attributes to constipation. The patient
drinks 1 or 2 beers each weekend and does not use tobacco or illicit drugs. His mother was diagnosed with lupus
and his older sister had thyroid surgery. Blood pressure is 120/80 mm Hg, and pulse is 76/min. Examination shows
normal jugular venous pressure, no thyromegaly, clear lung fields, and normal first and second heart sounds. The
abdomen is soft and nontender. There is no hepatomegaly or splenomegaly. There is weakness on dorsiflexion of
bilateral wrists and feet. Upper and lower limb deep tendon reflexes are 1+. Laboratory results are as follows:

Complete blood count


Hemoglobin 9.6 g/dL
Mean corpuscular volume 76 µm3
Reticulocytes 4%
Platelets 200,000/mm3
Leukocytes 4,100/mm3

Serum chemistry
Blood urea nitrogen 12 mg/dL
Creatinine 1.0 mg/dL
Uric acid, serum 11 mg/dL

Creatine phosphokinase levels are normal. Which of the following is most likely to improve this patient's
symptoms?
A) Allopurinol

B) Calcium disodium EDTA

C) Cobalamin supplementation

D) Levothyroxine

E) Plasmapheresis

F) Thiamine supplementation
Explanation
Correct Answer:

B) Calcium disodium EDTA

Lead poisoning in adults

Risk
• Occupational exposure (eg, lead paint, batteries, ammunition, construction)
factors

• Gastrointestinal (abdominal pain, constipation, anorexia)


Clinical
• Neurologic (cognitive deficits, peripheral neuropathy)
features
• Hematologic (anemia)

• Anemia
Laboratory • Elevated venous lead level
findings • Elevated serum zinc protoporphyrin level
• Basophilic stippling on peripheral smear

This patient likely has acute lead toxicity from exposure to lead-based paint given his recent work renovating old
houses. His constellation of clinical manifestations, including nonspecific findings (eg, fatigue), consists of:

• Gastrointestinal manifestations: Constipation


• Neuropsychiatric manifestations: Sensorimotor neuropathy (eg, weakness with dorsiflexion), short-term
memory loss
• Hematologic manifestations: Microcytic anemia (often with basophilic stippling on peripheral smear),
hyperuricemia (due to impaired purine metabolism)

Chronic lead toxicity can cause hypertension, nephropathy, and more pronounced neuropsychiatric symptoms (eg,
psychosis).

Lead is absorbed predominantly via the lungs in adults. It is stored predominantly in the skeleton and is released
slowly, exerting its pathologic effects over decades. Diagnosis depends on establishing a history of lead exposure
accompanied by corroborating physical examination findings (eg, neurologic manifestations) and elevated blood
lead levels. The treatment for symptomatic lead poisoning is chelation therapy with an agent such as calcium
disodium EDTA. It is important to separate the patient from further lead exposure as the chelating agent may
otherwise increase lead absorption from the source.

(Choice A) Allopurinol is a xanthine oxidase inhibitor used as urate-lowering therapy to prevent recurrent gout.
This patient has hyperuricemia from lead-induced inhibition of purine synthesis, but no inflammatory arthritis
suggestive of gout. Urate-lowering therapy would be unlikely to improve the neuropsychiatric and other symptoms
related to lead toxicity.

(Choices C and F) Cobalamin (vitamin B12) deficiency (pernicious anemia, post-gastrectomy syndrome, intestinal
malabsorption) causes macrocytic (not microcytic) anemia and subacute combined degeneration (eg, spasticity,
incontinence, paraplegia). Thiamine (vitamin B1) deficiency, seen with alcohol abuse and white rice/cereal-based
diets, manifests as Wernicke-Korsakoff syndrome (eg, nystagmus, ophthalmoplegia, ataxia, confabulation) or
beriberi (eg, distal and symmetric peripheral neuropathy, cardiomyopathy).

(Choice D) Hypothyroidism can cause a sensory neuropathy and may be associated with motor weakness and
hyperuricemia. However, anemia due to hypothyroidism is generally either normocytic or macrocytic, without
reticulocytosis.

(Choice E) Plasmapheresis improves outcomes in patients with severe Guillain-Barré syndrome. Although the
syndrome presents with progressive motor neuropathy and decreased/absent deep-tendon reflexes, the sensory
and cognitive deficits are usually milder than those in this patient, and there is no direct association with
hyperuricemia or microcytic anemia.
Educational objective:
Chronic lead toxicity presents in adults with neuropsychiatric, gastrointestinal, and general symptoms, including
peripheral sensorimotor neuropathy, fatigue, abdominal pain, and constipation. It can lead to hypertension,
nephropathy, hyperuricemia, and microcytic anemia with basophilic stippling seen on peripheral smear. Treatment
for symptomatic patients involves chelation therapy with an agent such as calcium disodium EDTA.

Reference
• Evaluation and management of lead exposure.
Question #282

A 75-year-old man comes to the office due to intermittent right eye vision loss. The patient reports 3 episodes over
the past 2 months in which he suddenly experienced "a curtain falling over the right eye" for several minutes before
it spontaneously resolved. He has no eye pain or discomfort, focal weakness, numbness, or headache. His
medical history is significant for hypertension, hyperlipidemia, and osteoarthritis. He smoked cigarettes for 25 years
before quitting at age 50. Blood pressure is 140/85 mm Hg and pulse is 74/min. Visual acuity, pupillary reflex, and
funduscopic examinations are unremarkable. Further neurologic examination shows normal reflexes, motor
strength, and sensation. Which of the following is most likely to yield the diagnosis in this patient?

A) CT scan of the head

B) Duplex ultrasound of the neck

C) Echocardiography

D) Erythrocyte sedimentation rate

E) Lumbar puncture and CSF analysis

F) MRI of the brain

G) Ocular tonometry
Explanation
Correct Answer:

B) Duplex ultrasound of the neck

This patient has amaurosis fugax, which is characterized by painless, rapid, and transient (<10 minutes)
monocular vision loss. The description of a curtain descending over the visual field is highly suggestive of this
diagnosis. The most common etiology is retinal ischemia due to atherosclerotic emboli originating from the
ipsilateral carotid artery; therefore, patients with vascular risk factors (eg, hypertension, hyperlipidemia, smoking)
should be evaluated with a duplex ultrasound of the neck. Funduscopic examination is often normal but may
show embolic plaques and retinal whitening (due to ischemia). Patients with amaurosis fugax and concomitant
carotid artery disease have an increased risk of stroke.

Other causes of transient monocular vision loss are uncommon and include cardioembolic disease, giant cell
arteritis, retinal vein occlusion, retinal vasospasm, and papilledema. Retinal detachment also sometimes causes
the sensation of a curtain descending over the eye, but symptoms are unlikely to resolve within minutes.

(Choices A and F) Neuroimaging with CT scan or MRI may be considered in patients with suspected structural
brain disease (eg, stroke, tumor, infection, demyelinating plaques). However, the absence of headache, seizure,
mental status changes, or focal neurologic deficits makes this less likely in this patient.

(Choice C) Echocardiography can be helpful in evaluating for cardioembolic disease (eg, atrial thrombus in the
setting of atrial fibrillation), which typically leads to transient ischemic attack or stroke. Cardioembolic disease is a
much less common cause of amaurosis fugax compared to carotid atherosclerosis.

(Choice D) Erythrocyte sedimentation rate should be considered in patients age >50 with monocular vision loss to
evaluate for temporal (giant cell) arteritis. However, this diagnosis is unlikely in this patient as other features are not
evident, such as unilateral headache, jaw claudication, constitutional symptoms (eg, fever, fatigue), and polymyalgia
rheumatica (eg, proximal muscle stiffness).

(Choice E) Lumbar puncture and CSF analysis (eg, oligoclonal bands) may be considered in patients with
suspected multiple sclerosis, which often presents with optic neuritis. However, optic neuritis usually presents in
women age <50 with monocular vision loss and pain on eye movement. Patients also typically have an afferent
pupillary defect with optic disc swelling on funduscopy.

(Choice G) Ocular tonometry is indicated for measuring intraocular pressure in patients with acute angle-closure
glaucoma, which typically presents with painful monocular vision loss, headache, and nausea/vomiting.
Examination findings usually include conjunctival injection with a poorly reactive mid-dilated pupil.

Educational objective:
Amaurosis fugax is characterized by painless, rapid, and transient monocular vision loss. The description of a
curtain descending over the visual field is highly suggestive of this diagnosis. The most common etiology is retinal
ischemia due to atherosclerotic emboli originating from the ipsilateral carotid artery; therefore, patients with vascular
risk factors should receive a duplex ultrasound of the neck.

Reference
• A 65-year-old man with transient monocular vision loss.

• Transient monocular blindness.


Question #283

A 38-year-old woman comes to the office for evaluation of pain and tingling in her hands. She works as a
telemarketer for a local sales and marketing agency and has been having difficulty holding the phone. Review of
systems is positive for frequent muscle aches in the arms and legs, constipation, lethargy, and difficulty
concentrating at work. The patient has no other medical conditions and takes no medication. She does not use
tobacco, alcohol, or recreational drugs. Blood pressure is 130/80 mm Hg, pulse is 62/min, and respirations are 14/
min. BMI is 32 kg/m2. On examination, there is numbness in the thumb, index, and middle fingers of both hands.
Deep tendon reflexes are 1+ in all 4 extremities. The skin appears doughy and dry at both elbows. Mild edema is
present in both ankles. Which of the following is most likely responsible for this patient's hand symptoms?

A) Amyloid accumulation

B) Calcium phosphate deposits

C) Mucinous infiltration

D) Periventricular demyelination

E) Vascular steal phenomenon


Explanation
Correct Answer:

C) Mucinous infiltration

Pathophysiology of carpal tunnel syndrome

Cause Pathophysiologic features

Idiopathic/overuse • Swelling & fibrosis of tendons & soft tissue

Hypothyroidism • Soft tissue enlargement (mucopolysaccharides)

• Soft tissue enlargement


Diabetes mellitus
• Microvascular insufficiency & neovascularization

Rheumatoid arthritis • Extrinsic compression from joint deformity

Pregnancy • Edema/fluid accumulation


End-stage • Amyloid & calcium phosphate deposition
renal disease • Access related (bleeding, venous hypertension during HD, vascular steal)

• Tendon enlargement
Acromegaly
• Synovial edema

Gout • Compression from tophi

HD = hemodialysis.

This patient has myalgia, constipation, impaired concentration, dry skin, mild edema, and hyporeflexia, suggesting
hypothyroidism. She also has pain, paresthesia, and sensory loss in the distribution of the median nerve, findings
consistent with carpal tunnel syndrome (CTS). CTS is an entrapment mononeuropathy caused by compression of
the median nerve within the carpal tunnel in the wrist.

In addition to slowed metabolic activity, many symptoms of hypothyroidism (eg, myxedema, dry skin) are due to
deposition of mucinous material composed of glycosaminoglycans, hyaluronans, and mucopolysaccharides within
tissue. Direct infiltration of the median nerve and nerve sheath with mucinous material results in swelling and a
localized ischemic neuropathy. Deposition on the tendons within the carpal tunnel results in swelling and
compression on the nerve, leading to symptoms of CTS.

Patients with hypothyroidism are more likely to develop bilateral CTS, and their symptoms may be more severe.
The risk does not appear to correlate with the severity of the underlying thyroid disease, although it is greater in
patients with elevated BMI. Treatment of hypothyroidism with levothyroxine often leads to improved CTS
symptoms.

(Choices A, B, and E) Patients with end-stage renal disease (ESRD) on hemodialysis frequently develop CTS due
to accumulation of dialysis-related amyloid. When CTS occurs, it is more common in the arm used for vascular
access (eg, access graft, arteriovenous fistula), suggesting a contribution of vascular steal or venous hypertension.
ESRD can also lead to pathologic calcification with deposition of calcium phosphate, which can compress the
nerve. However, these factors are unlikely in this patient.

(Choice D) Multiple sclerosis is characterized by demyelination in the periventricular white matter; common
manifestations include weakness, sensory loss, incontinence, and optic neuropathy. However, symptoms are
episodic and affect multiple, noncontiguous domains; symptoms involving the distribution of the median nerve in the
hand are more consistent with CTS.

Educational objective:
Carpal tunnel syndrome is common in patients with hypothyroidism and is frequently bilateral. Hypothyroidism
causes soft tissue thickening and mucinous infiltration, which can lead to compression of the median nerve within
the carpal tunnel.

Reference
• Carpal tunnel syndrome in hypothyroidism.
Question #284

A 76-year-old man is brought to the office by his daughter due to disruptive behavior. Last night he behaved
normally, took his regular medications, and retired to his bedroom shortly after dinner. When the patient emerged
an hour later, he was agitated, irritable, and confused. When asked what was wrong, he became belligerent and
seemed unable to recognize some family members. The daughter describes similar episodes over the past 3
evenings. Although the patient seems calmer and less confused this morning, the daughter worries that he will
have another episode. His medical history is significant for essential hypertension, benign prostatic hyperplasia,
mild neurocognitive disorder, insomnia, and a recent increase in anxiety related to stock market losses.
Medications include terazosin and alprazolam. A thorough physical examination is unremarkable, with no focal
neurologic findings. Laboratory evaluation, including complete blood count and serum electrolytes, is normal.
Which of the following is the most appropriate next step in management of this patient?

A) Increase dose of alprazolam

B) Order CT scan of the brain

C) Prescribe donepezil

D) Prescribe quetiapine as needed

E) Taper and discontinue alprazolam

F) Taper and discontinue terazosin


Explanation
Correct Answer:

E) Taper and discontinue alprazolam

Although benzodiazepines are often prescribed for patients with insomnia and/or anxiety, they should be used
sparingly in the elderly due to increased risk of adverse effects. As people age, they metabolize
benzodiazepines more slowly and are more likely to experience confusion and increased risk of falls. In addition,
patients with baseline cognitive impairment are even more vulnerable to the adverse effects of benzodiazepines.

Another adverse effect of benzodiazepines is paradoxical agitation, which is suggested by this patient's recurrent
episodes of confusion and agitation shortly after taking his alprazolam dose. Paradoxical agitation is characterized
by increased agitation, confusion, aggression, and disinhibition, typically within an hour of administration. Although
paradoxical reactions to benzodiazepines are relatively uncommon (<1%), they are important to recognize as
increasing the benzodiazepine dose will only worsen the patient's condition (Choice A). Taper and eventual
discontinuation of this patient's alprazolam is the most appropriate next step in management.

(Choice B) CT scan of the head should not precede a decrease in alprazolam dose in a patient who develops
mental status changes following repeated benzodiazepine administration and who has no focal neurologic findings.
CT scan of the brain should be considered in individuals with focal neurologic findings, recent head injury, or no
clear precipitant for an acute-onset episode of confusion.

(Choice C) Donepezil is an acetylcholinesterase inhibitor used to slow cognitive decline in patients with Alzheimer
disease, but there is no data to support its use in mild cognitive impairment. It would not be of help in patients with
acute-onset cognitive impairment due to medication-induced delirium.

(Choice D) Quetiapine would be appropriate only in a patient with persistent psychosis or uncontrolled agitation.
This patient's alprazolam dose should be tapered before considering quetiapine.

(Choice F) Terazosin, an alpha blocker, may cause orthostatic hypotension or syncope, especially with the first
dose. Therefore, it should be used with caution in the elderly. Terazosin is not known to cause cognitive
impairment or agitation, and there is no indication to discontinue it in this patient at this time.

Educational objective:
Benzodiazepines should be used with extreme caution in the elderly due to increased risk of cognitive impairment,
falls, and paradoxical agitation.

Reference
• Paradoxical reaction to alprazolam in an elderly woman with a history of anxiety, mood disorders, and
hypothyroidism.

• What is the point of guidelines? Benzodiazepine and z-hypnotic use by an elderly population.
Question #285

A 29-year-old woman comes to the office due to frequent episodes of headache. She has moderate to severe, left-
sided, throbbing pain associated with nausea and occasional vomiting. Her headache is often preceded by a
tingling sensation in the right hand that gradually involves the right arm and face and is followed by numbness
lasting <1 hour. She feels irritable while experiencing the headaches. The pain typically improves following several
hours of rest in a dark and quiet room. The patient has had these episodes every 1-2 months for the past 5 years.
She has taken as-needed sumatriptan with some relief, but the symptoms have caused her to miss several days of
work. She has no other medical history and does not use tobacco, alcohol, or recreational drugs. There is a family
history of depression. Vital signs are within normal limits and physical examination is normal. Which of the
following is most likely to be helpful in preventing this patient's headaches?

A) Beta-interferon

B) Glucocorticoid

C) Levetiracetam

D) Metoclopramide

E) Sertraline

F) Topiramate
Explanation
Correct Answer:

F) Topiramate

Migraine therapies

• Triptans (eg, sumatriptan)


• NSAIDs (eg, naproxen)
Abortives • Acetaminophen
• Antiemetics (eg, metoclopramide, prochlorperazine)
• Ergotamines (eg, dihydroergotamine)

• Topiramate
• Divalproex sodium
Preventives
• Tricyclic antidepressants
• Beta blockers (eg, propranolol)

NSAIDs = nonsteroidal anti-inflammatory drugs.

This patient most likely has migraine headaches as evidenced by recurrent episodes of severe, left-sided,
throbbing headache associated with nausea and vomiting and often preceded by aura (eg, progressive right-
sided tingling sensation followed by numbness). Abortive medications (eg, sumatriptan) can be taken as needed at
the onset of an attack, but prophylactic medication should be considered in patients who:
• Have frequent (eg, >4/month) or long-lasting (eg, >12 hours) episodes
• Experience disabling symptoms that prevent regular activities despite abortive treatment
• Are unable to take or have had no relief with abortive medications
• Overuse abortive medication (eg, nonsteroidal anti-inflammatory drugs [NSAIDs]) and have rebound
headache

This patient has missed work due to her symptoms; therefore, it is reasonable to consider a migraine prophylactic
medication such as topiramate, beta blockers (eg, propranolol), or amitriptyline.

(Choice A) Beta-interferon is a disease-modifying drug indicated for chronic maintenance therapy in patients with
relapsing-remitting multiple sclerosis; it can decrease the frequency of relapses and reduce the development of
brain lesions. It is not indicated for the treatment or prophylaxis of migraine headaches.

(Choice B) Glucocorticoids can be given with other abortive migraine medications for acute migraines but play no
role in the prophylaxis of migraines due to their long-term side effects (eg, hyperglycemia, osteoporosis).

(Choice C) Levetiracetam is used for seizure treatment and prophylaxis and is not generally administered for the
treatment or prevention of migraine headache.

(Choice D) Metoclopramide is an antiemetic that can be used as monotherapy or as adjuvant therapy with NSAIDs
or triptans for the treatment of acute migraine headaches, but it is not usually used for migraine prophylaxis.

(Choice E) Sertraline, a selective serotonin reuptake inhibitor, is an antidepressant and is not commonly used in
migraine treatment. Antidepressants that can be used are amitriptyline and venlafaxine.

Educational objective:
Migraine should be suspected in patients with unilateral headaches with a pulsing or throbbing quality, particularly if
accompanied by nausea/vomiting and photophobia. Patients who have frequent attacks or disabling symptoms that
do not respond to abortive treatment should be considered for prophylactic treatment, including topiramate, beta
blockers, or amitriptyline.

Reference
• Evidence-based guideline update: pharmacologic treatment for episodic migraine prevention in adults:
report of the Quality Standards Subcommittee of the American Academy of Neurology and the American
Headache Society.
Question #286

A 55-year-old man comes to the office due to difficulty chewing food over the past 2 months. The patient initially
experienced coughing while drinking liquids and has recently had difficulty ingesting solid foods. In addition, several
people have told him they have difficulty understanding his speech. Review of systems is positive for a 5-kg (11-lb)
weight loss over the past 3 months. The patient smoked a pack of cigarettes daily for 35 years. Temperature is 37
C (98.6 F), blood pressure is 120/70 mm Hg, pulse is 78/min, and respirations are 14/min. BMI is 24 kg/m2. On
examination, there is pooling of secretions in the oral cavity. Involuntary muscle twitching is noted over the chin,
tongue, and upper and lower limbs. Power is 4/5 in both upper extremities. Deep tendon reflexes are 3+
throughout. Bilateral Babinski is present. Mental and sensory examinations are normal. Which of the following is
most likely present in this patient?

A) Antibodies against postsynaptic acetylcholine receptors

B) Antibodies against presynaptic Ca channels

C) Brainstem compression

D) Degeneration of corticospinal tract

E) Inflammatory muscle disease


Explanation
Correct Answer:

D) Degeneration of corticospinal tract


This patient with bulbar symptoms (eg, dysphagia, dysarthria), muscle wasting and fasciculations, and hyperreflexia
with positive Babinski has amyotrophic lateral sclerosis (ALS). ALS is a progressive disease that results from
degeneration of upper motor neurons (UMNs) (eg, motor nuclei in the precentral gyrus and their projections in
the corticospinal tract and corticobulbar tract) and lower motor neurons (LMNs) (eg, cranial nerves, spinal
nerves).
Because any motor neuron can be involved, the initial presentation may vary widely but typically includes
weakness in multiple muscle groups, with both LMN (eg, muscle atrophy, fasciculations) and UMN (eg,
increased muscle tone, hyperreflexia, upgoing Babinski) signs. In 20% of patients, cranial nerves (eg,
hypoglossal, ambiguus, motor trigeminal) are affected first, which results in bulbar symptoms (eg, coughing or
choking when eating, weight loss); these patients have a worse prognosis.

(Choices A and B) Disorders of the neuromuscular junction include myasthenia gravis, which results from
antibodies against postsynaptic acetylcholine receptors, and Lambert-Eaton myasthenic syndrome, which results
from antibodies against presynaptic Ca channels and is associated with malignancy (eg, weight loss, smoking
history). These disorders can present with asymmetric muscle weakness, including of the bulbar muscles.
However, symptoms are typically fluctuating, and, because they affect the neuromuscular junction, UMN signs
would not be present.

(Choice C) Brainstem compression can result in bulbar symptoms and UMN signs in the extremities. However,
compression would typically also affect other cranial nerves (eg, III, V, VII, XI) and would likely also affect the
respiratory drive (due to medullary compression). LMN signs such as fasciculations are not common with brainstem
compression.

(Choice E) Inflammatory myopathies (eg, polymyositis) cause widespread muscle weakness. However, because
they affect the musculature (rather than motor) neurons, hyporeflexia may occur but UMN signs (eg, hyperreflexia,
abnormal Babinski) would be unexpected.

Educational objective:
Amyotrophic lateral sclerosis leads to degeneration of the upper motor neurons (eg, motor nuclei in the precentral
gyrus and their projections in the corticospinal and corticobulbar tracts) and lower motor neurons (eg, cranial
nerves, spinal nerves). Because any motor neuron can be involved, patients may have a variety of symptoms,
including bulbar muscle weakness. Examination shows both upper motor neuron and lower motor neuron signs.
Question #287

A 72-year-old man is brought to the office by his niece due to medication nonadherence. The patient has type 2
diabetes mellitus that had been well controlled with a basal-bolus insulin regimen, but recently he has not been
taking his insulin regularly and his home glucometer readings have been high. In the past, he self-administered his
insulin without difficulty, but now he no longer knows how to draw up his insulin or inject it. The patient's niece says
he has been "letting things go" around the house since his wife's death from cancer 3 years ago. He is also
reluctant to leave the house after a fall several months prior, and his family has had to help him with grocery
shopping and other common errands. On physical examination, the patient appears unkempt and his clothes smell
of urine. The lungs are clear on auscultation and heart sounds are normal. The abdomen is soft and nontender.
Neurological examination reveals absence of bilateral ankle reflexes, and the patient's gait is slow with a wide
base. Brief cognitive assessment shows short-term memory impairment. Hemoglobin A1c is 10.2%. Which of the
following is the best next step in the management of this patient?

A) Arrange placement in a nursing home

B) Begin therapy for depression-related cognitive impairment

C) Change antidiabetic therapy to oral medications

D) Obtain MRI of the brain

E) Provide diabetes education and reinforce compliance


Explanation
Correct Answer:

D) Obtain MRI of the brain

Normal pressure hydrocephalus

• Gait instability (wide-based) with frequent falls


• Cognitive dysfunction
Clinical features • Urinary urgency/incontinence
• Depressed affect (frontal lobe compression)
• Upper motor neuron signs in lower extremities

• Marked improvement in gait with spinal fluid removal: Miller


Fisher (lumbar tap) test
Diagnosis
• Enlarged ventricles out of proportion to the underlying
brain atrophy on MRI

Treatment • Ventriculoperitoneal shunting

This elderly patient with cognitive impairment, urinary incontinence, and gait abnormalities (eg, slow, wide-
based) has the classic symptom triad of normal pressure hydrocephalus (NPH). NPH is a potentially reversible
cause of dementia characterized by the abnormal accumulation of cerebrospinal fluid. It is often idiopathic and
occurs most commonly in older adults (age >60). However, it may occur in any age group due to neurologic
trauma, infection, or subarachnoid hemorrhage resulting in scarring of the arachnoid granulations. Although upper
motor neuron signs may occur in advanced disease, many elderly patients have diminished lower extremity reflexes
(eg, ankles) related to age.

Neuroimaging (eg, MRI, CT scan) shows ventriculomegaly out of proportion to the sulci. The diagnosis is
confirmed with lumbar puncture, which demonstrates normal opening pressure and transient clinical improvement
following high-volume cerebrospinal fluid removal. Definitive therapy is a ventriculoperitoneal shunt.

(Choice A) Although nursing home placement may eventually be required for patients with progressive dementia,
this patient's presentation suggests NPH, which is potentially reversible and should be evaluated with MRI.

(Choice B) Depression can cause apathy and signs of cognitive impairment ("pseudodementia"), but patients
typically have other depressive symptoms (eg, sleep impairment, depressed mood) as well. Treating depression
without working up the patient's focal findings would likely lead to progression of dementia.

(Choices C and E) Changing to an oral diabetic regimen and providing patient and family education could be
considered after workup and diagnosis of this patient's cognitive decline. However, diagnosis and treatment of NPH
should take priority.

Educational objective:
Normal pressure hydrocephalus is a potentially reversible cause of dementia characterized by cognitive decline,
gait impairment, and urinary incontinence. Neuroimaging demonstrates ventriculomegaly out of proportion to the
sulci.
Question #288

A 54-year-old woman comes to the office due to unsteady gait. The patient's legs have felt "rubbery," and she fell
once while walking in the dark several weeks ago. She has had no headache, vision loss, focal weakness, or back
pain. The patient has also had irritability and crying spells, which she attributes to menopause. She has a 6-year
history of type 2 diabetes mellitus and takes metformin. The patient has had remarkable weight loss following
various diets, and her hemoglobin A1c levels have been below 7% over the past year. Blood pressure is 124/68
mm Hg and pulse is 66/min with no orthostatic changes. BMI is 26 kg/m2. Physical examination shows normal
pupillary reflexes and funduscopy. Muscle strength in the lower extremities is normal. Bilateral plantar reflexes are
upgoing. When the patient is instructed to stand with the feet together and eyes closed, she sways from side to
side. Laboratory results are as follows:

Hemoglobin 12.6 g/dL


Mean corpuscular volume 96 µm3
Platelets 380,000/mm3
Leukocytes 9,000/mm3
Serum creatinine 0.8 mg/dL
Hemoglobin A1c 6.5%

Which of the following is the best next step in management of this patient?

A) CT scan of the lumbosacral spine

B) Duloxetine therapy

C) Gadolinium-enhanced MRI of the brain


D) Serum vitamin B12 level

E) Thyroid function studies


Explanation
Correct Answer:

D) Serum vitamin B12 level

This patient on chronic metformin therapy has neuropsychiatric manifestations (eg, irritability, crying spells), sensory
ataxia (swaying when standing with eyes closed), and positive Babinski sign, raising strong suspicion for vitamin
B12 deficiency. Approximately 30% of patients on metformin develop clinically significant vitamin B12 deficiency
after ≥5 years of treatment. The mechanism is not entirely elucidated but may be in part due to metformin altering
calcium homeostasis in the gastrointestinal tract, thereby impairing calcium-dependent vitamin B12 absorption in the
terminal ileum.

Vitamin B12 is required for DNA synthesis and myelin sheath formation; deficiency usually results in megaloblastic
anemia (due to impaired hematopoiesis) and/or neurologic manifestations (due to myelin damage). A minority of
patients have neurologic findings alone (as in this case); therefore, the lack of hematologic abnormalities (eg,
macrocytosis) does not exclude the diagnosis. Neurologic abnormalities generally arise because of damage to the
following:

• Dorsal columns, leading to impaired vibratory sensation/proprioception and sensory ataxia (gait impairment
that worsens when the eyes are closed [positive Romberg sign])

• Lateral corticospinal tracts, leading to positive Babinski sign

• Myelinated peripheral nerves, leading to lower extremity paresthesias

• Myelinated fibers in the brain, leading to neuropsychiatric manifestations (eg, irritation, mood changes)

Workup begins with a serum vitamin B12 level, but methylmalonic acid and homocysteine levels may be required if
results are inconclusive.

(Choice A) Lumbar spinal stenosis can be diagnosed with CT scan of the lumbosacral spine. This condition can
cause positive Babinski sign due to compression of the corticospinal tracts, but most patients (>90%) have pain in
the legs exacerbated by activity and weakness.

(Choice B) Duloxetine can treat painful diabetic polyneuropathy. Most cases arise in patients with poorly controlled
diabetes mellitus and generally present with loss of sensation in a stocking-glove pattern. Because this is a
peripheral neuropathy, positive Babinski sign would not be seen.

(Choice C) Contrast-enhanced MRI of the brain could aid in the diagnosis of demyelinating brain lesions (eg,
multiple sclerosis), stroke, or brain neoplasm. Although brain lesions can cause positive Babinski sign,
manifestations are usually unilateral; motor impairments are also commonly seen. A parasagittal meningioma could
cause bilateral lower leg weakness but would not explain sensory ataxia.

(Choice E) Hypothyroidism can cause peripheral polyneuropathy, which frequently presents with lower extremity
paresthesia and sensory loss in a stocking-glove distribution. Although deep tendon reflexes may be diminished,
Babinski sign would not be seen.

Educational objective:
Long-term (≥5 years) metformin therapy can cause vitamin B12 deficiency due to alterations in calcium homeostasis,
leading to impaired absorption of vitamin B12 in the terminal ileum. Although megaloblastic anemia is often seen, a
minority of patients have isolated neurologic findings (eg, paresthesias, sensory ataxia, neuropsychiatric changes).
Question #289

A 52-year-old woman comes to the office due to intermittent pain on the right side of the face for 3 weeks. The
intense, sharp pain on the right cheek and lips lasts several seconds and recurs several times a day. The patient is
afraid to brush her teeth or drink cold water because both trigger the pain. She is pain free between episodes. She
has a history of hypertension, type 2 diabetes mellitus, and seasonal allergies. The patient does not use tobacco,
alcohol, or illicit drugs. Temperature is 36.9 C (98.4 F), blood pressure is 136/84 mm Hg, and pulse is 80/min. No
rash or facial asymmetry is noted. The remainder of the neurologic examination shows no abnormalities. Which of
the following is the most likely cause of this patient's current condition?

A) Demyelinating midbrain plaque

B) Functional basal ganglia abnormality

C) Peripheral nerve inflammation and edema

D) Sensory ganglion virus reactivation

E) Vascular nerve root compression


Explanation
Correct Answer:

E) Vascular nerve root compression

This patient with unilateral, intermittent sharp pain of the right cheek and lips that lasts several seconds and is
triggered by minor stimuli (eg, brushing teeth, drinking cold water) likely has trigeminal neuralgia (TN).
Characteristic clinical features of TN include the following:

• Paroxysmal severe attacks of neuropathic pain (eg, shock-like, stabbing) lasting a few seconds to 2
minutes
• Unilateral pain involving primarily the V2 and V3 distributions of the trigeminal nerve (CN V)
• Innocuous triggers (eg, chewing, talking, brushing teeth) and allodynia (pain to light touch)
• Absence of other neurologic deficits on examination

The pathophysiology of TN is thought to be related to compression of the trigeminal nerve root as it enters the
pons, leading to demyelination and atrophy of the nerve. Compression most commonly occurs due to a vascular
loop, neoplastic growth, or multiple sclerosis plaque.

(Choice A) A demyelinating plaque (eg, multiple sclerosis) in the midbrain could impact cranial nerves that
originate in the midbrain, including the oculomotor nerve (CN III) and the trochlear nerve (CN IV). However, CN V,
which conveys sensation from the face, originates in the pons. A demyelinating plaque in the pons is one cause of
TN.

(Choice B) A functional abnormality in the basal ganglia can lead to focal dystonia that can affect the face (eg,
blepharospasm, oromandibular dystonia). Although pain can occur due to sustained muscle contraction and
spasms can often be triggered or alleviated by sensory input (eg, "sensory trick"), severe neuropathic pain is
uncommon with focal dystonia. In addition, this patient has no evidence of abnormal muscular contraction.

(Choice C) The pathogenesis of Bell palsy involves inflammation and edema of the facial nerve (CN VII).
However, this usually does not cause pain but rather results in unilateral facial paralysis. In TN, compression of the
nerve root leads to atrophy and demyelination rather than edema.

(Choice D) Herpetic neuralgia occurs due to reactivation of a virus (eg, herpes zoster); dermatomal pain can
precede development of the classic vesicular rash by several days. However, seconds-long paroxysms of pain
triggered by brushing teeth and unaccompanied by a rash after >1 week are more consistent with TN.

Educational objective:
Trigeminal neuralgia is caused by compression of the trigeminal nerve (CN V) root as it enters the pons, usually by
an abnormal vessel loop. This leads to atrophy and demyelination of the nerve and causes short paroxysms of
neuropathic pain.

Reference
• Trigeminal neuralgia—a prospective systematic study of clinical characteristics in 158 patients.

• Increased risk of trigeminal neuralgia after hypertension: a population-based study.


Question #290

A 38-year-old woman is evaluated in the clinic due to swallowing difficulty that began a month ago while eating
dinner. Halfway through the meal, she had difficulty chewing her steak and was unable to swallow it. Since then,
the patient has had 2 similar episodes associated with food and water regurgitation through her nose. Medical
history is significant for gastroesophageal reflux disease. Blood pressure is 138/76 mm Hg and pulse is 82/min.
Oropharyngeal examination is unremarkable. Lung auscultation is normal. Heart sounds are normal with no
murmurs. The abdomen is soft and nontender with no organomegaly. The patient has no focal weakness or
sensory loss, and reflexes are normal. Upper gastrointestinal endoscopy and esophageal manometry are normal.
CT scan of the chest is shown in the image below:
Which of the following is the most likely cause of this patient's symptoms?
A) Decreased acetylcholine release from motor neurons

B) Decreased available acetylcholine receptors

C) Degeneration of myenteric plexus neurons

D) Fibrous tissue replacement of smooth muscles

E) Inflammatory demyelination of the axons


Explanation
Correct Answer:

B) Decreased available acetylcholine receptors


This patient's symptoms are concerning for myasthenia gravis (MG), a neuromuscular disorder that typically
presents with fluctuating, fatigable muscle weakness that worsens with repetitive motion and improves with rest.
Ocular symptoms (eg, ptosis, diplopia) and bulbar dysfunction are the most common manifestations, the latter of
which may lead to fatigable chewing or dysphagia with nasopharyngeal regurgitation, as in this patient. Early in the
disorder course, weakness is often limited to specific muscle groups (eg, ocular or bulbar), then becomes
generalized (eg, limb and respiratory involvement) over the next 2 years.

MG is caused by autoantibodies (originating in the thymus) directed against nicotinic acetylcholine receptors at the
neuromuscular junction. Antibody binding leads to complement activation with receptor degradation, causing
impaired action potential propagation and muscle weakness. Most patients with acetylcholine receptor antibodies
have thymic abnormalities (eg, thymoma, thymic hyperplasia), which may appear as an anterior mediastinal
mass on chest imaging. A CT scan of the chest is typically performed in patients with MG to evaluate for thymoma
because thymectomy can result in long-term disease remission.

(Choice A) Foodborne botulism and Lambert-Eaton syndrome (LES) result in decreased acetylcholine release
from the presynaptic terminal of motor neurons, which can affect cranial nerves and cause difficulty swallowing.
However, botulism typically causes rapidly progressive, symmetric descending flaccid paralysis, and LES typically
causes proximal lower extremity weakness (vs normal strength). Both would be expected to have decreased or
absent reflexes (vs normal reflexes).

(Choice C) Achalasia occurs due to degeneration of neurons within the myenteric plexus and can cause gradual
onset of solid and liquid dysphagia due to uncoordinated peristalsis and failed relaxation of the lower esophageal
sphincter (LES). However, esophageal manometry typically reveals high LES resting pressure.

(Choice D) Systemic sclerosis is a progressive connective tissue disease characterized by fibrosis of the skin and
visceral organs. Esophageal involvement can cause dysphagia due to fibrous tissue replacement of smooth
muscle; however, esophageal manometry classically reveals dysmotility and low LES pressure.

(Choice E) Multiple sclerosis (MS) is an autoimmune disorder characterized by inflammatory demyelination of


axons in the central nervous system, which can lead to variable relapsing-remitting neurologic deficits, including
dysphagia. However, initial manifestations are more commonly ocular (eg, optic neuritis, intranuclear
ophthalmoplegia) than bulbar, and MS is not associated with a mediastinal mass.

Educational objective:
Myasthenia gravis is caused by autoantibody-mediated degradation of acetylcholine receptors at the neuromuscular
junction, which often leads to fatigable ocular and bulbar muscle weakness. Most patients with acetylcholine
receptor antibodies have thymic abnormalities (eg, thymoma), which appear as an anterior mediastinal mass on
chest imaging.

Reference
• Myasthenia gravis: a review.
Question #291

A 27-year-old woman comes to the office due to facial weakness. When the patient woke up this morning, the left
side of her face seemed to be "pulling down." She had difficulty eating breakfast, and orange juice leaked out the
left side of her mouth. The patient had some minor left ear pain last night that responded to acetaminophen and is
fully resolved. She has no chronic medical conditions and takes no other medications. Vital signs are within normal
limits. On examination, the external ear, external auditory canal, and tympanic membrane are normal. There is
weakness of the entire left side of the face, including the forehead and incomplete eye closure. The pupils are
equal and reactive to light. Hearing is intact. Which of the following is the most appropriate next step in diagnosis
of this patient?

A) Chest x-ray

B) HIV testing

C) Lumbar puncture

D) MRI of the head

E) No additional testing needed


Explanation
Correct Answer:

E) No additional testing needed

Bell palsy

• Acute onset (within hours)


Clinical features • Idiopathic facial weakness involving upper & lower face
• Decreased tear production, altered taste sensation, hyperacusis

Diagnosis of exclusion (r/o other causes of facial weakness):

• Neurologic disease (eg, stroke, GBS)


Diagnosis
• Granulomatous disease (eg, sarcoidosis)
• Infections (eg, otitis media, herpes zoster, Lyme disease)
• Neoplasms (eg, cerebellopontine angle, parotid)

• High-dose glucocorticoids
Treatment • ± Antivirals (eg, acyclovir)
• Eye protection (eg, lubrication, taping of eye at night)

GBS = Guillain-Barré syndrome; r/o = rule out.


This patient likely has Bell palsy, the most common cause of facial nerve (CN VII) palsy. Bell palsy typically
presents with acute-onset (eg, over hours), unilateral, upper and lower facial drooping (peripheral pattern). Other
common findings include an inability to raise the eyebrow or close the eye, drooping of the mouth corner (with the
mouth drawn to the unaffected side), and disappearance of the nasolabial fold. The development of facial
weakness may be preceded by a prodrome of auricular pain or dysacusis (ie, distortion of sound).

Bell palsy represents >50% of cases of facial nerve palsy, rarely causes multisystemic involvement, and has a very
good prognosis (>85% completely recover with high-dose glucocorticoids). Therefore, further diagnostic workup
is not cost effective and is not recommended when patients have a classic presentation (eg, acute onset, diffuse,
peripheral nerve palsy with no systemic findings suggestive of other conditions). Other causes of facial nerve
weakness can often be excluded with a thorough history and physical examination. Patients with Bell palsy
typically recover within a few weeks. If facial function has not returned by 4 months, further workup can be
considered for secondary causes.

(Choice A) Chest x-ray is used to screen for sarcoidosis. Neurosarcoidosis can present with facial nerve palsy;
however, the majority of patients already have an established diagnosis of sarcoidosis or have multisystem (eg,
cutaneous, ophthalmologic, gastrointestinal) involvement. Therefore, routine chest-x-ray screening is not
recommended.

(Choice B) Patients with HIV are at risk for sensory peripheral neuropathy. Isolated motor mononeuropathy is an
extremely uncommon presentation. Therefore, routine HIV screening is not recommended.

(Choice C) Suspected Guillain-Barré syndrome (GBS) should be evaluated with lumbar puncture, which often
shows albuminocytologic dissociation. It is extremely unusual not to have limb weakness in GBS. When facial
palsy occurs as part of this condition, it is usually bilateral and symmetric.

(Choice D) MRI of the head may exclude stroke in patients with facial palsy. However, stroke is unlikely in this
patient with no risk factors (eg, advanced age, hypertension) or findings (eg, sparing of upper face, onset over
seconds or minutes) suggestive of stroke.

Educational objective:
Bell palsy, a peripheral neuropathy involving the facial nerve (CN VII), presents with acute, progressive, unilateral
facial weakness. In patients with a classic presentation, no further diagnostic workup is needed, and many other
causes of facial weakness can be eliminated through a thorough history and physical examination.

Reference
• Clinical practice guideline: Bell's palsy.
Question #292

A 56-year-old woman comes to the office due to eye irritation, painful eye movements, and diplopia. Over the last
few weeks, she has also experienced weight loss and fatigue. The patient smokes a pack of cigarettes a day and
has a 25-pack-year history. Other medical problems include chronic obstructive pulmonary disease treated with an
inhaled corticosteroid and bronchodilators. She also suffered a head injury in a motor vehicle accident 6 months
ago. Blood pressure is 146/70 mm Hg, pulse is 110/min, and respirations are 18/min. On physical examination, the
patient is unable to maintain eye convergence and experiences diplopia on upward gaze. Her eyes are shown in
the image below.

Which of the following is the most likely cause of this patient's examination findings?
A) Brain stem injury

B) Increased intracranial pressure

C) Increased intraocular pressure

D) Neuromuscular junction disorder

E) Orbital tissue expansion

F) Paraneoplastic syndrome
Explanation
Correct Answer:

E) Orbital tissue expansion

Clinical manifestations of Graves disease

General Heat intolerance, weight loss, sweating

Eyes Lid lag, proptosis, diplopia

Skin Hair loss, infiltrative dermopathy (pretibial myxedema)

Cardiovascular Tachycardia, hypertension, atrial fibrillation

Nails Onycholysis, clubbing (acropachy)

Endocrine Hyperglycemia, hypercalcemia, bone loss, menstrual irregularities

Gastrointestinal Diarrhea
Neurology Tremors, hyperreflexia, proximal muscle weakness

This patient has clinical features of hyperthyroidism (ie, weight loss, tachycardia) with proptosis and impaired
extraocular motion (decreased convergence, diplopia) consistent with Graves ophthalmopathy. Other common
symptoms include irritation (eg, gritty or sandy sensation), redness, photophobia, pain, and tearing.
Ophthalmopathy in Graves disease is typically diagnosed at the same time as hyperthyroidism but may occur
before or after the onset of hyperthyroidism. Risk factors include female sex, advancing age, and smoking.

In Graves disease, thyrotropin (TSH) receptor autoantibodies (TRAB) stimulate thyroid hormone production,
resulting in hyperthyroidism. Thyroid hormone increases sensitivity to catecholamines, and thyrotoxicosis of any
etiology may cause lid lag and retraction due to sympathetic activation and contraction of the superior tarsal
muscle. However, true exophthalmos with impaired extraocular motion is seen only in Graves disease and is due
to T cell activation and stimulation of orbital fibroblasts and adipocytes by TRAB, resulting in orbital tissue
expansion and lymphocytic infiltration.

(Choice A) Brain stem dysfunction can cause diplopia but not proptosis. Furthermore, because many neural tracts
are tightly packed in the brain stem, other neurologic findings (eg, weakness, sensory deficits, altered sensorium)
are usually present.

(Choice B) Increased intracranial pressure (eg, pseudotumor cerebri) can cause papilledema but would not cause
proptosis.

(Choice C) Graves ophthalmopathy can cause increased intraocular pressure due to orbital tissue expansion and
compression of the globe. However, increased intraocular pressure alone (eg, glaucoma) does not cause proptosis.

(Choices D and F) Myasthenia gravis (MG) is a disorder of the neuromuscular junction caused by autoantibodies
against acetylcholine receptors on the motor end plate. Ocular involvement may cause fluctuating diplopia and
ptosis. Lambert-Eaton myasthenic syndrome (LEMS) is a paraneoplastic syndrome that may also cause diplopia.
However, ocular involvement by LEMS is less common than involvement of the proximal limb muscles, and neither
MG nor LEMS would cause ocular irritation, painful movement, or proptosis.
Educational objective:
Graves ophthalmopathy is characterized by ocular irritation, impaired extraocular motion, and proptosis. It is
caused by T cell activation and stimulation of orbital fibroblasts by thyrotropin (TSH) receptor autoantibodies,
leading to expansion of orbital tissues.

Reference
• Evaluating Graves' orbitopathy.

• Graves' ophthalmopathy.
Question #293

A 65-year-old man is found to have an abnormal reading during screening tonometry. The patient has no visual
symptoms or headache. Medical history is significant for asthma, for which he takes inhaled fluticasone/salmeterol
routinely and inhaled albuterol on most days to control the symptoms. Vital signs are within normal limits. Eye
examination shows normal conjunctivae, corneas, and lenses. Intraocular pressure is 28 mm Hg (normal: 8-21).
Funduscopy reveals thinning of the optic disc rim and asymmetry of the cup/disc ratio between the eyes. Which of
the following is the most appropriate next step in management of this patient's ocular condition?

A) Advise to stop glucocorticoid therapy

B) Begin atropine eye drops

C) Prescribe oral acetazolamide therapy

D) Recommend treatment only if symptomatic

E) Start latanoprost ophthalmic solution


Explanation
Correct Answer:

E) Start latanoprost ophthalmic solution


This patient has open-angle glaucoma (OAG), a form of optic neuropathy typically (but not always) associated with
elevated intraocular pressure (IOP) and characterized by atrophy of the optic nerve head (eg, optic disc rim
thinning, increased cup/disc ratio [ie, "cupping"]). OAG is usually asymptomatic in the early stages, but patients can
develop gradual loss of peripheral vision. Because vision loss from OAG is generally irreversible, treatment
should be initiated before the patient develops significant symptoms (Choice D).

OAG can be treated with a number of topical agents that lower IOP by decreasing the formation of aqueous humor
or increasing its outflow from the anterior chamber.

• The first-line agent for most patients is a topical prostaglandin (eg, latanoprost, bimatoprost), which acts
by increasing drainage of aqueous humor through the uveoscleral pathway.

• If prostaglandins are ineffective, topical beta blockers (eg, timolol) can be added, although they should be
used with caution in patients with comorbid asthma.

• If topical agents are ineffective or not tolerated, surgical intervention (eg, laser trabeculoplasty) can be
considered.

(Choice A) Systemic or ophthalmic glucocorticoids (eg, prednisone and prednisolone eye drops, respectively) can
worsen OAG by decreasing the outflow of aqueous humor from the anterior chamber. However, this adverse effect
is much rarer with inhaled glucocorticoids and is unlikely to have caused this patient's elevated IOP. Discontinuing
the inhaled glucocorticoid would increase the risk of asthma exacerbation and is not recommended.

(Choice B) Ophthalmic atropine is used to dilate the pupil to facilitate examination of the retina. It is
contraindicated in patients with OAG because the dilation can lead to angle narrowing, which results in decreased
outflow of aqueous humor from the anterior chamber, leading to acute angle-closure glaucoma.

(Choice C) Systemic (ie, oral, intravenous) carbonic anhydrase inhibitors (eg, acetazolamide) are used acutely for
angle-closure glaucoma, which presents with headache, ocular pain, vomiting, and a fixed, middilated pupil. For
OAG, topical carbonic anhydrase inhibitors (eg, dorzolamide) are preferred.

Educational objective:
Open-angle glaucoma can be treated with topical agents that lower intraocular pressure. The first-line agent for
most patients is a topical prostaglandin (eg, latanoprost, bimatoprost), which increases drainage of aqueous humor
in the anterior chamber. If prostaglandins are ineffective, topical beta blockers (eg, timolol) can be added.
Reference
• Therapy for open-angle glaucoma.
Question #294

A 75-year-old man comes to the clinic due to a gradual onset of blurred vision in both eyes and difficulty reading
over the past 2 months. He also has difficulty driving at night and reports seeing halos around bright lights. Medical
history is notable for longstanding diabetes and hypertension. The patient has a 40 pack-year smoking history.
Vital signs are normal. Corrected vision is OD (right eye) 20/80 and OS (left eye) 20/100 with normal findings on
visual field testing. Which of the following is the most likely diagnosis in this patient?

A) Cataracts

B) Diabetic retinopathy

C) Hypertensive retinopathy

D) Macular degeneration

E) Open-angle glaucoma
Explanation
Correct Answer:

A) Cataracts

This patient has progressive, bilateral blurring of vision, suggesting cataracts. A cataract is a vision-impairing
opacification of the lens, most commonly related to oxidative damage of the lens with aging. Other risk factors for
cataract include diabetes, smoking, chronic sunlight exposure, and glucocorticoid use.
Cataracts are usually bilateral, but patients may become symptomatic in one eye before the other. Patients usually
report painless blurring of vision, bothersome glare, and often halos around lights at night due to scattering of
light in the lens. Ocular examination in early cataract formation may show a normal red reflex with clear
visualization of the retina; as the cataract progresses, the red reflex is lost and retinal detail may not be visible.

Cataracts typically follow a slowly progressive course, and treatment is indicated when loss of vision impairs
activities of daily living. Definitive treatment is lens extraction with artificial lens implantation.

(Choice B) Diabetic retinopathy is characterized by vascular proliferation with or without vitreous hemorrhage.
Symptoms are a late finding and may include floaters, decreased visual acuity, focal visual field defects, and acute
monocular vision loss. Gradually progressive blurring with glare and halos is more consistent with cataracts.

(Choice C) Hypertension can cause progressive arteriolar changes in the retina. Most patients with hypertensive
retinopathy are asymptomatic; symptoms can occur due to retinal hemorrhage but typically present acutely with
monocular vision loss.

(Choice D) Macular degeneration causes decreased acuity of central vision. It occurs in 2 primary forms: atrophic
(dry), which causes slowly progressive, vision loss that involves one or both eyes and is usually associated with
scotoma (ie, blind spot); and exudative/neovascular (wet), which causes unilateral, aggressive vision loss, often
starting with straight line distortion. However, difficulty with night driving, typically due to glare, and halos are more
suggestive of cataracts.

(Choice E) Open-angle glaucoma presents with insidious, gradual loss of peripheral vision. This may lead to near-
miss motor vehicle collisions, but night driving is not preferentially affected. Open-angle glaucoma also does not
cause halos around bright lights; these are sometimes seen with acute angle-closure glaucoma, which classically
presents with severe eye pain, conjunctival redness, and a poorly reactive mid-dilated pupil.

Educational objective:
A cataract is a vision-impairing opacification of the lens. Patients usually have painless blurred vision, glare, and
often halos around lights. Treatment with lens extraction and artificial lens implantation is indicated when loss of
vision impairs activities of daily living.

Reference
• Cataracts.
Question #295

A 53-year-old man comes to the office because of difficulty reading fine print over the last year. He now has to hold
books, menus, and magazines at an arms length in order to read them. He has never had visual problems before.
Which of the following is most likely abnormal in this patient?

A) Corneal shape

B) Lens elasticity

C) Lens opacity

D) Macula

E) Peripheral retina

F) Intraocular pressure
Explanation
Correct Answer:

B) Lens elasticity

This patient is most likely suffering from presbyopia, which is a common age-related disorder that results from the
loss of elasticity in the lens. This decrease in elasticity prohibits accommodation of the lens, which is required in
order to focus on near objects. The tendency of patients to hold reading material at a further distance is classic for
presbyopia. Patients often have no history of eye problems. Most patients will report the onset of presbyopia while
they are in their forties, and symptoms typically peak at some point in their sixties. The poor near vision associated
with presbyopia can easily be improved with reading glasses.

(Choice A) A nonspherical cornea can lead to astigmatism, which typically presents with blurry vision both at a
distance and up close.

(Choice C) Increased lens opacity is the etiology of cataracts. Patients may have difficulty reading, but a history of
difficulty with night vision or driving at night is more characteristic.

(Choice D) Age-related macular degeneration is a common cause of visual loss in patients over the age of 50, but
it is typically associated with central visual field loss.

(Choice E) Disease located in the peripheral retina is usually secondary to diabetes, which can lead to either
proliferative or non-proliferative retinopathy. This patient, however, has no known history of diabetes.

(Choice F) Elevated intraocular pressure is the etiology of glaucoma. Primary open-angle glaucoma usually
presents with peripheral visual field defects followed by central visual loss.

Educational objective:
Presbyopia is a common age-related decrease in lens elasticity that leads to difficulty with near vision. A history of
a middle-aged individual who has to hold books at an arms length to read is classic.
Question #296

A 9-year-old boy is brought to the emergency department due to a red eye. His mother states, "he woke up this
morning, and his right eye was red." The mother and patient deny any injury. He has no eye pain, itching, or
discharge. The patient has not had similar episodes in the past. He has no known drug or environmental allergies.
The patient has no medical conditions and takes no medications; his vaccines are up to date. Vital signs are
normal. On examination, the pupils are equal, round, and reactive to light. Visual acuity is normal. The photo of his
eye is shown in the image below. What is the best next step in management of this patient?
A) Coagulation studies

B) Fluorescein eye examination

C) Formal visual field testing

D) Measurement of intraocular pressure

E) Reassurance
Explanation
Correct Answer:

E) Reassurance
This patient with an atraumatic and painless red eye has a thin layer of blood on the eye surface consistent with
subconjunctival hemorrhage (SH). SH is a benign condition that occurs when a small conjunctival blood vessel
ruptures, leading to blood collecting in the space between the sclera (the white part of the eye) and the
conjunctiva (the mucous membrane covering the sclera). SH can be due to minor trauma (eg, rubbing the eyes
vigorously), but in many cases there is no apparent cause.
Patients with SH are typically asymptomatic and have a normal eye examination (eg, normal pupillary reflex,
extraocular movements, and visual acuity) except for the presence of a flat and focal area of blood on the ocular
surface. SH is a clinical diagnosis that does not require any further work-up. Patients can be reassured that no
medications or interventions are necessary; the hemorrhage typically spreads, fades, and becomes yellow in color,
and then disappears completely within 1-3 weeks.

(Choice A) Patients who develop SH while on anticoagulation therapy or who have recurrent, unexplained SH may
warrant coagulation studies. However, coagulation studies are not necessary in this healthy child with a first
episode of SH.

(Choice B) Fluorescein eye examination is performed when there is concern for corneal abrasion. Although
corneal abrasion is a common cause of a red eye, it typically causes eye pain and tearing, which are not present in
this patient.

(Choice C) Formal visual field testing (typically performed with visual field perimetry testing equipment) is
commonly used in the outpatient setting to screen for or monitor glaucoma, retinopathy, or conditions affecting the
optic nerve/visual pathways, which can cause scotomas (ie, areas of vision loss). It is not indicated in this healthy
child with normal visual acuity and a clinical examination consistent with SH.

(Choice D) Measurement of intraocular pressure is indicated in patients with suspected glaucoma. Acute angle
closure glaucoma can cause conjunctival erythema/injection; however, it is rare in children and typically also causes
ocular pain, headache, and decreased visual acuity.

Educational objective:
Subconjunctival hemorrhage is a benign condition in which blood collects in the space between the conjunctiva and
the sclera. Patients are typically asymptomatic. The diagnosis is clinical, and no treatment is necessary.

Reference
• Subconjunctival hemorrhage: risk factors and potential indicators.
Question #297

A 32-year-old man arrives at the emergency department with pain, watering, and redness in his left eye for the past
5 days. He had similar symptoms in the same eye a few months ago that resolved after about a week. The patient
is a construction worker and has been remodeling the attic and roof of an older home for the past few weeks.
Temperature is 36.8 C (98.2 F), blood pressure is 118/78 mm Hg, heart rate is 82/min. Clear drainage is visible
from the left eye, which is diffusely erythematous around the cornea. The right eye and bilateral periorbital regions
are unremarkable. There are no oral lesions. Slit-lamp examination of his left eye is shown below:

What is the most likely diagnosis for this patient?

A) Bacterial keratitis

B) Corneal abrasion

C) Fungal keratitis
D) Herpes simplex keratitis

E) Herpes zoster ophthalmicus


Explanation
Correct Answer:

D) Herpes simplex keratitis


This patient with pain, watering, and redness of the eye has branching ulcerations on slit-lamp examination with
fluorescein dye. These findings are consistent with herpes simplex virus (HSV) keratitis, a manifestation of viral
reactivation and a frequent cause of corneal blindness.

HSV keratitis occurs most commonly in adults and usually affects one eye in otherwise healthy patients. Symptoms
include the acute onset of eye pain, photophobia, tearing, and blurred vision. Conjunctival erythema around the
corneal border or limbus (ie, ciliary flush) is classic, as seen here. A history of prior episodes (usually affecting the
same eye) may be present, and the recurrences are often provoked by excessive sun exposure (eg, outdoor
occupation), fever, or immunodeficiency (eg, HIV).

Diagnosis of HSV keratitis is typically made by slit-lamp examination with fluorescein dye, which classically reveals
dendritic ulcers (linear branching lesions); corneal vesicles may also be present. Viral culture or PCR can confirm
the diagnosis in atypical cases. Management is antiviral therapy (oral or topical); topical glucocorticoid
monotherapy should be avoided if there is any suspicion of HSV keratitis due to associated risk of vision loss.
Patients with frequent recurrences may warrant oral suppressive therapy.

(Choice A) Bacterial keratitis usually occurs after improper contact lens use or corneal trauma. Patients typically
have mucopurulent drainage (not clear watering), and the cornea appears hazy with a round, white ulceration, not
branching lesions.

(Choice B) Corneal abrasion presents with severe pain, photophobia, and foreign-body sensation after trauma
(which did not occur in this case) and usually improves within 1-2 days. In addition, fluorescein staining may reveal
a linear lesion, but the branching pattern seen in this patient is not typical.

(Choice C) Fungal keratitis is rare and typically occurs after corneal injury (eg, thorn in the eye) in
immunocompromised patients. In addition, presentation is gradual and progressive rather than acute, and the
cornea shows multiple infiltrates with feathery edges, not dendritic branching.

(Choice E) Herpes zoster ophthalmicus is an infection caused by varicella zoster virus and most commonly affects
elderly individuals. Presentation includes fever, malaise, and burning/pruritus of the periorbital region. Dendritic
corneal ulcers can occur, but a vesicular rash around the eye in the distribution of the cutaneous branch of the first
division of the trigeminal nerve is almost always present.
Educational objective:
Herpes simplex keratitis presents acutely with eye pain, redness, photophobia, tearing, and blurred vision. Slit-lamp
examination reveals characteristic dendritic ulcers (linear branching).

Reference
• Herpes simplex keratitis.
Question #298

A 26-year-old man is evaluated for sudden onset of redness, watery discharge, and itching of both eyes. His vision
is slightly distorted from excess tearing but is not blurred. He feels fine otherwise. Medical history is significant for
obesity, asthma, sleep apnea, and chronic back pain. Eye examination shows bilateral injection with granular
appearance of the conjunctiva. Mild eyelid swelling and clear discharge are also noted. Pupils are equally round
and reactive to light. The lungs have a few scattered wheezes. What is the most likely diagnosis in this patient?

A) Allergic conjunctivitis

B) Anterior uveitis

C) Atopic keratoconjunctivitis

D) Bacterial conjunctivitis

E) Viral conjunctivitis
Explanation
Correct Answer:

A) Allergic conjunctivitis

Acute conjunctivitis*

Viral Bacterial Allergic

Distribution Unilateral or bilateral Unilateral or bilateral Bilateral

Discharge Watery/mucoid Purulent Watery

Conjunctival Diffuse injection;


Diffuse injection; nonfollicular Diffuse injection; follicular ("bumpy")
appearance follicular ("bumpy")

Associated Unremitting discharge Ocular pruritus, history of atopy (eg,


Viral prodrome
findings (reaccumulates within minutes) allergic rhinitis, asthma)

<30 minutes (often sudden onset) to


Duration 1-2 weeks 1-2 weeks
perennial
*Red flags suggestive of alternate etiology: decreased visual acuity, photophobia, pain with extraocular
movement & fixed/distorted pupil.

This patient has new-onset conjunctival injection and discharge, findings most commonly due to acute
conjunctivitis. Conjunctivitis is characterized by inflammation of the conjunctiva, the mucous membrane that lines
the inside of the eyelids (tarsal conjunctiva) and anterior sclera (bulbar conjunctiva).

Common causes of acute conjunctivitis include:

• Allergic conjunctivitis: Patients develop bilateral conjunctival injection and watery discharge, as seen in
this case. Because this condition is due to a type I hypersensitivity reaction to environmental allergens (eg,
pollen, dust), ocular pruritus is characteristic and begins soon after exposure to the allergen, which may
not always be readily identifiable; the sudden symptom onset in this case is consistent with the diagnosis.
Eyelid edema (due to histamine release and rubbing) is common, and the tarsal conjunctiva may have a
nonspecific, granular (follicular or "bumpy") appearance due to lymphoid aggregates. Findings
suggestive of other atopic conditions, such as allergic rhinitis (eg, rhinorrhea, sneezing) and asthma (eg,
wheezing), may be present (as in this patient) and are also often triggered by environmental allergens.

• Viral conjunctivitis: Like allergic conjunctivitis, patients develop watery discharge and granular tarsal
conjunctiva; upper respiratory symptoms (eg, rhinorrhea) usually accompany ocular findings. In contrast to
allergic conjunctivitis, ocular pruritus is uncommon, making this diagnosis less likely in this patient (Choice
E). Moreover, viral infections often start with unilateral conjunctival injection and involve the contralateral
side within 1-2 days.

• Bacterial conjunctivitis: Symptoms classically include unilateral conjunctival injection with thick, purulent
discharge that reaccumulates within minutes of wiping; contralateral involvement often occurs within 1-2
days (Choices D).

The presence of severe pain, photophobia, or blurry vision (none of which is seen in this patient) should raise
concern for a diagnosis other than conjunctivitis.
(Choice B) Inflammation of the anterior uveal tract causes iritis (or anterior uveitis). Redness of the eye is often
associated with pain, photophobia, and decreased visual acuity, none of which are seen in this patient. In addition,
discharge and pruritus are not expected.

(Choice C) Atopic keratoconjunctivitis is a form of allergic conjunctivitis in which redness, pruritus, and tearing are
common, but patients have chronic (not acute) symptoms, often with photophobia and a foreign body sensation.
Moreover, signs of chronic eyelid inflammation (eg, thickened, lichenified skin) are typical.

Educational objective:
Acute allergic conjunctivitis causes bilateral conjunctival injection with watery discharge and granular (follicular)
tarsal conjunctiva. Ocular pruritus is a key feature that distinguishes allergic conjunctivitis from viral conjunctivitis.

Reference
• Conjunctivitis: a systematic review of diagnosis and treatment.
Question #299

A 29-year-old man comes to the office for evaluation of blurred vision and right eye pain. The pain increases when
he is in bright light. The patient also has a yearlong history of generalized fatigue, sleep problems, back pain, and
occasional heel pain. He has no recent illnesses. Medical history is notable for obesity and hypertension treated
with lisinopril and chlorthalidone. The patient is monogamous. He lives in Texas and has not traveled outside the
state. Temperature is 36.8 C (98.2 F), blood pressure is 134/88 mm Hg, and pulse is 80/min. Examination of the
right eye shows redness of the sclera, which is most prominent at the junction between the cornea and the sclera.
The right pupil is constricted, and there is photophobia with penlight examination. The left eye is normal. There are
no skin lesions. In addition to treating this patient's eye condition, what additional workup is needed?

A) Anti-Ro/SSA antibody assay

B) Lyme serology

C) MRI of the brain

D) Rheumatoid factor

E) X-ray of the sacroiliac joints


Explanation
Correct Answer:

E) X-ray of the sacroiliac joints

Anterior uveitis

• Symptoms: ocular pain, photophobia, decreased acuity


Presentation
• Examination: ciliary flush, pupillary constriction, hypopyon

• Infections: herpesviruses, toxoplasmosis, syphilis


Common causes/ • Sarcoidosis
associations • Spondylarthritis (eg, ankylosing spondylitis, reactive arthritis)
• Inflammatory bowel disease

• Slit-lamp examination
Evaluation/testing
• As indicated: HLA-B27, pelvis/spine x-ray, chest x-ray, HIV, syphilis serology

• Dilating eye drops (eg, cyclopentolate)


Treatment
• Topical glucocorticoids

HLA-B27 = human leukocyte antigen B27.


This patient has anterior uveitis with eye pain, photophobia, decreased visual acuity, and scleral hyperemia at the
corneal margin (ie, ciliary flush). Anterior uveitis is inflammation of the anterior uveal tract (ie, iris, ciliary body) and
is sometimes termed iritis (or iridocyclitis if the ciliary body is involved). Other common findings include pupillary
constriction, a hazy flare in the aqueous humor, and layering of white cells in the anterior chamber (hypopyon). The
diagnosis can be confirmed on slit-lamp examination.

Anterior uveitis can be idiopathic but often occurs in association with systemic inflammatory disorders. Further
evaluation depends on the presence of associated features that could suggest a systemic disease:

• No evidence of systemic disease: Chest x-ray and serum rapid plasma reagin should be obtained because
both sarcoidosis and syphilis can be silent.

• Evidence of systemic symptoms: For back pain or enthesitis, x-ray of the sacroiliac joints should be
obtained to rule out spondylarthritis (eg, ankylosing spondylitis).

This patient also has chronic back pain and possible enthesitis (ie, Achilles tendonitis/heel pain), findings
suggestive of ankylosing spondylitis. Specific x-ray findings include widening of the sacroiliac joints and
subchondral sclerosis. Some experts also advise human leukocyte antigen B27 typing, although it has low
specificity.

(Choices A and D) Anti-Ro/SSA antibodies are seen in Sjögren syndrome, which can cause dry eyes and corneal
ulceration. Rheumatoid factor is associated with rheumatoid arthritis, which can cause episcleritis (acute redness
and mild irritation) and scleritis (severe ocular pain with diffuse scleral erythema and edema). However, these
conditions are not commonly associated with uveitis. In addition, although rheumatoid arthritis can affect the
cervical spine, back pain is unusual.

(Choice B) Lyme disease is occasionally associated with uveitis. However, this patient has no rash to suggest the
disease, and the causative organism (Borrelia burgdorferi) is not endemic to Texas and the Southcentral United
States.

(Choice C) MRI is used in the evaluation of multiple sclerosis, which is occasionally associated with uveitis,
although optic neuritis is more common. However, this patient has no neurologic findings (eg, sensory/motor
deficits) to suggest multiple sclerosis, and his musculoskeletal symptoms are more suggestive of ankylosing
spondylitis.

Educational objective:
Anterior uveitis often occurs in association with systemic inflammatory disorders, in particular spondylarthritis (eg,
ankylosing spondylitis). Patients with uveitis and features suggesting ankylosing spondylitis (eg, chronic back pain,
enthesitis) should undergo x-ray of the pelvis/sacroiliac joints and possibly human lymphocyte antigen B27 typing.
Question #300

A 34-year-old previously healthy woman comes to the office due to itchy eyes for the past several months. The
patient also reports frequent gritty sensations in her eyes and crusting of her eyelashes in the morning. She has
had no eye pain, photophobia, or vision disturbances. The patient has not been exposed to ill contacts, uses no
facial cosmetic products, and has not changed bath products recently. Eye examination is shown in the exhibit.
Seborrhea is present at the scalp and eyebrows, but the skin of the face is otherwise normal. Nasal and
oropharyngeal mucosa are normal. Which of the following is the most likely diagnosis?

A) Allergic conjunctivitis

B) Blepharitis

C) Dry eye disease

D) Hordeolum

E) Preseptal cellulitis
Explanation
Correct Answer:

B) Blepharitis

This patient has typical symptoms of chronic blepharitis. Blepharitis is a common disorder characterized by
inflammation at the eyelid margin, usually most prominent at the opening of the meibomian glands. The etiology is
variable and can be multifactorial; common contributors include seborrheic dermatitis, rosacea, allergic disorders,
bacterial infection (especially Staphylococcus species), viral infection (eg, herpes simplex), and Demodex mite
infestation.

Blepharitis presents with burning or itching of the lids, discharge (which may be associated with crusting of the
eyelashes in the morning), and often a foreign body sensation in the eye. Examination shows redness, swelling,
and scaling at the lid margin, and the adjacent conjunctiva may also be inflamed. Close inspection may reveal
lipoid plugs at the meibomian glands. Findings are frequently bilateral. The diagnosis is primarily based on clinical
features. Treatment is generally supportive and includes warm compresses, gentle scrubs, and lid massage.

(Choice A) Allergic conjunctivitis causes conjunctival erythema, discharge, and foreign body sensation; itching may
be severe. However, conjunctival edema is usually prominent (often with papilla formation), the findings are
primarily at the conjunctivae rather than lid margin, and patients typically have seasonal allergies or other atopic
disorders.

(Choice C) Dry eye disease (eg, Sjögren syndrome) can cause discomfort, a foreign body sensation, and
conjunctival erythema, and it can occur concurrently with blepharitis. However, it would not cause erythema,
swelling, or scaling of the lid margin.

(Choice D) A hordeolum (ie, stye) is an acute inflammatory disorder of the eyelash follicle or tear gland and
presents as a tender, erythematous nodule at the lid margin. Findings are focal, rather than diffuse/bilateral.

(Choice E) Preseptal cellulitis presents with acute, unilateral eyelid swelling and erythema. The swelling typically
involves the entire lid (not just the margin), and patients often have a recent insect bite or another break in the skin.
Educational objective:
Blepharitis presents with burning or itching of the lids, discharge, and often a foreign body sensation in the eye.
Examination shows redness, swelling, and scaling at the lid margin, and the adjacent conjunctiva may also be
inflamed. Common contributors include seborrheic dermatitis, rosacea, allergic disorders, bacterial infection, viral
infection, and Demodex mite infestation.

Reference
• Medical management of blepharitis.
Question #301

A 33-year-old man comes to the office due to painful swelling of the right lower eyelid for the past 2 days. He has
had no trauma or history of similar symptoms in the past. The patient says he recently visited his niece who was
having an upper respiratory tract infection. He has no significant prior medical problems other than occasional
episodes of herpes labialis. He is sexually active with his girlfriend who takes oral contraceptives. Temperature is
36.8 C (98.2 F), blood pressure is 130/80 mm Hg, and pulse is 80/min. Examination of the right eye shows
localized swelling along the margin of the lower eyelid with associated erythema and tenderness. There is no
conjunctival injection or ocular discharge. Visual acuity is normal, and the left eye appears unremarkable. Which of
the following is the most appropriate initial management of this patient's current condition?

A) Erythromycin ointment

B) Incision and curettage

C) Oral doxycycline

D) Topical ganciclovir

E) Warm compress only


Explanation
Correct Answer:

E) Warm compress only

This patient has an external hordeolum (stye). A hordeolum is an acute inflammatory disorder of the eyelash
follicle or tear gland and presents as an erythematous, tender nodule at the lid margin. It is often due to infection
with Staphylococcus aureus but can be sterile in many cases. A similar process arising in the meibomian gland
(internal hordeolum) presents as a tender nodule visible at the palpebral conjunctiva but is less common.

Within a few days, a minute pustule may appear at the lid margin (pointing), which will then rupture with discharge
of pus and relief of pain. Warm compresses are advised to accelerate the process. Following resolution of
infection, some patients have a residual granulomatous nodule (chalazion) that regresses slowly over several
months. For patients with a persistent hordeolum (eg, >1-2 weeks) or a large chalazion, additional management
options include incision and curettage (Choice B).

(Choice A) Erythromycin ophthalmic ointment is used for treatment of bacterial conjunctivitis and in the prevention
of ophthalmia neonatorum due to Neisseria gonorrhoeae. Topical antibiotics are often prescribed for hordeolum but
are usually unnecessary.

(Choice C) Preseptal cellulitis is an infection of the eyelid anterior to the orbital septum. It presents with fever and
leukocytosis as well as erythema and edema of the eyelid and is treated with oral antibiotics (eg, doxycycline).

(Choice D) Viral keratitis is infection of the cornea due to herpes simplex or varicella zoster virus. Patients have
corneal vesicles, opacification, and/or dendritic ulcers. Treatment includes topical ganciclovir or trifluridine.

Educational objective:
An external hordeolum is an acute inflammatory disorder of the eyelash follicle or tear gland and presents as an
erythematous, tender nodule at the lid margin. It is often due to infection with Staphylococcus aureus but can be
sterile. Initial treatment includes warm compresses.
Question #302

An 81-year-old man comes to the urgent care clinic due to severe pain across the right side of the forehead and the
right eye over the past 2 days. The pain is "burning" and, intermittently, "stabbing" in quality. The patient reports
feeling more fatigued with intermittent headaches and low-grade fevers over the same period. Since this morning,
he has also noted increased aching and increased tearing of the right eye. Temperature is 37.2 C (99 F), blood
pressure is 138/90 mm Hg, pulse is 92/min, and respirations are 14/min. On physical examination, the right side of
the forehead, nasal sidewall, and eyelid are hypersensitive to touch and contain scattered erythematous papules.
The right eye is red, with chemosis of the conjunctiva and increased tearing. The remainder of the physical
examination is normal. Which of the following is the most appropriate treatment for this patient's condition?

A) Broad-spectrum antibiotic therapy

B) Carbamazepine therapy

C) Inhaled 100% oxygen

D) Systemic antiviral therapy

E) Systemic glucocorticoid therapy


Explanation
Correct Answer:

D) Systemic antiviral therapy

This patient has neuropathic (eg, "burning," "stabbing") pain and rash (eg, erythematous papules) in the V1
distribution of the trigeminal nerve, as well as pain, increasing tearing, and conjunctival chemosis of the associated
eye. This presentation is most consistent with herpes zoster ophthalmicus (HZO), a condition caused by the
reactivation of latent varicella-zoster virus (VZV) in the trigeminal ganglion.

VZV reactivation is most common in elderly or immunocompromised patients and can cause systemic symptoms
(eg, headache, malaise, fever) as part of the prodrome, in addition to typical sensory disturbances (eg, burning,
itching, tingling). The classic rash typically begins with small, erythematous papules that become confluent and
evolve into vesicles or pustules, with subsequent ulceration and crusting. Involvement of the side or tip of the nose
(Hutchinson sign) correlates highly with ocular involvement.

Ocular involvement in HZO can lead to acute keratitis, corneal ulceration, and vision loss. Therefore, systemic
antiviral therapy (eg, acyclovir, famciclovir, valacyclovir) should be promptly initiated. Oral antiviral therapy is
typical, but intravenous therapy may be required for patients who are immunocompromised or have sight-
threatening corneal involvement.

(Choice A) Preseptal cellulitis (treated with broad-spectrum antibiotics) can cause low-grade fevers, unilateral
ocular pain, eyelid hypersensitivity, and, occasionally, conjunctival chemosis. However, this patient's dermatomal
rash and prodrome of neuropathic pain make HZO more likely.

(Choice B) Trigeminal neuralgia (often treated with carbamazepine) can cause short episodes of severe,
neuropathic pain. However, it is more common in the V2 and V3 (vs the V1) distributions of the trigeminal nerve
and is not associated with a dermatomal rash.

(Choice C) Cluster headaches (treated with 100% inhaled oxygen) can present with intermittent, severe, unilateral
periorbital pain accompanied by ipsilateral autonomic symptoms (eg, conjunctival injection, tearing). However,
additional automatic symptoms such as ptosis, miosis, and rhinorrhea are commonly present, and a dermatomal
rash is not expected.

(Choice E) Giant cell arteritis (treated with systemic glucocorticosteroids) occurs in elderly patients and can cause
constitutional symptoms (eg, fatigue, fever, headache) and skin (typically scalp) tenderness but does not cause a
rash. When vision complications occur, they are typically ischemic (eg, anterior ischemic optic neuropathy) and
painless, without involvement of the conjunctiva (eg, chemosis).

Educational objective:
Herpes zoster ophthalmicus is characterized by a painful dermatomal rash and eye involvement. It is caused by
reactivation of varicella-zoster virus in the V1 division of the trigeminal nerve and is treated with systemic antiviral
therapy.

Reference
• Eye and periocular skin involvement in herpes zoster infection.
Question #303

A 19-year-old woman comes to the university health center due to "feeling cold all the time." She wears sweaters
despite the warm weather and sleeps with multiple blankets at night. The patient also states that she has had
difficulty losing weight and wonders if a thyroid problem could be causing her symptoms. She says, "I go to the gym
at least 5 times a week, but I feel like I'm stuck at the same weight. I've even cut bread from my diet because it was
making me constipated." Menarche was at age 12, and the patient's last menstrual period was 2 months ago.
Family history is significant for Graves disease in her mother and Sjögren syndrome in her maternal grandmother.
Temperature is 36.2 C (97.2 F), blood pressure is 80/50 mm Hg, pulse is 38/min, and respirations are 16/min. BMI
is 15 kg/m2. The patient is alert and cooperative but appears tired. Laboratory results are as follows:

Serum chemistry
Sodium 136 mEq/L
Potassium 2.9 mEq/L
Chloride 96 mEq/L
Blood urea nitrogen 14 mg/dL
Creatinine 0.6 mg/dL
Calcium 8 mg/dL
Glucose 78 mg/dL
Magnesium 2.0 mg/dL
Phosphorus 2 mg/dL

Endocrine
Thyroid function tests
TSH 4.5 µU/mL
Thyroxine (T4), serum 4.0 µg/dL
Triiodothyronine (T3), serum 65 ng/dL

Urine hCG negative

Oral electrolyte supplements are administered. Which of the following is the best next step in management?

A) Administer fluoxetine

B) Administer levothyroxine

C) Administer low-dose olanzapine

D) Admit to hospital

E) Refer for cognitive-behavioral therapy, with follow-up in 2 weeks

F) Refer for nutritional rehabilitation, with follow-up in 2 weeks


Explanation
Correct Answer:

D) Admit to hospital

Anorexia nervosa

• Underweight (BMI <18.5 kg/m2)


Clinical features
• Fear of weight gain, distorted body image

• Psychotherapy (individual, family, group)


Treatment • Nutritional rehabilitation
• Olanzapine if severe/refractory

• Bradycardia (<40/min), dysrhythmia


• Hypotension (<80/60 mm Hg), orthostasis
• Hypothermia (<35 C)
Indications for hospitalization
• Electrolyte disturbance, marked dehydration
• Organ compromise (renal, hepatic, cardiac)
• <70% expected weight (BMI <15 kg/m2)

Anorexia nervosa is an eating disorder that usually begins in adolescence. It is characterized by distorted body
image, determined dieting, and phobic avoidance of many foods, resulting in unsafe weight loss. Risk factors
include high-achieving, type-A personalities and participation in activities in which there is pressure to be slender
(eg, ballet, running). To lose weight, patients may fast and/or exercise excessively (restricting subtype) or
compensate for binge eating with laxatives or vomiting (purging subtype). Many patients also have secondary
amenorrhea.

Nutritional rehabilitation and psychotherapy (ie, individual, group, family) are the primary interventions for
medically stable patients. Indications for hospitalization include signs of dehydration, unstable vital signs,
cardiac dysrhythmias, electrolyte disturbances, organ involvement due to malnutrition, or very low weight. This
patient is medically unstable and fulfills criteria for hospitalization (pulse < 40/min, blood pressure < 80/60 mm Hg,
hypokalemia, hypophosphatemia). Meals should be supervised; some patients may require nasogastric tube
feeding.

(Choices A and C) These medications are generally used as second-line adjunctive treatments. Olanzapine may
aid in weight restoration and can be considered if nutritional counseling and psychotherapy are ineffective.
Selective serotonin reuptake inhibitors such as fluoxetine can be helpful if comorbid depression or anxiety
symptoms do not resolve with refeeding. Fluoxetine is used as primary treatment for bulimia nervosa (not anorexia
nervosa).

(Choice B) Patients with anorexia nervosa often have low T3 and/or T4 levels (also known as euthyroid-sick
syndrome) due to the body's adaption to chronic nutritional depletion. Thyroid replacement therapy is not indicated
as these levels will normalize with adequate weight gain. Replacement therapy is also potentially dangerous in
these patients due to the risk for cardiac arrhythmias and osteopenia.

(Choices E and F) Psychotherapy and nutritional rehabilitation are the mainstay of treatment for medically stable
patients, but this patient requires prompt inpatient stabilization rather than outpatient referral.

Educational objective:
Indications for hospitalization in patients with anorexia nervosa include unstable vital signs, cardiac dysrhythmias,
electrolyte derangements, and severely low body weight.

Reference
• Medical complications of anorexia nervosa and their treatments: an update on some critical aspects.
• Medical complications of anorexia nervosa and bulimia.

• Initial evaluation, diagnosis, and treatment of anorexia nervosa and bulimia nervosa.
Question #304

A 79-year-old woman comes to the office for her yearly checkup. She reports feeling well physically but mentions
some sleep issues. The patient used to sleep 8-9 hours a night. Now, she wakes up in the middle of the night and
arises an hour earlier than she used to, around 5:30 AM. She has little difficulty falling asleep and does so most
evenings while sitting and watching television. She gets a total of 6½ to 7 hours of sleep and often awakens 1 or 2
times to go to the bathroom, returning to sleep within 10-15 minutes. The patient's husband says that she snores
softly but reports no breathing pauses, gasping, or choking. During daytime hours, the patient has energy in the
morning but often takes a 30- to 45-minute nap in the afternoon. She reports no persistent anxiety or depression.
She has a good appetite and is able to follow television programs, concentrate on crossword puzzles, and likes to
go out with friends in the morning. Medical conditions include osteoporosis and arthritis. Medications include
alendronate and ibuprofen as needed. The patient does not use alcohol or illicit drugs. Blood pressure is 124/82
mm Hg, pulse is 66/min, and respirations are 14/min. BMI is 27 kg/m2. In addition to sleep hygiene education,
which of the following is the next best step regarding this patient's sleep?

A) Lorazepam

B) Low-dose zolpidem

C) Melatonin

D) Polysomnogram

E) Ramelteon

F) Reassurance

G) Sleep restriction therapy


Explanation
Correct Answer:

F) Reassurance

This patient's sleep pattern is consistent with normal aging. Changes in sleep architecture that can occur with
age include increased nighttime sleep onset latency, decreased REM latency, and decreased slow-wave sleep.
Patients can experience decreased total sleep time, peak sleepiness earlier in the evening, nocturnal
awakenings, reduced sleep during early-morning hours, and daytime napping/reduced daytime sleep latency, as
seen in this patient. Due to these changes, elderly patients may experience difficulty falling asleep and/or
maintaining sleep.

Provided that patients do not have significant impairment in activities of daily living or cognition, no further workup is
necessary, and they should be reassured that these changes are normal. However, when insomnia is functionally
impairing, nonpharmacologic interventions and cognitive-behavioral therapy are used first line. Pharmacotherapy is
generally limited to short-term management.

(Choices A and B) Benzodiazepines (eg, lorazepam) and nonbenzodiazepine hypnotics (eg, zolpidem) are not
indicated in this patient with normal, age-related sleep changes. The elderly are at a particularly high risk of
adverse effects from hypnotic medications. Therefore, these medications should be avoided if possible because
they are associated with an increased risk of delirium, impaired cognition, over sedation, falls, and fractures.

(Choices C and E) Although these medications could be considered for patients with diagnosed insomnia, they are
not indicated for this patient with normal sleep changes and no evidence of daytime impairment. Melatonin may
have some, albeit limited, efficacy in the treatment of some circadian rhythm disorders (eg, delayed sleep-wake
phase disorder) and sleep onset insomnia in the elderly. This patient does not have any sleep onset insomnia.
Ramelteon, a melatonin agonist used for sleep onset insomnia, has demonstrated safety in older adults and could
be considered when nonpharmacological interventions are ineffective.

(Choice D) Polysomnography may help rule out possible underlying etiologies of disrupted sleep when the cause
is less clear. However, this patient has classic symptoms of normal, age-related sleep changes and does not snore
loudly or have breathing pauses.

(Choice G) Sleep restriction therapy is a nonpharmacologic treatment for patients with insomnia who spend too
much nonsleeping time in bed; this patient does not have insomnia and does not spend excessive time in bed.

Educational objective:
Normal, age-related sleep changes include decreased total sleep time; increased nighttime awakenings; sleepiness
earlier in the evening with early morning awakening; and increased daytime somnolence (napping).

Reference
• Aging and sleep: physiology and pathophysiology.

• Modelling changes in sleep timing and duration across the lifespan: changes in circadian rhythmicity or
sleep homeostasis?
Question #305

A 33-year-old man with an unremarkable medical history comes to the office due to fatigue and weight loss over the
past 3 months. He awakens at 4:00 AM each day and is unable to fall back asleep. The patient also mentions that
he had enjoyed golf and playing with his children but now has little interest in either activity. Although he used to
excel at work, he now has no motivation and "spaces out" at times. Physical examination and laboratory studies,
including TSH level, are normal. The patient is diagnosed with major depressive disorder and prescribed
fluoxetine. Two weeks later, he calls to report that he is sleeping and eating better but is still depressed. The
patient is discouraged about his lack of improvement and says, "I don't feel like the medication is doing very much."
Which of the following is the most appropriate next step in pharmacological management?

A) Add aripiprazole to augment antidepressant response

B) Add bupropion to augment antidepressant response

C) Continue fluoxetine at present dose

D) Discontinue fluoxetine and begin a different selective serotonin reuptake inhibitor (SSRI)

E) Discontinue fluoxetine and begin a nonselective SRI

F) Increase dose of fluoxetine


Explanation
Correct Answer:

C) Continue fluoxetine at present dose

Antidepressants take time to work, and this patient's lack of global improvement at the 2-week mark is not
uncommon. In addition, although he still feels depressed, he shows signs of improvement in some depressive
symptoms (ie, sleep and appetite) at his current dosage. An adequate antidepressant trial is generally
considered to be at least 4-6 weeks. The patient should be educated that 2 weeks is considered an inadequate
trial and be encouraged to continue the medication for at least 2-4 more weeks. Inadequate antidepressant dose or
duration is a common reason for perceived lack of response, and physicians must ensure that patients receive an
adequate trial before recommending next-step treatments.

(Choices A and B) Augmentation strategies such as adding aripiprazole or bupropion are generally considered
when a patient has experienced a partial response to an adequate trial of monotherapy. It would be premature to
add these medications after only 2 weeks of fluoxetine therapy.

(Choices D and E) Switching to a different antidepressant is a reasonable strategy if the patient shows little
improvement after a 4- to 6-week trial. Discontinuing fluoxetine would risk losing his partial response.

(Choice F) Increasing the dose of fluoxetine is not indicated in this patient who is already showing signs of
responding (ie, improved sleep and appetite) and would unnecessarily increase his risk of adverse effects.

Educational objective:
Adequate duration of an antidepressant trial is generally considered to be 6 weeks. Physicians should continue
antidepressants at therapeutic dosages for at least 4-6 weeks before considering the next step in treatment.

Reference
• Systematic review of clinical practice guidelines for failed antidepressant treatment response in major
depressive disorder, dysthymia, and sub threshold depression in adults.
• Serotonergic drugs for depression and beyond.

• When should you move beyond first-line therapy for depression?


Question #306

A 54-year-old woman was diagnosed with breast cancer 6 months ago after seeing her physician for a lump in her
breast. The patient underwent a mastectomy and has been receiving chemotherapy. She has had poor sleep and
decreased appetite for the last 3 weeks. She has some nausea that is relieved by prescription medications. The
patient feels down and says, "The chemo makes me feel weak and drained, so I don't want to do anything." She
spends most of her time at home and does not feel like going out or talking to family and friends. Some days, she
does not get out of bed and does not answer her phone. The patient has lost interest in reading and her favorite
television programs. She feels at fault for having breast cancer because she missed 2 mammogram appointments.
Which of the following is the most appropriate next step in management of this patient?

A) Aggressively manage chemotherapy side effects and monitor the patient

B) Encourage the patient to reach out to her family for support

C) Initiate treatment for major depressive disorder

D) Reassure the patient that it is not her fault and that grief is expected and will improve

E) Refer the patient to a breast cancer support group


Explanation
Correct Answer:

C) Initiate treatment for major depressive disorder

It is normal for patients with cancer to have feelings of sadness, and adverse effects of chemotherapy can include
low energy and decreased appetite. However, recognition of major depression in medically ill patients is
important, as it decreases quality of life and can negatively affect treatment adherence and medical outcome.
This patient's 3-week history of sadness; sleep and appetite disturbance; low energy; decreased interest in friends,
family, and activities; and excessive guilt is consistent with a major depressive episode (≥ 2 weeks of at least 5 of 9
symptoms: Depressed mood plus SIGECAPS [Sleep disturbance, loss of Interest, Guilt, low Energy, impaired
Concentration, change in Appetite, Psychomotor retardation or agitation, and Suicidal thoughts]).

Although several of this patient's symptoms can be explained by both depression and medication adverse effects,
there should be a low threshold for initiating treatment for depression, given the potential to provide relief and
improve quality of life. Patients with cancer can be safely and effectively treated with antidepressant medications
with or without psychotherapy.

(Choice A) Although some of this patient's symptoms (eg, decreased appetite, fatigue) may be attributed to
chemotherapy, she has other symptoms that raise concern for major depression. The patient should be offered
treatment to alleviate her depression.

(Choice B) Although improving communication with family would be beneficial, depression can cause patients to
withdraw from those who care for them the most. Treatment of depression may restore this patient's interest in
family and friends, making communication easier.

(Choice D) Reassurance might be appropriate if this patient had only normal grief or an adjustment disorder. The
development of symptoms many months after the diagnosis and the severity of the symptoms make major
depression more likely. Treatment is indicated.

(Choice E) Cancer support groups are an important adjunctive intervention and should be recommended in
addition to aggressive treatment of the depression. However, this patient struggles to talk with family and friends
and hardly leaves the house. She will likely find it difficult to attend a meeting until her depression has improved.
Recommending a breast cancer support group alone would not be sufficient at this time, given the severity of this
patient's depression and her difficulty communicating with others.

Educational objective:
Patients with cancer may have somatic symptoms that overlap those of depression (eg, sleep disturbance, appetite
change, poor energy). However, if there are additional symptoms such as persistent sadness, guilt, loss of interest,
and suicidal thoughts, major depression should be considered, with a low threshold for beginning treatment.

Reference
• Pharmacological treatment of depression in women with breast cancer: a systematic review.

• Efficacy and acceptability of antidepressants on the continuum of depressive experiences in patients with
cancer: systematic review and meta-analysis.
Question #307

A 59-year-old woman comes to the office to follow up for chronic back pain caused by a motor vehicle collision. A
month ago, the patient was given scheduled-dose oral morphine. Her pain level has decreased from 8 to 3 on a
scale of 0-10, and pain no longer interferes with activities of daily living. There has been no drowsiness,
constipation, or nausea. Other medications include naproxen, acetaminophen, and a lidocaine patch as needed.
Temperature is 37 C (98.6 F), blood pressure is 114/76 mm Hg, pulse is 64/min, and respirations are 12/min. On
examination, the back has greater range of motion than it did during the last visit. Which of the following would be
most helpful in the early detection of prescription opioid misuse in this patient?

A) Assessing back range of motion at each visit

B) Assessing pain level at each visit

C) Monitoring vital signs at each visit

D) Obtaining a scheduled urine drug screening at each visit

E) Querying the prescription drug monitoring database at each visit


Explanation
Correct Answer:

E) Querying the prescription drug monitoring database at each visit

Opioid prescribing best practices

• Screen for risk factors for misuse


• Perform urine drug screening
Prior to
• Query PDMP database
initiation
• Sign controlled substance agreement (eg, 1
prescriber, 1 pharmacy)

• Follow up every 1-3 months


During
• Query PDMP database before each refill
therapy
• Perform random urine drug screenings

PDMP = prescription drug monitoring program.

Opioids account for three-quarters of all fatal prescription drug overdoses; early identification of misuse is critical in
ensuring patient safety. Opioid misuse is defined as taking the medication in a way that deviates from the manner
(eg, overuse, diversion) and purpose (eg, relieve anxiety) for which it was prescribed. Obtaining opioids from
multiple prescribers, taking nonprescribed opioids, and concurrently using recreational drugs or other nonprescribed
controlled substances also constitute misuse.
The prescription drug monitoring program (PDMP) is a helpful tool for detecting misuse. The state-specific
PDMP is a record of prescribed controlled substances that the patient has obtained, including the quantities, dates
the prescriptions were filled, and prescriber information. The PDMP can identify patients who are receiving opioids
from multiple prescribers, suggesting possible overuse or diversion. The database should be queried before
each refill to help ensure appropriate use.

(Choices A and B) Functional capacity (eg, range of motion) and pain level should be assessed at each visit to
determine the efficacy of therapy and to guide dose adjustments when necessary. These evaluations do not
provide information on misuse.

(Choice C) Patient vital signs can demonstrate evidence of acute opioid intoxication (eg, bradypnea, bradycardia,
hypotension) but would not indicate misuse without acute intoxication.

(Choice D) Urine drug screenings can identify diversion (eg, test is negative for opioids), recreational drug use,
and concurrent use of other controlled substances undivulged by the patient. However, they are best done
randomly because scheduled screenings can lead to patient preparation that would misrepresent screening results.

Educational objective:
The prescription drug monitoring program records prescribed controlled substances that the patient received,
including quantities, dates the prescriptions were filled, and prescriber information. This database can identify
patients who are obtaining opioids from multiple prescribers, thereby raising concern for overuse or diversion.

Reference
• Prescription drug monitoring programs, nonmedical use of prescription drugs, and heroin use: evidence from
the National Survey of Drug Use and Health.

• Impact of prescription drug monitoring programs (PDMPs) on opioid utilization among Medicare
beneficiaries in 10 U.S. states.
Question #308

A 27-year-old man comes to the office for postdischarge follow-up of a recent psychiatric hospitalization for a
depressive episode with suicidal ideation. The patient's depressive episode remitted with intensive inpatient
psychotherapy and the initiation of a new medication regimen 1 month ago. The patient has been home for a week
and is feeling well. He has a history of migraine headaches, bipolar disorder, and tobacco use disorder. Family
history is significant for major depressive disorder in his father. Temperature is 37.5 C (99.5 F), blood pressure is
110/75 mm Hg, pulse is 72/min, and respirations are 14/min. Examination is significant for a maculopapular rash
over the patient's left scapula. The patient is unaware of the rash, and discharge paperwork from the previous week
does not document any rash. Which of the following medications is the most likely cause of this patient's condition?

A) Escitalopram

B) Haloperidol

C) Lamotrigine

D) Lurasidone

E) Olanzapine

F) Quetiapine

G) Sertraline
Explanation
Correct Answer:

C) Lamotrigine

The anticonvulsant lamotrigine is used as a mood stabilizer in bipolar disorder and is often used specifically to
target bipolar depression. One of the most significant side effects of lamotrigine is a drug rash, a mild form of
which occurs in up to 10% of those treated. The more severe mucocutaneous rashes of Stevens-Johnson
syndrome (<10% body surface area skin detachment) and toxic epidermal necrolysis (>30% detachment) occur at
a rate of 0.1% (10%-30% detachment is known as Stevens-Johnson syndrome/toxic epidermal necrolysis overlap).

Because of the challenge of predicting the eventual severity of lamotrigine-induced rash at the outset, lamotrigine
should be discontinued at the first sign of rash and substituted with another agent. Most cases of lamotrigine-
induced rash develop within the first 2 months of therapy. Risk of rash is greater in children and appears to be
higher when coprescribed with valproate or when the dose is increased rapidly.

(Choices A and G) Antidepressants such as the selective serotonin reuptake inhibitors escitalopram and sertraline
should be used with caution in bipolar depression, given the limited evidence of benefit and the risk of mood-
switching from depression to mania. Neither escitalopram nor sertraline is associated with a significant risk of rash.

(Choices B, D, E, and F) Haloperidol, lurasidone, olanzapine, and quetiapine are all antipsychotic medications that
may be used in the treatment of bipolar disorder; however, none are associated with a clinically significant risk of
rash.

Educational objective:
A mild rash may develop in up to 10% of those treated with lamotrigine, whereas life-threatening Stevens-Johnson
syndrome or toxic epidermal necrolysis may occur in 0.1%. Any occurrence of rash during the treatment of
lamotrigine requires immediate discontinuation of the drug.

Reference
• Risk of a lamotrigine-related skin rash: current meta-analysis and postmarketing cohort analysis.

• Management of adverse effects of mood stabilizers.


Question #309

An 18-year-old woman comes to the office due to difficulty sleeping and fatigue. The patient started attending
college 2 months ago and lives alone in an apartment near campus. For the past month, she has been increasingly
anxious at night and thinks that someone is going to break into her home despite living on the eighth floor in a safe
neighborhood. The patient makes sure to check that the front and balcony doors are secure every night and locks
her bedroom door before going to bed. She wakes up multiple times a night when her air conditioner turns on or
when she hears street noises and often has difficulty falling back asleep. The patient struggles to stay awake
during class and is worried that her lack of sleep is beginning to affect her grades. She feels tense when walking
alone on campus and spends time with friends only during the day. Vital signs are within normal limits. Physical
examination shows no abnormalities. She appears anxious and reports that her mood has been okay. Which of the
following is the most likely diagnosis?

A) Acute stress disorder

B) Adjustment disorder

C) Generalized anxiety disorder

D) Obsessive-compulsive disorder

E) Paranoid personality disorder


Explanation
Correct Answer:

B) Adjustment disorder

This patient's onset of anxiety and insomnia in response to a major life change (living alone for the first time) is most
consistent with an adjustment disorder. Her insomnia and increased anxiety developed in relation to safety
concerns about being alone and fears of intruders at night. Her fatigue, in turn, negatively impacts her studies.

An adjustment disorder involves emotional or behavioral symptoms (eg, anxiety, depression, disturbance of
conduct) developing within 3 months of an identifiable stressor and lasting no longer than 6 months once the
stressor ceases. Symptoms of adjustment disorder are distressing and impairing but do not meet criteria for
another psychiatric disorder (ie, symptoms are insufficient in number, severity, or duration for another specific
diagnosis). Stressors can be single or multiple; in this case, they are multiple (eg, moving away from home, living
alone for the first time, starting college, feeling academic stress).

(Choice A) Acute stress disorder (ASD) involves exposure to a severe and life-threatening traumatic event,
intrusive reminders of the trauma (eg, flashbacks, nightmares, memories), dissociative symptoms (eg, altered sense
of reality), and/or avoidance behavior. Duration is ≥3 days and <1 month. In adjustment disorder, the stressor can
be of any severity. This patient lacks both the traumatic stressor and intrusive symptoms required for an ASD
diagnosis.

(Choice C) Although this patient's sleep disturbance, fatigue, and tension are also seen in generalized anxiety
disorder (GAD), her monthlong history of symptoms is insufficient for this diagnosis. GAD is characterized by
excessive, uncontrollable worry about multiple issues (eg, school, family, finances, health) for ≥6 months. Patients
typically worry about minor matters and experience a chronic course.

(Choice D) This patient's worrisome thoughts are time consuming and distressing; however, she does not describe
them as intrusive, which would be typical of obsessive-compulsive disorder (OCD). In addition, she does not
display the repetitive, ritualistic compulsive behaviors characteristic of OCD (eg, she checks the doors only once
rather than repeatedly in a compulsive manner).

(Choice E) This patient's new, acute-onset symptoms in response to a stressor cannot be explained by a
personality disorder alone. Paranoid personality disorder involves a lifelong pattern of pervasive distrust and
suspiciousness across a wide range of interpersonal contexts.

Educational objective:
Adjustment disorder is characterized by the development of emotional or behavioral symptoms in response to an
identifiable stressor. The diagnosis is appropriate when the patient does not meet the criteria for another psychiatric
disorder.

Reference
• Adjustment disorder: a diagnosis whose time has come.

• Adjustment disorder: current developments and future directions.


Question #310

A 40-year-old man with a history of hypertension comes to the office for a checkup. The patient has been under a
lot of stress due to work issues and marital problems. He says, "My wife is always criticizing me, and we fight a lot.
Last weekend, she was angry that I didn't make it to my daughter's basketball game. I wanted to go but was too
tired because I had stayed out late with friends the night before. A man has to relax sometimes." The patient
consumes 5 or 6 alcoholic drinks a night and more on weekends. He says that he never gets intoxicated as he is a
"big guy with a high tolerance." Blood pressure is 170/90 mm Hg. Physical examination is normal. Laboratory
evaluation indicates persistent mild elevation of liver function tests. The physician previously explained that the
patient's hypertension is worsened by alcohol use and that the abnormal liver tests are likely due to alcohol use, but
the patient remains unconcerned. Which of the following responses is most appropriate for initiating a discussion of
the patient's alcohol use?

A) "Although you enjoy drinking, do you think it may have played a role in missing your daughter's game?"

B) "Continuing to drink these amounts of alcohol will likely result in liver damage."

C) "I am concerned that it will be difficult to control your blood pressure if you don't decrease your drinking."

D) "I am concerned that you are in denial about your alcohol use."

E) "I recommend trying to reduce your drinking with a long-term goal of complete abstinence."

F) "Is your wife upset that you are drinking too much?"

G) "Would you consider attending an Alcoholics Anonymous meeting?"


Explanation
Correct Answer:

A) "Although you enjoy drinking, do you think it may have played a role in missing your daughter's game?"

All patients should be screened for unhealthy alcohol use, and those identified should receive a brief counseling
intervention geared to the patient's level of awareness of the problem and degree of motivation to change. This
patient has been drinking excessively (≥4 drinks/day is considered problematic in men) with evidence of liver
toxicity, worsening hypertension, and possible negative consequences to his marriage and family.

In counseling this patient, the physician should take into account that the patient does not recognize his drinking as
being problematic and is therefore probably not going to consider changing his behavior (stage of
precontemplation). However, the patient does acknowledge that he would have liked to attend his daughter's
basketball game; having him connect his alcohol use to negative effects on next-day functioning may increase his
motivation to change (motivational interviewing). Once this patient is able to recognize that his use is causing
problems, he will likely be more receptive to a trial of decreasing, or abstaining from, alcohol.

(Choices B and C) It is important to educate patients about the negative medical effects of alcohol. Even though
this patient was educated about his hypertension and laboratory abnormalities, he continues to disregard the
correlation between his health issues and his alcohol consumption. He is more likely to change his behavior if he
can acknowledge the negative effects of his alcohol use on his relationships.

(Choice D) Pointing out that the patient is in denial would likely be perceived as confrontational and judgmental. A
collaborative approach is preferred.

(Choices E and G) This patient has not accepted that he has problems with alcohol and is therefore unlikely to be
receptive to recommendations to change his behavior or for treatment interventions. The first step is to develop his
concern about his alcohol use.

(Choice F) This patient describes a confrontational relationship with his wife. Initiating the discussion by pointing
out his wife's perspective would likely be perceived as taking her side, causing the patient to become defensive and
negatively affecting the physician-patient alliance.

Educational objective:
Effective brief interventions for alcohol abuse require an understanding of the patient's level of awareness of the
problem and readiness to change. When the patient does not acknowledge the problem, the physician's role is to
increase the patient's awareness that alcohol use may be interfering with personal goals.

Reference
• Motivational interviewing with primary care populations: a systematic review and meta-analysis.

• Can motivational interviewing in emergency care reduce alcohol consumption in young people? A
systematic review and meta-analysis.
Question #311

A 30-year-old man comes to the office due to chronic fatigue. When he is not working, the patient is often at home
sleeping; he estimates that he sleeps approximately 12 hours a day. He has felt down since being passed over for
a promotion last month and copes by overeating. The patient says he has never had a lot of energy and can
remember only a few times since high school when he felt happy—a month he spent with his grandparents in
Florida and a brief period when he felt really good after starting his first job. He has worked as a computer
programmer since graduating from college. The patient socializes with friends but avoids dating due to his weight,
saying, "I just know I will be rejected because I'm fat." Apart from obesity, the patient has no other medical
conditions. He smokes marijuana 3 times a week, which he began doing in his mid-20s, to "help me relax and feel
better about myself." He also drinks 4 cups of coffee before going to work on weekdays, which he says helps with
his energy level. There is a family history of bipolar disorder in a maternal uncle. Vital signs and physical
examination are unremarkable. Which of the following is the most likely diagnosis?

A) Adjustment disorder with depressed mood

B) Bipolar II disorder

C) Chronic fatigue syndrome

D) Cyclothymic disorder

E) Hypothyroidism

F) Persistent depressive disorder (dysthymia)

G) Substance-induced depressive disorder


Explanation
Correct Answer:

F) Persistent depressive disorder (dysthymia)

Persistent depressive disorder (dysthymia)

• Chronic depressed mood ≥2 years (1 year in children/adolescents)


• No symptom-free period for >2 months
• Presence of ≥2 of the following:
◦ Poor appetite or overeating
Clinical
◦ Insomnia or hypersomnia
features
◦ Low energy or fatigue
◦ Low self-esteem
◦ Poor concentration or difficulty making decisions
◦ Feelings of hopelessness

• With pure dysthymic syndrome: criteria for major depressive episode never met
• With intermittent major depressive episodes
Specifiers
• With persistent major depressive episodes: criteria for major depressive episode met
throughout previous 2 years

This patient's chronic depression since adolescence, accompanied by overeating, hypersomnia, and low self-
esteem, is consistent with persistent depressive disorder (dysthymia). Diagnosis requires that the patient has
had persistent depressive symptoms for ≥2 years and has never been without depressive symptoms for >2 months
at a time. In DSM-5, persistent depressive disorder also encompasses patients who have met the full criteria for
major depressive disorder at some point during the illness.

(Choice A) Although this patient came to the office after being passed over for a promotion, his depressive
symptoms have been present since adolescence and meet the criteria for persistent depressive disorder.
Adjustment disorders are not diagnosed if the patient meets the criteria for another disorder.

(Choices B and D) This patient's brief periods of feeling better likely represent normal (not elevated) mood, and he
lacks other symptoms of hypomania (eg, inflated self-esteem, increased energy, decreased need for sleep,
pressured speech) required for the diagnosis of bipolar II disorder. Cyclothymic disorder involves ≥2 years of
numerous periods with hypomanic and depressive symptoms that do not meet the full criteria for hypomanic or
major depressive episodes.

(Choices C and E) Although medical conditions should always be considered before making a psychiatric
diagnosis, this patient's long-standing history of chronic sadness, low energy, and low self-esteem are more likely
due to dysthymia. Chronic fatigue syndrome (also referred to as systemic exertion intolerance disease) is
characterized by relatively sudden onset of overwhelming fatigue, often associated with an infection such as
mononucleosis. Hypothyroidism is less likely in this patient who does not report other typical symptoms (eg, cold
intolerance, constipation, dry skin, recent weight gain) and has a normal physical examination.

(Choice G) Although the use of substances such as marijuana and caffeine can affect mood, energy levels, and
sleep patterns, this patient reports long-standing symptoms beginning in high school, prior to the use of substances,
making dysthymia the more likely diagnosis. Patients with mood disorders often self-medicate to manage their
symptoms.

Educational objective:
Persistent depressive disorder (dysthymia) refers to a depressed mood lasting most days for ≥2 years. It includes
patients with pure dysthymia and those with intermittent or persistent major depressive episodes.

Reference
• Impairment in psychosocial functioning associated with dysthymic disorder in the NESARC study.

• Efficacy and acceptability of acute treatments for persistent depressive disorder: a network meta-analysis.
Question #312

A 57-year-old woman comes to the office with intermittent headaches, shoulder and neck pain, fatigue, and
insomnia. The headaches sometimes interfere with the patient's ability to concentrate at work. The symptoms are
not new but have worsened over the past 8 months due to the stress of starting a new job and placing her elderly
mother in an assisted living facility. The patient lies awake at night worrying about her mother, her own health
problems, and the finances of her 2 adult children, who are having difficulty supporting themselves. During the day,
she is tired due to poor sleep and worries about her job performance. The patient has a history of hypertension,
irritable bowel syndrome, and tension headaches. She drinks 1 or 2 glasses of wine before bedtime a few times per
week to help her relax and fall asleep. Temperature is 36.7 C (98 F), blood pressure is 130/80 mm Hg, pulse is 88/
min, and respirations are 16/min. Examination shows sweaty palms and mild, diffuse abdominal tenderness; no
other abnormalities are noted. Complete blood count, chemistry panel, TSH, urinalysis, and ECG are normal.
Which of the following is the most appropriate next step in management of this patient?

A) Begin clonazepam and supportive psychotherapy

B) Begin escitalopram and recommend cognitive-behavioral therapy

C) Obtain noncontrast CT scan of the head and neck

D) Obtain urinary fractionated catecholamines and metanephrines

E) Obtain urinary porphobilinogens

F) Reassure the patient that serious medical illness has been ruled out

G) Schedule monthly visits to monitor symptoms


Explanation
Correct Answer:

B) Begin escitalopram and recommend cognitive-behavioral therapy

Generalized anxiety disorder

• Excessive worry, anxiety (multiple issues) ≥6 months


• Difficult to control
• ≥3 of the following symptoms:
◦ Restlessness or feeling on edge
◦ Fatigue
DSM-5 criteria ◦ Difficulty concentrating
◦ Irritability
◦ Muscle tension
◦ Sleep disturbance
• Significant distress or impairment
• Not due to substances, another mental disorder, or medical condition

• Cognitive-behavioral therapy
Treatment
• SSRIs or SNRIs

SNRI = serotonin-norepinephrine reuptake inhibitor; SSRI = selective serotonin reuptake inhibitor.

This patient's pain, insomnia, fatigue, and poor concentration are consistent with generalized anxiety disorder
(GAD). Although excessive worry about multiple issues is the key symptom in GAD, patients frequently come to
their primary care provider with somatic symptoms, including fatigue and physical symptoms related to muscular
tension (eg, headaches; neck, shoulder, and back pain). Other physical manifestations of anxiety commonly seen
in GAD include trembling, sweating, and gastrointestinal symptoms.

GAD typically has a lifelong, chronic course, with symptom severity increasing during periods of stress, as in this
patient. GAD is effectively treated with a serotonergic antidepressant, cognitive-behavioral therapy (CBT), or a
combination of the two.

(Choice A) Benzodiazepines are generally reserved for patients whose GAD symptoms persist despite treatment
with a serotonergic antidepressant. These drugs are also less preferred in patients who have used alcohol to self-
medicate. Supportive psychotherapy can be helpful but is not as effective as CBT.

(Choice C) An imaging study would be low yield in this patient, who has a known history of tension headaches, a
common physical manifestation of GAD, and no focal neurological signs on examination.

(Choice D) Pheochromocytoma may cause anxiety. However, it is a rare disease that typically presents with
paroxysmal symptoms and would not explain this patient's chronic anxiety about multiple issues.

(Choice E) Acute intermittent porphyria can result in autonomic arousal (eg, tachycardia, hypertension, sweating)
as well as anxiety, but these symptoms tend to occur in acute attacks accompanied by neuropathy (eg, weakness,
numbness) and severe abdominal pain.

(Choices F and G) Reassurance that serious medical illness has been ruled out and regularly scheduled visits are
appropriate interventions for patients with somatic symptom disorder. In contrast to the generalized worries seen in
GAD, patients with somatic symptom disorder have anxiety that specifically revolves around their somatic
symptoms with the fear that they have a serious medical illness. This patient's somatic symptoms are a
manifestation of GAD and should improve with effective treatment.

Educational objective:
Generalized anxiety disorder is characterized by excessive and persistent worry about multiple issues. Patients
frequently come to their primary care provider with insomnia, fatigue, and physical symptoms related to muscular
tension.
Reference
• Diagnosis and management of generalized anxiety disorder and panic disorder in adults.

• The network structure of major depressive disorder, generalized anxiety disorder and somatic
symptomatology.
Question #313

An 89-year-old man is brought to the office by his daughter for a routine visit. The daughter says she is worried
about her father's continued insistence on driving. The patient has had 2 minor accidents in the past 6 months, one
in which he hit a mailbox while backing up in the dark and another in a supermarket parking lot where he backed
into another car that was also backing out. The patient has not gotten lost while driving. Medical history includes
mild cognitive impairment, hypertension, diminished hearing, osteoarthritis, and benign prostatic hyperplasia.
Corrected vision is 20/40 in both eyes. Current medications include lisinopril, hydrochlorothiazide, and as-needed
ibuprofen. He does not drink alcohol. Vital signs are normal. Physical examination shows decreased range of
motion in his neck and mild pain in his hands and knees bilaterally. Mini-Mental State Examination is 23/30, 1 point
lower than his score last year. Which of the following is the most appropriate next step in management of this
patient?

A) Advise patient to limit his driving to familiar, local trips

B) Recommend an on-road driving evaluation

C) Recommend revoking the patient's driver's license

D) Refer for neuropsychological testing

E) Restrict driving to daylight hours


Explanation
Correct Answer:

B) Recommend an on-road driving evaluation

Decisions regarding driving restrictions in elderly patients are complex, requiring a careful evaluation of the
patient's functional capacity to drive and balancing patient autonomy with public safety. Patients often minimize or
are unaware of their deficits and see cessation of driving as a loss of independence and self-esteem. Factors that
can contribute to impaired driving in older patients include medical conditions (eg, cerebrovascular disease,
movement disorders, arthritis, epilepsy), medication effects (eg, benzodiazepines, anticholinergics), delayed
reaction time, hearing and vision loss, and cognitive impairment (eg, impaired spatial processing, executive
dysfunction, impaired judgment).

This patient insists on driving but has several risk factors for impaired driving, including history of recent accidents,
decreased mobility (eg, decreased range of motion in his neck), and mild cognitive impairment. Although degree of
cognitive impairment correlates to impaired driving, mild cognitive impairment is not in and of itself a reason to
restrict driving since these patients may be able to drive safely for some period of time. Because it is difficult to
adequately simulate driving performance in an office visit, the best approach is recommending an on-road driving
evaluation, which can provide an objective assessment of the patient's functional driving ability.

(Choices A and E) Although restricting driving to local trips and daylight hours can be considered, it is unclear
whether this patient's impaired driving is limited to unfamiliar surroundings or nighttime driving. Further evaluation
of his driving ability with an on-road driving test is the appropriate next step.

(Choice C) It would be premature to recommend revoking this patient's license without conducting further
evaluation of his functional driving ability. Legal statutes regarding a physician's authority to intervene in cases of
concern for driving ability vary by state.

(Choice D) Although neuropsychological testing can provide more detailed information regarding cognitive
dysfunction, it would not adequately evaluate the patient's functional driving ability.
Educational objective:
Evaluating older drivers requires a careful evaluation of the patient's functional capacity to drive and balancing
patient autonomy and confidentiality with public safety. In addition to the performance of a history and physical
examination, an on-road driving evaluation is recommended in complex cases.

Reference
• Evaluation of the older driver.
Question #314

A 22-year-old man comes to the office due to difficulty concentrating and sleeping. The patient lost his job and
moved in with his parents 2 months ago. Although he has been looking for work and revising his resume, he gets
distracted and easily loses focus. The patient has not received any interview invitations and is worried that he will
be living with his parents for a long time. He feels that he has "reverted to my high school self," playing video
games most evenings and going out with friends on the weekends. The patient gets annoyed with his parents
occasionally but states that he knows they are "just trying to be helpful." He has been eating more than usual since
moving, gaining 3 kg (6.6 lb) and feeling tired during the day. The patient takes 2-3 hours to fall asleep each night
and frequently checks the time while in bed. He drinks 3 or 4 beers a week and does not use recreational
substances. Vital signs are within normal limits. Physical examination shows no abnormalities. The patient states
that his mood is "okay," and he has a full range of affect. He reports no suicidal ideation. In addition to
recommending psychotherapy, which of the following is the most appropriate pharmacotherapy for this patient?

A) Alprazolam

B) Lithium

C) Methylphenidate

D) Quetiapine

E) Zolpidem
Explanation
Correct Answer:

E) Zolpidem

Adjustment disorder

• Mood & behavioral symptom onset within 3 months of identifiable stressor


Clinical features • Marked distress &/or functional impairment
• Does not meet criteria for another DSM-5 disorder

• Normal stress response


◦ Not excessive, no significant impairment
Differential • Acute stress disorder
diagnosis ◦ Traumatic event, intrusive reexperiencing
• Major depressive disorder
◦ ≥2 weeks, 5/9 symptoms: depressed mood + SIGECAPS

• Psychotherapy is treatment of choice


• Adjunctive pharmacotherapy
Management
◦ Short-term, reserved for rapid relief of impairing symptoms (eg, sleep aid,
anxiolytic)
SIGECAPS = Sleep disturbance, loss of Interest, excessive Guilt, low Energy, impaired Concentration, Appetite
disturbance, Psychomotor agitation/retardation, Suicidal ideation.

This patient's symptoms of insomnia and impaired concentration, developing within 3 months of an identifiable
stressor (eg, job loss, moving back in with his parents), are consistent with an adjustment disorder. Patients with
adjustment disorder experience an excessive response to a stressful life event that impairs daily functioning.
Short-term psychotherapy is the primary form of treatment and focuses on stress reduction, cognitive
reframing, definition of realistic goals, and coping skills enhancement. In general, pharmacotherapy is used as an
adjunct to psychotherapy when more rapid symptom relief is desired.

In this patient, short-term or as-needed treatment with zolpidem can be used as an adjunct to sleep hygiene and
psychotherapy to help obtain the restorative sleep needed to function. Zolpidem, a nonbenzodiazepine
benzodiazepine receptor agonist, has demonstrated efficacy in the treatment of sleep-onset and sleep-maintenance
insomnia.

(Choice A) Alprazolam is a benzodiazepine that can be used for short-term management of generalized anxiety
and panic disorders. Alprazolam has a fast onset and short duration of action; therefore, it is less likely than
zolpidem to improve sleep hygiene. In addition, because of the risk of misuse, it is not recommended in the
treatment of maladjustment to stressful life events.

(Choice B) Lithium is a mood stabilizer that can be used to treat bipolar disorder. Although this patient has
difficulty focusing, he has intermittent irritability and low energy; he is not experiencing the sustained affective
symptoms (eg, persistent irritability, elevated mood) or excessive energy seen during a manic episode.

(Choice C) Methylphenidate is a stimulant used to treat attention deficit hyperactivity disorder, which is
characterized by childhood onset and chronic impairments in attention and concentration. This patient's recent
onset of distractibility and inability to maintain focus following a stressful life event is more consistent with an
adjustment disorder.

(Choice D) Although quetiapine is sedating, an antipsychotic medication would not be appropriate as a sleep aid in
a patient without a comorbid psychotic or mood disorder when safer choices are available.
Educational objective:
Patients with adjustment disorder experience an excessive response to a stressful life event that impairs daily
functioning, including sleep. Short-term pharmacotherapy for insomnia can be used as an adjunct to psychotherapy
when more rapid symptom relief is desired.

Reference
• Zolpidem for insomnia.
Question #315

A 22-year-old man comes to the office due to fatigue, poor appetite, and weakness. The patient recently emigrated
from India and has few social supports. He feels tired throughout the day despite sleeping 8-9 hours every night.
He is worried about his new job and thinks that his coworkers don't like him. The patient reports no significant
medical history but is very worried that he is losing semen when he urinates. Temperature is 36.7 C (98 F), blood
pressure is 110/78 mm Hg, pulse is 62/min, and respirations are 14/min. BMI is 24 kg/m2. Physical examination is
unremarkable. The patient is alert and oriented. His thought process is organized, linear, and coherent. He does
not appear to be responding to internal stimuli. Which of the following would be the most appropriate initial
response to the patient's concerns?

A) Do you ever hear voices when you're alone?

B) Do you think you might be depressed?

C) Have you lost your ability to enjoy your usual interests?

D) I can reassure you that you're not losing semen when you urinate.

E) What do you think is causing your illness?

F) Your physical examination is reassuring, but it may be helpful to meet with a counselor.
Explanation
Correct Answer:

E) What do you think is causing your illness?

This patient's somatic symptoms (eg, fatigue, weight loss), anxiety, cultural background (ie, South Asia), and belief
that he is losing semen during urination are consistent with Dhat syndrome. Dhat syndrome is a culture-bound
syndrome found in South Asian cultures in which sufferers attribute a range of psychological and somatic
complaints to the loss of semen via various methods (eg, loss in urine, excess masturbation). Culture-bound
syndromes are illnesses or explanations of illnesses that are unique to specific cultures. Due to the challenge of
becoming familiar with all the various cultural manifestations of illness, physicians must approach the treatment of
unfamiliar culture-bound syndromes from a patient-centered approach.

Patient-centered cross-cultural care empowers patients to educate the physician on beliefs about their illness.
Physicians facilitate this education by adopting an open and inquisitive communication style that explores the
cultural explanations, manifestations, and consequences of the illness. This patient has provided a description of
his illness but has not been given an opportunity to share his beliefs about its causes. Therefore, the most
appropriate next step would be to ask an open-ended question to explore these beliefs.

(Choices A, B, and C) Although further assessment of specific depressive and psychotic symptoms is necessary,
the most appropriate initial step is to engage the patient by conveying the physician's willingness to understand the
patient's cultural perspective. Subsequently, open-ended questions that ask the patient to describe his experience
would be preferable to close-ended questions that can be answered yes or no.

(Choices D and F) At this point in the physician-patient relationship, reassurance is unlikely to be an effective
intervention, and it may even be perceived as invalidating the patient's concerns. Once a complete patient-centered
evaluation has been completed, it would be appropriate to provide reassurance and discuss treatment options,
including referral to a counselor.

Educational objective:
Dhat syndrome is a culture-bound syndrome of South Asia that manifests with psychological and somatic symptoms
that are attributed to loss of semen. When evaluating culture-bound syndromes, it is important to take a patient-
centered approach and allow the patient to explain the cultural explanations, manifestations, and consequences of
a given syndrome.

Reference
• Relevance of culture-bound syndromes in the 21st century.

• Phenomenology and beliefs of patients with Dhat syndrome: a nationwide multicentric study.
Question #316

A 19-year-old man comes to the office due to overwhelming "stress." The patient recently started working as a
busboy in a restaurant and is afraid that he will be fired because he is often late. He says, "I live close to work, but
the drive takes me a long time—I have to make a lot of stops." Every time the patient drives over a bump in the
road, he is convinced that he has accidentally run over someone and immediately pulls over to check underneath
the car. Even at work, he is unable to focus and must frequently check the news to see if any hit-and-run accidents
have been reported. The patient has no previous psychiatric history, suicide attempts, or violent behavior; family
history is notable for schizophrenia in a maternal uncle. The first-line pharmacotherapy for this patient's current
condition primarily affects which of the following neurotransmitters?

A) Dopamine

B) Gamma-aminobutyric acid

C) Glutamate

D) Norepinephrine

E) Serotonin
Explanation
Correct Answer:

E) Serotonin

Obsessive-compulsive disorder

• Obsessions
◦ Recurrent, intrusive, anxiety-provoking thoughts, urges, or images
• Compulsions
Clinical features
◦ Response to obsessions with repeated behaviors or mental acts
◦ Behaviors not connected realistically with preventing feared event
• Time-consuming (>1 hr/day) or causing significant distress or impairment

• Selective serotonin reuptake inhibitor


Treatment
• Cognitive-behavioral therapy (exposure & response prevention)

This patient's behavior is highly suggestive of obsessive-compulsive disorder (OCD). OCD is characterized by
the presence of persistent intrusive thoughts/urges/images and/or compulsive acts performed in response to
obsessions. Common themes involve fears of contamination, aggressive and sexual impulses, symmetry
obsessions, and fears of harm. Images of horrific scenes (eg, running over a person) or violent urges (eg, to stab
someone) can occur. Related compulsions include cleaning rituals, checking behaviors (eg, this patient checking
underneath his car and then the news), and mental acts performed in response to the obsession (eg, counting,
repeating words silently, praying). Patients with OCD often perform multiple time-consuming rituals and recognize
the irrational nature of their behavior but feel unable to stop and are likely to suffer significant distress and functional
impairment.
Although multiple neurotransmitters and regulatory mechanisms are thought to contribute to the clinical expression
of OCD, serotonin is the primary pharmacological target. First-line treatment is a selective serotonin reuptake
inhibitor antidepressant (eg, fluoxetine, fluvoxamine, paroxetine, citalopram, escitalopram, sertraline). The
serotonergic tricyclic antidepressant clomipramine is generally used second line because it is less well tolerated.

(Choice A) Dopamine is the primary pharmacological target of antipsychotic medications used in the treatment of
psychotic disorders. Antipsychotic monotherapy is not a first-line treatment for OCD. Antipsychotics may be used
as adjunctive medications in patients with inadequate response to a serotonergic antidepressant.

(Choice B) Gamma-aminobutyric acid is the major inhibitory neurotransmitter in the CNS and is the primary target
of benzodiazepines.

(Choice C) Glutamate is the major excitatory neurotransmitter in the CNS and is thought to play a role in many
psychiatric disorders. However, the efficacy of medications targeting glutamate receptors in OCD has not been
established.

(Choice D) Norepinephrine is a catecholamine involved in mood, anxiety, alertness, learning, and memory.
Although drugs that affect both serotonin and norepinephrine have been used in OCD (eg, serotonin-norepinephrine
reuptake inhibitors), the role of serotonin is primary.

Educational objective:
Serotonergic antidepressants (eg, selective serotonin reuptake inhibitors) are the first-line medication treatment for
obsessive-compulsive disorder.

Reference
• Obsessive-compulsive disorder.

• Pharmacological treatment of obsessive-compulsive disorder.


Question #317

A 76-year-old woman comes to the office for a routine checkup. Her medical history is significant for hypertension,
type 2 diabetes mellitus, and hypothyroidism that are treated with oral agents. The patient had a stroke a year ago
and has mild residual weakness in the right arm. Otherwise, she has no physical symptoms but is concerned about
becoming more forgetful, especially because her mother died of Alzheimer disease. She says, "Sometimes I can't
remember people's names, and occasionally during conversations, I can't seem to find the right word." The patient
is widowed and lives alone. She drives herself to the grocery store weekly and manages her own finances. She
describes her mood as "just okay," misses her husband, and regrets that she lives far from her children and
grandchildren. The patient plays cards with her close friends regularly and is looking forward to a trip to visit her
family. Her appetite is good, but she falls asleep earlier in the evening than before, awakening earlier in the
morning and having less energy during the day. Blood pressure is 135/76 mm Hg and pulse is 65/min. Finger-stick
glucose is 94 mg/dL and TSH is 2.4 µU/mL. She scores 26/30 on the Montreal Cognitive Assessment. Which of
the following is the most likely diagnosis in this patient?

A) Alzheimer disease

B) Depression-related cognitive impairment

C) Hypothyroid-induced cognitive impairment

D) Mild cognitive impairment

E) Normal aging

F) Vascular dementia
Explanation
Correct Answer:

E) Normal aging

Cognitive impairment in elderly patients

• Slight decrease in fluid intelligence (ability to process new


Normal aging information quickly)
• Normal functioning in all activities of daily living

Mild neurocognitive disorder • Mild decline in ≥1 cognitive domains


(mild cognitive impairment) • Normal functioning in all activities of daily living with compensation

• Significant decline in ≥1 cognitive domains


Major neurocognitive disorder • Irreversible global cognitive impairment
(dementia) • Marked functional impairment
• Chronic & progressive, months to years

• Reversible mild-moderate cognitive impairment


Major depression • Features of depression (mood, interest, energy)
• Episodic, weeks to months

As patients age, they tend to have a lower energy level and slower processing speed that affects memory and
executive functioning (eg, planning, working memory, mental flexibility) to a small degree. This patient—with well-
controlled diabetes, hypertension, and hypothyroidism and no new focal neurologic deficits (baseline right arm
weakness is due to prior stroke)—likely has normal age-related changes.

Such changes do not impact activities of daily living (ADLs) (eg, self-care, finances, medication management)
and are associated with normal scores on objective measures of cognition (eg, Montreal Cognitive Assessment
[MoCA] score ≥26/30). Patients can have occasional word-finding difficulty (expressive aphasia) or mild
memory loss but are able to provide details about incidents of forgetfulness. Changes in sleep pattern, including
lighter sleep and phase advance (ie, falling asleep earlier in the evening and awakening earlier), are also common.

By contrast, with mild cognitive impairment (MCI), there is a objective evidence of impairment (eg, MoCA score
<26/30) and modest but notable decline in cognitive function (eg, complex attention, executive function, learning
and memory, language). Although independence is generally preserved, ADLs take greater effort, time, and/or
compensatory strategies.

This patient does not demonstrate impairment in ADLs, does not need to compensate to function, and shows no
objective evidence of impairment on neuropsychological screening (Choice D).

(Choices A and F) Dementia (also known as major neurocognitive disorder) may be differentiated from MCI by the
severity of cognitive dysfunction as well as by the functional impairment of ADLs. Alzheimer disease presents with
insidious onset and gradual progression of memory impairment (particularly memory for recent events or of specific
events at specific times). Vascular dementia manifests with prominent executive dysfunction and mild memory
impairment, focal neurologic findings, and signs of cerebrovascular disease on brain imaging.

(Choice B) Depression is a common cause of reversible cognitive impairment in the elderly and is sometimes
referred to as pseudodementia. This patient's lack of core depression symptoms (dysphoria or anhedonia) makes
pseudodementia an unlikely explanation for her symptoms.

(Choice C) Overt hypothyroidism (ie, elevated TSH with low free T4) may be associated with a reversible
hypothyroid-induced cognitive impairment characterized by deficits in short-term memory and concentration. This
patient has no evidence of hypothyroidism on laboratory testing.

Educational objective:
Normal age-related cognitive changes include occasional forgetfulness, word-finding difficulty, and sleep pattern
changes that do not impact activities of daily living.

Reference
• Normal cognitive aging.

• What is normal in normal aging? Effects of aging, amyloid and Alzheimer's disease on the cerebral cortex
and the hippocampus.
Question #318

A 74-year-old man comes to the office for a scheduled follow-up after hospitalization for a myocardial infarction 4
weeks ago. The patient has had no new cardiac symptoms and has adhered to his outpatient medication regimen.
When asked how he has been, he says, "I just don't feel like my old self." The patient's usual routine has been
"disrupted" since he was discharged; he is not meeting with friends and has not been able to read the newspaper all
the way through. He naps on his couch during the day and wakes up earlier than he wants to in the morning. The
patient's medical history includes diet-controlled diabetes mellitus, hypertension, and obesity. Medications include
aspirin, metoprolol, atorvastatin, lisinopril, and sublingual nitroglycerin as needed. Vital signs are within normal
limits. Physical examination is unremarkable except for moderate obesity. He reports low mood and appears
slowed and subdued. The patient declines psychotherapy. Which of the following is the most appropriate
pharmacotherapy?

A) Citalopram

B) Desipramine

C) Mirtazapine

D) Phenelzine

E) Sertraline

F) Trazodone

G) Venlafaxine
Explanation
Correct Answer:

E) Sertraline

Major depressive disorder

• ≥5 of the following symptoms lasting ≥2 weeks (at least one symptom must be
either depressed mood or loss of interest/pleasure):
◦ Depressed mood
◦ Loss of interest or pleasure
◦ Change in appetite or weight
◦ Insomnia or hypersomnia
Diagnosis ◦ Psychomotor retardation or agitation
◦ Low energy
◦ Poor concentration or indecisiveness
◦ Thoughts of worthlessness or inappropriate guilt
◦ Recurrent thoughts of death or suicide
• No history of mania or hypomania
• Not due to substances or another medical condition

The patient exhibits symptoms of major depressive disorder (MDD), which include low mood, anhedonia,
impaired concentration, sleep disturbance, and psychomotor retardation. MDD may occur in up to one-fifth of
patients who have had a recent myocardial infarction (MI). It may affect adherence to post-MI medical treatments
and can increase the risk of cardiac mortality. Treatment options include psychotherapy and antidepressant
medication. Combination therapy is likely to be more effective and is therefore recommended.
Selective serotonin reuptake inhibitors (SSRIs) are used as first-line treatments after an acute MI because they
are generally well tolerated and, compared with other classes of antidepressants, less likely to cause adverse
cardiac effects. Among SSRIs, sertraline is preferred because it carries a very low risk of adverse drug
interactions, especially with cardiac medications. The SSRI citalopram is generally avoided in patients with a recent
MI due to its potential for dose-dependent QT prolongation (Choice A).

(Choice B) Tricyclic antidepressants such as desipramine can slow cardiac conduction by inhibition of fast sodium
channels and therefore are not indicated in patients with severe cardiovascular disease.

(Choice C) Although mirtazapine, an atypical antidepressant with noradrenergic and serotonergic effects, is not
contraindicated in patients with cardiovascular disease, its propensity to cause sedation and weight gain make it a
poor choice for this patient.

(Choice D) Monoamine oxidase inhibitors such as phenelzine are not considered first-line treatment for MDD due
to their side effect burden (eg, sedation, weight gain) and potential for drug and food interactions (eg, serotonin
syndrome, tyramine hypertensive crisis).

(Choice F) Trazodone, a serotonin-modulating antidepressant, is extremely sedating. It is used at a low dose


primarily for sleep induction rather than for its antidepressant effects. Its orthostatic effects are problematic in the
elderly.

(Choice G) Venlafaxine, a serotonin-norepinephrine reuptake inhibitor, is associated with tachycardia and


increased blood pressure; therefore, it is not recommended in a patient with a recent MI.

Educational objective:
Depression after myocardial infarction can increase the overall risk of cardiac mortality. Psychotherapy and
antidepressant treatment (especially in combination) are likely to be effective. The selective serotonin reuptake
inhibitor sertraline is considered a first-line treatment.

Reference
• Treatment of affective disorders in cardiac disease.
• Treatment of depression in cardiovascular disease.

• QT prolongation, torsades de pointes, and psychotropic medications: a 5-year update.


Question #319

A 33-year-old woman comes to the office for "help losing weight." She says, "I'm really trying, but I haven't seen
any results." The patient has been on restrictive diets that cut out various food groups but gained 5 pounds in the
past month. She feels "defeated" and says, "Nothing I try works. I end up feeling sorry for myself, and before I
know it, I've eaten an entire cake. Even though I feel disgusting afterward, I still do it a few times a week." She
reports no difficulty with sleep, energy, or concentration. The patient has been avoiding meals with friends and
family because of embarrassment about overeating. Medical history is insignificant. Mental status examination
reveals an overweight woman wearing loose clothing. She is cooperative and has a reactive affect. Treatment
options are discussed, and the patient prefers to start medication. Which of the following medications would be
most appropriate for this patient?

A) Bupropion

B) Lamotrigine

C) Orlistat

D) Phentermine

E) Sertraline
Explanation
Correct Answer:

E) Sertraline

Eating disorders

Diagnosis Clinical features Treatment

• Cognitive-behavioral therapy
• BMI <18.5 kg/m2
Anorexia • Nutritional rehabilitation
• Intense fear of weight gain
nervosa • Olanzapine if no response to
• Distorted views of body weight & shape
above

• Recurrent episodes of binge eating • Cognitive-behavioral therapy


Bulimia • Binge eating & inappropriate compensatory • Nutritional rehabilitation
nervosa behavior to prevent weight gain • SSRI (fluoxetine), often in
• Excess worrying about body shape & weight combination with above

• Cognitive-behavioral therapy
• Recurrent episodes of binge eating
Binge-eating • Behavioral weight loss therapy
• No inappropriate compensatory behaviors
disorder • SSRI
• Lack of control during eating
• Lisdexamfetamine
SSRI = selective serotonin reuptake inhibitor.

This patient's repeated episodes of eating an excessive quantity of food in a discrete period, associated with loss
of control, physical discomfort, and distress, are consistent with binge eating disorder (BED). The most
prevalent eating disorder in the United States, BED has negative repercussions for physical health, including
increased risk for obesity, diabetes, hyperlipidemia, and hypertension, as well as comorbid psychiatric disorders
such as depression, anxiety, and illicit drug use. It can be differentiated from bulimia nervosa by the absence of
inappropriate compensatory behaviors (eg, vomiting, excessive exercise, starvation). Although patients with BED
are more likely to be overweight or obese, it also occurs in those of normal weight.

The mainstay of treatment for BED is psychotherapy. Cognitive-behavioral therapy has the most evidence of
benefit. Medications can be considered in patients who do not have access to psychotherapy, prefer medications,
or fail psychotherapy alone. Options include selective serotonin reuptake inhibitors such as sertraline, the
stimulant lisdexamfetamine, and the antiepileptic agent topiramate.

(Choice A) The antidepressant bupropion has the advantage of not causing weight gain and may be associated
with weight loss in some patients. However, it lacks evidence for use in BED. SSRIs are considered first-line
pharmacotherapy for BED.

(Choice B) The antiepileptic agent topiramate has shown benefit in the treatment of BED. However, there is lack
of evidence to recommend the use of lamotrigine.

(Choice C) The antiobesity agent orlistat has not shown benefit in decreasing binge eating episodes and is not
recommended for use in BED.

(Choice D) The sympathomimetic weight loss agent, phentermine, is not recommended for use in BED because of
a lack of evidence of efficacy and the potential for adverse effects, which include hypertension, tachycardia, and
cardiac ischemia.

Educational objective:
Binge eating disorder is characterized by repeated episodes of excessive eating accompanied by a loss of control
and subsequent distress. The most effective treatment is psychotherapy. Medication options include selective
serotonin reuptake inhibitors (eg, sertraline), lisdexamfetamine, and topiramate.

Reference
• Binge eating disorder.

• Psychological, pharmacological, and combined treatments for binge eating disorder: a systematic review
and meta-analysis.

• Pharmacological treatment of binge eating disorder: update review and synthesis.


Question #320

A 55-year-old woman comes to the office due to insomnia and fatigue that began shortly after her divorce was
finalized a year ago. The patient used to sleep 7-8 hours without difficulty; however, over the past year she has had
increasing difficulty falling asleep and started drinking 2-3 glasses of wine before bedtime to help. Despite falling
asleep more quickly, she has recently started to regularly wake up around 3:00 AM. Her symptoms have
progressively worsened and she now lies awake for several hours in the middle of the night, thinking and worrying
about her future, but she rarely experiences anxiety during the day. She reports mild difficulty with concentration
and no change in appetite. The patient experiences occasional brief sadness and loneliness. She has recently
started showing up late for work and has stopped attending her weekly exercise class. Medical history includes
hypothyroidism, treated with levothyroxine, and gastroesophageal reflux disease, diagnosed 3 months ago and
treated with famotidine. Temperature is 37.2 C (99 F), blood pressure is 140/90 mm Hg, pulse is 90/min, and
respirations are 12/min. Physical examination shows a mild tremor but no other abnormalities. Laboratory results
are as follows:

Complete blood count


Hemoglobin 11.4 g/dL
Hematocrit 34%
Mean corpuscular volume 106 µm3
Platelets 150,000/mm3
Leukocytes 7,500/mm3

Serum chemistry
Sodium 142 mEq/L
Potassium 3.9 mEq/L
Chloride 102 mEq/L
Bicarbonate 26 mEq/L
Blood urea nitrogen 20 mg/dL
Creatinine 0.8 mg/dL
Glucose 100 mg/dL

Liver function studies


Total protein 6.5 g/dL
Total bilirubin 0.7 mg/dL
Alkaline phosphatase 78 U/L
Aspartate aminotransferase (SGOT) 85 U/L
Alanine aminotransferase (SGPT) 42 U/L

Endocrine
TSH 2.7 µU/mL

Which of the following is the most likely diagnosis in this patient?

A) Adjustment disorder

B) Alcohol use disorder

C) Anxiety disorder due to another medical condition

D) Generalized anxiety disorder

E) Insomnia disorder

F) Persistent depressive disorder (dysthymia)


Explanation
Correct Answer:

B) Alcohol use disorder

The National Institute on Alcohol Abuse and Alcoholism has found evidence of negative health effects for women of
all ages and men age ≥65 who consume >7 drinks in a week or >3 in a day (for men age <65, the cutoffs are >14
drinks in a week or >4 drinks in a day). This patient's alcohol consumption pattern (≥14 drinks per week), abnormal
liver enzymes (aspartate aminotransferase/alanine aminotransferase ratio ≥2:1), macrocytosis, alcohol
tolerance (ie, decreasing sedative effects of alcohol over time), and impaired functioning (eg, work tardiness) are
consistent with a diagnosis of alcohol use disorder. This diagnosis is also supported by her history of
gastroesophageal reflux disease and hypertension as well as a tremor on examination.

Individuals with alcohol use disorder frequently seek primary care due to sleep disturbance and/or anxiety
symptoms from mild withdrawal, as in this patient. These individuals may use alcohol to fall asleep, but as the
blood alcohol level drops, CNS hyperarousal occurs and results in awakenings.

(Choice A) Although this patient's alcohol use may have initially escalated in response to her divorce a year ago,
adjustment disorder cannot be diagnosed in a patient whose symptoms have lasted >6 months after a stressor.

(Choice C) There is insufficient evidence that this patient's insomnia and anxiety are a direct result of the
physiological effects of a medical condition. Her normal TSH level indicates that her hypothyroidism is well
controlled.

(Choice D) Generalized anxiety disorder is characterized by pervasive and excessive worry about multiple issues
and is typically seen in patients who report lifelong anxiety. It is less likely in this patient with signs/symptoms of
alcohol use disorder, no history of psychiatric illness, and circumscribed anxiety (ie, only worries at night when
unable to sleep).

(Choice E) Although this patient may have initially developed difficulty falling asleep in the context of a divorce, it
developed into a classic pattern of alcohol-induced sleep disruption with the onset of heavy drinking. Insomnia
disorder is diagnosed only when the sleep disturbance is not better explained by another disorder.

(Choice F) A diagnosis of dysthymia would require a depressed mood for at least 2 years and substance-induced
causes to be ruled out.

Educational objective:
Patients with unhealthy alcohol use are frequently seen in the primary care setting for insomnia and/or anxiety.
These patients should be carefully screened for alcohol use disorder. Findings of increased liver enzymes and
macrocytosis can also assist in screening.

Reference
• Screening for substance abuse in women's health: a public health imperative.

• Screening for alcohol and drug use disorders among adults in primary care: a review.

• Alcohol dependence and its relationship with insomnia and other sleep disorders.
Question #321

A 58-year-old man with a 45-pack-year smoking history comes to the clinic due to a persistent cough. The cough
started about 3 months ago and is distinctly worse than his usual smoker's cough. During this period, he had
episodes of hemoptysis and lost 9.1 kg (20 lb). He has had no nausea, vomiting, fever, chills, night sweats,
pruritus, seizures, or gait abnormalities. Temperature is 36.9 C (98.4 F), blood pressure is 134/85 mm Hg without
an orthostatic drop, and pulse is 76/min. Temporal wasting is evident. The mucous membranes are moist, and the
neck is supple without lymphadenopathy. Heart and breath sounds are normal. The abdomen is soft and
nontender. No peripheral edema is present. The patient is oriented to time, place, and person. Pupils are equal
and reactive, and no nystagmus is elicited. Neurologic examination, including gait, is normal. Laboratory results
are as follows:

Sodium 124 mEq/L


Potassium 4.0 mEq/L
Chloride 96 mEq/L
Bicarbonate 24 mEq/L
Blood urea nitrogen 11 mg/dL
Creatinine 0.9 mg/dL
Glucose 91 mg/dL
Calcium 8.9 mg/dL
Albumin 2.9 mg/dL

Which of the following is the best initial treatment for this patient?

A) Albumin infusion

B) Demeclocycline
C) Fluid restriction

D) Hypertonic saline

E) Lactated Ringer solution

F) No additional intervention

G) Normal saline
Explanation
Correct Answer:

C) Fluid restriction

Syndrome of inappropriate antidiuretic hormone

• CNS disturbance (eg, stroke, hemorrhage, trauma)


• Medications (eg, carbamazepine, SSRIs, NSAIDs)
Etiologies • Lung disease (eg, pneumonia)
• Ectopic ADH secretion (eg, small cell lung cancer)
• Pain &/or nausea

• Mild/moderate hyponatremia: nausea, forgetfulness


Clinical features • Severe hyponatremia: seizures, coma
• Euvolemia (eg, moist mucous membranes, no edema, no JVD)

• Hyponatremia
• Serum osmolality <275 mOsm/kg H2O (hypotonic)
Laboratory findings
• Urine osmolality >100 mOsm/kg H2O
• Urine sodium >40 mEq/L

Management • Fluid restriction ± salt tablets


• Hypertonic (3%) saline for severe hyponatremia

ADH = antidiuretic hormone; JVD = jugular venous distension; NSAIDs = nonsteroidal anti-inflammatory drugs;
SSRIs = selective serotonin reuptake inhibitors.

This patient, who has a history of extensive smoking, cough, hemoptysis, weight loss, and temporal wasting, likely
has lung cancer. His laboratory results are remarkable for hyponatremia (serum sodium <135 mEq/L). One
approach to determining the etiology of hyponatremia is an assessment of the patient's volume status. Evidence
of volume overload (eg, peripheral edema, jugular venous distension) is consistent with hypervolemic
hyponatremia, which occurs in heart failure, renal failure, and liver cirrhosis. Evidence of volume depletion (eg, dry
mucous membranes) suggests hypovolemic hyponatremia, which occurs in patients with dehydration. This patient
appears clinically euvolemic (eg, moist mucous membranes, absence of peripheral edema), which is characteristic
of hyponatremia due to the syndrome of inappropriate antidiuretic hormone secretion (SIADH).

Small cell lung cancer is strongly associated with smoking and is the most common cause of SIADH due to
ectopic release of antidiuretic hormone. This patient requires workup for probable small cell lung cancer. In the
meantime, fluid restriction (<800 mL/day) is the best initial treatment for patients with asymptomatic or mildly
symptomatic (eg, lethargy, nausea, forgetfulness) hyponatremia due to SIADH. Salt tablets are sometimes given as
well.

(Choice A) Intravenous albumin may provide some benefit in the management of hypervolemic hyponatremia in
patients with cirrhosis. However, it is not useful in the management of SIADH, even in those who are
hypoalbuminemic, such as this patient.

(Choice B) Demeclocycline acts at the level of the renal collecting tubule to decrease responsiveness to
antidiuretic hormone. It is sometimes used in the treatment of SIADH; however, it can be nephrotoxic and should
be considered only if fluid restriction and high salt intake fail to correct the hyponatremia.

(Choice D) Use of hypertonic saline can be dangerous due to the risk of sodium overcorrection; it should be
reserved for severe symptomatic hyponatremia (eg, seizures, coma, profound confusion) or cases of hyponatremia
that are resistant to other treatment.

(Choices E and G) Both lactated Ringer solution and normal saline are forms of isotonic fluid. Infusion of isotonic
fluid is appropriate for hypovolemic patients; however, it often results in net free water retention and worsening of
hyponatremia in those with SIADH.

(Choice F) Further intervention is required as this patient's hyponatremia is likely to worsen without fluid restriction.

Educational objective:
Hyponatremia due to the syndrome of inappropriate antidiuretic hormone secretion is a common complication of
small cell lung cancer. Fluid restriction is the initial treatment of choice in asymptomatic or mildly symptomatic
patients.

Reference
• Diagnosing and treating the syndrome of inappropriate antidiuretic hormone secretion.

• Paraneoplastic syndromes associated with lung cancer.


Question #322

A 67-year-old man comes to the office due to slowly worsening fatigue and shortness of breath. The symptoms
began after a chest cold 8 months ago. Medical history is significant for type 2 diabetes mellitus, for which the
patient takes metformin. He has a 25-pack-year smoking history and quit 2 years ago. The patient is a retired
attorney. He started taking trumpet lessons 3 months ago. Temperature is 36.6 C (97.9 F), blood pressure is 122/
80 mm Hg, pulse is 70/min, and respirations are 16/min. Oxygen saturation is 96% on room air. Heart sounds are
normal. Pulmonary examination demonstrates fine inspiratory crackles. Muscle strength is 5/5 throughout. The
joints appear normal with full range of motion. Skin examination is normal. A high-resolution CT scan of the chest
reveals lower lobe subpleural honeycombing and reticular markings greatest at the periphery. Pulmonary function
tests demonstrate moderate restriction and reduced diffusion lung capacity for carbon monoxide. Hemoglobin A1c
is 8.2%. All other laboratory test results are within normal limits. What is the most appropriate therapy for this
patient?

A) Antifibrotic therapy

B) Corticosteroid-based immunosuppression

C) Exposure avoidance

D) Noncorticosteroid-based immunosuppression

E) Referral for hospice care


Explanation
Correct Answer:

A) Antifibrotic therapy

Idiopathic pulmonary fibrosis

• Microscopic alveolar epithelial injury (smoking, GERD, silent aspiration)


Pathophysiology • Inappropriate repair by fibrosis instead of epithelial regeneration
& risk factors • Risk factors/demographics: male, age >60, smoking, polymorphisms in genes
regulating epithelial regeneration & health

• Chronic progressive dyspnea, nonproductive cough, fatigue


Clinical
• Fine "Velcro" inspiratory crackles, digital clubbing
presentation
• HRCT: UIP (subpleural honeycombing, reticular opacities); PFTs: restrictive pattern

• Antifibrotic therapy (eg, pirfenidone, nintedanib)


• Smoking cessation & treatment of GERD
Management
• Supplemental oxygen & pulmonary rehabilitation
• Lung transplantation

GERD = gastroesophageal reflux disease; HRCT = high-resolution computed tomography; PFT = pulmonary
function test; UIP = usual interstitial pneumonia.
This patient with dyspnea and inspiratory crackles has pulmonary restriction and a high-resolution CT (HRCT) of the
chest showing a pattern of subpleural honeycombing and reticular opacities (ie, usual interstitial pneumonia [UIP]).
There is no evidence for underlying connective tissue disease (ie, normal skin and joint examinations) or history of
relevant exposure (eg, asbestos, pneumotoxic drugs). This presentation is most consistent with idiopathic
pulmonary fibrosis (IPF), diagnosed when UIP is present on HRCT and secondary causes of UIP (eg,
autoimmune disease, exposures) have been excluded. IPF can be a devastating diagnosis; average survival is 2-3
years.

Central to IPF pathogenesis is alveolar epithelial injury that is inappropriately repaired by fibrosis (scarring).
Antifibrotic therapy has revolutionized IPF treatment. These small-molecule inhibitors block profibrotic growth
factor signaling (eg, transforming growth factor β), reducing fibroblast proliferation and collagen production. They
are approved for IPF patients with diminished lung function, such as reduced forced vital capacity (FVC).
Antifibrotic drugs slow the rate of FVC decline but cannot restore lost lung function. Candidates for treatment
should be identified early and monitored closely for progression.

(Choices B and D) Because low levels of lung inflammation are common in IPF, pharmacologic
immunosuppression was once regarded as a promising treatment strategy. However, inflammatory responses are
likely reactive to the primary epithelial injury and fibrosis. Clinical trials of immunosuppression (eg, prednisone
[corticosteroid], azathioprine [noncorticosteroid]) demonstrated harm, including increased mortality. If lung fibrosis
is suspected to be driven by a systemic autoimmune disease (eg, scleroderma-associated UIP),
immunosuppression is considered on an individual basis. Forms of ILD that tend to be corticosteroid-responsive
include cryptogenic organizing pneumonia (eg, groundglass/consolidative opacities on HRCT, alveolar granulation
tissue on histopathology).

(Choice C) Exposure to organic antigens can cause hypersensitivity pneumonitis (HP). Acute HP presents with
short-lived episodes following exposure. Chronic HP can develop with continued exposure causing fibrotic
evolution; it can be seen in wind instrument (eg, bagpipe, rather than trumpet) players due to mold overgrowth.
However, the timeline of this patient's symptoms (8 months) does not correspond to exposure (started lessons 3
months ago), and the radiographic pattern (ie, lower lobe honeycombing, peripheral reticular markings) revealed by
HRCT scanning is specific for UIP rather than HP.

(Choice E) Although IPF survival is limited, antifibrotic therapy should be considered, especially in patients with
good functional status who may also be eligible for lung transplantation. Hospice care may become appropriate
when life expectancy is ≤6 months.

Educational objective:
Idiopathic pulmonary fibrosis is a disorder of epithelial injury that has been pathologically repaired by fibrotic
responses. Antifibrotic drugs inhibit fibroblasts to slow the rate of lung function decline and are first-line therapy.
Question #323

A 38-year-old man comes to the office due to progressive shortness of breath and cough with mucoid sputum over
the past 6 months. His shortness of breath is worse with exertion. The patient has no chest pain, weight loss, or
night sweats. Past medical history is otherwise unremarkable, and he takes no medications. He smoked cigarettes
for 5 years but quit 13 years ago. The patient does not use alcohol or illicit drugs. Temperature is 36.7 C (98 F),
blood pressure is 128/78 mm Hg, pulse is 78/min, and respirations are 16/min. His pulse oximetry is 96% on room
air. BMI is 32 kg/m2. Breath sounds are decreased at the bases, and there are no crackles or wheezes. Heart
sounds are normal. There is no jugular venous distension or peripheral edema. Chest imaging reveals bilateral
basilar hyperlucency. Complete blood count and basic metabolic panel are normal. Which of the following is the
most appropriate next step in management?

A) In-laboratory polysomnography

B) Methacholine challenge test

C) Transthoracic echocardiogram

D) Serum alpha-1 antitrypsin level

E) Sweat chloride testing

F) Video-assisted lung biopsy


Explanation
Correct Answer:

D) Serum alpha-1 antitrypsin level

This patient with chronic shortness of breath, productive cough, and evidence of destruction of the lower lung lobes
likely has alpha-1 antitrypsin (AAT) deficiency. AAT deficiency-related lung disease presents similarly to other
forms of chronic obstructive pulmonary disease (COPD) but results in panacinar (panlobular) emphysema.
Smoking-induced centriacinar (centrilobular) emphysema most commonly causes disease in the upper lobes of the
lungs, whereas the panacinar emphysema of AAT deficiency classically results in greater destruction of the lower
lobes. On average, smokers present in their 30s, whereas nonsmokers present in their 40s. AAT deficiency is
frequently associated with liver disease, most commonly resulting in neonatal hepatitis, cirrhosis, or hepatocellular
carcinoma.

AAT deficiency should be considered in a number of situations, including in patients with:

• COPD at a young age (age ≤45)


• COPD with minimal or no smoking history
• basilar-predominant COPD
• history of unexplained liver disease

Diagnosis is confirmed by measuring serum AAT levels, and pulmonary function testing should also be performed.
Treatment includes intravenous supplementation with pooled human AAT.

(Choice A) Sleep disordered breathing due to obesity such as obstructive sleep apnea (confirmed by
polysomnography) more commonly presents with daytime sleepiness, impaired concentration, and morning
headaches.

(Choice B) Bronchoprovocation testing with methacholine can be used to confirm asthma diagnosis in selected
groups of patients such as those with typical asthma symptoms but normal spirometry. Hyperinflation may be seen
on chest x-ray, but basilar hyperlucency is unlikely.
(Choice C) An echocardiogram is very useful to evaluate cardiac sources of dyspnea. However, this patient's
pulmonary findings along with the lack of jugular venous pressure elevation and peripheral edema make a cardiac
cause less likely.

(Choice E) Sweat chloride testing is used to confirm the diagnosis of cystic fibrosis (CF); however, adult
presentations of CF are more commonly characterized by gastrointestinal, endocrine, and fertility problems. The
absence of bronchiectasis on lung imaging also makes CF less likely.

(Choice F) Lung biopsy is an invasive procedure with associated morbidity. Measurement of the serum AAT level
is less invasive, and low values are highly diagnostic for AAT deficiency.

Educational objective:
Alpha-1 antitrypsin (AAT) deficiency should be considered in patients who lack typical risk factors for chronic
obstructive pulmonary disease (eg, age <45) or those with atypical features (eg, basilar-predominant disease).
Diagnosis is confirmed by measuring serum AAT levels.

Reference
• Diagnosis and management of patients with a1-antitrypsin (A1AT) deficiency.

• Alpha-1 antitrypsin deficiency.


Question #324

A 19-year-old woman comes to the office for an athletic fitness evaluation. The patient is a competitive sprinter.
Over the past few weeks, she has had dyspnea, cough, and chest tightness that begin several minutes after she
starts running. These symptoms occur only during very intense training sessions, approximately once per week;
they have begun to negatively affect her race performance. She otherwise feels well and has no nocturnal
symptoms. The patient takes intranasal fluticasone for allergic rhinitis. She transferred schools from across the
country 3 months ago. The patient does not use tobacco, alcohol, or recreational drugs. Vital signs are within
normal limits. The head and neck are normal on physical examination, and chest auscultation is unremarkable.
Baseline spirometry is normal. However, FEV1 decreases by 25% after 10 minutes of treadmill running. Which of
the following is the most appropriate treatment strategy for this patient's condition?

A) Inhaled corticosteroid and beta agonist 10 minutes before exercise

B) Inhaled corticosteroid and beta agonist at onset of symptoms

C) Inhaled long-acting muscarinic antagonist daily

D) Inhaled short-acting muscarinic antagonist 10 minutes before exercise

E) Oral leukotriene receptor antagonist daily


Explanation
Correct Answer:

A) Inhaled corticosteroid and beta agonist 10 minutes before exercise

Exercise-induced bronchoconstriction

• Hyperventilation → incomplete heating & humidification of inspired air → cooler, dry air
Pathophysiology triggers mast cell degranulation & bronchospasm
• Can occur in isolation or in underlying asthma

• ↓ Exercise tolerance; asthma symptoms appear within 5-10 min & improve* after 20 min
of exercise
Clinical features • Diagnosis:
& diagnosis ◦ Supportive: empiric response to preexercise bronchodilator
◦ Confirmatory: bronchoprovocation testing (eg, spirometry before & after
exercise) showing ≥15% decline in FEV1

• Improve control of underlying asthma (ie, step-up therapy)


• Premedicate before exercise: ICS-formoterol (10 min prior) preferred over SABA, LTRA
Management
(2 hr prior)
• Daily ICS-beta agonist or LTRA may be needed for frequent, prolonged exercise
*Refractory period (↓ preformed mast cell mediators & ↑ inhibitory prostaglandins).

ICS = inhaled corticosteroid; LTRA = leukotriene receptor antagonist; SABA = short-acting beta agonist.

This patient has exercise-induced respiratory symptoms causing decreased athletic performance. Symptoms are
exclusive to exercise but absent at other times of day. Bronchoprovocation testing reveals an exercise-induced
decline in expiratory airflow of 25% (normal: <10%). Taken together, this presentation is consistent with exercise-
induced bronchoconstriction (EIB).

During exercise, minute ventilation increases. Rapid movement of large volumes of air overwhelms the normal
heating and humidification capacity of the upper airway (eg, mouth instead of nose breathing). Therefore, the air
entering the lower respiratory tract is relatively cool and dry. This disrupts the airway surface lining fluid to trigger
mast cell degranulation, leading to bronchospasm.

In addition to these thermophysical effects on the airway surface, a role for leukotrienes and T-helper cell
2–predominant inflammation has been identified in patients with recurrent episodes of EIB. Therefore, EIB is often
considered to be a subtype of intermittent asthma, and premedication is recommended with an inhaled
corticosteroid (ICS) and beta agonist (eg, budesonide-formoterol).

(Choice B) In EIB, bronchoconstriction typically occurs 5-10 minutes after the onset of exercise, reflecting the time
required to cool and dehydrate the airway lining sufficiently to cause mast cell mediator release. Because fast-
onset bronchodilators require several minutes to take effect, premedication is recommended 10 minutes prior to
exercise instead of at onset of symptoms, with the goal of preventing EIB entirely.

(Choice C) Long-acting muscarinic antagonists (eg, tiotropium) are used as add-on controller therapy for persistent
asthma that remains uncontrolled despite maximum doses of daily ICS-formoterol and leukotriene receptor
antagonists (LTRAs). They have no established role for EIB.

(Choice D) Short-acting muscarinic antagonists (eg, ipratropium) are typically not used for quick relief because
their effects are more delayed and they are less effective than other agents (eg, beta agonists). They are
sometimes used for EIB caused by extremely cold, dry air (eg, during alpine skiing or ice hockey), possibly due to
their central role for cholinergic bronchospasm in these environments.
(Choice E) LTRAs (eg, montelukast) can be taken 2 hours prior to exercise on an as-needed basis as an alternate
to ICS-formoterol. However, daily treatment is not indicated in this patient who has symptoms sporadically (eg,
once a week during very intense workouts) and can be treated with prophylaxis as needed rather than on a
scheduled basis. Daily LTRAs can be considered in athletes who have near-daily EIB despite as-needed
prophylaxis.

Educational objective:
Exercise-induced bronchoconstriction is very effectively prevented by premedication with an inhaled corticosteroid
and beta agonist (eg, budesonide-formoterol) 10 minutes prior to exercise.

Reference
• An official American Thoracic Society clinical practice guideline: exercise-induced bronchoconstriction.

• Asthma and exercise-induced bronchoconstriction in athletes.


Question #325

A 38-year-old man is being evaluated due to shortness of breath, cough, and chest tightness. He reports that his
breathing "gets noisy" sometimes. The symptoms appeared 3 months ago, last throughout most of the day, and
have gradually become more severe. The patient started working as a research laboratory technician 2 years ago,
spending most of his day maintaining rodent colonies. He is unsure whether the respiratory symptoms are related
to his arrival at or departure from the animal facility. The patient's breathing improved when he attended a
weeklong conference in another city. He reports no fevers, night sweats, weight loss, or unusual fatigue. Vital signs
are within normal limits. Nasal and pharyngeal mucosae are normal. Pulmonary examination shows normal lung
sounds. Chest x-ray is normal. Baseline spirometry demonstrates mild airflow obstruction that is reversed after the
administration of albuterol. Which of the following diagnostic tests will be most useful for establishing the
diagnosis?

A) CT scan of the chest

B) Evaluation of vocal cord function

C) No further testing

D) Peak expiratory flow measurements at home and work

E) Therapeutic trial of intranasal steroid


Explanation
Correct Answer:

D) Peak expiratory flow measurements at home and work

Occupational asthma

• Workplace antigens → inflammation (IgE-dependent or independent) →


Pathogenesis bronchoconstriction
& high-risk • Classic antigens: animal proteins (eg, animal workers, seafood processors), grain
occupations carbohydrates (eg, bakers), isocyanates (eg, painters), metals (eg, welders)
• Latency (months/years) due to gradual sensitization

• Initially, symptoms clearly exacerbated at workplace & relieved at home


Clinical • Over time, symptoms persist throughout week & subside only after sustained work absence
features • Diagnosis: confirm asthma or BHR (eg, methacholine bronchoprovocation) → confirm
workplace-specific worsening of obstruction (eg, serial PEFR at home & work)

• Antigen avoidance (change of workplace if possible) or reduction (eg, respirator, shift


rotation)
Management
• Bronchodilators & inhaled corticosteroids
• Desensitization (immunotherapy)
BHR = bronchial hyperresponsiveness; PEFR = peak expiratory flow rate.

This patient has respiratory symptoms and reversible airflow obstruction on spirometry at baseline, findings
consistent with asthma. Symptoms improve on sustained absence from the workplace (ie, weeklong conference).
This key history element is suggestive of occupational asthma, defined as asthma specifically driven by
workplace antigen exposure.

Classic occupational antigens include animal proteins (eg, laboratory rodent workers), isocyanates (eg, painters),
and metals (eg, welders). Because of the long latency period required for immune sensitization, occupational
asthma may not manifest for months to years. Initially, patients may link symptoms closely to the workplace.
Eventually, when airway inflammation becomes established, symptoms may persist even at home throughout the
week and subside only after prolonged absence (eg, vacations).

Therefore, diagnosis requires a detailed occupational history, careful symptom timeline, and 2-step confirmatory
testing:

• First, asthma must be confirmed (eg, reversible obstruction on spirometry). For patients with normal
baseline spirometry, bronchoprovocation (eg, methacholine) is performed to confirm bronchial
hyperresponsiveness.

• Next, an occupational relationship is established by confirming workplace-specific worsening of airflow


obstruction (Choice C). This can be accomplished with serial peak expiratory flow rate (PEFR)
measurements using a portable peak flow meter.

Patients record their PEFR at home and at work (along with their symptoms): a PEFR decline by ≥20% at the
workplace relative to at home is consistent with occupational asthma.

(Choice A) Occupational hypersensitivity pneumonitis can present with respiratory symptoms waxing and waning
with workplace exposure. However, chest CT demonstrates ground-glass opacities (reflecting alveolitis), spirometry
shows restriction or a mixed pattern, and constitutional symptoms are common.
(Choice B) Vocal cord dysfunction (ie, paradoxic inspiratory adduction) can present with dyspnea and noisy
breathing (stridor). However, it tends to affect young women, attacks occur in discrete episodes (instead of lasting
the day), and spirometric flow-volume loop abnormalities would be expected.

(Choice E) Occupational allergic rhinitis can be relieved with intranasal steroids. The condition can cause
postnasal drip, producing chronic cough with no obvious physiologic abnormalities. However, nasal and sinus
symptoms would be expected, and signs of upper airway allergy would be seen on physical examination (eg,
turbinate hypertrophy, pharyngeal cobblestoning).

Educational objective:
Occupational asthma is induced by workplace antigen exposure (eg, animal proteins, isocyanates). Chronic
occupational asthma may produce persistent airway inflammation and bronchoconstriction that subside only after
prolonged absence from work. Once asthma is diagnosed (eg, baseline spirometry), occupational asthma is
confirmed by demonstrating workplace-specific worsening of airflow obstruction (eg, fall in peak expiratory flow).
Question #326

A 43-year-old woman with congestive heart failure, rheumatoid arthritis, and chronic hepatitis C with early liver
cirrhosis complains of difficulty breathing. Her temperature is 37.8 C (100 F), blood pressure is 126/72 mm Hg,
pulse is 97/min, and respirations are 28/min. Pulse oximetry is 94% on room air. Physical examination shows
dullness to percussion at the right lung base. Chest x-ray shows moderate-size right pleural effusion and cannot
rule out underlying infiltrates. Thoracentesis shows pleural fluid with glucose of 28 mg/dL and lactate
dehydrogenase of 252 U/L. Which of the following explains the low pleural fluid glucose concentration?

A) High amylase content of the pleural fluid

B) High white blood cell content of the pleural fluid

C) Increased capillary hydrostatic pressure

D) Increased permeability of the right hemidiaphragm

E) Increased pleural membrane permeability


Explanation
Correct Answer:

B) High white blood cell content of the pleural fluid

This patient's presentation is concerning for likely pneumonia with parapneumonic effusion. Pleural effusions are
divided into transudates and exudates. Transudates are due to an imbalance between hydrostatic and oncotic
pressures that increases fluid movement across the capillaries into the visceral pleura and pleural space.
Transudative fluid requires no further intervention except for treatment directed at the underlying disease.
Exudative effusions are due to pleural and lung inflammation resulting in increased capillary and pleural membrane
permeability. Exudative effusions require more extensive diagnostic investigation. The Light criteria define an
exudate as having at least one of the following:

• Pleural fluid protein/serum protein ratio >0.5


• Pleural fluid lactate dehydrogenase (LDH)/serum LDH ratio >0.6
• Pleural fluid LDH >2/3 of the upper limit of normal for serum LDH

This patient's pleural fluid LDH of 252 U/L is >2/3 of the upper limit of normal for serum LDH (2/3*90 = 60 U/L). The
low pleural glucose also indicates an exudative pleural effusion. Pleural fluid glucose <60 mg/dL is usually due to
rheumatoid pleurisy, complicated parapneumonic effusion or empyema, malignant effusion, tuberculous pleurisy,
lupus pleuritis, or esophageal rupture. (Pleural glucose <30 mg/dL in particular suggests an empyema or rheumatic
effusion.) Glucose concentration in empyema is decreased due to the high metabolic activity of leukocytes (and/
or bacteria) in the fluid.

(Choice A) Elevated pleural amylase concentrations are found in pleural effusions due to esophageal rupture or
pancreatitis. However, high pleural amylase is not responsible for the low glucose found in these effusions.

(Choice C) Increased capillary hydrostatic pressure is the mechanism responsible for transudative pleural
effusions. However, this patient has an exudative effusion due to the low pleural glucose and high pleural LDH.

(Choice D) Increased permeability of the right hemidiaphragm leads to right-sided pleural effusions in cirrhosis,
also known as a hepatic hydrothorax. Effusions due to cirrhosis are typically transudates.

(Choice E) Inflammatory conditions cause pleural effusions by increasing the pleural membrane and capillary
permeability. This mechanism establishes the effusion but does not directly decrease pleural glucose
concentration.

Educational objective:
Empyemas are exudative effusions with a low glucose concentration due to the high metabolic activity of leukocytes
and bacteria within the pleural fluid.

Reference
• Useful tests on the pleural fluid in the management of patients with pleural effusions.

• Diagnosis of pleural effusion: a systematic approach.


Question #327

A 25-year-old, previously healthy man comes to the office due to 3 months of progressive dyspnea and
nonproductive cough. The patient feels short of breath when cycling 2 miles to his office, which he could do without
difficulty in the past. He does not use tobacco or alcohol. The patient works at a publishing firm in New York and
has not traveled recently. Family history is significant for asthma in his father. Temperature is 36.7 C (98.1 F),
blood pressure is 130/80 mm Hg, pulse is 78/min, and respirations are 16/min. Pulse oximetry is 97% on room air.
The lungs are clear on auscultation and heart sounds are normal. Laboratory results are as follows:

Complete blood count


Hemoglobin 13 g/dL
Platelets 310,000/mm3
Leukocytes 8,000/mm3

Serum chemistry
Sodium 142 mEq/L
Potassium 4.6 mEq/L
Creatinine 0.8 mg/dL
Calcium 11.4 mg/dL
Glucose 98 mg/dL

Chest x-ray shows mediastinal fullness and scattered reticular opacities in the upper lungs. Which of the following
pulmonary function test patterns is most likely to be observed in this patient?

FEV1 Total lung capacity DLCO

A)
Decreased Decreased Normal

B)

Normal Normal Increased

C)

Normal Decreased Decreased

D)

Decreased Increased Decreased

E)

Decreased Increased Increased


Explanation
Correct Answer:

C)

Normal Decreased Decreased


This patient's presentation is consistent with sarcoidosis, a systemic, granulomatous disorder of unknown etiology
that commonly involves the lungs. Clues to the diagnosis include the patient's hypercalcemia (due to vitamin D
conversion by lung macrophages) and chest x-ray findings suggesting hilar lymphadenopathy (described as
mediastinal fullness and scattered reticular opacities in the upper lobe). Lung examination is often normal; patients
may have only mild symptoms.

Sarcoidosis can result in abnormal pulmonary function tests (PFTs) in up to 70% of patients. Granuloma-induced
scarring and fibrosis result in the typical restrictive pattern (ie, normal or reduced FEV1, normal or reduced FVC,
normal or increased FEV1/FVC ratio, reduced total lung capacity [TLC]). The inflammation and scarring intrinsically
impede alveolar gas exchange, leading to decreased diffusion capacity for carbon monoxide (DLCO). Among
patients demonstrating restrictive physiology on PFTs, the findings for DLCO help differentiate intrinsic lung disease
(eg, fibrosis) from extrinsic causes of hypoventilation (eg, chest wall weakness).

(Choice A) Obesity-hypoventilation syndrome, in which excess adipose tissue restricts diaphragmatic excursion,
can produce PFT results demonstrating restrictive physiology with chest wall weakness. Findings include
decreased FEV1 and TLC with preserved DLCO because the physiology of alveolar gas exchange is unaffected.

(Choice B) In diffuse pulmonary or alveolar hemorrhage (eg, pulmonary vasculitis), an abnormal increase in
hemoglobin within the alveolar space increases carbon monoxide transfer, resulting in increased DLCO. However,
lung volumes and airway mechanics are unaffected; therefore, FEV1 and TLC are normal.

(Choices D and E) Obstructive lung conditions (eg, asthma, chronic bronchitis, emphysema) result in lung
hyperinflation and airway obstruction during exhalation. Expected PFT findings include increased TLC and
decreased FEV1. The results for DLCO allow differentiation between chronic obstructive pulmonary disease
(COPD) and severe asthma. COPD is associated with reduced DLCO due to alveolar destruction, whereas severe
asthma is associated with increased DLCO due to increased thoracic blood flow caused by increased negative
intrathoracic pressures during inspiration.

Educational objective:
Pulmonary function tests in patients with sarcoidosis-associated lung disease typically follow a restrictive pattern:
normal or reduced FEV1, normal or reduced FVC, normal or increased FEV1/FVC ratio, reduced total lung capacity,
and decreased diffusion capacity for carbon monoxide.

Reference
• Pulmonary sarcoidosis.

• Pulmonary fibrosis in sarcoidosis: clinical features and outcomes.


Question #328

A 36-year-old woman comes to the office due to worsening shortness of breath. The patient has a long-standing
history of nasal congestion and frontal headaches that partly improved after nasal polyp removal a year ago. Over
the past several months, she has experienced increasingly frequent episodes of chest tightness and wheezing. The
patient has noted that the symptoms worsen after she takes over-the-counter analgesics. She has no pets, and
there have been no changes in her environment. She does not use tobacco or alcohol. Which of the following is
the main driver of pathogenesis of this patient's symptoms?

A) Autoantibodies

B) Bradykinin

C) IgE

D) IL-5

E) Leukotrienes
Explanation
Correct Answer:

E) Leukotrienes
This patient's presentation is most consistent with aspirin-exacerbated respiratory disease (AERD), a
nonimmunologic reaction to aspirin and other nonsteroidal anti-inflammatory drugs (NSAIDs). AERD should be
suspected when the Samter triad is present:

• Asthma (eg, intermittent wheezing, chest tightness)

• Chronic rhinosinusitis (eg, nasal congestion, frontal headaches) with nasal polyps

• NSAID-induced respiratory reactions (eg, worsening symptoms after over-the-counter analgesics [eg,
ibuprofen])

AERD pathogenesis involves increased production of proinflammatory leukotrienes and decreased production of
anti-inflammatory prostaglandins. NSAIDs inhibit the cyclooxygenase arm of the arachidonic acid pathway (COX-1,
COX-2) to block prostaglandin synthesis, shunting it toward the leukotriene production arm. AERD symptoms
are due to this leukotriene excess and mimic asthma or rhinosinusitis flares. Patients may not always relate the
symptoms to NSAID use, especially if they are taken habitually. Some patients may exhibit more dramatic
symptoms, including facial flushing, urticarial eruptions, angioedema, and anaphylactoid reactions (eg,
bronchospasm, hypotension), usually occurring 30 minutes to 3 hours after NSAID ingestion. Unlike anaphylactic
reactions, these pseudoallergic reactions are not IgE-mediated.

Treatment includes avoidance of NSAIDs. If NSAIDs are absolutely required, desensitization is sometimes
necessary. Leukotriene pathway inhibitors (eg, zileuton, montelukast) can also improve respiratory and nasal
symptoms.

(Choice A) Antineutrophil cytoplasmic autoantibodies are associated with eosinophilic granulomatosis with
polyangiitis, which often presents with prodromal asthma and nasal polyposis before the onset of overt systemic
vasculitis. However, the history of NSAID sensitivity is more strongly suggestive of AERD.

(Choice B) Bradykinin is an inflammatory vasoactive peptide involved in the pathogenesis of ACE inhibitor cough,
as well as ACE inhibitor and hereditary angioedema. The patient does not take an ACE inhibitor and has no
evidence of angioedema.

(Choice C) IgE is the primary upstream player in type 1 (immediate) hypersensitivity reactions such as
anaphylaxis, angioedema, extrinsic allergic asthma (eg, dust mites, pollen), and allergic rhinitis and conjunctivitis
(ie, hay fever). However, it is not involved in the pathogenesis of AERD.

(Choice D) IL-5 is a T helper type 2 (Th2) cytokine that plays a central role in the pathogenesis of intrinsic asthma
(eg, in response to weather, exercise) and chronic rhinosinusitis with nasal polyposis. With intrinsic asthma, the
sinus and pulmonary symptoms would not be aggravated by NSAID exposure.

Educational objective:
Aspirin-exacerbated respiratory disease is a nonimmunologic reaction that occurs when cyclooxygenase inhibitors
(eg, aspirin, other nonsteroidal anti-inflammatory drugs [NSAIDs]) promote the imbalanced production of
leukotrienes over prostaglandins. The classic triad includes asthma, chronic rhinosinusitis with nasal polyposis, and
NSAID sensitivity.

Reference
• Aspirin-Exacerbated Respiratory Disease
Question #329

A 25-year-old man comes to the emergency department due to dry cough, wheezing, and congestion. The patient
says his symptoms have been worsening over the past 3 days; he has no fever or chills. Medical history includes
well-controlled asthma and no hospitalizations. The patient says normally he uses his reliever inhaler only as
needed, usually 3 or 4 times a year, but it has not relieved his symptoms during this episode. Temperature is 37.1
C (98.8 F), blood pressure is 138/78 mm Hg, pulse is 112/min, and respirations are 24/min. Oxygen saturation is
95% on room air. Diffuse wheezing is noted on examination. The patient is appropriately treated and says he feels
much better 2 hours later. On reexamination, respirations are 18/min, oxygen saturation is 99% on room air, and
the wheezing has decreased. The patient would like to go home. In addition to optimizing his rescue inhaler
therapy, which of the following medications is indicated on discharge to promote resolution of this patient's
exacerbation?

A) Daily inhaled ipratropium

B) No additional medication

C) Five days of oral azithromycin

D) Daily oral loratadine

E) Short course of oral prednisone


Explanation
Correct Answer:

E) Short course of oral prednisone

Initial pharmacotherapy for acute asthma exacerbation

Class Medication Use

• Bronchodilator
Short-acting ꞵ2-agonist • Inhaled albuterol • Continuous (if severe) or
repeated dosing

• Bronchodilator
• Repeated dosing only in acute
Anticholinergic • Inhaled ipratropium bromide
setting & in conjunction with
albuterol

• Prednisone, • Anti-inflammatory
methylprednisolone, • Delayed effect (6 hr)
Glucocorticoid
dexamethasone (oral/ • Multiday dosing for control of
intravenous) late-phase inflammation

An acute asthma exacerbation is defined as an acute worsening of symptoms (eg, cough, wheezing, dyspnea)
accompanied by a significant decline in expiratory flow rate. Acute exacerbations occur when sensitized
bronchioles are exposed to a trigger (eg, respiratory tract infection, allergens), causing mast cell degranulation.

In the immediate phase (minutes to hours) of an acute asthma exacerbation, release of preformed mediators
(histamine, leukotriene) causes immediate bronchoconstriction, airway edema, and mucus production. Initial
treatment of an acute asthma exacerbation focuses on relaxing bronchoconstriction with a short-acting beta agonist
(eg, albuterol), a short-acting muscarinic antagonist (eg, ipratropium), and intravenous magnesium.

Hours after the immediate response, a late-phase reaction occurs, characterized by proinflammatory gene
expression, de novo synthesis of TH2 cytokines, and further recruitment of leukocytes, all contributing to ongoing
bronchospasm and inflammation. Systemic glucocorticoids (eg, oral prednisone, dexamethasone) reduce late-
phase inflammation.

Because the anti-inflammatory activity of glucocorticoids is delayed by several hours (due to the time required to
change nuclear gene expression), they are often given with bronchodilators in the emergency department.
Corticosteroids are continued at home (eg, prednisone 40-60 mg daily for 5-7 days) to prevent relapse and
decrease the need for hospitalization (Choice B).

(Choice A) Bronchial smooth muscle relaxation (eg, via albuterol, ipratropium) can improve airflow in the short-
term but does not sufficiently treat the underlying airway inflammation and edema during an acute asthma
exacerbation.

(Choice C) Antibiotics are not routinely indicated for asthma exacerbations unless there is evidence for an
underlying lower respiratory tract infection (eg, fever, purulent sputum, rales, egophony). Macrolide antibiotics (eg,
azithromycin) can improve recovery in some patients with acute exacerbations of chronic obstructive pulmonary
disease, partially because they inhibit neutrophil-mediated airway inflammation. In multiple clinical trials, macrolides
provided no benefit for acute asthma exacerbations, which are driven by eosinophil-mediated airway inflammation.

(Choice D) An oral antihistamine (eg, H1 receptor antagonist) is used to treat allergic rhinitis. Although allergic
rhinitis can trigger an asthma exacerbation, antihistamines do not have a role in acute asthma management.

Educational objective:
Treatment of an acute asthma exacerbation includes a short-acting bronchodilator to manage immediate
bronchoconstriction and a systemic glucocorticoid (eg, prednisone) to dampen late-phase inflammation and prevent
symptom relapse.

Reference
• Asthma exacerbations: pathogenesis, prevention, and treatment.
Question #330

A 60-year-old man comes to the office due to cough and dyspnea on exertion. The patient has had a cough for
over 2 years, but it has become worse recently and is accompanied by increasing shortness of breath. The cough
is productive with a small volume of mucoid sputum. He has no hemoptysis, chest pain, or leg swelling. The
patient has a 40-pack-year smoking history and drinks 1 or 2 glasses of wine daily. He has worked in a shipyard for
10 years. Other medical conditions include rheumatoid arthritis and hypertension. Temperature is 37.2 C (99 F),
blood pressure is 140/80 mm Hg, pulse is 80/min, and respirations are 20/min. Chest x-ray reveals prominent
bronchovascular markings and mild diaphragmatic flattening. Pulmonary function test results are as follows:

FEV1 67% of predicted


FVC 95% of predicted
FEV1/FVC ratio 0.65

Diffusion capacity of the lung for carbon monoxide is 100% of predicted value. Which of the following is the most
likely cause of this patient's symptoms?

A) Asbestosis

B) Bronchiectasis

C) Chronic bronchitis

D) Panacinar emphysema

E) Pulmonary fibrosis

F) Silicosis
Explanation
Correct Answer:

C) Chronic bronchitis

Differential diagnosis based on DLCO

Restrictive pattern
Obstructive pattern
(FEV1/FVC >70% predicted, Normal spirometry
(FEV1/FVC <70% predicted)
FVC <80% predicted)

• Anemia
• Interstitial lung diseases
• Pulmonary
Low • Sarcoidosis
• Emphysema embolism
DLCO • Asbestosis
• Pulmonary
• Heart failure
hypertension

Normal • Chronic bronchitis • Musculoskeletal deformity


DLCO • Asthma • Neuromuscular disease

• Pulmonary
Increased
• Asthma • Morbid obesity hemorrhage
DLCO
• Polycythemia
DLCO = diffusing capacity of the lung for carbon monoxide.

Chronic obstructive pulmonary disease (COPD) encompasses a spectrum of pathology ranging from chronic
bronchitis to emphysema, eventually leading to nonreversible airflow obstruction (eg, pulmonary function testing
[PFT] with decreased FEV1/FVC ratio, decreased FEV1). The resultant air trapping can lead to lung hyperinflation,
seen as diaphragmatic flattening on chest x-ray.

This patient has a classic presentation of COPD with a predominant chronic bronchitis phenotype. Chronic
bronchitis occurs due to irritant-induced airway mucosal inflammation, respiratory epithelium metaplasia, and mucus
hypersecretion. It is defined as productive cough for ≥3 months over 2 consecutive years (not due to other
causes) with or without airflow limitation. By contrast, emphysema involves proteolytic destruction of the alveolar
walls. Some distinguishing features include the following:

• The diffusing capacity of the lung for carbon monoxide (DLCO), which measures gas transfer between the
alveoli and pulmonary capillary blood, remains normal in chronic bronchitis (intact alveolar and capillary
membrane) but is decreased in emphysema, where destruction of the alveolar wall reduces the available
gas exchange surface area.

• The chest x-ray findings in chronic bronchitis reflect airway inflammation, including prominent thickened
bronchovascular markings. In contrast, chest x-ray findings in emphysema reflect obliteration of the
alveolar septa and include decreased lung tissue density (hyperlucency) and bullae.

Management of these 2 subgroups is similar, although the distinction can be useful in treating advanced cases. For
example, patients with frequent exacerbations of chronic bronchitis gain more benefit from mucus clearance therapy
(eg, chest percussion) and anti-inflammatory agents (eg, inhaled corticosteroids).

(Choices A, E, and F) Asbestosis, pulmonary fibrosis, and silicosis are examples of interstitial lung diseases that
demonstrate a restrictive pattern on PFT, characterized by reduced lung volumes, roughly proportional decreases in
FEV1 and FVC, normal or supranormal FEV1/FVC ratio, and low DLCO.

(Choice B) Bronchiectasis is a disease of abnormal bronchial widening in the setting of recurrent infection and
inflammation. Its presentation can be similar to that of chronic bronchitis. However, sputum production is more
prominent and voluminous, the chest x-ray reveals dilated (rather than thickened) conducting airways, and episodes
of bacterial superinfection are common.

(Choice D) Panacinar emphysema is typical for alpha-1-antitrypsin deficiency; centriacinar emphysema is more
characteristic for smoking-induced COPD. Both forms of emphysema usually cause a low DLCO.

Educational objective:
Chronic obstructive pulmonary disease (COPD) encompasses a spectrum of chronic bronchitis and emphysema.
The diffusing capacity of the lung for carbon monoxide is normal in chronic bronchitis-predominant COPD and
decreased in emphysema-predominant COPD.
Question #331

A 38-year-old woman comes to the office due to a dry cough and malaise for the past 2 months. She has no prior
medical issues and takes no medications. The patient reports no allergies and does not smoke. Vital signs are
normal. Examination shows no abnormalities. Her chest x-ray is shown below.
Which of the following is the most likely pathophysiology of this patient's condition?

A) Allergen-induced bronchospasm
B) Chronic granulomatous inflammation

C) Loss of elastin in the lung matrix

D) Necrotizing pulmonary vasculitis

E) Pulmonary venous congestion

F) Recurrent pulmonary embolism


Explanation
Correct Answer:

B) Chronic granulomatous inflammation

Manifestations of sarcoidosis

• Bilateral hilar adenopathy


Pulmonary
• Interstitial infiltrates

• Papular, nodular, or plaquelike lesions


Cutaneous
• Erythema nodosum

• Anterior uveitis (iridocyclitis or iritis)


Ophthalmologic • Posterior uveitis
• Keratoconjunctivitis sicca

• Peripheral lymphadenopathy
Reticuloendothelial • Hepatomegaly
• Splenomegaly

Musculoskeletal • Acute polyarthritis (especially ankles)


• Chronic arthritis

• Atrioventricular block
Cardiovascular
• Dilated or restrictive cardiomyopathy

• Facial nerve palsy


CNS/endocrine • Central diabetes insipidus
• Hypercalcemia

• Erythema nodosum
• Hilar adenopathy
Löfgren syndrome
• Migratory polyarthralgia
• Fever

This patient's symptoms of dry cough and malaise for 2 months with findings of bilateral hilar adenopathy on chest
x-ray strongly suggest sarcoidosis. Sarcoidosis is a chronic multisystem disorder due to noncaseating
granulomatous inflammation. It typically occurs in young adults, is 3-4 times more common in African Americans,
and affects more women than men. Sarcoidosis is frequently detected incidentally on chest x-ray, and bilateral hilar
adenopathy is the first manifestation of disease in 50% of patients. It can also present with symptoms of cough,
dyspnea, fever, fatigue, and weight loss.

Definitive diagnosis requires consistent radiographic and clinical findings, and a biopsy demonstrating noncaseating
granulomas. Bronchoscopy with transbronchial biopsy is often performed to obtain tissue diagnosis. If there is
evidence of systemic involvement, more accessible sites such as peripheral lymph nodes or cutaneous lesions can
be biopsied. Patients with incidental bilateral hilar adenopathy are typically monitored without biopsy unless
symptoms develop.
(Choice A) Allergen-induced bronchospasm occurs in asthma. Patients typically present with cough and
intermittent dyspnea (absent in this patient). Hilar adenopathy is not a typical finding on chest x-ray.

(Choice C) Loss of elastin in the lung matrix occurs in alpha-1-antitrypsin (AAT) deficiency, leading to emphysema.
Nonsmoking patients with AAT typically present in their mid-40s with dyspnea, cough, and sputum production.
Chest x-ray reveals hyperinflation, and bilateral hilar adenopathy is not a typical finding.

(Choice D) Necrotizing pulmonary vasculitis occurs in granulomatosis with polyangiitis (formerly Wegener
granulomatosis). Fever, weight loss, and rhinosinusitis are commonly seen on presentation. Pulmonary nodules
and alveolar consolidation are typical findings on chest x-ray but are not present in this patient.

(Choice E) Pulmonary venous congestion occurs in patients with congestive heart failure. Characteristic chest x-
ray findings include cardiomegaly, cephalization of pulmonary vessels with prominent vascular markings, and
pleural effusions, which are not present in this patient.

(Choice F) Pulmonary embolism typically is accompanied by chest pain and dyspnea, and hilar adenopathy on
chest x-ray is not typically seen.

Educational objective:
Sarcoidosis typically affects young adults and is more common in African Americans. It is often discovered
incidentally on routine chest x-ray by the presence of bilateral hilar adenopathy. The underlying histopathologic
lesion is noncaseating granulomas on tissue biopsy.

Reference
• A concise review of pulmonary sarcoidosis.
Question #332

A 54-year-old man comes to the office due to fatigue, a productive cough with blood-tinged sputum, and a 7-kg
(15.4-lb) weight loss over the past few months. He has no fever, chest pain, or night sweats. Ten years ago, the
patient emigrated from Vietnam, where he received treatment for pulmonary tuberculosis. He has no other medical
history. He does not use tobacco or illicit drugs but drinks alcohol occasionally. Temperature is 36.7 C (98 F), blood
pressure is 120/80 mm Hg, and pulse is 82/min. Lung auscultation reveals right-sided rhonchi and crackles.
Creatinine is 1.1 mg/dL and urinalysis is normal. CT scan of the chest is shown in the image below.

Which of the following is the most likely diagnosis for this patient?
A) Actinomyces israelii infection

B) Bacterial lung abscess

C) Chronic pulmonary aspergillosis

D) Granulomatosis with polyangiitis

E) Histoplasmosis

F) Small cell lung cancer


Explanation
Correct Answer:

C) Chronic pulmonary aspergillosis

Invasive aspergillosis Chronic pulmonary aspergillosis*

• Immunocompromise (neutropenia, • Lung disease/damage (cavitary


Risk factors
glucocorticoids, HIV) tuberculosis)

• Triad of fever, chest pain, hemoptysis


• >3 months: weight loss (>90%), cough,
• Pulmonary nodules with halo sign
hemoptysis, fatigue
Manifestations • Positive cultures
• Cavity lesion ± fungus ball
• Positive cell wall biomarkers
• Positive Aspergillus IgG serology
(galactomannan, β-D-glucan)

• Resect aspergilloma (if possible)


Management • Voriconazole ± caspofungin • Azole medication (voriconazole)
• Embolization (if severe hemoptysis)

*Simple aspergilloma (fungus ball in preexisting lung cavity) is a form of chronic pulmonary aspergillosis but is
usually quiescent with occasional hemoptysis.

Aspergillus is a ubiquitous fungus that most people encounter daily. Conidia are inhaled into the lung and convert to
potentially pathogenic hyphae. Patients with immunocompetency rapidly clear the organism and rarely develop
infection; however, a subset of immunocompetent patients with a history of pulmonary disease (eg, cavitary
tuberculosis) may develop chronic pulmonary aspergillosis (CPA) at sites of lung damage. Diagnosis is made by
the presence of all 3 of the following:

• >3 months of symptoms - fever, weight loss, fatigue, cough, hemoptysis, and/or dyspnea
• Cavitary lesion(s) containing debris, fluid, or an aspergilloma (fungus ball)
• Positive Aspergillus IgG serology

Therapy depends on symptoms and severity of disease; antifungal medication (eg, itraconazole, voriconazole),
surgery (to prevent hemoptysis), and bronchial artery embolization (for hemoptysis with extensive disease) may be
used together or separately.

A simple aspergilloma is sometimes categorized as CPA but is typically quiescent (aside from occasional mild
cough or hemoptysis) with no systemic symptoms. This patient has CPA rather than simple aspergilloma.

(Choice A) Actinomyces israelii is an anaerobic bacterium that usually causes cervicofacial infections. Rarely,
Actinomyces may cause an indolent pneumonia with weight loss, fever, cough, and CT scan findings of pulmonary
nodules or lobar consolidation (not a cavitary lesion with a fungus ball).

(Choice B) Bacterial lung abscess causes indolent symptoms of fever, cough, dyspnea, weight loss and CT scan
findings of an infiltrate with a cavity. Lung abscesses usually have necrotic tissue with air fluid levels and do not
have evidence of fungal balls.

(Choice D) Granulomatosis with polyangiitis often causes chronic fever, fatigue, and weight loss. Pulmonary
disease is common, and imaging may reveal cavitary nodules. However, most patients also have nasal, sinus, or
ear symptoms, and many have renal insufficiency and active urine sediment (unlike this patient).

(Choice E) Histoplasmosis is an endemic mycosis of the central and midwestern United States that rarely causes
illness in immunocompetent individuals but may occasionally cause subacute pulmonary symptoms (<5%).
However, pulmonary histoplasmosis usually causes focal infiltrates with hilar lymphadenopathy (not cavitary
lesions).
(Choice F) Small cell lung cancer almost never causes cavitation. It most commonly presents as a hilar mass with
lymphadenopathy and is often characterized by widespread metastases at presentation due to its rapid growth rate.

Educational objective:
Chronic pulmonary aspergillosis manifests with >3 months of weight loss, cough, hemoptysis, and fatigue in
patients with a history of underlying lung disease. Imaging usually reveals >1 apical cavitary lesions with or without
an aspergilloma. Diagnosis is made with Aspergillus IgG serology in the setting of characteristic symptoms and
radiographic findings.

Reference
• Pulmonary aspergillosis: a clinical review.
Question #333

A 53-year-old man comes to the office due to nonspecific chest pain localized to the left chest along the fifth
intercostal space. His other medical problems include autosomal dominant polycystic kidney disease and
hypertension. He has a 10-pack-year smoking history but quit 14 years ago. He consumes alcohol occasionally
and does not use illicit drugs. Blood pressure is 160/90 mm Hg and pulse is 90/min. Physical examination shows
no abnormalities. Laboratory results are as follows:

Hemoglobin 10.8 g/dL


Leukocytes 10,000/mm3
Creatinine 1.2 mg/dL
Blood urea nitrogen 28 mg/dL
Sodium 142 mEq/L
Potassium 4.2 mEq/L

Chest x-ray reveals a solitary round lesion in the left upper lung field 1 cm in diameter with sharp borders. The
lesion does not touch the pleura. A chest x-ray from 3 years earlier was normal. Which of the following is the most
appropriate next step in evaluating this patient's pulmonary findings?

A) Computed tomography scan of the chest

B) Flexible bronchoscopy

C) Percutaneous biopsy of the lesion

D) Pulmonary function testing

E) Repeat chest x-ray in 2 months


Explanation
Correct Answer:

A) Computed tomography scan of the chest


A solitary pulmonary nodule (SPN) is defined as a round opacity up to 3 cm in diameter within and surrounded by
pulmonary parenchyma. By convention, there must be no associated pleural effusion, adenopathy, or atelectasis.
SPN is often discovered incidentally on chest x-ray and can be due to many causes. The first step is to compare
previous x-rays or scans. An SPN with stable size and appearance for 2 to 3 years has a low malignancy risk and
the patient may be given reassurance without further workup. Ground-glass lesions have a higher malignancy risk
and likely require yearly assessment, even with stable appearance and size.

Unstable or new lesions require further assessment for malignancy risk. The main objective in assessing SPNs is
to promptly detect and surgically resect malignancies and to leave benign lesions intact. Nodules <0.8 cm with
smooth margins are less likely to be malignant; nodules ≥2 cm with irregular or spiculated margins are more likely to
be malignant. Other factors to consider are age and smoking status. Patients age >60 have a higher likelihood of
malignancy. In addition, current smokers have increasing malignancy risk as the daily cigarette consumption
increases.

This patient has a new SPN since a previous chest x-ray and requires chest CT scan to further assess the
malignancy risk. CT scan is more sensitive than chest x-ray to identify these features and also can detect other
small nodules that may represent metastasis. If the CT scan findings are suspicious for malignancy, biopsy
followed by likely surgical excision should be performed. Lesions that appear less likely to be malignant on CT
scans can be followed with serial CT scans for 2 to 3 years to monitor for growth.

(Choice B) Bronchoscopy can be used to biopsy accessible lesions (central lesions) but is not as sensitive as
percutaneous biopsy unless the lesion is >2 cm. In general, percutaneous biopsies and surgical excision carry a
superior diagnostic yield.

(Choice C) Percutaneous biopsy is recommended for lesions that appear malignant and in patients with high
clinical suspicion for malignancy. If the CT findings are suspicious for malignancy, a biopsy can be attempted by CT
guidance. An initial CT scan will also help evaluate whether the lesion can be accessed percutaneously without risk
for pneumothorax.

(Choice D) Pulmonary function testing will provide no additional information regarding the biologic behavior of the
mass. It is often obtained as part of the routine follow-up of chronic obstructive pulmonary disease and in patients
with lung cancer who are evaluated for surgical fitness.

(Choice E) CT scan offers better resolution and growth detection than chest x-ray (0.5 mm versus 3.5 mm) and is
preferred for following an SPN.

Educational objective:
Evaluation of a solitary pulmonary nodule detected on chest x-ray includes comparison with old imaging studies, if
available, followed usually by chest computed tomography. The decision to observe, biopsy, or surgically resect the
nodule is based on its size and characteristics as well as the patient's age and smoking history.

Reference
• The solitary pulmonary nodule-deciding when to act?
Question #334

A 56-year-old woman comes to the office due to progressive asthma symptoms. The patient describes nighttime
cough and wheezing that have increased in recent months. Sometimes she has to use her albuterol inhaler right
after a meal. She also reports feeling tired each morning as she works late and has no time to relax after dinner.
The patient has no dyspnea on exertion but says that her throat has been sore lately and describes hoarseness in
the morning that clears during the day. She has gained 5 kg (11 lb) over the last 6 months. Her medical history is
significant for bronchial asthma, type 2 diabetes mellitus, and hypertension. Medications include low-dose inhaled
fluticasone, an albuterol inhaler, lisinopril, amlodipine, and aspirin with no change in her medications for years.
Blood pressure is 140/90 mm Hg, pulse is 80/min, and respirations are 16/min. BMI is 32 kg/m2. Lung auscultation
indicates normal breath sounds without wheezing. Heart sounds are normal. Which of the following is the most
appropriate next step in management of this patient?

A) Add benzocaine lozenges

B) Add diphenhydramine

C) Add esomeprazole

D) Add oral corticosteroids

E) Discontinue aspirin

F) Discontinue lisinopril
Explanation
Correct Answer:

C) Add esomeprazole

Comorbid gastroesophageal reflux disease (GERD) is common in patients with asthma and can exacerbate
asthma symptoms through microaspiration of gastric contents, leading to an increase in vagal tone and
bronchial reactivity. This patient with asthma has several clues in her history suggesting comorbid GERD,
including sore throat, morning hoarseness, worsening cough only at night, and increased need for her albuterol
inhaler following meals. In addition, obesity increases the risk of developing GERD, and this patient's worsening
asthma symptoms coincide with her recent weight gain. Other symptoms suggestive of GERD that are not present
in this patient include dysphagia, chest pain/heartburn, and sensation of regurgitation.

Proton-pump inhibitor (PPI) therapy has been shown to improve both asthma symptoms and peak expiratory flow
rate in asthma patients with evidence of comorbid GERD, and a PPI trial (eg, esomeprazole) should be initiated in
this patient.

(Choice A) Benzocaine lozenges can provide temporary relief of throat pain but would not address the primary
cause of the problem.

(Choice B) The anticholinergic effect of diphenhydramine is useful in the treatment of chronic cough caused by
rhinitis in patients with upper airway cough syndrome (UACS), previously known as postnasal drip syndrome. This
patient's symptoms are more suggestive of comorbid GERD than comorbid UACS (no rhinorrhea, no sensation of
something "dripping" into throat).

(Choice D) Oral corticosteroids are used for acute asthma exacerbation (eg, dyspnea at rest, wheezing), which this
patient does not have.

(Choice E) Aspirin-exacerbated respiratory disease occurs in patients with asthma and chronic rhinosinusitis.
Symptoms include a sudden worsening of asthma and nasal congestion 30 minutes to 3 hours after ingestion of
nonsteroidal anti-inflammatories. This patient does not have symptoms suggestive of significant nasal congestion.
(Choice F) Cough associated with ACE inhibitors may occur at any time during therapy (usually within 2 weeks of
drug initiation). However, it would be unlikely to occur only at night and to be associated with wheezing and
morning hoarseness.

Educational objective:
Comorbid gastroesophageal reflux disease (GERD) is common in patients with asthma and can worsen asthma
symptoms as a result of microaspiration. In asthma patients with signs and/or symptoms suggestive of comorbid
GERD, proton-pump inhibitor therapy has shown benefit in improving asthma symptoms and peak expiratory flow
rate.

Reference
• Effects of esomeprazole 40 mg twice daily on asthma: a randomized placebo-controlled trial.
Question #335

A 45-year-old woman is evaluated for fatigue and exertional dyspnea. She has had difficulty walking for more than
1 or 2 blocks without becoming short of breath. She uses 2 pillows to sleep. Past medical history is significant for
hyperlipidemia, hypertension, and type 2 diabetes mellitus. The patient smoked a pack a day for 15 years but quit
10 years ago. She does not use alcohol or illicit drugs. Temperature is 36.7 C (98 F), blood pressure is 150/90 mm
Hg, pulse is 80/min, and respirations are 16/min. BMI is 55 kg/m2. On physical examination, jugular venous
distension is difficult to visualize due to a thick neck. Lungs are clear to auscultation without wheezes or crackles.
Heart sounds are distant. The abdomen is obese and nontender. There is trace bilateral lower-extremity edema.
Neurological examination is within normal limits. Chest x-ray is of poor quality due to under-penetration but shows
no obvious abnormalities. ECG shows low-voltage QRS complexes but no significant ST-segment or T-wave
abnormalities. Laboratory results are as follows:

Complete blood count


Hemoglobin 16.0 g/dL
Platelets 224,000/mm3
Leukocytes 6,600/mm3

Arterial blood gases


pH 7.3
PaO2 70 mm Hg
PaCO2 59 mm Hg

Which of the following is the predominant cause of this patient's dyspnea?

A) Airway inflammation and bronchospasm


B) Alveolar hypoventilation

C) Left ventricular contractile dysfunction

D) Neuromuscular dysfunction

E) Ventilation-perfusion mismatch
Explanation
Correct Answer:

B) Alveolar hypoventilation

Obesity hypoventilation syndrome

• Obesity with BMI ≥30 kg/m2


Diagnostic criteria • Awake daytime hypercapnia (PaCO2 >45 mm Hg)
• No alternate cause of hypoventilation

• ABG on room air (hypercapnia, normal A-a gradient)


• No intrinsic pulmonary disease on chest x-ray
Workup • Restrictive pattern on PFTs
• Normal TSH
• Polysomnography

• Nocturnal positive pressure ventilation as first-line therapy


• Weight loss (bariatric surgery in select cases)
Treatment
• Avoidance of sedative medications
• Respiratory stimulants (eg, acetazolamide) as last resort

ABG = arterial blood gas; PFT = pulmonary function test.


Obesity hypoventilation syndrome (OHS) is defined as daytime hypercapnia (PaCO2 >45 mm Hg) in an obese
patient (BMI ≥30 kg/m2, often >40 kg/m2) without another explanation for the hypercapnia. Most patients have
coexisting obstructive sleep apnea with frequent apneic events and daytime hypersomnolence. Other features of
OHS include dyspnea, polycythemia, respiratory acidosis with compensatory metabolic alkalosis, pulmonary
hypertension, and cor pulmonale.

Several mechanisms likely contribute to hypoventilation and resultant hypercapnia. Obesity reduces chest wall
and lung compliance, leading to a decrease in tidal volumes and total lung capacity and an increase in airway
resistance. As a result, higher levels of ventilatory drive are required to maintain normocapnia, but there is an
inability to exhale excess CO2 during the day (due to persistent restriction). This leads to CO2 accumulation
overnight, with subsequent chronic respiratory acidosis. Renal bicarbonate excretion is decreased as a
compensatory mechanism; this blunts the ventilatory response to the increased CO2 and contributes to
hypoventilation.

In sum, patients with OHS "can't breathe" (due to excess weight and altered lung mechanics) and "won't breathe"
(due to decreased chemosensitivity to hypercapnia from persistent nocturnal hypoventilation).

(Choice A) Airway inflammation and bronchospasm occur with reactive airway disease, which would cause
wheezing or cough.

(Choice C) Left ventricular contractile dysfunction results in systolic left heart failure. This patient is at risk for
diastolic heart failure as a result of likely prolonged uncontrolled hypertension. However, she has clear lungs and
minimal lower-extremity edema, making left-sided heart failure less likely.

(Choice D) Neuromuscular dysfunction (eg, due to myasthenia gravis, amyotrophic lateral sclerosis, Guillain-Barré
syndrome) can cause hypoventilation and dyspnea. However, this patient's elevated BMI and lack of neurologic
signs or symptoms make OHS a more likely diagnosis.

(Choice E) Ventilation/perfusion mismatch occurs in disease processes such as pulmonary embolism, pneumonia,
and pulmonary edema. Hypoxemia with an elevated alveolar-arterial gradient should be present. Hypoventilation
causes hypoxemia (PaO2 is often <70 mm Hg), but the alveolar-arterial gradient remains normal.

Educational objective:
Obesity hypoventilation syndrome is defined by obesity (BMI ≥30 kg/m2), daytime hypercapnia (PaCO2 >45 mm
Hg), and alveolar hypoventilation. Arterial blood gas will show hypercapnia, hypoxemia, and respiratory acidosis as
a result of altered lung mechanics due to obesity and hypoventilation.

Reference
• Obesity hypoventilation syndrome: a state-of-the-art review.

• Chronic hypoventilation and its management.

• Obesity hypoventilation syndrome--the big and the breathless


Question #336

A 64-year-old man comes to the office for evaluation of persistent cough and worsening dyspnea over the last 6
months. The cough is dry, nagging, and present throughout the day. He reports mild dyspnea with exertion, which
has become increasingly bothersome. The patient has no chest pain, syncope, orthopnea, or paroxysmal nocturnal
dyspnea. He had an inferior wall myocardial infarction due to right coronary artery occlusion 5 years ago and was
treated with a drug-eluting stent. The patient's other medical concerns include a hiatal hernia and hypertension.
Medications include aspirin, losartan, atorvastatin, and famotidine. Temperature is 36.8 C (98.2 F), blood pressure
is 130/78 mm Hg, pulse is 82/min, and respirations are 15/min. Pulse oximetry is 97% on room air. Head and neck
examination is normal. Jugular venous pressure is normal. There are no murmurs, extra sounds, or gallops. Fine,
dry inspiratory crackles are heard in the lower lung zones. Diaphragm excursion is normal and symmetric. There is
no peripheral edema. Chest x-ray is normal. What is the most appropriate next step in immediate management?

A) Barium esophagram

B) High-resolution CT scan of the chest

C) Stress echocardiography

D) Substitute clopidogrel for aspirin

E) Twice-daily proton pump inhibitor therapy


Explanation
Correct Answer:

B) High-resolution CT scan of the chest

This patient with ischemic heart disease has dry cough, exertional dyspnea, and inspiratory crackles. Given the
lack of clinical evidence for heart failure (eg, normal jugular venous pressure, no peripheral edema), he should be
evaluated for early interstitial lung disease (ILD).

Inspiratory "Velcro" (eg, fine, dry) crackles are sensitive for interstitial fibrosis, which is present in many forms of
ILD. They are typically audible long before abnormalities on chest x-ray (CXR) or pulmonary function testing
emerge, underscoring the value of careful auscultation. Early into disease progression, interstitial lung
abnormalities may not be seen on CXR. Therefore, patients with suspected ILD should undergo high-resolution
computed tomography (HRCT), a thin-slice (usually 1 mm) CT scan protocol developed to visualize subtle
interstitial lung features (eg, reticulation, honeycombing).

Compared to conventional CT, HRCT provides sharper images because slices are taken in a "step-and-shoot"
format (as opposed to helical continuous scanning). A prone sequence is often acquired to remove compressive
atelectasis, allowing better visualization of subtle reticulation or honeycombing in the dorsal dependent lung.
The crisp detail offered by HRCT allows for specific classification of ILD (eg, usual interstitial pneumonia,
cryptogenic organizing pneumonia).

In addition to HRCT, workup of suspected ILD involves full pulmonary function testing (eg, lung volumes,
diffusing capacity).

(Choices A and E) Barium esophagography and a proton pump inhibitor (PPI) are useful in evaluation and
management of patients with gastroesophageal reflux (which can cause cough) or certain forms of dysphagia
(associated with aspiration and cough). Reflux-induced lung microinjury can promote pulmonary fibrosis (eg,
idiopathic pulmonary fibrosis [IPF], a form of ILD). However, simple reflux would not explain dyspnea or Velcro
crackles. Although PPIs are often recommended for reflux-driven IPF, HRCT should be obtained to establish the
diagnosis first.

(Choice C) Patients with known coronary atherosclerotic disease are at risk for cardiac ischemia and/or congestive
heart failure (CHF). Stress imaging is often complementary to pulmonary workup. However, although ischemia can
cause dyspnea (angina equivalent), it does not explain dry cough and fine crackles. Although ischemic left-sided
CHF can cause dyspnea, it is classically associated with wet (vs dry) crackles, frothy sputum, and orthopnea; in
addition, an inferior wall myocardial infarction is most likely to cause right-sided (eg, peripheral edema), rather than
left-sided, symptoms.

(Choice D) P2Y12 inhibitors (eg, clopidogrel) are occasionally substituted for aspirin in patients with aspirin-
exacerbated respiratory disease (AERD). AERD presents with asthma (episodic wheezing, rather than crackles
and progressive dyspnea), recurrent nasal polyps, and hypersensitivity to nonsteroidal anti-inflammatory drugs.

Educational objective:
In patients with suspected early interstitial lung disease, initial workup includes high-resolution computed
tomography scanning and full pulmonary function testing.

Reference
• High-resolution CT of interstitial lung disease: a continuous evolution

• Role of imaging in progressive-fibrosing interstitial lung diseases


Question #337

A 44-year-old man comes to the office due to persistent cough and dyspnea on exertion for approximately the last 3
months. He has had no fever, chills, rhinorrhea, or sputum production. Over the past year, the patient has had
progressively worsening shortness of breath and is unable to sleep lying flat. He has no chest pain or diaphoresis
but has had palpitations previously. The patient does not use tobacco or alcohol. He moved to the United States
from Southeast Asia 10 years ago. On examination, the patient is alert but in mild distress. Temperature is 36.7 C
(98 F), blood pressure is 110/70 mm Hg, and pulse is 100/min. BMI is 34 kg/m2. Heart sounds are distant due to
body habitus. The lungs demonstrate crackles on auscultation. Chest x-ray reveals an enlarged cardiac silhouette
with vascular congestion. The left main stem bronchus appears to be elevated. There is no other visible lung
pathology. ECG shows an irregularly irregular rhythm. Which of the following is the most likely cause of this
patient's presentation?

A) Acute pericarditis

B) Interstitial lung disease

C) Lung cancer

D) Rheumatic heart disease

E) Sarcoidosis
Explanation
Correct Answer:

D) Rheumatic heart disease

Mitral stenosis in adults

• Rheumatic heart disease (vast majority of cases)


Etiology
• Age-related calcification, radiation induced

• Exertional dyspnea, orthopnea, paroxysmal nocturnal dyspnea, hemoptysis


Clinical presentation • Pulmonary edema ± right-sided heart failure (eg, lower extremity edema)
• Atrial fibrillation, ↑ risk for systemic embolization

• Opening snap with middiastolic rumble at the apex


Diagnosis
• Echocardiography: ↑ transmitral flow velocity

Treatment • Percutaneous valvotomy or surgical repair/replacement

This patient likely has decompensated heart failure due to mitral stenosis. Mitral stenosis usually occurs in the
setting of rheumatic heart disease, which in the United States is most often seen in patients who emigrated from
Latin America, Africa, or Asia.

Mitral stenosis causes eventual backflow of blood into the left atrium, leading to elevated left atrial and pulmonary
vascular pressures. This classically presents as dyspnea on exertion, often the only symptom of mitral stenosis,
and can progress to pulmonary edema. Manifestations vary from cough (productive or nonproductive, occasionally
with hemoptysis) to orthopnea, paroxysmal nocturnal dyspnea, and other features of decompensated heart failure
(eg, vascular congestion, enlarged cardiac silhouette). The increased left atrial pressure also leads to left atrial
dilation that causes upward displacement of the left mainstem bronchus on chest x-ray.

Left atrial dilation can also disrupt atrial electrical conduction, commonly leading to atrial fibrillation (eg,
palpitations, irregularly irregular rhythm on ECG). The loss of organized atrial contraction often precipitates an
acute decompensation because not enough pressure can be generated to mitigate the elevated left atrial pressures
and provide adequate blood flow across the stenotic valve.

(Choice A) Patients with acute pericarditis can go on to develop pericardial effusion and cardiac tamponade, with
associated dyspnea, orthopnea, and an enlarged cardiac silhouette on chest x-ray. However, this patient has no
history of recent infection or chest pain that would suggest acute pericarditis.

(Choice B) Interstitial lung disease can present with progressive dyspnea and persistent cough. However, chest x-
ray typically shows increased reticular or nodular interstitial markings without significant cardiac enlargement or left
main stem bronchus elevation.

(Choice C) Primary lung malignancy can present with symptoms of fatigue, chest pain, hemoptysis, weight loss,
progressive dyspnea, and persistent cough. However, it is less likely in the absence of a smoking history and is
less likely to explain cardiac enlargement on chest x-ray.

(Choice E) Sarcoidosis is a systemic granulomatous disease that may be asymptomatic or present with fatigue,
weight loss, cough, dyspnea, and chest pain. Chest x-ray usually shows bilateral hilar adenopathy with or without
parenchymal reticular opacities.

Educational objective:
Mitral stenosis usually occurs in the setting of rheumatic heart disease and presents with exertional dyspnea,
cough, and orthopnea. Left atrial dilation is typical and may be recognized by elevation of the left main stem
bronchus on chest x-ray. Atrial fibrillation is a common complication and can precipitate acute decompensated
heart failure in previously well-compensated patients.
Reference
• Rheumatic heart disease.

• Mitral stenosis.
Question #338

A 62-year-old woman comes to the office due to a nonproductive cough that is "quite disturbing." The cough has
been present for 2 months and is accompanied by a tickling or scratching sensation in the throat. The patient
cannot identify a provoking factor for the cough, which occurs mostly during the day. She has no fever, rhinorrhea,
or chest pain. The patient has had no changes in appetite or weight, and her lifestyle is mostly sedentary. She has
difficult-to-control hypertension. Her medications, which were last readjusted 3 months ago, include chlorthalidone,
ramipril, and amlodipine. The patient, who emigrated from China 30 years ago, does not use tobacco, alcohol, or
illicit drugs. Her temperature is 36.8 C (98.2 F), blood pressure is 130/90 mm Hg, pulse is 80/min, and respirations
are 16/min. The patient's conjunctivae are normal, oropharynx is clear without erythema or cobblestoning, lungs
are clear without wheezes or crackles, and heart sounds are normal. She has no peripheral edema. Complete
blood count and chest x-ray are normal. Which of the following is the most likely cause of this patient's current
symptoms?

A) Airway inflammation and reversible obstruction

B) Destruction and permanent dilation of airways

C) Impaired metabolism of kinins and substance P

D) Laryngeal stimulation by food and liquids

E) Pulmonary venous hypertension

F) Stimulation of cough receptors by nasal secretions


Explanation
Correct Answer:

C) Impaired metabolism of kinins and substance P

This patient's chronic nonproductive cough is most likely an adverse effect of ACE inhibitor (eg, ramipril)
therapy, which occurs in 5%-20% of patients. The cough typically begins within a week of drug initiation or dosage
increase but can be delayed for months. The pathophysiologic mechanism of the cough is not completely
understood. ACE is usually involved in the metabolism of kinins and substance P, so ACE inhibitor therapy
leads to accumulation of these molecules. Kinin accumulation is thought to cause bronchial irritation through
increased production of prostaglandins. ACE inhibition also activates the arachidonic acid pathway, which
may contribute to cough through increased levels of thromboxane, leading to bronchoconstriction.

Women and individuals of Chinese descent have a higher chance of developing cough due to ACE inhibitor therapy,
and a tickling or scratching sensation in the throat (as reported by this patient) is a common feature. If the cough is
significantly bothersome, discontinuing the drug typically results in resolution of the cough within days.

(Choice A) Airway inflammation and reversible obstruction occur in asthma, which is a common cause of chronic
cough. Ramipril use and the absence of wheezing make ACE inhibitor-induced cough more likely.

(Choice B) Destruction and permanent damage of the conducting airways occur in bronchiectasis, which is
characterized by a cough productive of large amounts of sputum. Nonproductive cough makes bronchiectasis very
unlikely.

(Choice D) Laryngeal stimulation by regurgitated foods and liquids occurs in patients with laryngopharyngeal
reflux, which is a common cause of chronic cough. Laryngopharyngeal reflux is unlikely in the absence of other
suggestive signs and symptoms (eg, oropharyngeal erythema, heartburn, hoarseness).

(Choice E) Pulmonary venous hypertension occurs in congestive heart failure and often presents with cough due
to pulmonary edema. The absence of peripheral edema and crackles on lung auscultation makes heart failure less
likely. Additionally, cough due to heart failure is typically productive rather than nonproductive.
(Choice F) The stimulation of cough receptors by nasal secretions occurs in upper airway cough syndrome
(postnasal drip), which is a common cause of chronic cough. The absence of oropharyngeal cobblestoning on
physical examination and lack of rhinorrhea make this condition less likely.

Educational objective:
Chronic nonproductive cough is a common side effect of ACE inhibitors that is likely due to increased circulating
levels of kinins, substance P, prostaglandins, and thromboxane.

Reference
• A prospective study of frequency and characteristics of cough during ACE inhibitor treatment.
Question #339

A 61-year-old man is evaluated due to progressive exertional dyspnea and decreased exercise tolerance. He is no
longer able to walk at a brisk pace or jog gently. He has also noticed some bilateral leg swelling. The patient has a
40-pack-year history but no illicit drug use or occupational exposure. Examination shows increased anteroposterior
diameter of the chest. Mild neck accessory muscle use is evident when the patient becomes dyspneic while
speaking. Breath sounds are decreased throughout. Heart sounds are distant. The abdomen is soft and
nontender. Mild pitting edema of the calves is present. Upright posteroanterior and lateral chest x-rays are shown
in the images below:
Which of the following is expected to be present in this patient?

A) Decreased diaphragm force

B) Decreased lung compliance

C) Increased chest wall recoil

D) Increased inspiratory flow

E) Increased left ventricular preload


Explanation
Correct Answer:

A) Decreased diaphragm force


This patient with a heavy smoking history has exertional dyspnea, distant heart sounds, and evidence of
hyperinflation on chest x-ray. This presentation is highly suggestive of emphysema, an obstructive lung disease,
in which lung emptying is slowed due to loss of elastic recoil and premature expiratory airway closure. The
resultant air trapping and hyperinflation can be recognized by an increased anteroposterior diameter ("barrel chest")
and oblique sternophrenic angle (≥90 degrees, normal: <90 degrees ), indicating a flattened diaphragm.

Normally, the diaphragm assumes a curved, dome-shaped configuration. During contraction, it flattens and
descends toward the abdomen, resembling a sheet being pulled taut. This produces a vacuum in the pleural cavity,
allowing for inspiratory flow to begin. When hyperinflation is present (eg, emphysema), the already-flattened
diaphragm has difficulty contracting further to produce force. Therefore, it is less capable of generating
inspiratory flow (Choice D).

Because the flattened diaphragm is disadvantaged, the accessory muscles of inspiration (sternocleidomastoid,
serratus, latissimus) are recruited to supply additional power. The work of breathing is the total muscular effort
required to move the tidal volume into and out of the lungs. Because of the poor movement of the diaphragm and
activation of these accessory muscles, more work is required to yield the same tidal respiration. Obstructive lung
disease can be summarized as follows: higher work to get the next breath in due to difficulty getting the previous
breath out (hyperinflation).

(Choice B) Lung compliance (change in volume per change in pressure) is the inverse of elastance. In
emphysema, loss of alveolar elastin leads to decreased elastance and increased lung compliance ("loose and
baggy" lungs).

(Choice C) The chest wall normally springs outward to balance the inward elastic recoil of the lungs. Because
inward elastic recoil is reduced in emphysema, the respiratory system (lung and chest wall unit) equilibrates to a
higher volume. At this higher volume, both the (outward) thoracic wall recoil and the (inward) elastic recoil are
reduced by equal amounts.

(Choice E) Hyperinflation is associated with a rise in intrathoracic pressure, limiting venous return to the right side
of the heart. Emphysema also causes capillary bed destruction, leading to increased pulmonary vascular
resistance. As a result, left ventricular filling (ie, preload) is reduced, rather than increased, in patients with
emphysema. This leads to a clinical picture of right ventricular dysfunction (cor pulmonale), causing peripheral
lower extremity edema.
Educational objective:
Obstructive lung disease leads to slower lung emptying, hyperinflation, and diaphragm flattening. The work of
breathing is increased because the diaphragm has difficulty contracting further to generate inspiratory force.
Question #340

A 20-year-old woman comes to the office for follow-up after being discharged from the hospital 2 weeks ago. The
patient has been hospitalized twice in the past 3 months for right lower lobe pneumonia; treatment with antibiotics
resolved her symptoms on both occasions, and she currently feels well. Family history is not significant. She has
never smoked cigarettes. Temperature is 37.5 C (99.5 F), blood pressure is 120/76 mm Hg, pulse is 88/min, and
respirations are 15/min. Oxygen saturation is 96% on ambient air. BMI is 24 kg/m2. Oropharynx is clear, and nasal
passages are normal. There is no lymphadenopathy. Localized wheezes are present in right lower lobe. There is
no clubbing. Complete blood count with differential is normal. Chest radiograph reveals a persistent right lower
lobe infiltrate. Which of the following is the most likely diagnosis in this patient?

A) Bronchial carcinoid tumor

B) Polyarteritis nodosa

C) Pulmonary tuberculosis

D) Squamous cell lung cancer


Explanation
Correct Answer:

A) Bronchial carcinoid tumor

Bronchial carcinoid tumor

• Most common lung cancer in adolescents/young adults


Epidemiology
• Neuroendocrine tumor derived from bronchial Kulchitsky cells

• Proximal airway obstruction (eg, dyspnea, wheezing, cough)


• Recurrent pneumonia distal to obstruction
Manifestations
• Hemoptysis
• Carcinoid syndrome less common than with midgut carcinoid

• Chest imaging: contrast enhanced (vascular) tumor with endobronchial component


Diagnosis
• Bronchoscopy with biopsy

This patient has had 2 episodes of right lower lobe pneumonia over a short time frame. Both infections
symptomatically improved with antibiotic therapy, but she continues to have wheezing and a pulmonary infiltrate in
that region. Recurrent pneumonia in the same anatomic portion of the lung is usually caused by a structural
lesion inside (eg, foreign body, bronchial stenosis) or around (eg, mediastinal lymphadenopathy, neoplasm) the
proximal bronchus. Obstruction of the bronchus impairs clearance of fluid from the alveoli, leading to microbial
aggregation and a high risk of infection.
Although bronchial carcinoid tumors account for 1%-2% of total lung malignancies, they are the leading cause of
lung cancer in adolescents and young adults (particularly in those who do not smoke). Tumors are typically slow-
growing and round and generally form in the proximal airway; they can present with wheezing, dyspnea, cough
due to bronchial obstruction or hemoptysis due to tumor bleeding from hypervascularity. Compared to
gastrointestinal carcinoid tumors, bronchial carcinoid tumors produce far less serotonin/vasoactive amines and do
not often cause carcinoid syndrome.

Recurrent pneumonia can also be seen with systemic disorders (eg, immunodeficiency, cystic fibrosis); however,
patients with systemic causes of recurrent pneumonia often have pneumonia in different anatomic regions and
infections in other portions of the respiratory tract (eg, sinusitis).

(Choice B) Polyarteritis nodosa typically causes gastrointestinal (eg, mesenteric ischemia), renal (eg, infarction),
dermatologic (eg, livedo reticularis), and neurologic (eg, mononeuritis multiplex) manifestations. Lung involvement
is extremely rare.

(Choice C) Pulmonary tuberculosis can cause fever and pulmonary infiltrate; however, it requires multidrug
antibiotic therapy over several months for appropriate treatment (unlike in this patient).

(Choice D) Squamous cell lung cancer can cause bronchial obstruction with recurrent pneumonia. However, most
cases occur in patients who smoke and arise in those with advanced age. This young patient who does not smoke
cigarettes is unlikely to have squamous cell cancer.

Educational objective:
Bronchial carcinoid is the most common lung cancer in adolescents and young adults. It typically presents with
proximal airway obstruction, leading to dyspnea, wheezing, and recurrent pneumonia in the same lobe of the lung.
Carcinoid syndrome is much less common than with gastrointestinal carcinoid tumors.
Question #341

A 68-year-old woman comes to the office due to gradually worsening shortness of breath. Over the past several
months, she has bothersome dyspnea on exertion, now developing with even light housework. She also reports
cough in the mornings that is productive of minimal white sputum. The patient has had no chest pain or fever. She
smoked approximately 10 cigarettes daily from age 18 to 50. Blood pressure is 128/76 mm Hg, pulse is 80/min,
and respirations are 14/min. Oxygen saturation is 94% on room air. On physical examination, the patient is
breathing comfortably without using accessory muscles of respiration. The trachea is central, and jugular venous
pressure is normal. Lung auscultation reveals bilateral vesicular breath sounds with occasional rhonchi that clears
after the patient is asked to cough. Heart sounds are normal. Chest x-ray reveals no abnormalities. Pulmonary
function testing shows that FEV1 is 65% of predicted, and the FEV1/FVC ratio is 50%. These values are unchanged
after administration of albuterol. Which of the following is the best next step in management of this patient?

A) Inhaled muscarinic antagonist

B) Low dose inhaled corticosteroid

C) Oral leukotriene antagonist

D) Pulmonary rehabilitation

E) Short course of oral prednisone


Explanation
Correct Answer:

A) Inhaled muscarinic antagonist


This patient has a significant smoking history, chronic respiratory symptoms (eg, dyspnea, productive cough), and
fixed airflow limitation (ie, FEV1/FVC ratio <70%, unchanged after albuterol), consistent with chronic obstructive
pulmonary disease (COPD).

Selection of initial bronchodilator therapy for stable COPD depends on 2 parameters:


• Symptom severity is graded using validated instruments such as the COPD Assessment Test. These
scales measure the impact of respiratory symptoms on daily life (higher severity: dyspnea during light
housework).

• Disease risk is assessed by the frequency of COPD acute exacerbations and/or hospitalizations (higher
risk: ≥1 hospitalization or ≥2 exacerbations [requiring oral corticosteroids] annually).

Although pulmonary function tests are crucial for COPD diagnosis, FEV1 is not a primary consideration when
deciding initial therapy because it correlates poorly with severity and risk. The 2-by-2 strata of severity and risk
partitions patients into 4 groups (groups A to D). This patient has higher symptom severity with lower disease risk
(group B).

Except for low risk/low severity patients (ie, group A), the preferred initial therapy for patients with COPD includes a
daily long-acting muscarinic antagonist (LAMA). LAMAs (eg, tiotropium) block airway M3 receptors to stabilize
against bronchoconstriction and reduce mucus hypersecretion. LAMAs are superior to long-acting beta agonists
(LABAs) in COPD.

In addition to bronchodilators, patients should be assessed for smoking cessation, supplemental oxygen, and
influenza and pneumococcal vaccination.

(Choice B) Inhaled corticosteroids (ICSs), such as budesonide, are preferred for asthma (reversible obstruction) or
as part of triple therapy (LABA + LAMA + ICS) for refractory COPD. ICSs are not first line for COPD because
LAMAs are more effective and have fewer adverse effects. ICSs increase pneumonia risk in COPD due to local
pulmonary immunosuppression.

(Choice C) Leukotriene antagonists (eg, montelukast) are an adjuvant therapy for asthma (reversible obstruction
with albuterol administration), rather than COPD. They are especially helpful for aspirin-exacerbated respiratory
disease (triad of asthma, nonsteroidal anti-inflammatory drug sensitivity, and nasal polyposis) due to the central role
of leukotrienes in this condition.

(Choice D) All patients with COPD should be screened for pulmonary rehabilitation, a supervised exercise program
that improves aerobic tolerance. It is indicated for patients with deconditioning and life-limiting dyspnea (eg, light
housework or at rest) despite optimal medical therapy.
(Choice E) Oral corticosteroids are indicated for acute exacerbations of COPD (increased wheezing, sputum, and
dyspnea) but not as initial therapy for stable disease.

Educational objective:
Daily long-acting muscarinic antagonists are the best initial therapy for stable chronic obstructive pulmonary disease
with higher symptom burden (ie, dyspnea on light exertion) and/or disease risk (history of exacerbations and
hospitalizations).
Question #342

A 65-year-old man complains of two years of persistent cough. He says that he coughs up whitish sputum almost
every morning on waking, and then continues coughing throughout the day. He also complains of exertional
shortness of breath that becomes disabling if he gets an upper respiratory infection. He has smoked one pack of
cigarettes daily for the past 40 years. Pulmonary function testing reveals a vital capacity that is 65% of his
predicted. Which of the following best explains this finding?

A) Alveolar-capillary membrane thickening

B) Decreased functional residual capacity

C) Air trapping during expiration

D) Decreased lung distensibility

E) Respiratory muscle fatigue


Explanation
Correct Answer:

C) Air trapping during expiration

This patient's history and presentation are classic for chronic obstructive pulmonary disease (COPD), with
components of both chronic bronchitis (chronic cough with sputum production) and emphysema (dyspnea).
Typically caused by cigarette smoking, COPD is marked by progressive decreases in the expiratory airflow rate,
which manifests as a forced expiratory volume in 1 second (FEV1) to forced vital capacity (FVC) ratio of less than
0.7. As airflow limitation increases, more air is trapped during expiration and the residual and total lung volumes
increase. Air trapping and airflow obstruction in severe disease also decrease the vital capacity (VC) as described
in this patient. An accompanying process is destruction of the alveolar-capillary membrane, possibly due to
excessive lysis of lung structural proteins.

(Choice A) The alveolar-capillary membrane is the site of respiratory gas exchange. This membrane is destroyed in
COPD.

(Choice B) Due to air trapping, COPD patients have increased functional residual and total lung capacities.

(Choice D) The alveolar-capillary membrane is destroyed in COPD, resulting in increased lung distensibility and
compliance. Lung distensibility is decreased in pulmonary fibrosis.

(Choice E) Respiratory muscle fatigue occurs in a number of diseases like botulism, Guillain-Barré syndrome,
amyotrophic lateral sclerosis, and myasthenia gravis. Respiratory muscle failure is not a hallmark of early COPD.

Educational objective:
COPD is characterized by progressive expiratory airflow limitation which causes air trapping, decreased VC and
increased total lung capacity. FEV1 is disproportionately decreased as compared to VC.
Question #343

A 45-year-old woman comes to the office due to frequent nighttime cough and wheezing over the past several
months. Lately, she has also experienced similar episodes during the daytime. Medical history includes
hypertension, hyperlipidemia, and obesity. Medications are amlodipine, lisinopril, and simvastatin. She has a
15-pack-year smoking history but quit 10 years ago. There is no personal or family history of lung disease. Blood
pressure is 130/84 mm Hg, pulse is 72/min, and respirations are 16/min. Pulse oximetry shows 95% on room air.
BMI is 31 kg/m2. Jugular venous pressure is 3 cm H2O above the sternal angle. The heart has a regular rate and
rhythm with no murmurs. The lungs show good air movement with expiratory wheezes and no crackles. Chest x-
ray is unremarkable. The results of pulmonary function testing are as follows:

Pulmonary
Pre-bronchodilator Post-bronchodilator
Forced vital capacity (FVC) (liters) 3.52 (86% of predicted) 3.69 liters (90% of predicted)
Forced expiratory volume in 1 second
2.28 (61% of predicted) 2.95 liters (84% of predicted)
(FEV1) (liters)
FEV1/FVC ratio (%) 58% 70%
Diffusing capacity of lung for carbon monoxide
94% of predicted (normal: >80%)
(DLCO) (mL/mmHg/min)

Which of the following is the primary pathogenesis of this patient's symptoms?

A) Gastric acid–induced laryngeal irritation

B) Hypertrophic occlusion of the pulmonary arteries

C) Impaired bradykinin degradation


D) Leukocyte-induced bronchoconstriction

E) Protease-induced alveolar destruction


Explanation
Correct Answer:

D) Leukocyte-induced bronchoconstriction
This former smoker has episodic dyspnea, wheezing, and cough. Pulmonary function testing shows reversible
airflow obstruction: FEV1/FVC ≤70% with positive bronchodilator response (≥12% increase in FEV1 or FVC). The
combination of her symptoms and spirometry is diagnostic of asthma.

Although asthma is typically diagnosed in younger individuals, a significant minority are diagnosed later in life.
Adult-onset asthma is characterized by a different immunophenotype with greater airway remodeling. Patients
have a positive bronchodilator response, but FEV1/FVC ratio may not fully normalize (eg, may not increase to
>70%).

Distinguishing asthma from chronic obstructive pulmonary disease (COPD) is a common clinical scenario,
particularly in adult ever smokers. Both diseases may show airflow obstruction, but patients with COPD have no
bronchodilator responsiveness; emphysematous COPD is associated with decreased DLCO due to protease-
induced tissue destruction leading to loss of alveolar surface area (Choice E).

Proper distinction guides the selection of initial inhaled therapy:

• The hallmark of asthma is leukocyte-induced bronchoconstriction. A combination of inhaled


corticosteroid (targets the leukocytes) plus beta agonist (targets the bronchoconstriction) is recommended
for asthma.

• The hallmark of COPD is goblet cell hyperplasia, mucus hypersecretion, and alveolar septal destruction. A
long-acting muscarinic antagonist (targets airway hypersecretion) is recommended for COPD.

(Choice A) Nocturnal cough and wheezing are also key symptoms of gastroesophageal reflux disease (GERD).
GERD can exacerbate asthma due to constant aspiration of acid fumes into the lower airways. GERD can also
cause laryngeal irritation, contributing to vocal cord dysfunction (VCD) that manifests as episodic dysphonia and
inspiratory stridor. However, neither GERD by itself nor GERD-induced VCD would cause expiratory airflow
obstruction.

(Choice B) Pulmonary arterial hypertension (PAH) can cause dyspnea, chest tightness, and wheezing due to
stretching of the pulmonary vessels and lung mechanoreceptors. It most frequently affects young adult women and
often masquerades as asthma for many years prior to diagnosis. However, PAH is associated with a low DLCO
value (decreased distal lung perfusion) and does not cause airflow obstruction.
(Choice C) ACE inhibitors (eg, lisinopril) cause decreased bradykinin clearance; this is a common cause of dry
cough, even among patients who have been taking ACE inhibitors chronically. However, these medications do not
induce bronchoconstriction, wheezing, or obstruction on pulmonary function testing. Despite the increased
bradykinin levels, ACE inhibitors are generally well tolerated in patients with asthma.

Educational objective:
Bronchodilator-responsive airflow obstruction (≥12% increase in FEV1 or FVC) suggests asthma, regardless of age.
Because late adult-onset asthma features greater airway remodeling, the FEV1/FVC ratio may not fully normalize.

Reference
• Bronchodilator reversibility in COPD.

• Adverse respiratory effect of acute ß-blocker exposure in asthma: a systematic review and meta-analysis of
randomized controlled trials.
Question #344

A 25-year-old man comes to the office due to a dry cough that has persisted since he contracted an upper
respiratory tract infection 10 days ago. The cough is worse at night and is not associated with nasal congestion,
facial pressure, sore throat, abdominal discomfort, or vomiting. Medical history is notable only for childhood
asthma, which the patient reports he "outgrew." All recommended vaccinations are up to date. The patient is
afebrile, and other vital signs are normal. Nasal and pharyngeal examinations are unremarkable. Scattered
wheezes are heard on lung auscultation. There are no heart murmurs. Which of the following is the best next step
in management of this patient?

A) Chest x-ray

B) Inhaled bronchodilator

C) Macrolide antibiotic

D) Penicillin antibiotic

E) Second-generation antihistamine
Explanation
Correct Answer:

B) Inhaled bronchodilator

Acute bronchitis

Etiology • Preceding respiratory illness (90% viral)

• Cough for >5 days to 3 weeks (± purulent sputum)


Clinical presentation • Absent systemic findings (eg, fever, chills)
• Wheezing or rhonchi, chest wall tenderness

• Clinical diagnosis, CXR only when pneumonia suspected


Diagnosis & treatment • Symptomatic treatment (eg, NSAIDs &/or bronchodilators)
• Antibiotics not recommended

CXR = chest x-ray; NSAIDs = nonsteroidal anti-inflammatory drugs.

This patient has a persistent, dry cough following an acute upper respiratory infection. He has mild wheezing on
examination, but clinical findings are otherwise unremarkable. This presentation is consistent with acute
bronchitis.

Acute bronchitis is a self-limited infection that can last for 3-4 weeks. More than 90% of cases are caused by
viruses (eg, rhinovirus, coronavirus, influenza); purulent yellow or green sputum can occur due to epithelial
sloughing and is not a sign of bacterial infection. Patients may also have mild dyspnea and chest wall discomfort
(from coughing). Crackles from airway secretions may be audible but typically clear with cough (unlike in
pneumonia). Fever is not typical and should raise suspicion for bacterial pneumonia.

Treatment of acute bronchitis is primarily supportive and may include throat lozenges and over-the-counter cough
suppressants (eg, dextromethorphan/guaifenesin). Patients with associated wheezing in the setting of underlying
pulmonary disease (eg, asthma) may benefit from inhaled bronchodilators (eg, albuterol); otherwise, these
medications are not generally recommended. Oral corticosteroids are not indicated.

(Choice A) Chest x-ray to rule out pneumonia is indicated for patients with acute cough associated with high fever,
tachycardia, tachypnea, or signs of lung consolidation on examination (eg, crackles, dullness). It should be
considered for elderly patients, in whom clinical signs of pneumonia may be subtle, and those with advanced lung
disease (eg, chronic obstructive pulmonary disease). However, minor wheezing in a young patient who is otherwise
clinically stable does not warrant chest imaging.

(Choice C) Macrolide antibiotics (eg, azithromycin) are indicated for treatment of community-acquired pneumonia
(eg, fever, tachypnea, lung crackles) or pertussis (eg, paroxysmal bouts of coughing, posttussive emesis). Although
macrolide-sensitive species (eg, Mycoplasma) are responsible for a minority of acute bronchitis cases, treatment
with antibiotics does not improve outcomes.

(Choice D) Penicillin antibiotics (eg, amoxicillin/clavulanate) are indicated for acute bacterial sinusitis that fails to
improve with symptomatic treatment. However, sinusitis is typically associated with nasal congestion, sinus
tenderness, and fever.

(Choice E) First-generation antihistamines (eg, diphenhydramine) are useful in combination with nasal
decongestants for acute upper respiratory symptoms, although they are associated with significant side effects (eg,
sedation). Second-generation antihistamines (eg, loratadine) alone are less effective for upper respiratory
symptoms and are not indicated for acute bronchitis.

Educational objective:
Acute bronchitis is a self-limited illness usually caused by a viral infection. Treatment is supportive, although
patients with associated wheezing due to underlying asthma may benefit from an inhaled bronchodilator (eg,
albuterol). Antibiotics are not helpful and should be avoided.

Reference
• Beta2-agonists for acute cough or a clinical diagnosis of acute bronchitis.
Question #345

A 32-year-old man comes to the office due to 3 days of fever, malaise, and cough productive of clear sputum. He
has had no nasal congestion, rhinorrhea, sore throat, or chest pain. The patient has a history of mild, intermittent
asthma and seasonal allergic rhinitis. He does not use tobacco but drinks alcohol occasionally. His 2-year-old son
had fever, cough, and rhinorrhea last week. Temperature is 38.3 C (101 F), blood pressure is 120/80 mm Hg, pulse
is 92/min, respirations are 20/min, and pulse oximetry is 96% on room air. The oropharynx is normal, and palpation
of the neck shows no cervical lymphadenopathy. Lung auscultation reveals crackles at the right lung base and
occasional expiratory wheezing. Heart sounds are normal. Which of the following is the best next step in
management of this patient?

A) Azithromycin only

B) Ceftriaxone and azithromycin

C) Chest x-ray

D) Cold agglutinin test

E) Rapid influenza test

F) Sputum Gram stain and culture

G) Symptomatic treatment with follow-up in 3 days


Explanation
Correct Answer:

C) Chest x-ray

This patient has fever, malaise, and cough productive of clear sputum. Although these findings are often seen in
viral upper respiratory infection, most patients have concurrent upper airway symptoms (eg, rhinorrhea, coryza,
sore throat). In addition, the presence of crackles at the right lung base indicate that the alveoli in this area are
collapsed by fluid or exudate. This raises concern for pneumonia, a potentially life threatening infection that
requires further evaluation with chest x-ray for diagnosis.

CAP is a pulmonary parenchymal infection that may be caused by bacterial (majority), viral (30%), or fungal
pathogens. Symptoms often develop acutely and frequently include fever, cough, pleuritic chest pain, and
dyspnea. Tachycardia, tachypnea, and pulmonary auscultation abnormalities (eg, focal crackles) may be present.
Clinical and physical findings are notoriously poor at predicting the presence of pneumonia (<50% sensitivity).
Therefore, even though patients are sometimes diagnosed clinically in practice, guidelines require a lobar,
interstitial, or cavitary infiltrate on chest imaging (usually chest x-ray) for confirmation.

Patients with a pulmonary infiltrate are generally treated with empiric antibiotics because rapid differentiation
between a viral and bacterial etiology is often difficult; in addition, antibiotics reduce risk of secondary bacterial
pneumonia in the setting of primary viral lung infection (Choice G). The presence of a focal infiltrate makes viral
bronchitis much less likely than CAP; therefore, symptomatic treatment would not be appropriate.

(Choices A and B) Azithromycin can be used for the outpatient treatment of CAP when local resistance rates in
Streptococcus pneumoniae are <25%. Ceftriaxone plus azithromycin is first-line for hospitalized patients (not in
intensive care). However, diagnosis with chest x-ray is required prior to treatment.

(Choice D) Mycoplasma pneumoniae is the most common cause of atypical pneumonia (indolent fever, malaise,
and cough) and frequently results in the formation of cold agglutinins (not routinely used for diagnosis). However, M
pneumoniae is less likely to cause focal findings on lung examination, can be accompanied by extrapulmonary (eg,
gastrointestinal) manifestations, and usually causes indolent rather than acute symptoms.

(Choice E) Influenza tends to cause the abrupt onset of systemic (eg, fever, malaise, myalgias) and upper
respiratory (eg, rhinorrhea, sore throat) symptoms. This patient has no upper respiratory symptoms.

(Choice F) Sputum and blood cultures are typically not required in the outpatient setting as empiric oral antibiotics
(eg, amoxicillin, levofloxacin) are usually curative. An infiltrate on chest x-ray confirms CAP.

Educational objective:
The diagnosis of community-acquired pneumonia requires the presence of a lobar, interstitial, or cavitary infiltrate
on chest imaging (eg, chest x-ray). Sputum and blood cultures are typically not required in the outpatient setting as
empiric oral antibiotics are almost always curative.

Reference
• Testing strategies in the initial management of patients with community-acquired pneumonia.
Question #346

A 56-year-old man is evaluated in the preoperative clinic 4 weeks prior to an elective right knee replacement for
severe osteoarthritis. Medical history is notable for hypertension and chronic obstructive pulmonary disease. The
patient uses inhaled bronchodilators, inhaled corticosteroids, and amlodipine. He has never required hospitalization
for respiratory-related issues. The patient is able to perform all basic activities of daily living without dyspnea and
was playing golf once a week prior to the knee pain limiting his activity. He previously smoked a pack of cigarettes
daily beginning at age of 21 but has cut down to 3 or 4 cigarettes per day over the last 2 years. Vital signs are
within normal limits. Oxygen saturation is 92% on room air. Lung auscultation shows slightly diminished breath
sounds with prolonged expiration but no wheezes or crackles. Heart sounds are normal. The abdomen is soft and
nontender with no organomegaly. There is no extremity edema. The right knee has crepitus and decreased range
of motion. Blood cell counts and serum chemistry studies are within normal limits. Which of the following
interventions is most appropriate to decrease the risk of postoperative pulmonary complications in this patient?

A) Immediate smoking cessation

B) Long-term oxygen therapy

C) Preoperative moxifloxacin

D) Systemic glucocorticoids

E) Theophylline therapy
Explanation
Correct Answer:

A) Immediate smoking cessation

This patient with chronic obstructive pulmonary disease (COPD) is at increased risk for postoperative pulmonary
complications, including pneumonia and respiratory failure requiring mechanical ventilation. Medical optimization
of COPD symptoms (eg, dyspnea, wheezing) prior to surgery helps reduce this risk. Active smoking is also an
independent risk factor for postoperative pulmonary complications, and this effect is amplified in patients with
underlying lung disease.

Even a small amount of active smoking (eg, 3 or 4 cigarettes per day) is associated with increased postoperative
pulmonary risk. In all patients anticipating elective surgery, immediate smoking cessation is recommended as
those who quit smoking >4-8 weeks prior to surgery substantially decrease their postoperative pulmonary risk.
Lesser durations of preoperative smoking cessation do not reduce the risk, likely because the airway inflammation
and increased bronchial mucus production induced by smoking requires some time to improve.

(Choice B) Long-term oxygen therapy improves mortality in patients with COPD and significant chronic hypoxemia
(ie, resting arterial oxygen saturation ≤88%); however, it has not been shown to benefit patients with lesser degrees
of hypoxemia.

(Choice C) There is no evidence that preoperative antibiotics active against respiratory pathogens (eg,
moxifloxacin) decrease the risk of pneumonia, COPD exacerbation, or other postoperative pulmonary
complications.

(Choice D) Systemic glucocorticoids are indicated in an acute exacerbation of COPD, but preoperative use has not
been shown to decrease the risk of postoperative pulmonary complications.

(Choice E) Theophylline is sometimes used as an additional therapy in patients with COPD who have persistent
symptoms despite smoking cessation and optimal usage of inhaled bronchodilators and corticosteroids. However, it
is not indicated in this patient whose symptoms are controlled by his current therapy.
Educational objective:
Active smoking increases the risk of postoperative pulmonary complications (eg, pneumonia, respiratory failure
requiring mechanical ventilation), especially in patients with underlying lung disease (eg, chronic obstructive
pulmonary disease). Smoking cessation is recommended >4-8 weeks prior to elective surgery as it has been
shown to reduce the risk of postoperative pulmonary complications.

Reference
• A prospective survey of the incidence of postoperative pulmonary complications.

• Impact of smoking on perioperative outcomes after major surgery.


Question #347

A 30-year-old family physician moves from Florida to a town in the Colorado mountains to establish a medical
practice. On the night of arrival, she experiences headache and has difficulty falling asleep. The next day she
develops fatigue, nausea, dizziness, and mild dyspnea on exertion. She has no chronic medical conditions.
Temperature is 37.2 C (99 F), blood pressure is 116/68 mm Hg, pulse is 90/min, and respirations are 20/min. Pulse
oximetry shows 90% on ambient air. Medication with which of the following effects is most appropriate to improve
her symptoms?

A) Direct suppression of the respiratory center

B) Increased erythrocyte levels of hemoglobin F

C) Increased sensitivity of peripheral chemoreceptors

D) Increased urinary excretion of HCO3−

E) Normalization of the alveolar-arterial oxygen gradient


Explanation
Correct Answer:

D) Increased urinary excretion of HCO3−


This patient is experiencing symptoms of high-altitude illness (HAI), including fatigue, nausea, headache,
lightheadedness, and dyspnea. These symptoms most commonly occur within 1-2 days of ascent to an elevation
>2,500 m (~8,000 ft) and are driven by the hypoxemia that results from low partial pressure of inspired oxygen
(PiO2) at high altitude.

Hyperventilation is the most immediate and important physiologic adjustment to high altitude. Hypoxemia
stimulates peripheral chemoreceptors in the aorta and carotid body to increase minute ventilation, which helps
increase PaO2. However, this comes at the expense of increased CO2 removal and respiratory alkalosis, which
create a ceiling for the ventilatory response because central chemoreceptors sense the elevated blood pH and
stimulate negative feedback to limit ventilation. The kidneys compensate for the respiratory alkalosis by
increasing HCO3− excretion to decrease pH and increase the ventilatory ceiling, but this process requires
approximately 72 hours to optimize.

Acetazolamide, a carbonic anhydrase inhibitor, helps prevent and treat HAI by accelerating the increase in the
ventilatory ceiling. The drug blocks sodium bicarbonate (NaHCO3) reabsorption in the proximal tubule to increase
HCO3−excretion and promptly assist with metabolic compensation for respiratory alkalosis. The reduction in pH is
accelerated, alleviating central inhibition of the ventilatory response and facilitating the maintenance of adequate
PaO2 to reduce the symptoms of HAI.

(Choice A) Direct suppression of the respiratory center, as occurs with central respiratory depressants (eg, opioids,
benzodiazepines, alcohol), decreases the ventilatory response and worsens HAI.

(Choice B) Hydroxyurea increases erythrocyte hemoglobin F levels, which increases overall hemoglobin affinity for
O2 and shifts the hemoglobin dissociation curve to the left. However, one of the initial responses to high altitude is
increased production of 2,3-bisphosphoglycerate, which shifts the curve to the right to facilitate O2 unloading in the
tissues. In addition, months are required for hydroxyurea to take effect.

(Choice C) Peripheral chemoreceptors are already fully activated at high elevation in response to hypoxemia.
Further sensitization is not a potential mechanism for improving HAI.

(Choice E) Hypoxemia at high altitude results from low PiO2 leading to low alveolar partial pressure of oxygen.
The alveolar-arterial oxygen gradient is already normal and cannot be further reduced.
Educational objective:
Acetazolamide helps prevent and treat high altitude illness by increasing renal HCO3− excretion to reduce blood pH
and alleviate central chemoreceptor inhibition of the hypoxic ventilatory response.
Question #348

A 18-year-old woman comes to the office for follow-up of shortness of breath during exercise. The patient
competes in track and field. After about 30 minutes of practice, the patient has severe shortness of breath with
throat tightness and noisy breathing that requires her to stop working out. Symptoms resolve within 3 minutes after
stopping; she has no symptoms when she is not exercising. At her last appointment, the patient was prescribed
inhaled albuterol and oral montelukast but has had no improvement in symptoms. She has no other medical
conditions. Physical examination during exercise reveals normal heart sounds, normal lung sounds, and loud
inspiratory stridor. Which of the following is the most likely diagnosis?

A) Adenotonsillar hypertrophy

B) Choanal atresia

C) Paradoxical vocal fold motion

D) Performance anxiety

E) Tracheomalacia
Explanation
Correct Answer:

C) Paradoxical vocal fold motion

This young athlete has episodic dyspnea and noisy breathing during exercise. Empiric treatment for exercise-
induced asthma had no effect, and examination during exercise revealed inspiratory stridor (in contrast to
expiratory wheezing seen in asthma). This presentation is suspicious for paradoxical vocal fold motion (PVFM)
(also called vocal cord dysfunction).

The vocal cords normally abduct (open) during inspiration. During PVFM episodes, the vocal cords
inappropriately adduct (close) during inspiration, causing laryngeal obstruction. Episodes may be triggered by
strong smells, stress, or exercise. A classic scenario is a young, female athlete, particularly in the setting of a
transition to a higher level of competition. In addition to dyspnea and noisy breathing, patients typically report throat
tightness and impaired inspiration. Diagnosing PVFM during an acute episode of respiratory distress is difficult;
patients may have a history of repeated intubations. Patients may also pass out (due to hypoxia), after which the
vocal cords move normally, and respiratory distress immediately resolves.

Laryngoscopy showing vocal cord adduction during inspiration is diagnostic. Pulmonary function tests may show
flattening of the inspiratory loop but may be normal if done between episodes. Acute episodes are managed by
encouraging the patient to sniff or pant, which activates the posterior cricoarytenoid to abduct the vocal cords, or
with noninvasive positive-pressure ventilation. Endotracheal intubation ideally should be avoided. Long-term
treatment primarily consists of education and therapy with a speech-language pathologist.

(Choice A) Adenotonsillar hypertrophy may cause airway obstruction but typically manifests as nighttime snoring,
not exercise-induced inspiratory stridor.

(Choice B) Bilateral choanal atresia, or congenital blockage of the posterior nasal aperture, presents shortly after
birth with cyclic cyanosis that worsens when newborns cannot breathe through the mouth (eg, during feeding) and
improves when they do (eg, while crying). Unilateral choanal atresia commonly presents during childhood as
persistent, unilateral nasal obstruction and discharge.

(Choice D) Performance anxiety in athletes can manifest as fatigue and difficulty competing. However, it often
preferentially affects competitions rather than occurring during practice. In addition, this patient has objective
inspiratory stridor, which makes intermittent airway obstruction due to PVFM more likely.

(Choice E) Acquired tracheomalacia can present with noisy breathing during periods of increased respiratory
effort. However, stridor is expiratory (rather than inspiratory). In addition, this patient has no risk factors for
development of tracheomalacia (eg, intubation, recurrent pulmonary infections, relapsing polychondritis).

Educational objective:
Paradoxical vocal cord movement is an episodic, inappropriate closing of the vocal cords during inspiration, which
results in laryngeal obstruction and inspiratory stridor. It can present in elite athletes and often mimics exercise-
induced asthma.

Reference
• Vocal cord dysfunction in athletes: clinical presentation and review of the literature

• Multidisciplinary team clinic for vocal cord dysfunction directs therapy and significantly reduces healthcare
utilization.
Question #349

A 54-year-old man is being evaluated for shortness of breath and cough. His medical problems include rheumatoid
arthritis, hypertension, and a history of deep vein thrombosis and pulmonary embolism. He takes multiple
medications and has smoked for 30 years. BMI is 34 kg/m2. Examination shows dullness to percussion over the
right lower lobe. Breath sounds are increased, especially during expiration, over the right lung base compared to
the left. Cardiac examination shows regular rate and rhythm with normal S1 and S2. There is no murmur.
Moderate peripheral edema is present. Which of the following is the most likely cause of his shortness of breath
and lung examination findings?

A) Consolidation of the lung

B) Emphysema

C) Interstitial lung disease

D) Mucus plugging

E) Pleural effusion

F) Pneumothorax
Explanation
Correct Answer:

A) Consolidation of the lung

Pulmonary examination findings

Condition Breath sounds Tactile fremitus Percussion Mediastinal shift

Bronchovesicular (hilar),
Normal lung Normal Resonance None
vesicular (peripheral)

Consolidation
(eg, lobar Increased Increased Dullness None
pneumonia)

Pleural Away from effusion (if


Decreased or absent Decreased Dullness
effusion large)

Away from tension


Pneumothorax Decreased or absent Decreased Hyperresonance
pneumothorax
Atelectasis
Toward atelectasis (if
(eg, mucus Decreased or absent Decreased Dullness
large)
plugging)

This patient with shortness of breath, cough, and dullness to percussion with increased breath sounds over the
right lower lung field likely has lobar pneumonia causing focal lung consolidation. A normal lung is resonant to
percussion, and auscultation at the periphery demonstrates vesicular breath sounds that consist of a quiet
inspiratory phase and an almost inaudible expiratory phase. When a portion of the lung is consolidated (eg, lobar
pneumonia), the density of tissue/fluid increases and dullness to percussion is detected. In addition, sound
conducts more rapidly and efficiently through consolidated lung, resulting in increased intensity of breath sounds
and a more prominent expiratory component.

More rapid and efficient sound conduction also results in increased tactile fremitus (vibration generated by air
movement in the lungs) as well as egophony (sounds like the letter "A" when the patient says the letter "E") in
areas of lung consolidation. Crackles are also often heard.

(Choice B) The lungs are bilaterally hyperresonant to percussion in emphysema, and the intensity of breath
sounds is decreased. Wheezing is commonly present as well.

(Choice C) Patients with interstitial lung disease have lungs that are resonant to percussion and breath sounds of
normal or decreased intensity. Fine crackles are heard at the end of inspiration.

(Choice D) Mucus plugging decreases airflow in the affected part of the bronchial tree resulting in collapse of the
downstream lung parenchyma (atelectasis). Breath sounds will be absent in the affected area.

(Choice E) Unlike lung consolidation, pleural effusions are characterized by decreased breath sounds and
decreased tactile fremitus as the fluid in the thoracic cavity acts to insulate sound and vibration originating in the
lungs. As with lung consolidation, dullness to percussion is present due to increased tissue/fluid density compared
to normal air-filled lung tissue.

(Choice F) Air in the thoracic cavity also acts to insulate sounds originating from the lung; therefore, pneumothorax
is characterized by decreased breath sounds and decreased tactile fremitus. Hyperresonance to percussion occurs
due to a relatively lower density of air alone compared to normal air-filled lung tissue.

Educational objective:
Lung consolidation (eg, lobar pneumonia) presents with dullness to percussion, increased intensity of breath
sounds, and increased tactile fremitus.
Question #350

A 42-year-old woman with rheumatoid arthritis returns to the clinic for follow-up. The patient reports improvement in
joint pain after methotrexate therapy was begun 6 months ago. Over the past 2 months, she has felt increasingly
winded, causing her to stop jogging. She has a nonproductive cough and low-grade fevers. She has no other
medical conditions and is a lifelong nonsmoker. Temperature is 37.5 C (99.5 F), blood pressure is 120/70 mm Hg,
pulse is 88/min, and respirations are 18/min. Oxygen saturation is 89% on room air at rest. Jugular venous
pressure is normal. Lung auscultation reveals fine bibasilar inspiratory crackles. Heart sounds are normal.
Musculoskeletal examination shows mild interphalangeal synovitis. High-resolution CT scan of the chest reveals
patchy areas of consolidation and reticular thickening. A respiratory sample obtained by bronchoscopy yields
lymphocyte-predominant fluid negative for bacterial, viral, and fungal pathogens. Which of the following is the best
next step in management of this patient?

A) Discontinue methotrexate

B) Measure serum ACE levels

C) Measure serum IgE levels

D) Obtain lung biopsy

E) Start antifibrotic therapy


Explanation
Correct Answer:

A) Discontinue methotrexate

Methotrexate-induced lung injury

Pathogenesis • Idiosyncratic (ie, not dose dependent) hypersensitivity pneumonitis


& risk factors • Risk factors: rheumatoid arthritis, parenchymal lung disease

• Onset 1-12 months: pneumonitis → fibrosis (restrictive PFTs)


• CT scan with variable mix of patterns: inflammation (eg, GGO, consolidation) & fibrosis (eg,
Clinical
reticulation)
features
• BAL: lymphocytosis; peripheral blood: eosinophilia
• Trial of MTX cessation is diagnostic & therapeutic

BAL = bronchoalveolar lavage; GGO = ground-glass opacities; MTX = methotrexate; PFTs = pulmonary function
tests.

This patient taking methotrexate (MTX) for rheumatoid arthritis (RA) now has exertional dyspnea, hypoxemia, and
pulmonary infiltrates with potential fibrosis (ie, reticular thickening). Microbiologic workup is negative for infection.
In this context, methotrexate lung toxicity (aka MTX pneumonitis) becomes the primary diagnostic consideration.

MTX lung injury is an idiosyncratic (ie, not dose dependent) reaction. The mechanism is akin to hypersensitivity
pneumonitis, with granulomatous lymphocytic lung inflammation. It manifests with often overlapping patterns,
including acute inflammatory pneumonitis (ground glass), organizing pneumonia (consolidations), and rapidly
progressive pulmonary fibrosis (reticulation, honeycomb changes).

There is no specific diagnostic test for MTX pneumonitis. Infection must be excluded first, often through
bronchoalveolar lavage (BAL). Empiric antimicrobials are often administered while awaiting culture data. BAL fluid
typically demonstrates lymphocytic pleocytosis, a finding consistent with hypersensitivity. A provisional diagnosis
is made when an MTX cessation trial leads to clinical improvement. Systemic corticosteroids are sometimes
indicated if respiratory status does not improve with MTX cessation.

(Choice B) Pulmonary sarcoidosis (which is sometimes treated with MTX) can present with dyspnea, mixed
consolidative and fibrotic infiltrates, and lymphocytic BAL. However, sarcoidosis usually features hilar adenopathy
and peribronchovascular nodules. Serum ACE levels may track with disease severity but are neither sensitive nor
specific for diagnosis.

(Choice C) Allergic bronchopulmonary aspergillosis, associated with elevated IgE, can present with low-grade
fever and persistent cough. However, underlying asthma, bronchiectasis, and thick, mucinous sputum would be
expected.

(Choice D) Lung biopsy, a last resort in the diagnosis of interstitial lung disease, is reserved for rare occasions
when noninvasive studies are inconclusive (eg, atypical imaging features, non-improvement or deterioration despite
MTX cessation). This patient has a classic presentation of MTX pneumonitis based on history (exposure time
frame) and chest imaging, so monitoring clinical response after drug cessation is diagnostic. Therefore, lung biopsy
is not immediately required.

(Choice E) Antifibrotic agents (eg, pirfenidone) are used for idiopathic pulmonary fibrosis, which is characterized by
fibrotic interstitial pneumonia (ie, predominant honeycombing without consolidations) without an underlying
autoimmune disorder (ie, no RA). They have no established role in MTX pneumonitis, in which drug cessation
quells the hypersensitivity response, leading to spontaneous resolution.

Educational objective:
Methotrexate (MTX) pneumonitis is an idiosyncratic lung toxicity sharing similar pathogenesis to hypersensitivity
pneumonitis. Evaluation for pulmonary infection is obligatory. MTX cessation is both diagnostic and therapeutic.
Question #351

A 38-year-old woman comes to the office with cough and blood-tinged sputum. She developed malaise, nasal
congestion, sore throat, and dry cough 10 days ago. Most of her symptoms improved, but the cough persisted and
became productive of yellowish sputum. For the past 2 days, she has had several episodes of blood-tinged
sputum. The patient has no chest pain except the discomfort from coughing. Her appetite is normal, and she has
no weight loss. She feels anxious as she has never had blood in the sputum before. She is a nonsmoker, has no
other medical problems, and denies international travel. Her vaccinations are up to date. The patient's temperature
is 36.7 C (98 F), blood pressure is 132/80 mm Hg, pulse is 82/min, and respirations are 18/min. Pulse oximetry is
98% on room air. Her BMI is 29 kg/m2, and the oropharynx is clear. Scattered bilateral wheezes and crackles that
clear with coughing are heard on chest auscultation. Chest x-ray reveals clear lung fields. Which of the following is
the most appropriate next step in management of this patient?

A) Bronchoscopy

B) CT scan of the chest

C) Pulmonary function tests

D) Sputum acid-fast stain

E) Sputum cytology

F) Sputum Gram stain and culture

G) Symptomatic treatment only


Explanation
Correct Answer:

G) Symptomatic treatment only

Acute bronchitis

Etiology • Preceding respiratory illness (90% viral)

• Cough for >5 days to 3 weeks (± purulent sputum)


Clinical presentation • Absent systemic findings (eg, fever, chills)
• Wheezing or rhonchi, chest wall tenderness

• Clinical diagnosis, CXR only when pneumonia suspected


Diagnosis & treatment • Symptomatic treatment (eg, NSAIDs &/or bronchodilators)
• Antibiotics not recommended

CXR = chest x-ray; NSAIDs = nonsteroidal anti-inflammatory drugs.

This nonsmoking patient with symptoms of recent upper respiratory infection (URI) and persistent cough productive
of yellow, blood-tinged sputum likely has acute bronchitis.

Cough lasting >5 days is characteristic of acute bronchitis, and a viral URI is the usual cause. Sputum production
occurs in roughly half of patients; the typical yellow/purulent sputum is due to epithelial sloughing and is not a
sign of bacterial infection. Small amounts of blood in the sputum can occur due to inflammation and epithelial
damage. Mild dyspnea and chest wall discomfort are common, and physical examination often shows wheezing as
well as crackles that clear with cough, suggesting that secretions are easily mobilized (unlike in pneumonia). Fever
is not typical and, when present, should raise suspicion for bacterial pneumonia or influenza.

The illness is self-limiting (although cough and airway hypersensitivity may persist for weeks), and only
symptomatic treatment (eg, nonprescription pain relievers) is indicated. Antibiotics should generally be avoided
as they provide no significant benefit (including in acute bronchitis due to Mycoplasma species) and are associated
with adverse effects.

(Choices A and B) Chest CT and flexible bronchoscopy can be useful in the diagnosis and treatment of recurrent
or large-volume hemoptysis. This patient with small-volume hemoptysis in the setting of a URI, unremarkable chest
x-ray, and no risk factors for malignancy (eg, smoking) or bronchiectasis (eg, recurrent infection) requires no further
investigation.

(Choice C) Patients with chronic cough may be evaluated with pulmonary function testing if asthma or chronic
obstructive pulmonary disease is suspected. Cough due to acute bronchitis is usually self-limiting, and pulmonary
function testing is not indicated.

(Choice D) Sputum acid-fast stain is used to diagnose tuberculous infection in patients with chronic cough and
constitutional symptoms such as weight loss or night sweats. An unremarkable chest x-ray and absence of risk
factors (eg, international travel) make tuberculosis unlikely.

(Choice E) Sputum cytology can occasionally help in the diagnosis of lung cancer, but sensitivity is poor. Lung
cancer is very unlikely in this young patient with no smoking history.

(Choice F) Most cases of acute bronchitis are viral, and sputum culture is not recommended unless pneumonia is
suspected. The absence of fever and consolidation on chest x-ray makes pneumonia very unlikely.

Educational objective:
Acute bronchitis is a common cause of cough that may be productive of purulent, blood-tinged sputum. Upper
respiratory viral infection is the typical etiology, and symptoms are usually self-limiting. Symptomatic treatment and
close clinical follow-up are the best management strategies.
Reference
• Diagnosis and treatment of acute bronchitis.

• Hemoptysis: diagnosis and management.


Question #352

A 34-year-old man comes to the office to reestablish routine healthcare. The patient has a 9-year history of asthma,
for which he uses an inhaler containing a combination of both inhaled corticosteroid (ICS) and long-acting beta
agonist (LABA) as needed. For the past 2 years, he has been using this combination inhaler about 3 times per
week. His asthma symptoms wake him from sleep approximately once every week. The patient has seasonal
allergies to ragweed, but he has no other medical conditions and takes no other medications. He demonstrates
excellent inhaler technique in the office, with good synchronization and breath holding. The patient does not use
tobacco, alcohol, or illicit drugs. Family history is significant for asthma in his grandfather. Vital signs are normal.
Lung auscultation indicates normal breath sounds without wheezing. Heart sounds are normal. Which of the
following is the most appropriate next step in management of this patient?

A) Add a long-acting muscarinic antagonist daily

B) Add an oral systemic corticosteroid daily

C) Continue his current ICS/LABA combination inhaler as needed

D) Perform allergy desensitization

E) Schedule his ICS/LABA combination inhaler regimen daily


Explanation
Correct Answer:

E) Schedule his ICS/LABA combination inhaler regimen daily


This patient has persistent asthma, diagnosed when any of the following criteria are present during the week:
• Nocturnal waking with asthma on any night (often the earliest indicator of poor disease control)
• Symptoms requiring reliever inhaler use on most days
• Reduced FEV1 (while the patient is taking bronchodilator therapy)

Many patients may be falsely reassured that their asthma is well controlled because their symptoms do not interfere
with daily life and are alleviated with acute reliever therapy alone. Therefore, they may self-treat intermittently
despite having persistent asthma, which leads to chronic undermedication and poor long-term control.
Undertreatment of persistent asthma is associated with greater lung function decline and a higher likelihood of
severe acute exacerbations (Choice C).

Patients with persistent asthma should be treated with a scheduled combination inhaler that contains both an
inhaled corticosteroid (ICS) and a long-acting fast-onset beta agonist (formoterol preferred). Clinical response
should be reassessed using validated tools, such as the Asthma Control Test questionnaire. If the asthma remains
uncontrolled (eg, persistent nocturnal awakenings), the next step would be to increase the dose of the ICS
component.

(Choice A) Long-acting antimuscarinic agents (eg, tiotropium) are indicated for step-up therapy in patients with
poorly controlled persistent asthma despite scheduled high-dose ICS-formoterol.

(Choice B) Short courses of oral prednisone are indicated in acute asthma exacerbations (eg, wheezing) and
cases of poorly controlled symptoms. Chronic use of oral prednisone is not recommended due to significant
adverse effects (eg, glucose intolerance, osteopenia, adrenal suppression).

(Choice D) Allergy desensitization is an adjunctive controller treatment that can benefit a minority of patients with
asthma and rhinitis driven by a specific aeroallergen (eg, immunotherapy with house dust mite proteins). It is less
effective for patients, such as this one, who have seasonal allergies but experience persistent asthma symptoms.
Immunotherapy is not a substitute for optimization of inhaled ICS–beta agonist therapy; in the absence of
appropriate asthma control, allergen challenge may cause asthma to worsen.

Educational objective:
Persistent asthma is treated with a step-up in asthma therapy involving scheduled inhaled corticosteroids and long-
acting (fast-onset) beta agonists.
Question #353

A 64-year-old man comes to the office due to 2 months of weight loss, dry cough, and progressive pain in his right
arm. He was treated for community-acquired pneumonia a year ago. The patient has a history of type 2 diabetes
mellitus and hypertension. He used to travel internationally for work but retired several years ago. The patient has
a 30-pack-year smoking history but quit 5 years ago. He drinks alcohol occasionally. Vital signs are normal.
Physical examination is unremarkable. Imaging of the chest is shown below.
Which of the following is most likely responsible for this patient's symptoms?

A) Aspergilloma

B) Bronchial carcinoid

C) Cryptogenic organizing pneumonia

D) Mesothelioma

E) Superior pulmonary sulcus tumor

F) Tuberculosis
Explanation
Correct Answer:

E) Superior pulmonary sulcus tumor

Common manifestations of superior pulmonary sulcus tumor

• Shoulder pain
• Horner syndrome (invasion of paravertebral sympathetic chain/stellate ganglion)
◦ Ipsilateral ptosis, miosis, enophthalmos & anhidrosis
• Neurologic symptoms in the arm (invasion of C8-T2 nerves)
◦ Weakness/atrophy of intrinsic hand muscles
◦ Pain/paresthesia of 4th/5th digits & medial arm/forearm
• Supraclavicular lymphadenopathy
• Weight loss

This patient with weight loss, dry cough, and arm pain has imaging evidence of a superior pulmonary sulcus
(Pancoast) tumor. Pancoast tumors occur in the apical pleuropulmonary groove and are generally caused by
adenocarcinoma or squamous cell carcinoma of the lung. Smoking is the single greatest risk factor.
Manifestations generally arise when the tumor compresses or invades adjacent structures, leading to:

• Shoulder pain due to invasion of the brachial plexus, pleura, ribs, or vertebral bodies. Because shoulder
pain is common and often due to a benign cause (eg, osteoarthritis), the diagnosis of Pancoast tumor is
frequently delayed.

• Horner syndrome (eg, miosis, ptosis, anhidrosis) due to invasion of the paravertebral sympathetic chain or
stellate ganglion.
• Neurologic manifestations (eg, weakness/atrophy of hand muscles, paresthesia/pain) in an ulnar distribution
due to invasion of the inferior (C8-T1) portion of the brachial plexus

• Hoarseness due to recurrent laryngeal nerve invasion

• Superior vena cava syndrome

Weight loss is also common; pulmonary symptoms (eg, cough) do not typically occur until the tumor is large (as in
this patient).

(Choices A and F) Reactivation tuberculosis can cause nodules or cavitary lesions in the lung apex that may be
colonized by Aspergillus species, creating an aspergilloma. However, arm pain would be very atypical because
invasion into adjacent structures is highly uncommon in immunocompetent patients. In addition, most patients have
fever.

(Choice B) Bronchial carcinoid tumors are rare and usually occur centrally in the central airways, not the lung
apex. Most cases present with hemoptysis, cough, or dyspnea from denudation/obstruction of the airway.

(Choice C) Cryptogenic organizing pneumonia causes dry cough and systemic symptoms that last for months.
Cross-sectional chest imaging usually demonstrates ground-glass infiltrates (typically bilateral) rather than a solid
mass.

(Choice D) Mesothelioma is a malignant neoplasm derived from the mesothelial lining of the thoracic cavity. It is
strongly linked to asbestos exposure and usually presents with cough, chest pain, and dyspnea. Chest imaging
typically reveals pleural thickening with an effusion.

Educational objective:
Superior pulmonary sulcus (Pancoast) tumors arise in the apical pleuropulmonary groove and typically present with
shoulder pain, Horner syndrome, or neurologic manifestations in the arm due to invasion of adjacent structures (eg,
brachial plexus, paravertebral sympathetic chain).

Reference
• Pancoast tumors: characteristics and preoperative assessment.
Question #354

A 60-year-old man comes to the office due to shortness of breath over the past 5 months. He also has a chronic
cough that is more pronounced in the morning and is productive of whitish sputum. The patient's shortness of
breath initially occurred only with heavy exertion, but he now experiences dyspnea with daily tasks. He has no
fever, chills, hemoptysis, chest pain, or orthopnea. Medical history includes hypertension, benign prostatic
hyperplasia, and major depressive disorder. The patient has smoked cigarettes for the past 40 years and has
recently reduced his consumption from one pack to a half pack per day. Temperature is 37.2 C (99 F), blood
pressure is 130/86 mm Hg, pulse is 78/min, and respirations are 18/min. Pulse oximetry shows 96% oxygen
saturation on room air and 92% after a short brisk walk. BMI is 18.4 kg/m2. Examination shows a thin man in no
respiratory distress. Breath sounds are mildly decreased throughout. Spirometry results are as follows:

FEV1 75%
FEV1/FVC 57% (normal: ≥70%)
Total lung capacity 114% (normal: 80%-120%)
Residual volume 132% (normal: 80%-120%)
Diffusion capacity (DLCO) 68% (normal: ≥80%)

No significant change is noted after administration of albuterol. Complete blood count is normal. Which of the
following is the most effective measure to decrease mortality in this patient?

A) Chronic macrolide prophylaxis

B) Complete smoking cessation

C) Continuous supplemental oxygen

D) Daily systemic corticosteroid


E) Optimal bronchodilator therapy
Explanation
Correct Answer:

B) Complete smoking cessation

Management of stable chronic obstructive pulmonary disease

• Smoking cessation*
• Pulmonary rehabilitation*
Nonpharmacologic • Vaccination (influenza*, pneumococcal, COVID-19)
• Long-term oxygen therapy* if SaO2 ≤88% at rest
• Nocturnal noninvasive ventilation if daytime hypercapnia*

Surgical • Lung volume reduction or lung transplantation for advanced disease*

• Bronchodilators (LAMA, LABA) ± ICS


• Long-term macrolide (eg, azithromycin 3× weekly)**
Medications
• PDE4 inhibitor (eg, roflumilast)**
• Biologic therapy (eg, anti–IL-5 mAb) if eosinophilic/asthma overlap**

*Reduces mortality.

**Reduces hospitalizations for patients with frequent exacerbations.


ICS = inhaled corticosteroid; LABA = long-acting β2 agonist; LAMA = long-acting muscarinic antagonist; mAb =
monoclonal antibody; PDE4 = phosphodiesterase 4.

This patient with a heavy smoking history has chronic dyspnea, productive cough, and pulmonary function testing
showing airflow obstruction (ie, FEV1/FVC <0.7) that is irreversible (ie, no response to bronchodilator) and
accompanied by air trapping (ie, elevated residual volume). These findings are diagnostic of chronic obstructive
pulmonary disease (COPD).

Smoking cessation reduces mortality in all patients with COPD. The mortality reduction reflects the collective
effect of slowing the decline of lung function (eg, FEV1) and lowering the incidence of cardiovascular events and
lung cancer. All major public health societies endorse a goal of complete and permanent abstinence because the
survival benefit appears to be effective with complete cessation only.

This patient should be commended on his partial achievement (reduced consumption) and supported toward the
ultimate goal of reducing cigarette usage to zero. The projected average life expectancy of successful quitters
begins to exceed that of current smokers within 5 years of complete cessation.

Additional interventions with consistent mortality benefit in COPD include long-term supplemental oxygen for
patients with significant hypoxemia, pulmonary rehabilitation, and vaccination against respiratory pathogens.

(Choices A and D) Prophylaxis with macrolides (eg, azithromycin), phosphodiesterase-4 inhibitors (eg,
roflumilast), or low-dose systemic corticosteroids (eg, prednisone) can suppress airway inflammation to prevent
acute exacerbations of COPD. However, none of these therapies have a proven impact on mortality. Long-term
use of systemic corticosteroids is associated with increased harm (eg, osteoporosis, pulmonary infections).

(Choice C) Long-term oxygen therapy (LTOT) decreases mortality in patients with COPD and significant
hypoxemia (ie, SaO2 ≤88% at rest). There is no expected mortality benefit or indication for LTOT in this patient with
only mild exertional desaturation to 92%.

(Choice E) Optimal bronchodilator therapy (eg, long-acting beta agonist + long-acting muscarinic antagonist)
based on COPD classification improves symptoms, stabilizes lung function, and may decrease the rate of acute
exacerbations. However, bronchodilators have not been shown to decrease mortality.

Educational objective:
Smoking cessation slows the rate of lung function decline and reduces mortality in patients with chronic obstructive
pulmonary disease (COPD). A goal of complete cessation is recommended for all patients, at any age, and with
any stage of COPD.
Question #355

A 54-year-old man comes to the emergency department with cramping lower abdominal pain, mild nausea, and 2
episodes of watery diarrhea. The patient has no fever, vomiting, or urinary symptoms. His medical history is
unremarkable. The patient's father died at age 60 from abdominal aortic aneurysm rupture. He is a lifetime
nonsmoker. His vital signs are normal. CT scan of the abdomen with contrast is shown in the image below:
All gastrointestinal symptoms resolve in 2 hours without any intervention, and the patient wants to go home. Which
of the following is the most appropriate management for the renal findings on the CT scan?

A) Antibiotics

B) Percutaneous aspiration

C) Reassurance only

D) Surgical excision

E) Ureteral stent placement


Explanation
Correct Answer:

C) Reassurance only

Characteristics of renal cysts

Simple renal cyst Malignant cystic mass

Thin, smooth, regular wall Thick, irregular wall

Unilocular Multilocular

No septae Multiple septae, occasionally thick & calcified

Homogeneous content Heterogeneous content (solid & cystic)

Absence of contrast enhancement on CT/MRI Presence of contrast enhancement on CT/MRI

Usually asymptomatic May cause pain, hematuria, or hypertension


Requires follow-up imaging & urological evaluation for
No follow-up needed
malignancy

This patient likely had a viral gastrointestinal illness that resolved. However, his CT scan of the abdomen shows a
simple renal cyst on the inferior aspect of the right kidney. Such cysts are most commonly seen in patients age
>50. They are benign and often discovered incidentally by radiological examination. Most of the time, cysts do not
cause hypertension, flank pain, hematuria, or proteinuria; infection occurs rarely.

When a renal cyst is found on imaging, it is often possible to differentiate between a simple cyst and a malignant
cystic mass based on the radiological features. Features suggesting malignancy include irregular or multilocular
structure with multiple septations, heterogeneous content, and contrast enhancement on CT scan or MRI.
Incidentally discovered cysts with benign features require no additional follow-up evaluation or imaging, and the
patient may be reassured.

(Choice A) Antibiotics are not indicated because there is no evidence of bacterial infection (either gastrointestinal
or genitourinary).

(Choice B) Percutaneous aspiration of the cyst is not indicated because the patient has no symptoms from it.
Percutaneous aspiration may be considered if the cyst is large and painful or infected with purulent material.

(Choice D) Surgical excision or nephrectomy is not indicated, as this patient has a simple benign cyst with good
prognosis.

(Choice E) Ureteral stent placement is indicated when there is urinary obstruction at the ureter or renal pelvis.
Dilation of the renal pelvis (hydronephrosis) or ureter (hydroureter) is a typical sign of urinary obstruction. This
patient has no symptoms of urinary obstruction or radiological signs of obstruction on CT scan.

Educational objective:
Simple renal cysts are almost always benign and do not require further evaluation. Features concerning for
malignant renal mass include a multilocular mass, irregular walls, thickened septa, and contrast enhancement.
Reference
• A clinical view of simple and complex renal cysts.
Question #356

An 80-year-old woman is brought to the emergency department from a nursing home due to progressive lethargy.
The patient has a history of Alzheimer disease and at baseline is conversant and able to indicate her needs. For
the past several days, she has been somnolent and lying in bed most of the time. She has had no fever, vomiting,
or diarrhea. Two weeks ago, the patient was treated with ciprofloxacin for a urinary tract infection. No other
medications have been recently prescribed. Her other medical conditions include hypertension, type 2 diabetes
mellitus, and coronary artery disease. Blood pressure is 100/60 mm Hg, pulse is 100/min, and respirations are 20/
min. Mucous membranes are dry. Lungs are clear to auscultation, and heart sounds are normal. The abdomen is
soft and nontender. The patient has no skin rash. Laboratory results are as follows:

Complete blood count


Hematocrit 44%
Leukocytes 8,200/mm3
Serum chemistry
Blood urea nitrogen 61 mg/dL
Creatinine 2.1 mg/dL
Glucose 140 mg/dL
Urinalysis
Protein trace
White blood cells 1-2/hpf
Casts none

A month ago, her serum creatinine was 0.9 mg/dL. Which of the following is the most likely cause of this patient's
current renal abnormality?
A) Age-related renal functional decline

B) Atherosclerotic renovascular disease

C) Drug hypersensitivity reaction

D) Intravascular volume depletion

E) Nodular glomerulosclerosis

F) Obstructive uropathy

G) Suppression of antidiuretic hormone release

H) Toxin-mediated renal tubular damage


Explanation
Correct Answer:

D) Intravascular volume depletion

Prerenal acute kidney injury

• Decreased renal perfusion


◦ True volume depletion
◦ Decreased EABV (eg, heart failure, cirrhosis)
Etiology
◦ Displacement of intravascular fluid (eg, sepsis, pancreatitis)
◦ Bilateral renal artery stenosis with ACE inhibition
◦ Afferent arteriole vasoconstriction (eg, NSAIDs)

• Increase in serum creatinine (eg, 50% from baseline)


• Decreased urine output
Clinical features • Blood urea nitrogen/creatinine ratio >20:1
• Fractional excretion of sodium <1%
• Unremarkable ("bland") urine sediment

Treatment • Restoration of renal perfusion

EABV = effective arterial blood volume; NSAIDs = nonsteroidal anti-inflammatory drugs.


This patient with tachycardia, mild hypotension, and dry mucous membranes most likely has intravascular volume
depletion leading to prerenal acute kidney injury (AKI). Elderly patients can be especially susceptible to volume
depletion due to an impaired thirst response and often an inability to obtain food and water without assistance (eg,
due to dementia).

Hypovolemia leads to decreased renal blood flow and causes activation of the renin-angiotensin-aldosterone
system. Increased resorption of salt and water occurs, leading to an increase in the passive resorption of urea and
resulting in an elevated blood urea nitrogen/creatinine ratio >20:1. The term "injury" is somewhat inaccurate in
the prerenal state as the kidneys do not experience actual injury unless decreased renal perfusion is prolonged
(leading to acute tubular necrosis). Therefore, an unremarkable urinalysis (absence of significant protein, cells, or
casts) is typical.

(Choice A) Age-related decline in renal function occurs due to loss of renal mass and thickening of the glomerular
basement membrane. However, the process is gradual and cannot account for this patient's relatively abrupt
decrease in renal function.

(Choice B) Atherosclerotic renovascular disease can lead to bilateral renal artery stenosis, which predisposes
patients to AKI following initiation of an ACE inhibitor or angiotensin receptor blocker. However, the only new
medication prescribed for this patient with previously normal renal function is ciprofloxacin.

(Choice C) Drug hypersensitivity reaction can cause intrinsic AKI due to acute interstitial nephritis (AIN).
Ciprofloxacin can cause AIN; however, absence of leukocyte casts on urinalysis and no skin rash make AIN
unlikely.

(Choice E) Nodular glomerulosclerosis occurs in diabetic nephropathy; renal dysfunction typically develops
gradually (rather than over 1 month), and significant proteinuria is expected.

(Choice F) Obstructive uropathy (postrenal AKI) is generally uncommon in women because it is most commonly
due to benign prostatic hyperplasia.

(Choice G) Suppression of antidiuretic hormone occurs in the setting of low blood osmolality. However, antidiuretic
hormone release will be stimulated in this patient due to nonosmotic stimulation from hypovolemia.
(Choice H) Aminoglycosides (eg, gentamicin) can cause intrinsic AKI due to renal tubular toxicity; however,
ciprofloxacin is not associated with direct renal tubular damage.

Educational objective:
The elderly can be especially susceptible to intravascular volume depletion leading to prerenal acute kidney injury.
There is often a history of poor oral intake or excessive volume loss (eg, diarrhea). Laboratory results typically
demonstrate a blood urea nitrogen/creatinine ratio >20:1 and an unremarkable urine sediment.

Reference
• Management of acute renal failure in the elderly patient: a clinician's guide.
Question #357

A 55-year-old man comes to the office due to a 2-week history of reddish-brown urine associated with joint pain,
fatigue, and a lower extremity rash. He reports no chest pain, dyspnea, sore throat, dysuria, or abdominal pain. He
has no known chronic medical conditions and takes no medications. Temperature is 37.8 C (100 F), blood pressure
is 152/80 mm Hg, pulse is 80/min, and respirations are 12/min. Oropharyngeal examination is unremarkable. Heart
sounds are normal, and the lungs are clear to auscultation bilaterally. The abdomen is nontender and
nondistended. Diffuse palpable purpura is identified on both lower extremities. There is mild pain with movement of
the knees and elbows, but no swelling, erythema, or warmth of the joints is present. Laboratory analysis reveals the
following:

Complete blood count


Hemoglobin 9.8 g/dL
Platelets 120,000/mm3
Leukocytes 7,200/mm3

Serum chemistry
Creatinine 1.7 mg/dL

Urinalysis
Protein +2
Blood gross
Leukocyte esterase negative
Nitrites negative
White blood cells 1-2/hpf
Red blood cells many/hpf
Casts RBC casts
Complement
C3 low
C4 very low

Which of the following is most likely to establish the diagnosis?

A) Anti-cyclic citrullinated peptide antibody levels

B) Anti-glomerular basement membrane antibody levels

C) Antineutrophil cytoplasmic antibody levels

D) Antistreptolysin O antibody levels

E) Serum cryoglobulin levels


Explanation
Correct Answer:

E) Serum cryoglobulin levels


This patient with hematuria, acute kidney injury, and hypertension has abnormal urinalysis findings (eg, erythrocyte
casts, leukocyturia, proteinuria) consistent with glomerulonephritis (GN). GN results from immune-mediated injury
to small renal vessels and may be part of a systemic small-vessel vasculitis with widespread manifestations (eg,
fever, palpable purpura, arthralgias).
Given GN has multiple potential etiologies, evaluation involves many laboratory studies. Complement levels are
often helpful because hypocomplementemia (eg, low C3 and C4, as in this patient) usually results from significant
complement consumption due to immune complex (IC)-mediated GN. The primary causes of
hypocomplementemic, IC-mediated forms of GN and their corresponding laboratory findings include the following:

• Poststreptococcal GN (PSGN): Antistreptolysin O antibodies


• Membranoproliferative GN (MPGN): Serum cryoglobulins
• Lupus nephritis: Antinuclear antibodies (ANAs), anti-double-stranded DNA (anti-dsDNA)
• Endocarditis-associated GN: Positive blood cultures

IC-mediated MPGN results from the renal deposition of ICs containing cryoglobulins, which are immunoglobulins
generated by chronic viral infections or autoimmune diseases. MPGN is classically associated with
cryoglobulinemic small-vessel vasculitis (eg, palpable purpura, arthralgia), as in this patient. MPGN can be
caused by chronic hepatitis C (most common), hepatitis B, and HIV. Therefore, serum cryoglobulin testing should
be accompanied by viral serologies (hepatitis C might also explain this patient's thrombocytopenia).

Lupus nephritis can also cause IC-mediated GN with associated hypocomplementemia, vasculitis, and
thrombocytopenia and should be excluded in this patient (eg, ANA, anti-dsDNA). Blood cultures to rule out infective
endocarditis should be obtained as well. IgA nephropathy/IgA vasculitis also cause IC-mediated GN; however,
complement is typically normal because IgA activates complement poorly.

(Choice D) Although PSGN is a common cause of IC-mediated GN, it is unlikely in this patient because it typically
causes renal-limited disease without other manifestations of small-vessel vasculitis; a history of antecedent throat/
skin infection would also be expected.

(Choices B and C) Anti-glomerular basement membrane (anti-GBM) GN and antineutrophil cytoplasmic antibody
(ANCA)-associated GN do not involve circulating ICs, so complement levels are typically normal. Anti-GBM GN is
classically associated with pulmonary hemorrhage. ANCA-associated GN often has variable accompanying lung,
heart, and sinus involvement.

(Choice A) Anti-cyclic citrullinated peptide antibodies are seen with rheumatoid arthritis, which only rarely causes
GN, and polyarticular synovitis (ie, swollen, tender joints) would be expected.
Educational objective:
Glomerulonephritis with hypocomplementemia is typical of immune complex-mediated glomerulonephritis (except
IgA nephropathy). Poststreptococcal glomerulonephritis causes renal-limited disease, so hypocomplementemic
glomerulonephritis associated with systemic vasculitis is usually due to membranoproliferative glomerulonephritis
(positive serum cryoglobulins) or lupus nephritis (positive antinuclear antibodies).

Reference
• Hepatitis C virus associated glomerulopathies.

• Membranoproliferative glomerulonephritis.
Question #358

A 72-year-old man comes to the office for a routine preventive visit. The patient feels well and reports no recent
health issues. He has a history of hypertension and hyperlipidemia; current medications include
hydrochlorothiazide, felodipine, and simvastatin. The patient smoked a pack of cigarettes daily for 40 years but quit
5 years ago. He drinks alcohol occasionally. Vital signs and physical examination are normal. The patient's brother
was recently diagnosed with bladder cancer, and the patient asks whether he should be screened for the disease.
Which of the following is the most appropriate response to this patient's question?

A) You do not require any screening for bladder cancer.

B) You need screening for bladder cancer because of your age.

C) You need screening for bladder cancer because of your family and smoking histories.

D) You need screening for bladder cancer because of your family history.

E) You need screening for bladder cancer because of your smoking history.
Explanation
Correct Answer:

A) You do not require any screening for bladder cancer.

Ideal screening program

• Common
• Serious
Disease characteristics
• Long asymptomatic phase
• Correctable if identified early

• High prevalence in screened population


Patient characteristics
• Appropriate age range & gender

• Low cost relative to disease severity


Test characteristics • High sensitivity & specificity
• Acceptable to broad range of patients

• Identifies population to be screened


Program • Avoids "overdiagnosis" (detects benign/indolent disease)
characteristics • Avoids "lead-time" bias (detects disease early but does not alter ultimate
prognosis)
• Improves outcomes of screened vs nonscreened population

Bladder cancer is the second most common urologic cancer (after prostate cancer) in the United States. Most
cases arise in older individuals who have had chronic exposure to chemical carcinogens in tobacco smoke, the
workplace (eg, aromatic amines, aluminum), or drinking water (eg, arsenic from well water).

Although smoking and a family history of bladder cancer in first-degree relatives are associated with increased risk,
multiple studies have found that bladder cancer screening does not provide survival benefit, even among high-
risk populations. This is likely due to the following:

• Current screening tests such as dipstick urinalysis (for hematuria), cytology of urine sediment, and urinary
tumor biomarkers have relatively low sensitivity/specificity, leading to missed cases (low sensitivity) or
unnecessary workup (false-positive testing).

• Most bladder cancers are detected at an early stage, progress slowly, and are associated with relatively high
rates of 5-year survival; therefore, earlier detection (via screening) provides minimal mortality benefit.

Therefore, the United States Preventive Services Task Force does not recommend screening for bladder cancer
at any age in any patient group (Choices B, C, D, and E).

Educational objective:
The risk of bladder cancer is greatest in those with chronic exposure to chemical carcinogens in tobacco smoke, the
workplace, or drinking water; family history can also indicate increased risk. However, screening for bladder cancer
(eg, urinalysis, urine cytology, tumor markers) has not been shown to improve outcomes and is not recommended.

Reference
• Bladder cancer: diagnosis and treatment.
Question #359

A 62-year-old woman comes to the office due to urinary frequency and burning during urination. The patient reports
no fever, chills, nausea, back pain, or abdominal pain. Her medical history is significant for long-standing diabetes
mellitus and hypertension, for which she takes metformin and lisinopril. The patient does not use tobacco or
alcohol. Blood pressure is 149/100 mm Hg and heart rate is 70/min. Hematocrit is 43% and WBC count is 8,500/
mm3. Creatinine is 1.1 mg/dL. Urinalysis results are as follows:

Glucose negative
Ketones negative
Nitrites positive
Protein 2+
White blood cells 20-25/hpf
Red blood cells 3-5/hpf

The patient is given a course of trimethoprim-sulfamethoxazole. Two weeks later, urinalysis reveals 2+ protein but
no nitrites, white blood cells, or red blood cells. Her urinary symptoms have resolved. Repeat serum creatinine is
1.1 mg/dL. Which of the following is most likely responsible for her persistent urinalysis abnormality?

A) Acute inflammatory infiltrate in the kidney interstitium

B) Atherosclerotic narrowing of the renal arteries

C) Glomerular basement membrane changes

D) Insoluble crystal precipitation in the tubular lumen

E) Retrograde passage of urine into the renal pelvis


Explanation
Correct Answer:

C) Glomerular basement membrane changes


This patient's presenting symptoms of dysuria and urinary frequency, combined with abnormal urinalysis (eg,
hematuria, leukocytes, positive nitrites), suggest urinary tract infection. However, after treatment and resolution of
urinary symptoms, she continues to have persistent proteinuria. In the context of her long-standing diabetes and
poorly controlled hypertension, persistent proteinuria suggests diabetic nephropathy (DN).
Metabolic by-products (eg, advanced glycation end-products, reactive oxygen species) associated with diabetes
lead to irreversible changes of DN, primarily affecting the glomerular basement membrane and surrounding
structures. These include:

• Glomerular hyperfiltration
• Glomerular basement membrane fibrosis and thickening
• Interstitial fibrosis, mesangial thickening, and nodules (Kimmelstiel-Wilson lesion)

A common clinical abnormality in DN is persistently elevated albumin excretion, known as moderately increased
albuminuria (ie, albumin-creatinine ratio of 30-300 mg/g). Therefore, screening via random urine albumin-
creatinine ratio should be performed at least annually, starting either at diagnosis (type 2) or 5 years after diagnosis
(type 1). Risk factor modification (eg, diet optimization, exercise, smoking cessation), glycemic control (A1c ≤7.0%),
and medical management of hyperlipidemia and hypertension (eg, ACE inhibitors) also help slow the progression of
DN.

(Choice A) Acute interstitial nephritis (AIN) can be caused by antibiotics (eg, methicillin), infections (eg,
Legionella), or systemic disease (eg, sarcoidosis). Trimethoprim-sulfamethoxazole can be a potential cause of AIN;
however, this patient lacks the typical AIN findings of acutely elevated creatinine, pyuria (eg, eosinophiluria), and
white cell casts in the urine.

(Choice B) Atherosclerotic vascular disease can lead to renal artery stenosis, the most common cause of
secondary hypertension in adults. Poorly controlled hypertension is a hallmark of this condition; however, this
patient's persistent proteinuria is an uncommon finding.

(Choice D) Sulfonamide antibiotics can lead to insoluble crystal precipitation in kidney tubules and subsequent
obstruction and acute kidney injury. Other causes include acyclovir, excessive vitamin C, and methotrexate.
However, crystal-induced obstruction would not cause persistent proteinuria; moreover, findings of acute creatinine
elevation or crystalluria would be expected.

(Choice E) In adults, vesicoureteral reflux (ie, retrograde passage of urine into the renal pelvis) is associated with
neurogenic bladder and can cause frequent urinary tract infections. Long-standing diabetes may cause autonomic
bladder dysfunction; however, this patient does not have associated symptoms of chronic incontinence, and
persistent proteinuria would be uncommon.
Educational objective:
Diabetic nephropathy is characterized by glomerular hyperfiltration, basement membrane thickening, and mesangial
nodules. Persistent proteinuria in a patient with long-standing diabetes and poorly controlled hypertension should
raise suspicion for this diagnosis.

Reference
• Markers of inflammation and oxidative stress in the development and progression of renal disease in
diabetic patients.
Question #360

A 73-year-old man comes to the office due to 3 months of progressive urinary urgency, hesitancy, nocturia, and
weak urinary stream. He has no fever, abdominal pain, hematuria, malaise, or weight loss. His only medication is
lisinopril for essential hypertension. The patient has no history of diabetes mellitus or ischemic heart disease. He
does not use tobacco, alcohol, or illicit drugs. Digital rectal examination reveals a smooth, firm, and enlarged
prostate without induration or asymmetry. Neurological examination is normal. Urinalysis shows no proteinuria or
hematuria. The patient's serum creatinine is 2.1 mg/dL, which is higher than his baseline creatinine of 1.2 mg/dL 4
months ago. Prostate-specific antigen is normal. Which of the following is the most appropriate next step in
evaluation of this patient's acute kidney injury?

A) Cystoscopy

B) Kidney biopsy

C) Prostate biopsy

D) Renal ultrasound

E) Urine cytology
Explanation
Correct Answer:

D) Renal ultrasound

This patient has lower urinary tract symptoms (LUTS) (eg, urinary urgency, hesitancy, nocturia, weak urinary
stream) with a smooth, enlarged prostate on examination consistent with benign prostatic hyperplasia (BPH).
Initial evaluation of patients with LUTS should include a urinalysis (to exclude infection and hematuria) and serum
prostate-specific antigen (PSA) to assess risk for prostate cancer. A serum creatinine test is not required in the
routine evaluation of uncomplicated BPH. However, it is recommended by some expert panels, especially for
patients with more significant symptoms or additional risk factors (eg, hypertension, diabetes) for chronic kidney
disease.

This patient has evidence of acute kidney injury, as indicated by an interval rise in serum creatinine. Creatinine is
generally not elevated in unilateral urinary obstruction (eg, ureteral calculus) but can be elevated in bilateral
obstruction, such as in patients with severe bladder outlet obstruction due to BPH. Patients with acutely elevated
creatinine require imaging (preferably renal ultrasound) to assess for hydronephrosis and exclude other causes of
obstruction. Placement of a urinary catheter in patients with hydronephrosis can provide quick relief of the
obstruction.

(Choices A and E) Cystoscopy in patients with BPH can reveal signs of chronic bladder obstruction, but it is
nonspecific and usually reserved for those who have failed initial management. It is also used to visualize the lower
urinary tract in patients with hematuria, usually in combination with imaging of the upper tract (eg, CT scan of the
kidneys and ureters). Urine cytology is sometimes performed in place of cystoscopy for low-risk patients with
hematuria. Cytology can also be done for patients with LUTS and additional risk factors for bladder cancer (eg,
smoking).

(Choice B) Kidney biopsy is used to diagnose intrinsic renal causes of acute kidney injury. This patient's
presentation is more consistent with obstructive (post-renal) acute kidney injury.
(Choice C) Prostate biopsy is indicated for patients with signs of prostate cancer, such as grossly asymmetric
enlargement of the prostate, palpable nodules, or persistently elevated PSA levels >4 ng/dL.

Educational objective:
Patients with severe bladder outlet obstruction due to benign prostatic hyperplasia can develop acute kidney injury.
A renal ultrasound is advised for assessment of hydronephrosis in those with worsening kidney function.

Reference
• Clinical utility of gray scale renal ultrasound in acute kidney injury.
Question #361

A 67-year-old man comes to the office due to 2 weeks of intermittent blood in the urine. He has had no pain,
urgency, dysuria, frequency, urethral discharge, or nocturia. The bleeding is present throughout micturition. The
patient has a history of coronary artery disease, hypertension, and hyperlipidemia. He has smoked a pack of
cigarettes daily for 40 years. He is sexually active. Temperature is 36.9 C (98.4 F), blood pressure is 120/80 mm
Hg, pulse is 78/min, and respirations are 16/min. Abdomen is soft and nontender with no masses; bowel sounds
are normal. Rectal examination shows an enlarged prostate with no tenderness, asymmetry, or induration.
Urinalysis shows >50 red blood cells/hpf; there are no leukocytes, casts, or dysmorphic erythrocytes. Urine
cytology shows no abnormal cells. Which of the following is the most appropriate next step in evaluation of this
patient?

A) Abdominal ultrasound

B) Cystoscopy

C) Prostate-specific antigen level

D) Reassurance and routine follow-up

E) Urine culture
Explanation
Correct Answer:

B) Cystoscopy

Bladder cancer

• >90% urothelial carcinoma


Epidemiology
• ↑ Risk with smokers & exposure to industrial carcinogens

• Painless hematuria throughout micturition


Manifestations • Irritative voiding symptoms (eg, frequency, urgency, dysuria)
• Regional pain

• Flexible cystoscopy with biopsy (gold standard)


Diagnosis
• Urine cytology

• TURBT
Staging
• Upper urinary tract imaging (eg, IVP, MRI, CT)

• No muscle invasion: TURBT & intravesical immunotherapy


Treatment
• Muscle invasion: radical cystectomy & systemic chemotherapy
• Metastatic: systemic chemotherapy & immunotherapy

IVP = intravenous pyelogram; TURBT = transurethral resection of bladder tumor.

This older patient with a 40-pack-year smoking history has painless, gross hematuria throughout micturition, raising
strong suspicion for bladder cancer, the most common malignancy of the urinary tract. Most cases occur due to
malignant transformation of the urothelium in response to chronic carcinogen exposure (eg, tobacco smoke,
industrial chemicals). Because tumor growth is associated with the formation of friable new blood vessels, bleeding
into the urinary tract is common and often results in painless hematuria that lasts throughout micturition (tumors
in the bladder neck occasionally present with only terminal hematuria).

The workup of gross hematuria begins with urinalysis to confirm the diagnosis (>3 red blood cells/hpf) and to
evaluate for common underlying causes, such as urinary tract infection (eg, pyuria, bacteriuria) and
glomerulonephritis (eg, red blood cell casts, dysmorphic erythrocytes). Patients who have no clear cause of
hematuria and are age >40 require urgent evaluation for urinary tract malignancy; the first tests of choice are
generally cystoscopy (to visualize the bladder wall and biopsy/resect suspicious lesions) and CT urography (to
visualize the kidney and to evaluate for metastases). Although urine cytology is often performed as part of the
evaluation for hematuria, it has low sensitivity (<35%) and cannot be relied on to rule out bladder or kidney cancer.

(Choice A) Abdominal ultrasound is not routinely used in the evaluation of hematuria because it frequently misses
small bladder masses and cannot evaluate tumor depth, extension to surrounding tissues, or nodal spread.

(Choice C) Prostate-specific antigen level can aid in the diagnosis of prostate cancer, which is usually
asymptomatic and is found during routine digital rectal examination (eg, prostatic nodule) or prostate-specific
antigen testing. Hematuria is uncommon because most prostate cancers grow in the periphery of the gland rather
than in the central, periurethral zone.

(Choice D) Painless hematuria in an adult age >40 must be evaluated urgently with cystoscopy because delays in
diagnosis of bladder or kidney cancer are associated with poor prognosis.

(Choice E) Urine culture can diagnose urinary tract infection, which usually causes irritative bladder symptoms (eg,
dysuria, frequency), pyuria, and bacteriuria. This patient has painless hematuria with no leukocytes in his urine,
making an infection unlikely.

Educational objective:
Bladder cancer often presents with painless hematuria that lasts throughout micturition in an adult age >40.
Urinalysis should be performed to confirm hematuria (>3 red blood cells/hpf) and to rule out infection/
glomerulonephritis. Those with no clear cause for hematuria require urgent evaluation with cystoscopy to visualize
the bladder for lesions.
Question #362

An 18-year-old girl comes to the office with a new-onset skin rash and malaise. She is concerned as her aunt has
lupus treated with corticosteroids. Her medical history is unremarkable except for dysuria and increased urinary
frequency a week ago, which was effectively treated with sulfamethoxazole-trimethoprim. The patient takes no
medications and uses no illicit drugs. She has had the same sexual partner for the past 4 months. Her temperature
is 38.0 C (100.4 F), pulse is 86/min, and respirations are 16/min. Physical examination reveals a disseminated
maculopapular rash. There is no costovertebral tenderness. The patient has no joint swelling or effusion. Her
serum creatinine is 2.0 mg/dL. Urinalysis reveals 2-5 red blood cells/hpf, numerous white blood cell casts, and mild
proteinuria. Which of the following is the most likely diagnosis?

A) Disseminated gonococcemia

B) Interstitial nephritis

C) Lupus nephritis

D) Postinfectious acute glomerulonephritis

E) Pyelonephritis
Explanation
Correct Answer:

B) Interstitial nephritis
The clinical presentation is typical of allergic interstitial nephritis (AIN): acute renal failure, fever, rash, a recent
history of para-aminobenzoic acid analogue antibiotic (sulfonamide) ingestion, and white blood cell (WBC) casts
on urinalysis. Patients may also have arthralgias and eosinophiluria. Symptoms usually appear 5 days to several
weeks after use of an offending agent. Drugs cause the majority of AIN cases; examples include antibiotics (eg,
penicillins, cephalosporins, trimethoprim, rifampin), nonsteroidal anti-inflammatory drugs, and diuretics. Less
commonly, AIN may be caused by infectious agents (eg, Legionella, Mycobacterium tuberculosis, Streptococcus).

(Choice A) Disseminated gonococcemia causes a vesicopustular rash. Renal failure with WBC casts is not usually
seen.

(Choice C) The mucocutaneous manifestations of lupus include malar rash and discoid rash, but this patient has a
maculopapular rash. Lupus nephritis is characterized by hypertension, mild proteinuria, and red blood cell (RBC)
casts.

(Choice D) Postinfectious glomerulonephritis produces hematuria, mild proteinuria, RBC casts, and fluid retention
resulting in hypertension and periorbital edema. It usually occurs 1-2 weeks after an episode of streptococcal
pharyngitis/skin infection and not after a urinary infection.

(Choice E) Pyelonephritis can complicate untreated cystitis, but manifests with chills, fever, and tenderness in the
flanks and/or the costovertebral angle. Disseminated rash is not usually seen.

Educational objective:
Drug-induced interstitial nephritis is usually caused by antibiotics (eg, penicillins, cephalosporins, trimethoprim,
rifampin), nonsteroidal anti-inflammatory drugs, and diuretics. Patients present with a fever, maculopapular rash,
and renal failure. Urinalysis may reveal white blood cell casts and, less frequently, eosinophils.
Question #363

A 56-year-old man comes to the office for follow up of hypertension. The patient had undergone kidney
transplantation one year ago for focal segmental glomerulosclerosis. He was originally seen in the office 8 weeks
ago for hypertension and had a blood pressure of 210/110 mm Hg; initially he was given amlodipine, but the patient
continued to have hypertension. Two weeks ago, he was also started on lisinopril. In addition, he takes tacrolimus
and low-dose prednisone. The patient currently feels well. Today, blood pressure is 160/90 mm Hg, pulse is 78/
min, and respirations are 16/min. Examination shows normal jugular venous pressure, vesicular breath sounds,
and normal heart sounds. The transplant site is nontender. Laboratory studies reveal a serum creatinine level of
2.4 mg/dL (2 weeks ago: 1.5 mg/dL). Serum tacrolimus level is within the normal therapeutic range. Which of the
following is the most appropriate next step in management of this patient?

A) Kidney biopsy

B) Lisinopril dose increase

C) Prednisone discontinuance

D) Renal vascular imaging

E) Tacrolimus discontinuance
Explanation
Correct Answer:

D) Renal vascular imaging

This patient with a history of renal transplant has severe, persistent hypertension. In association with the acute
kidney injury (AKI) that developed after initiation of lisinopril, this presentation suggests renovascular
hypertension due to transplant renal artery stenosis (RAS). Although most cases of RAS occur in elderly men
with diffuse atherosclerotic disease, it can also occur in patients with a transplanted kidney and is commonly
associated with operative abnormalities (eg, trauma during organ procurement, abnormal suture placement), viral
infection (cytomegalovirus, BK virus), and atherosclerosis of the donor artery. Transplant RAS typically occurs in
the first 2 years after transplantation.

Similar to RAS due to other causes, transplant RAS typically manifests with resistant hypertension. A decline in
renal function after the addition of ACE inhibitors or angiotensin II receptor blockers is highly suggestive of the
diagnosis. Other findings that suggest RAS include a lateralizing abdominal bruit and recurrent flash pulmonary
edema. The diagnosis is made with renal vascular imaging (eg, renal Doppler ultrasonography). Management of
transplant RAS usually includes angioplasty, possibly with stent placement.

(Choice A) Kidney biopsy is indicated to evaluate for AKI due to acute allograft rejection, which is somewhat less
likely in the absence of graft tenderness or fever. In addition, biopsy is invasive, and noninvasive methods of
assessment (eg, imaging) should be performed prior to biopsy.

(Choice B) Because transplant RAS is the equivalent of bilateral RAS in a non-transplant patient, the initiation of
an ACE inhibitor is likely to trigger AKI. Increased lisinopril dosing would likely further worsen this patient's renal
function.

(Choices C and E) High-dose prednisone is a common cause of hypertension; however, low doses are not
typically associated with marked hypertension. Toxicity to calcineurin inhibitors (eg, tacrolimus) can cause
hypertension and AKI; however, such adverse effects are more common with elevated plasma drug levels. In
addition, the timing of this patient's AKI following initiation of lisinopril makes RAS more likely than an adverse effect
of immunosuppression.

Educational objective:
Renal artery stenosis (RAS) can occur in the renal allograft, typically within 2 years of transplant. Like other forms
of RAS, suggestive findings include persistently elevated blood pressure, decline in renal function with the addition
of ACE inhibitors, a lateralizing abdominal bruit, and recurrent flash pulmonary edema. The diagnosis is made with
renal vascular imaging (eg, renal Doppler ultrasonography).
Question #364

A 76-year-old man is evaluated after a recent hospitalization for septic shock due to acute pyelonephritis. CT scan
on admission showed right-sided perinephric stranding and urolithiasis with mild hydronephrosis. During a 5-day
hospitalization, the patient received intravenous fluids, vasopressors, and broad-spectrum antibiotics. His condition
gradually improved, the stone was spontaneously passed, and he was discharged a week ago to a skilled nursing
facility on continued intravenous amikacin, which he completed 3 days ago. Physical examination today shows no
abnormalities. Serum creatinine is 3.6 mg/dL (0.9 at discharge). Fractional excretion of sodium is >2%. Urinalysis
shows occasional epithelial cell casts and no white blood cells. Which of the following is the most likely cause of
this patient's renal dysfunction?

A) Contrast-associated acute kidney injury

B) Drug-induced acute kidney injury

C) Postinfectious glomerulonephritis

D) Renal scarring after pyelonephritis

E) Septic shock–induced tubular injury


Explanation
Correct Answer:

B) Drug-induced acute kidney injury

Acute tubular necrosis due to nephrotoxins

• Antibiotics: aminoglycosides (eg, gentamicin), vancomycin


Common
• Antivirals: cidofovir, foscarnet
nephrotoxins
• Other: radiographic contrast, cisplatin, heavy metals, heme pigment

• Tubular epithelial cell necrosis with cell detachment, cast formation & tubular lumen
Histology obstruction
• Proximal tubules primarily affected

• BUN/creatinine ratio <20:1, FENa >2%


Presentation • Muddy brown (granular) or epithelial cell casts, low urine osmolality
• Oliguria or polyuria, ± electrolyte abnormalities

BUN = blood urea nitrogen; FENa = fractional excretion of sodium.

This patient has developed acute kidney injury (AKI) following hospitalization and treatment for septic shock due to
acute pyelonephritis. The use of amikacin raises suspicion for aminoglycoside nephrotoxicity, a common cause
of acute tubular necrosis (ATN).
After glomerular filtration, aminoglycosides (eg, amikacin, gentamicin) are taken up by tubular cells in the cortical
segments (ie, S1, S2) of the proximal convoluted tubule (PCT) and accumulate in these cells over time.
Accumulation to toxic levels typically occurs after >5-7 days of aminoglycoside use. As with other causes of ATN,
fractional excretion of sodium (FENa) is usually >2%, and urinalysis typically shows no blood cells with muddy
brown (granular) or epithelial cell casts.

The risk for nephrotoxicity can be reduced with once-daily dosing regimens; however, because nephrotoxicity can
occur even with close monitoring of serum drug levels, aminoglycosides are usually avoided unless absolutely
necessary (eg, multi–drug-resistant, gram-negative organisms). Renal function usually returns to baseline within 3
weeks of drug cessation as PCT cells regenerate.

(Choice A) Contrast-associated AKI usually results in a mild creatinine elevation 24-48 hours after contrast
administration, with most patients returning to baseline renal function 3-7 days afterward. However, this patient's
creatinine elevation is substantial and started following hospital discharge rather than shortly after his admission CT
scan.

(Choice C) Although postinfectious glomerulonephritis may occur 10 days following infection and might also cause
FENa >2%, urinalysis would show red blood cells (RBCs) and, often, RBC casts. Postinfectious glomerulonephritis
in adults typically occurs following infections caused by staphylococci or streptococci, which are gram-positive
organisms not treated with aminoglycoside monotherapy.

(Choice D) Renal scarring is a potential cause of chronic kidney disease in patients with chronic pyelonephritis.
Scarring takes months to develop and is unlikely to abruptly increase serum creatinine levels, as seen with this
patient's AKI.

(Choice E) Septic shock is a common cause of ischemic ATN, but the onset is typically 24-48 hours following the
ischemic insult. This patient was at risk for ischemic ATN shortly after his initial presentation, but his normal
creatinine at hospital discharge makes septic shock–induced tubular injury unlikely.

Educational objective:
Aminoglycosides (eg, amikacin) can cause acute kidney injury in the form of toxic acute tubular necrosis, usually
after >5-7 days of use. Fractional excretion of sodium is typically >2%, and urinalysis usually shows muddy brown
(granular) or epithelial cell casts.
Reference
• Aminoglycoside-induced nephrotoxicity.
Question #365

A 45-year-old woman comes to the office for follow-up after a recent visit to the emergency department due to
vomiting and severe right flank pain 6 weeks ago. Imaging revealed a 4-mm stone in the right midureter, which she
passed at home a week later. The patient has been to the emergency department twice in the past 5 years due to
similar episodes of renal colic. Both times, the stones passed spontaneously, and she did not seek further medical
care. The patient has been following dietary instructions and drinking plenty of fluids. Vital signs are normal.
Physical examination shows no abnormalities. A 24-hour urine collection shows urinary calcium excretion of 350
mg (normal: <250 in women). Laboratory results show a serum calcium concentration of 8.9 mg/dL; serum
parathyroid hormone is normal. Further investigations fail to demonstrate a cause of the hypercalciuria. Which of
the following is the most appropriate pharmacotherapy for this patient?

A) Acetazolamide

B) Chlorthalidone

C) Low-dose furosemide

D) Sodium bicarbonate

E) Sodium citrate
Explanation
Correct Answer:

B) Chlorthalidone

Prevention of calcium stone (calcium oxalate, calcium phosphate) recurrence

Intervention Mechanism

All calcium stones:

↑ Fluid (produce >2 L/day urine) ↑ Urine flow, ↓ solute concentration

↓ Sodium (<2,300 mg/day) ↑ Renal calcium reabsorption


Dietary
interventions
↑ Citrate (fruits & vegetables) Binds urinary calcium to inhibit stone formation

↑ Potassium ↑ Urinary citrate excretion

↓ Animal protein ↓ Urinary calcium excretion


Calcium oxalate stones:

Adequate calcium intake (1,200 mg/day) ↓ Oxalate absorption in GI tract

↓ Oxalate (spinach) ↓ Urinary oxalate excretion

Thiazide diuretics ↑ Renal calcium reabsorption


Pharmacologic
interventions
Potassium citrate ↑ Urinary citrate concentration

GI = gastrointestinal.

This patient has recurrent nephrolithiasis with elevated urinary calcium (ie, hypercalciuria) and an otherwise
normal medical evaluation. She most likely has idiopathic hypercalciuria, which is characterized by hypercalciuria
in the presence of normal serum calcium levels and the absence of an identifiable disease process (eg,
hyperparathyroidism) that alters calcium metabolism and/or absorption.

Hypercalciuria greatly increases the risk for recurrent calcium (eg, calcium oxalate, calcium phosphate) stone
formation and can be treated with increased fluid intake (eg, ≥2 L/day) and dietary measures alone in most
patients. Dietary measures include low sodium (↑ renal calcium reabsorption), low animal protein (↓ urinary calcium
excretion), high citrate (fruits/vegetables) consumption (binds urinary calcium), and adequate dietary calcium (↓
oxalate absorption). If these measures fail (eg, recurrent stones, increased size of old stones, persistent
hypercalciuria) after a 3- to 6-month trial, as seen in this patient, pharmacotherapy with a thiazide diuretic is
recommended.
Thiazide diuretics (eg, chlorthalidone) decrease urinary calcium excretion by up to 50% and significantly reduce
the formation of new calcium stones. In addition to increasing calcium reabsorption at the level of the distal tubule,
thiazides produce relative hypovolemia that results in a compensatory increase in sodium reabsorption, which is
tightly coupled to calcium reabsorption, at the level of the proximal tubule and loop of Henle. A low-sodium diet is
crucial for these medications to work effectively.

(Choice A) Carbonic anhydrase inhibitors (eg, acetazolamide) block bicarbonate reabsorption in the proximal
tubule, creating an alkaline urine that increases the risk for precipitation of calcium phosphate stones.

(Choice C) Loop diuretics (eg, furosemide) promote calcium wasting by the kidney. The resulting increase in
urinary calcium excretion increases the risk for calcium stone formation.

(Choices D and E) Sodium citrate increases urinary citrate levels; and sodium bicarbonate alkalinizes the serum,
which promotes renal citrate excretion. Urinary citrate can bind calcium (creating a soluble complex) and inhibit
calcium stone formation. However, the increased sodium load with sodium citrate and sodium bicarbonate
counteracts this positive effect because it leads to increased urinary calcium excretion due to the tight coupling of
sodium and calcium reabsorption in the proximal tubule and loop of Henle. Potassium citrate or potassium
bicarbonate can be used instead.

Educational objective:
Hypercalciuria increases the risk for calcium stone recurrence. If increased fluid intake and dietary measures are
ineffective at preventing stones, thiazide diuretics should be added. Thiazide diuretics increase renal calcium
reabsorption, decrease urinary calcium concentration, and significantly reduce the formation of new calcium stones.

Reference
• Effectiveness of treatment modalities on kidney stone recurrence.

• Nutrition and kidney stone disease.


Question #366

A 55-year-old man comes to the office for hypertension follow-up. He was diagnosed with hypertension 5 years ago
and since then has been treated with benazepril and amlodipine. His blood pressure has been consistently high
over the past 6 months despite dietary modifications to decrease salt intake. Medical history is significant for
hypercholesterolemia. The patient does not smoke or consume alcohol. Blood pressure is 152/94 mm Hg and
pulse is 80/min. BMI is 30 kg/m2 with increased waist-to-hip ratio. Adding chlorthalidone to the patient's current
therapy is considered to improve control of the hypertension. Which of the following metabolic effects may be
expected as a result of this therapy?

A) Decreased LDL cholesterol

B) Decreased plasma triglycerides

C) Hyperglycemia

D) Hyperkalemia

E) Hypermagnesemia

F) Hypocalcemia
Explanation
Correct Answer:

C) Hyperglycemia

Metabolic effects of thiazide diuretics

Adverse effect Mechanism

Inhibition of Na+/Cl− cotransporters in the distal


Hyponatremia
convoluted tubule

Hypokalemia Compensatory rise in renin & aldosterone secretion

Hypercalcemia Increased reabsorption in distal tubule

Hyperglycemia
Decreased insulin secretion & increased insulin
resistance
Hypercholesterolemia
Hyperuricemia Increased reabsorption in proximal tubule

Thiazide diuretics (eg, chlorthalidone, hydrochlorothiazide) are widely used antihypertensive medications that are
effective as monotherapy or in combination with other agents. At lower doses, chlorthalidone is more effective than
other thiazides and is associated with a lower rate of heart failure compared with other antihypertensive agents
(possibly due to its ability to achieve lower blood pressure as a single agent).

However, thiazides can raise the patient's blood sugar level due to impairment of both insulin release from the
pancreas and glucose utilization in peripheral tissues. Thiazide-induced glucose intolerance is seen more
commonly in patients with diabetes mellitus and metabolic syndrome (ie, hypertension, dyslipidemia, and abdominal
obesity). The metabolic adverse effects of thiazide diuretics are dose-dependent, therefore significant blood
pressure reduction can be achieved with minimal side effects when low doses are used.

(Choices A and B) Thiazide diuretics can unfavorably affect lipid metabolism by increasing (not decreasing) LDL
cholesterol and plasma triglyceride levels.

(Choices D, E, and F) Thiazides increase renal excretion of sodium, potassium, and magnesium and decrease
excretion of calcium. This can potentially lead to hyponatremia, hypokalemia, hypomagnesemia, and
hypercalcemia. Thiazides also reduce renal uric acid excretion, and the resulting hyperuricemia predisposes to
gout.

Educational objective:
Adverse metabolic effects of thiazide diuretics include hyperglycemia, increased LDL cholesterol and plasma
triglycerides, and hyperuricemia. Electrolyte abnormalities that can be induced by thiazide diuretics include
hyponatremia, hypokalemia, hypomagnesemia, and hypercalcemia.

Reference
• Thiazide and loop diuretics.

• Best thiazide diuretic for hypertension.


Question #367

A 70-year-old woman comes to the office due to worsening bilateral shoulder pain for 3 months that is not
responsive to analgesics. The patient also reports tingling in her right hand and forearm for the past 3 weeks.
Medical history is significant for diabetic nephropathy and end-stage renal disease. The patient has been receiving
hemodialysis 3 times a week for 20 years. Temperature is 37 C (98.6 F), blood pressure is 130/78 mm Hg, pulse is
80/min, and respirations are 16/min. Examination shows bilateral hypertrophy of the shoulders. Range of motion of
the shoulder joints is restricted in all directions. Weakness and atrophy of the muscles of the thenar eminence, as
well as decreased sensation in the thumb and index, middle, and ring fingers, are noted. Heart sounds are normal.
There is no hepatosplenomegaly. X-ray reveals radiolucent bone cysts in both shoulders. Ultrasound reveals
increased rotator cuff thickness and deposits with increased echogenicity between the muscles and tendons of the
rotator cuff. Serum calcium is 8.1 mg/dL. Which of the following is the most likely diagnosis?

A) Acromegaly

B) Amyloidosis

C) Polymyalgia rheumatica

D) Pseudogout

E) Tertiary hyperparathyroidism
Explanation
Correct Answer:

B) Amyloidosis

This patient on long-term dialysis has shoulder pain/hypertrophy and carpal tunnel syndrome due to dialysis-
related amyloidosis (DRA). The unifying feature of amyloidoses is misfolded precursor proteins that deposit in
extracellular tissues, causing organ dysfunction. When these precursor proteins undergo excessive production,
inadequate clearance, or inherited mutation, amyloidosis results.

In DRA, beta2-microglobulin (B2-m) is the precursor protein deposited as amyloid due to inadequate elimination
despite dialysis. Along with sustained elevation of B2-m, several connective tissue components (eg,
glycosaminoglycans, type I collagen) participate in amyloid fibril stabilization. This may explain DRA's affinity for
osteoarticular structures, resulting in the following classic triad:

• Scapulohumeral periarthritis: shoulder pain/hypertrophy with increased rotator cuff thickness and
hyperechogenic deposits on imaging

• Carpal tunnel syndrome: median neuropathy, with thenar eminence atrophy and weakness, tingling and
decreased sensation in the first 3 fingers, as well as possible flexor tendosynovitis (finger contractures)

• Bone cysts: possible pathologic fracture

DRA prevalence increases with age and dialysis duration.

(Choice A) Acromegaly frequently causes carpal tunnel syndrome, and soft tissue hypertrophy is characteristic.
However, enlargement is not isolated to the shoulders. Tissue overgrowth involves the jaw (macrognathia), brow
ridge, tongue (macroglossia), and hands and feet. Headaches are a prominent symptom, and acromegaly is not
associated with dialysis.

(Choice C) Polymyalgia rheumatica prevalence increases with age, and shoulder pain is characteristic; however,
the hip girdle is also involved, and there is no hypertrophy or bone cysts.

(Choice D) Pseudogout (acute attacks of calcium pyrophosphate crystal deposition in synovial tissues) typically
results in a monoarticular, self-limited inflammatory arthritis. Imaging can reveal chondrocalcinosis (calcification of
articular cartilage appearing as linear radiodensities on x-ray) and hyperechoic bands within the tendons, not
between the muscles and tendons, as in this patient. Bilateral shoulder involvement would be uncommon; it rarely
causes bone cysts and is unrelated to dialysis.

(Choice E) Tertiary hyperparathyroidism results from inadequately controlled secondary hyperparathyroidism in


end-stage kidney disease. Parathyroid hyperplasia results in the autonomous secretion of parathyroid hormone
unresponsive to plasma calcium levels, typically resulting in hypercalcemia. Although tertiary hyperparathyroidism
can cause bone cysts, this patient's low calcium makes it highly unlikely, and soft tissue deposits are not typical.

Educational objective:
Dialysis-related amyloidosis results from inadequate clearance of beta2-microglobulin despite dialysis, and the
prevalence increases with age and dialysis duration. It has a predilection for osteoarticular structures presenting as
scapulohumeral periarthritis (shoulder pain/hypertrophy), carpal tunnel syndrome, and bone cysts.
Question #368

A 38-year-old woman comes to the emergency department due to right-sided flank pain radiating to the groin. She
also has hematuria but no dysuria or urinary frequency. She has no other medical conditions and takes no
medications. Temperature is 36.7 C (98.1 F), blood pressure is 110/80 mm Hg, and pulse is 68/min. The patient
has no costovertebral angle tenderness. Abdominal imaging shows a 6-mm calculus in the distal right ureter. The
patient's symptoms improve with intravenous hydration and analgesics. Which of the following pharmacotherapy is
recommended to facilitate stone passage?

A) Bethanechol

B) Finasteride

C) Oxybutynin

D) Phenazopyridine

E) Tamsulosin
Explanation
Correct Answer:

E) Tamsulosin
This patient has a 6-mm stone in the right distal ureter. The resulting obstruction of urine flow increases
intraureteral pressure and induces ureteral smooth muscle spasms that cause the characteristic pain of renal colic
(eg, flank pain radiating to the groin).

The likelihood of a stone passing spontaneously is dependent on both stone size and location within the
ureter.

• Small stones (≤5 mm) that are distally located have a high probability of passing with expectant
management alone (eg, fluids, pain control).

• Stones that are large (>10 mm), regardless of their location, almost always require (outpatient) urologic
intervention (eg, lithotripsy, stent placement) for resolution.

• For stones that are moderately sized (>5 mm and ≤10 mm), medical expulsive therapy with an alpha-1
antagonist (eg, tamsulosin) should be considered.

Alpha-1 adrenergic receptors, which are located along the length of the ureter, mediate smooth muscle contraction.
Blocking these receptors can increase ureteral smooth muscle relaxation, decreasing spasms (thus reducing
analgesic requirements) and facilitating stone passage. The beneficial effects have been observed mainly in the
treatment of distal ureteral stones; however, alpha-1 antagonists are a reasonable option for proximal stones as well
given the relatively low risk of adverse effects.

(Choices A and C) Activation of cholinergic receptors within the bladder smooth muscle stimulates contraction and
facilitates voiding. Bethanechol, a cholinergic agonist, promotes bladder emptying in patients with urinary retention
from bladder hypotonia. Conversely, oxybutynin, which has anticholinergic effects, promotes bladder muscle
relaxation. Oxybutynin is often prescribed for overactive bladder (detrusor instability).

(Choice B) Finasteride is a 5-alpha reductase inhibitor that blocks the conversion of testosterone to
dihydrotestosterone. It is used in the management of benign prostatic hyperplasia (BPH) to decrease prostatic
volume. Reducing prostate size usually takes months to years; finasteride is not used for acute symptom control
even in BPH.

(Choice D) Phenazopyridine is a urinary tract analgesic that provides symptom relief of dysuria (eg, due to urinary
tract infection or instrumentation). It has no effect on kidney stone passage.

Educational objective:
The likelihood of spontaneous ureteral stone passage decreases with increased stone size and proximal location.
Alpha-1 antagonists (eg, tamsulosin) may aid in stone expulsion by relaxing ureteral smooth muscle. Their
beneficial effects are seen most often in treatment of distal ureteral stones sized >5 mm and ≤10 mm.

Reference
• Alpha-blockers as medical expulsive therapy for ureteral stones.

• Efficacy and safety of tamsulosin in medical expulsive therapy for distal ureteral stones with renal colic: a
multicenter, randomized, double-blind, placebo-controlled trial.

• Is tamsulosin effective for the passage of symptomatic ureteral stones: a systematic review and meta-
analysis.
Question #369

A 21-year-old woman comes to the office with her mother due to progressive weakness and fatigue. She nearly
collapsed yesterday during one of her routine 3-hour workouts. Her mother adds that the patient is not doing well
academically despite persistent attempts to improve her grades. Blood pressure is 106/58 mm Hg and pulse is 110/
min. BMI is 21 kg/m2. Physical examination shows dry skin, fine facial hair, and dental enamel erosions. The lungs
are clear to auscultation and heart sounds are normal. The abdomen is soft and nontender. There is no lower
extremity edema. Laboratory results are as follows:

Sodium 134 mEq/L


Potassium 2.4 mEq/L
Chloride 90 mEq/L
Bicarbonate 40 mEq/L
Urine chloride 9 mEq/L

In addition to potassium supplementation, which of the following is the best treatment for correcting the laboratory
abnormalities in this patient?

A) Acetazolamide

B) Ammonium chloride

C) Breathing into a paper bag

D) Fludrocortisone

E) Normal saline
F) Spironolactone
Explanation
Correct Answer:

E) Normal saline

Hypokalemic, hypochloremic metabolic alkalosis

Common • Gastric suction or severe vomiting


etiologies • Loop or thiazide diuretic overuse

• Gastric or renal H+ losses initiate alkalosis


• Volume depletion activates RAAS
• ↑ Renal K+ & H+ losses cause hypokalemia & worsen
alkalosis
Pathophysiology
• Relatively greater loss of Cl− than Na+ → profound Cl−
depletion
• ↓ Cl− impairs renal HCO3− excretion to perpetuate
alkalosis

• Remove or treat initiating factor


Management
• Cl− replenishment with normal saline corrects alkalosis
RAAS = renin-angiotensin-aldosterone system.

This patient's fatigue, fine facial hair (ie, lanugo), dental erosions, and excessive exercise (likely as a compensatory
behavior) are suggestive of bulimia nervosa. Bulimia nervosa is characterized by episodes of binge eating followed
by purging, most often via self-induced vomiting. Chronic vomiting leads to volume depletion (eg, near-syncope,
sinus tachycardia) and metabolic alkalosis (eg, elevated serum bicarbonate [HCO3−]).

Vomiting leads to the loss of hydrogen (H+) and chloride (Cl−) (ie, hydrochloric acid) from the gastrointestinal tract.
This results in a net gain in HCO3− that initiates metabolic alkalosis. The kidneys respond by attempting to eliminate
this alkali load; however, they are unable to do so because the low Cl− (from vomiting) impairs renal HCO3−
excretion. Therefore, Cl− depletion perpetuates metabolic alkalosis; it can be confirmed by a low urine Cl−.

Treatment with normal saline replenishes both intravascular volume and Cl−, thereby restoring the kidneys' ability
to excrete HCO3− and correcting the alkalosis (ie, saline-responsive metabolic alkalosis).

(Choice A) Acetazolamide is a carbonic anhydrase inhibitor that decreases proximal tubular reabsorption of
sodium (Na+) and HCO3−, thereby promoting mild metabolic acidosis. It is used in volume-overloaded patients who
develop loop diuretic–induced metabolic alkalosis but should be avoided in volume-depleted patients.

(Choice B) Ammonium chloride is a strong acid that can be used to treat severe metabolic alkalosis in patients
unable to receive normal saline (eg, volume overload). This volume-depleted patient should be given normal saline.

(Choice C) Breathing into a paper bag promotes mild CO2 retention and can induce a mild, temporary respiratory
acidosis, but it would not address the volume depletion and underlying chloride depletion in this patient.

(Choice D) Fludrocortisone is used as mineralocorticoid replacement for primary adrenal insufficiency (Addison
disease), which typically presents with hyperkalemia and metabolic acidosis. Although this patient's hyponatremia
and fatigue can be seen with adrenal insufficiency, hypokalemia and metabolic alkalosis are not consistent with
adrenal insufficiency.
(Choice F) Spironolactone is a mineralocorticoid antagonist used to treat mineralocorticoid excess (eg, primary
hyperaldosteronism). Primary hyperaldosteronism can cause hypokalemia and metabolic alkalosis (due to
aldosterone-mediated K+ and H+ loss); however, hypertension and high urine Cl− would be expected.

Educational objective:
Chronic vomiting leads to volume depletion and metabolic alkalosis, which is initiated by a loss of hydrogen and
perpetuated by chloride depletion. Treatment with normal saline restores intravascular volume and replenishes
chloride, thereby allowing renal elimination of bicarbonate.

Reference
• Metabolic alkalosis: a brief pathophysiologic review
Question #370

A 61-year-old man comes to the office for evaluation of urinary symptoms. Review of systems is positive for
nocturnal urinary frequency, occasional dribbling, and a weak urinary stream for the past 3 months. The patient was
diagnosed with type 2 diabetes mellitus 16 years ago. Other medical conditions include hypertension, myocardial
infarction 2 years ago, and moderately decreased visual acuity. Blood pressure is 160/100 mm Hg and pulse is 60/
min. Examination shows a left-sided carotid bruit and trace bilateral ankle edema. Postvoid bladder residual
volume is 40 mL. Dipstick urinalysis reveals 2+ protein and no blood. Serum creatinine level is 2.1 mg/dL.
Hemoglobin A1c is 7.3%. Which of the following is the most likely cause of this patient's chronic kidney disease?

A) Microangiopathy

B) Minimal change nephropathy

C) Neurogenic bladder

D) Obstructive uropathy

E) Renal papillary necrosis


Explanation
Correct Answer:

A) Microangiopathy
This patient's age, nocturia, dribbling, and weak urinary stream initially suggest benign prostatic hyperplasia (BPH).
However, laboratory evaluation shows proteinuria and elevated creatinine, which are unexpected with BPH alone.
Given his long-standing diabetes mellitus, these findings make diabetic kidney disease (DKD) the most likely
cause.

DKD is a microvascular process (microangiopathy) that predominantly affects glomeruli. It is commonly


asymptomatic in early stages but found on routine studies. This patient's risk factors include:

• long-standing diabetes mellitus (eg, >5-10 years).

• evidence of another microvascular disease (eg, retinopathy suggested by decreased visual acuity).

• poor glycemic control (ie, A1c >7.0%; higher levels recommended only for patients who are elderly or have
increased risk for hypoglycemia).

• poorly controlled hypertension (>130/80 mm Hg).

Proteinuria (reflecting glomerular damage), particularly if persistent, is an important clue for diagnosis. Although a
positive urine dipstick test revealed this patient's proteinuria, the preferred screening test is the random urine
albumin/creatinine ratio, which detects excess albuminuria earlier. Proteinuria is associated with increased risk for
cardiovascular mortality and concurrent macrovascular disease (cardiovascular disease), like that in this patient
(eg, carotid bruit, myocardial infarction).

(Choice B) Minimal change disease (MCD) causes nephrotic syndrome (proteinuria >3.5 g/day, edema,
hyperlipidemia) due to podocyte damage. However, 2+ proteinuria on urine dipstick is unlikely to represent
nephrotic-range proteinuria. MCD is more common in children. Although nephrotic syndrome can occur in diabetes
mellitus, it usually involves glomerular (eg, glomerulosclerosis) rather than podocyte damage.

(Choice C) Neurogenic bladder can cause urinary symptoms but typically results from spinal cord injury or multiple
sclerosis, is often accompanied by bladder spasms, and would not cause elevated creatinine or proteinuria.
Diabetic autonomic neuropathy can affect the bladder but causes overflow incontinence (continuous dribbling) with
high postvoid residuals; this would not explain proteinuria or elevated creatinine.
(Choice D) BPH explains this patient's urinary symptoms and increases the risk for obstructive uropathy (ie, urinary
outflow tract obstruction), but postvoid residual <50 mL is inconsistent with significant obstruction. In addition, BPH
is not associated with proteinuria and does not cause elevated creatinine unless complete obstruction occurs.

(Choice E) Renal papillary necrosis is a complication of sickle cell nephropathy, analgesic use, and diabetes
mellitus. It typically causes flank pain and hematuria.

Educational objective:
Diabetic kidney disease, a microangiopathic glomerular process suggested by persistent proteinuria, more likely
occurs in long-standing diabetes mellitus, suboptimal glycemic and blood pressure control, and other systemic
microvascular damage (eg, retinopathy). It is associated with increased risk for cardiovascular mortality.

Reference
• Albuminuria changes and cardiovascular and renal outcomes in type 1 diabetes: the DCCT/EDIC study.
Question #371

A 27-year-old man comes to the physician because of a 1-day history of fever and joint pains. He is being treated
with cephalexin for a skin infection. His urine has turned darker. His temperature is 38.5° C (101.3° F), blood
pressure is 125/70 mm Hg, pulse is 90/min, and respirations are 15/min. Examination shows a skin rash;
examination otherwise shows no abnormalities. Urinalysis shows: 8 RBCs/HPF, 12 WBCs/HPF with white cell
casts, eosinophiluria, and a mild degree of proteinuria. Laboratory studies show a BUN of 40 mg/dl and serum
creatinine of 2.2 mg/dl. Which of the following is the most appropriate next step in management?

A) Discontinue cephalexin

B) Start ampicillin and gentamicin

C) Start oral ciprofloxacin

D) Start intravenous steroids

E) Start oral steroids


Explanation
Correct Answer:

A) Discontinue cephalexin

The patient presented in this clinical vignette is most likely suffering from drug-induced interstitial nephritis. Drug-
induced interstitial nephritis occurs with many drugs such as penicillins, cephalosporins and sulfonamides. Clinical
features include fever, rash and arthralgias. Other features are peripheral eosinophilia, hematuria, sterile pyuria
and eosinophiluria. WBC casts may be present in the urine, but red cell casts are rare. Discontinuing the offending
agent is the treatment of drug-induced interstitial nephritis.

(Choice D) Steroids may hasten recovery in cases of drug-induced interstitial nephritis, but they may aggravate the
underlying infection.

(Choice C) Oral ciprofloxacin or IV ampicillin and gentamicin are used to treat acute pyelonephritis.

Educational Objective:
70% of cases with interstitial nephritis are caused by drugs such as cephalosporins, penicillins, sulfonamides,
sulfonamide containing diuretics, NSAIDs, rifampin, phenytoin, and allopurinol. Discontinuing the offending agent is
the treatment of drug-induced interstitial nephritis.
Question #372

A 16-year-old girl is brought to the office due to dark brown urine and fatigue for a week. Temperature is 37.2 C (99
F), blood pressure is 140/94 mm Hg, pulse is 80/min, and respirations are 16/min. Physical examination shows no
abnormalities. Laboratory results are as follows:

Serum chemistry
Blood urea nitrogen 24 mg/dL
Creatinine 1.7 mg/dL

Urinalysis
Specific gravity 1.016
Protein +2
Blood moderate
Leukocyte esterase negative
Nitrites negative
White blood cells 1-2/hpf
Red blood cells 20-30/hpf

Which of the following is the most appropriate next step in management?

A) Bladder ultrasound

B) CT scan of the abdomen

C) Cystoscopy
D) Hemoglobin electrophoresis

E) Repeat urinalysis in 2 weeks

F) Serum complement levels

G) Urine culture
Explanation
Correct Answer:

F) Serum complement levels

Hematuria can be due to a nonglomerular or glomerular source. The most common cause of gross hematuria in
children is nonglomerular (eg, urinary tract infection, perineal/meatal irritation, trauma). However, glomerular
disease should be suspected with any of the following:
• Red blood cell casts (pathognomonic)
• Proteinuria
• Hypertension
• Edema
• Brown, cola-colored urine (in contrast to red/pink urine with lower urinary tract bleeding)

Therefore, this patient with dark brown urine, in addition to 2+ proteinuria, hypertension, and elevated creatinine,
likely has a glomerular source of bleeding.

Initial evaluation of glomerulonephritis includes serum complement (C3, C4) levels, as well as a complete
blood count and albumin level. Low C3 is suggestive of poststreptococcal glomerulonephritis (PSGN), the most
common glomerulonephritis in children, or lupus nephritis. Antistreptolysin O (seen in PSGN) and antinuclear
antibody (seen in systemic lupus erythematosus) testing can help differentiate these conditions. If laboratory
evaluation is unrevealing and symptoms persist, renal biopsy may be indicated for diagnosis (eg, IgA nephropathy,
membranoproliferative glomerulonephritis).

(Choices A, B, and C) Bladder imaging may be indicated for painless hematuria to evaluate for a tumor; because
of radiation exposure, ultrasound, not CT scan, is typically preferred as the initial imaging study in children.
Visualization/tissue sampling via cystoscopy is diagnostic. However, bladder tumors are extremely rare in children,
cause pink/red (not brown) urine, and are not typically associated with proteinuria, hypertension, or elevated
creatinine.

(Choice D) Hemoglobin electrophoresis is diagnostic of sickle cell trait, which can cause papillary necrosis from
sickling within capillaries of the renal medulla. The classic presentation is isolated, painless, gross hematuria;
hypertension, proteinuria, and elevated creatinine are atypical. Moreover, in children, PSGN is far more common
than renal papillary necrosis.

(Choices E and G) Management of patients with hematuria and other symptoms of a urinary tract infection (eg,
dysuria, urgency, frequency) includes empiric antibiotics, urine culture, and repeat urinalysis in 2 weeks to confirm
resolution of hematuria. Urine is typically pink or red, not brown, and urinalysis classically shows pyuria (white
blood cells ≥5/hpf), leukocyte esterase, and/or nitrites, none of which is seen here. In addition, asymptomatic
microscopic hematuria may also be followed by serial urinalyses, but this patient has symptomatic gross hematuria
that warrants further evaluation.
Educational objective:
Glomerular sources of gross hematuria should be considered in a patient with brown urine, red blood cell casts,
proteinuria, hypertension, and/or edema. Initial evaluation includes serum complement (C3, C4) levels.

Reference
• Approach to the child with hematuria.
Question #373

A 53-year-old man comes to the office due to occasional red urine for the last 3 months. He states that his urine
stream appears normal initially but turns red by the end of voiding. He has also noticed small clots in his urine. The
patient has not had any fever, edema, flank pain, or weight loss. Medical history is significant for chronic back pain.
He currently smokes a pack of cigarettes daily but does not use alcohol. His temperature is 37.5 C (99.5 F) and
blood pressure is 140/90 mm Hg. Physical examination is within normal limits. Urinalysis is positive only for blood.
Which of the following is the most likely cause of this patient's symptoms?

A) Bladder disease

B) Glomerular disease

C) Nephrolithiasis

D) Polycystic kidney disease

E) Urethral injury

F) Urinary tract infection


Explanation
Correct Answer:

A) Bladder disease
Gross (ie, visible or macroscopic) hematuria can be classified based on the stage of voiding at which bleeding
predominates:

• Initial hematuria is characterized by blood at the beginning of the voiding cycle and often reflects a urethral
source.
• Total hematuria is characterized by blood during the entire voiding cycle and can reflect bleeding from
anywhere in the urinary tract (eg, bladder, kidneys).
• Terminal hematuria is characterized by blood at the end of voiding cycle and often suggests bleeding from
the prostate, bladder neck or trigone, or posterior urethra.

Urothelial (bladder) cancer is associated with blood vessel formation and often presents with painless, total
hematuria; however, terminal hematuria can also be seen if the cancer originates from the bladder neck or trigone.
This patient with several risk factors for urothelial cancer (age >40, sex, smoking) has terminal hematuria with clots
(which suggest a nonglomerular—and usually lower urinary tract—source of bleeding). Therefore, he should
undergo cystoscopy to evaluate for bladder cancer and other sources of terminal hematuria. Delays in diagnosis of
bladder cancer are associated with poor prognosis.

(Choice B) Glomerular diseases can cause nephritic syndrome with microscopic or gross hematuria. Patients can
also present with total hematuria. However, clots are unusual in glomerular disease, and urinalysis frequently
shows red blood cell casts and may show proteinuria.

(Choice C) Nephrolithiasis can cause hematuria. However, stones usually present with flank or groin pain,
depending on the location of the stone.

(Choice D) Polycystic kidney disease (PCKD) is the leading heritable cause of renal disease in adults. Although it
can cause hypertension, it usually presents as abdominal or flank pain with microscopic or gross total hematuria
and, occasionally, a bulky mass on abdominal examination. The patient does not have any of these features, and
his elevated blood pressure is more likely due to essential hypertension than to PCKD.

(Choice E) Urethritis or urethral injury (eg, Foley catheterization) typically manifests as initial hematuria.

(Choice F) All urinary tract infections (pyelonephritis, cystitis, urethritis) may present with microscopic or gross
hematuria. However, pyelonephritis usually presents with flank pain and systemic illness (eg, fever, nausea,
vomiting); cystitis and urethritis present with irritative voiding symptoms (eg, dysuria, urinary frequency, hesitancy).

Educational objective:
Initial hematuria suggests urethral damage, terminal hematuria indicates bladder or prostatic damage, and total
hematuria reflects damage anywhere in the urinary tract. Clots are not usually seen with renal causes of hematuria
(eg, glomerular diseases).
Question #374

A 50-year-old man comes to the office for a routine follow-up visit. He has hypertension, diabetes mellitus,
depression, secondary hyperparathyroidism, and end-stage renal disease. The patient has been on hemodialysis
for the past 3 years. He was admitted 3 months ago for catheter-related infection, which was treated with
antibiotics. The patient also had a right below-the-knee amputation 2 years ago following a nonhealing foot ulcer.
Physical examination shows a right carotid bruit. Over the next 5 years, this patient is at greatest risk of death from
which of the following?

A) Cardiovascular disease

B) Infection

C) Suicide

D) Venous thromboembolism

E) Voluntary withdrawal of dialysis


Explanation
Correct Answer:

A) Cardiovascular disease

Cardiovascular risk factors in ESRD patients

• Hypertension, diabetes mellitus & dyslipidemia


Traditional risk factors • Left ventricular hypertrophy
• Advanced age & low physical activity

• Anemia of chronic kidney disease


ESRD-specific risk factors • Vascular calcifications (↑ phosphorus, ↑ calcium)
• Oxidative stress related to uremia and dialysis

ESRD = end-stage renal disease.

Cardiovascular disease (CVD) (eg, coronary artery disease, stroke) is the most common cause of death in
patients with end-stage renal disease (ESRD), accounting for approximately 50% of deaths even in younger
patients. Traditional cardiovascular risk factors including hypertension (96% of patients), dyslipidemia, diabetes
mellitus, left ventricular hypertrophy, sedentary lifestyle, and advanced age (average age of dialysis initiation is >60
years) are highly prevalent in patients on dialysis.

In addition, chronic kidney disease is an independent risk factor for CVD, likely due to factors related to renal
dysfunction and/or renal replacement therapy, including the following:
• Metabolic abnormalities (eg, hyperphosphatemia) and increased calcium load (eg, vitamin D
supplementation) cause arterial calcification, coronary artery stiffening/narrowing, and left ventricular
hypertrophy (diastolic dysfunction).

• Uremia and renal replacement therapy cause oxidative stress, resulting in accelerated atherogenesis and
inhibition of nitric oxide synthesis.

• Anemia, which is common in patients with ESRD, is a risk factor for CVD and is associated with worse
outcomes in acute coronary syndrome.

CVD also accounts for significant morbidity in patients with ESRD; peripheral artery disease with extensive vascular
calcification and poor arterial supply can lead to nonhealing foot ulcers and result in amputation. Because of the
increased morbidity and mortality, patients with ESRD require aggressive management of both traditional and
renal-specific cardiovascular risk factors.

(Choice B) Infection is the second most common cause of death in patients with ESRD, accounting for
approximately 15%-20% of deaths. Dialysis catheter-related infections are common; mortality can be reduced with
appropriate antibiotic treatment and replacement of infected catheters.

(Choice C) Patients with ESRD are at increased risk for psychiatric conditions (prevalence of up to 70%),
especially depression and anxiety. The incidence of death by suicide is higher than in the general population but
much lower than death from CVD.

(Choice D) The risk of venous thromboembolism in patients with ESRD is twice that for the general population, but
death attributable to CVD is much higher.

(Choice E) Although some elderly patients with poor functional status and quality of life may withdraw from
dialysis, this is an uncommon outcome in a 50-year-old patient.

Educational objective:
Cardiovascular disease is the most common cause of death (up to 50%) in patients with end-stage renal disease
(ESRD). Contributing factors include an increased prevalence of traditional cardiovascular risk factors (eg,
hypertension, diabetes) and ESRD-related risks (eg, anemia, oxidative stress). Aggressive risk-factor modification
is required to reduce mortality.
Question #375

A 26-year-old, previously healthy woman comes to the office due to worsening fatigue, constipation, and
nonproductive cough for the past several months. The patient also reports occasional reddish, painful bumps on
her legs and nonpainful, nonpruritic facial eruptions for the past week. She has had no fever, chest pain, orthopnea,
or pedal edema. The patient does not use tobacco, alcohol, or illicit drugs and is in a monogamous relationship.
Vital signs and pulse oximetry are within normal limits. Physical examination shows normal oropharyngeal mucosa
and jugular venous pressure. The lungs are clear on auscultation and heart sounds are normal. The abdomen is
soft and nontender. There are enlarged cervical and axillary lymph nodes. Examination of the skin reveals 3
violaceous, indurated nodules on the anterior shins, as well as facial eruptions shown in the exhibit. Chest imaging
and laboratory studies are ordered. Which of the following sets of laboratory findings are expected in this patient?

Serum calcitriol Parathyroid hormone Urinary calcium

A)

Decreased Decreased Increased

B)

Decreased Increased Increased

C)

Increased Decreased Increased

D)

Increased Increased Decreased

E)
Increased Increased Increased
Explanation
Correct Answer:

C)

Increased Decreased Increased

Regulation of calcium metabolism

• ↑ Bone resorption
Parathyroid hormone • ↑ Renal reabsorption of calcium & ↓ reabsorption of phosphate
• ↑ Conversion of 25-hydroxyvitamin D to 1,25-dihydroxyvitamin D

• ↑ Intestinal absorption of calcium & phosphate


• ↑ Renal reabsorption of calcium & phosphate
Vitamin D
• ↓ Parathyroid hormone secretion
• ↑ Mineralization of bone

• ↓ Bone resorption
Calcitonin
• ↓ Renal calcium reabsorption

This young African American patient with nonproductive cough, lymphadenopathy, rash, and symptoms of
hypercalcemia (eg, constipation, fatigue) likely has sarcoidosis, a multisystem, granulomatous disorder. Skin
involvement occurs in up to 25% of patients, as seen in this patient with erythema nodosum (eg, violaceous,
indurated nodules on the shins) and papular sarcoidosis (eg, multiple scattered papular lesions) on the face.
Impairments in calcium homeostasis in sarcoidosis (and other granulomatous diseases) are primarily driven by
abnormal activity of 1-alpha hydroxylase, which normally converts calcidiol (25-dihydroxyvitamin D) to calcitriol
(1,25-dihydroxyvitamin D) only in the kidneys. In sarcoidosis, abnormal extrarenal calcitriol conversion occurs in
activated macrophages in the lungs and lymph nodes, causing a cascade of effects:

• Serum calcitriol increases, which stimulates intestinal calcium absorption, causing hypercalcemia.

• Parathyroid hormone (PTH) decreases due to negative feedback by hypercalcemia and increased
calcitriol.

• Renal calcium reabsorption decreases due to low PTH; because serum calcium is high, renal calcium
excretion increases (hypercalciuria).

Hypercalcemia in sarcoidosis results from abnormal calcitriol (not PTH) production; therefore, it is PTH-
independent. In contrast, PTH-dependent hypercalcemia involves failed negative feedback, such that PTH is
inappropriately high (unsuppressed) despite high calcitriol and hypercalcemia. Primary hyperparathyroidism is the
most common cause; urinary calcium levels in hyperparathyroidism can be increased or normal, depending on the
severity of hypercalcemia (Choices D and E).

(Choice A) Low PTH and low serum calcitriol indicate impaired PTH production, which can occur with primary
hypoparathyroidism.

(Choice B) Low calcitriol (which corresponds to low serum calcium) and hypercalciuria in the setting of high PTH
(which corresponds to high renal calcium reabsorption) suggest intrinsic renal injury blocking reabsorption (eg,
distal renal tubular acidosis).

Educational objective:
Sarcoidosis is a multisystem, granulomatous disorder that can manifest with skin findings, including papular lesions
and erythema nodosum. Hypercalcemia in sarcoidosis occurs due to extrarenal calcitriol production in the lungs
and lymph nodes and is independent of parathyroid hormone (PTH). It is associated with high serum calcitriol, low
PTH, and increased urinary calcium.
Question #376

A 58-year-old woman comes to the office due to increasing fatigue and muscle cramps over the past 2 months.
She has a 5-year history of Sjögren syndrome and reports dryness of her eyes and mouth but no other symptoms.
Blood pressure is 130/74 mm Hg, and pulse is 86/min. Physical examination shows enlarged parotid glands and
dry oral mucosa. Laboratory results are as follows:

Serum chemistry
Sodium 139 mEq/L
Potassium 3.1 mEq/L
Chloride 116 mEq/L
Bicarbonate 13 mEq/L
Creatinine 1.1 mg/dL

Arterial blood gases


pH 7.32
PaO2 100 mm Hg
PaCO2 26 mm Hg

Urine pH 6.8

Which of the following is the most likely cause of this patient's acid-base imbalance?

A) Impaired respiratory muscle function

B) Impaired urinary acid excretion


C) Increased gastrointestinal bicarbonate loss

D) Increased ketoacid production

E) Increased lactic acid production

F) Intracellular shift of bicarbonate


Explanation
Correct Answer:

B) Impaired urinary acid excretion

Metabolic acidosis

Type Normal anion gap Elevated anion gap

• Accumulation of unmeasured
Mechanism • Loss of bicarbonate
acidic compounds

• Severe diarrhea • Lactic acidosis


• Renal tubular acidosis • Diabetic ketoacidosis
Etiologies • Excess saline infusion • Renal failure (uremia)
• Intestinal or pancreatic fistula • Methanol, ethylene glycol
• CAI & MRA diuretics • Salicylate toxicity

CAI = carbonic anhydrase inhibitor; MRA = mineralocorticoid receptor antagonist.

This patient has primary metabolic acidosis, evidenced by pH <7.35 and serum bicarbonate (HCO3−) <24 mEq/L.
The normal anion gap of 10 mEq/L is consistent with nonanion gap metabolic acidosis (NAGMA), which results
from loss of HCO3−. This patient's NAGMA is likely due to type 1 renal tubular acidosis (RTA), a common
complication of Sjögren syndrome.
There are 3 major types of RTA, all of which cause NAGMA due to a net loss of HCO3−.

• Type 1 (distal) RTA results from impaired H+ excretion by alpha-intercalated cells in the distal tubule.
Hypokalemia is typically present due to reduced K+ reabsorption, and urine pH >5.5 is expected due to
markedly impaired capacity to acidify the urine.

• Type 2 (proximal) RTA results from impaired HCO3− reabsorption in the proximal tubule, and hypokalemia is
usually present. Because distal tubular H+ excretion is able to somewhat compensate for the impaired
proximal HCO3− reabsorption, urine pH is variable and often <5.5.

• Type 4 RTA results from reduced aldosterone activity, leading to impaired H+ and K+ excretion in the
collecting duct. In contrast to type 1 and type 2 RTA, hyperkalemia is typical. The urine pH is usually <5.5.

(Choice A) Impaired respiratory muscle function causes hypoventilation and primary respiratory acidosis,
evidenced by pH <7.35 but elevated PaCO2 (>40 mm Hg). This patient's low PaCO2 represents compensatory
respiratory alkalosis for primary metabolic acidosis.

(Choices C and F) Increased gastrointestinal HCO3− loss (eg, severe diarrhea) is a common cause of NAGMA.
Intracellular shift of HCO3− occurs with infusion of excess sodium chloride and leads to NAGMA. These and most
other etiologies of acidosis occur with intact distal tubular H+ excretion; therefore, low urine pH (<5.5) is expected as
the kidneys can appropriately acidify the urine and excrete excess acid. This patient's high urine pH in the setting of
metabolic acidosis suggests a defect in renal acid handling.

(Choices D and E) Anion gap metabolic acidosis (AGMA) results from the accumulation of unmeasured acidic
compounds that increase the anion gap. Increased ketoacid production is responsible for AGMA in severe insulin
deficiency (ie, diabetic ketoacidosis), and increased lactic acid production causes AGMA during periods of poor
organ and tissue perfusion (eg, sepsis).

Educational objective:
Renal tubular acidosis (RTA) involves a net loss of HCO3− and is a common cause of nonanion gap metabolic
acidosis. Type 1 RTA results from impaired H+ excretion in the distal tubule and can be recognized by
inappropriately high urine pH (>5.5) in the setting of acidosis.
Reference
• Hyperchloremic acidosis.
Question #377

An 18-year-old African American man comes to the office due to several months of excessive urination. The patient
voids every few hours during the day and wakes 2 or 3 times per night to urinate despite restricting his evening fluid
intake. He says that his urine appears clear and has no visible blood. Medical history is significant for seasonal
allergies, for which he frequently takes an antihistamine. He takes no other medications and does not use tobacco,
alcohol, or illicit drugs. The patient is sexually active and does not use condoms. He was adopted at a young age;
records indicate that his birth mother had sickle cell disease and died from a stroke at age 32. The patient is
afebrile and normotensive. Hematocrit is 41% and serum sodium is 138 mEq/L. Urinalysis of a first-morning
specimen reveals the following:

Specific gravity 1.001 (normal: 1.010-1.030)


pH 6.6 (normal: 4.5-8)
Protein none
Blood negative
Glucose negative
Ketones negative
Leukocyte esterase negative
Nitrites negative

This patient's polyuria is most likely caused by which of the following?

A) Central diabetes insipidus

B) Chlamydia infection

C) Hyposthenuria
D) Medication adverse effect

E) Primary polydipsia

F) Type 1 diabetes mellitus

G) Type 2 diabetes mellitus


Explanation
Correct Answer:

C) Hyposthenuria

Sickle cell trait

• Usually asymptomatic
Clinical features
• No change in overall life expectancy

• Normal hemoglobin, reticulocyte count, RBC indices & morphology


Laboratory findings
• Hemoglobin electrophoresis: Hb A > Hb S

• Hematuria/papillary necrosis, hyposthenuria


Complications • Splenic infarction (especially at higher altitudes), venous thromboembolism, priapism
• Exertional rhabdomyolysis

Hb A = hemoglobin A; Hb S = hemoglobin S; RBC = red blood cell.

This patient has a low urine specific gravity consistent with hyposthenuria, the inability of the kidneys to
concentrate urine. His family history is significant for early-onset stroke, which can be a severe complication of
sickle cell disease (SCD) due to cerebrovascular occlusion from sickling; however, he has a normal hematocrit level
and no other symptoms. Therefore, sickle cell trait (SCT) is the most likely cause of this patient's hyposthenuria.
Hyposthenuria is common in patients with SCD and may also develop in those with SCT. In response to hypoxic,
hyperosmolar conditions of the renal medulla, red blood cells sickle in the vasa recta, impairing free water
reabsorption and countercurrent exchange. Patients typically have polyuria and nocturia despite fluid restriction.
Urine osmolality is low; however, normal serum sodium is maintained due to intact antidiuretic hormone (ADH).
Urinary diluting capacity is also intact as it is a function of the superficial loop of Henle, which is not supplied by the
vasa recta.

Typically, mild hyposthenuria due to SCT requires no treatment. In patients with SCD, red blood cell transfusions
often improve urine-concentrating ability and provide relief of symptoms.

(Choice A) Central diabetes insipidus causes polyuria and hyposthenuria due to insufficient ADH production.
However, thirst mechanism is impaired and serum sodium is elevated in patients with central diabetes insipidus.

(Choice B) Chlamydia infection in men can cause dysuria and urethral discharge. Impaired urine-concentrating
ability is not associated with chlamydia infection, and urinalysis would likely be positive for leukocyte esterase.

(Choice D) Antihistamines (eg, diphenhydramine) treat allergic rhinitis and can cause anticholinergic adverse
effects such as dry mouth, tachycardia, and urinary retention, none of which is consistent with this patient's
presentation.

(Choice E) Primary (psychogenic) polydipsia causes hyposthenuria due to excessive water intake. However,
serum sodium is typically low, and this condition is unlikely in a patient restricting evening fluid intake.

(Choices F and G) Diabetes mellitus is characterized by elevated serum glucose due to lack of insulin (type 1) or
insulin resistance (type 2). When serum glucose exceeds the capacity of kidney resorption, glucose draws water
into the collecting system, causing polyuria and glucosuria. Negative urine glucose makes this diagnosis unlikely.

Educational objective:
Hyposthenuria is the inability of the kidneys to concentrate urine and can occur in patients with sickle cell disease
and sickle cell trait. Patients have polyuria, low urine specific gravity, and normal serum sodium.

Reference
• Sickle cell disease: renal manifestations and mechanisms.
Question #378

A 34-year-old woman comes to the office due to excessive urination at night. For the past 2 weeks, the patient has
been getting up at least 3 times at night to urinate. She has a history of hypothyroidism treated with levothyroxine
and bipolar disorder treated with lithium carbonate. Blood pressure is 134/78 mm Hg and pulse is 88/min. BMI is
27 kg/m2. Physical examination shows decreased skin turgor and dry mucous membranes. Laboratory results after
overnight fasting are as follows:

Serum chemistry
Sodium 146 mEq/L
Potassium 4 mEq/L
Chloride 106 mEq/L
Bicarbonate 26 mEq/L
Blood urea nitrogen 26 mg/dL
Creatinine 1.1 mg/dL
Calcium 10.0 mg/dL
Glucose 112 mg/dL

Osmolality, serum 308 mOsmol/kg H2O

Thyroid function tests


Thyroid stimulating hormone 7.0 µU/mL

Osmolality, urine 200 mOsmol/kg H2O

This patient's symptoms are most likely caused by a defect in which of the following sites?
A) Hypothalamus

B) Pancreatic islet cells

C) Posterior pituitary lobe

D) Proximal tubule of the kidney

E) Renal collecting ducts

F) Thyroid follicular cells


Explanation
Correct Answer:

E) Renal collecting ducts

Causes of polyuria & dilute urine

Primary polydipsia Central DI Nephrogenic DI

• ADH independent • ADH deficiency


• ADH resistance (renal
Cause (excessive water (CNS
disease)
intake) pathology)

• Idiopathic
• Chronic lithium use
• Antipsychotics (dry • Trauma
• Hypercalcemia
mouth) • Pituitary
Etiology • Hereditary (AVPR2 &
• Psychiatric surgery
aquaporin 2
conditions • Ischemic
mutations)
encephalopathy

Results
• Low urine
of water • High urine osmolality • Low urine osmolality
osmolality
deprivation
Response to • Increased urine
• No change • No change
desmopressin osmolality

ADH = antidiuretic hormone; AVPR2 = vasopressor V2; DI = diabetes insipidus.

This patient with nocturia and persistently dilute urine (urine osmolality <300 mOsmol/kg) with overnight fasting
likely has lithium-induced nephrogenic diabetes insipidus (DI).

Normally, antidiuretic hormone (ADH) stimulates renal water reabsorption when serum osmolality is high (eg,
following water deprivation). ADH resistance or deficiency causes DI, which presents with dilute urine (low urine
osmolality) and polyuria (often worse at night).

• Nephrogenic DI is caused by renal ADH resistance. One of the most common causes in adults is lithium,
which accumulates in renal collecting ducts and blocks responsiveness to ADH. After water deprivation
(eg, overnight fasting), serum osmolality rises (eg, >295 mOsmol/kg, often with hypernatremia) and ADH
increases appropriately; however, urine osmolality remains low.

• Central DI can occur following hypothalamic or posterior pituitary damage (eg, trauma, infection) and leads
to deficient ADH secretion. After water deprivation, serum osmolality rises but ADH remains low; therefore,
urine osmolality remains low. Lithium use is only rarely associated with central DI (Choice C).

A desmopressin challenge can differentiate central from nephrogenic DI (as desmopressin causes increased urine
osmolality only in central DI).

(Choice A) Psychogenic polydipsia is classically associated with psychiatric conditions or disorders affecting thirst.
Polyuria and dilute urine are seen; however, in contrast to DI, they represent appropriate responses to excessive
water intake, and both ADH sensitivity and production are intact. Therefore, following water deprivation, ADH rises
and urine becomes concentrated (ie, urine osmolality increases >300 mOsmol/kg). This patient's dilute urine
following water deprivation makes psychogenic polydipsia unlikely.
(Choice B) Pancreatic cell destruction causes type 1 diabetes mellitus; severe hyperglycemia (eg, diabetic
ketoacidosis) can precipitate glucosuria and associated osmotic diuresis. However, osmotic diuresis is associated
with high, rather than low, urine osmolality, and this patient's fasting blood sugar is only mildly elevated.

(Choice D) Sodium-glucose cotransporter-2 (SGLT-2) inhibitors are diabetic medications that act on the proximal
tubule of the kidney, leading to increased excretion of glucose and associated osmotic diuresis. This patient has no
history of SGLT-2 inhibitor use, and her low urine osmolality makes osmotic diuresis unlikely.

(Choice F) This patient's elevated TSH suggests suboptimally treated hypothyroidism (possibly from chronic
lithium therapy). However, hypothyroidism is not associated with nocturia or persistently dilute urine.

Educational objective:
Chronic lithium therapy is commonly associated with nephrogenic diabetes insipidus (DI), presenting with polyuria
and persistently dilute urine. Lithium-induced nephrogenic DI results from lithium accumulation in the renal
collecting ducts, which leads to antidiuretic hormone resistance and impaired renal water reabsorption.

Reference
• Pathophysiology, diagnosis and management of nephrogenic diabetes insipidus.
Question #379

An 88-year-old woman is evaluated for progressive decrease in appetite and weight loss of 4.5 kg (10 lb) over the
past 3 months. She has had no nausea, vomiting, abdominal pain, or diarrhea. Medical history is significant for a
long history of hypertension and slowly declining kidney function. Her medications have not been changed or
adjusted over the past 6 months. The patient does not use tobacco or alcohol. Temperature is 37.1 C (98.8 F),
blood pressure is 140/90 mm Hg, and pulse is 78/min. BMI is 18 kg/m2. She appears thin and frail. Examination
shows 1+ bilateral lower extremity edema. Mental status examination shows a normal affect. Cognition is intact.
Laboratory studies are as follows:

Complete blood count


Hemoglobin 10.3 g/dL
Mean corpuscular volume 90 µm3
Platelets 240,000/mm3
Leukocytes 7,500/mm3

Serum chemistry
Sodium 140 mEq/L
Potassium 4.6 mEq/L
Bicarbonate 22 mEq/L
Blood urea nitrogen 40 mg/dL
Creatinine 3.2 mg/dL
Calcium 8 mg/dL
Glucose 100 mg/dL
eGFR 11 mL/min/1.73 m2
Serum TSH and liver function studies are normal. Fecal occult blood testing is negative. What is the most
appropriate next step in the management of weight loss in this patient?

A) Nutritional supplementation

B) Trial of a selective serotonin reuptake inhibitor

C) Upper and lower gastrointestinal endoscopy

D) Upper gastrointestinal series with small bowel follow-through

E) Whole-body CT scan with contrast


Explanation
Correct Answer:

A) Nutritional supplementation

Unintentional weight loss in elderly patients

Definition • >5% involuntary weight loss over 6-12 months

• Malignancy, depression
• Chronic end-organ disease (eg, CKD, COPD)
Causes • Neurologic disease (eg, oropharyngeal dysphagia)
• Social factors (eg, poverty, isolation)
• Diminished smell & taste, poor dentition

CKD = chronic kidney disease; COPD = chronic obstructive pulmonary disease.

Unintentional weight loss is generally defined as a >5% involuntary weight loss over 6-12 months. There are
many causes of unintentional weight loss in elderly patients (eg, age >75), including malignancy, depression, social
isolation, dysphagia, diminished sense of taste and smell, and advanced end-organ disease.

Advanced end-organ disease contributes to weight loss via components of both anorexia (ie, loss of appetite) and
cachexia (ie, inflammatory loss of muscle mass). Chronic heart failure, chronic kidney disease (CKD), and
chronic obstructive pulmonary disease are among the most notable conditions associated with this phenomenon.
This patient's initial workup, including history, physical examination, and basic laboratory testing (including TSH,
fecal occult blood testing), is unremarkable other than findings of advanced CKD (ie, mild normocytic anemia,
estimated glomerular filtration rate <15 mL/min/1.73 m2).

In such patients without findings to suggest malignancy (eg, blood in the stool, hypercalcemia, unexplained
anemia), watchful waiting is typically most appropriate. This is especially true for elderly patients with limited life
expectancy (such as this frail, 88-year-old patient) and other explanations for weight loss (eg, CKD) because
watchful waiting avoids diagnostic testing that is unlikely to improve quality of life or life expectancy.
Furthermore, unnecessary diagnostic testing may increase patient anxiety, physical discomfort, and financial
stress. In addition, such patients with evidence of malnutrition (eg, BMI <18.5 kg/m2) often benefit from nutritional
supplementation (eg, high-calorie supplement drinks), which can help reverse cachexia progression (protein-
energy wasting).

(Choice B) Selective serotonin reuptake inhibitors (eg, sertraline) are used to treat major depressive disorder
(MDD), a common cause of anorexia and unintentional weight loss. In elderly patients, MDD may also present with
cognitive difficulties. However, this patient's normal affect and cognition make MDD unlikely.

(Choices C and D) Upper and lower gastrointestinal endoscopy and upper gastrointestinal series with small bowel
follow-through are used to evaluate for malignancy (eg, gastric cancer, colon cancer) and other causes of
unintentional weight loss (eg, peptic ulcer disease). However, these tests are not indicated at this time because this
frail, elderly patient has no signs or symptoms to suggest an underlying gastrointestinal disorder (eg, abdominal
pain, blood in the stool), and her weight loss and normocytic anemia are likely explained by advanced CKD.

(Choice E) Whole-body CT scan is more likely to reveal incidental findings than an occult malignancy in this patient
without localizing signs or symptoms and is not indicated.

Educational objective:
Advanced end-organ disease (eg, chronic kidney disease) is a common cause of unintentional weight loss in elderly
patients. In the absence of signs, symptoms, or unexplained laboratory findings to suggest malignancy, watchful
waiting and nutritional supplementation are appropriate in such patients.

Reference
• Wasting in chronic kidney disease.
• Unintentional weight loss in older adults.
Question #380

A 60-year-old man comes to the office for follow-up of hypertension and type 2 diabetes mellitus. The patient
reports that his blood pressure is usually 110-130/70-85 mm Hg and his fasting glucose 85-95 mg/dL when he
measures them at home. He has had no headaches, chest pain, or vision changes but reports increased urinary
frequency and nocturia 2-3 times per night over the past 3 months. The patient has had no dysuria but says it takes
him longer to finish urinating and he often urinates again within 1-2 hours. His current medications are metformin
and lisinopril. Temperature is 37 C (98.6 F), blood pressure is 122/78 mm Hg, and pulse is 68/min. BMI is 23
kg/m2. Physical examination shows normal funduscopy findings, normal heart and lung sounds, and a soft and
nontender abdomen with a palpable bladder. Rectal examination reveals a smooth, enlarged prostate with no
induration. Today's and previous laboratory results are as follows:

Today 6 months ago 12 months ago 24 months ago


Hemoglobin A1c (%) 6.0 5.8 6.1 6.5
Serum creatinine (mg/dL) 1.5 1.3 1.1 1.0

Urine studies reveal no significant albuminuria, hematuria, or pyuria. Which of the following is the best next step in
management of this patient?

A) Additional antihypertensive therapy

B) Lisinopril discontinuation

C) Percutaneous kidney biopsy

D) Renal ultrasonography

E) Stricter glycemic control


Explanation
Correct Answer:

D) Renal ultrasonography

This patient with lower urinary tract symptoms (eg, urinary frequency, nocturia, impaired flow) consistent with
benign prostatic hyperplasia (BPH) has a slowly rising creatinine level; urinalysis does not show evidence of
albuminuria, hematuria, or pyuria, making intrinsic kidney disease less likely. This presentation raises suspicion for
BPH-induced obstructive uropathy (eg, enlarged prostate, palpable bladder), which may result in permanent
kidney damage due to blockage of free flow of urine.

With obstructive uropathy, a renal ultrasound examination (which should be performed in all patients being
evaluated for creatinine elevation or chronic kidney disease) typically reveals hydronephrosis; it can also help
assess the extent of kidney injury. If irreversible kidney damage (eg, cortical atrophy on sonogram due to increased
pressure) has not yet occurred, management of BPH may improve creatinine levels.

In this patient, other possible causes of the rising creatinine level are diabetic and hypertensive nephropathies.
However, his hemoglobin A1c and blood pressure measurements are within goal, and he has no evidence of
albuminuria or retinopathy. Therefore, his creatinine elevation is not likely caused by diabetes or hypertension, and
additional antihypertensive therapy and stricter glycemic control are not indicated (Choices A and E).

(Choice B) ACE inhibitors (eg, lisinopril) often cause an elevation in creatinine level, but this elevation would be
rapid and would occur within 3-5 days of starting the medication, not over 12-24 months, as seen in this patient.

(Choice C) A kidney biopsy would be appropriate to evaluate glomerular hematuria (eg, red cell casts and
dysmorphia), nephrotic syndrome (eg, heavy proteinuria), or other acute kidney damage that has not been
explained by less invasive testing. It can be considered in this patient if renal ultrasonography is unrevealing.
However, skipping an ultrasound examination and performing a biopsy to determine the cause of a rising serum
creatinine level in the absence of any additional laboratory abnormalities would not be appropriate.

Educational objective:
Benign prostatic hyperplasia can cause an obstructive uropathy that presents as a slowly rising creatinine level. A
renal ultrasound examination, which should be performed in all patients being evaluated for creatinine elevation or
chronic kidney disease, typically reveals hydronephrosis and may help assess the extent and reversibility of kidney
injury.

Reference
• Approach to managing elevated creatinine.

• Diagnosis and management of non-dialysis chronic kidney disease in ambulatory care: a systematic review
of clinical practice guidelines.
Question #381

A 70-year-old man comes to the physician complaining of 4-6 months of almost continuous urinary dribbling. It
occurs during the day and at night and is progressively worsening. He has no dysuria or hematuria. The patient
has a 20-year history of type 2 diabetes, hypertension, alcoholic hepatitis, and coronary artery disease. He had a
gastric emptying study done a few weeks ago due to continuous nausea and early satiety. He had laser
photocoagulation of both eyes for diabetic retinopathy. The patient has a 50-pack-year smoking history and drinks
4-6 beers daily. His medications include insulin glargine, lispro insulin, metformin, aspirin, metoprolol, lisinopril, and
erythromycin. Physical examination shows a normal-size prostate, decreased sensation in both legs below the
knees, and absent Achilles tendon and knee reflexes bilaterally. Postvoid residual volume is 550 mL.

Urinalysis results are as follows:

Specific gravity 1.020


Blood Trace
Glucose Positive
Ketones Negative
Protein Moderate
Leukocyte esterase Negative
Nitrites Negative
White blood cells 1-2/hpf
Red blood cells 3-4/hpf

Which of the following is the most likely cause of this patient's incontinence?

A) Diabetic nephropathy
B) Neurogenic bladder dysfunction

C) Overflow incontinence due to medication

D) Overflow incontinence from bladder outlet obstruction

E) Pelvic floor muscle weakness

F) Urethral instability

G) Urinary diverticulum

H) Urinary fistula

I) Urinary tract infection


Explanation
Correct Answer:

B) Neurogenic bladder dysfunction

Clinical features of diabetic autonomic neuropathy

• Tachycardia, impaired exercise tolerance


Cardiovascular
• Postural hypotension with loss of diurnal blood pressure variation

• Dry skin, pruritus, callus formation


Peripheral
• Foot ulcers & poor wound healing
nerves
• Charcot arthropathy (increased fracture risk with resultant secondary ulceration)

• Gastroparesis with delayed gastric emptying


Gastrointestinal • Esophageal dysmotility with possible dyspepsia
• Intestinal involvement with possible diarrhea, constipation, or fecal incontinence

• Erectile dysfunction & retrograde ejaculation in men, decreased libido & dyspareunia in
women
Genitourinary • Decreased ability to sense full bladder leading to incomplete emptying & decreased
urination
• Eventual recurrent urinary tract infections &/or overflow incontinence (eg, dribbling, poor
urinary stream)

This patient's presentation is consistent with likely diabetic autonomic neuropathy (DAN) affecting the genitourinary
tract. Risk factors for DAN include poor glucose control and other vascular risk factors (eg, hypertension, elevated
triglycerides, smoking, obesity). This patient's microvascular complications (peripheral neuropathy, retinopathy, and
nephropathy) indicate poorly controlled diabetes.

Parasympathetic innervation of the bladder regulates detrusor muscle contraction and internal sphincter relaxation
to allow for voiding urine. DAN can cause a neurogenic bladder with decreased ability to sense a full bladder,
incomplete emptying, urinary retention, and distended bladder. Patients with a higher bladder than urethral
pressure develop overflow incontinence and lose urine until the pressures equalize. The symptoms can occur
cyclically both day and night. Physical examination may show a distended bladder with high post-void residual
urine volume (>50 mL).

(Choice A) This patient's urinalysis showing proteinuria suggests glomerular disease due to diabetic nephropathy.
However, diabetic nephropathy does not typically cause urinary incontinence.

(Choice C) Medications (eg, anticholinergics, antipsychotics, tricyclic antidepressants, sedative-hypnotics) can


block the parasympathetic pathways that initiate micturition and lead to overflow incontinence. However, this
patient's current medications are not usually associated with incontinence.

(Choice D) Bladder outlet obstruction due to an enlarged prostate is the most common cause of overflow
incontinence in men. This patient's normal prostate exam makes this less likely than a neurogenic cause,
especially in the setting of retinopathy, neuropathy and nephropathy.

(Choice E) Pelvic floor weakness commonly causes stress incontinence in women. Patients develop incontinence
due to increased intraabdominal pressure after coughing or sneezing. Stress incontinence can occur in older men
after a radical or transurethral prostatectomy. However, patients with this condition usually have normal post-void
residual volume.

(Choice F) Urethral instability refers to involuntary fluctuations in the urethral pressure with or without urinary
incontinence. This can occur even in normal individuals. However, it usually does not cause urinary retention or a
high post-void residual volume.

(Choice G) Urinary diverticulum is an outpouching of the bladder or urethra into the adjacent tissues. It is more
common in women than men. Patients usually develop post-void dribbling, dysuria, and dyspareunia. The urinary
stasis can lead to possible recurrent cystitis/urinary tract infection but does not cause a high post-void residual
volume.

(Choice H) Urinary fistula is a communication between the urethra/bladder and uterus or vagina. It occurs more
commonly in women after pelvic surgery. Patients develop continuous leakage of urine without significant urinary
retention.

(Choice I) Urinary tract infection may cause incontinence without typical dysuria in patients with neuropathy, but
patients will still have an abnormal urinalysis (eg, pyuria, positive nitrite/leukocyte esterase) and sometimes
systemic symptoms (eg fever, chills).

Educational objective:
Diabetic autonomic neuropathy can affect the genitourinary tract to cause a neurogenic bladder with urinary
retention and distended bladder. Patients can then develop overflow incontinence (eg, dribbling, poor urinary
stream) with a high post-void residual volume.

Reference
• Bladder dysfunction in diabetes mellitus.
Question #382

A 65-year-old man comes to the office due to right upper extremity pain after falling on his right outstretched hand 2
days ago. The patient has mild abrasions over his wrist and elbow with full range of motion and no tenderness over
these joints. He has severe pain when he tries to lift his arm above shoulder level or when he pulls or pushes with
his right arm. He has been unable to sleep the last 2 nights due to pain and is unable to lie on the affected side.
The patient has a 40-pack-year smoking history. His father died of multiple myeloma. Vital signs are normal.
Examination shows limited active abduction and external rotation of the humerus; however, passive motion is
comparable to the contralateral side. Palpation of the right shoulder, collarbone, and neck reveal no step-off
deformities or point tenderness. Radial pulse, extremity sensation, muscle tone, and bulk are normal. Which of the
following would most likely confirm the diagnosis in this patient?

A) Bone scan

B) Chest x-ray

C) CT scan of the shoulder

D) MRI of the cervical spine

E) MRI of the shoulder

F) Shoulder x-ray
Explanation
Correct Answer:

E) MRI of the shoulder

Rotator cuff tendinopathy & tear

• Pain with abduction, external rotation


Rotator cuff impingement or • Subacromial tenderness
tendinopathy • Normal range of motion with positive impingement tests (eg,
Neer, Hawkins)

• Similar to rotator cuff tendinopathy


Rotator cuff tear • Weakness with abduction & external rotation
• Age >40

This patient has post-traumatic shoulder pain with limited active abduction, most likely due to a rotator cuff tear.
Rotator cuff tears are most common in patients age >40, often after a fall on an outstretched arm. Patients typically
have pain at the lateral shoulder that is worsened with raising the arm overhead or with external rotation and
abduction. Examination shows the drop arm sign (inability to smoothly control shoulder adduction) and weakness
with external rotation (passive range of motion is normal). Rotator cuff tendinopathy (tendinitis) may also present
with pain on abduction, but tendinopathy without tear does not cause weakness.

X-ray of the shoulder in a rotator cuff tear is often performed as an initial study to rule out fracture; in large, chronic
tears it may demonstrate migration of the humeral head, but it has low sensitivity for confirming the diagnosis of an
acute tear (Choice F). MRI can visualize the soft tissues around the humeral head and can accurately diagnose a
rotator cuff tear. Treatment of an acute tear usually involves surgery, with best results if performed within 6 weeks
of the injury.

(Choice A) Bone scans are most often used to diagnose infectious (eg, osteomyelitis) or metastatic disease.
However, these conditions would not usually present with acute post-traumatic pain and decreased range of motion.

(Choice B) With this patient's extensive smoking history, lung cancer involving the brachial plexus (Pancoast
syndrome) could cause shoulder pain. However, this would typically present with radicular pain radiating to the
wrist, atrophy of the muscles, and additional compressive symptoms (eg, edema).

(Choice C) CT of the shoulder is effective in diagnosing fractures and other disorders involving cortical bone but is
less sensitive than MRI for visualization of tendons and other soft tissue structures.

(Choice D) MRI of the spine would identify the cause of cervical radiculopathy. This condition usually results from
disc herniation (or osteophytes) and presents with arm weakness, paresthesias, and neck pain.

Educational objective:
Rotator cuff tears occur most commonly in patients age >40, often after a fall on an outstretched arm. These
injuries are characterized by pain and weakness with abduction and external rotation of the humerus. MRI can
confirm the diagnosis.

Reference
• The diagnostic accuracy of MRI for the detection of partial- and full-thickness rotator cuff tears in adults.
Question #383

A 45-year-old woman comes to the office due to right hand and wrist pain. The patient has sharp pain at the lateral
wrist and thumb that began 6 weeks ago after an out-of-state trip during which she played slot machines at a
casino. The pain is worse when opening jars. There is no history of falls or recent trauma. The patient has no
fever, pain in other joints, or recent illness. Medical history is unremarkable, but she has a history of alcohol use
disorder and smokes a pack of cigarettes a day. Vital signs are normal. Examination shows tenderness of the
lateral wrist at the radial styloid. With the hand held in ulnar deviation, passive flexion of the thumb elicits severe,
sharp pain. Which of the following is the most appropriate next step in this patient?

A) Erythrocyte sedimentation rate

B) MRI of the wrist

C) No additional testing

D) Nucleic acid amplification test of vaginal swab

E) X-ray of the hand and wrist


Explanation
Correct Answer:

C) No additional testing

De Quervain tendinopathy

• Myxoid degeneration of tendons/tendon sheaths


Pathogenesis
• Involves abductor pollicis longus & extensor pollicis brevis

• Lateral hand/wrist pain


Symptoms
• Provoked by lifting objects, thumb or wrist movement

• Based on clinical features (eg, positive provocative testing)


Diagnosis
• Tenderness at radial styloid
• Pain with thumb flexion + ulnar wrist deviation

• Thumb spica splint


Management • Nonsteroidal anti-inflammatory drugs
• Persistent/severe cases: corticosteroid injection

This patient has de Quervain tendinopathy (DQT), an overuse syndrome involving the tendons of the abductor
pollicis longus and extensor pollicis brevis. DQT typically presents with lateral hand and wrist pain often
aggravated by resisted thumb abduction or extension (eg, lifting an infant under the arms) or forceful radial/ulnar
deviation (eg, opening jar lids, working with hand-tools).

The diagnosis of DQT is based on clinical features. Examination typically shows tenderness at the radial styloid
and lateral wrist at the base of the hand. In addition, the pain can be reproduced with provocative maneuvers that
stretch the affected tendons by flexing the thumb with the wrist in ulnar deviation (eg, Finkelstein/Eichhoff tests).
In such cases, no additional testing is needed. Conservative management with nonsteroidal anti-inflammatory
drugs (eg, ibuprofen) and thumb spica splinting is usually adequate.

(Choice A) Inflammatory markers (eg, erythrocyte sedimentation rate, C-reactive protein) are elevated in the
inflammatory arthropathies (eg, rheumatoid arthritis), which typically present with swelling of the wrist and hand
joints. Most, although not all, cases are polyarticular and symmetric. Inflammatory markers are normal in DQT;
they are not indicated for this patient with a clinical examination consistent with DQT.

(Choices B and E) Imaging is not generally necessary in DQT. X-ray is typically normal or may reveal common
degenerative changes that can be misleading. Ultrasonography and MRI may reveal thickening of the involved
tendons, swelling of the tendon sheath, or peritendinous edema but add little to examination findings.

(Choice D) Nucleic acid amplification testing of vaginal and pharyngeal secretions for Neisseria gonorrhoeae is
useful for ruling out disseminated gonococcal infection, which can cause tenosynovitis but typically involves multiple
tendon systems and is characteristically associated with fever, polyarthralgia, and pustular skin lesions.

Educational objective:
De Quervain tendinopathy typically presents with lateral hand and wrist pain, often aggravated by resisted thumb
abduction or extension or forceful radial/ulnar deviation. The diagnosis is based on clinical features, including
tenderness at the lateral wrist and reproduction of pain with provocative maneuvers (eg, flexing the thumb with the
wrist in ulnar deviation).

Reference
• Finkelstein's test is superior to Eichhoff's test in the investigation of de Quervain's disease.
Question #384

A 35-year-old man comes to the office due to right ankle pain. The pain has been present for the past 3 months,
worsens with walking or running, and improves slightly with rest. There is no history of trauma, although the patient
has been training for a marathon for the past 6 months. He took over-the-counter naproxen for a week, which
provided little help. The patient has no chronic medical conditions. Vital signs are normal. Examination shows
tenderness at the right Achilles tendon 4 cm proximal to its insertion. No swelling, redness, or deformity is
apparent. Plantar flexion is intact. Which of the following would be the most appropriate therapy?

A) Eccentric calf-strengthening exercises

B) Local corticosteroid injection

C) One month of oral NSAIDs

D) Oral corticosteroids

E) Surgical repair
Explanation
Correct Answer:

A) Eccentric calf-strengthening exercises

Achilles tendinopathy

• Athletic activity, increase in activity


Risk factors • Systemic disorders: psoriasis, ankylosing spondylitis
• Medications: glucocorticoids, fluoroquinolones

• Swelling, warmth, pain at posterior heel


Clinical
• Tendon rupture: "popping" sensation & acute pain following rapid acceleration/direction
features
change

Examination • Swelling, tenderness 2-6 cm proximal to tendon insertion


findings • Rupture: positive Thompson test*

• Clinical findings
Diagnosis • Ultrasound: swelling, neovascularization
• MRI
• Acute: activity modification, ice, NSAIDs
Management
• Chronic: eccentric resistance exercises

*With the patient prone and feet off the end of the table, squeeze the calf muscles; the absence of plantar flexion
indicates tendon rupture.

NSAIDs = nonsteroidal anti-inflammatory drugs.

This patient most likely has Achilles tendinopathy (AT). AT is often referred to as a tendinitis, though true
inflammation is not always present. AT occurs most commonly in patients who abruptly increase athletic activity,
particularly in running and jumping sports. The risk is also increased following use of fluoroquinolone antibiotics,
and these patients have an increased risk of tendon rupture.

The diagnosis is based primarily on clinical findings. AT presents with ankle pain, swelling, and tenderness that are
usually most prominent approximately 2-6 cm proximal to the insertion of the tendon, where perfusion of the
gastrocnemius/Achilles tendon complex is the lowest. Management of acute AT primarily includes activity
modification, cold compresses/icing, and nonsteroidal anti-inflammatory drugs. For chronic symptoms, a
rehabilitation program emphasizing eccentric resistance exercises (ie, lengthening the muscle while under load)
is recommended.

(Choices B and D) Corticosteroid injections can be helpful for AT, but the benefits are inconsistent and carry a risk
of tendon rupture. Systemic glucocorticoids (eg, prednisone) also increase the risk of tendon rupture and are not
recommended.

(Choice C) A short trial (<10 days) of a nonsteroidal anti-inflammatory drug (NSAID) is reasonable for Achilles
tendinopathy, especially for acute symptoms. However, this patient already attempted NSAIDs and had no relief;
additional NSAID treatment is unlikely to be helpful.

(Choice E) Surgical repair can be considered for patients with complete tendon rupture who anticipate returning to
strenuous occupation or athletic activity. It is not indicated for uncomplicated tendinopathy and is frequently
unnecessary for patients who are able to participate in a prolonged rehabilitation program.

Educational objective:
Achilles tendinopathy (AT) presents with ankle pain and tenderness in the tendon proximal to its insertion on the
calcaneus. It is most common in patients who abruptly increase athletic activity. Management of acute AT includes
activity modification, cold compresses/icing, and nonsteroidal anti-inflammatory drugs. For chronic symptoms, a
rehabilitation program emphasizing eccentric resistance exercises is recommended.

Reference
• Management of chronic tendon injuries.
Question #385

A 75-year-old man comes to the office for a routine preventive examination. He has no ongoing symptoms. The
patient's medical history is significant for osteoarthritis, hypertension, and hyperlipidemia. Current medications
include hydrochlorothiazide and atorvastatin. He does not smoke and consumes 1 or 2 beers on weekends. The
patient walks 2 miles every morning and eats a balanced diet. Vital signs are within normal limits. His
cardiopulmonary examination is normal, and his abdomen is soft and nontender. Rectal examination shows a
diffusely enlarged prostate without nodules. The distal interphalangeal joints are enlarged, and his gait is normal.
Laboratory results are as follows:

Serum chemistry
Creatinine 0.8 mg/dL
Calcium 8.8 mg/dL
Serum total prostate-specific 2.1 ng/mL (≤6.5 ng/
antigen mL)

Liver function studies


Total bilirubin 1.0 mg/dL
Alkaline phosphatase 420 U/L
Aspartate aminotransferase 20 U/L
Alanine aminotransferase 25 U/L

Which of the following is the most likely cause of the elevated alkaline phosphatase in this patient?

A) Adverse effect of medication

B) Alcohol use
C) Metastatic bone disease

D) Paget disease of bone

E) Plasma cell neoplasia


Explanation
Correct Answer:

D) Paget disease of bone

Paget disease of bone

• Most patients are asymptomatic


• Bone pain & deformity
Clinical ◦ Skull: headache, hearing loss
features ◦ Spine: spinal stenosis, radiculopathy
◦ Long bones: bowing, fracture, arthritis of adjacent joints
• Giant cell tumor, osteosarcoma

• Osteoclast dysfunction
Pathogenesis
• Increased bone turnover

• Elevated alkaline phosphatase


Laboratory
• Elevated bone turnover markers (eg, PINP, urine hydroxyproline)
testing
• Calcium & phosphorus: usually normal

• X-ray: osteolytic or mixed lytic/sclerotic lesions


Imaging
• Bone scan: focal increase in uptake
Treatment • Bisphosphonates

PINP = procollagen type I N-terminal propeptide.

Alkaline phosphatase is a ubiquitous enzyme that has its highest expression in bone and hepatobiliary tissues. It
is a useful marker for cholestatic liver disease and diseases of bone causing increased bone turnover and new
bone formation. The most common cause of an isolated, asymptomatic elevation of alkaline phosphatase in an
elderly patient is Paget disease of bone (osteitis deformans). Paget disease is caused by osteoclast dysfunction,
with defective osteoid formation, increased bone remodeling, and focal hypertrophy (eg, enlarged distal
interphalangeal joints in this patient). The most commonly affected bones are the skull, clavicles, pelvis, and long
bones.

Although the classic clinical manifestations are related to areas of anatomic involvement (eg, hearing loss due to
cochlear involvement, radiculopathy due to nerve compression), Paget disease is most frequently discovered
incidentally on routine blood tests. Alkaline phosphatase levels can be strikingly elevated, often >10 times the
upper limit of normal. Fractionation of alkaline phosphatase would show a predominance of bone rather than liver
isoenzymes. Laboratory markers for bone turnover (eg, procollagen type I N propeptide, urine hydroxyproline) are
frequently elevated. X-rays of affected bones will show osteolytic or mixed lytic-sclerotic lesions, and radionuclide
bone scan is useful to fully stage the disease.

(Choices A and B) Statins (eg, atorvastatin) can cause minor elevations in liver markers, but this typically
manifests with a hepatocellular pattern (predominant elevations in transaminases) rather than a cholestatic pattern
(predominant elevation in alkaline phosphatase). Alcoholic liver disease also is typically associated with a
hepatocellular pattern, classically with a 2:1 ratio of aspartate aminotransferase to alanine aminotransferase.

(Choice C) Prostate cancer with bone metastasis may cause an elevation of alkaline phosphatase but will usually
cause additional features such as bone pain or systemic symptoms (eg, weight loss, fatigue). Also, prostate-
specific antigen will usually be >4 ng/mL, and prostate nodularity or asymmetry may be palpable on examination.
Diffuse (smooth) prostatic enlargement is common in older men and is usually benign.
(Choice E) Multiple myeloma typically presents with bone pain and systemic symptoms. However, calcium is
usually elevated, renal dysfunction is often present, and alkaline phosphatase is typically normal.

Educational objective:
The most common cause of an asymptomatic elevation of alkaline phosphatase in an elderly patient is Paget
disease of bone, which is frequently discovered incidentally on routine blood tests. X-rays will show osteolytic or
mixed lytic-sclerotic lesions, and radionuclide bone scan can fully stage the disease.
Question #386

A 23-year-old woman comes to the office with right forefoot pain that started 5 weeks ago and has slowly
worsened. She recalls no trauma or other inciting events. The patient is an avid runner and is currently training for
a long-distance race. The pain has progressed over the past week and has prevented her from daily running. She
takes no medications. The patient does not use tobacco, alcohol, or recreational drugs. She is not sexually active,
and her last menstrual period was 8 weeks ago. Temperature is 37 C (98.6 F), blood pressure is 100/60 mm Hg,
pulse is 68/min, and respirations are 12/min. BMI is 17 kg/m2. Examination of the right foot shows normal range of
motion without erythema, swelling, or bruising. There is tenderness to palpation along the dorsal surface of the
second metatarsal bone. Which of the following is the most likely diagnosis?

A) Morton neuroma

B) Plantar fasciitis

C) Stress fracture

D) Tarsal tunnel syndrome

E) Tenosynovitis
Explanation
Correct Answer:

C) Stress fracture

Clinical features of stress fracture

• Repetitive activities (eg, running, military marching)


• Abrupt increase in physical activity volume & intensity
Risk
• Low BMD from inadequate nutrition, malabsorption,
factors
estrogen deficiency in women (eg, functional hypothalamic
amenorrhea)

• Insidious, gradual worsening onset of localized pain


Clinical
• Point tenderness at fracture site
presentation
• Usually negative x-ray in the first 2-3 weeks

• Analgesia & reduced weight bearing


Management • Referral to orthopedic surgeon for high-risk fracture
(eg, anterior tibial cortex, 5th metatarsal)

BMD = bone mineral density.

This patient with worsening right forefoot pain, history of intense exercise, and tenderness on palpation of the
metatarsal bone likely has a stress fracture, which can occur in patients who suddenly increase their physical
activity (eg, military recruits, runners). Bone responds to mechanical stress by remodeling, but an abrupt increase
in intensity, duration, or frequency of physical activity causes repeated tension or compressive stress to the bone.
This can lead to microfractures that eventually coalesce within the cortical bone. Female runners with low BMI and
functional hypothalamic amenorrhea precipitated by intense exercise are more likely to develop stress fractures.

Other potential causes of forefoot pain include muscular strain, arthritis, bursitis, or Morton neuroma. A stress
fracture can be distinguished from these causes with direct palpation or axial loading of the metatarsal joint,
which usually causes sharp and localized pain over a bony surface (commonly the dorsal aspect of the
metatarsals). In contrast, arthritis typically occurs in the metatarsal-phalangeal joints and is not localized to a single
bony surface. Bursitis is usually caused by wearing poor-fitting shoes for an extended period, leading to
inflammation between the metatarsal heads.

(Choice A) Morton neuroma presents with pain between the metatarsal bones (typically between the third and
fourth) on the plantar surface. Often, a clicking sensation occurs when simultaneously palpating this space and
squeezing the metatarsal joints (Mulder sign).

(Choice B) Plantar fasciitis causes burning pain in the plantar area of the foot that typically is most severe with the
first few steps in the morning. It is common in runners who develop local point tenderness on the plantar aspect of
the foot.

(Choice D) Tarsal tunnel syndrome is due to compression of the tibial nerve as it passes through the ankle. It is
usually caused by a fracture of the ankle bones. Patients develop burning, numbness, and aching of the distal
plantar surface of the foot/toes that sometimes radiate up to the calf.

(Choice E) Tenosynovitis is an inflammation of the tendon and its synovial sheath, presenting with pain and
tenderness along the tendon, particularly with flexion and extension. It is usually seen in the hands and wrists,
often due to overuse or following a bite or puncture wound.

Educational objective:
Stress fractures are common in patients who suddenly increase their physical activity, especially runners with low
BMI, hypogonadism (eg, amenorrhea in female athletes), and insufficient caloric intake. Typical symptoms include
localized pain to palpation.
Reference
• Stress fractures: definition, diagnosis and treatment.
Question #387

A 54-year-old woman comes to the office due to left wrist pain 3 days after a fall. She was walking her dog when it
suddenly ran after another dog, causing her to fall. She landed forcefully on her left palm. She had no other
injuries. The patient has been taking acetaminophen, which provides partial relief. She does not use tobacco or
illicit drugs and drinks alcohol socially. Blood pressure is 120/70 mm Hg and pulse is 84/min. Physical examination
shows left wrist tenderness with maximal tenderness at the dorsoradial wrist lateral to the tendon of the extensor
pollicis longus. Left wrist radiographs reveal a radiolucent line across the waist of the scaphoid bone. She is
treated nonsurgically with analgesics and immobilization. This patient should be monitored closely during treatment
due to which of the following potential complications?

A) Bacterial tenosynovitis

B) Fat embolism

C) Ganglion cyst

D) Nerve injury

E) Osteonecrosis
Explanation
Correct Answer:

E) Osteonecrosis

This patient has a fracture of the scaphoid, the most commonly fractured carpal bone. Scaphoid fractures
typically present with pain at the radial wrist proximal to the base of the thumb. Examination shows tenderness in
the shallow depression at the dorsoradial wrist bounded medially by the tendon of the extensor pollicis longus and
laterally by the tendons of the abductor pollicis longus and extensor pollicis brevis ("anatomic snuffbox"). Scaphoid
fractures carry a significant risk of osteonecrosis because the blood supply enters at the distal pole and flows
proximally, and can be disrupted by the fracture.

Initial x-rays can be normal in nondisplaced scaphoid fractures. If scaphoid fracture is suspected, CT scan or MRI
can confirm the diagnosis, or repeat x-rays can be performed in 7-10 days. Displaced fractures should be
considered for surgical intervention. Wrist immobilization with a cast can be considered for nondisplaced fractures,
but patients should be monitored with serial x-ray to rule out osteonecrosis of the proximal segment and nonunion
of the fracture.

(Choice A) Bacterial tenosynovitis typically occurs following penetrating injury to the hand (eg, cat bite) or from
hematogenous spread of a distant infection (eg, Neisseria gonorrhoeae).

(Choice B) Fat embolism is most common following major fracture of long bones (eg, femur) or extensive soft-
tissue injury (eg, burns).

(Choice C) Synovial cysts of the wrist (ganglion cysts) are most common in individuals age 15-40 and are thought
to arise in most cases due to repetitive stress or inflammation.

(Choice D) Lunate dislocation following a fall on an outstretched hand can cause compressive neuropathy of the
median nerve. Nerve injury is not common following scaphoid fracture.

Educational objective:
Displaced scaphoid fractures should be considered for surgical intervention. Wrist immobilization with a cast can be
considered for nondisplaced fractures, but patients should be monitored with serial x-ray to rule out osteonecrosis of
the proximal segment and nonunion of the fracture.

Reference
• Scaphoid fractures.
Question #388

A 75-year-old man comes to the office for evaluation of right hip pain. The patient first noticed the pain several
months ago and it has become increasingly worse. He now has trouble putting on his shoes and socks. The
patient used to walk his dog for 2 miles each night, but he now must use a cane and can only walk a half-mile. The
patient has a history of hypertension. BMI is 32 kg/m2. X-ray of the right hip is shown below:
Which of the following is the most likely cause of this patient's hip pain?
A) Avascular necrosis

B) Femur fracture

C) Greater trochanteric pain syndrome

D) Hip dislocation

E) Osteoarthritis

F) Osteosarcoma
Explanation
Correct Answer:

E) Osteoarthritis
This patient has chronic joint pain that is worse with activity and weight bearing. The x-ray shows loss of the
normal joint space, periarticular osteophytes, and sclerosis of the acetabular surface. Together, these are typical
findings of osteoarthritis of the hip.
Osteoarthritis is characterized by progressive destruction of the articular cartilage. The pain of hip osteoarthritis is
typically felt in the groin, buttock, or lateral hip region, and can radiate to the lower thigh. Patients may have mild
pain and brief stiffness with prolonged rest, but the worst pain usually occurs with activity and weight bearing.
Examination findings often include decreased rotational range of motion with no synovitis (redness, warmth).

(Choice A) Avascular necrosis (osteonecrosis) is characterized by reduced perfusion of the femoral head and
collapse of the periarticular bone and can present with hip pain and reduced range of motion. Most atraumatic
cases are associated with glucocorticoid or alcohol use. X-ray findings include subchondral lucency and loss of the
normal spherical contour of the femoral head, but the joint space is preserved and osteophytes are not present.

(Choice B) Fracture of the femoral neck is most common in elderly patients following an acute fall. Examination
typically shows a shortened and externally rotated leg, with severe pain on range of motion. X-ray reveals
shortening and angulation of the femoral neck.

(Choice C) Greater trochanteric pain syndrome (formerly trochanteric bursitis) is caused by friction of the tendons
of the gluteus medius and tensor fascia lata over the greater trochanter of the femur. Pain is localized to the lateral
hip and is worsened by direct pressure. X-ray is typically normal.

(Choice D) Dislocation of the hip is uncommon in adults and is typically seen only in the context of major trauma.
X-ray would show displacement of the femoral head from the acetabulum, whereas this patient's femoral head is
well centered in the acetabulum.

(Choice F) Osteosarcoma is most common in children and adolescents, although it is occasionally seen in adults
age >65. The femur is a common location in pediatric patients, but in adults, osteosarcoma more commonly affects
the axial skeleton. X-ray shows destruction of trabecular and cortical bone with formation of new periosteal bone.

Educational objective:
Osteoarthritis of the hip is characterized by chronic pain in the groin, buttock, or lateral hip that is worse with activity
and weight bearing. Examination often shows decreased rotational range of motion. X-ray reveals loss of the
normal joint space, periarticular osteophytes, and sclerosis of the acetabular surface.

Reference
• Radiological aspects of osteoarthritis.
Question #389

A 30-year-old man comes to the office due to lower extremity muscle pain and weakness. The patient tripped and
fell while climbing stairs yesterday. He has had pain in both legs, usually with activity but occasionally at rest. The
patient works as a repair person and has been having a hard time performing routine tasks such as fixing
doorknobs or chandeliers. He is sexually active but has had decreased libido. The patient has had no double
vision or paresthesia. He has no chronic medical conditions and takes no medications. The patient does not use
tobacco, alcohol, or recreational drugs. Family history is significant for sudden cardiac death due to an arrhythmia
in his father. Vital signs are normal. Examination shows a long, narrow face; the cheeks are hollowed and the jaws
sag. The small muscles of the hands and dorsal flexors of the foot are weak bilaterally. Percussion over the thenar
eminence displays thumb abduction and delayed relaxation. Deep tendon reflexes are 2+. Gait is unsteady. The
testicles are small in volume. Which of the following is most likely to reveal the diagnosis of this patient?

A) Antinuclear antibodies

B) Echocardiography

C) Genetic testing

D) Karyotype analysis

E) Nerve conduction studies


Explanation
Correct Answer:

C) Genetic testing

Myotonic dystrophy

• Autosomal dominant CTG trinucleotide repeat expansion


Pathogenesis
• Longer repeat length correlating with earlier & more severe disease

• Classic (adult): myotonia & weakness (eg, face, hands, ankles)


Presentation • Childhood: cognitive & behavioral difficulties (classic symptoms develop over time)
• Infantile: hypotonia, respiratory failure, inverted V–shaped upper lip

• Cardiac: arrhythmias, cardiomyopathy


• Gastrointestinal: dysphagia, constipation
Associated
• Respiratory: pharyngeal weakness, hypoventilation
findings
• Nonmuscular: insulin resistance, hypogonadism, cataracts, frontal balding, excessive
daytime sleepiness

Diagnosis & • Genetic testing


treatment • Supportive management
CTG = cytosine-thymine-guanine.

This patient's muscle pain, weakness, and delayed relaxation are concerning for myotonic dystrophy. Myotonic
dystrophy is an autosomal dominant disorder in which a trinucleotide (CTG) repeat in the DMPK gene is transcribed
into untranslatable mRNA that is toxic to cellular function. Longer repeat expansion generally correlates with an
earlier onset of more severe disease (ie, anticipation).

Presentation classically involves muscle weakness, pain, and atrophy, particularly in the distal (eg, hands, feet)
and facial (eg, long and narrow face, sagging jaw, hollow cheeks) muscles; footdrop may result in frequent falls and
an unsteady gait. Myotonia is a characteristic feature and can be elicited by percussion of the thenar eminence,
which results in thumb abduction followed by delayed relaxation (percussion myotonia), or by sustained muscle
contraction after the release of a handshake (grip myotonia). Involvement of cardiac muscles can result in
cardiomyopathy or arrhythmias, and sudden cardiac death in this patient's father raises suspicion for undiagnosed
myotonic dystrophy. Nonmuscular manifestations can include hypogonadism with decreased libido and testicular
atrophy, as seen in this patient.

The diagnosis is suspected based on clinical findings and family history, and DMPK genetic testing is
confirmatory. Management, including physiotherapy for weakness and testosterone replacement for symptomatic
hypogonadism, is generally supportive.

(Choice A) Antinuclear antibodies are frequently positive with inflammatory myopathies (eg, polymyositis);
however, proximal, not distal, muscle weakness predominates in these conditions, and myotonia and hypogonadism
would not be expected.

(Choice B) In myotonic dystrophy, ECG and echocardiographic findings (eg, atrial fibrillation/flutter, atrioventricular
block, left ventricle dysfunction) are nonspecific, making genetic testing to confirm the diagnosis the priority.
Hypertrophic cardiomyopathy (eg, left ventricular outflow tract obstruction on echocardiography) could explain a
family history of sudden cardiac death but is not classically associated with myotonia.

(Choice D) Karyotype analysis is diagnostic of Klinefelter (XXY) syndrome, which is a cause of primary
hypogonadism but is not associated with muscle pain/weakness or myotonia.
(Choice E) Nerve conduction studies are helpful in evaluating weakness due to peripheral neuropathies. However,
peripheral neuropathies do not cause percussion myotonia and would not explain this patient's testicular atrophy.

Educational objective:
Myotonic dystrophy is an autosomal dominant disorder characterized by myotonia, as well as pain and weakness of
the distal and facial muscles. Cardiac arrythmias and hypogonadism are also common features, and genetic testing
detecting the presence of a trinucleotide (CTG) repeat expansion in the DMPK gene confirms the diagnosis.

Reference
• Cardiac conduction disorders as markers of cardiac events in myotonic dystrophy type 1.
Question #390

A 24-year-old woman comes to the office with several months of low back and buttock pain. Her pain is most
severe early in the morning and fades as the day progresses. There is no recent history of significant illness or
trauma. Vital signs are normal. On examination, tenderness is present in the lower lumbar spine. Muscle strength
and deep tendon reflexes are normal and equal in all extremities. Plain x-ray findings are shown in the image
below:
Which of the following is the most appropriate next step in management?

A) Colchicine
B) Nonsteroidal anti-inflammatory drug

C) Oxycodone

D) Prednisone

E) Tumor necrosis factor inhibitor


Explanation
Correct Answer:

B) Nonsteroidal anti-inflammatory drug

Treatment of ankylosing spondylitis

• Exercise (postural exercises, ROM/


Nonpharmacologic
stretching exercises)
measures
• Physical therapy

• NSAIDs (eg, ibuprofen, naproxen)


Initial treatment
• COX-2 inhibitors (eg, celecoxib)

• TNF-α inhibitors (eg, etanercept,


Treatment failure/
infliximab)
disease
• Anti–IL-17 antibodies (eg,
progression
secukinumab)

COX-2 = cyclooxygenase-2; NSAIDs = nonsteroidal anti-


inflammatory drugs; ROM = range of motion; TNF-α = tumor
necrosis factor-alpha.

This patient has inflammatory back pain (ie, onset age <40, chronic/insidious onset, worse with rest/better with
activity) and spinal tenderness. Her pelvic x-rays show findings of sacroiliitis, including widening of the sacroiliac
joint space and sclerosis of the subchondral bone. This is a characteristic presentation of ankylosing spondylitis
(AS).

In AS, prostaglandin E2, acting primarily via the EP4 receptor, has a synergistic effect with the inflammatory
cytokines (eg, tumor necrosis factor-alpha [TNF-alpha], IL-17) in promoting syndesmophyte formation and
ankylosis. Nonsteroidal anti-inflammatory drugs (NSAIDs) (eg, ibuprofen, naproxen) and cyclooxygenase-2
inhibitors (eg, celecoxib) reduce the formation of prostaglandin E2. These agents often provide substantial relief of
symptoms in AS and inhibit progression of the disease. In many patients (70%-80%), NSAIDs are the only agents
needed to reduce pain and stiffness. NSAIDs also help the peripheral arthritis often seen in association with AS.

(Choice A) Colchicine is a microtubule inhibitor that suppresses mitosis and inflammatory cell migration. It is
commonly used in the treatment of gout but is not recommended for AS.

(Choice C) Options for pain relief in patients who have persistent symptoms on NSAID therapy include
acetaminophen and tramadol. Opioids (eg, oxycodone) are not well studied for AS and, unlike NSAIDs, do not
address the underlying inflammatory process.

(Choice D) Intraarticular glucocorticoid injections can be used in select patients with AS for treatment of sacroiliitis,
peripheral arthritis, or enthesitis (inflammation at tendon insertions). Systemic glucocorticoids (eg, prednisone) are
generally not effective.

(Choice E) Anti-TNF (eg, etanercept, infliximab) and anti–IL-17 (eg, secukinumab) agents are effective for patients
who have progressive disease despite NSAIDs. However, these medications have significant toxicity and are not
used as first-line treatment because many patients do extremely well with NSAIDs alone.

Educational objective:
First-line treatment options for ankylosing spondylitis include nonsteroidal anti-inflammatory drugs and
cyclooxygenase-2 inhibitors, which provide substantial relief of symptoms and may inhibit progression of the
disease for long periods.
Question #391

A 67-year-old man comes to the office due to 2 days of back pain. The patient was moving boxes in his garage
when the pain started. It is not relieved by lying down and increases in intensity when he strains or coughs. The
patient cannot sleep due to pain and has taken acetaminophen several times without relief. He has never before
had such pain. Physical examination shows 2+ symmetric knee and ankle jerk reflexes. Straight-leg raising test is
negative. There is point tenderness to palpation and percussion along the midline at the fourth lumbar vertebra.
Which of the following is the most likely underlying etiology of this patient's current condition?

A) Apophyseal joint arthritis

B) Intervertebral disc degeneration

C) Ligamentous sprain

D) Loss of bone mineral density

E) Nerve root demyelination


Explanation
Correct Answer:

D) Loss of bone mineral density

Clinical features of vertebral compression fracture

• Trauma (often trivial)


• Osteoporosis, osteomalacia
Etiologies • Bone metastases
• Metabolic (eg, hyperparathyroidism)
• Paget disease

Acute

• Back pain & decreased spinal mobility


• Pain increasing with standing, walking, lying on back
• Referred pain to abdomen/flank
Clinical • Spinal tenderness at affected level
presentation
Chronic/gradual

• Painless
• Progressive kyphosis
• Loss of height
• Increased risk for future fractures
Complications • Hyperkyphosis → protuberant abdomen, early satiety, weight
loss, decreased respiratory capacity

This older patient has acute back pain consistent with vertebral compression fracture (VCF). Acute VCF
commonly occurs from minor trauma, such as lifting, twisting, coughing, or falling from standing height. Pain is
typically localized to the midline, although it can radiate to the flank or upper abdomen. Pain often worsens with
movement, coughing, or straining and may persist at night. Examination typically demonstrates vertebral point
tenderness; spinal cord or nerve root compression is rare, and patients usually do not have neurologic deficits.

VCF is most commonly due to decreased bone density from osteoporosis. The risk is greatest in
postmenopausal women and men age >65, such as this patient. Other etiologies include osteomalacia, vertebral
osteomyelitis, malignancy with bone metastases, metabolic abnormalities, and trauma.

The diagnosis is usually established with plain x-ray of the spine; MRI or CT scan are also effective if the diagnosis
is uncertain or underlying malignancy is likely. Imaging commonly identifies additional asymptomatic
compressions. Initial evaluation should also include screening studies to identify additional contributing factors (eg,
hyperthyroidism, hyperparathyroidism, vitamin D deficiency). DXA should also be obtained to guide chronic
management.

(Choice A) Apophyseal joint arthritis can occur in spondyloarthropathy (eg, ankylosing spondylitis), which typically
presents with chronic, progressive pain that is worse at night and with prolonged rest; symptoms usually begin at
age <40.

(Choice B) Degenerative disc disease can cause low back pain, but the pain is usually chronic, worsens with
activity, and is relieved with rest. Disc degeneration can also lead to acute disc herniation, but this typically
presents with acute lumbosacral radiculopathy (ie, pain radiating in a dermatomal distribution) because most
herniations are posterolateral. Vertebral point tenderness is more consistent with VCF.

(Choice C) Ligamentous back sprain can occur following exertion. However, the pain is usually relieved with rest,
and tenderness would be seen in the paraspinal tissues rather than at the midline (over the vertebra).
(Choice E) Nerve root demyelination (eg, Guillain-Barré syndrome) presents with paresthesia, weakness, and
absent deep tendon reflexes corresponding to the affected nerve root. Although back pain may be present due to
nerve root inflammation, it typically is not associated with focal vertebral (bone) tenderness.

Educational objective:
Acute vertebral compression fracture can be caused by twisting, lifting, or other minimal trauma and presents with
back pain and vertebral point tenderness. It typically occurs in patients with osteoporosis or other conditions
associated with decreased bone mineral density.

Reference
• Evaluation and management of vertebral compression fractures.
Question #392

A 35-year-old man comes to the office for evaluation of left knee pain that is worse when kneeling and when walking
up or down stairs. Medical history is significant for type 1 diabetes mellitus and vitamin D deficiency. In high
school, the patient had reconstruction of the left anterior cruciate ligament. He works as a software engineer and
rarely exercises. He drinks 1 or 2 beers a night but does not use tobacco or illicit drugs. Temperature is 36.7 C (98
F), blood pressure is 120/70 mm Hg, pulse is 76/min, and respirations are 14/min. BMI is 24 kg/m2. On
examination, the left knee has full range of motion. There is mild tenderness at the medial joint line, and the
Lachman test is negative. There is no clicking, locking, or swelling. Laboratory studies show a low serum vitamin D
level and a hemoglobin A1c of 8%. Which of the following is the most significant contributor to this patient's knee
pain?

A) Alcohol use

B) Physical inactivity

C) Poor glucose control

D) Prior trauma

E) Vitamin D deficiency
Explanation
Correct Answer:

D) Prior trauma

Risk factors for osteoarthritis

Modifiable Nonmodifiable

• Advanced age
• Sedentary lifestyle
• Female sex
• Obesity
• Family history
• Occupational joint loading
• Abnormal joint alignment
• Diabetes mellitus
• Prior joint trauma

This patient most likely has osteoarthritis (OA) of the knee, presenting with monarticular arthritis that is worse with
joint loading during kneeling and negotiating stairs. OA is very common in patients with prior joint injury or
surgery (secondary OA); contributing factors include altered joint mechanics and both acute and chronic
inflammation, which lead to altered expression of matrix metalloproteinases and accelerated chondrocyte death.

OA develops within 10 years in most patients with anterior cruciate ligament injury, and the risk is increased
whether or not the tear is surgically corrected. Other causes of secondary OA include joint infection, inflammatory
disorders (eg, rheumatoid arthritis), congenital or acquired bony deformities, and neuromuscular weakness.

(Choice A) Alcohol use can contribute to the degradation of articular cartilage. Heavy alcohol intake (>3 drinks/
day) is associated with a significant increase in the risk for OA of the knee; however, moderate intake, as in this
patient, has only a minor effect on risk.

(Choice B) Physical inactivity is associated with an increased risk for primary OA, likely due to abnormal joint
loading from muscular weakness or associated obesity. However, in the absence of trauma or other secondary
causes of OA, the risk of primary OA is low before age 40.

(Choice C) Chronic hyperglycemia can accelerate the progression of OA, likely due to the glycosylation of
components of articular cartilage. However, this patient's hemoglobin A1c is only slightly elevated and likely has
little effect on his OA risk.

(Choice E) Vitamin D deficiency can lead to osteomalacia manifesting with bone pain, muscle weakness,
disordered gait, and increased risk of fracture; subclinical deficiency is associated with osteoporosis. It does not
cause monarthritis of the knee.

Educational objective:
Osteoarthritis is common in patients with prior joint injury (especially anterior cruciate ligament injury) or surgery.
Other secondary causes of osteoarthritis include infection, inflammatory disorders, bony deformities, and
neuromuscular weakness.

Reference
• Treatment for acute anterior cruciate ligament tear: five year outcome of randomised trial.
Question #393

A 50-year-old man comes to the office due to right hand pain and swelling for 3 days. The pain started suddenly
and woke him from sleep. Today, he developed fever. There is no recent history of trauma to the hand. The patient
has hypertension and takes lisinopril and hydrochlorothiazide. He is sexually active with multiple partners.
Temperature is 38.9 C (102 F). Examination shows right wrist redness, swelling, tenderness, and decreased range
of motion. There are no oral, cutaneous, genital, or perianal lesions or rash. Laboratory results reveal a leukocyte
count of 15,000/mm3 with 85% neutrophils. Right wrist arthrocentesis is performed; fluid analysis shows a
leukocyte count of 48,000/mm3 with 86% neutrophils and no crystals; Gram stain reveals no organisms. Which of
the following is the next best step to establish the diagnosis in this patient?

A) MRI of the hand

B) Nucleic acid amplification testing of synovial fluid

C) Serology for Borrelia burgdorferi

D) Serum antinuclear antibody titer

E) Serum uric acid concentration


Explanation
Correct Answer:

B) Nucleic acid amplification testing of synovial fluid

Disseminated gonococcal infection

• Purulent monoarthritis

Manifestations OR

• Triad of tenosynovitis, dermatitis, migratory polyarthralgia

• Detection of Neisseria gonorrhoeae in urine, cervical, or urethral sample


Diagnosis
• Culture of blood, synovial fluid (less sensitive)

Treatment • 3rd-generation cephalosporin intravenously

This patient has fever and purulent monoarthritis of the right wrist with no recent trauma and no evidence of
synovial fluid crystals or organisms on microscopy. This constellation of features in an individual at risk for sexually
transmitted infections (ie, multiple sexual partners) raises suspicion for disseminated gonococcal infection (DGI).

DGI typically presents with either the triad of migratory arthralgia, tenosynovitis, and pustules or purulent oligo- or
monoarthritis. Manifestations of arthritis typically begin suddenly and are marked by joint swelling and pain; the
knees, wrists, and ankles are most often affected. Fever and malaise are frequently present.
The diagnosis of DGI requires sampling from multiple areas because Neisseria gonorrhoeae is a fastidious
organism and culture results are frequently negative. Two sets of blood cultures should be obtained, and synovial
fluid sampling is required when joint effusion is present. Synovial fluid leukocyte count usually is ~50,000/mm3 with
a preponderance of neutrophils, but organisms may be absent on Gram stain. Synovial fluid should be sent for
nucleic acid amplification testing (NAAT) or culture. Mucosal sites (eg, rectum, urethra, throat) should also be
sampled and specimens sent for NAAT; although patients with DGI do not typically have symptomatic infection at
the portal of entry (eg, urogenital tract), mucosal NAAT is often positive and can provide a presumptive diagnosis.

(Choice A) Although MRI can help identify septic joints that are difficult to visualize on examination (eg, hip, spine),
patients with septic arthritis of the wrist would usually undergo x-ray before additional imaging is considered. In
addition, imaging characteristics are not particularly helpful in clarifying the etiology of septic arthritis. MRI would
not be the best next test.

(Choice C) Late Lyme disease can cause purulent monoarthritis, but it usually occurs in the large joints (eg, knee).
Synovial samples would also show no organisms or crystals, but leukocyte count is usually 10,000-25,000/mm3. In
addition, fever is typically absent.

(Choice D) Antinuclear antibody testing is part of the evaluation for systemic lupus erythematosus (SLE), which
often causes symmetric, polyarticular arthritis. This patient with acute-onset purulent monoarthritis is less likely to
have SLE than DGI.

(Choice E) Patients taking hydrochlorothiazide are at increased risk for gout flares. Although gout often causes
sudden-onset, intensely inflammatory monoarthritis, the lower extremity joints are primarily affected, and
monosodium urate crystals are typically visualized in synovial fluid.

Educational objective:
Disseminated gonococcal infection can present with purulent monoarthritis and fever. Synovial fluid analysis
typically shows a leukocyte count of approximately 50,000/mm3 with no organisms or crystals on microscopy.
Diagnosis requires synovial fluid nucleic acid amplification testing (NAAT), blood cultures, and NAAT of samples
from mucosal sites (eg, urethra, rectum, pharynx, cervix).
Question #394

A 42-year-old woman comes to the office with a 4-month history of heartburn. She describes a periodic "sticking
sensation" in her chest during meals. In addition, the patient has recently been unable to participate in her normal
exercise routine due to dyspnea on exertion and joint pain in her hands and feet. She does not use tobacco,
alcohol, or illicit drugs. Lung examination reveals bilateral end-inspiratory crackles. Endoscopic evaluation shows
mild hyperemia in the distal esophagus. Esophageal manometry shows lack of peristaltic waves in the lower two-
thirds of the esophagus and a significant decrease in lower esophageal sphincter tone. Which of the following is the
most likely mechanism responsible for this patient's manometric findings?

A) Dysfunctional impairment of inhibitory neurons

B) Eosinophilic infiltration of esophageal mucosa

C) Loss of intramural neurons

D) Smooth muscle atrophy and fibrosis

E) Striated muscle inflammation


Explanation
Correct Answer:

D) Smooth muscle atrophy and fibrosis

Systemic sclerosis

• Progressive tissue fibrosis


Pathogenesis
• Vascular dysfunction

• Skin: telangiectasia, sclerodactyly, digital ulcer, calcinosis cutis


• Extremities: arthralgia, myalgia, contracture
Clinical features • Gastrointestinal tract: esophageal dysmotility, dysphagia, acid reflux
• Lungs: dyspnea, dry cough
• Vascular system: Raynaud phenomenon

• Antinuclear
Autoantibodies • Anti–topoisomerase type I (anti–Scl-70) (diffuse SSc)
• Anticentromere (limited SSc)

• Lungs: interstitial lung disease, pulmonary arterial HTN


Complications • Kidneys: HTN, scleroderma renal crisis (severe HTN, AKI, MAHA)
• Heart: myocardial fibrosis, pericarditis, pericardial effusion
AKI = acute kidney injury; HTN = hypertension; MAHA = microangiopathic hemolytic anemia; SSc = systemic
sclerosis.

This patient has a number of findings (eg, esophageal dysmotility, fibrotic lung disease, arthralgias) consistent with
extradermal manifestations of systemic sclerosis (SSc). Classic early skin manifestations of SSc include
thickening or hardening, edema, and pruritus. However, if skin symptoms are mild, patients may first seek attention
due to gastrointestinal (GI), joint, or respiratory disease.

GI complications are common in SSc and primarily affect the esophagus. SSc causes smooth muscle atrophy
and fibrosis in the lower esophagus; the upper third of the esophagus is made of striated muscle and seldom
affected by SSc. Common symptoms include dysphagia, choking, heartburn, and hoarseness. Esophageal
manometry in affected patients typically shows hypomotility and incompetence of the lower esophageal sphincter
(LES).

(Choice A) Dysfunction of inhibitory neurons causes diffuse esophageal spasm and presents with chest pain and
dysphagia rather than heartburn. Manometry is characterized by periodic, high-amplitude, non-peristaltic
contractions.

(Choice B) Eosinophilic esophagitis is characterized by heartburn that does not respond to standard medications
for gastroesophageal reflux disease. Manometry most often shows esophageal hypercontractility.

(Choice C) Achalasia presents with dysphagia and regurgitation of undigested food. As in SSc, manometry shows
aperistalsis in the distal esophagus. However, achalasia causes increased LES pressure and incomplete LES
relaxation, whereas SSc causes decreased LES pressure.

(Choice E) Esophageal involvement may occur in polymyositis. It can involve both the upper and lower esophagus
and is characterized by dysphagia, regurgitation, and aspiration. Manometry results are often functionally similar to
SSc, but most patients will have symmetric proximal muscle weakness, not distal arthralgias.

Educational objective:
Systemic sclerosis can cause atrophy and fibrosis of the smooth muscle in the lower esophagus. This leads to
decreased peristalsis and decreased tone in the lower esophageal sphincter. Typical symptoms include heartburn
and dysphagia.

Reference
• Gastrointestinal complications: the most frequent internal complications of systemic sclerosis.
Question #395

A 58-year-old man comes to the office due to a hard mass on the posterior left elbow. The mass has been growing
for the past 3 years. The patient has had no fever, pain, weight loss, or trauma. Medical history includes
hypertension, chronic kidney disease, and gout. On examination, there is a 3 × 3 cm, nontender, hard mass just
under the skin in the posterior left elbow. The elbow is not red, warm, or tender. Elbow imaging shows a 3-cm, soft
tissue mass and bone erosions with overhanging edges of cortical bone at the olecranon process. Serum
creatinine is 1.7 mg/dL. Which of the following conditions most likely explains this patient's findings?

A) Chronic kidney disease–mineral and bone disorder

B) Chronic olecranon bursitis

C) Osteoarthritis

D) Osteosarcoma

E) Tophaceous gout
Explanation
Correct Answer:

E) Tophaceous gout

This patient has tophaceous gout affecting the olecranon bursa. The olecranon bursa is a fluid-filled synovial sac
between the olecranon process and the skin that alleviates friction at the bony prominence. Gout can affect
superficial bursae (eg, olecranon, prepatellar) in multiple ways:

• Acute bursitis in a gout flare, presenting with erythema, warmth, and swelling

• Chronic bursitis, presenting with a large, rounded, fluctuant effusion

• Bursal tophus, manifesting as a slowly enlarging, hard mass (as in this patient), sometimes with mild
inflammatory changes

Acute bursitis is painful due to the rapidly escalating inflammation; in contrast, chronic bursitis and bursal tophus
typically have minimal or no pain because the accompanying inflammation is generally indolent.

Tophi (ie, large aggregates of monosodium urate crystals) may also form in the nearby humeroulnar joint or other
types of soft tissues (eg, ligaments, tendons). Tophi often trigger a chronic inflammatory response in the
surrounding tissue and bone, resulting in bone erosions with overhanging edges of cortical bone on imaging. Urate
is intrinsically radiolucent but can become radiopaque when calcium precipitates with urate in a tophus. Risk
factors for tophus formation include chronically untreated gout and chronic kidney disease (CKD) (due to decreased
uric acid excretion); conversely, chronic hyperuricemia can cause or worsen CKD due to nephrolithiasis and urate
deposition in the renal interstitium (ie, urate nephropathy).

(Choice A) Phosphorus retention and reduced production of 1,25-dihydroxyvitamin D in CKD can lead to
secondary hyperparathyroidism and abnormal bone turnover. The resulting skeletal manifestations (ie,
CKD–mineral and bone disorder) can include bone pain, fragility fractures, and cystic lesions. However, these
findings are usually seen in end-stage renal disease, and soft tissue mass lesions are not typical.
(Choice B) Chronic olecranon bursitis is characterized by synovial fluid accumulation within the bursa, most often
due to repetitive friction. It presents with a round, fluctuant swelling, not a hard mass.

(Choice C) Osteoarthritis is associated with osteophytes, joint space narrowing (due to cartilage degeneration),
subchondral sclerosis, and subchondral cysts. A soft tissue mass and bone erosions are not characteristic.
Osteoarthritis typically affects the knees, hips, and hands; elbow involvement is rare.

(Choice D) Osteosarcoma presents as a primary bone malignancy in adolescents or as a result of malignant


transformation of underlying bone pathology (eg, Paget disease) in those age >65. On x-ray, periosteal reaction
resulting in sunburst or Codman triangle patterns is typically present.

Educational objective:
Gout can cause acute bursal (eg, prepatellar, olecranon) inflammation, chronic bursal swelling, or tophus deposition
in the bursa. Tophus induces chronic inflammation in the surrounding soft tissue and bones, which can result in
erosions and overhanging edges of cortical bone on imaging.

Reference
• Common superficial bursitis.
Question #396

A 68-year-old woman is brought to the clinic by her daughter due to severe pain in her fingers. Her daughter says,
"Mom has had horrible problems with her joints for a long time, but she has never tried to get help." The patient
adds that her fingers have become more swollen and painful over the last few weeks. She had similar symptoms in
her foot last year. She was given an unknown pain pill, but it was ineffective. Medical history is notable for
hypertension, hypothyroidism, and early Alzheimer dementia. Vital signs are normal. Examination findings of the
hands are shown in the image below.
What is the most likely diagnosis in this patient?

A) Calcinosis cutis

B) Chondrosarcoma

C) Gout

D) Osteoarthritis

E) Psoriatic arthritis

F) Rheumatoid nodules
Explanation
Correct Answer:

C) Gout

This patient with multiple white nodules in the hands and a history of painful arthritis in the fingers and feet has
tophaceous gout. In most cases, gout is characterized by hyperuricemia and precipitation of uric acid crystals in
the joints, leading to episodic monoarticular arthritis (especially in the first metatarsophalangeal joint and knee).
The diagnosis is confirmed with arthrocentesis showing an inflammatory effusion with uric acid crystals.

Urate crystals may also deposit in the soft tissues, forming tumors known as tophi. Tophi can ulcerate and drain a
chalky material. Uric acid tophi are virtually pathognomonic for gout, and even in the absence of microscopic
confirmation of crystals, the diagnosis can be made provisionally in patients with visible tophi and a history of
episodic monoarthritis. An elevated serum uric acid level is nonspecific but can also contribute to the diagnosis.

(Choice A) Calcinosis cutis is characterized by deposition of calcium and phosphorus in the skin. It presents with
scattered whitish papules, plaques, or nodules. This patient's history of painful arthritis is more typical for gout.

(Choice B) Chondrosarcomas are bone tumors occurring in the 5th or 6th decade of life. They most commonly
affect the pelvis, femur, or proximal humerus and would likely not be as multifocal and bilateral as the disease
pictured.

(Choice D) Severe osteoarthritis can produce Heberden and Bouchard nodes, which are hard, bony nodules over
the distal and proximal interphalangeal joints, respectively. This patient's tophi involving several joint spaces and
soft tissue structures are more consistent with gout.

(Choice E) Psoriatic arthritis can present with distal interphalangeal joint arthritis, asymmetric oligoarthritis,
symmetric polyarthritis, spondyloarthropathy, or aggressively destructive arthritis mutilans. Most patients have
established psoriasis, and nail changes (eg, pitting) are common.

(Choice F) Rheumatoid nodules are firm, flesh-colored, and nontender. They typically occur over pressure points
such as the elbow and extensor surface of the proximal ulna.

Educational objective:
In chronic tophaceous gout, urate crystals can be deposited in the skin, resulting in the formation of tumors with a
chalky white appearance.
Question #397

A 61-year-old woman comes to the office due to a 2-week history of low back pain. The patient has a constant, dull,
aching pain that is more pronounced at night and has awakened her on several occasions. She has had no trauma
or other back conditions. There is no associated fever, chills, bowel or bladder incontinence, or lower extremity
weakness or numbness. Medical history is notable for hypertension and breast cancer at age 55, which was
treated with lumpectomy, radiation therapy, and hormone therapy. The patient does not use tobacco, alcohol, or
illicit drugs. Temperature is 36.7 C (98.1 F), blood pressure is 134/86 mm Hg, pulse is 76/min, and respirations are
12/min. Head and neck, cardiac, lung, breast, and abdominal examinations show no abnormalities. Spinal
examination shows no deformities or focal tenderness. Lower extremity motor strength and reflexes are normal and
symmetric. Straight-leg raising test is negative. Which of the following is the most appropriate next step in
management of this patient?

A) Epidural corticosteroid injection

B) Lumbosacral spinal imaging

C) Opioid analgesic at bedtime

D) Supervised exercise program

E) Trial of nonsteroidal anti-inflammatory drugs and follow-up


Explanation
Correct Answer:

B) Lumbosacral spinal imaging


Most cases of acute (ie, <4 weeks) low back pain have a benign etiology and resolve spontaneously; imaging
generally does not improve outcomes and is not recommended. However, spinal imaging is indicated for patients
with significant neurologic deficits or red-flag features suggesting increased risk for infection, malignancy, or bony
abnormalities (eg, compression fracture).

This patient has features that raise concern for malignant back pain, including nocturnal pain and history of
malignancy; therefore, she warrants imaging despite her unremarkable neurological examination (given the urgency
in treating potential metastatic or bony lesions). The preferred test is spinal MRI, which has a high sensitivity for
lytic bone lesions and metastasis in the surrounding soft tissues. If MRI cannot be performed, CT scan has good
sensitivity for bony disruption and is a reasonable alternate test. Plain film x-rays have lower sensitivity for bone
metastasis, but some guidelines suggest x-ray combined with inflammatory markers (eg, erythrocyte sedimentation
rate, C-reactive protein) to increase sensitivity for patients with moderate clinical suspicion for malignancy.

(Choice A) Epidural corticosteroid injection is indicated for chronic radicular pain (eg, due to a herniated disc) that
has failed noninvasive treatment, but it is not indicated for acute nonradicular pain, as in this patient.

(Choices C and E) Uncomplicated pain with no red-flag features can be managed symptomatically with
nonsteroidal anti-inflammatory drugs (NSAIDs). Opioids are not more effective than NSAIDs, so they are not
recommended for initial therapy. Regardless, this patient requires additional evaluation first.

(Choice D) Physical therapy referral for a supervised exercise program is used primarily for patients with persistent
(ie, >4 weeks) back pain and can be considered for those with risk factors for chronic pain disorders (eg, poor
functional status, psychiatric comorbidity).

Educational objective:
Most patients with acute low back pain do not require imaging. However, spinal imaging is indicated for patients
with significant neurologic deficits or clinical features suggesting increased risk for infection, malignancy, or bony
abnormalities. The preferred test for patients with a history of malignancy is spinal MRI, which has high sensitivity
for lytic bone lesions and metastasis in the surrounding soft tissues.

Reference
• Diagnosis and treatment of acute low back pain.
Question #398

A 38-year-old man comes to the office for evaluation of bilateral progressive hand pain with intermittent swelling
over the past 6 months. The patient has no other symptoms or history of trauma or family history of arthropathy.
Medical history is significant for type 2 diabetes mellitus, for which he takes metformin. Vital signs are normal. BMI
is 24 kg/m2. Examination reveals mild swelling and tenderness over the second and third metacarpophalangeal
(MCP) joints bilaterally. There is no muscle weakness. X-ray reveals joint space narrowing and chondrocalcinosis
involving the second MCP joints with osteophyte formation at the second and third metacarpal heads. Serum
alanine and aspartate aminotransferases are elevated. Which of the following therapies would be of most benefit to
this patient?

A) Etanercept

B) Methotrexate

C) Phlebotomy

D) Prednisone
Explanation
Correct Answer:

C) Phlebotomy

Arthropathy of hereditary hemochromatosis

• Onset at age <40


• Chronic pain & bony swelling
Clinical features
• Most common at 2nd & 3rd MCP joints
• Occasional acute flare

• Joint space narrowing


X-ray signs • Chondrocalcinosis
• Hook-shaped osteophytes at metacarpal heads

• Oral analgesics (eg, acetaminophen, NSAIDs)


Management
• Therapeutic phlebotomy (to prevent other complications)

MCP = metacarpophalangeal; NSAIDs = nonsteroidal anti-inflammatory drugs.

This patient has chronic arthritis with joint space narrowing similar to osteoarthritis (OA), but several features
suggest hereditary hemochromatosis (HH) arthropathy, including:
• Onset at age <40 versus >40 with OA

• Predilection for the second and third metacarpophalangeal (MCP) joints and wrists versus distal
interphalangeal joints and thumb bases with OA

• Presence of chondrocalcinosis (ie, cartilage calcification)

HH is also suggested by comorbid diabetes, which is due to pancreatic iron deposition and is otherwise unusual in
nonobese patients at this age. Elevated liver transaminases also suggest HH and make primary joint disorders
less likely.

In addition to the hand and wrist, other joints (eg, hips, knees) can be affected by HH arthropathy; characteristic x-
ray findings include periarticular bony enlargement and subchondral lucency. Joint examination typically shows no
signs of synovitis (eg, erythema, warmth), but acute inflammatory flares may occasionally occur. Arthropathy
typically becomes apparent early in the course of HH and may be the presenting feature, although later
presentations are possible.

Management of HH arthropathy is primarily focused on symptom relief. Simple oral analgesics (eg, acetaminophen,
nonsteroidal anti-inflammatory drugs [NSAIDs]) are usually used first. However, patients benefit most from
therapeutic phlebotomy, which is necessary to prevent further joint injury (with limited effect on the established
arthropathy) and minimize the systemic complications of HH (eg, liver disease).

(Choices A, B, and D) Rheumatoid arthritis (RA) is typically treated initially with NSAIDs or systemic
glucocorticoids (eg, prednisone) to provide acute symptom relief. Thereafter, disease-modifying antirheumatic
drugs are initiated for long-term management; methotrexate is commonly recommended, with tumor necrosis factor
inhibitors (eg, etanercept) and other biologic agents used for patients with refractory symptoms. Although RA
commonly causes joint space narrowing in the MCP joints, x-ray typically shows periarticular osteopenia and bony
erosions rather than the chondrocalcinosis and osteophytes seen with HH.

Educational objective:
Hereditary hemochromatosis (HH) arthropathy often resembles osteoarthritis but differs in age at onset (<40);
predilection for the second and third metacarpophalangeal joints and wrists; and presence of chondrocalcinosis.
Management is primarily symptomatic (eg, acetaminophen, nonsteroidal anti-inflammatory drugs), but therapeutic
phlebotomy is necessary to minimize other systemic complications of HH (eg, liver disease).
Question #399

A 29-year-old woman comes to the office due to back pain. For the last week, she has had moderate, nonradiating
pain in the low back that started after she spent the weekend raking leaves in her yard. The patient attempted
treatment with a heating pad and ibuprofen but had no relief. She has no associated fever, lower extremity
weakness, or urinary symptoms. Medical history is unremarkable, and the patient takes no other medications. Vital
signs are normal. BMI is 34 kg/m2. Examination shows tenderness in the right lumbar paraspinal region with
normal neurologic findings in the lower extremities. Straight-leg raising test is negative. Which of the following is
the most appropriate next step?

A) Add a nonbenzodiazepine muscle relaxant

B) Add an opioid analgesic

C) Advise strict bed rest

D) Order an MRI of the lumbar spine

E) Order an x-ray of the lumbar spine


Explanation
Correct Answer:

A) Add a nonbenzodiazepine muscle relaxant

Management of acute nonspecific back pain

• Maintain normal, moderate activity


Nonpharmacologic
• Education (eg, activity, prognosis)
measures
• Consider: heat, massage, spinal manipulation, acupuncture

• NSAIDs (eg, naproxen, ibuprofen)


First-line medication
• Consider acetaminophen

• Nonbenzodiazepine muscle relaxant (eg, cyclobenzaprine, tizanidine)


Second-line medication • Opioids, tramadol (not preferred; consider short course for severe, refractory
pain)

NSAIDs = nonsteroidal anti-inflammatory drugs.

This patient has acute low back pain (LBP) with no evidence of neurologic injury and no features (eg, fever,
history of malignancy) that would suggest a potentially serious cause. Her pattern of moderate, nonradiating pain
with paraspinal tenderness suggests a benign cause, likely paraspinal muscle strain.
Most cases of acute LBP resolve spontaneously; management is focused primarily on interventions to provide
temporary relief with minimal risk for adverse effects. Nonpharmacologic interventions include heat, massage, and
spinal manipulation. In addition, patients should be advised to maintain normal, moderate activity and exercise
(Choice C).

Nonsteroidal anti-inflammatory drugs (NSAIDs) (eg, ibuprofen) are recommended as first-line drug therapy;
acetaminophen is less effective but can be considered for patients with contraindications to NSAIDs (eg, chronic
kidney disease). If NSAIDs are ineffective, as in this patient, a short course of a nonbenzodiazepine muscle
relaxant (eg, cyclobenzaprine, tizanidine) can provide additional pain relief. Primary adverse effects include
drowsiness and anticholinergic symptoms (eg, dry mouth, constipation).

(Choice B) Opioid analgesics are no more effective than NSAIDs for mechanical low back pain; they are not
recommended over muscle relaxants due to the risk of physical dependency and addiction.

(Choice D) Indications for MRI include significant neurologic deficits (eg, saddle anesthesia) or concern for
epidural abscess (eg, fever, intravenous drug use) or spinal metastasis (eg, recent history of cancer). It is not
recommended for patients with uncomplicated, acute LBP, who are highly unlikely to have a serious etiology. In
addition, incidental imaging findings are common, which can lead to further unnecessary interventions.

(Choice E) X-ray is helpful for patients with back pain due to suspected compression fracture or spondylolisthesis;
it also can be considered for suspected vertebral metastasis though it has lower sensitivity than MRI. However, x-
ray has limited utility in uncomplicated LBP.

Educational objective:
Nonsteroidal anti-inflammatory drugs (NSAIDs) are first-line drug therapy for acute, uncomplicated low back pain. If
NSAIDs are ineffective, a short course of a nonbenzodiazepine muscle relaxant (eg, cyclobenzaprine, tizanidine)
can provide additional pain relief.

Reference
• Noninvasive treatments for acute, subacute, and chronic low back pain: a clinical practice guideline from the
American College of Physicians.
Question #400

A 28-year-old woman comes to the office due to right shoulder pain and weakness. The patient recently returned
from a 2-week hike in the Appalachian Mountains and began noticing the symptoms during the trip. She has had
no falls or trauma. The patient has no prior medical conditions and takes no medications. Vital signs are within
normal limits. Physical examination reveals weakness of right shoulder abduction and external rotation. Passive
range of motion is full. Right upper extremity sensation and deep tendon reflexes are normal. The remainder of the
physical examination shows no abnormalities. Which of the following is the most likely cause of this patient's
current symptoms?

A) Acromioclavicular ligament sprain

B) Biceps tendon degeneration

C) Borrelia burgdorferi infection

D) Cervical nerve root compression

E) Suprascapular nerve injury


Explanation
Correct Answer:

E) Suprascapular nerve injury


This patient most likely has suprascapular nerve entrapment (SNE). The suprascapular nerve provides motor
supply to the supraspinatus and infraspinatus muscles. It originates from the brachial plexus and traverses the
suprascapular notch below the superior transverse scapular ligament (which effectively transforms the notch into a
foramen).
Therefore, SNE can develop due to external compression of the nerve at the suprascapular notch, such as from
excessive use of a heavy backpack when hiking or commuting; other causes include a direct blow to the nerve (eg,
from a fall) or repetitive motion at the shoulder (eg, weight lifting, baseball). Clinical features suggestive of SNE at
the suprascapular notch include:

• shoulder pain
• weakness of shoulder abduction (supraspinatus muscle)
• weakness of external rotation (infraspinatus muscle)

Passive range of motion is preserved. Examination may show tenderness at the suprascapular notch. Cross-body
adduction of the humerus, which abducts the scapula, may reproduce pain along the top of the scapula. In
advanced cases, sensory deficits and atrophy of the supraspinatus and infraspinatus muscles may be seen. Initial
management includes nonsteroidal anti-inflammatory drugs and activity modification (eg, avoiding the use of
backpacks).

(Choice A) Acromioclavicular sprain is usually caused acutely by a fall or direct blow to the shoulder. As in SNE,
pain may be exacerbated by cross-body adduction of the arm; however, weakness of shoulder abduction is not
seen.

(Choice B) Biceps tendinopathy presents with anterior shoulder pain radiating to the upper arm. Although the long
head of the biceps crosses the glenohumeral joint and rupture may cause partial weakness of shoulder flexion,
abduction is not generally affected. Also, this injury is typically associated with a noticeable bulge in the anterior
arm.

(Choice C) Borrelia burgdorferi infection (ie, Lyme disease) can cause a variety of neuropathic syndromes in the
early disseminated phase. However, isolated acute mononeuropathy is a rare presentation, and this patient has no
other findings (eg, history of erythema migrans, fever, headache) to suggest Lyme disease.

(Choice D) Compression of the C5 and/or C6 nerve roots can present with shoulder pain and weakness of
abduction but would typically also cause dermatomal numbness, paresthesia, and diminished biceps and
brachioradialis reflexes.

Educational objective:
Suprascapular nerve entrapment presents with shoulder pain and weakness of shoulder abduction and external
rotation. It can be caused by external compression of the nerve at the suprascapular notch (eg, use of a heavy
backpack), a direct blow, or repetitive motion at the shoulder.

Reference
• Suprascapular nerve injury: a cause to consider in shoulder pain and dysfunction.
Question #401

A 40-year-old man comes to the office due to muscle and joint pains for 2 months. Review of systems is otherwise
negative. The patient underwent Roux-en-Y gastric bypass surgery for obesity 2 years ago and has lost 40 kg (88
lb) since the surgery. Vital signs are normal. BMI is 28 kg/m2. There is no pallor, lymphadenopathy, thyromegaly,
or jaundice. Abdominal examination reveals no tenderness and no organomegaly. Muscle strength is 5/5, and no
focal muscle tenderness is present. Deep tendon reflexes are normal. Laboratory results are as follows:

Hematocrit 34%
Calcium 8.2 mg/dL
Creatinine 1.1 mg/dL
Albumin 3.9 g/dL
Alkaline phosphatase 280 U/L

Which of the following is the most appropriate next step in evaluation of this patient?

A) Measurement of 25-hydroxyvitamin D level

B) Measurement of antinuclear antibodies

C) Measurement of serum creatine kinase

D) Measurement of serum transferrin saturation

E) Radionuclide bone scanning


Explanation
Correct Answer:

A) Measurement of 25-hydroxyvitamin D level

Clinical features of osteomalacia

• Malabsorption
• Intestinal bypass surgery
Causes • Celiac disease
• Chronic liver disease
• Chronic kidney disease

• May be asymptomatic
Symptoms/ • Bone pain & muscle weakness
signs • Muscle cramps
• Difficulty walking, waddling gait

• ↑ Alkaline phosphatase, ↑ PTH


• ↓ Serum calcium & phosphorus, ↓ urinary
calcium
Diagnosis • ↓ 25(OH)D levels
• Imaging
◦ Cortical thinning & reduced bone density
◦ Bilateral symmetric pseudofractures
(Looser zones)

25(OH)D = 25-hydroxycholecalciferol; PTH = parathyroid hormone.

This patient has musculoskeletal pain and an elevated alkaline phosphatase level, suggesting increased bone
turnover. Given his history of gastric bypass surgery, this most likely represents osteomalacia due to vitamin D
deficiency. Patients who have undergone Roux-en-Y bariatric procedures can develop deficiency of fat-soluble
vitamins due to reduced exposure of food to gastric acid, bacterial overgrowth, and voluntary restriction of certain
foods.

Vitamin D deficiency reduces intestinal calcium absorption, leading to secondary hyperparathyroidism and renal
phosphate wasting. The resulting hypophosphatemia causes impaired mineralization of newly deposited osteoid
matrix. Although bone contours are generally normal, the newly formed bone is weak and prone to fracture.
Initially, patients may be asymptomatic or have only nonspecific arthralgias and myalgias, but chronic deficiency
can lead to proximal muscle wasting, gait abnormalities, and deformity of weight-bearing bones.

Vitamin D deficiency should be confirmed with a serum 25-hydroxyvitamin D level, which reflects total body
vitamin D stores. In contrast, 1,25-dihydroxyvitamin D primarily reflects parathyroid hormone levels and renal
function and is not an accurate marker for deficiency.

(Choice B) Primary biliary cholangitis (PBC) is an autoimmune disorder that causes significant elevations in
alkaline phosphatase. Patients often have an abnormal antinuclear antibody test, although antimitochondrial
antibodies have more utility. However, this disorder is uncommon in men, and the primary skeletal complication is
osteoporosis, which is asymptomatic in the absence of fracture. Most patients with PBC have pruritus.

(Choice C) Muscle pain with an elevated creatine kinase level is seen in various inflammatory and metabolic
myopathies (eg, hypothyroid myopathy, statin myopathy). However, alkaline phosphatase, which reflects bone
turnover, is usually normal, and this patient's history of gastric bypass makes osteomalacia more likely.
(Choice D) The serum transferrin saturation can be used to screen for hemochromatosis. The liver disease in this
disorder causes elevations in hepatic transaminases (eg, aspartate aminotransferase, alanine aminotransferase),
but alkaline phosphatase is usually normal or only minimally elevated.

(Choice E) Radionuclide bone scanning is useful in assessing Paget disease of bone, which can cause bone pain
and an elevated alkaline phosphatase level but is uncommon at age <50 and usually causes localized bone
symptoms rather than generalized muscle and joint pains.

Educational objective:
Vitamin D deficiency in patients who have had Roux-en-Y gastric bypass surgery (and other malabsorptive
conditions) can lead to osteomalacia, presenting with diffuse musculoskeletal pain and an elevated alkaline
phosphatase level. The diagnosis is confirmed by measuring serum 25-hydroxyvitamin D.
Question #402

A 68-year-old woman comes to the office due to a 4-year history of pain and stiffness in the fingers of both hands.
The pain is worse after she does housework and improves with rest. She also has morning stiffness lasting 10-15
minutes after awakening. The patient has no other joint pains, weight loss, rash, or shortness of breath. Medical
history is notable for type 2 diabetes mellitus and Graves disease treated with radioiodine therapy. She smoked
cigarettes for 10 years and quit at age 40. Vital signs are normal. The image below shows examination findings in
the hands, and the remainder of the physical examination is normal.
Which of the following is the most appropriate next step in management?

A) Antinuclear antibody assay

B) Chest x-ray
C) No additional testing

D) Rheumatoid factor

E) Serum uric acid measurement

F) X-ray of the hands


Explanation
Correct Answer:

C) No additional testing
This patient has osteoarthritis (OA) of the hands, which presents with chronic joint pain that is worse with
activity and relieved with rest. Examination shows Heberden and Bouchard nodes at the distal interphalangeal
(DIP) and proximal interphalangeal (PIP) joints, respectively, which represent bony enlargement adjacent to the
articular cartilage and are pathognomonic for OA. Bony hypertrophy is often also found at the first carpometacarpal
joint (base of thumb).

OA of the hands is primarily a clinical diagnosis and is often apparent at the time of initial presentation. X-ray may
be performed in ambiguous cases and may show decreased joint space, subchondral sclerosis, and periarticular
osteophytes. However, imaging is less sensitive than examination findings and is usually unnecessary for diagnosis
or management (Choice F). This patient has classic examination findings, there are no additional diagnostic
considerations, and imaging does not change how the OA is treated. Therefore, no additional testing is needed.

(Choice A) Antinuclear antibody (ANA) is used to screen for systemic lupus erythematosus (SLE). However, SLE
arthritis is typically migratory and nondeforming, and patients usually have manifestations in multiple domains (eg,
skin, kidneys, lungs); ANA has low specificity (many false positives) and should not be used in patients without a
high pretest probability of SLE.

(Choice B) Chest x-ray is indicated for evaluating hypertrophic osteoarthropathy, which is commonly associated
with adenocarcinoma of the lung and other pulmonary diseases (eg, bronchiectasis). However, this condition
presents subacutely with digital clubbing, periostosis of long bones, and synovial effusions. This patient has DIP
and PIP joint enlargement, which is classic for OA.

(Choice D) Rheumatoid arthritis often affects the hands but typically presents with boggy synovitis and deformity at
the metacarpophalangeal joints and wrists; DIP joint involvement is uncommon. Patients typically experience
prolonged (>1 hr) morning stiffness; brief (<30 min) stiffness is more characteristic of OA.

(Choice E) Tophi are subcutaneous nodules composed of urate crystals in patients with gout. They often occur in
the hands but are typically isolated or irregular and asymmetric, not uniform across the DIP and PIP joints, as with
Heberden and Bouchard nodes.

Educational objective:
Heberden and Bouchard nodes represent bony enlargement at the distal and proximal interphalangeal joints and
are pathognomonic for osteoarthritis. Osteoarthritis of the hands is primarily a clinical diagnosis; imaging (eg, x-ray)
is less sensitive than examination findings and is usually unnecessary for diagnosis or management.
Question #403

A 53-year-old man comes to the emergency department at 6:00 AM due to severe pain in his right great toe for the
past 3 hours. His pain began suddenly as a dull ache, and rapidly worsened to severe throbbing that is not relieved
by acetaminophen. Medical history is notable for hypertension, type 2 diabetes mellitus, and hypercholesterolemia.
The patient has smoked 2 packs of cigarettes daily for 30 years. He drinks 2-3 beers and 3-4 cups of coffee daily,
consumes fast food often, and does not exercise regularly. Current medications include metformin, losartan,
amlodipine, sitagliptin, and atorvastatin. The patient's temperature is 36.8 C (98.2 F), blood pressure is 160/90 mm
Hg, pulse is 88/min, and respirations are 16/min. On examination, the right great toe appears markedly swollen,
red, and warm to the touch. Which of the following interventions would be most appropriate to prevent development
of further similar episodes in this patient?

A) Alcohol cessation

B) Discontinue atorvastatin

C) Discontinue losartan

D) Low-dose prednisone

E) Moderation of coffee intake

F) Smoking cessation
Explanation
Correct Answer:

A) Alcohol cessation

Prevention of future gout attacks

• Weight loss to achieve BMI <25 kg/m2


• Low-fat diet
• Decreased seafood & red meat intake
• Protein intake preferably from vegetable & low-fat dairy products
• Avoidance of organ-rich foods (eg, liver & sweetbreads)
• Avoidance of beer & distilled spirits
• Avoidance of diuretics when possible

This patient has sudden-onset severe pain in the first metatarsophalangeal joint (podagra) consistent with acute
gout. Initial treatment can include nonsteroidal anti-inflammatory drugs (eg, indomethacin), glucocorticoids, or
colchicine. Urate-lowering medications (eg, allopurinol, febuxostat) are indicated for patients with recurrent attacks
or complicated disease (eg, tophi, uric acid kidney stones). This patient is having his first attack and does not
require urate-lowering medication; however, he has multiple risk factors for recurrent gout (ie, obesity, hypertension,
insulin resistance) and should be counseled on lifestyle modification to reduce his risk.

Weight loss and caloric restriction can reduce the risk of recurrent gout. In addition, specific dietary changes are
also recommended. The risk of attacks is increased with intake of red meat and seafood but is reduced in diets
emphasizing protein from vegetarian and low-fat dairy sources. Risk is also increased by heavy intake of
beverages and foods containing fructose and other refined sugars. Heavy alcohol intake, especially from beer
and distilled spirits, is also associated with an increased risk of gout attack. Ethanol increases uric acid production
and may also decrease renal elimination of uric acid.

(Choices B and C) A number of medications, including thiazide and loop diuretics, aspirin, and beta blockers, can
increase the risk of gouty attacks. Losartan and calcium channel blockers lower uric acid levels and likely reduce
the risk of an attack. Atorvastatin and rosuvastatin also lower serum uric acid slightly.

(Choice D) Glucocorticoids may be used for acute treatment of gout as well as short-term prevention of flare-ups in
patients starting urate-lowering therapy. Otherwise, glucocorticoids are not recommended for routine gout
prophylaxis and would be relatively contraindicated in patients with diabetes.

(Choice E) Higher consumption of coffee (but not tea) is associated with lower uric acid levels and a decreased
risk of gout attack.

(Choice F) Smoking is associated with a lower risk of gout due to decreased endogenous production of uric acid.

Educational objective:
Lifestyle modifications, including alcohol cessation and weight loss, are recommended to prevent recurrent gout
attacks. Medications for lowering serum urate are indicated for patients with repeated attacks of gouty arthritis or
complicated disease (eg, tophi, uric acid kidney stones).

Reference
• Alcohol quantity and type on risk of recurrent gout attacks: An internet-based case-crossover study.
Question #404

A 24-year-old woman comes to the office with a 4-week history of joint pain. She has had moderate, achy pain and
swelling in multiple metacarpophalangeal joints of both hands, with lesser pain in the wrists, knees, and elbows. In
addition, the patient experienced intermittent fevers at the onset of her pain, but has been afebrile since then. She
has had relief of her symptoms with over-the-counter ibuprofen, but in the week since she scheduled her
appointment, she has not required medication as the pain has resolved. The patient is married and has 2 children.
Vital signs are normal. Examination shows normal range of motion in all joints tested, with no redness, warmth, or
swelling in any hand joints. X-rays of the hands are normal. Which of the following is the most likely diagnosis?

A) Crystalline arthritis

B) Fibromyalgia

C) Gonococcal arthritis

D) Lyme disease

E) Polymyalgia rheumatica

F) Rheumatoid arthritis

G) Spondyloarthropathy

H) Systemic lupus erythematosus

I) Viral arthritis
Explanation
Correct Answer:

I) Viral arthritis

Parvovirus B19 infection

• Most patients are asymptomatic or have flulike symptoms


• Erythema infectiosum (fifth disease): Fever, nausea & “slapped cheek” rash (more
Signs & common in children)
symptoms • Acute, symmetric arthralgia/arthritis: Hands, wrists, knees & feet (resembles RA)
• Transient pure red cell aplasia; aplastic crisis in patients with underlying hematologic
disease (eg, sickle cell)

• Acute infection
◦ B19 IgM antibodies in immunocompetent patients
Diagnosis ◦ NAAT for B19 DNA in immunocompromised patients
• Previous infection: B19 IgG antibodies (documents immunity)
• Reactivation of previous infection: NAAT for B19 DNA

NAAT = nucleic acid amplification testing; RA = rheumatoid arthritis.

This patient has symmetric polyarticular arthritis with a brief, self-limited course. In light of her exposure to
young children, this is likely viral arthritis due to parvovirus B19. Viral arthritis is often associated with joint
swelling and tenderness, but objective findings may be subtle or absent. Whereas children with parvovirus B19
typically present with the characteristic "slapped cheek" rash, adults are more likely to develop a nonspecific
morbilliform exanthem and may have no skin manifestations at all. A similar arthritis may also be seen with HIV,
mumps, rubella, and other viruses.

The arthritis of parvovirus B19 may resemble early rheumatoid arthritis (RA) or systemic lupus erythematosus
(SLE), and may be associated with a weakly positive rheumatoid factor or antinuclear antibody titer. However, RA
and SLE are significantly less common, usually produce more obvious synovitis, and typically follow a more chronic,
protracted course (Choices F and H). In addition, SLE is usually associated with manifestations in multiple
systems (eg, malar rash, oral ulcers, serositis) and is less likely to resolve spontaneously. If there is doubt
regarding the diagnosis, parvovirus B19 infection can be confirmed with assay for anti-parvovirus IgM or nucleic
acid amplification testing.

(Choices A and C) Crystalline arthritis (ie, gout, pseudogout) and purulent gonococcal arthritis cause acute,
inflammatory monoarthritis. Disseminated gonococcal infection can also present as chronic polyarticular arthralgias
but is asymmetric, often favors large joints, and is associated with tenosynovitis and pustular skin lesions.

(Choice B) Fibromyalgia is characterized by chronic widespread pain, often with fatigue and affective symptoms.
Localized joint swelling is not seen.

(Choice D) Untreated Lyme disease can lead to arthritis. However, symptoms are chronic and favor the large
joints (eg, knee).

(Choice E) Polymyalgia rheumatica causes pain and stiffness in the neck, shoulder, and hip muscles. Localized
arthralgias may occur but are uncommon. Polymyalgia rheumatica occurs almost exclusively in patients age >50;
this patient's age effectively rules out PMR.

(Choice G) Seronegative spondyloarthropathies (eg, ankylosing spondylitis, reactive arthritis, psoriatic arthritis) are
characterized by chronic asymmetric arthritis and inflammatory back pain.

Educational objective:
The self-limited arthritis of parvovirus B19 may resemble early rheumatoid arthritis (RA) or systemic lupus
erythematosus (SLE), and may be associated with a weakly positive rheumatoid factor or antinuclear antibody titer.
However, RA and SLE are significantly less common, typically produce more obvious synovitis, and usually follow a
chronic, protracted course.
Question #405

A 44-year-old woman comes to the office due to pain, tingling, and numbness in her hands for the past several
months. The symptoms are worse at night. She often takes ibuprofen or soaks her hands in warm water for
symptom relief, but her wrists and fingers are stiff and painful again by the next morning. The patient has difficulty
doing morning chores but feels better by midday. She also has excessive fatigue. The patient has no prior medical
conditions and takes no prescription medications. Temperature is 37.6 C (99.7 F), blood pressure is 118/72 mm Hg,
and pulse is 68/min. BMI is 28 kg/m2. Physical examination shows that the bilateral wrists and joints of the fingers
are mildly swollen and tender. Tapping on the flexor surface of the wrist elicits tingling in the first 3 digits of each
hand. The remainder of the joints are normal. Lungs are clear on auscultation and heart sounds are normal. The
abdomen is soft and nontender with no hepatosplenomegaly. The patient has no rashes or lower extremity edema.
Which of the following is the best next step in evaluation of this patient's symptoms?

A) CT scan of the chest

B) Hemoglobin A1c assay

C) MRI of the cervical spine

D) Rheumatoid factor assay

E) Serum cryoglobulin test

F) Serum vitamin B12 test


Explanation
Correct Answer:

D) Rheumatoid factor assay

Extraarticular manifestations of rheumatoid arthritis

• Fever
Systemic
• Weight loss

• Pulmonary fibrosis
Pulmonary
• Pulmonary hypertension

• Atherosclerosis
Cardiovascular
• Vasculitis

Musculoskeletal • Osteopenia/osteoporosis

Dermatologic • Rheumatoid nodules

Hematologic • Anemia
• Compressive neuropathy (eg, carpal tunnel syndrome)
Neurologic
• Cervical myelopathy

• Sjögren syndrome
Other • Raynaud phenomenon
• Scleritis, episcleritis

This patient has paresthesias in the distribution of the median nerve and a positive Tinel sign, which are consistent
with carpal tunnel syndrome (CTS). CTS is caused by compression of the median nerve as it passes through
the carpal tunnel in the wrist. Potential contributing factors include overuse and fibrosis, soft tissue swelling in the
tunnel (eg, hypothyroidism, acromegaly), and extrinsic compression from arthritis in the wrist.

This patient also has prolonged morning pain and stiffness involving the fingers and wrists, systemic symptoms (eg,
fatigue, elevated temperature), and polyarticular synovitis, suggesting rheumatoid arthritis (RA). RA can cause
tenosynovitis of the flexor tendons that pass within the carpal tunnel; in addition, the synovium can expand within
the carpal space. The resultant inflammation leads to compression of the median nerve under the flexor
retinaculum, producing the symptoms of CTS. Patients with RA are 2-3 times as likely as the general population to
develop CTS.

Initial evaluation of RA should include inflammatory markers (eg, erythrocyte sedimentation rate, C-reactive
protein), serologic studies (eg, rheumatoid factor, cyclic citrullinated peptide antibodies), and x-rays of the
symptomatic joints.

(Choice A) CT scan can confirm malignancy at the superior pulmonary sulcus (ie, Pancoast tumor), which can
cause hand pain and paresthesias due to compression of the C8-T2 nerve roots. However, the features do not
follow the distribution of the median nerve, and typical associated findings include shoulder pain, Horner syndrome
(ipsilateral ptosis, miosis, anhidrosis), and supraclavicular lymphadenopathy.

(Choice B) Diabetes mellitus (ie, elevated fasting blood glucose, elevated hemoglobin A1c) is a risk factor for CTS
but would not cause synovitis or morning stiffness.

(Choice C) MRI of the neck can evaluate a cervical radiculopathy and is warranted for suspected malignancy, rapid
progression of symptoms, or significant weakness or diminished reflexes. However, cervical radiculopathy would
not cause features confined to the median nerve at wrist.

(Choice E) Cryoglobulinemic vasculitis can cause fatigue and arthralgias and can occasionally cause
mononeuropathy or polyneuropathy. However, CTS is less typical, and most patients have purpuric skin lesions
and liver disease (eg, hepatitis C).

(Choice F) Vitamin B12 deficiency typically causes peripheral neuropathy of the lower extremities rather than CTS
and is often associated with cognitive changes (eg, memory loss).

Educational objective:
Carpal tunnel syndrome is a common manifestation of rheumatoid arthritis involving the wrist. Initial evaluation
should include inflammatory markers (eg, erythrocyte sedimentation rate, C-reactive protein), serologic studies (eg,
rheumatoid factor, cyclic citrullinated peptide antibodies), and x-rays of the symptomatic joints.

Reference
• Neurologic manifestations of rheumatoid arthritis.
Question #406

A 54-year-old man comes to the office due to morning facial puffiness and bilateral leg swelling. Medical history
includes psoriasis and recurrent pulmonary infections due to bronchiectasis. Blood pressure is 143/92 mm Hg and
pulse is 92/min. BMI is 22 kg/m2. Physical examination shows hepatosplenomegaly, bilateral ballottable enlarged
kidneys, and 2+ pitting edema of the bilateral lower extremities to the knees. Urinalysis shows 4+ proteinuria and
normal urinary sediment. Which of the following is the most likely diagnosis in this patient?

A) Amyloidosis

B) Hepatorenal syndrome

C) Hypertensive nephrosclerosis

D) IgA nephropathy

E) Polycystic kidney disease


Explanation
Correct Answer:

A) Amyloidosis

Major types of systemic amyloidosis

Dialysis
Primary Secondary Age related
related

Disease Chronic infection/ ESRD on


Plasma cell dyscrasia Elderly men
association inflammation dialysis

Precursor
Immunoglobulin Serum Transthyretin
protein
light chains amyloid A (prealbumin) β2-Microglobulin


↓ ↓ ↓
Decreased
Misfolding
Increased Increased Accumulation clearance
mechanism
production production over time


↓ ↓ ↓
Aβ2M
Amyloid
AL AA ATTR*
fibril type
Nephropathy, hepatosplenomegaly, cardiomyopathy Cardiomyopathy, CTS,
Clinical Scapulohumeral
(AL > AA), peripheral neuropathy, macroglossia, skin other peripheral
manifestations arthritis, CTS
bruising neuropathy

Amyloid names begin with "A" followed by the precursor protein abbreviation.

*ATTR also occurs in a hereditary form that affects younger patients.

CTS = carpal tunnel syndrome; ESRD = end-stage renal disease.

This patient's clinical presentation of facial swelling, bilateral lower extremity edema, and massive proteinuria is
consistent with nephrotic syndrome. A common unifying diagnosis is amyloidosis given the multisystemic
findings suggestive of infiltrative disease, including palpable and enlarged kidneys, hepatomegaly, and
splenomegaly. Specifically, this patient likely has secondary (AA) amyloidosis, which can be seen with chronic
inflammatory disease (eg, rheumatoid arthritis, psoriatic arthritis, inflammatory bowel disease) or chronic
infection (eg, bronchiectasis, tuberculosis, osteomyelitis), as serum amyloid A protein is an acute-phase reactant.

Similar to amyloidosis light-chain (AL amyloidosis), deposition of the abnormally folded protein fibrils can affect
many organ systems; cardiac involvement is much less common with AA amyloidosis than AL amyloidosis. The
diagnosis is confirmed by the presence of amyloid on abdominal fat pad biopsy; renal biopsy can be considered if
fat pad biopsy is nondiagnostic. Histology demonstrates amorphous hyaline material that stains with Congo red.
Treatment is usually directed at the underlying inflammatory disease.

(Choice B) Hepatorenal syndrome refers to acute renal failure in patients with severe acute or chronic liver disease
and portal hypertension. The renal failure results from reduced renal blood flow and does not cause significant
proteinuria.

(Choice C) Hypertensive nephrosclerosis is seen in patients with chronic hypertension and is associated with
retinopathy, progressive renal failure, and mild proteinuria (generally <1 g/day). The kidneys are usually small and
hepatomegaly is not present.

(Choice D) IgA nephropathy is a type of glomerulonephritis that usually presents with hematuria on urinalysis. It is
not typically associated with other organ involvement and would not explain this patient's hepatomegaly and
splenomegaly.

(Choice E) Patients with polycystic kidney disease typically have flank pain, hematuria, progressive renal failure,
hypertension, and large, palpable kidneys. Hepatomegaly can also be present due to cystic liver involvement.
However, splenomegaly is not typical, and overt nephrotic syndrome, as seen in this patient, is unusual.

Educational objective:
Secondary amyloidosis is a complication of a chronic inflammatory condition (eg, chronic infection, inflammatory
bowel disease, rheumatoid arthritis) resulting in extracellular tissue deposition of protein fibrils into tissues and
organs. Patients can develop multiorgan dysfunction (eg, kidneys, liver). Management usually involves treating the
underlying inflammatory disease.

Reference
• AA amyloidosis: basic knowledge, unmet needs and future treatments.
Question #407

A healthy 24-year-old woman comes to the office due to pain behind her right heel for 5 days. The pain developed
gradually and is burning in quality, worse with activity, and partially relieved with rest. There is no history of trauma.
The patient has no fever, back pain, swelling, or rash. She eats a healthy diet and walks 2 miles every morning.
She has no chronic medical problems. The patient's only medication is naproxen, which provides some relief. Her
father has gout. Vital signs are within normal limits. BMI is 24 kg/m2. Examination shows tenderness 3 cm above
the posterior calcaneus. There is no leg swelling, deformity or redness. Dorsiflexion and plantar flexion of the foot
are intact. Which of the following is the most likely cause of this patient's leg pain?

A) Achilles tendinopathy

B) Calcaneal apophysitis

C) Crystal-induced synovitis

D) Enthesitis

E) Subcutaneous calcaneal bursitis


Explanation
Correct Answer:

A) Achilles tendinopathy

Achilles tendinopathy

• Athletic activity, increase in activity


Risk factors • Systemic disorders: psoriasis, ankylosing spondylitis
• Medications: glucocorticoids, fluoroquinolones

• Swelling, warmth, pain at posterior heel


Clinical
• Tendon rupture: "popping" sensation & acute pain following rapid acceleration/direction
features
change

Examination • Swelling, tenderness 2-6 cm proximal to tendon insertion


findings • Rupture: positive Thompson test*

• Clinical findings
Diagnosis • Ultrasound: swelling, neovascularization
• MRI
• Acute: activity modification, ice, NSAIDs
Management
• Chronic: eccentric resistance exercises

*With the patient prone and feet off the end of the table, squeeze the calf muscles; the absence of plantar flexion
indicates tendon rupture.

NSAIDs = nonsteroidal anti-inflammatory drugs.

This patient has Achilles tendinopathy, presenting with acute posterior ankle pain and tenderness. Achilles
tendinopathy is often referred to as a tendinitis, although inflammatory infiltrates are not always present. Achilles
tendinopathy is most common in athletes and other active people and is often triggered by an increase in exercise
regimen. The risk is also increased following use of fluoroquinolone antibiotics.

The diagnosis of Achilles tendinopathy is based primarily on clinical findings, particularly the location and pattern
of symptoms. Pain (eg, burning sensation), swelling, and tenderness are usually most prominent approximately
2-6 cm proximal to the insertion of the tendon, where perfusion of the gastrocnemius/Achilles tendon complex is
the lowest. Management of acute Achilles tendinopathy includes activity modification, cold compresses/icing, and
nonsteroidal anti-inflammatory drugs.

(Choice B) Calcaneal apophysitis (Sever disease) is a common cause of posterior heel pain and tenderness.
However, it usually occurs in children and adolescents who participate in running or jumping sports (eg, basketball)
because the apophysis is most susceptible to stress during periods of rapid growth. Also, the findings are typically
seen within 2 cm of the insertion of the Achilles tendon.

(Choice C) Crystal-induced synovitis (eg, gout, pseudogout) typically presents as acute monoarthritis with redness,
swelling, and severe pain with passive movement. Although gout commonly involves the ankle joint, it usually does
not affect the heel or the Achilles tendon, the area of concern in this patient.

(Choice D) Enthesitis is an inflammatory disorder characterized by local pain, tenderness, and swelling at the
insertion of a tendon or ligament. It is a relatively specific finding for ankylosing spondylitis and other
spondyloarthropathies and is usually associated with back pain and stiffness.

(Choice E) Subcutaneous calcaneal (superficial to the tendon) bursitis presents with pain and tenderness at the
calcaneal prominence (Achilles tendon insertion point). It is typically associated with swelling, warmth, and, often,
erythema. Pain 3 cm proximal to the calcaneus makes Achilles tendinopathy more likely.

Educational objective:
Acute Achilles tendinopathy presents with posterior ankle pain and tenderness and typically occurs in athletes and
other active people following an increase in activity or exercise. The diagnosis is based on clinical findings, which
include pain, swelling, and tenderness approximately 2-6 cm proximal to the insertion of the tendon on the
calcaneus.

Reference
• Impact of age, sex, obesity, and steroid use on quinolone-associated tendon disorders.
Question #408

A 52-year-old man comes to the physician with a long history of joint pain. The patient has pain and stiffness of the
small joints of his hands. The symptoms are worse in the morning and can last several hours. He also has digit
swelling. The patient's hands are shown in the image below:

Which of the following is the most likely diagnosis?

A) Crystalline arthritis

B) Dermatomyositis
C) Enteropathic arthritis

D) Neuropathic arthropathy

E) Osteoarthritis

F) Psoriatic arthritis

G) Rheumatoid arthritis

H) Sarcoidosis
Explanation
Correct Answer:

F) Psoriatic arthritis

Clinical features of psoriatic arthritis

• Distal interphalangeal joints


• Asymmetric oligoarthritis
Arthritis • Symmetric polyarthritis, similar to rheumatoid arthritis
• Arthritis mutilans (deforming & destructive arthritis)
• Spondylarthritides (sacroiliitis & spondylitis)

• Enthesitis (inflammation at site of tendon insertion into bone)


• Dactylitis ("sausage digits") of toe or finger
Soft tissue & nail involvement
• Nail pitting & onycholysis
• Swelling of the hands or feet with pitting edema

• Arthritis precedes skin disease in 15% of patients


Skin lesions
• Skin lesions are present but not yet diagnosed in 15% of patients

The clinical history and image provided are most consistent with psoriatic arthritis. Psoriatic arthritis occurs in
5%-30% of patients who have psoriasis. Psoriatic arthritis can present with many different patterns, but the classic
presentation involves the distal interphalangeal (DIP) joints. Morning stiffness is present, as it is in all
inflammatory arthritides. Deformity of involved joints, dactylitis ("sausage digit," a diffusely swollen finger), and
nail involvement are common. This patient shows all of the classic signs: All of the fingers (particularly the left
index finger) are swollen, consistent with dactylitis; many of the nails show pitting and onycholysis (separation of
nail bed); and the DIP joints are prominently involved. Furthermore, well-demarcated red plaques with silvery
scaling—the classic lesions of psoriasis—are seen on the dorsum of each hand. Current treatment options for
psoriatic arthritis include nonsteroidal anti-inflammatory agents, methotrexate, and anti-tumor necrosis factor
agents.

(Choice A) Nail changes do not accompany crystalline arthritis (gout and pseudogout). Cutaneous deposits of
monosodium urate occur in chronic gout. These lesions, known as tophi, most commonly occur over joints and the
helix of the ear, and they may ulcerate.

(Choice B) Dermatomyositis can present with Gottron's papules (violaceous plaques, slightly scaly) overlying the
metacarpophalangeal (MCP) joints, which at times can look similar to psoriasis. However, dermatomyositis is not
associated with dactylitis or nail changes (onycholysis or nail pitting).

(Choice C) Enteropathic arthritis occurs in 10%-20% of patients with Crohn disease and ulcerative colitis. It most
often affects the lower extremities and sacroiliac joints and tends to wax and wane with the symptoms of bowel
disease. These patients have prominent gastrointestinal symptoms and other extraintestinal manifestations of
inflammatory bowel disease.

(Choice D) Neuropathic arthropathy (Charcot joint) most commonly affects the lower limbs. Most cases occur with
diabetes, but syphilis and alcoholism are other causes. Peripheral neuropathy is believed to result in decreased
proprioception, which leads to frequent trauma and eventual destruction of the joint.

(Choice E) Osteoarthritis often affects the DIP joints. However, the pain of osteoarthritis is worse with activity and
improves with rest (the opposite of this patient's symptoms). Osteoarthritis is noninflammatory, and dactylitis should
not occur.

(Choice G) Rheumatoid arthritis also presents with morning stiffness. The MCP and proximal interphalangeal
joints are prominently involved, whereas the DIP joints classically are not. Dactylitis and nail involvement are also
not expected in rheumatoid arthritis.

(Choice H) Polyarthritis affecting the ankles and knees sometimes occurs in sarcoidosis. Sarcoidosis causes
protean cutaneous manifestations, the most common of which is erythema nodosum. These patients also often
have cough, chest pain, and dyspnea.

Educational objective:
Psoriatic arthritis occurs in 5%-30% of patients with psoriasis. The classic presentation involves the distal
interphalangeal joints. Morning stiffness, deformity, dactylitis ("sausage digit"), and nail involvement are common.
Current treatment options for psoriatic arthritis include nonsteroidal anti-inflammatory agents, methotrexate, and
anti-tumor necrosis factor agents.
Question #409

A 78-year-old woman comes to the office due to a 3-week history of achy pain and morning stiffness in the
shoulders and hips. Associated symptoms include fatigue and low-grade fever, but the patient has had no
headache, swelling in the joints, or other new symptoms. Medical history is unremarkable. Physical examination
shows pain with active abduction at the shoulders and inability to abduct the humerus above horizontal. Motor
strength and deep tendon reflexes are normal and symmetric. Laboratory results show a markedly elevated
erythrocyte sedimentation rate and mild normocytic anemia; a serum chemistry panel and TSH are normal. Low-
dose prednisone therapy is prescribed. A week later, the patient returns and reports substantial improvement of
symptoms. Which of the following additional studies is needed to confirm the diagnosis?

A) Cyclic citrullinated peptide antibody assay

B) Echocardiography

C) Muscle biopsy

D) No additional studies

E) Temporal artery biopsy


Explanation
Correct Answer:

D) No additional studies

Polymyalgia rheumatica

• Rapid-onset pain & stiffness in shoulders & hips ± neck involvement


Clinical
• Fatigue, weight loss, low-grade fever
features
• ~10% associated with GCA (eg, headache, jaw claudication, visual symptoms)

Diagnostic • Elevated acute-phase markers (eg, ESR, CRP)


testing • Temporal artery biopsy if symptoms of GCA

Treatment • Very rapid response to oral glucocorticoids

CRP = C-reactive protein; ESR = erythrocyte sedimentation rate; GCA = giant cell (temporal) arteritis.

This patient has aching pain and stiffness in the shoulders and hips consistent with polymyalgia rheumatica
(PMR). PMR is an inflammatory disorder of the proximal joints, bursae, and tendons; the muscle itself is
unaffected (despite the name "polymyalgia") and strength remains normal, although range of motion maybe limited
in the proximal joints (eg, shoulder abduction). Morning stiffness and gelling (stiffness with inactivity) are hallmark
features. Patients also often have systemic symptoms (eg, fever, weight loss). PMR occurs almost exclusively in
patients age ≥50. Symptoms typically have a discrete onset, although if mild, patients may wait several weeks
before coming for evaluation.

There are no specific diagnostic tests for PMR, but acute-phase inflammatory markers (eg, erythrocyte
sedimentation rate, C-reactive protein) are very sensitive and, when markedly elevated, generally adequate for
initiating treatment. A rapid response to low-dose glucocorticoid therapy (eg, prednisone 20 mg daily) supports
the diagnosis; persistent symptoms despite glucocorticoids should prompt consideration of other conditions. For an
older patient with typical symptoms, elevated inflammatory markers, and rapid improvement with treatment, no
additional testing is required.

(Choice A) Cyclic citrullinated peptide antibodies are specific for rheumatoid arthritis, which can cause morning
stiffness and systemic symptoms. However, the hands are more commonly affected than the shoulders, and
swelling is typically seen at affected joints.

(Choice B) Infective endocarditis can cause myalgia, fever, and elevated inflammatory markers. However, this
patient has no risk factors (eg, valvular heart disease, intravenous drug use) or embolic manifestations (eg, splinter
hemorrhages) to suggest endocarditis. Echocardiography is not required.

(Choice C) Myopathies (eg, statin-induced myopathy, thyroid myopathy) can present with muscle pain but usually
cause weakness and tenderness; stiffness is less common. A serum creatine kinase level is often performed in the
initial evaluation of PMR, especially if the presentation is ambiguous, but muscle biopsy is unnecessary.

(Choice E) A minority of PMR cases are associated with giant cell (temporal) arteritis (GCA), characterized by
headache, temporal tenderness, jaw claudication, and visual disturbances. When GCA is suspected, patients
should be prescribed high-dose glucocorticoid therapy and referred for temporal artery biopsy. However, this
patient has no features of GCA, and temporal artery biopsy is not indicated in uncomplicated PMR.

Educational objective:
Polymyalgia rheumatica is an inflammatory disorder that occurs in patients age ≥50; it is characterized by aching
pain and stiffness in the hips, shoulders, and neck. The diagnosis is suggested by elevated inflammatory markers
(eg, erythrocyte sedimentation rate, C-reactive protein), and the condition responds rapidly to low-dose
glucocorticoid therapy.

Reference
• Pathogenesis, diagnosis and management of polymyalgia rheumatica.
Question #410

A 25-year-old man with ankylosing spondylitis comes to the office for reevaluation of persistent low back pain and
stiffness. The patient was first seen 6 months ago with intermittent pain that responded well to naproxen. The pain
has since progressed and now occurs throughout the day and is no longer relieved by naproxen. The patient is
unable to fully participate in an exercise program due to his symptoms. He is a lifetime nonsmoker and does not
use alcohol or illicit drugs. Erythrocyte sedimentation rate is 90 mm/hr. Which of the following is the most
appropriate next step in management of this patient?

A) Ibuprofen as needed

B) Methotrexate

C) Mycophenolate

D) Rituximab

E) Tumor necrosis factor antagonist


Explanation
Correct Answer:

E) Tumor necrosis factor antagonist

Treatment of ankylosing spondylitis

• Exercise (postural exercises, ROM/


Nonpharmacologic
stretching exercises)
measures
• Physical therapy

• NSAIDs (eg, ibuprofen, naproxen)


Initial treatment
• COX-2 inhibitors (eg, celecoxib)

• TNF-α inhibitors (eg, etanercept,


Treatment failure/
infliximab)
disease
• Anti–IL-17 antibodies (eg,
progression
secukinumab)

COX-2 = cyclooxygenase-2; NSAIDs = nonsteroidal anti-


inflammatory drugs; ROM = range of motion; TNF-α = tumor
necrosis factor-alpha.

This patient has ankylosing spondylitis (AS) with chronic back pain and stiffness associated with elevated
inflammatory markers (eg, erythrocyte sedimentation rate). All patients with AS should be advised on maintaining a
regular exercise program that includes postural and range-of-motion exercises; supervised physical therapy is
often beneficial, especially when initiating exercise. In addition, nonsteroidal anti-inflammatory drugs (NSAIDs) (eg,
ibuprofen, naproxen) or cyclooxygenase-2 inhibitors (eg, celecoxib) are typically very helpful and may be adequate
for many patients.

The pathogenesis of AS is driven in part by inflammatory cytokines, particularly tumor necrosis factor-alpha (TNF-
alpha) and IL-17. Specific large-molecule biologic inhibitors of these cytokines, including TNF-alpha inhibitors (eg,
etanercept, infliximab) and IL-17 inhibitors (eg, secukinumab), are indicated for patients who have severe and
persistent symptoms despite NSAID therapy.

(Choice A) When initial treatment with one NSAID fails, the condition sometimes responds to a different drug from
the class. However, this patient's symptoms are progressing despite an initial good response, and trials of
additional NSAIDs (eg, ibuprofen) are unlikely to be successful.

(Choice B) Most first-line, disease-modifying, antirheumatic drugs (eg, sulfasalazine, methotrexate), as commonly
used in the treatment of rheumatoid arthritis (RA), have little to no benefit for spinal disease in AS, although they are
occasionally used for individuals with predominantly peripheral joint symptoms.

(Choice C) Mycophenolate is most commonly used to prevent rejection in patients with allogeneic transplants. It is
also prescribed for certain autoimmune disorders (eg, RA, systemic sclerosis). Although mycophenolate is
occasionally used for uveitis associated with AS, it is not standard treatment for spinal manifestations.

(Choice D) Rituximab is a chimeric anti-CD20 monoclonal antibody that suppresses B-cell activity in patients with
RA. However, the immune response in AS is mediated primarily by T-cell activity.

Educational objective:
The initial treatment of ankylosing spondylitis includes regular exercise and nonsteroidal anti-inflammatory drugs.
Tumor necrosis factor inhibitors and IL-17 inhibitors are used in patients whose conditions do not respond to less
aggressive treatment.
Question #411

A 35-year-old man comes to the office for follow-up of low back pain. He has aching pain in the left lumbar
paraspinal area that is worse at the end of the day and relieved overnight with rest. The pain began 3 months ago
with no precipitating trauma. The patient was initially treated with intermittent doses of acetaminophen and
naproxen but continues to have moderate residual pain. There is no associated fever, weight loss, radicular pain,
lower extremity weakness, or urinary symptoms. Medical history is unremarkable. The patient is employed as a
factory line worker, requiring him to lift 5-7 kg (11-15 lb) several times daily. Vital signs are normal. On physical
examination, the patient appears comfortable. Cervical and thoracic spine range of motion is normal, but flexion
and extension of the lumbar spine elicit pain. Straight-leg raising test is normal. There is mild tenderness in the left
lumbar paraspinal tissues but no midline tenderness. Upper and lower extremity strength and deep tendon reflexes
are normal. Which of the following is the best recommendation for management of this patient's pain?

A) Discontinue working until the pain is resolved

B) Epidural glucocorticoid injection

C) Exercise therapy

D) Lumbar support brace

E) Scheduled dose of benzodiazepine


Explanation
Correct Answer:

C) Exercise therapy

Management of chronic nonspecific lower back pain

• Pain >12 weeks


Clinical features
• No evidence of specific etiology (eg, malignancy, infection, fracture)

• Maintain normal activity as tolerated


• Stretching & strengthening exercise program
Nonpharmacologic measures
• Heat
• Address maladaptive coping (eg, CBT, mind-body interventions)

• Intermittent NSAID or acetaminophen


Medication • Duloxetine, tricyclic antidepressants, or tramadol
• Nonbenzodiazepine muscle relaxants

CBT = cognitive-behavioral therapy; NSAID = nonsteroidal anti-inflammatory drug.

This patient has uncomplicated low back pain (LBP). Following initial onset of acute (<4 weeks) LBP, patients
should be advised to maintain moderate activity and use short courses of acetaminophen or nonsteroidal anti-
inflammatory drugs (NSAIDs) as needed; most patients experience complete resolution of symptoms. However,
those with subacute (4-12 weeks) or chronic (>12 weeks) LBP, such as this patient, are more likely to have
recurrent or persistent pain and therefore warrant additional intervention.

In patients with chronic LBP, exercise is beneficial in reducing pain and improving function. Patients often start with
a supervised exercise program (eg, physical therapy) that emphasizes stretching and strengthening of the back
muscles. Aerobic exercise may also be helpful. Subsequently, patients can transition to a home exercise program,
which should be continued indefinitely. Short courses of acetaminophen or NSAIDs can be used intermittently.
Some patients also may benefit from duloxetine or tricyclic antidepressants, but opioids are not advised for routine
use.

(Choice A) Prolonged activity restriction in patients with uncomplicated LBP is associated with increased long-term
pain and stiffness. Patients should be counseled to maintain normal activity.

(Choice B) Epidural glucocorticoid injections can be considered for patients with lumbosacral radiculopathy who
have not responded to initial treatment. They are not helpful for nonradicular LBP.

(Choice D) Lumbar support braces are commonly recommended in the lay media for chronic LBP. However, such
braces may discourage activity, and there is limited evidence of reduced pain, improved function, or lower
recurrence rates. The long-term benefit, if any, is likely much less than that from an appropriate exercise program.

(Choice E) Nonbenzodiazepine muscle relaxants (eg, cyclobenzaprine) can be used as an adjunct therapy for
LBP. However, benzodiazepines are not indicated for this purpose because they have minimal evidence of benefit,
carry significant abuse potential, and can cause severe respiratory depression in patients on concurrent opioid
therapy.

Educational objective:
Management of chronic back pain should include an exercise program emphasizing stretching and strengthening of
the back muscles. Acetaminophen or nonsteroidal anti-inflammatory drugs can be used intermittently. Some
patients may benefit from duloxetine or tricyclic antidepressants, but opioids and benzodiazepines are not advised.

Reference
• Exercise interventions for the treatment of chronic low back pain: a systematic review and meta-analysis of
randomised controlled trials.

• Clinical practice guidelines for the management of non-specific low back pain in primary care: an updated
overview.
Question #412

A 48-year-old man comes to the office due to pain in his right hand. He has had symptoms intermittently for the
past year, but they have become worse over the past 3 months, particularly at night. The pain radiates to the
anterior aspect of the forearm. Medical history is notable for type 2 diabetes mellitus; his most recent hemoglobin
A1c was 8.5%. The patient does not use tobacco, alcohol, or recreational drugs. He works at a retail supermarket
where he is responsible for stocking heavy canned goods. Blood pressure is 148/95 mm Hg, pulse is 76/min, and
respirations are 12/min. BMI is 32 kg/m2. On examination, the patient appears comfortable. There is weakness of
thumb opposition and slightly decreased light touch sensation over the palmar surface of the tip of the thumb, index
finger, and middle finger. Sensation is normal elsewhere. Which of the following is the most likely cause of this
patient's symptoms?

A) Complex regional pain syndrome

B) Focal demyelination in the white matter

C) Glycosylation of proteins in the vasa nervorum

D) Nerve compression at the wrist

E) Nerve compression in the forearm


Explanation
Correct Answer:

D) Nerve compression at the wrist

Carpal tunnel syndrome

• Obesity
• Pregnancy
• Diabetes mellitus
Risk factors
• Hypothyroidism
• Rheumatoid arthritis
• End-stage renal disease/hemodialysis

• Pain & paresthesia in median nerve distribution (first 3½ digits)


• Positive Phalen, Tinel, or Durkan (carpal compression) test
Clinical presentation
• Severe disease: weakness of thumb abduction & opposition, atrophy of
thenar eminence

Confirmatory test • Nerve conduction studies

• Wrist splinting
Treatment
• Glucocorticoid injection
• Surgery for severe or refractory symptoms

This patient has pain and decreased sensation in the hand, consistent with carpal tunnel syndrome (CTS). CTS
is caused by compression of the median nerve as it passes deep to the flexor retinaculum in the wrist and is
common in patients with diabetes mellitus, as well as in individuals with repetitive, forceful use of the wrist and
hand.

Manifestations of CTS include paresthesia, pain, and sensory loss in the palmar aspect of the first 3 digits and
sometimes in the lateral aspect of the fourth; patients may also have referred pain to the lateral palm, forearm, or
even upper arm. However, because the palmar cutaneous branch of the median nerve passes outside the carpal
tunnel, sensation over the thenar eminence typically remains intact.

In the clinic, symptoms may be provoked by percussion (Tinel test) or manual compression (Durkan test) over the
nerve, or by forceful flexion of the wrist (Phalen test). Early CTS typically has an intermittent course with symptoms
often worse at night. However, manifestations can be highly variable, particularly with severe disease. As the
disorder progresses, symptoms may become persistent and/or associated with motor features (eg, thenar atrophy,
weakness of thumb abduction and opposition).

(Choice A) Complex regional pain syndrome causes pain in a regional (vs specific peripheral nerve) distribution. It
typically follows trauma or surgery and is associated with edema, vasomotor signs, and trophic changes in skin and
hair.

(Choice B) Multiple sclerosis is caused by focal demyelination in the white matter. Clinical features (eg, sensory
disturbances, incontinence, optic neuritis) are intermittent and progressive, and occur in multiple neural
distributions.

(Choice C) Glycosylation of proteins in the vasa nervorum leads to diabetic peripheral neuropathy. It causes pain
in the extremities but usually occurs in a stocking-and-glove pattern; findings tend to be most prominent in the feet.

(Choice E) Median nerve compression in the forearm (eg, pronator teres syndrome) can cause forearm pain and
sensory loss in the palmar aspects of the first 3 digits; however, additional sensory loss over the lateral palm and
thenar eminence would be expected due to involvement of the palmar cutaneous branch.

Educational objective:
Carpal tunnel syndrome is caused by compression of the median nerve at the wrist. It initially has an intermittent
course, but with progression, the symptoms may become more persistent and associated with motor features. Pain
may radiate to the forearm or upper arm, but sensory loss in these areas is not seen.

Reference
• Pathophysiology of carpal tunnel syndrome.

• Occupational and biopsychosocial risk factors for carpal tunnel syndrome.


Question #413

A 40-year-old woman comes to the office for follow-up. The patient has a 4-year history of rheumatoid arthritis and
has been taking a disease-modifying therapy. She reports significant improvement in joint pain and stiffness with
treatment and can now perform daily activities without difficulty. The patient has no other medical conditions and
does not use tobacco, alcohol, or illicit drugs. She consumes a balanced diet and exercises most days of the
week. Vital signs are normal. Physical examination shows no significant joint swelling, erythema, or tenderness.
Laboratory results are as follows:

Complete blood count


Hemoglobin 11.2 g/dL
Mean corpuscular volume 108 µm3
Platelets 226,000/mm3
Leukocytes 7,800/mm3

Serum chemistry
Sodium 140 mEq/L
Potassium 4.0 mEq/L
Bicarbonate 24 mEq/L
Creatinine 0.8 mg/dL
Calcium 9.6 mg/dL
Glucose 98 mg/dL

Laboratory studies were within normal limits 6 months ago. Which of the following is the most likely additional
adverse effect of this patient's pharmacotherapy?
A) Hepatotoxicity

B) Neurotoxicity

C) Osteoporosis

D) Retinal toxicity

E) Tuberculosis reactivation
Explanation
Correct Answer:

A) Hepatotoxicity

Disease-modifying antirheumatic drugs

Agent Mechanism Adverse effects

• Hepatotoxicity
Methotrexate • Folate antimetabolite • Stomatitis
• Cytopenias

• Hepatotoxicity
Leflunomide • Pyrimidine synthesis inhibitor
• Cytopenias

Hydroxychloroquine • TNF & IL-1 suppressor • Retinopathy

• Hepatotoxicity
Sulfasalazine • TNF & IL-1 suppressor • Stomatitis
• Hemolytic anemia
• Infection
TNF inhibitors
• Demyelination
(eg, adalimumab, certolizumab,
• Congestive heart failure
etanercept, golimumab, infliximab)
• Malignancy

IL-1 = interleukin 1;TNF = tumor necrosis factor.

This patient with rheumatoid arthritis (RA) is on disease-modifying antirheumatic drug (DMARD) therapy, which
improves long-term joint function and is initiated as soon as practical after diagnosis. Methotrexate is the preferred
first-line DMARD for most patients with RA.

However, this patient now has macrocytic anemia (mean corpuscular volume >100 µm3), a potential adverse effect
of methotrexate. Methotrexate inhibits dihydrofolate reductase, which can lead to cellular folate depletion.
Hematologic effects can range from mild macrocytosis to severe pancytopenia. Methotrexate is also associated
with hepatotoxicity, especially in patients with comorbid liver disease. Mild elevations in hepatic transaminases
are common, and chronic liver disease and cirrhosis may occur over time. Other adverse effects of methotrexate
include nausea, stomatitis, rash, interstitial lung disease, alopecia, and fever.

Therefore, patients on methotrexate should have regular monitoring with complete blood counts and hepatic
function markers (eg, serum albumin, transaminases). Much of the toxicity of methotrexate, including
hepatotoxicity, can be mitigated by concurrent administration of folic (or folinic) acid, which does not reduce the
effectiveness of the drug. Due to the risk of hepatotoxicity, patients should avoid alcohol intake while on treatment.

(Choice B) Calcineurin inhibitors (eg, cyclosporine, tacrolimus) are associated with neurotoxicity; manifestations
include headache, seizures, tremor, encephalopathy, and peripheral pain. However, these medications do not
commonly cause macrocytic anemia and are rarely used for RA.

(Choice C) Glucocorticoids (eg, prednisone) are used in acute management of RA. Major adverse effects include
Cushing syndrome, osteoporosis, adrenocortical atrophy, and poor wound healing; however, they do not commonly
cause macrocytosis. Unlike DMARDs, glucocorticoids do not alter the course of joint destruction and are not
continued chronically in most patients.

(Choice D) Hydroxychloroquine is an antimalarial DMARD that is well tolerated in management of RA and other
autoimmune disorders. It can cause irreversible retinal toxicity and warrants regular ophthalmologic examination.
Hematologic effects are uncommon.

(Choice E) Tumor necrosis factor (TNF) inhibitors (eg, etanercept, adalimumab) are large-molecule biologic
DMARDs that are very effective in the treatment of RA. They have potent immunosuppressive qualities and are
associated with increased risk for reactivation of latent tuberculosis. TNF inhibitors commonly cause neutropenia,
but macrocytic anemia is not a common effect.

Educational objective:
Methotrexate is a disease-modifying antirheumatic drug used for rheumatoid arthritis. Macrocytic anemia and
hepatotoxicity are common adverse effects. The toxicity of methotrexate (including hepatotoxicity) can be mitigated
by the administration of folic acid, which does not reduce the effectiveness of the drug.

Reference
• Hepatic and hematological adverse effects of long-term low-dose methotrexate therapy in rheumatoid
arthritis: an observational study.
Question #414

A 56-year-old woman comes to the office with her daughter to discuss knee pain, which has been ongoing for
several years. The patient has a high tolerance for pain and has previously refused to see a doctor for her
condition. Lately, the pain has been affecting the patient's daily activities, and she spends most of the day sitting in
a chair. The knee pain and stiffness are more pronounced on the right side and are worse in the morning but
gradually improve by the afternoon. She takes no medications. Review of systems is positive for fatigue. Vital
signs are within normal limits. BMI is 31 kg/m2. Lower extremity examination shows that both knees are swollen
and tender but more so on the right side. The patient is unable to fully flex or extend the right knee. There is mild,
bilateral atrophy of the quadriceps. Radiographs of the knees reveal osteopenia of the distal femur, multiple
periarticular erosions, and soft tissue swelling. Which of the following is the best long-term management for the
most likely diagnosis in this patient?

A) Antifolate immunosuppressant

B) Knee braces and weight loss

C) Systemic glucocorticoid

D) Total knee replacement

E) Urate-lowering therapy
Explanation
Correct Answer:

A) Antifolate immunosuppressant

Osteoarthritis vs rheumatoid arthritis

Osteoarthritis Rheumatoid arthritis

Age of onset >40; increases with age 40-60; often younger

• Knees & hips • MCP joint


Joint involvement • DIP joint • PIP joint
• First CMC joint • Wrists

Morning stiffness None/brief Prolonged

• Fever
Systemic symptoms Absent • Fatigue
• Weight loss
Examination • Hard, bony enlargement of joints • Soft/spongy, warm joints

• Narrowed joint space


X-ray • Periarticular erosions
• Osteophytes

CMC = carpometacarpal; DIP = distal interphalangeal; MCP = metacarpophalangeal; PIP = proximal


interphalangeal.

The 2 most prevalent forms of arthritis are osteoarthritis (OA) and rheumatoid arthritis (RA), and individuals may
have both conditions concurrently. OA is much more common and characteristically involves the knees, hips, and
distal finger joints. RA, an inflammatory arthritis, typically involves the metacarpophalangeal, proximal
interphalangeal, and wrist joints. However, any joint may be affected. This patient has multiple clinical and
radiographic features suggestive of RA: Her prolonged morning stiffness, systemic symptoms (ie, fatigue), and
soft tissue swelling involving multiple joints are more consistent with RA than OA; she also has X-ray findings that
favor a diagnosis of RA over OA, including periarticular erosions (seen here on hand x-rays), distal osteopenia, and
soft tissue swelling.

Additional evaluation of patients with suspected RA should include inflammatory markers (eg, erythrocyte
sedimentation rate, C-reactive protein), which are typically elevated in RA but normal in OA. Cyclic citrullinated
peptide antibodies are present in most patients with RA and can help confirm the diagnosis; rheumatoid factor
should also be tested but has lower specificity.

Initial management of RA includes nonsteroidal anti-inflammatory drugs and systemic glucocorticoids (eg,
prednisone) to acutely reduce pain; however, these interventions do not alter the long-term prognosis and are
inadequate for definitive therapy (Choice C). As soon as possible after the diagnosis is made, disease-modifying
antirheumatic drugs (DMARDs) should be initiated to slow disease progression; first-line options include
methotrexate, a folate antimetabolite that is cost-effective and well tolerated by most patients. Alternate options
include sulfasalazine, leflunomide, and hydroxychloroquine.
(Choice B) Nonpharmacologic interventions for OA include regular exercise and weight loss; knee braces may
reduce pain in some patients but are unlikely to alter the long-term course of the condition. However, none of these
interventions are adequate treatment for RA.

(Choice D) Total knee arthroplasty is most commonly performed for patients with advanced OA. It is occasionally
needed for patients with severe deformity due to RA, but early initiation of DMARD therapy may eliminate the need
for later joint replacement.

(Choice E) Urate-lowering therapy is indicated for long-term treatment of gout. Gout commonly affects the knee
but typically causes episodic symptoms with brief but severe inflammatory changes (eg, redness, warmth,
swelling). X-rays in early gout are generally normal, although chronic findings include punched-out erosions with an
overhanging rim of cortical bone ("rat bite" lesion).

Educational objective:
Clinical findings that favor the diagnosis of rheumatoid arthritis (RA) over osteoarthritis include prolonged morning
stiffness, systemic symptoms, and soft tissue swelling. Other characteristic findings in RA include periarticular
erosions on x-ray, elevated inflammatory markers, and positive cyclic citrullinated peptide antibodies or rheumatoid
factor. Disease-modifying antirheumatic drugs (eg, methotrexate) slow disease progression and should be initiated
on diagnosis.

Reference
• Early diagnosis and treatment of rheumatoid arthritis.

• Conventional radiology in rheumatoid arthritis.


Question #415

A 55-year-old woman comes to the office with right shoulder pain for the past 2 months. The patient is unable to
raise her arm or reach for objects. Her right hand is dominant, but she has recently been using her left more due to
the problems on the right. The patient lives with her disabled spouse, who had a stroke 8 months ago. Before the
pain, she had taken care of his daily activities and helped transfer him from his bed to wheelchair but is now unable
to do so. The patient's other medical conditions include type 2 diabetes mellitus, Graves disease, hypertension,
and hypercholesterolemia. On examination, there is markedly decreased passive and active abduction, flexion, and
rotation at the right shoulder. Palpation of the shoulder and acromioclavicular joint do not reproduce the pain. The
remainder of the physical examination is unremarkable. X-ray of the shoulder is shown below.
Which of the following is the most likely cause of this patient's current symptoms?
A) Adhesive capsulitis

B) Calcific tendinopathy

C) Osteoarthritis

D) Rotator cuff tear

E) Rotator cuff tendonitis


Explanation
Correct Answer:

A) Adhesive capsulitis

Common causes of shoulder pain

• Pain with abduction, external rotation


Rotator cuff impingement or • Subacromial tenderness
tendinopathy • Normal range of motion with positive impingement tests (eg,
Neer, Hawkins)

• Similar to rotator cuff tendinopathy


Rotator cuff tear • Weakness with abduction & external rotation
• Age >40

Adhesive capsulitis • Decreased passive & active range of motion


(frozen shoulder) • Stiffness ± pain

• Anterior shoulder pain


Biceps tendinopathy or rupture • Pain with lifting, carrying, or overhead reaching
• Weakness (less common)
• Uncommon & usually caused by trauma
Glenohumeral osteoarthritis • Gradual onset of anterior or deep shoulder pain
• Decreased active & passive abduction & external rotation

This patient has chronic shoulder pain with decreased passive and active range of motion (ROM), consistent with
adhesive capsulitis (sometimes termed frozen shoulder). Adhesive capsulitis is characterized by loss of the
normal distensibility of the glenohumeral joint capsule due to chronic inflammation, fibrosis, and contracture. It is
most common in middle-aged and older individuals, and the risk is increased in patients with diabetes mellitus,
thyroid disorders, and chronic immobility. It is also common in patients with rotator cuff tendinopathy or fracture of
the proximal humerus.

Patients with adhesive capsulitis typically report gradual-onset shoulder stiffness that limits the ability to flex, abduct,
or rotate the humerus. The diagnosis is confirmed by demonstrating a reduction in both passive and active ROM
in multiple planes. X-ray findings, as in this patient, are usually normal, and imaging is used primarily to rule out
other causes of pain.

(Choice B) Calcific tendinopathy of the shoulder is caused by hydroxyapatite deposits in the rotator cuff tendons. It
is characterized by pain during abduction, which may be severe. Active ROM may be limited because of pain, but
passive ROM remains normal, and calcifications of the tendons are visible on x-ray.

(Choice C) Osteoarthritis of the glenohumeral joint can decrease both active and passive ROM. However, this
condition is relatively uncommon, and most patients have a history of significant past shoulder trauma.

(Choices D and E) Rotator cuff tears often follow an injury (eg, fall on outstretched hand) and typically cause
weakness with active shoulder abduction and rotation. Rotator cuff (primarily supraspinatus) tendonitis is an
overuse syndrome characterized by painful abduction and external rotation (eg, heavy lifting above the shoulder).
Patients with these disorders may resist manipulation of the arm because of pain, but passive ROM is normal.

Educational objective:
Adhesive capsulitis is characterized by pain and loss of distensibility of the glenohumeral joint capsule due to
chronic inflammation, fibrosis, and contracture. It is often seen in patients with diabetes mellitus, thyroid disorders,
chronic immobility, rotator cuff tendinopathy, and fracture of the proximal humerus. Examination shows decreased
active and passive range of motion in multiple planes. X-ray findings are normal.

Reference
• Adhesive capsulitis of the shoulder.
Question #416

A 60-year-old man comes to the office due to fatigue and muscle weakness in the extremities. The weakness has
progressively worsened over the past few months, and is associated with a 7-kg (15.4-lb) weight loss. He has no
other medical conditions. The patient smokes 2 packs of cigarettes daily and consumes alcohol occasionally. On
examination, he has difficulty rising from the chair to sit on the examining table. Muscle strength is 3/5 in the
proximal muscle groups symmetrically and is not improved by repeated muscle contractions. Reflexes are intact
and symmetric bilaterally. No sensory abnormality is present. There are erythematous patches and violaceous
papules involving the dorsum of the fingers. Chest x-ray reveals an ill-defined mass in the right lower lobe. The
patient's muscle weakness is most likely due to a pathologic process at which of the following?

A) Muscle fibers

B) Peripheral nerves

C) Postsynaptic membrane

D) Presynaptic membrane

E) Spinal cord
Explanation
Correct Answer:

A) Muscle fibers

Paraneoplastic
Involved site Clinical features
syndrome

• Fluctuating muscle weakness


◦ Ocular (ptosis, diplopia)
Acetylcholine receptor in ◦ Bulbar (dysphagia,
Myasthenia gravis
postsynaptic membrane dysarthria)
◦ Facial, neck & limb
muscles

• Proximal muscle weakness


• Autonomic dysfunction (eg, dry
Presynaptic membrane mouth)
Lambert-Eaton
voltage-gated calcium • Cranial nerve involvement
syndrome
channels (eg, ptosis)
• Diminished or absent deep-
tendon reflexes

Dermatomyositis/ • Symmetrical & more proximal


Muscle fiber injury
polymyositis muscle weakness
• ILD, Raynaud phenomenon
• Polyarthritis
• Dysphagia
• Skin findings (eg, Gottron
papules, heliotrope rash) in
dermatomyositis

ILD = interstitial lung disease.

This patient with progressive muscle weakness and a rash also has features concerning for lung cancer (weight
loss, smoking history, right lower lobe mass). Malignancies can cause paraneoplastic syndromes, which are not
directly related to the cancer's invasion but rather are due to tumor-produced hormones, cytokines, or cross-reactive
antibodies (eg, shared antigens between proteins expressed on tumor cells and human cells). They are most
commonly seen in cancers of the lung, breast, and ovaries and in lymphomas. Manifestations typically resemble
those of primary endocrine, metabolic, hematologic, or neuromuscular disorders.

Paraneoplastic syndromes affecting the peripheral nerve and/or muscle (eg, myasthenia gravis, Lambert-Eaton
syndrome, dermatomyositis) most commonly present with muscle weakness, as seen in this patient. This patient
likely has dermatomyositis, an idiopathic inflammatory myopathy with immune-mediated muscle fiber injury. It
typically presents with symmetrical, proximal muscle weakness and an erythematous rash on the dorsum of the
fingers (Gottron sign) and/or upper eyelids (heliotrope eruption). The weakness is often painless, although mild
tenderness can occur in 25%-50% of cases. Reflexes and sensation are preserved. Myositis-specific antibodies
(eg, anti–Jo-1) can be helpful in clarifying the condition, and muscle biopsy is often diagnostic, although not always
necessary.

Other paraneoplastic syndromes include the following:

• Myasthenia gravis, which is due to autoantibodies against acetylcholine receptors in the postsynaptic
membrane. It is characterized by fluctuating ocular (ptosis, diplopia), facial, and/or bulbar muscle weakness
that becomes worse with repetitive tasks. This patient's skin findings and absence of eye findings make
myasthenia gravis less likely (Choice C).

• Lambert-Eaton myasthenic syndrome, which is a paraneoplastic disorder (commonly occurring in lung


malignancy) characterized by antibody-mediated inflammation of the voltage-gated calcium channels on the
presynaptic membrane. It can cause proximal muscle weakness, but patients usually have autonomic
dysfunction (eg, dry mouth, erectile dysfunction) and absent or diminished reflexes, and symptoms usually
improve with repeated isometric contractions (Choice D).

• Paraneoplastic myelopathy of the spinal cord, which typically presents with flaccid or spastic paraplegia or
quadriplegia, sensory deficits, and/or urinary or fecal retention/incontinence (Choice E).

(Choice B) Peripheral nerve injury can cause motor weakness but typically manifests with sensory (eg, numbness,
paresthesia) abnormalities and abnormal reflexes on physical examination. This patient's normal reflexes and
sensory examination make this unlikely.

Educational objective:
Dermatomyositis is an inflammatory myopathy characterized by immune-mediated injury to muscle fibers that often
occurs as a paraneoplastic syndrome in malignancy. Patients typically have symmetrical, proximal muscle
weakness and an erythematous rash over the dorsum of the fingers (Gottron sign) and/or upper eyelids (heliotrope
rash).

Reference
• Dermatomyositis: clinical features and pathogenesis.
Question #417

A 34-year-old man comes to the office due to low back pain and tightness for the past 3 months. The pain is dull,
has an aching quality, and has become progressively worse. It is worse late at night and in the early morning hours
but improves gradually during the day as he moves around. The patient has had no recent injury. Medical history is
unremarkable. He lives with his wife and does not use tobacco, alcohol, or recreational drugs. Temperature is 36.1
C (97 F), blood pressure is 120/76 mm Hg, and pulse is 80/min. Examination shows no tenderness over the lumbar
spine or lower extremity weakness. Straight-leg raising test is negative. Which of the following most likely accounts
for this patient's symptoms?

A) Decreased bone mineralization

B) Inflammation at ligamentous insertions

C) Ligamentous sprain

D) Lumbar disc degeneration

E) Lumbar spinal stenosis


Explanation
Correct Answer:

B) Inflammation at ligamentous insertions

Causes of chronic low back pain

Mechanical
• Normal neurologic examination
(eg, muscle strain,
• Paraspinal tenderness
disk degeneration)

• Radiation below the knee


Radiculopathy
• Positive straight-leg raising test
(eg, herniated disc)
• Neurologic deficits

• Pseudoclaudication
Spinal stenosis
• Relief with leaning forward

• Worse with rest, better with


Inflammatory
activity
(eg, spondyloarthropathy)
• Sacroiliitis
• Age >50
Metastatic cancer • Worse at night
• Unrelieved with rest

Infectious • Recent infection or IVDU


(eg, osteomyelitis, discitis) • Fever, focal spine tenderness

IVDU = intravenous drug use.

Most acute low back pain is due to mechanical causes (eg, muscle strain, ligamentous sprain); it typically improves
with rest and resolves spontaneously over 2-4 weeks. However, this patient has inflammatory back pain, which is
characterized by chronic, gradual-onset pain and stiffness that worsens with rest (eg, in the later part of the night)
and improves with activity. Inflammatory back pain typically begins at age <40 and is a characteristic feature of
spondylarthritis, an inflammatory arthritis of the spine.

The most common spondylarthritis is ankylosing spondylitis, which characteristically affects the sacroiliac and
apophyseal joints of the axial spine; however, spinal involvement can also occur with psoriatic arthritis, reactive
arthritis, and arthritis associated with inflammatory bowel disease. Spondyloarthropathies affect the sites of
ligamentous insertion (enthesitis) and eventually destroy articular cartilage, resulting in progressive pain and
stiffness. Examination findings are often normal early in the disease course, but patients may show reduced spinal
range of motion and tenderness at the sacroiliac joints and at tendon insertion sites (eg, along the vertebrae, hands,
insertion of Achilles tendon).

(Choice A) Osteomalacia is characterized by decreased mineralization of osteoid. It presents with bone pain and
muscle weakness and, over time, can lead to fractures. Back pain due to vertebral deformity or fracture in
osteomalacia would likely be worse with activity and better with rest, and osteomalacia is uncommon in young
patients without an underlying cause (eg, malabsorption, chronic kidney disease).
(Choice C) Ligamentous sprains can often be traced to a specific event or action. Patients typically experience
more pain with movement and improvement with rest. Symptoms generally resolve within days to weeks.

(Choice D) Lumbar disc degeneration can present with mechanical back pain or lumbosacral radiculopathy (ie,
pain radiating below the knee along the sciatic nerve). The pain is typically worse with movement. Atraumatic
lumbar disc degeneration is uncommon in young patients.

(Choice E) Lumbar spinal stenosis is usually seen in patients age >60 due to degenerative arthritis. The pain,
which often radiates to the distal lower extremity, is typically worse when ambulating with the spine extended and
relieved with spinal flexion (ie, pseudoclaudication).

Educational objective:
Inflammatory back pain is characterized by the gradual onset of pain and stiffness that begins at a young age (<40),
worsens with rest (eg, at night), and improves with activity. It suggests spondyloarthropathy (eg, ankylosing
spondylitis, psoriatic arthritis, reactive arthritis, arthritis associated with inflammatory bowel disease).
Question #418

A 25-year-old man comes to the clinic due to a 3-month history of right shoulder pain. He also has swelling and
pain at his heels. The patient has tried using a topical "arthritis cream" without improvement, although over-the-
counter analgesics have provided some relief. Medical history includes influenza pneumonia at age 12, but he has
otherwise been healthy and takes no regular medications. The patient does not use tobacco, alcohol, or illicit
drugs. His temperature is 37.3 C (99.2 F), blood pressure is 115/75 mm Hg, pulse is 56/min and regular, and
respirations are 12/min. On examination, there is pain with resisted abduction at the shoulder and tenderness at
the acromioclavicular junction. Palpation over the heels, iliac crests, and tibial tuberosities also elicits tenderness.
Which of the following additional findings is most likely in this patient?

A) Hand joint deformities

B) Limited spine mobility

C) Positive rheumatoid factor

D) Proteinuria

E) Subcutaneous nodules
Explanation
Correct Answer:

B) Limited spine mobility

Ankylosing spondylitis

• Insidious onset at age <40


• Symptoms >3 months
Inflammatory back pain
• Relieved with exercise but not rest
• Nocturnal pain

• Arthritis (sacroiliitis)
• Reduced chest expansion & spinal mobility
Examination findings • Enthesitis (tenderness at tendon insertion sites)
• Dactylitis (swelling of fingers & toes)
• Uveitis

• Osteoporosis/vertebral fractures
Complications • Aortic regurgitation
• Cauda equina syndrome

Laboratory • Elevated ESR & CRP


• HLA-B27 association

• X-ray of sacroiliac joints


Imaging
• MRI of sacroiliac joints

CRP = C-reactive protein; ESR = erythrocyte sedimentation rate.

Enthesitis is characterized by inflammation and pain at sites where tendons and ligaments attach to bone. It is
commonly due to tendon or ligament stress and can manifest as acute swelling and pain. Examination findings
include tenderness at tendon insertion sites. Enthesitis at the insertion of the Achilles tendon at the heel is often
the most prominent presentation. However, enthesitis can also be seen at the costosternal junction, shoulders,
elbows, hips, iliac crests, tibial tuberosities, and other locations. Chronic complications of enthesitis include fibrosis
and calcification.

Enthesitis may occur as an isolated disorder (eg, plantar fasciitis) but is a characteristic finding in the
spondyloarthropathies such as ankylosing spondylitis (AS), psoriatic arthritis, and reactive arthritis. It may
manifest with isolated heel pain at the insertion of the Achilles tendon, but can also involve multiple regions as in
this case. Low back pain (that classically improves with activity), loss of spinal mobility, and peripheral arthritis are
other common musculoskeletal manifestations of AS.

(Choices A, C, and E) Rheumatoid arthritis (RA) is characterized by chronic synovitis leading to hand joint
deformities (eg, ulnar deviation of the fingers, swan-neck and boutonnière deformities). Many patients have a
positive rheumatoid factor, and some will have painless, slow-growing subcutaneous nodules. Enthesitis is not a
prominent feature of RA. Enthesitis can be seen with psoriatic arthritis, which can lead to sausage digits; however,
this patient has no history of skin problems to suggest psoriasis.

(Choice D) Proteinuria and arthritis are common manifestations of systemic lupus erythematosus. Lupus is a
systemic illness that is likely to present with other findings, including weight loss, fatigue, rash, serositis, and
neurological symptoms. Enthesitis is not expected.

Educational objective:
Enthesitis is characterized by inflammation and pain at sites of tendon and ligament attachment to bone. It is a
common finding in ankylosing spondylitis and other spondyloarthropathies.

Reference
• Enthesitis: The clue to the pathogenesis of spondyloarthritis?
Question #419

A 45-year-old man comes to the office for evaluation of pain in his right lateral thigh. The pain has been present for
2 weeks and is described as a "pins and needles" sensation; it is exacerbated when he strains for bowel
movements. The patient has never had this pain before and has no associated leg weakness or back pain.
Medical history is significant for recently diagnosed type 2 diabetes mellitus, for which he was prescribed metformin
3 months ago. Vital signs are normal. BMI is 37 kg/m2. Examination shows loss of light touch and pinprick
sensation in an area of the upper lateral thigh. There is no tenderness to palpation of the lateral femoral
epicondyle. Knee and hip examinations are normal. A straight leg raising test is negative bilaterally and strength is
5/5 in both extremities. Which of the following is the most appropriate next step in management?

A) Discontinue metformin

B) MRI of the spine

C) Nerve conduction studies

D) Physical therapy

E) Plain x-ray of the hip and pelvis

F) Reassurance and conservative therapy


Explanation
Correct Answer:

F) Reassurance and conservative therapy

Meralgia paresthetica

Lateral femoral cutaneous nerve entrapment or injury due to:


Etiology
• Compression (eg, tight clothing, obesity, pregnancy)
• Iatrogenic injury (eg, total hip arthroplasty)

Clinical • Paresthesia & ↓ sensation at lateral thigh


features • Normal motor & reflex examination

• Avoidance of tight garments, weight loss


• Nonopioid analgesics (eg, NSAIDs)
Management
• Anticonvulsants (eg, gabapentin), nerve block, or surgical
release for refractory cases

NSAIDs = nonsteroidal anti-inflammatory drugs.

This patient has localized pain and paresthesia in the lateral thigh consistent with meralgia paresthetica (MP). MP
is caused by compression of the lateral femoral cutaneous nerve (LFCN) where it passes under the inguinal
ligament into the thigh. Patients experience symptoms in the distribution of the nerve but have no weakness
because the nerve contains no motor fibers.

MP is often caused by tight clothing or belts or by abdominal straining; the risk is increased in patients who are
pregnant, obese, or have diabetes mellitus. The diagnosis is made clinically; no imaging is needed. Because MP
is a benign and self-limited condition, conservative treatment (eg, avoiding tight garments, weight loss) is usually
adequate and improves symptoms in >90% of cases. Patients with persistent symptoms may respond to
anticonvulsants (eg, gabapentin) or local nerve block.

(Choice A) Long-term use of metformin has been associated with vitamin B12 deficiency and a possible increased
risk of peripheral neuropathy, but this would typically affect the distal extremities and would not be seen within 3
months. However, metformin can facilitate weight loss and may therefore alleviate MP.

(Choice B) MRI of the lumbar spine is indicated for suspected spinal pathology, as in certain patients with sciatica
(eg, back pain radiating to posterior leg, positive straight leg raise test) who have concerning neurologic deficits (eg,
saddle anesthesia, urinary retention) or in whom malignancy (eg, weight loss) or epidural abscess (eg, fever,
intravenous drug use) is suspected. MP is due to peripheral nerve compression.

(Choice C) Nerve conduction studies (NCS) can be helpful to diagnose MP in doubtful cases; however, this patient
has classic symptoms, and in most patients MP is a clinical diagnosis.

(Choice D) Physical therapy is sometimes prescribed to certain patients with sciatica (eg, persistent symptoms),
although the effectiveness may be variable. For MP, aerobic exercise can facilitate weight loss and reduction of
symptoms, but skilled physical therapy is not needed.

(Choice E) X-ray of the hip and pelvis can identify osteoarthritis of the hip joint. Osteoarthritis and other intrinsic
disorders of the hip occasionally cause upper lateral thigh pain, but deep anterior groin pain is more typical, and
loss of sensation is not seen.

Educational objective:
Meralgia paresthetica is caused by compression of the lateral femoral cutaneous nerve under the inguinal ligament
and presents with localized pain and paresthesia in the lateral thigh without weakness. Patients are usually treated
conservatively with weight loss and avoidance of tight garments.
Question #420

A 32-year-old man comes to the office due to a 3-week history of escalating low back pain. The pain is dull, achy,
present at rest, and worse with activity. The patient has no significant medical history. He smokes a pack of
cigarettes daily and drinks alcohol occasionally. He recently began using intravenous heroin. Temperature is 37.7
C (99.9 F), blood pressure is 120/80 mm Hg, pulse is 90/min, and respirations are 16/min. Gentle percussion over
lumbar vertebrae L3 and L4 elicits exquisite tenderness. Spine mobility is decreased and there is spasm of the
paravertebral muscles. A full neurologic examination, including straight-leg raising test, is normal. Laboratory
results are as follows:

Hematocrit 46%
Platelets 550,000/mm3
Leukocytes 8,500/mm3

Which of the following is the most likely diagnosis?

A) Ankylosing spondylitis

B) Lumbar disk herniation

C) Lumbar spinal stenosis

D) Vertebral compression fracture

E) Vertebral osteomyelitis
Explanation
Correct Answer:

E) Vertebral osteomyelitis

Vertebral osteomyelitis

• Staphylococcus aureus most common


• Hematogenous seeding (eg, distant infection, IVDU, intravenous catheter)
Pathogenesis
• Contiguous spread from local infection
• Penetrating injury, invasive procedure/iatrogenic

• Subacute-chronic pain & midline spinal tenderness


Clinical features • +/- Fever
• Possible radiculopathy, LE weakness

• Elevated CRP/ESR
Diagnosis • MRI
• Blood culture, CT-guided bone biopsy/culture

• Abscess/epidural extension/neurologic deficits


Complications
• Vertebral collapse
• Intravenous antibiotics
Management
• Drainage of any associated abscesses

CRP = C-reactive protein; ESR = erythrocyte sedimentation rate; IVDU = intravenous drug use; LE = lower
extremity.

This patient most likely has vertebral osteomyelitis. Patients with injection drug use, sickle cell anemia, and
immunosuppression are at highest risk. The spine is a frequent site in those with injection drug use (reflecting
hematogeneous spread to the highly vascularized spine); Staphylococcus aureus is the most common pathogen,
but infections with gram-negative organisms also occur.

Many cases of vertebral osteomyelitis are chronic (>6 weeks) and insidious with minimal, nonspecific
symptoms, often initilally attributed to muscular back pain. Many patients have worsening back pain unrelieved by
rest; fever is present in <50% of cases. Physical examination often shows few findings, but focal tenderness to
gentle percussion over the spinous process of involved vertebrae (eg, L3-L4 in this patient) can be an important
clue; paravertebral spasm and reduced spinal range of motion may be seen.

A high index of suspicion should be maintained (eg, worsening back pain with localized spine tenderness in a
patient with injection drug use). In addition to fever being an unreliable finding, leukocytes may be elevated or
normal. Elevated platelet count may be present (marker of inflammation), and the erythrocyte sedimentation rate
is often significantly elevated. MRI is the most sensitive diagnostic study. Treatment includes long-term
intravenous antibiotics with or without surgery.

(Choice A) Ankylosing spondylitis, characterized by pain and progressive limitation of back motion, occurs most
often in young men. Symptoms are usually most severe in the early morning and improve as the day progresses
(activity relieves pain). Exquisite focal tenderness to palpation is not a classic finding.

(Choice B) Lumbar disc herniation typically presents with acute-onset back pain with or without radiation down the
leg. Patients can usually recall an inciting event. The pain worsens with activity and improves with rest. Focal
spine tenderness and reactive thrombocytosis are not common.

(Choice C) Lumbar spinal stenosis is a narrowing of the spinal canal with spinal root compression. Most patients
are age >60 and experience back pain that radiates to the buttocks and thighs. Symptoms often worsen with
walking and lumbar extension and improve with lumbar flexion. Numbness and paresthesias may occur.

(Choice D) Vertebral compression fracture can cause focal tenderness to palpation as well, but it is most common
in elderly patients with osteoporosis and is typically acute pain (vs progressively worsening). Reactive
thrombocytosis in this patient also indicates a chronic inflammatory process.

Educational objective:
Vertebral osteomyelitis presents with insidious onset of back pain unrelieved by rest. Focal tenderness to
percussion over the spinous process of the involved vertebra, when present, is a clue. Fever and leukocytosis are
unreliable findings, but the erythrocyte sedimentation rate is significantly elevated.
Question #421

A 51-year-old woman comes to the office due to pain in her hands. For the past 2 years, she has had daily pain
and stiffness in the fingers that are worse when working with hand tools or opening food jars. The patient has tried
massaging her hands and doing stretching exercises, with only partial relief. Medical history is unremarkable, and
she takes no medications. Vital signs are normal. Examination reveals mild, hard periarticular enlargement at
multiple proximal and distal interphalangeal joints of both hands, as shown in the image below:
Examination of the metacarpophalangeal joints and wrists is normal. Which of the following is the most appropriate
therapy for management of this patient's symptoms?

A) Glucosamine

B) Methotrexate

C) Prednisone

D) Topical diclofenac

E) Tramadol
Explanation
Correct Answer:

D) Topical diclofenac

Hand osteoarthritis

• Age >40
Clinical
• Chronic progressive hand pain & stiffness
presentation
• DIP & PIP joints; 1st carpometacarpal joint

• Examination: bony enlargement (Heberden & Bouchard nodes), crepitus with range of
Diagnosis motion
• X-ray: joint space narrowing, subchondral sclerosis, periarticular osteophytes

Initial • Stretching & strengthening exercises


management • Topical or oral NSAIDs

DIP = distal interphalangeal; NSAIDs = nonsteroidal anti-inflammatory drugs; PIP = proximal interphalangeal.

This patient has pain and stiffness consistent with early osteoarthritis (OA) of the hands. OA typically becomes
symptomatic at age >40, and patients commonly report worsening of symptoms with extensive use of hand tools or
other strenuous motor tasks (eg, opening jars). OA of the hands is primarily a clinical diagnosis; distal
interphalangeal joint involvement is classic, and bony enlargement at the distal and proximal interphalangeal joints
(ie, Heberden and Bouchard nodes) is pathognomonic for OA and often associated with a strong family history of
the condition.

Initial management of hand OA includes stretching and strengthening exercises. Nonsteroidal anti-
inflammatory drugs (NSAIDs) reduce prostaglandin-induced sensitization of peripheral nociceptors and are
effective for pain relief in patients with OA. Topical NSAIDs (eg, diclofenac) are adequate for most patients; oral
NSAIDs are also effective but carry a greater risk for toxicity (eg, peptic ulcer).

(Choice A) Glucosamine is sometimes suggested for management of knee OA, although evidence of its benefit is
mixed. It is not well studied for OA of the hands, and any benefits are likely negligible.

(Choices B and C) Systemic glucocorticoids (eg, prednisone) are used for acute symptomatic management of
rheumatoid arthritis (RA), whereas methotrexate is used chronically to minimize long-term RA joint destruction.
Although RA can affect the interphalangeal joints, it is more common in the metacarpophalangeal joints and wrists,
and the joint swelling is typically rubbery or spongy, rather than hard and bony. In addition, distal interphalangeal
joint involvement can occur in RA but should raise suspicion for OA. Although intraarticular glucocorticoids are
occasionally used for patients with refractory hand (especially thumb) OA, systemic glucocorticoids are not
recommended.

(Choice E) Other medications that can provide pain relief in OA include duloxetine, tramadol, and low-dose
opioids. However, these drugs are not usually necessary for patients with early or mild disease and can cause
nausea, vomiting, dizziness, and constipation. Therefore, they are generally considered only when OA does not
respond to NSAIDs.

Educational objective:
Initial management of hand osteoarthritis includes stretching and strengthening exercises. Topical nonsteroidal
anti-inflammatory drugs (NSAIDs) (eg, diclofenac) are beneficial for most patients; oral NSAIDs are also effective
but carry a greater risk for toxicity (eg, peptic ulcer).

Reference
• Efficacy and safety of non-pharmacological, pharmacological and surgical treatment for hand osteoarthritis:
a systematic literature review informing the 2018 update of the EULAR recommendations for the
management of hand osteoarthritis.

• Use of NSAIDs in treating patients with arthritis.


Question #422

A 38-year-old woman comes to the office due to left knee swelling for the past 3 weeks. The patient has type 2
diabetes mellitus and recently started an exercise program consisting of aerobics and running to lose weight and
improve glycemic control. She reports no falls or trauma. BMI is 35 kg/m2. On examination, the knees are not red
or warm and demonstrate full range of motion. There is a 4-cm area of swelling in the anteromedial knee region 3
cm below the joint line. The swollen area is mildly tender on palpation. There is no joint line tenderness. A left
knee x-ray reveals no abnormalities. Which of the following is the most appropriate next step in management of this
patient's condition?

A) Diagnostic aspiration of the swollen area

B) Immobilization with a rigid knee brace

C) MRI evaluation of the knee

D) Quadriceps and hamstring muscle exercises

E) Serum inflammatory marker testing


Explanation
Correct Answer:

D) Quadriceps and hamstring muscle exercises


This patient has pes anserine bursitis (PAB) related to overuse. The anserine bursa is located over the medial
tibial condyle distal to the joint line and deep to the pes anserinus tendon complex. Repetitive friction,
exacerbated by mechanical derangements of the knees (eg, medial meniscus protrusion, medial collateral ligament
displacement) and obesity, is thought to cause bursal swelling. PAB is part of a broader syndrome, pes anserinus
pain syndrome, that also includes nonbursal causes of upper medial tibial pain.

Initial management includes rest and nonsteroidal anti-inflammatory drugs. To enhance overall fitness and
address underlying mechanical derangements, patients should also perform exercises that increase strength and
flexibility of the quadriceps and hamstring muscles, which stabilize the knees. Weakness and tightness in these
muscles increase the stress on the articular (eg, bone, ligament, cartilage) and periarticular (eg, bursa, tendon)
tissues. An imbalance between these opposing muscles (eg, quadriceps stronger than hamstring muscles) can
lead to injury to the weaker muscle (and tendon) when the stronger muscle contracts.

(Choice A) When redness, warmth, and severe tenderness accompany bursal swelling, an aspiration should be
done to rule out septic bursitis. This patient does not have these acute inflammatory changes.

(Choice B) Rigid knee immobilizers are indicated for short-term management of patella fracture or rupture of the
quadriceps/patellar ligament complex. Immobilization of the knee is not indicated in PAB because it may promote
muscular atrophy and increase the risk of additional injury.

(Choice C) MRI is useful in diagnosing ligament or meniscus tears, which typically present with significant knee
pain and effusion following trauma. Stress fractures can be missed by x-ray and can be diagnosed with MRI,
especially in early stages (ie, first 1-2 weeks). However, this patient has had symptoms for 3 weeks; stress fracture
significant enough to cause localized swelling would have been identified on the x-ray.

(Choice E) Serum inflammatory markers can be useful when rheumatologic disease (eg, gout) or infection is
suspected to be the cause of the bursal swelling. Redness, warmth, and significant pain typically accompany bursal
swelling.

Educational objective:
Pes anserine bursitis presents as a swelling over the upper medial tibial region. It is typically caused by overuse
and poor knee mechanics. Treatment includes quadriceps and hamstring strengthening exercises to optimize
strength, flexibility, and stability of the knees.
Reference
• Pes anserine bursitis.
Question #423

A 67-year-old woman comes to the office due to right hand pain. For the past 3 months, she has had mild to
moderate pain at the base of the thumb that is worse when working with hand tools in her garden. In addition, the
patient has stiffness in the hands for 5-10 minutes after awakening. Medical history is notable for type 2 diabetes
mellitus. She drinks 2 or 3 glasses of wine a day. Vital signs are normal. Examination shows tenderness at both
the palmar and the dorsal aspects of the base of the thumb that is associated with hard enlargement at the
carpometacarpal junction. X-ray of the hand is shown in the exhibit. Which of the following is the most likely
diagnosis?

A) Chronic osteomyelitis

B) Hemochromatosis

C) Osteoarthritis

D) Psoriatic arthritis

E) Rheumatoid arthritis

F) Tophaceous gout
Explanation
Correct Answer:

C) Osteoarthritis

Osteoarthritis

Age of onset • >40; prevalence increases with age

• Knees
Joint • Hips
involvement • Distal interphalangeal joints
• 1st carpometacarpal joint

Morning
• None/brief (<30 min)
stiffness

Systemic
• Absent
symptoms

Examination • Hard, bony enlargement of joints


findings • Reduced range of motion with crepitus
This patient has osteoarthritis (OA) at the first carpometacarpal (CMC) joint. Hand OA typically presents with
chronic, progressive stiffness and pain that worsen when working with tools or performing fine motor tasks. Risk
factors include age >40, repetitive joint stresses, diabetes mellitus, and excessive alcohol intake.

Examination typically shows hard, periarticular enlargement, crepitus with movement, and mild tenderness. Other
affected joints include the distal interphalangeal (DIP) and proximal interphalangeal (PIP), where bony enlargement
classically manifests as Heberden and Bouchard nodes. X-ray reveals decreased joint space, subchondral
sclerosis, and periarticular osteophytes.

(Choice A) Chronic osteomyelitis presents with erythema, swelling, and pain, often with an overlying ulcer or sinus
tract. However, osteomyelitis typically occurs in the foot (only rarely in the hand), and bone destruction, rather than
enlargement, is typical.

(Choice B) Hemochromatosis can cause arthritis resembling OA, and although men are usually symptomatic
before age 50, women often develop symptoms later. However, hemochromatosis predominantly affects the
second and third metacarpophalangeal joints with hook-like osteophytes on the metacarpal heads; involvement of
the CMC joints is rare.

(Choice D) Psoriatic arthritis (PsA) typically presents with inflammatory arthritis that may involve the entire digit (ie,
dactylitis). Symptoms are commonly cyclical/episodic rather than chronic/slowly progressive. Prolonged (>30 min)
morning stiffness and pain lessen, not worsen, with activity. PsA commonly involves the DIP and PIP joints; CMC
arthritis is atypical. X-ray reveals narrowing of the proximal phalanx with erosion of the distal phalanx (pencil-in-cup
deformity) and new bone formation. It is uncommon (<10% of cases) for PsA to present before skin symptoms.

(Choice E) Rheumatoid arthritis commonly affects the hands, but usually affects the metacarpophalangeal joints
and wrists. Swelling is typically firm or rubbery rather than hard and bony. Morning stiffness lasts >30 minutes; this
patient's brief stiffness is more typical of OA.

(Choice F) Gouty tophi involving the joint capsule, articular cartilage, or ligament insertions may cause pain with a
hard mass adjacent to the joint. X-ray typically reveals soft tissue opacification and punched-out lytic lesions with a
rim of overhanging bone (rat-bite lesion). Most patients have a history of acute, episodic gout.

Educational objective:
Osteoarthritis of the hands presents with hard, periarticular enlargement of the joints; crepitus with range of motion;
and mild tenderness. It most commonly involves the first carpometacarpal joint and distal and proximal
interphalangeal joints. X-ray reveals decreased joint space, subchondral sclerosis, and periarticular osteophytes.
Question #424

A 65-year-old man is admitted to the hospital for hematemesis. The patient has had abdominal pain for the past 3
months and a 4.5 kg (10 lb) weight loss. He has no other medical history and has not seen a physician in 30 years.
The patient is a retired car mechanic and lives with his wife; they attend church service every week. After he is
stabilized, an esophagogastroduodenoscopy is performed, which shows an ulcerated mass; biopsy confirms gastric
adenocarcinoma. When the physician approaches the patient to discuss the biopsy result, he is adamant that he
does not want to know the result. Which of the following is the most appropriate next step in management of this
patient?

A) Ask the patient whether there is a family member to whom the physician can disclose the diagnosis

B) Discuss the biopsy result with the patient's wife and allow her to make subsequent management decisions

C) Document that the patient does not wish to know the diagnosis and discharge the patient with an oncology
appointment

D) Offer to have the patient's pastor or the hospital chaplain be present when the physician discusses the
result

E) Warn the patient there is bad news, allow him time to mentally prepare, and set up a time tomorrow to
discuss the result
Explanation
Correct Answer:

A) Ask the patient whether there is a family member to whom the physician can disclose the diagnosis

This patient has gastric adenocarcinoma but declines to discuss the diagnosis. Although patients have the right to
know their diagnoses, they also may choose not to be informed of their medical information, including potentially
life-threatening diagnoses. Such refusal may be due to fear, avoidance, or cultural preferences. Therefore, this
patient's request to decline discussion of the biopsy result should be honored by the physician (Choices D and
E).

However, relinquishing care (eg, discharging the patient) without a plan for communication risks discontinuity of
information and suboptimal follow-up care. In addition, deferring the disclosure to a consultant whose specialty (eg,
medical oncology) would imply the diagnosis would be inappropriate and would violate the patient's right not to be
informed (Choice C). Instead, suitable next steps include:

• Ask the patient to choose a family member or friend with whom the physician can disclose the diagnosis

• Schedule a follow-up appointment with a primary care physician (PCP) and notify that physician of the
patient's diagnosis and of the patient's decision not to be told the diagnosis

Should the patient change his mind, he can obtain information on his diagnosis from his family member and follow
up with the PCP who can coordinate consultation with an oncologist.

(Choice B) The patient's wife is his default surrogate decision-maker if he is incapacitated and unable to make
decisions (and if he has not specified another decision-maker). However, this patient has capacity to make his own
decisions, including who should receive medical information on his behalf. The physician should ask the patient to
designate the person with whom to share the diagnosis.

Educational objective:
Patients have the right to know or to decline discussion of their diagnoses. When a patient declines, the physician
should ask the patient to choose a family member with whom the diagnosis can be disclosed; doing so ensures
continuity of information should the patient choose to be informed later.

Reference
• End-of-life care: guidelines for patient-centered communication.

• The right to information.


Question #425

A 53-year-old man comes to the emergency department of an academic medical center with back pain after lifting a
heavy box a day earlier. The patient appears anxious and in pain. He says, "I'm worried I've twisted a nerve in my
back, and I really hope you are going to do an MRI." A second-year resident obtains the patient's history and
conducts an initial physical examination. Palpation of the lower back reveals paravertebral tenderness on the left
side at the L4 level; there is no tenderness to palpation over the spine. Strength and sensation in both extremities
are intact, and deep tendon reflexes are symmetric. The resident presents his findings to the attending physician,
mentioning the patient's concerns and request for MRI. As the attending physician examines the patient, which of
the following statements is most recommended to reduce patient requests for unnecessary imaging?

A) "I will now test your nerves and spinal cord, to confirm both are functioning normally."

B) "MRI can be very expensive and is unnecessary for you; it is better to try exercise and physical therapy
instead."

C) "Since the findings from the resident physician's examination are normal, MRI is not likely to show anything
meaningful."

D) "We see patients with this condition all the time, and it will get better in a few days."

E) "Your pain appears to be caused by tight back muscles and can be treated with muscle relaxants."
Explanation
Correct Answer:

A) "I will now test your nerves and spinal cord, to confirm both are functioning normally."

Resource overuse (eg, unnecessary imaging for uncomplicated low back pain) is associated with increased health
care costs and suboptimal patient outcomes. Current guidelines specify that imaging is not indicated for
uncomplicated, acute back pain (<4-6 weeks) lacking "red flags" (eg, constitutional symptoms, prior malignancy).
However, patient concerns about serious but unlikely causes of pain (eg, significant nerve injury) may precipitate
expectations for imaging that conflict with optimal care and physician judgment.

Patients with low back pain are less likely to request imaging studies when they believe the physician has
performed a thorough physical examination that checks for serious pathology and they receive a specific
explanation of their condition. Recommended practices include the following:

• Apply patient-centered (rather than paternalistic) approaches to build a patient-physician partnership and
directly address patient concerns.

• Perform a detailed physical examination, giving special attention to areas that are concerning to the
patient and providing ongoing commentary on what structures are being assessed and the significance of
findings.

This patient's request for imaging is likely linked to his concern for nerve damage; providing specific commentary
during the examination confirming that his nerve function is normal is likely to reassure the patient and support
subsequent discussions explaining why imaging is unnecessary.

(Choice B) This statement simply reiterates the physician's perspective regarding imaging but does not address
the concerns that precipitated the patient's imaging request.

(Choice C) This statement is vague (eg, physical examination is "normal"), does not address the patient's specific
concerns, and is less likely to reassure the patient.
(Choice D) This statement is physician- rather than patient-centered, does not address the patient's specific
concern, and is likely to be perceived as dismissive.

(Choice E) This statement describes a potential cause of the pain; however, it does not reassure the patient that
the physician has assessed for nerve damage and is likely to be less effective than specifically addressing nerve
function.

Educational objective:
Patients with low back pain are less likely to request unnecessary imaging studies when they believe the physician
has thoroughly checked for serious pathology and has addressed their specific concerns (eg, nerve damage). The
physician should perform a detailed physical examination with specific, ongoing commentary on the structures
being assessed and the significance of findings.

Reference
• Diagnostic imaging for low back pain: advice for high-value health care from the American College of
Physicians.
Question #426

A 65-year-old woman comes to the clinic due to markedly decreased urine output for the past 24 hours. The patient
underwent coronary angiography by the physician's colleague 5 days ago and was discharged the next day in
stable condition. Medical conditions include type 2 diabetes mellitus, hypertension, and coronary artery disease.
Temperature is 37 C (98.6 F), blood pressure is 140/92 mm Hg, pulse is 88/min, and respirations are 14/min.
Laboratory results show creatinine of 3.4 mg/dL and blood urea nitrogen of 40 mg/dL. Acute tubular necrosis
secondary to the contrast used during angiography is suspected. Review of the medical chart before and after the
procedure reveals that the patient did not receive adequate hydration prior to angiography. The patient is very
concerned and asks why she is unable to urinate. Which of the following is the most appropriate response?

A) "Given your underlying health conditions, this is unfortunately an expected event that occurs occasionally
after angiography."

B) "Let me ask your regular physician to call you and discuss what might have happened."

C) "The cause of your condition is unlikely to ever be known, but let's focus on what we can do now with the
best possible care."

D) "This complication could have been prevented if your physician had given intravenous hydration prior to the
procedure. Unfortunately, the physician forgot to do this."

E) "This may be related to your recent angiography, but I need to discuss it with your regular physician before I
can give you a definitive answer."
Explanation
Correct Answer:

E) "This may be related to your recent angiography, but I need to discuss it with your regular physician before I
can give you a definitive answer."

This patient likely has contrast-induced acute kidney injury (AKI), and her risk may have been unnecessarily
increased due to lack of periprocedural intravenous hydration. Discussing a potential medical mistake made by a
colleague can be awkward, especially when the patient or family is pressing for detailed information. Nevertheless,
the physician should discuss the essential truth with the patient in a tactful and concise manner.

Ideally, patient discussions regarding an adverse outcome should be conducted by the physician who had primary
responsibility for the initial intervention. If the physician who performed the procedure is unavailable, a complete
and detailed explanation may not be possible. Therefore, a brief and honest explanation should be provided to
the patient immediately ("This may be related to your recent angiography"), with the assurance that a definitive
answer will be provided once the matter has been fully discussed with the colleague. In the meantime, appropriate
care and follow-up for the acute complication should be provided.

(Choice A) AKI can be associated with administration intravenous contrast agents, and the risk may be greater in
patients with diabetes mellitus. However, this only increases the importance of proper care and does not excuse a
serious medical mistake.

(Choice B) This response completely evades the patient's direct question and is not appropriate. Even though the
physician has not had the opportunity to discuss the patient's care with the colleague, a brief acknowledgment of
the potential relationship of the procedure to the complication should be made.

(Choice C) Claiming that the cause of this patient's condition may never be known is deceptive, particularly since
the sequence of events has a potentially clear explanation. The physician should discuss the basic facts of the
case in a tactful but honest way.

(Choice D) Although intravenous hydration was not given prior to the procedure, additional information should be
obtained from the responsible physician before determining that it was mistake; there might have been other
reasons for not giving intravenous fluids. Also, patients can develop AKI following intravenous contrast even with
appropriate precautionary measures.

Educational objective:
Patient discussions regarding treatment complications should ideally be conducted by the physician who had
primary responsibility for the initial intervention. If that physician is unavailable, a brief and honest explanation
should be provided to the patient by the covering physician, with the assurance that a definitive answer will be
provided once the matter has been fully discussed with the colleague.
Question #427

A 34-year-old man calls the office requesting an urgent appointment due to cough and cold symptoms. His
temperature is 37.2 C (99 F), but he feels "miserable" and is unsure if he should go to work the next day. The
patient has a history of seasonal allergies and knee pain from a past sports injury. He has no other medical
problems but does have a history of frequent office visits. The receptionist informs him that the physician is
preparing to leave as it is near closing time but that an appointment is available for the next morning. The patient
declines the offer of an appointment and hangs up. As the physician leaves the office 30 minutes later to make
evening rounds at the hospital, the patient arrives and insists on being seen immediately. He sounds congested
and coughs occasionally but is breathing normally. He has no other symptoms. He again asks to be examined.
Which of the following is the most appropriate response?

A) "Although I understand your concern, we should address it tomorrow because it is not an emergency."

B) "I am sorry. I would normally see you, but I have another appointment that can't wait."

C) "I am sorry you drove all this way, but your problem can wait until tomorrow."

D) "I insist that you go to the nearest emergency department for evaluation."

E) "I regret that I cannot see you now; didn't my staff inform you that it is closing time?"

F) "Let me take a quick look; it's probably nothing to worry about."


Explanation
Correct Answer:

A) "Although I understand your concern, we should address it tomorrow because it is not an emergency."

This physician is faced with a patient who inappropriately demands to be seen after hours for a non-urgent
concern. A stable patient who fails to arrive at the appointed time should be advised to reschedule the
appointment; the physician is under no obligation to immediately see this patient. Performing an examination after
hours is inappropriate and reinforces the patient's sense of entitlement and poor boundaries. Acquiescing to the
patient's demands would only encourage similar behavior in the future and create further tension in the physician-
patient relationship due to the patient's lack of respect for the physician's time (Choice F).

The most appropriate response is to calmly set limits and establish clear professional boundaries. The
physician should explain in a polite yet firm tone that the examination will be deferred until the next day because the
condition is not an emergency.

(Choice B) This response suggests that the patient's behavior is appropriate and offers an excuse not to see the
patient rather than explaining that non-emergency concerns can wait.

(Choices C and E) These responses set limits but are dismissive of the patient's concerns. It is better to
acknowledge the concerns but explain that non-urgent issues are addressed during regular office hours.

(Choice D) Recommending that the patient go to the emergency department for a non-urgent condition is
inappropriate and places an unnecessary burden on emergency services.

Educational objective:
Physicians should respond politely but firmly to inappropriate patient requests. Maintaining professional boundaries
is an important component of the physician-patient relationship.

Reference
• Managing difficult encounters: understanding physician, patient, and situational factors.
Question #428

A 44-year-old psychology professor with a history of chronic rheumatoid arthritis comes to the office for a follow-up
examination. She is currently taking prednisone and infliximab, a regimen that has successfully stabilized her
condition. She enjoys swimming 3 or 4 times a week, which helps with her strength and keeps her weight on
target. Temperature is 37.2 C (99 F), blood pressure is 110/70 mm Hg, pulse is 78/min, and respirations are 16/
min. The patient weighs 65 kg (143.3 lb) and is 162.5 cm (5 ft 4 in) tall. Physical examination reveals no significant
changes. Toward the end of the visit, the patient mentions that a family friend who suffers with chronic pain
secondary to a spinal fusion has had a good response to acupuncture. She would like to try acupuncture instead of
the medications she is taking currently. Which of the following is the most appropriate response to this patient's
request?

A) Acupuncture is definitely something we can try; alternative treatments can be very beneficial.

B) Acupuncture should not be used as it has not demonstrated efficacy for rheumatoid arthritis.

C) I am concerned that your condition will worsen rapidly if you stop your medications.

D) I am not familiar with acupuncture but can refer you to a clinician experienced in the procedure.

E) Tell me more about your interest in trying acupuncture for your condition.
Explanation
Correct Answer:

E) Tell me more about your interest in trying acupuncture for your condition.

Patients are increasingly interested in complementary and alternative medicine (CAM), and physicians must be
prepared to discuss CAM modalities with them and integrate treatments that have demonstrated safety and
efficacy. The initial response to a patient who requests an alternative approach is to explore the reasons for
requesting a different treatment.

This patient may be dissatisfied or have concerns about the quality of current care. She may be experiencing
bothersome medication side effects of which the physician is unaware. Once the patient's reasons are clarified, the
physician can discuss the risks and benefits of the proposed CAM treatment in addition to suggesting
modifications to the current treatment regimen that may address her concerns.

(Choice A) This response enthusiastically endorses the patient's request for acupuncture without first
understanding what is prompting her request. In addition, it makes a general statement about the benefits of CAM
that is inaccurate in her case given the lack of evidence for acupuncture in rheumatoid arthritis and the risks of
discontinuing the current treatments.

(Choice B) Although studies have not demonstrated acupuncture's efficacy for rheumatoid arthritis, this response
rejects the patient's suggestion outright without first exploring the reasons for her request. The risks and benefits of
the proposed treatment can be discussed after the physician has gained a better understanding of the patient's
perspective.

(Choice C) Warning the patient about the risks of stopping her current treatment regimen is dismissive of her
request and likely to damage the physician-patient relationship. The first step is to understand why she is
requesting a change in her treatment. The risks of stopping her current medications can be addressed at a later
point.

(Choice D) It would be premature to refer the patient for acupuncture without first understanding the reasons
behind her request.

Educational objective:
The initial response to patients suggesting complementary and alternative medicine is to explore their reasons for
requesting such treatment. Physicians must be prepared to discuss the risks and benefits of complementary and
alternative approaches to treatment and be open to integrating modalities with proven safety and efficacy.

Reference
• A qualitative systematic review of patients' experiences of acupuncture.

• Integrative medicine and patient-centered care.

• Integration of acupuncture into conventional medicine from health professionals' perspective: a thematic
synthesis of qualitative studies.
Question #429

A 50-year-old man comes to the office for a preventive examination. The patient is accompanied by his wife and
daughter. He currently feels well and takes no medications. The patient works as an accountant at a large
technology corporation. He smoked a pack of cigarettes daily for 5 years but quit 15 years ago. The patient drinks
1 or 2 alcoholic beverages per week. He emigrated from Colombia 5 years ago and is fluent in English. Physical
examination is within normal limits. The provider recommends colon cancer screening, but the patient appears
hesitant and states, "I don't think I really need that because I feel good right now, and I'm pretty sure I don't have
cancer." Which of the following is the best next step?

A) Engage the patient in counseling on the scientific benefits of cancer screening

B) Explain that the patient's smoking history increases his cancer risk

C) Hold a family-centered discussion on the importance of cancer screening

D) Identify the patient's perceptions of cancer causes

E) Provide the patient with culturally appropriate materials on cancer screening


Explanation
Correct Answer:

D) Identify the patient's perceptions of cancer causes

Significant health care disparities can undermine the health of minority and immigrant patients. Cross-cultural
care improves minority health outcomes, provider-patient communication, and health care quality (eg, completing
recommended cancer screening). Cross-cultural care reflects core principles of high-value care common to all
populations:

• Patient-centeredness: understanding and aligning care with patient values and priorities

• Cultural sensitivity: exploring and responding to a patient's cultural perspective (eg, avoiding stereotyping
or assumptions, asking questions) and understanding sociocultural influences on perceptions of health

• Enhanced communication: addressing language barriers (eg, use of interpreter services) and tailoring
communication styles (eg, adjusting eye contact, addressing family) to patient preferences

This patient is hesitant to undergo colon cancer screening, but his reasons need clarification. The clinician should
engage in cross-cultural care by first understanding the patient's perceptions and beliefs about cancer and its
causes. For example, non-White patients may perceive themselves as less susceptible to cancer if prevention
messaging (eg, television advertising) primarily features images of White American communities. In addition,
patients may associate screening with fear (eg, "tempting fate" when symptoms are absent) or stigma (eg, relation
of cervical cancer to sexual activity).

Understanding the patient's perspective will enable the clinician to provide more meaningful counseling, which
increases likelihood of screening acceptability and completion.

(Choice A) Patient perceptions about disease prevention and causation may extend beyond scientific paradigms
(eg, spirituality, nature). The patient's perspective on disease must first be understood before proceeding with
scientific explanations.
(Choice B) Beginning counseling by explaining that the patient's cancer risk is increased by his smoking history
can exacerbate fears about screening and is less effective as a first step compared to identifying the patient's
understanding of cancer causes.

(Choice C) Family-centered discussions can be useful in complex situations (eg, cancer treatment); certain
cultures may also value family involvement. However, the provider should first work with the patient directly and
identify his preferences.

(Choice E) Culturally inclusive materials can promote cross-cultural care but should be used as an adjunct to
active, patient-centered discussion. Offering materials may be ineffective if this patient perceives screening to be
unnecessary; therefore, soliciting patient perspectives is a better first step.

Educational objective:
Cross-cultural care is patient-centered (reflects patient values) and culturally sensitive (responds to a patient's
sociocultural paradigm). It involves first exploring individual health perceptions (eg, cancer causes), allowing
tailoring of further counseling (eg, cancer screening).

Reference
• Addressing disparities in cancer screening among U.S. immigrants: progress and opportunities.
Question #430

A 42-year-old woman comes to the office for evaluation of weight management. The patient notes that she has not
been able to lose any weight despite making dietary changes and exercising 3 times a week for the past 6 months.
She is planning to start taking ephedra, an herbal supplement containing ephedrine, that she purchased online to
assist with weight loss. The risks of ephedra are discussed, and the patient is advised not to take the supplement.
Prescription weight loss medications are reviewed, but the patient declines, stating that she would like to first try
ephedra because many of her friends have had sustained weight loss with the supplement. Medical history is
otherwise noncontributory. Vital signs are normal. BMI is 36 kg/m2. Physical examination shows obese habitus but
is otherwise normal. Which of the following is the most appropriate statement?

A) "I am not comfortable with you taking ephedra, but if you would like, I can transfer you to another provider
who is more comfortable with this supplement."

B) "I don't recommend starting ephedra, but I'm available to talk about prescription weight loss medications if
you change your mind in the future."

C) "If you start taking ephedra, I would like to see you for regular follow-up visits so that I can monitor for any
adverse effects."

D) "It seems you plan to start taking ephedra; will you please sign a form saying that you understand the risks
of taking this supplement?"

E) "Supplements are more commonly used at weight loss treatment centers; perhaps you should look into
getting a consultation at one."
Explanation
Correct Answer:

C) "If you start taking ephedra, I would like to see you for regular follow-up visits so that I can monitor for any
adverse effects."

This patient is planning to take a weight loss supplement (ie, ephedra) that can cause significant cardiovascular
toxicity (eg, hypertension, stroke). Dietary supplements containing ephedrine are banned in the United States,
although patients are occasionally able to obtain them from international and online sources. In addition to toxicity
concerns, such nonstandard medications are often produced under unstandardized manufacturing practices, which
may be associated with significant variation in the quantity and purity of the active ingredient and possible
adulteration.

When discussing a patient's use of nonstandard therapies that are known to be harmful, the physician should
clearly explain the risks to the patient. However, despite the risks, some patients are insistent to proceed with
such treatments, often based on anecdotal reports or testimonials.

In such cases, the physician should respect patient autonomy, understanding and acknowledging the patient's
motivations for taking the therapy. The physician should continue to be involved in the patient's care to monitor for
toxicity (eg, elevated blood pressure) and develop a stronger therapeutic alliance (Choice A). Frequent follow-up
is often helpful, and in time, the patient may come to better trust the physician's advice and discontinue the harmful
therapy.

(Choices B and E) Offering standard obesity treatment or consulting a specialist provider may eventually lead to
use of safer, more appropriate therapies. However, this patient has already stated a commitment to using a
potentially harmful treatment, and close follow-up should be provided to monitor for severe toxicity.

(Choice D) The physician should carefully document in the patient's chart that the risks of the patient's intended
treatment have been explained and that continued care to monitor for toxicity is needed. However, asking the
patient to sign a written consent or waiver is not necessary because the physician is neither providing nor
advocating for the treatment.

Educational objective:
When discussing a patient's use of nonstandard therapies that are known to be harmful, the physician should
clearly explain the risks to the patient. If the patient insists on using such therapies, the physician should schedule
close follow-up to strengthen the therapeutic alliance and monitor for toxicity.

Reference
• Common herbal dietary supplement-drug interactions.

• Online marketing of ephedra weight loss supplements: labeling and marketing compliance with the U.S.
Food and Drug Administration ban on ephedra.
Question #431

A 73-year-old woman comes to the emergency department due to shortness of breath and productive cough for the
past 2 weeks. The patient lives independently and is active in her retirement community. She was seen by her
primary care physician last year and has no medical conditions. The patient has a history of smoking but quit 20
years ago. Temperature is 37.8 C (100 F). Lung auscultation reveals right-sided crackles. Chest x-ray reveals
right lower lobe consolidation and a solitary 1-cm, round lesion in the left upper lung field, which was not seen on
previous x-rays. The patient is admitted and treated for community-acquired pneumonia, becomes medically
stable, and is ready to be discharged home with oral antibiotics. The physician records the pulmonary lesion finding
on the discharge summary, places an outpatient radiology referral for chest CT scan, and informs the patient that
she will be contacted regarding her radiology appointment after discharge. Several months later, the patient is
readmitted to the hospital with dyspnea and weight loss. Records show that she was called about the scheduled
appointment for the CT scan but did not attend it, so the test was not completed. Repeat imaging reveals
enlargement of the pulmonary nodule, and biopsy confirms lung adenocarcinoma. Which of the following
interventions by hospital providers is most appropriate for reducing future occurrences of similar errors?

A) Explaining the importance of primary care follow-up after discharge

B) Informing patients' family members about follow-up imaging studies

C) Monitoring results from pending appointments for all discharged patients

D) Postponing discharge until imaging studies and biopsy results are complete
Explanation
Correct Answer:

A) Explaining the importance of primary care follow-up after discharge

Fewer than 20% of inpatients with incidentally detected lung nodules undergo recommended postdischarge follow-
up (eg, CT scan, management based on malignancy risk), leading to preventable adverse outcomes, including
delayed cancer diagnoses. A common cause is loss to follow-up after transfers of care (eg, hospital discharge),
as seen in this patient. Because inpatient care focuses on acute medical conditions, patients may not recognize the
importance of assessing unrelated findings (eg, incidental lung or adrenal nodule) after discharge, especially
when symptoms resolve.

Timely (<4 weeks) primary care follow-up promotes completion of pending studies (eg, imaging, laboratory tests)
and health management (eg, new medications, diagnoses). During the discharge process, patients should receive
high-quality provider communication and counseling explaining the importance of follow-up, as this increases
patient likelihood of attending primary care visits and completing follow-up recommendations. High-quality
communication includes the following:

• Face-to-face discourse, allowing responses to nonverbal cues

• Patient-centered interaction tailored to the patient's current motivations, goals, and ability

• Interactive approaches for verifying patient understanding (eg, inviting questions, encouraging teach-back
that summarizes patient understanding of the conversation)

Discharge summaries can be routed to primary care providers for review when patients attend their
posthospitalization appointment.

(Choice B) Including family members in follow-up care is useful if the patient has cognitive impairment or has
consented to sharing medical information with them. However, for independent, cognitively intact patients (eg, this
woman), direct engagement through counseling is preferred for building awareness and promoting follow-up.
(Choice C) Monitoring results from pending appointments on all discharged patients extends beyond hospital
providers' scope of responsibility (eg, inpatient management of acute health concerns) and potentially compromises
patient safety by excessively increasing provider workload. Primary care providers often receive a log of their
patients' abnormal outpatient tests through the electronic medical record, enabling superior systematic monitoring
and follow-up.

(Choice D) Postponing discharge of medically stable patients until completion of nonurgent studies and biopsy
results, which may take several extra days, is not recommended because it increases nosocomial infection risk and
undermines resource stewardship (eg, use of efficient health care resources).

Educational objective:
Loss to follow-up after hospital discharge can lead to adverse outcomes. High-quality patient counseling that
encourages early (<4 weeks) primary care follow-up increases the likelihood of patient attendance, completion of
pending items (eg, imaging studies), and management of patient health status (eg, new diagnoses).

Reference
• Post-hospitalization transitions: examining the effects of timing of primary care provider follow-up.

• Impact of a health information technology intervention on the follow-up management of pulmonary nodules.

• Patient and clinician characteristics associated with adherence. A cohort study of veterans with incidental
pulmonary nodules.
Question #432

A 38-year-old woman comes to the office for a routine health maintenance evaluation. She moved to the area a
month ago with her 8-year-old son for work. The patient states, "I feel okay overall, but I know I've gained weight. I
used to live right next door to a grocery store, but now it takes 45 minutes by bus just to get fresh groceries that I
can afford. I usually buy junk food from convenience stores because I just don't have time to make the trip." She is
worried that her son is eating "too much junk and sugar" and is glad he receives healthy lunches from school.
Medical history is noncontributory. Vital signs are within normal limits. BMI is 28 kg/m2. Which of the following is
the most appropriate statement by the physician?

A) "Eating unhealthy foods can have long-term consequences; are you in the habit of checking nutrition
labels?"

B) "It sounds like you've noticed changes in your weight; what is your understanding of the effects of an
unhealthy diet?"

C) "Living far away from fresh and affordable food is difficult, but you should still be able to find healthy options
closer to home."

D) "There are options available if you really want to make changes for you and your family; perhaps you could
get your groceries delivered."

E) "You want to make healthy choices for you and your family; let me look into what resources are available
near you."
Explanation
Correct Answer:

E) "You want to make healthy choices for you and your family; let me look into what resources are available
near you."

The US Department of Agriculture (USDA) identifies low-income communities with limited access to healthy foods,
commonly known as food deserts, as regions with increased risk of diet-related morbidity and mortality. Food
deserts are designated as areas with a poverty rate of at least 20% (or a median family income <80% of the
surrounding area median income) and with at least 500 (or 33%) people in the area living >1 mile (or >10 miles in
rural areas) from a grocery store. According to this, approximately 10% of the US population lives in a food desert.

Social determinants of health must be kept in mind when providing patient care. Part of competent health
promotion is being aware of access limitations to health-supporting interventions in one's area of practice.
Developing awareness of pertinent matters, like food deserts, allows for more effective planning and direction to
available resources. Helpful information can be found within the USDA's Economic Research Service on both
food deserts and food and nutrition assistance programs (eg, which may help address the patient's financial
burden).

These issues require a nonjudgmental, unassuming approach to foster patient openness and promote practical,
effective interventions. In this case, recognizing the patient's barrier allows a point of intervention and clarifying
her health-promoting goals ("You want to make healthy choices for you and your family") enhances rapport and her
motivation to follow-up with the resources provided.

(Choices A and B) Education about dietary choices and consequences can be helpful in some situations;
however, this patient is aware of her dietary changes, the impact on her weight, the potential consequence to her
son, and expresses concern about her situation. Suggesting she read the food labels or asking her to repeat the
understanding she has already demonstrated is likely to be off-putting while neglecting the core concern (limited
access to affordable healthy options).
(Choices C and D) Although necessary to identify and acknowledge the barriers to health-promoting choices,
insinuating the patient is not trying hard enough to solve her dilemma or is not dedicated enough to making healthy
changes undermines rapport and decreases the likelihood she will follow treatment advice. Furthermore, it neglects
the patient's financial limitations and does not offer adequate, practical solutions.

Educational objective:
Food deserts are identified as low-income communities that have limited access to healthy food. When addressing
this barrier, communication should be nonjudgmental to foster collaboration, enhance patient motivation, and
develop a practical treatment plan.

Reference
• Distance to store, food prices, and obesity in urban food deserts.
Question #433

An 81-year-old woman comes to the office for follow-up of type 2 diabetes mellitus, which has been well controlled
on a stable medication regimen for several years. Medical history is notable for hypertension and osteoporosis. Six
months ago, she suffered a nondisplaced orbital fracture. She is widowed and has lived with her daughter and son-
in-law for the last year. She claims that she is independent and "takes care" of herself. Blood pressure is 176/102
mm Hg and pulse is 84/min. Examination shows bruising at the right forearm and right flank. No cardiac murmurs
are heard. Abdominal examination is unremarkable. Gait is steady and the patient does well on the Timed Up and
Go test. Laboratory results show a fasting glucose of 220 mg/dL and hemoglobin A1c of 9%. Four months ago,
glucose was 105 mg/dL and hemoglobin A1c was 7.2%. On further discussion, the patient says, "The pharmacy
charges me too much these days. I just can't afford my medication anymore." Which of the following is the most
likely explanation for this patient's clinical findings?

A) Alcohol abuse

B) Early-stage dementia

C) Elder abuse

D) Late-life depression

E) Nutritional deficiency

F) Psychotic disorder
Explanation
Correct Answer:

C) Elder abuse

Elder abuse

• Female
• Dementia, chronic mental illness
• Functional impairments
Risk factors
• Social isolation
• Shared living environment
• Poor socioeconomic status/financial stress

Physical & sexual abuse

• Atypical abrasions, lacerations, contusions, fractures


• Pain not consistent with reported etiology
• Anogenital injuries
• Newly acquired STI
Manifestations of abuse

Psychological & verbal abuse

• Change in behavior/personality
• Depression/anxiety
Neglect

• Inadequate nutrition or hydration


• Pressure injuries
• Deterioration in comorbid conditions

Financial exploitation

• Failure to adhere to medication regimen


• Multiple missed appointments
• Unpaid expenses or rent payments

STI = sexually transmitted infection.

This patient has findings that raise concern for elder abuse, including multiple injuries and an acute change in
financial status (ie, discontinuing longstanding medications due to cost), possibly due to diversion of finances by
family members. Abuse is most common in older women, especially those who are socially isolated or have a
shared living environment (ie, live with multiple nonspousal others). Dementia and chronic mental illness also
increase the risk.

Manifestations of physical abuse in the elderly are similar to those in younger patients, and may include
unexplained pain, injuries in multiple stages of healing, or injuries in unusual locations. Although osteoporosis
increases the risk of fracture with minor trauma, orbital fractures (as in this patient) are atypical. Elderly
individuals are also at risk for financial exploitation, psychological abuse, sexual abuse, and neglect. Most
jurisdictions require that suspected abuse be reported to adult protective services or other governmental agency.

(Choice A) Late-onset alcohol abuse is more common in elders who live alone. Living with multiple family
members increases the risk for elder abuse. Elder abuse is also more likely to explain this patient's injury pattern
(orbital fracture, multiple bruises in unusual locations).

(Choices B, D, and F) Dementia, affective disorders, and psychotic disorders are common in elderly patients and
can lead to nonadherence to medical therapy. However, this patient has experienced multiple injuries that suggest
abuse.

(Choice E) Nutritional deficiencies are usually seen in elders who live alone and do not have outside nutritional
support. Although a variety of vitamin deficiencies can cause bruising, this patient's orbital fracture is more
suspicious for abuse.

Educational objective:
Manifestations of physical abuse in the elderly may include unexplained pain, injuries in multiple stages of healing,
or injuries in unusual locations. Elderly individuals are also at risk for financial exploitation, psychological abuse,
sexual abuse, and neglect.

Reference
• Elder abuse.

You might also like